You are on page 1of 816

14/05/2015

Reference ranges

Question 1 of 542

Next

A 20 year old lady presents with pain on the medial aspect of her thigh. Investigations show a
large ovarian cyst. Compression of which of the nerves listed below is the most likely
underlying cause?

Question stats

End and review

Score: 100%

6.5%

20%

45.7%

13.9%

13.8%

A. Sciatic
45.7% of users answered this
question correctly

B. Genitofemoral
C. Obturator
D. Ilioinguinal

Search

E. Femoral cutaneous

Go

Next question

Theme from April 2012 Exam


The cutaneous branch of the obturator nerve is frequently absent. However, the obturator
nerve is a recognised contributor to innervation of the medial thigh and large pelvic tumours
may compress this nerve with resultant pain radiating distally.
Obturator nerve
The obturator nerve arises from L2, L3 and L4 by branches from the ventral divisions of
each of these nerve roots. L3 forms the main contribution and the second lumbar branch is
occasionally absent. These branches unite in the substance of psoas major, descending
vertically in its posterior part to emerge from its medial border at the lateral margin of the
sacrum. It then crosses the sacroiliac joint to enter the lesser pelvis, it descends on obturator
internus to enter the obturator groove. In the lesser pelvis the nerve lies lateral to the
internal iliac vessels and ureter, and is joined by the obturator vessels lateral to the ovary or
ductus deferens.
Supplies
Medial compartment of thigh
Muscles supplied: external obturator, adductor longus, adductor brevis, adductor
magnus (not the lower part-sciatic nerve), gracilis
The cutaneous branch is often absent. When present, it passes between gracilis and
adductor longus near the middle part of the thigh, and supplies the skin and fascia of
the distal two thirds of the medial aspect.

Obturator canal
Connects the pelvis and thigh: contains the obturator artery, vein, nerve which divides
into anterior and posterior branches.

Cadaveric cross section demonstrating relationships of the obturator nerve

Image sourced from Wikipedia

Rate question:

Next question

Comment on this question

All contents of this site are 2012 E-Medical Revision Ltd

http://www.emrcs.com/question/question.php?q=0

Terms and Conditions

Privacy policy

1/1

14/05/2015

Reference ranges

Question 4 of 538

Previous

Next

A 56 year old man is undergoing a superficial parotidectomy for a pleomorphic adenoma.


During the dissection of the parotid, which of the following structures will be encountered
lying most superficially?

Question stats

End and review

Score: 100%
1

52.2%

9.3%

5.6%

10.3%

22.6%

A. Facial nerve
52.2% of users answered this
question correctly

B. External carotid artery


C. Occipital artery
D. Maxillary artery

Search

E. Retromandibular vein

Go

Next question

Most superficial structure on the parotid gland = facial nerve

The facial nerve is the most superficial structure in the parotid gland. Slightly deeper to this
lies the retromandibular vein, with the arterial layer lying most deeply.
Parotid gland
Anatomy of the parotid gland
Location

Overlying the mandibular ramus; anterior and inferior to the ear.

Salivary duct

Crosses the masseter, pierces the buccinator and drains adjacent to the
2nd upper molar tooth (Stensen's duct).

Structures passing
through the gland

Facial nerve (Mnemonic: The Zebra Buggered My Cat; Temporal


Zygomatic, Buccal, Mandibular, Cervical)
External carotid artery
Retromandibular vein
Auriculotemporal nerve

Relations

Anterior: masseter, medial pterygoid, superficial temporal and


maxillary artery, facial nerve, stylomandibular ligament
Posterior: posterior belly digastric muscle, sternocleidomastoid,
stylohyoid, internal carotid artery, mastoid process, styloid
process

Arterial supply

Branches of external carotid artery

Venous drainage

Retromandibular vein

Lymphatic
drainage

Deep cervical nodes

Nerve innervation

Parasympathetic-Secretomotor
Sympathetic-Superior cervical ganglion
Sensory- Greater auricular nerve

Parasympathetic stimulation produces a water rich, serous saliva. Sympathetic stimulation


leads to the production of a low volume, enzyme-rich saliva.
Rate question:

Next question

Comment on this question

All contents of this site are 2012 E-Medical Revision Ltd

http://www.emrcs.com/question/question.php?q=0

Terms and Conditions

Privacy policy

1/1

14/05/2015

Reference ranges

Question 5 of 538

Previous

Next

A 43 year old man is stabbed outside a nightclub. He suffers a transection of his median
nerve just as it leaves the brachial plexus. Which of the following features is least likely to
ensue?

Question stats

End and review

Score: 100%
1

17.8%

39.6%

14%

10.7%

17.9%

A. Ulnar deviation of the wrist


39.6% of users answered this
question correctly

B. Complete loss of wrist flexion


C. Loss of pronation
D. Loss of flexion at the thumb joint

Search

E. Inability to oppose the thumb

Go

Next question

Loss of the median nerve will result in loss of function of the flexor muscles. However, flexor
carpi ulnaris will still function and produce ulnar deviation and some residual wrist flexion.
High median nerve lesions result in complete loss of flexion at the thumb joint.
Median nerve
The median nerve is formed by the union of a lateral and medial root respectively from the
lateral (C5,6,7) and medial (C8 and T1) cords of the brachial plexus; the medial root passes
anterior to the third part of the axillary artery. The nerve descends lateral to the brachial
artery, crosses to its medial side (usually passing anterior to the artery). It passes deep to
the bicipital aponeurosis and the median cubital vein at the elbow.
It passes between the two heads of the pronator teres muscle, and runs on the deep surface
of flexor digitorum superficialis (within its fascial sheath).
Near the wrist it becomes superficial between the tendons of flexor digitorum superficialis and
flexor carpi radialis, deep to palmaris longus tendon. It passes deep to the flexor retinaculum
to enter the palm, but lies anterior to the long flexor tendons within the carpal tunnel.
Branches
Region

Branch

Upper
arm

No branches, although the nerve commonly communicates with the


musculocutaneous nerve

Forearm

Pronator teres
Flexor carpi radialis
Palmaris longus
Flexor digitorum superficialis
Flexor pollicis longus
Flexor digitorum profundus (only the radial half)

Distal
forearm

Palmar cutaneous branch

Hand
(Motor)

Motor supply (LOAF)

Hand
(Sensory)

Lateral 2 lumbricals
Opponens pollicis
Abductor pollicis brevis
Flexor pollicis brevis

Over thumb and lateral 2 fingers


On the palmar aspect this projects proximally, on the dorsal aspect only the
distal regions are innervated with the radial nerve providing the more proximal
cutaneous innervation.

Patterns of damage
Damage at wrist
e.g. carpal tunnel syndrome
paralysis and wasting of thenar eminence muscles and opponens pollicis (ape hand
deformity)
sensory loss to palmar aspect of lateral (radial) 2 fingers

Damage at elbow, as above plus:


unable to pronate forearm
weak wrist flexion

http://www.emrcs.com/question/question.php?q=0

1/2

14/05/2015

ulnar deviation of wrist

Anterior interosseous nerve (branch of median nerve)


leaves just below the elbow
results in loss of pronation of forearm and weakness of long flexors of thumb and index
finger

Topography of the median nerve

Image sourced from Wikipedia

Rate question:

Next question

Comment on this question

All contents of this site are 2012 E-Medical Revision Ltd

http://www.emrcs.com/question/question.php?q=0

Terms and Conditions

Privacy policy

2/2

14/05/2015

Reference ranges

Previous

Question 6 of 538

Next

A 78 year old man is due to undergo an endarterectomy of the internal carotid artery. Which
of the following nervous structures are most at risk during the dissection?

A. Recurrent laryngeal nerve


B. Sympathetic chain

Question stats

Score: 100%
1

24.4%

19%

38.4%

10.7%

7.5%

38.4% of users answered this


question correctly

C. Hypoglossal nerve

End and review

D. Phrenic nerve
E. Lingual nerve

Search
Next question

Go

Nerves at risk during a carotid endarterectomy:


Hypoglossal nerve
Greater auricular nerve
Superior laryngeal nerve

Theme from May 2011 exam


Theme from January 2013 Exam
During a carotid endarterectomy the sternocleidomastoid muscle is dissected, with ligation of
the common facial vein and then the internal jugular is dissected exposing the common and
the internal carotid arteries. The nerves at risk during the operation include:
Hypoglossal nerve
Greater auricular nerve
Superior laryngeal nerve
The sympathetic chain lies posteriorly and is less prone to injury in this procedure.
Internal carotid artery
The internal carotid artery is formed from the common carotid opposite the upper border of
the thyroid cartilage. It extends superiorly to enter the skull via the carotid canal. From the
carotid canal it then passes through the cavernous sinus, above which it divides into the
anterior and middle cerebral arteries.
Relations in the neck
Posterior

Medially

Laterally

Anteriorly

Longus capitis
Pre-vertebral fascia
Sympathetic chain
Superior laryngeal nerve

External carotid (near origin)


Wall of pharynx
Ascending pharyngeal artery

Internal jugular vein (moves posteriorly at entrance to skull)


Vagus nerve (most posterolaterally)

Sternocleidomastoid
Lingual and facial veins
Hypoglossal nerve

Relations in the carotid canal


Internal carotid plexus
Cochlea and middle ear cavity
Trigeminal ganglion (superiorly)
Leaves canal lies above the foramen lacerum

Path and relations in the cranial cavity


The artery bends sharply forwards in the cavernous sinus, the aducens nerve lies close to its
inferolateral aspect. The oculomotor, trochlear, opthalmic and, usually, the maxillary nerves

http://www.emrcs.com/question/question.php?q=0

1/2

14/05/2015

lie in the lateral wall of the sinus. Near the superior orbital fissure it turns posteriorly and
passes postero-medially to pierce the roof of the cavernous sinus inferior to the optic nerve.
It then passes between the optic and oculomotor nerves to terminate below the anterior
perforated substance by dividing into the anterior and middle cerebral arteries.
Branches
Anterior and middle cerebral artery
Ophthalmic artery
Posterior communicating artery
Anterior choroid artery
Meningeal arteries
Hypophyseal arteries

Image demonstrating the internal carotid artery and its relationship to the external carotid
artery

Image sourced from Wikipedia

Rate question:

Next question

Comment on this question

All contents of this site are 2012 E-Medical Revision Ltd

http://www.emrcs.com/question/question.php?q=0

Terms and Conditions

Privacy policy

2/2

14/05/2015

Reference ranges

Question 7 of 538

Previous

Next

Question stats

End and review

Score: 100%
1

9.8%

41.2%

12%

A. Waldeyers fascia

22.3%

B. Sibsons fascia

14.7%

C. Pretracheal fascia

41.2% of users answered this


question correctly

Which of the following fascial structures encases the apex of the lungs?

D. Clavipectoral fascia

6
7

E. None of the above


Search
Next question
Go

Sibson's fascia overlies the apices of both lungs

The suprapleural fascia (Sibson's fascia) runs from C7 to the first rib and overlies the apex of
both lungs.It lies between the parietal pleura and the thoracic cage.
Lung anatomy
The right lung is composed of 3 lobes divided by the oblique and transverse fissures. The
left lung has two lobes divided by the oblique fissure.The apex of both lungs is approximately
4cm superior to the sterno-costal joint of the first rib. Immediately below this is a sulcus
created by the subclavian artery.
Peripheral contact points of the lung
Base: diaphragm
Costal surface: corresponds to the cavity of the chest
Mediastinal surface: Contacts the mediastinal pleura. Has the cardiac impression.
Above and behind this concavity is a triangular depression named the hilum, where
the structures which form the root of the lung enter and leave the viscus. These
structures are invested by pleura, which, below the hilum and behind the pericardial
impression, forms the pulmonary ligament

Right lung
Above the hilum is the azygos vein; Superior to this is the groove for the superior vena cava
and right innominate vein; behind this, and nearer the apex, is a furrow for the innominate
artery. Behind the hilum and the attachment of the pulmonary ligament is a vertical groove
for the oesophagus; In front and to the right of the lower part of the oesophageal groove is a
deep concavity for the extrapericardiac portion of the inferior vena cava.
The root of the right lung lies behind the superior vena cava and the right atrium, and below
the azygos vein.
The right main bronchus is shorter, wider and more vertical than the left main bronchus and
therefore the route taken by most foreign bodies.

Image sourced from Wikipedia

Left lung
Above the hilum is the furrow produced by the aortic arch, and then superiorly the groove
accommodating the left subclavian artery; Behind the hilum and pulmonary ligament is a
vertical groove produced by the descending aorta, and in front of this, near the base of the
lung, is the lower part of the oesophagus.
The root of the left lung passes under the aortic arch and in front of the descending aorta.

http://www.emrcs.com/question/question.php?q=0

1/2

14/05/2015

Image sourced from Wikipedia

Inferior borders of both lungs


6th rib in mid clavicular line
8th rib in mid axillary line
10th rib posteriorly
The pleura runs two ribs lower than the corresponding lung level.
Bronchopulmonary segments
Segment number

Right lung

Left lung

Apical

Apical

Posterior

Posterior

Anterior

Anterior

Lateral

Superior lingular

Medial

Inferior lingular

Superior (apical)

Superior (apical)

Medial basal

Medial basal

Anterior basal

Anterior basal

Lateral basal

Lateral basal

10

Posterior basal

Posterior basal

Rate question:

Next question

Comment on this question

All contents of this site are 2012 E-Medical Revision Ltd

http://www.emrcs.com/question/question.php?q=0

Terms and Conditions

Privacy policy

2/2

14/05/2015

tornadoamr@gmail.com - MRCS Part A - My account


End and review
Reference ranges

Question stats
A
B
C
D
E

10.2%
15.2%
42.7%
18.5%
13.4%

42.7% of users answered this question correctly

Search
Go
Score: 100%
1
2
3
4
5
6
7

8
Previous

Question 8 of 538

Next

With regards to the internal jugular vein, which of the following statements is untrue?
1 A. It lies within the carotid sheath
2 B. It is the continuation of the sigmoid sinus
3 C. The terminal part of the thoracic duct crosses anterior to it to
insert into the right subclavian vein
4 D. The hypoglossal nerve is closely related to it as it passes near the
atlas
5 E. The vagus nerve is closely related to it within the carotid sheath

Next question
Theme from April 2013 Exam
Internal jugular vein
Each jugular vein begins in the jugular foramen, where they are the continuation of the sigmoid sinus. They terminate at the medial end
of the clavicle where they unite with the subclavian vein.
The vein lies within the carotid sheath throughout its course. Below the skull the internal carotid artery and last four cranial nerves are
anteromedial to the vein. Thereafter it is in contact medially with the internal (then common) carotid artery. The vagus lies
posteromedially.
At its superior aspect, the vein is overlapped by sternocleidomastoid and covered by it at the inferior aspect of the vein.
Below the transverse process of the atlas it is crossed on its lateral side by the accessory nerve. At its mid point it is crossed by the
inferior root of the ansa cervicalis.
Posterior to the vein are the transverse processes of the cervical vertebrae, the phenic nerve as it descends on the scalenus anterior,
and the first part of the subclavian artery.
On the left side its also related to the thoracic duct.
Image sourced from Wikipedia

http://www.emrcs.com/question/question.php?q=0

1/2

14/05/2015

Submit answer
3.32
1
2
Rate question: 3
4
5

Next question

Comment on this question


All contents of this site are 2012 E-Medical Revision Ltd

http://www.emrcs.com/question/question.php?q=0

Terms and Conditions

Privacy policy

2/2

14/05/2015

Reference ranges

Question 8 of 538

Previous

Next

Question stats

End and review

Score: 100%
1

10.2%

15.2%

42.7%

A. It lies within the carotid sheath

18.5%

B. It is the continuation of the sigmoid sinus

13.4%

C. The terminal part of the thoracic duct crosses anterior to it to insert into
the right subclavian vein

42.7% of users answered this


question correctly

With regards to the internal jugular vein, which of the following statements is untrue?

D. The hypoglossal nerve is closely related to it as it passes near the atlas

6
7
8

E. The vagus nerve is closely related to it within the carotid sheath


Search
Go

Next question

Theme from April 2013 Exam


Internal jugular vein
Each jugular vein begins in the jugular foramen, where they are the continuation of the
sigmoid sinus. They terminate at the medial end of the clavicle where they unite with the
subclavian vein.
The vein lies within the carotid sheath throughout its course. Below the skull the internal
carotid artery and last four cranial nerves are anteromedial to the vein. Thereafter it is in
contact medially with the internal (then common) carotid artery. The vagus lies
posteromedially.
At its superior aspect, the vein is overlapped by sternocleidomastoid and covered by it at the
inferior aspect of the vein.
Below the transverse process of the atlas it is crossed on its lateral side by the accessory
nerve. At its mid point it is crossed by the inferior root of the ansa cervicalis.
Posterior to the vein are the transverse processes of the cervical vertebrae, the phenic
nerve as it descends on the scalenus anterior, and the first part of the subclavian artery.
On the left side its also related to the thoracic duct.

Image sourced from Wikipedia

Rate question:

Next question

Comment on this question

All contents of this site are 2012 E-Medical Revision Ltd

http://www.emrcs.com/question/question.php?q=0

Terms and Conditions

Privacy policy

1/1

14/05/2015

tornadoamr@gmail.com - MRCS Part A - My account


End and review
Reference ranges

Question stats
A
B
C
D
E

18.5%
40%
11.4%
23.5%
6.5%

40% of users answered this question correctly

Search
Go
Score: 100%
1
2
3
4
5
6
7
8

9
Question 9 of 538

Previous

Next

A 28 year old man requires a urethral catheter to be inserted prior to undergoing a splenectomy. Where is the first site of resistance to
be encountered on inserting the catheter?
1
2
3
4
5

A. Bulbar urethra
B. Membranous urethra
C. Internal sphincter
D. Prostatic urethra
E. Bladder neck

Next question
Theme from January 2012 Exam
Theme from April 2014 Exam
The membranous urethra is the least distensible portion of the urethra. This is due to the fact that it is surrounded by the external
sphincter.

Urethral anatomy
Female urethra
The female urethra is shorter and more acutely angulated than the male urethra. It is an extra-peritoneal structure and embedded in the
endopelvic fascia. The neck of the bladder is subjected to transmitted intra-abdominal pressure and therefore deficiency in this area
may result in stress urinary incontinence. Between the layers of the urogenital diaphragm the female urethra is surrounded by the
external urethral sphincter, this is innervated by the pudendal nerve. It ultimately lies anterior to the vaginal orifice.

Male urethra
In males the urethra is much longer and is divided into four parts.

Pre-prostatic Extremely short and lies between the bladder and prostate gland.It has a stellate lumen and is between 1 and 1.5cm
long.Innervated by sympathetic noradrenergic fibres, as this region is composed of striated muscles bundles they may
urethra
contract and prevent retrograde ejaculation.

Prostatic
This segment is wider than the membranous urethra and contains several openings for the transmission of semen (at
the midpoint of the urethral crest).
urethra
Membranous Narrowest part of the urethra and surrounded by external sphincter. It traverses the perineal membrane 2.5cm
http://www.emrcs.com/question/question.php?q=0

1/2

14/05/2015

urethra
Penile
urethra

postero-inferior to the symphysis pubis.


Travels through the corpus spongiosum on the underside of the penis. It is the longest urethral segment.It is dilated at
its origin as the infrabulbar fossa and again in the gland penis as the navicular fossa. The bulbo-urethral glands open
into the spongiose section of the urethra 2.5cm below the perineal membrane.

The urothelium is transitional in nature near to the bladder and becomes squamous more distally.

Submit answer
4.3
1
2
Rate question: 3
4
5

Next question

Comment on this question


All contents of this site are 2012 E-Medical Revision Ltd

http://www.emrcs.com/question/question.php?q=0

Terms and Conditions

Privacy policy

2/2

14/05/2015

Reference ranges

Previous

Question 9 of 538

Next

A 28 year old man requires a urethral catheter to be inserted prior to undergoing a


splenectomy. Where is the first site of resistance to be encountered on inserting the
catheter?

Question stats

Score: 100%
1

18.5%

40%

11.4%

23.5%

6.5%

A. Bulbar urethra
40% of users answered this
question correctly

B. Membranous urethra

End and review

6
7

C. Internal sphincter
8

D. Prostatic urethra
9

Search

E. Bladder neck

Go

Next question

Theme from January 2012 Exam


Theme from April 2014 Exam
The membranous urethra is the least distensible portion of the urethra. This is due to the
fact that it is surrounded by the external sphincter.
Urethral anatomy
Female urethra
The female urethra is shorter and more acutely angulated than the male urethra. It is an
extra-peritoneal structure and embedded in the endopelvic fascia. The neck of the bladder is
subjected to transmitted intra-abdominal pressure and therefore deficiency in this area may
result in stress urinary incontinence. Between the layers of the urogenital diaphragm the
female urethra is surrounded by the external urethral sphincter, this is innervated by the
pudendal nerve. It ultimately lies anterior to the vaginal orifice.
Male urethra
In males the urethra is much longer and is divided into four parts.
Pre-prostatic
urethra

Extremely short and lies between the bladder and prostate gland.It has a
stellate lumen and is between 1 and 1.5cm long.Innervated by sympathetic
noradrenergic fibres, as this region is composed of striated muscles bundles
they may contract and prevent retrograde ejaculation.

Prostatic
urethra

This segment is wider than the membranous urethra and contains several
openings for the transmission of semen (at the midpoint of the urethral crest).

Membranous
urethra

Narrowest part of the urethra and surrounded by external sphincter. It traverses


the perineal membrane 2.5cm postero-inferior to the symphysis pubis.

Penile
urethra

Travels through the corpus spongiosum on the underside of the penis. It is the
longest urethral segment.It is dilated at its origin as the infrabulbar fossa and
again in the gland penis as the navicular fossa. The bulbo-urethral glands open
into the spongiose section of the urethra 2.5cm below the perineal membrane.

The urothelium is transitional in nature near to the bladder and becomes squamous more
distally.
Rate question:

Next question

Comment on this question

All contents of this site are 2012 E-Medical Revision Ltd

http://www.emrcs.com/question/question.php?q=0

Terms and Conditions

Privacy policy

1/1

14/05/2015

Reference ranges

Previous

Question 10 of 538

Next

Question stats

End and review

Score: 100%
1

9.2%

62.3%

8.2%

A. Median nerve

8.4%

B. Radial nerve

11.9%

C. Tendon of triceps

62.3% of users answered this


question correctly

Which of the following anatomical structures lies within the spiral groove of the humerus?

D. Musculocutaneous nerve
E. Axillary nerve

6
7
8
9

Search
Next question
Go

10

Theme from April 2014 Exam


The radial nerve lies in this groove and may be compromised by fractures involving the shaft.
Humerus
The humerus extends from the scapula to the elbow joint. It has a body and two ends. It is
almost completely covered with muscle but can usually be palpated throughout its length.
The smooth rounded surface of the head articulates with the shallow glenoid cavity. The
head is connected to the body of the humerus by the anatomical neck. The surgical neck is
the region below the head and tubercles and where they join the shaft and is the commonest
site of fracture. The capsule of the shoulder joint is attached to the anatomical neck
superiorly but extends down to 1.5cm on the surgical neck.
The greater tubercle is the prominence on the lateral side of the upper end of the bone. It
merges with the body below and can be felt through the deltoid inferior to the acromion. The
tendons of the supraspinatus and infraspinatus are inserted into impressions on its superior
aspect. The lesser tubercle is a distinct prominence on the front of the upper end of the
bone. It can be palpated through the deltoid just lateral to the tip of the coracoid process.
The intertubercular groove passes on the body between the greater and lesser tubercles,
continuing down from the anterior borders of the tubercles to form the edges of the groove.
The tendon of biceps within its synovial sheath passes through this groove, held within it by a
transverse ligament.
The posterior surface of the body is marked by a spiral groove for the radial nerve which
runs obliquely across the upper half of the body to reach the lateral border below the deltoid
tuberosity. Within this groove lie the radial nerve and brachial vessels and both may be
affected by fractures involving the shaft of the humerus.
The lower end of the humerus is wide and flattened anteroposteriorly, and inclined anteriorly.
The middle third of the distal edge forms the trochlea. Superior to this are indentations for
the coronoid fossa anteriorly and olecranon fossa posteriorly. Lateral to the trochlea is a
rounded capitulum which articulates with the radius.
The medial epicondyle is very prominent with a smooth posterior surface which contains a
sulcus for the ulnar nerve and collateral vessels. It's distal margin gives attachment for the
ulnar collateral ligament and, in front of this, the anterior surface has an impression for the
common flexor tendon.
Rate question:

Next question

Comment on this question

All contents of this site are 2012 E-Medical Revision Ltd

http://www.emrcs.com/question/question.php?q=0

Terms and Conditions

Privacy policy

1/1

14/05/2015

Reference ranges

Question 11 of 538

Previous

Next

A 24 year old man falls and sustains a fracture through his scaphoid bone. From which of the
following areas does the scaphoid derive the majority of its blood supply?

A. From its proximal medial border


B. From its proximal lateral border

Question stats

Score: 100%
1

12.2%

14.4%

11.1%

15.6%

46.7%

46.7% of users answered this


question correctly

C. From its proximal posterior surface

End and review

D. From the proximal end

6
7
8

E. From the distal end

Search
Go

Next question

10
11

Theme from April 2012 Exam


Theme from April 2014 Exam
The blood supply to the scaphoid enters from a small non articular surface near its distal
end. Transverse fractures through the scaphoid therefore carry a risk of non union.
Scaphoid bone
The scaphoid has a concave articular surface for the head of the capitate and at the edge of
this is a crescentic surface for the corresponding area on the lunate.
Proximally, it has a wide convex articular surface with the radius. It has a distally sited
tubercle that can be palpated. The remaining articular surface is to the lateral side of the
tubercle. It faces laterally and is associated with the trapezium and trapezoid bones.
The narrow strip between the radial and trapezial surfaces and the tubercle gives rise to the
radial collateral carpal ligament. The tubercle receives part of the flexor retinaculum. This
area is the only part of the scaphoid that is available for the entry of blood vessels. It is
commonly fractured and avascular necrosis may result.
Scaphoid bone

Image sourced from Wikipedia

Rate question:

Next question

Comment on this question

All contents of this site are 2012 E-Medical Revision Ltd

http://www.emrcs.com/question/question.php?q=0

Terms and Conditions

Privacy policy

1/1

14/05/2015

Reference ranges

Question 12 of 538

Previous

Next

A 21 year old man has an inguinal hernia and is undergoing a surgical repair. As the
surgeons approach the inguinal canal they expose the superficial inguinal ring. Which of the
following forms the lateral edge of this structure?

Question stats

Score: 100%
1

17.8%

17%

8.2%

45.3%

11.8%

A. Inferior epigastric artery


45.3% of users answered this
question correctly

B. Conjoint tendon

End and review

6
7

C. Rectus abdominis muscle


8

D. External oblique aponeurosis


9

Search

E. Transversalis fascia

Go

Next question

10
11
12

The external oblique aponeurosis forms the anterior wall of the inguinal canal and also the
lateral edge of the superficial inguinal ring. The rectus abdominis lies posteromedially and
the transversalis posterior to this.
Inguinal canal
Location
Above the inguinal ligament
The inguinal canal is 4cm long
The superficial ring is located anterior to the pubic tubercle
The deep ring is located approximately 1.5-2cm above the half way point between the
anterior superior iliac spine and the pubic tubercle

Boundaries of the inguinal canal


Floor

External oblique aponeurosis


Inguinal ligament
Lacunar ligament

Roof

Internal oblique
Transversus abdominis

Anterior wall
Posterior wall

Laterally

Medially

External oblique aponeurosis


Transversalis fascia
Conjoint tendon

Internal ring
Fibres of internal oblique

External ring
Conjoint tendon

Contents
Males

Spermatic cord and ilioinguinal


nerve

As it passes through the canal the spermatic


cord has 3 coverings:
External spermatic fascia from external
oblique aponeurosis
Cremasteric fascia
Internal spermatic fascia

Females

Round ligament of uterus and


ilioinguinal nerve

Related anatomy of the inguinal region


The boundaries of Hesselbachs triangle are commonly tested and illustrated below:

http://www.emrcs.com/question/question.php?q=0

1/2

14/05/2015

Image sourced from Wikipedia

The image below demonstrates the close relationship of the vessels to the lower limb with the
inguinal canal. A fact to be borne in mind when repairing hernial defects in this region.

Image sourced from Wikipedia

Rate question:

Next question

Comment on this question

All contents of this site are 2012 E-Medical Revision Ltd

http://www.emrcs.com/question/question.php?q=0

Terms and Conditions

Privacy policy

2/2

14/05/2015

Reference ranges

Question 13 of 538

Previous

Next

Question stats

End and review

Score: 100%
1

13.7%

46.2%

16.7%

A. Transverse sinus

13.2%

B. Superior sagittal sinus

10.2%

C. Cavernous sinus

46.2% of users answered this


question correctly

Which of the following cranial venous sinuses is unpaired?

D. Sigmoid sinus
E. Inferior petrosal sinus

6
7
8
9

Search
Next question
Go

10
11

The superior sagittal sinus is unpaired

12
13

The superior sagittal sinus is unpaired. It begins at the crista galli, where it may communicate
with the veins of the frontal sinus and sometimes with those of the nasal cavity. It arches
backwards in the falx cerebri to terminate at the internal occipital protuberance (usually into
the right transverse sinus). The parietal emissary veins link the superior sagittal sinus with
the veins on the exterior of the cranium.
Cranial venous sinuses
The cranial venous sinuses are located within the dura mater. They have no valves which is
important in the potential for spreading sepsis. They eventually drain into the internal jugular
vein.
They are:
Superior sagittal sinus
Inferior sagittal sinus
Straight sinus
Transverse sinus
Sigmoid sinus
Confluence of sinuses
Occipital sinus
Cavernous sinus
Topography of cranial venous sinuses

Image sourced from Wikipedia

Rate question:

Next question

Comment on this question

All contents of this site are 2012 E-Medical Revision Ltd

http://www.emrcs.com/question/question.php?q=0

Terms and Conditions

Privacy policy

1/1

14/05/2015

Reference ranges

Previous

Question 14 of 538

Next

Which of the following laryngeal tumours will not typically metastasise to the cervical lymph
nodes?

A. Glottic
B. Supraglottic

Question stats

Score: 100%
1

32.2%

17.9%

19.1%

12%

18.8%

32.2% of users answered this


question correctly

C. Subglottic

End and review

D. Transglottic

6
7
8

E. Aryepiglottic fold

Search
Next question

Go

10
11

The vocal cords have no lymphatic drainage and therefore this region serves as a lymphatic
watershed. The supraglottic part drains to the upper deep cervical nodes through vessels
piercing the thyrohyoid membrane. The sub glottic part drains to the pre laryngeal, pre
tracheal and inferior deep cervical nodes. The aryepiglottic and vestibular folds have a rich
lymphatic drainage and will metastasise early.

12
13
14

Larynx
The larynx lies in the anterior part of the neck at the levels of C3 to C6 vertebral bodies. The
laryngeal skeleton consists of a number of cartilagenous segments. Three of these are
paired; arytenoid, corniculate and cuneiform. Three are single; thyroid, cricoid and epiglottic.
The cricoid cartilage forms a complete ring (the only one to do so).
The laryngeal cavity extends from the laryngeal inlet to the level of the inferior border of the
cricoid cartilage.
Divisions of the laryngeal cavity
Laryngeal vestibule

Superior to the vestibular folds

Laryngeal ventricle

Lies between vestibular folds and superior to the vocal cords

Infraglottic cavity

Extends from vocal cords to inferior border of the cricoid cartilage

The vocal folds (true vocal cords) control sound production. The apex of each fold projects
medially into the laryngeal cavity. Each vocal fold includes:
Vocal ligament
Vocalis muscle (most medial part of thyroarytenoid muscle)
The glottis is composed of the vocal folds, processes and rima glottidis. The rima glottidis is
the narrowest potential site within the larynx, as the vocal cords may be completely opposed,
forming a complete barrier.
Muscles of the larynx
Muscle

Origin

Insertion

Innervation

Action

Posterior
cricoarytenoid

Posterior aspect
of lamina of
cricoid

Muscular process of
arytenoid

Recurrent
Laryngeal

Abducts vocal
fold

Lateral
cricoarytenoid

Arch of cricoid

Muscular process of
arytenoid

Recurrent
laryngeal

Adducts vocal
fold

Thyroarytenoid

Posterior aspect
of thyroid
cartilage

Muscular process of
arytenoid

Recurrent
laryngeal

Relaxes vocal
fold

Transverse
and oblique
arytenoids

Arytenoid
cartilage

Contralateral
arytenoid

Recurrent
laryngeal

Closure of
intercartilagenous
part of the rima
glottidis

Vocalis

Depression
between lamina
of thyroid
cartilage

Vocal ligament and


vocal process of
arytenoid cartilage

Recurrent
laryngeal

Relaxes posterior
vocal ligament,
tenses anterior
part

Cricothyroid

Anterolateral part
of cricoid

Inferior margin and


horn of thyroid
cartilage

External
laryngeal

Tenses vocal fold

http://www.emrcs.com/question/question.php?q=0

1/2

14/05/2015

Blood supply
Arterial supply is via the laryngeal arteries, branches of the superior and inferior thyroid
arteries. The superior laryngeal artery is closely related to the internal laryngeal nerve. The
inferior laryngeal artery is related to the inferior laryngeal nerve. Venous drainage is via
superior and inferior laryngeal veins, the former draining into the superior thyroid vein and
the latter draining into the middle thyroid vein, or thyroid venous plexus.
Lymphatic drainage
The vocal cords have no lymphatic drainage and this site acts as a lymphatic watershed.
Supraglottic part

Upper deep cervical nodes

Subglottic part

Prelaryngeal and pretracheal nodes and inferior deep cervical nodes

The aryepiglottic fold and vestibular folds have a dense plexus of lymphatics associated with
them and malignancies at these sites have a greater propensity for nodal metastasis.

Topography of the larynx

Image sourced from Wikipedia

Rate question:

Next question

Comment on this question

All contents of this site are 2012 E-Medical Revision Ltd

http://www.emrcs.com/question/question.php?q=0

Terms and Conditions

Privacy policy

2/2

14/05/2015

Reference ranges

Question 15 of 538

Previous

Next

Question stats

End and review

Score: 100%
1

12.9%

27%

12.5%

A. Pectineal ligament

38.5%

B. Adductor longus

9.2%

C. Sartorius

38.5% of users answered this


question correctly

Which of the following forms the medial wall of the femoral canal?

D. Lacunar ligament
E. Inguinal ligament

6
7
8
9

Search
Next question
Go

10
11

The femoral canal and the femoral triangle are distinct anatomical structures. Do not
confuse them, especially in the time pressured exam situation.

12
13
14

Theme from September 2011 exam


Theme from September 2012 exam
Theme from April 2014 exam

15

Femoral canal
The femoral canal lies at the medial aspect of the femoral sheath. The femoral sheath is a
fascial tunnel containing both the femoral artery laterally and femoral vein medially. The
canal lies medial to the vein.
Borders of the femoral canal
Laterally

Femoral vein

Medially

Lacunar ligament

Anteriorly

Inguinal ligament

Posteriorly

Pectineal ligament

Image showing dissection of femoral canal

Image sourced from Wikipedia

Contents
Lymphatic vessels
Cloquet's lymph node

Physiological significance
Allows the femoral vein to expand to allow for increased venous return to the lower limbs.

http://www.emrcs.com/question/question.php?q=0

1/2

14/05/2015

Pathological significance
As a potential space, it is the site of femoral hernias. The relatively tight neck places these at
high risk of strangulation.
Rate question:

Next question

Comment on this question

All contents of this site are 2012 E-Medical Revision Ltd

http://www.emrcs.com/question/question.php?q=0

Terms and Conditions

Privacy policy

2/2

14/05/2015

Reference ranges

Previous

Question 16 of 538

Next

A 67 year old man is undergoing a transurethral resection of a bladder tumour using


diathermy. Suddenly during the procedure the patient's thigh begins to twitch. Stimulation of
which of the following nerves is the most likely cause?

Question stats

Score: 100%
1

13.3%

22.2%

13.7%

43.9%

6.8%

A. Femoral
43.9% of users answered this
question correctly

B. Pudendal

End and review

6
7

C. Sciatic
8

D. Obturator
9

Search

E. Gluteal

Go

Next question

10
11
12

Theme from January 2011 Exam


Theme from January 2013 Exam
The obturator nerve is most closely related to the bladder (see below)

13
14
15
16

Image sourced from Wikipedia

Obturator nerve
The obturator nerve arises from L2, L3 and L4 by branches from the ventral divisions of
each of these nerve roots. L3 forms the main contribution and the second lumbar branch is
occasionally absent. These branches unite in the substance of psoas major, descending
vertically in its posterior part to emerge from its medial border at the lateral margin of the
sacrum. It then crosses the sacroiliac joint to enter the lesser pelvis, it descends on obturator
internus to enter the obturator groove. In the lesser pelvis the nerve lies lateral to the
internal iliac vessels and ureter, and is joined by the obturator vessels lateral to the ovary or
ductus deferens.
Supplies
Medial compartment of thigh
Muscles supplied: external obturator, adductor longus, adductor brevis, adductor
magnus (not the lower part-sciatic nerve), gracilis
The cutaneous branch is often absent. When present, it passes between gracilis and
adductor longus near the middle part of the thigh, and supplies the skin and fascia of
the distal two thirds of the medial aspect.

Obturator canal
Connects the pelvis and thigh: contains the obturator artery, vein, nerve which divides
into anterior and posterior branches.

Cadaveric cross section demonstrating relationships of the obturator nerve

http://www.emrcs.com/question/question.php?q=0

1/2

14/05/2015

Image sourced from Wikipedia

Rate question:

Next question

Comment on this question

All contents of this site are 2012 E-Medical Revision Ltd

http://www.emrcs.com/question/question.php?q=0

Terms and Conditions

Privacy policy

2/2

14/05/2015

Reference ranges

Previous

Question 16 of 538

Next

A 67 year old man is undergoing a transurethral resection of a bladder tumour using


diathermy. Suddenly during the procedure the patient's thigh begins to twitch. Stimulation of
which of the following nerves is the most likely cause?

Question stats

Score: 100%
1

13.3%

22.2%

13.7%

43.9%

6.8%

A. Femoral
43.9% of users answered this
question correctly

B. Pudendal

End and review

6
7

C. Sciatic
8

D. Obturator
9

Search

E. Gluteal

Go

Next question

10
11
12

Theme from January 2011 Exam


Theme from January 2013 Exam
The obturator nerve is most closely related to the bladder (see below)

13
14
15
16

Image sourced from Wikipedia

Obturator nerve
The obturator nerve arises from L2, L3 and L4 by branches from the ventral divisions of
each of these nerve roots. L3 forms the main contribution and the second lumbar branch is
occasionally absent. These branches unite in the substance of psoas major, descending
vertically in its posterior part to emerge from its medial border at the lateral margin of the
sacrum. It then crosses the sacroiliac joint to enter the lesser pelvis, it descends on obturator
internus to enter the obturator groove. In the lesser pelvis the nerve lies lateral to the
internal iliac vessels and ureter, and is joined by the obturator vessels lateral to the ovary or
ductus deferens.
Supplies
Medial compartment of thigh
Muscles supplied: external obturator, adductor longus, adductor brevis, adductor
magnus (not the lower part-sciatic nerve), gracilis
The cutaneous branch is often absent. When present, it passes between gracilis and
adductor longus near the middle part of the thigh, and supplies the skin and fascia of
the distal two thirds of the medial aspect.

Obturator canal
Connects the pelvis and thigh: contains the obturator artery, vein, nerve which divides
into anterior and posterior branches.

Cadaveric cross section demonstrating relationships of the obturator nerve

http://www.emrcs.com/question/question.php?q=0

1/2

14/05/2015

Image sourced from Wikipedia

Rate question:

Next question

Comment on this question

All contents of this site are 2012 E-Medical Revision Ltd

http://www.emrcs.com/question/question.php?q=0

Terms and Conditions

Privacy policy

2/2

14/05/2015

Reference ranges

Question 17 of 538

Previous

Next

A 5 year old boy is playing with some small ball bearings. Unfortunately he inhales one. To
which of the following lung regions is the ball most likely to settle?

A. Right lower lobe


B. Left main bronchus

Question stats

Score: 100%
1

55.8%

10.3%

14.4%

7.5%

12%

55.8% of users answered this


question correctly

C. Right upper lobe

End and review

D. Left lower lobe

6
7
8

E. None of the above

Search
Next question

Go

10
11

Theme from September 2011 Exam


Theme from January 2013 Exam
As the most dependent part of the right lung a small object is most likely to lodge here. Most
objects will preferentially enter the right lung owing to the angle the right main bronchus
takes from the trachea.

12
13
14
15
16

Lung anatomy

17

The right lung is composed of 3 lobes divided by the oblique and transverse fissures. The
left lung has two lobes divided by the oblique fissure.The apex of both lungs is approximately
4cm superior to the sterno-costal joint of the first rib. Immediately below this is a sulcus
created by the subclavian artery.
Peripheral contact points of the lung
Base: diaphragm
Costal surface: corresponds to the cavity of the chest
Mediastinal surface: Contacts the mediastinal pleura. Has the cardiac impression.
Above and behind this concavity is a triangular depression named the hilum, where
the structures which form the root of the lung enter and leave the viscus. These
structures are invested by pleura, which, below the hilum and behind the pericardial
impression, forms the pulmonary ligament

Right lung
Above the hilum is the azygos vein; Superior to this is the groove for the superior vena cava
and right innominate vein; behind this, and nearer the apex, is a furrow for the innominate
artery. Behind the hilum and the attachment of the pulmonary ligament is a vertical groove
for the oesophagus; In front and to the right of the lower part of the oesophageal groove is a
deep concavity for the extrapericardiac portion of the inferior vena cava.
The root of the right lung lies behind the superior vena cava and the right atrium, and below
the azygos vein.
The right main bronchus is shorter, wider and more vertical than the left main bronchus and
therefore the route taken by most foreign bodies.

Image sourced from Wikipedia

Left lung
Above the hilum is the furrow produced by the aortic arch, and then superiorly the groove
accommodating the left subclavian artery; Behind the hilum and pulmonary ligament is a
vertical groove produced by the descending aorta, and in front of this, near the base of the
lung, is the lower part of the oesophagus.
The root of the left lung passes under the aortic arch and in front of the descending aorta.

http://www.emrcs.com/question/question.php?q=0

1/2

14/05/2015

Image sourced from Wikipedia

Inferior borders of both lungs


6th rib in mid clavicular line
8th rib in mid axillary line
10th rib posteriorly
The pleura runs two ribs lower than the corresponding lung level.
Bronchopulmonary segments
Segment number

Right lung

Left lung

Apical

Apical

Posterior

Posterior

Anterior

Anterior

Lateral

Superior lingular

Medial

Inferior lingular

Superior (apical)

Superior (apical)

Medial basal

Medial basal

Anterior basal

Anterior basal

Lateral basal

Lateral basal

10

Posterior basal

Posterior basal

Rate question:

Next question

Comment on this question

All contents of this site are 2012 E-Medical Revision Ltd

http://www.emrcs.com/question/question.php?q=0

Terms and Conditions

Privacy policy

2/2

14/05/2015

Reference ranges

Previous

Question 18 of 538

Next

A patient presents with superior vena caval obstruction. How many collateral circulations exist
as alternative pathways of venous return?

A. None
B. One

Question stats

Score: 100%
1

22.8%

14.9%

19.2%

14.4%

28.7%

28.7% of users answered this


question correctly

C. Two

End and review

D. Three

6
7
8

E. Four

Search
Go

Next question

10
11
12

There are 4 collateral venous systems:

13

Azygos venous system


Internal mammary venous pathway
Long thoracic venous system with connections to the femoral and vertebral veins (2
pathways)

14
15
16
17

Despite this, venous hypertension still occurs.

18

Superior vena cava


Drainage
Head and neck
Upper limbs
Thorax
Part of abdominal walls

Formation
Subclavian and internal jugular veins unite to form the right and left brachiocephalic
veins
These unite to form the SVC
Azygos vein joins the SVC before it enters the right atrium

Relations
Anterior

Anterior margins of the right lung and pleura

Posteromedial

Trachea and right vagus nerve

Posterolateral

Posterior aspects of right lung and pleura


Pulmonary hilum is posterior

Right lateral

Right phrenic nerve and pleura

Left lateral

Brachiocephalic artery and ascending aorta

Developmental variations
Anomalies of the connection of the SVC are recognised. In some individuals a persistent left
sided SVC drains into the right atrium via an enlarged orifice of the coronary sinus. More
rarely the left sided vena cava may connect directly with the superior aspect of the left
atrium, usually associated with an un-roofing of the coronary sinus. The commonest lesion of
the IVC is for its abdominal course to be interrupted, with drainage achieved via the azygos
venous system. This may occur in patients with left sided atrial isomerism.
Rate question:

Next question

Comment on this question

All contents of this site are 2012 E-Medical Revision Ltd

http://www.emrcs.com/question/question.php?q=0

Terms and Conditions

Privacy policy

1/1

14/05/2015

Reference ranges

Previous

Question 19 of 538

Next

An 18 year old man is cutting some plants when a small piece of vegetable matter enters his
eye. His eye becomes watery. Which of the following is responsible for relaying
parasympathetic neuronal signals to the lacrimal apparatus?

Question stats

Score: 100%
1

33.1%

17.8%

6.8%

33.7%

8.7%

A. Pterygopalatine ganglion
33.1% of users answered this
question correctly

B. Otic ganglion

End and review

6
7

C. Submandibular ganglion
8

D. Ciliary ganglion
9

Search

E. None of the above

Go

10
11

Next question

12

Theme from January 2013 Exam


The parasympathetic fibres to the lacrimal apparatus transit via the pterygopalatine ganglion.

13
14
15

Lacrimal system

16
17

Lacrimal gland
Consists of an orbital part and palpebral part. They are continuous posterolaterally around
the concave lateral edge of the levator palpebrae superioris muscle.
The ducts of the lacrimal gland open into the superior fornix. Those from the orbital part
penetrate the aponeurosis of levator palpebrae superioris to join those from the palpebral
part. Therefore excision of the palpebral part is functionally similar to excision of the entire
gland.

18
19

Blood supply
Lacrimal branch of the opthalmic artery. Venous drainage is to the superior opthalmic vein.
Innervation
The gland is innervated by the secretomotor paraympathetic fibres from the pterygopalatine
ganglion which in turn may reach the gland via the zygomatic or lacrimal branches of the
maxillary nerve or pass directly to the gland. The preganglionic fibres travel to the ganglion
in the greater petrosal nerve (a branch of the facial nerve at the geniculate ganglion).
Nasolacrimal duct
Descends from the lacrimal sac to open anteriorly in the inferior meatus of the nose.
Lacrimation reflex
Occurs in response to conjunctival irritation (or emotional events). The conjunctiva will send
signals via the opthalmic nerve. These then pass to the superior salivary centre. The
efferent signals pass via the greater petrosal nerve (parasympathetic preganglionic fibres)
and the deep petrosal nerve which carries the post ganglionic sympathetic fibres. The
parasympathetic fibres will relay in the pterygopalatine ganglion, the sympathetic fibres do
not synapse. They in turn will relay to the lacrimal apparatus.
Rate question:

Next question

Comment on this question

All contents of this site are 2012 E-Medical Revision Ltd

http://www.emrcs.com/question/question.php?q=0

Terms and Conditions

Privacy policy

1/1

14/05/2015

Reference ranges

Previous

Question 20 of 538

Next

Which of the nerves listed below is directly responsible for the innervation of the lateral
aspect of flexor digitorum profundus?

A. Ulnar nerve
B. Anterior interosseous nerve

Question stats

Score: 94.4%
1

17.7%

29.4%

10.2%

34.4%

8.2%

29.4% of users answered this


question correctly

C. Radial nerve

End and review

D. Median nerve

6
7
8

E. Posterior interosseous nerve

Search
Go

Next question

10
11
12

The anterior interosseous nerve is a branch of the median nerve and is responsible for
innervation of the lateral aspect of the flexor digitorum profundus.

13
14

Forearm flexor muscles

15

Muscle

Flexor carpi
radialis

Origin

Insertion

16

Nerve
supply

Action

Flexes and abducts


the carpus, part
flexes the elbow and
part pronates forearm

17

Common
flexor origin
and
surrounding
fascia

Front of bases of
second and third
metacarpals

Median

Palmaris
longus

Common
flexor origin

Apex of palmar
aponeurosis

Median

Wrist flexor

Flexor carpi
ulnaris

Small humeral
head arises
from the
common flexor
origin and
adjacent
fascia. Ulnar
head comes
from medial
border of
olecranon and
posterior
border of ulna

Pisiform and base of


the fifth metacarpal

Ulnar nerve

Flexes and adducts


the carpus

Flexor
digitorum
superficialis

Long linear
origin from
common flexor
tendon,
adjacent
fascia and
septa and
medial border
of the coronoid
process

Via tendons in the


fibrous flexor sheath.
At the level of the
metacarpophalangeal
joint each tendon split
into two, these bands
pass distally to their
insertions

Median

Flexor of
metacarpophalangeal
joint and proximal
interphalangeal joint

Flexor
digitorum
profundus

Upper two
thirds of the
medial and
anterior
surface of the
ulna, medial
side of the
olecranon,
medial half of
the
interosseous
membrane

Via tendons that lie


deep to those of flexor
digitorum superficialis
to insert into the
distal phalanx

Medial part=
ulnar, lateral
part=anterior
interosseous
nerve

Flexes the distal


interphalangeal joints
and the wrist

Rate question:

18
19
20

Next question

Comment on this question

All contents of this site are 2012 E-Medical Revision Ltd

http://www.emrcs.com/question/question.php?q=0

Terms and Conditions

Privacy policy

1/1

14/05/2015

Reference ranges

Question 21 of 538

Previous

Next

A 45 year old lady is undergoing a Whipples procedure for carcinoma of the pancreatic
head. The bile duct is transected. Which of the following vessels is mainly responsible for the
blood supply to the bile duct?

Question stats

Score: 94.7%
1

34.9%

38.7%

8.1%

10.7%

7.7%

A. Cystic artery
38.7% of users answered this
question correctly

B. Hepatic artery

End and review

6
7

C. Portal vein
8

D. Left gastric artery


9

Search

E. None of the above

Go

Next question

10
11
12

Do not confuse the blood supply of the bile duct with that of the cystic duct.

13
14

Theme from April 2014 exam


The bile duct has an axial blood supply which is derived from the hepatic artery and from
retroduodenal branches of the gastroduodenal artery. Unlike the liver there is no contribution
by the portal vein to the blood supply of the bile duct. Damage to the hepatic artery during a
difficult cholecystectomy is a recognised cause of bile duct strictures.

15
16
17
18
19
20

Gallbladder

21

Fibromuscular sac with capacity of 50ml


Columnar epithelium

Relations of the gallbladder


Anterior

Liver

Posterior

Covered by peritoneum
Transverse colon
1st part of the duodenum

Laterally

Right lobe of liver

Medially

Quadrate lobe of liver

Arterial supply
Cystic artery (branch of Right hepatic artery)
Venous drainage
Directly to the liver
Nerve supply
Sympathetic- mid thoracic spinal cord, Parasympathetic- anterior vagal trunk
Common bile duct
Origin

Confluence of cystic and common hepatic ducts

Relations at
origin

Medially - Hepatic artery


Posteriorly- Portal vein

Relations distally

Arterial supply

Duodenum - anteriorly
Pancreas - medially and laterally
Right renal vein - posteriorly
Branches of hepatic artery and retroduodenal branches of gastroduodenal
artery

Hepatobiliary triangle
Medially

Common hepatic duct

http://www.emrcs.com/question/question.php?q=0

1/2

14/05/2015

Inferiorly

Cystic duct

Superiorly

Inferior edge of liver

Contents

Cystic artery

Calots triangle has the cystic artery as the superior border.


Rate question:

Next question

Comment on this question

All contents of this site are 2012 E-Medical Revision Ltd

http://www.emrcs.com/question/question.php?q=0

Terms and Conditions

Privacy policy

2/2

15/05/2015

Reference ranges

Question 1 of 517

Next

A 43 year old lady is undergoing a total thyroidectomy for an extremely large goitre. The
surgeons decide that access may be improved by division of the infra hyoid strap muscles. At
which of the following sites should they be divided?

Question stats

End and review

Score: 100%

35%

22.3%

17.8%

13.4%

11.6%

A. In their upper half


35% of users answered this
question correctly

B. In their lower half


C. In the middle
D. At their origin from the hyoid

Search

E. At the point of their insertion

Go

Next question

Theme from 2009 Exam


Should the strap muscles require division during surgery they should be divided in their
upper half. This is because their nerve supply from the ansa cervicalis enters in their lower
half.
Anterior triangle of the neck

Boundaries
Anterior border of the Sternocleidomastoid
Lower border of mandible
Anterior midline
Sub triangles (divided by Digastric above and Omohyoid)
Muscular triangle: Neck strap muscles
Carotid triangle: Carotid sheath
Submandibular Triangle (digastric)

Contents of the anterior triangle


Digastric triangle

Submandibular gland
Submandibular nodes
Facial vessels
Hypoglossal nerve

Muscular triangle

Strap muscles
External jugular vein

Carotid triangle

Carotid sheath (Common carotid, vagus and internal jugular vein)


Ansa cervicalis

Nerve supply to digastric muscle


Anterior: Mylohyoid nerve
Posterior: Facial nerve

Image sourced from Wikipedia

Rate question:

Next question

Comment on this question

All contents of this site are 2012 E-Medical Revision Ltd

http://www.emrcs.com/question/question.php?q=0

Terms and Conditions

Privacy policy

1/2

15/05/2015

http://www.emrcs.com/question/question.php?q=0

2/2

15/05/2015

Reference ranges

Previous

Question 2 of 517

Next

A 7 year old boy presents with right iliac fossa pain and there is a clinical suspicion that
appendicitis is present. From which of the following embryological structures is the appendix
derived?

Question stats

End and review

Score: 100%

18.7%

9%

8.8%

13.6%

49.8%

1
2

A. Vitello-intestinal duct
49.8% of users answered this
question correctly

B. Uranchus
C. Foregut
D. Hindgut

Search

E. Midgut

Go

Next question

The appendix is derived from the midgut

Theme from April 2014 exam


It is derived from the midgut which is why early appendicitis may present with periumbilical
pain.
Appendix

Location: Base of caecum.


Up to 10cm long.
Mainly lymphoid tissue (Hence mesenteric adenitis may mimic appendicitis).
Caecal taenia coli converge at base of appendix and form a longitudinal muscle cover
over the appendix. This convergence should facilitate its identification at surgery if it is
retrocaecal and difficult to find (which it can be when people start doing
appendicectomies!)
Arterial supply: Appendicular artery (branch of the ileocolic).
It is intra peritoneal.

McBurney's point
1/3 of the way along a line drawn from the Anterior Superior Iliac Spine to the
Umbilicus

6 Positions:

Retrocaecal 74%
Pelvic 21%
Postileal
Subcaecal
Paracaecal
Preileal

Rate question:

Next question

Comment on this question

All contents of this site are 2012 E-Medical Revision Ltd

http://www.emrcs.com/question/question.php?q=0

Terms and Conditions

Privacy policy

1/1

15/05/2015

Reference ranges

Previous

Question 3 of 517

Next

A 22 year old women has recently undergone a surgical excision of the submandibular gland.
She presents to the follow up clinic with a complaint of tongue weakness on the ipsilateral
side to her surgery. Which nerve has been damaged?

Question stats

End and review

Score: 100%
1

53.9%

20.2%

7.9%

10.3%

7.7%

A. Hypoglossal nerve
53.9% of users answered this
question correctly

B. Lingual nerve
C. Inferior alveolar nerve
D. Facial nerve

Search

E. Lesser petrosal nerve

Go

Next question

Three cranial nerves may be injured during submandibular gland excision.


Marginal mandibular branch of the facial nerve
Lingual nerve
Hypoglossal nerve

Theme from April 2012 Exam


Hypoglossal nerve damage may result in paralysis of the ipsilateral aspect of the tongue.
The nerve itself lies deep to the capsule surrounding the gland and should not be injured
during an intracapsular dissection. The lingual nerve is probably at greater risk of injury.
However, the effects of lingual nerve injury are sensory rather than motor.

Image sourced from Wikipedia

Submandibular gland
Relations of the submandibular gland
Superficial

Platysma, deep fascia and mandible


Submandibular lymph nodes
Facial vein (facial artery near mandible)
Marginal mandibular nerve
Cervical branch of the facial nerve

Deep

Facial artery (inferior to the mandible)


Mylohoid muscle
Sub mandibular duct
Hyoglossus muscle
Lingual nerve
Submandibular ganglion
Hypoglossal nerve

Submandibular duct (Wharton's duct)


Opens lateral to the lingual frenulum on the anterior floor of mouth.
5 cm length
Lingual nerve wraps around Wharton's duct. As the duct passes forwards it crosses
medial to the nerve to lie above it and then crosses back, lateral to it, to reach a
position below the nerve.

Innervation
http://www.emrcs.com/question/question.php?q=0

1/2

15/05/2015

Sympathetic innervation- Derived from superior cervical ganglion


Parasympathetic innervation- Submandibular ganglion via lingual nerve

Arterial supply
Branch of the facial artery. The facial artery passes through the gland to groove its deep
surface. It then emerges onto the face by passing between the gland and the mandible.
Venous drainage
Anterior facial vein (lies deep to the Marginal Mandibular nerve)
Lymphatic drainage
Deep cervical and jugular chains of nodes
Rate question:

Next question

Comment on this question

All contents of this site are 2012 E-Medical Revision Ltd

http://www.emrcs.com/question/question.php?q=0

Terms and Conditions

Privacy policy

2/2

15/05/2015

Reference ranges

Previous

Question 4 of 517

Next

You decide to take an arterial blood gas from the femoral artery. Where should the needle
be inserted to gain the sample?

A. Mid point of the inguinal ligament


B. Mid inguinal point

Question stats

End and review

Score: 100%
1

22.3%

45.9%

11.3%

7.9%

12.6%

45.9% of users answered this


question correctly

C. 2cm inferomedially to the pubic tubercle


D. 2cm superomedially to the pubic tubercle
E. 3cm inferolaterally to the deep inguinal ring

Search
Next question

Go

The mid inguinal point is midway between the anterior superior iliac spine and the
symphysis pubis

Theme from September 2012 Exam


Theme from April 2014 Exam
The mid inguinal point in the surface marking for the femoral artery.
Femoral triangle anatomy
Boundaries
Superiorly

Inguinal ligament

Laterally

Sartorius

Medially

Adductor longus

Floor

Iliopsoas, adductor longus and pectineus

Roof

Fascia lata and Superficial fascia


Superficial inguinal lymph nodes (palpable below the inguinal ligament)
Great saphenous vein

Image sourced from Wikipedia

Contents
Femoral vein (medial to lateral)
Femoral artery-pulse palpated at the mid inguinal point
Femoral nerve
Deep and superficial inguinal lymph nodes
Lateral cutaneous nerve
Great saphenous vein
Femoral branch of the genitofemoral nerve

http://www.emrcs.com/question/question.php?q=0

1/2

15/05/2015

Rate question:

Next question

Comment on this question

All contents of this site are 2012 E-Medical Revision Ltd

http://www.emrcs.com/question/question.php?q=0

Terms and Conditions

Privacy policy

2/2

15/05/2015

Reference ranges

Previous

Question 5 of 517

Next

A 67 year old man undergoes a carotid endarterectomy and seems to recover well following
surgery. When he is reviewed on the ward post operatively he complains that his voice is
hoarse. What is the most likely cause?

Question stats

End and review

Score: 100%
1

8.5%

9.2%

18.1%

17.6%

46.6%

A. Damage to the accessory nerve


46.6% of users answered this
question correctly

B. Damage to the cervical plexus


C. Damage to the glossopharyngeal nerve
D. Damage to the hypoglossal nerve

Search

E. Damage to the vagus

Go

Next question

Theme from April 2013 Exam


Many of these nerves are at risk of injury during carotid surgery. However, only damage to
the vagus would account for difficulties in speech.
Vagus nerve
The vagus nerve has mixed functions and supplies the structures from the fourth and sixth
pharyngeal arches. It also supplies the fore and midgut sections of the embryonic gut tube. It
carries afferent fibres from these areas (viz; pharynx, larynx, oesophagus, stomach, lungs,
heart and great vessels). The efferent fibres of the vagus are of two main types. The first are
preganglionic parasympathetic fibres distributed to the parasympathetic ganglia that
innervate smooth muscle of the innervated organs (such as gut). The second type of
efferent fibres have direct skeletal muscle innervation, these are largely to the muscles of the
larynx and pharynx.
Origin and course
The vagus arises from the lateral surface of the medulla oblongata by a series of rootlets. It
is related to the glossopharyngeal nerve cranially and the accessory nerve caudally. It exits
through the jugular foramen and is contained within its own dural sheath alongside the
accessory nerve. In the neck it descends vertically in the carotid sheath where it is closely
related to the internal and common carotid arteries. It leaves the neck and enters the
mediastinum. On the right it passes anterior to the first part of the subclavian artery, on the
left it lies in the interval between the common carotid and subclavian arteries.
In the mediastinum both nerves pass postero-inferiorly and reach the posterior surface of the
corresponding lung root. These then branch into both lungs. At the inferior end of the
mediastinum these plexuses reunite to form the formal vagal trunks that pass through the
oesophageal hiatus and into the abdomen. The anterior and posterior vagal trunks are
formal nerve fibres these then splay out once again sending fibres over the stomach and
posteriorly to the coeliac plexus. Branches pass to the liver, spleen and kidney.
Communications and branches
Communication

Details

Superior ganglion

Located in jugular foramen


Communicates with the superior cervical sympathetic ganglion, accessory
nerve
Two branches; meningeal and auricular (the latter may give rise to vagal
stimulation following instrumentation of the external auditory meatus)

Inferior ganglion

Communicates with the superior cervical sympathetic ganglion, hypoglossal


nerve and loop between first and second cervical ventral rami
Two branches; pharyngeal (supplies pharyngeal muscles) and superior
laryngeal nerve (inferomedially- deep to both carotid arteries)

Branches in the neck


Branch

Detail

Superior and
inferior cervical
cardiac
branches

Arise at various points and descend into thorax


On the right these pass posterior to the subclavian artery
On the left the superior branch passes between the arch of the aorta and the
trachea to connect with the deep cardiac plexus. The inferior branch descends
with the vagus itself.

Right recurrent
laryngeal nerve

Arises from vagus anterior to the first part of the subclavian artery, hooks
under it, and ascends superomedially. It passes close to the common carotid
and finally the inferior thyroid artery to insert into the larynx

http://www.emrcs.com/question/question.php?q=0

1/2

15/05/2015

Branches in the thorax


Branch

Details

Left
recurrent
laryngeal
nerve

Arises from the vagus on the aortic arch. It hooks around the inferior surface of the
arch, posterior to the ligamentum arteriosum and passes upwards through the
superior mediastinum and lower part of the neck. It lies in the groove between
oesophagus and trachea (supplies both). It passes with the inferior thyroid artery and
inserts into the larynx.

Thoracic
and
cardiac
branches

There are extensive branches to both the heart and lung roots. These pass
throughout both these viscera. The fibres reunite distally prior to passing into the
abdomen.

Abdominal branches
After entry into the abdominal cavity the nerves branch extensively. In previous years the
extensive network of the distal branches (nerves of Laterjet) over the surface of the distal
stomach were important for the operation of highly selective vagotomy. The use of modern
PPI's has reduced the need for such highly selective procedures. Branches pass to the
coeliac axis and alongside the vessels to supply the spleen, liver and kidney.

Rate question:

Next question

Comment on this question

All contents of this site are 2012 E-Medical Revision Ltd

http://www.emrcs.com/question/question.php?q=0

Terms and Conditions

Privacy policy

2/2

15/05/2015

Reference ranges

Question 6 of 517

Previous

Next

A 25 year old man has an inguinal hernia, which of the following structures must be divided
(at open surgery) to gain access to the inguinal canal?

A. Transversalis fascia
B. External oblique aponeurosis

Question stats

Score: 100%
1

13.7%

63.1%

8.4%

7.5%

7.3%

63.1% of users answered this


question correctly

C. Conjoint tendon

End and review

D. Rectus abdominis
E. Inferior epigastric artery

Search
Go

Next question

Theme from January 2013 Exam


This question is asking what structure forms the anterior wall of the inguinal canal. The
anterior wall is formed by the external oblique aponeurosis. Once this is divided the canal is
entered, the cord can be mobilised and a hernia repair performed. The transversalis fascia
and conjoint tendons form the posterior wall and would not routinely be divided to gain
access to the inguinal canal itself.
External oblique muscle
External oblique forms the outermost muscle of the three muscles comprising the
anterolateral aspect of the abdominal wall. Its aponeurosis comprises the anterior wall of the
inguinal canal.
Origin

Outer surfaces of the lowest eight ribs

Insertion

Anterior two thirds of the outer lip of the iliac crest.


The remainder becomes the aponeurosis that fuses with the linea alba in the
midline.

Nerve
supply

Ventral rami of the lower six thoracic nerves

Actions

Contains the abdominal viscera, may contract to raise intra abdominal pressure.
Moves trunk to one side.

Rate question:

Next question

Comment on this question

All contents of this site are 2012 E-Medical Revision Ltd

http://www.emrcs.com/question/question.php?q=0

Terms and Conditions

Privacy policy

1/1

15/05/2015

Reference ranges

Previous

Question 7 of 517

Next

Question stats

End and review

Score: 100%
1

8.7%

8.1%

55.6%

A. Infraspinatus

19.3%

B. Latissimus dorsi

8.4%

C. Supraspinatus

55.6% of users answered this


question correctly

Which muscle initiates abduction of the shoulder?

D. Deltoid

6
7

E. Teres major
Search
Next question
Go

Theme from September 2012 exam


Theme from April 2014 Exam
The intermediate portion of the deltoid muscle is the chief abductor of the humerus.
However, it can only do this after the movement has been initiated by supraspinatus.
Damage to the tendon of supraspinatus is a common form of rotator cuff disease.
Shoulder joint

Shallow synovial ball and socket type of joint.


It is an inherently unstable joint, but is capable to a wide range of movement.
Stability is provided by muscles of the rotator cuff that pass from the scapula to insert
in the greater tuberosity (all except sub scapularis-lesser tuberosity).

Glenoid labrum
Fibrocartilaginous rim attached to the free edge of the glenoid cavity
Tendon of the long head of biceps arises from within the joint from the supraglenoid
tubercle, and is fused at this point to the labrum.
The long head of triceps attaches to the infraglenoid tubercle

Fibrous capsule
Attaches to the scapula external to the glenoid labrum and to the labrum itself
(postero-superiorly)
Attaches to the humerus at the level of the anatomical neck superiorly and the surgical
neck inferiorly
Anteriorly the capsule is in contact with the tendon of subscapularis, superiorly with the
supraspinatus tendon, and posteriorly with the tendons of infraspinatus and teres
minor. All these blend with the capsule towards their insertion.
Two defects in the fibrous capsule; superiorly for the tendon of biceps. Anteriorly there
is a defect beneath the subscapularis tendon.
The inferior extension of the capsule is closely related to the axillary nerve at the
surgical neck and this nerve is at risk in anteroinferior dislocations. It also means that
proximally sited osteomyelitis may progress to septic arthritis.

Movements and muscles


Flexion

Anterior part of deltoid


Pectoralis major
Biceps
Coracobrachialis

Extension

Posterior deltoid
Teres major
Latissimus dorsi

Adduction

Pectoralis major
Latissimus dorsi
Teres major
Coracobrachialis

Abduction

Mid deltoid
Supraspinatus

Medial rotation

Subscapularis
Anterior deltoid
Teres major
Latissimus dorsi

Lateral rotation

Posterior deltoid

http://www.emrcs.com/question/question.php?q=0

1/2

15/05/2015

Infraspinatus
Teres minor
Important anatomical relations
Anteriorly

Brachial plexus
Axillary artery and vein

Posterior

Suprascapular nerve
Suprascapular vessels

Inferior

Axillary nerve
Circumflex humeral vessels

Rate question:

Next question

Comment on this question

All contents of this site are 2012 E-Medical Revision Ltd

http://www.emrcs.com/question/question.php?q=0

Terms and Conditions

Privacy policy

2/2

15/05/2015

Reference ranges

Question 8 of 517

Previous

Next

A 34 year old man is shot in the postero- inferior aspect of his thigh. Which of the following
lies at the most lateral aspect of the popliteal fossa?

A. Popliteal artery
B. Popliteal vein

Question stats

Score: 90%
1

9.4%

10.8%

55.1%

13.3%

11.4%

55.1% of users answered this


question correctly

C. Common peroneal nerve

End and review

D. Tibial nerve

6
7
8

E. Small saphenous vein

Search
Go

Next question

Theme from April 2012 exam


Theme from April 2014 exam
The contents of the popliteal fossa are (from medial to lateral):
Popliteal artery
Popliteal vein
Tibial nerve
Common peroneal nerve
The sural nerve is a branch of the tibial nerve and usually arises at the inferior aspect of the
popliteal fossa. However, its anatomy is variable.
Popliteal fossa
Boundaries of the popliteal fossa
Laterally

Biceps femoris above, lateral head of gastrocnemius and plantaris below

Medially

Semimembranosus and semitendinosus above, medial head of gastrocnemius


below

Floor

Popliteal surface of the femur, posterior ligament of knee joint and popliteus muscle

Roof

Superficial and deep fascia

Image showing the popliteal fossa

Image sourced from Wikipedia

Contents
Popliteal artery and vein
Small saphenous vein
Common peroneal nerve
Tibial nerve
Posterior cutaneous nerve of the thigh
Genicular branch of the obturator nerve
Lymph nodes

Rate question:

Next question

Comment on this question

All contents of this site are 2012 E-Medical Revision Ltd

http://www.emrcs.com/question/question.php?q=0

Terms and Conditions

Privacy policy

1/1

15/05/2015

Reference ranges

Previous

Question 9 of 517

Next

A 67 year old man has an abdominal aortic aneurysm which displaces the left renal vein.
Which branch of the aorta is most likely to affected at this level?

A. Inferior mesenteric artery


B. Superior mesenteric artery

Question stats

Score: 81.8%
1

17.9%

39.2%

11.3%

23.7%

7.9%

39.2% of users answered this


question correctly

C. Coeliac axis

End and review

D. Testicular artery

6
7
8

E. None of the above

Search
Go

Next question

Theme from April 2013 exam


Theme from April 2014 exam
The left renal vein lies behind of the SMA as it branches off the aorta. Whilst juxtarenal AAA
may sometimes require the division of the left renal vein, direct involvement of the SMA may
require a hybrid surgical bypass and subsequent endovascular occlusion.
Abdominal aortic branches

Branches

Level

Paired

Type

Inferior phrenic

T12 (Upper border)

Yes

Parietal

Coeliac

T12

No

Visceral

Superior mesenteric

L1

No

Visceral

Middle suprarenal

L1

Yes

Visceral

Renal

L1-L2

Yes

Visceral

Gonadal

L2

Yes

Visceral

Lumbar

L1-L4

Yes

Parietal

Inferior mesenteric

L3

No

Visceral

Median sacral

L4

No

Parietal

Common iliac

L4

Yes

Terminal

Rate question:

Next question

Comment on this question

All contents of this site are 2012 E-Medical Revision Ltd

http://www.emrcs.com/question/question.php?q=0

Terms and Conditions

Privacy policy

1/1

15/05/2015

Reference ranges

Previous

Question 10 of 517

Next

A 12 year old boy undergoes surgery for recurrent mastoid infections. Post operatively he
complains of an altered taste sensation. Which of the following nerves has been injured?

A. Glossopharyngeal
B. Greater petrosal

Question stats

Score: 83.3%
1

21%

9.4%

8.9%

11.8%

48.8%

48.8% of users answered this


question correctly

C. Olfactory

End and review

D. Trigeminal

6
7
8

E. Chorda tympani

Search
Next question

Go

10

Theme from April 2012 exam


Theme from April 2014 exam
The chorda tympani branch of the facial nerve passes forwards through itrs canaliculus into
the middle ear, and crosses the medial aspect of the tympanic membrane. It then passes
antero-inferiorly in the infratemporal fossa. It distributes taste fibres to the anterior two thirds
of the tongue.

Image sourced from Wikipedia

Facial nerve
The facial nerve is the main nerve supplying the structures of the second embryonic
branchial arch. It is predominantly an efferent nerve to the muscles of facial expression,
digastric muscle and also to many glandular structures. It contains a few afferent fibres which
originate in the cells of its genicular ganglion and are concerned with taste.
Supply - 'face, ear, taste, tear'
Face: muscles of facial expression
Ear: nerve to stapedius
Taste: supplies anterior two-thirds of tongue
Tear: parasympathetic fibres to lacrimal glands, also salivary glands

Path
Subarachnoid path
Origin: motor- pons, sensory- nervus intermedius
Pass through the petrous temporal bone into the internal auditory meatus with the
vestibulocochlear nerve. Here they combine to become the facial nerve.

Facial canal path


The canal passes superior to the vestibule of the inner ear
At the medial aspect of the middle ear, it becomes wider and contains the geniculate
ganglion.
- 3 branches:
1. greater petrosal nerve
2. nerve to stapedius
3. chorda tympani

http://www.emrcs.com/question/question.php?q=0

1/2

15/05/2015

Stylomastoid foramen
Passes through the stylomastoid foramen (tympanic cavity anterior and mastoid
antrum posteriorly)
Posterior auricular nerve and branch to posterior belly of digastric and stylohyoid
muscle

Face
Enters parotid gland and divides into 5 branches:
Temporal branch
Zygomatic branch
Buccal branch
Marginal mandibular branch
Cervical branch

Rate question:

Next question

Comment on this question

All contents of this site are 2012 E-Medical Revision Ltd

http://www.emrcs.com/question/question.php?q=0

Terms and Conditions

Privacy policy

2/2

15/05/2015

Reference ranges

Question 11 of 517

Previous

Next

Question stats

End and review

Score: 84.6%
1

9.4%

61.4%

12.2%

A. C6

9%

B. C5

8.1%

C. C3

61.4% of users answered this


question correctly

The first root of the brachial plexus commonly arises at which of the following levels?

D. C2
E. C8

6
7
8
9

Search
Next question
Go

It begins at C5 and has 5 roots. It ends with a total of 15 nerves of these 5 are the main
nerves to the upper limb (axillary, radial, ulnar, musculocutaneous and median)

10
11

Brachial plexus

Origin

Anterior rami of C5 to T1

Sections of the
plexus

Roots

Roots, trunks, divisions, cords, branches


Mnemonic:Real Teenagers Drink Cold Beer

Located in the posterior triangle


Pass between scalenus anterior and medius

Trunks

Located posterior to middle third of clavicle


Upper and middle trunks related superiorly to the subclavian
artery
Lower trunk passes over 1st rib posterior to the subclavian
artery

Divisions

Apex of axilla

Cords

Related to axillary artery

Diagram illustrating the branches of the brachial plexus

Image sourced from Wikipedia

Cutaneous sensation of the upper limb

Image sourced from Wikipedia

Rate question:

Next question

Comment on this question

http://www.emrcs.com/question/question.php?q=0

1/2

15/05/2015

All contents of this site are 2012 E-Medical Revision Ltd

http://www.emrcs.com/question/question.php?q=0

Terms and Conditions

Privacy policy

2/2

15/05/2015

Reference ranges

Previous

Question 12 of 517

Next

Question stats

End and review

Score: 85.7%
1

8.3%

15.9%

48.7%

A. T11

8.5%

B. T12

18.6%

C. L1

48.7% of users answered this


question correctly

What is the anatomical level of the transpyloric plane?

D. L4
E. T10

6
7
8
9

Search
Next question
Go

10
11

Theme from September 2012 Exam


Theme from January 2013 Exam
Theme from September 2013 Exam

12

Transpyloric plane
Transpyloric plane
Level of the body of L1

Pylorus stomach
Left kidney hilum (L1- left one!)
Right hilum of the kidney (1.5cm lower than the left)
Fundus of the gallbladder
Neck of pancreas
Duodenojejunal flexure
Superior mesenteric artery
Portal vein
Left and right colic flexure
Root of the transverse mesocolon
2nd part of the duodenum
Upper part of conus medullaris
Spleen

Rate question:

Next question

Comment on this question

All contents of this site are 2012 E-Medical Revision Ltd

http://www.emrcs.com/question/question.php?q=0

Terms and Conditions

Privacy policy

1/1

15/05/2015

Reference ranges

Previous

Question 13 of 517

Next

A 35 year old man falls and sustains a fracture to the middle third of his clavicle. Which
vessel is at greatest risk of injury?

A. Subclavian vein
B. Subclavian artery

Question stats

Score: 86.7%
1

44.7%

34.2%

7%

7.3%

6.8%

44.7% of users answered this


question correctly

C. External carotid artery

End and review

D. Internal carotid artery

6
7
8

E. Vertebral artery

Search
Go

Next question

10
11
12

Theme from April 2013 exam


Theme from April 2014 exam
The subclavian vein lies behind subclavius and the medial part of the clavicle. It rests on the
first rib, below and in front of the third part of the subclavian artery, and then on scalenus
anterior which separates it from the second part of the artery (posteriorly).

13

Clavicle
The clavicle extends from the sternum to the acromion and helps prevent the shoulder falling
forwards and downwards.
The inferior surface is irregular and strongly marked by ligaments at each end. Laterally, lies
the trapezoid line and this runs anterolaterally. Posteriorly, lies the conoid tubercle. These
give attachment to the conoid and trapezoid parts of the coracoclavicular ligament. The
medial part of the inferior surface has an irregular surface which marks the surface
attachment of the costoclavicular ligament. The intermediate portion is marked by a groove
for the subclavius muscle.
Medially, the superior part of the bone has a raised surface which gives attachment to the
clavicular head of sternocleidomastoid. Sternohyoid gains attachment to the posterior
surface.
Laterally there is an oval articular facet for the acromion and a disk lies between the clavicle
and acromion. The capsule of the joint is attached to the ridge on the margin of the facet.

Rate question:

Next question

Comment on this question

All contents of this site are 2012 E-Medical Revision Ltd

http://www.emrcs.com/question/question.php?q=0

Terms and Conditions

Privacy policy

1/1

15/05/2015

Reference ranges

Previous

Question 14 of 517

Next

Question stats

End and review

Score: 87.5%
1

15.7%

34.2%

34.3%

A. Right ventricle

6.9%

B. Left ventricle

8.9%

C. Right atrium

34.3% of users answered this


question correctly

Where are the greatest proportion of musculi pectinati found?

D. Pulmonary valve
E. Aortic valve

6
7
8
9

Search
Next question
Go

The musculi pectinati are found in the atria, hence the reason that the atrial walls in the right
atrium are irregular anteriorly.
The musculi pectinati of the atria are internal muscular ridges on the anterolateral surface of
the chambers and they are only present in the area derived from the embryological true
atrium.

10
11
12
13
14

Heart anatomy
The walls of each cardiac chamber comprise:
Epicardium
Myocardium
Endocardium

Cardiac muscle is attached to the cardiac fibrous skeleton.


Relations
The heart and roots of the great vessels within the pericardial sac are related anteriorly to
the sternum, medial ends of the 3rd to 5th ribs on the left and their associated costal
cartilages. The heart and pericardial sac are situated obliquely two thirds to the left and one
third to the right of the median plane.
The pulmonary valve lies at the level of the left third costal cartilage.
The mitral valve lies at the level of the fourth costal cartilage.
Coronary sinus
This lies in the posterior part of the coronary groove and receives blood from the cardiac
veins. The great cardiac vein lies at its left and the middle and small cardiac veins lie on its
right. The smallest cardiac vein (anterior cardiac vein) drains into the right atrium directly.
Aortic sinus
Right coronary artery arises from the right aortic sinus, the left is derived from the left aortic
sinus and no vessel emerges from the posterior sinus.
Right and left ventricles
Structure

Left Ventricle

A-V Valve

Mitral (double leaflet)

Walls

Twice as thick as right

Trabeculae carnae

Much thicker and more numerous

Right coronary artery


The RCA supplies:
Right atrium
Diaphragmatic part of the left ventricle
Usually the posterior third of the interventricular septum
The sino atrial node (60% cases)
The atrio ventricular node (80% cases)

Left coronary artery


The LCA supplies:
Left atrium
Most of left ventricle
Part of the right ventricle
Anterior two thirds of the inter ventricular septum
The sino atrial node (remaining 40% cases)

Innervation of the heart


Autonomic nerve fibres from the superficial and deep cardiac plexus. These lie anterior to
the bifurcation of the trachea, posterior to the ascending aorta and superior to the
bifurcation of the pulmonary trunk. The parasympathetic supply to the heart is from
presynaptic fibres of the vagus nerves.
Valves of the heart
Mitral valve

http://www.emrcs.com/question/question.php?q=0

Aortic valve

Pulmonary

Tricuspid valve

1/2

15/05/2015

valve
2 cusps

3 cusps

3 cusps

3 cusps

First heart sound

Second heart
sound

Second heart
sound

First heart sound

1 anterior cusp

2 anterior cusps

2 anterior cusps

2 anterior cusps

Attached to chordae
tendinae

No chordae

No chordae

Attached to chordae
tendinae

Rate question:

Next question

Comment on this question

All contents of this site are 2012 E-Medical Revision Ltd

http://www.emrcs.com/question/question.php?q=0

Terms and Conditions

Privacy policy

2/2

15/05/2015

Reference ranges

Question 15 of 517

Previous

Next

Question stats

End and review

Score: 88.2%
1

9.2%

9%

56.2%

A. Digastric muscle

16.5%

B. Prevertebral fascia

9%

C. Anterior scalene muscle

56.2% of users answered this


question correctly

Which of the following structures separates the subclavian artery and vein?

D. Middle scalene muscle


E. Omohyoid

7
8
9

Search
Next question
Go

10
11

The anterior scalene muscle is an important anatomical landmark and separates the
subclavian vein (anterior) from the subclavian artery (posterior).

12
13

Scalene muscles

14
15

The 3 paired muscles are:


Scalenus anterior: Elevate 1st rib and laterally flex the neck to same side
Scalenus medius: Same action as scalenus anterior
Scalenus posterior: Elevate 2nd rib and tilt neck to opposite side

Innervation Spinal nerves C4-6


Origin

Transverse processes C2 to C7

Insertion

First and second ribs

Important
relations

The brachial plexus and subclavian artery pass betw een the anterior
and middle scalenes through a space called the scalene
hiatus/fissure.
The subclavian vein and phrenic nerve pass anteriorly to the anterior
scalene as it crosses over the first rib.

Image sourced from Wikipedia

Thoracic outlet syndrome


The scalenes are at risk of adhering to the fascia surrounding the brachial plexus or
shortening causing compression of the brachial plexus when it passes between the clavicle
and 1st rib causing thoracic outlet syndrome.
Rate question:

Next question

Comment on this question

All contents of this site are 2012 E-Medical Revision Ltd

http://www.emrcs.com/question/question.php?q=0

Terms and Conditions

Privacy policy

1/1

15/05/2015

Reference ranges

Question 16 of 517

Previous

Next

A 33 year old man is stabbed in the right chest and undergoes a thoracotomy. The right lung
is mobilised and the pleural reflection at the lung hilum is opened. Which of the structures
listed below does not lie within this region?

Question stats

Score: 83.3%
1

9.5%

52.4%

9.6%

12.3%

16.4%

A. Pulmonary artery
52.4% of users answered this
question correctly

B. Azygos vein

End and review

6
7

C. Pulmonary vein
8

D. Bronchus
9

Search

E. None of the above

Go

Next question

10
11
12

The pleural reflections encase the hilum of the lung and continue inferiorly as the pulmonary
ligament. It encases the pulmonary vessels and bronchus. The azygos vein is not contained
within it.

13
14
15

Lung anatomy

16

The right lung is composed of 3 lobes divided by the oblique and transverse fissures. The
left lung has two lobes divided by the oblique fissure.The apex of both lungs is approximately
4cm superior to the sterno-costal joint of the first rib. Immediately below this is a sulcus
created by the subclavian artery.
Peripheral contact points of the lung
Base: diaphragm
Costal surface: corresponds to the cavity of the chest
Mediastinal surface: Contacts the mediastinal pleura. Has the cardiac impression.
Above and behind this concavity is a triangular depression named the hilum, where
the structures which form the root of the lung enter and leave the viscus. These
structures are invested by pleura, which, below the hilum and behind the pericardial
impression, forms the pulmonary ligament

Right lung
Above the hilum is the azygos vein; Superior to this is the groove for the superior vena cava
and right innominate vein; behind this, and nearer the apex, is a furrow for the innominate
artery. Behind the hilum and the attachment of the pulmonary ligament is a vertical groove
for the oesophagus; In front and to the right of the lower part of the oesophageal groove is a
deep concavity for the extrapericardiac portion of the inferior vena cava.
The root of the right lung lies behind the superior vena cava and the right atrium, and below
the azygos vein.
The right main bronchus is shorter, wider and more vertical than the left main bronchus and
therefore the route taken by most foreign bodies.

Image sourced from Wikipedia

Left lung
Above the hilum is the furrow produced by the aortic arch, and then superiorly the groove
accommodating the left subclavian artery; Behind the hilum and pulmonary ligament is a
vertical groove produced by the descending aorta, and in front of this, near the base of the
lung, is the lower part of the oesophagus.
The root of the left lung passes under the aortic arch and in front of the descending aorta.

Image sourced from Wikipedia

Inferior borders of both lungs


6th rib in mid clavicular line
http://www.emrcs.com/question/question.php?q=0

1/2

15/05/2015

6th rib in mid clavicular line


8th rib in mid axillary line
10th rib posteriorly
The pleura runs two ribs lower than the corresponding lung level.
Bronchopulmonary segments
Segment number

Right lung

Left lung

Apical

Apical

Posterior

Posterior

Anterior

Anterior

Lateral

Superior lingular

Medial

Inferior lingular

Superior (apical)

Superior (apical)

Medial basal

Medial basal

Anterior basal

Anterior basal

Lateral basal

Lateral basal

10

Posterior basal

Posterior basal

Rate question:

Next question

Comment on this question

All contents of this site are 2012 E-Medical Revision Ltd

http://www.emrcs.com/question/question.php?q=0

Terms and Conditions

Privacy policy

2/2

15/05/2015

Reference ranges

Previous

Question 17 of 517

Next

A 56 year old man requires long term parenteral nutrition and the decision is made to insert
a PICC line for long term venous access. This is inserted into the basilic vein at the region of
the elbow. As the catheter is advanced, into which venous structure is the tip of the catheter
most likely to pass from the basilic vein?

Question stats

Score: 84.2%
1

16.2%

53.3%

8.3%

13.9%

8.4%

53.3% of users answered this


question correctly

A. Subclavian vein

End and review

B. Axillary vein

6
7
8

C. Posterior circumflex humeral vein

Search

D. Cephalic vein
E. Superior vena cava

Go

10
11

Next question

12
13

The basilic vein drains into the axillary vein and although PICC lines may end up in a variety
of fascinating locations the axillary vein is usually the commonest site following from the
basilic. The posterior circumflex humeral vein is encountered prior to the axillary vein.
However, a PICC line is unlikely to enter this structure because of its angle of entry into the
basilic vein.

14
15
16
17

Basilic vein
The basilic and cephalic veins both provide the main pathways of venous drainage for the
arm and hand. It is continuous with the palmar venous arch distally and the axillary vein
proximally.
Path
Originates on the medial side of the dorsal venous network of the hand, and passes
up the forearm and arm.
Most of its course is superficial.
Near the region anterior to the cubital fossa the vein joins the cephalic vein.
Midway up the humerus the basilic vein passes deep under the muscles.
At the lower border of the teres major muscle, the anterior and posterior circumflex
humeral veins feed into it.
It is often joined by the medial brachial vein before draining into the axillary vein.

Rate question:

Next question

Comment on this question

All contents of this site are 2012 E-Medical Revision Ltd

http://www.emrcs.com/question/question.php?q=0

Terms and Conditions

Privacy policy

1/1

15/05/2015

Reference ranges

Previous

Question 18 of 517

Next

An individual is noted to have a left sided superior vena cava. By which pathway is blood
from this system most likely to enter the heart?

A. Via the coronary sinus


B. Via the azygos venous system and into the superior vena cava

Question stats

Score: 80%
1

28.6%

22.3%

7.9%

18.6%

22.5%

28.6% of users answered this


question correctly

C. Via anomalies in the pumonary vascular bed

End and review

D. Via the left atrium and persistent foramen ovale

6
7
8

E. Directly into the roof of the right atrium

Search
Go

Next question

10
11
12

Theme from September 2013 Exam


Persistent left superior vena cava is the most common anomaly of the thoracic venous
system. It is prevalent in 0.3% of the population and is a benign entity of failed involution
during embryogenesis.

13
14
15

Superior vena cava

16
17

Drainage

18

Head and neck


Upper limbs
Thorax
Part of abdominal walls

Formation
Subclavian and internal jugular veins unite to form the right and left brachiocephalic
veins
These unite to form the SVC
Azygos vein joins the SVC before it enters the right atrium

Relations
Anterior

Anterior margins of the right lung and pleura

Posteromedial

Trachea and right vagus nerve

Posterolateral

Posterior aspects of right lung and pleura


Pulmonary hilum is posterior

Right lateral

Right phrenic nerve and pleura

Left lateral

Brachiocephalic artery and ascending aorta

Developmental variations
Anomalies of the connection of the SVC are recognised. In some individuals a persistent left
sided SVC drains into the right atrium via an enlarged orifice of the coronary sinus. More
rarely the left sided vena cava may connect directly with the superior aspect of the left
atrium, usually associated with an un-roofing of the coronary sinus. The commonest lesion of
the IVC is for its abdominal course to be interrupted, with drainage achieved via the azygos
venous system. This may occur in patients with left sided atrial isomerism.
Rate question:

Next question

Comment on this question

All contents of this site are 2012 E-Medical Revision Ltd

http://www.emrcs.com/question/question.php?q=0

Terms and Conditions

Privacy policy

1/1

15/05/2015

Reference ranges

Previous3 / 3

Question 19-21 of 517

Next

Question stats

End and review

Score: 84.6%

Average score for registered users:

1
2

Theme: Nerve injury


A. Ulnar nerve

19

81.6%

20

69.8%

21

76.4%

B. Musculocutaneous nerve

3
4
5

C. Radial nerve

D. Median nerve

Search

E. Axillary nerve

7
Go

F. Intercostobrachial nerve

8
9

What is the most likely nerve injury for the scenario given? Each option may be used once,
more than once or not at all.

10
11
12
13

19.

A 23 year old man is involved in a fight outside a nightclub and sustains a


laceration to his right arm. On examination he has lost extension of the fingers
in his right hand.

14
15
16

Radial nerve
17

The radial nerve supplies the extensor muscle group.


20.

18

A 40 year old lady trips and falls through a glass door and sustains a severe
laceration to her left arm. Amongst her injuries it is noticed that she has lost the
ability to adduct the fingers of her left hand.

19-21 3 / 3

Ulnar nerve
The interossei are supplied by the ulnar nerve.
21.

A 28 year old rugby player injures his right humerus and on examination is
noted to have a minor sensory deficit overlying the point of deltoid insertion
into the humerus.
Axillary nerve
This patch of skin is supplied by the axillary nerve

Next question

Theme from April 2014 exam

Brachial plexus

Origin
Sections of the
plexus

Roots

Trunks

Anterior rami of C5 to T1
Roots, trunks, divisions, cords, branches
Mnemonic:Real Teenagers Drink Cold Beer

Located in the posterior triangle


Pass between scalenus anterior and medius

Located posterior to middle third of clavicle


Upper and middle trunks related superiorly to the subclavian
artery
Lower trunk passes over 1st rib posterior to the subclavian
artery

Divisions

Apex of axilla

Cords

Related to axillary artery

Diagram illustrating the branches of the brachial plexus

http://www.emrcs.com/question/question.php?q=0

1/2

15/05/2015

Image sourced from Wikipedia

Cutaneous sensation of the upper limb

Image sourced from Wikipedia

Rate question:

Next question

Comment on this question

All contents of this site are 2012 E-Medical Revision Ltd

http://www.emrcs.com/question/question.php?q=0

Terms and Conditions

Privacy policy

2/2

15/05/2015

Reference ranges

Question 22 of 517

Previous

Next

A 53 year old man is undergoing a radical gastrectomy for carcinoma of the stomach. Which
of the following structures will need to be divided to gain access to the coeliac axis?

A. Lesser omentum
B. Greater omentum

Question stats

Score: 81.5%
1

45.8%

18.9%

9.9%

10.7%

14.6%

45.8% of users answered this


question correctly

C. Falciform ligament

End and review

D. Median arcuate ligament

6
7
8

E. Gastrosplenic ligament

Search
Go

Next question

10
11
12

The lesser omentum will need to be divided. During a radical gastrectomy this forms one of
the nodal stations that will need to be taken.

13
14

Coeliac axis

15
16

The coeliac axis has three main branches.

17

Left gastric
Hepatic: branches-Right Gastric, Gastroduodenal, Right Gastroepiploic, Superior
Pancreaticoduodenal, Cystic (occasionally).
Splenic: branches- Pancreatic, Short Gastric, Left Gastroepiploic

18
19-21 3 / 3
22

It occasionally gives off one of the inferior phrenic arteries.

Image sourced from Wikipedia

Relations
Anteriorly

Lesser omentum

Right

Right coeliac ganglion and caudate process of liver

Left

Left coeliac ganglion and gastric cardia

Inferiorly

Upper border of pancreas and renal vein

Rate question:

Next question

Comment on this question

All contents of this site are 2012 E-Medical Revision Ltd

http://www.emrcs.com/question/question.php?q=0

Terms and Conditions

Privacy policy

1/1

15/05/2015

Reference ranges

Previous

Question 23 of 517

Next

A 76 year old man complains of symptoms of claudication. The decision is made to measure
his ankle brachial pressure index. The signal from the dorsalis pedis artery is auscultated
with a hand held doppler device. This vessel is the continuation of which of the following?

Question stats

Score: 82.1%
1

14.2%

64.3%

8.6%

7.1%

5.8%

A. Posterior tibial artery


64.3% of users answered this
question correctly

B. Anterior tibial artery

End and review

6
7

C. Peroneal artery
8

D. Popliteal artery
9

Search

E. None of the above

Go

Next question

10
11
12

The dorsalis pedis is a continuation of the anterior tibial artery.

13
14

Foot- anatomy

15

Arches of the foot


The foot is conventionally considered to have two arches.
The longitudinal arch is higher on the medial than on the lateral side. The posterior
part of the calcaneum forms a posterior pillar to support the arch. The lateral part of
this structure passes via the cuboid bone and the lateral two metatarsal bones. The
medial part of this structure is more important. The head of the talus marks the summit
of this arch, located between the sustentaculum tali and the navicular bone. The
anterior pillar of the medial arch is composed of the navicular bone, the three
cuneiforms and the medial three metatarsal bones.
The transverse arch is situated on the anterior part of the tarsus and the posterior
part of the metatarsus. The cuneiforms and metatarsal bases narrow inferiorly, which
contributes to the shape of the arch.

16
17
18
19-21 3 / 3
22
23

Intertarsal joints
Sub talar joint

Formed by the cylindrical facet on the lower surface of the body of


the talus and the posterior facet on the upper surface of the
calcaneus. The facet on the talus is concave anteroposteriorly, the
other is convex. The synovial cavity of this joint does not
communicate with any other joint.

Talocalcaneonavicular
joint

The anterior part of the socket is formed by the concave articular


surface of the navicular bone, posteriorly by the upper surface of the
sustentaculum tali. The talus sits within this socket

Calcaneocuboid joint

Highest point in the lateral part of the longitudinal arch. The lower
aspect of this joint is reinforced by the long plantar and plantar
calcaneocuboid ligaments.

Transverse tarsal joint

The talocalcaneonavicular joint and the calcaneocuboid joint extend


across the tarsus in an irregular transverse plane, between the talus
and calcaneus behind and the navicular and cuboid bones in front.
This plane is termed the transverse tarsal joint.

Cuneonavicular joint

Formed between the convex anterior surface of the navicular bone


and the concave surface of the the posterior ends of the three
cuneiforms.

Intercuneiform joints

Between the three cuneiform bones.

Cuneocuboid joint

Between the circular facets on the lateral cuneiform bone and the
cuboid. This joint contributes to the tarsal part of the transverse arch.

A detailed knowledge of the joints is not required for MRCS Part A. However, the contribution
they play to the overall structure of the foot should be appreciated
Ligaments of the ankle joint and foot

http://www.emrcs.com/question/question.php?q=0

1/3

15/05/2015

Image sourced from Wikipedia

Muscles of the foot


Muscle

Origin

Insertion

Nerve
supply

Action

Abductor
hallucis

Medial side of the calcaneus,


flexor retinaculum, plantar
aponeurosis

Medial side of
the base of
the proximal
phalanx

Medial
plantar
nerve

Abducts the great


toe

Flexor
digitorum
brevis

Medial process of the


calcaneus, plantar
eponeurosis.

Via 4 tendons
into the
middle
phalanges of
the lateral 4
toes.

Medial
plantar
nerve

Flexes all the joints


of the lateral 4 toes
except for the
interphalangeal joint.

Abductor
digit
minimi

From the tubercle of the


calcaneus and from the
plantar aponeurosis

Together with
flexor digit
minimi brevis
into the lateral
side of the
base of the
proximal
phalanx of the
little toe

Lateral
plantar
nerve

Abducts the little toe


at the
metatarsophalangeal
joint

Flexor
hallucis
brevis

From the medial side of the


plantar surface of the cuboid
bone, from the adjacent part
of the lateral cuneiform bone
and from the tendon of tibialis
posterior.

Into the
proximal
phalanx of the
great toe, the
tendon
contains a
sesamoid
bone

Medial
plantar
nerve

Flexes the
metatarsophalangeal
joint of the great toe.

Adductor
hallucis

Arises from two heads. The


oblique head arises from the
sheath of the peroneus longus
tendon, and from the plantar
surfaces of the bases of the
2nd, 3rd and 4th metatarsal
bones. The transverse head
arises from the plantar
surface of the lateral 4
metatarsophalangeal joints
and from the deep transverse
metatarsal ligament.

Lateral side of
the base of
the proximal
phalanx of the
great toe.

Lateral
plantar
nerve

Adducts the great


toe towards the
second toe. Helps
maintain the
transverse arch of
the foot.

Extensor
digitorum
brevis

On the dorsal surface of the


foot from the upper surface of
the calcaneus and its
associated fascia

Via four thin


tendons which
run forward
and medially
to be inserted
into the
medial four
toes. The
lateral three
tendons join
with hoods of
extensor
digitorum
longus.

Deep
peroneal

Extend the
metatarsophalangeal
joint of the medial
four toes. It is
unable to extend the
interphalangeal joint
without the
assistance of the
lumbrical muscles.

Detailed knowledge of the foot muscles are not needed for the MRCS part A
Nerves in the foot

http://www.emrcs.com/question/question.php?q=0

2/3

15/05/2015

Lateral plantar nerve


Passes anterolaterally towards the base of the 5th metatarsal between flexor digitorum brevis
and flexor accessorius. On the medial aspect of the lateral plantar artery. At the base of the
5th metatarsal it splits into superficial and deep branches.
Medial plantar nerve
Passes forwards with the medial plantar artery under the cover of the flexor retinaculum to
the interval between abductor hallucis and flexor digitorum brevis on the sole of the foot.
Plantar arteries
Arise under the cover of the flexor retinaculum, midway between the tip of the medial
malleolus and the most prominent part of the medial side of the heel.

Medial plantar artery. Passes forwards medial to medial plantar nerve in the space
between abductor hallucis and flexor digitorum brevis.Ends by uniting with a branch of
the 1st plantar metatarsal artery.
Lateral plantar artery. Runs obliquely across the sole of the foot. It lies lateral to the
lateral plantar nerve. At the base of the 5th metatarsal bone it arches medially across
the foot on the metatarsals

Dorsalis pedis artery


This vessel is a direct continuation of the anterior tibial artery. It commences on the front of
the ankle joint and runs to the proximal end of the first metatarsal space. Here is gives off the
arcuate artery and continues forwards as the first dorsal metatarsal artery. It is accompanied
by two veins throughout its length. It is crossed by the extensor hallucis brevis
Rate question:

Next question

Comment on this question

All contents of this site are 2012 E-Medical Revision Ltd

http://www.emrcs.com/question/question.php?q=0

Terms and Conditions

Privacy policy

3/3

15/05/2015

Reference ranges

Previous

Question 24 of 517

Next

A 67 year old man is due to undergo a revisional total hip replacement using a posterior
approach. After dividing gluteus maximus in the line of its fibres there is brisk arterial
bleeding. Which of the following vessels is likely to be responsible?

Question stats

Score: 82.8%
1

11.7%

9.2%

7.8%

14.9%

56.4%

A. Profunda femoris artery


56.4% of users answered this
question correctly

B. External iliac artery

End and review

6
7

C. Internal iliac artery


8

D. Obturator artery
9

Search

E. Inferior gluteal artery

Go

Next question

10
11
12

Theme from April 2014 Exam


The inferior gluteal artery runs on the deep surface of the gluteus maximus muscle. It is a
branch of the internal iliac artery. It is commonly divided during the posterior approach to the
hip joint.

13
14
15
16

Hip joint

17
18

Head of femur articulates with acetabulum of the pelvis


Both covered by articular hyaline cartilage
The acetabulum forms at the union of the ilium, pubis, and ischium
The triradiate cartilage (Y-shaped growth plate) separates the pelvic bones
The acetabulum holds the femoral head by the acetabular labrum

19-21 3 / 3
22
23
24

Normal angle between femoral head and femoral shaft is 130o

Ligaments
Transverse ligament: joints anterior and posterior ends of the articular cartilage
Head of femur ligament (ligamentum teres): acetabular notch to the fovea. Contains
arterial supply to head of femur in children.

Image sourced from Wikipedia

Image sourced from Wikipedia

http://www.emrcs.com/question/question.php?q=0

1/2

15/05/2015

Extracapsular ligaments
Iliofemoral ligament: inverted Y shape. Anterior iliac spine to the trochanteric line
Pubofemoral ligament: acetabulum to lesser trochanter
Ischiofemoral ligament: posterior support. Ischium to greater trochanter.

Blood supply
Medial circumflex femoral and lateral circumflex femoral arteries (Branches of profunda
femoris). Also from the inferior gluteal artery. These form an anastomosis and travel to up
the femoral neck to supply the head.

Rate question:

Next question

Comment on this question

All contents of this site are 2012 E-Medical Revision Ltd

http://www.emrcs.com/question/question.php?q=0

Terms and Conditions

Privacy policy

2/2

15/05/2015

Reference ranges

Previous

Question 25 of 517

Next

A 17 year old lady presents with right iliac fossa pain and diagnosed as having acute
appendicitis. You take her to theatre to perform a laparoscopic appendicectomy. During the
procedure the scrub nurse distracts you and you inadvertently avulse the appendicular
artery. The ensuing haemorrhage is likely to be supplied directly from which vessel?

Question stats

Score: 83.3%
1

10.6%

16.9%

58.8%

6.1%

7.6%

58.8% of users answered this


question correctly

A. Inferior mesenteric artery

End and review

B. Superior mesenteric artery

6
7
8

C. Ileo-colic artery

Search

D. Internal iliac artery


E. None of the above

Go

10
11

Next question

12
13

Theme from April 2014 exam


The appendicular artery is a branch of the ileocolic artery.

14
15

Appendix

16
17
18

Location: Base of caecum.


Up to 10cm long.
Mainly lymphoid tissue (Hence mesenteric adenitis may mimic appendicitis).
Caecal taenia coli converge at base of appendix and form a longitudinal muscle cover
over the appendix. This convergence should facilitate its identification at surgery if it is
retrocaecal and difficult to find (which it can be when people start doing
appendicectomies!)
Arterial supply: Appendicular artery (branch of the ileocolic).
It is intra peritoneal.

19-21 3 / 3
22
23
24
25

McBurney's point
1/3 of the way along a line drawn from the Anterior Superior Iliac Spine to the
Umbilicus

6 Positions:

Retrocaecal 74%
Pelvic 21%
Postileal
Subcaecal
Paracaecal
Preileal

Rate question:

Next question

Comment on this question

All contents of this site are 2012 E-Medical Revision Ltd

http://www.emrcs.com/question/question.php?q=0

Terms and Conditions

Privacy policy

1/1

15/05/2015

Reference ranges

Question 26 of 517

Previous

Next

A 63 year old man who smokes heavily presents with dyspepsia. He is tested and found to be
positive for helicobacter pylori infection. One evening he has an episode of haematemesis
and collapses. What is the most likely vessel to be responsible?

Question stats

Score: 83.9%
1

11.5%

16%

8.7%

51.1%

12.8%

A. Portal vein
51.1% of users answered this
question correctly

B. Short gastric arteries

End and review

6
7

C. Superior mesenteric artery


8

D. Gastroduodenal artery
9

Search

E. None of the above

Go

10
11

Next question

12

Theme from January 2012 exam

13
14

He is most likely to have a posteriorly sited duodenal ulcer. These can invade the
gastroduodenal artery and present with major bleeding. Although gastric ulcers may invade
vessels they do not tend to produce major bleeding of this nature.

15
16
17

Gastroduodenal artery

18
19-21 3 / 3

Supplies
Pylorus, proximal part of the duodenum, and indirectly to the pancreatic head (via the
anterior and posterior superior pancreaticoduodenal arteries)

22
23
24

Path
Most commonly arises from the common hepatic artery of the coeliac trunk
Terminates by bifurcating into the right gastroepiploic artery and the superior
pancreaticoduodenal artery

25
26

Image showing stomach reflected superiorly to illustrate the relationship of the


gastroduodenal artery to the first part of the duodenum

Image sourced from Wikipedia

Rate question:

Next question

Comment on this question

All contents of this site are 2012 E-Medical Revision Ltd

http://www.emrcs.com/question/question.php?q=0

Terms and Conditions

Privacy policy

1/1

15/05/2015

Reference ranges

Previous

Question 27 of 517

Next

Question stats

End and review

Score: 84.4%
1

8.4%

7.7%

12.4%

A. Lateral femoral cutaneous nerve

63.6%

B. Femoral nerve

8%

C. Obturator nerve

63.6% of users answered this


question correctly

Which of the following nerves is responsible for the cremasteric reflex?

D. Genitofemoral nerve
E. None of the above

7
8
9

Search
Next question
Go

10
11

The motor and sensory fibres of the genitofemoral nerve are tested in the cremasteric reflex.
A small contribution is also played by the ilioinguinal nerve and thus the reflex may be lost
following an inguinal hernia repair.

12
13
14

Genitofemoral nerve

15
16

Supplies
Small area of the upper medial thigh.

17
18

Path

19-21 3 / 3

Arises from the first and second lumbar nerves.


Passes obliquely through psoas major, and emerges from its medial border opposite
the fibrocartilage between the third and fourth lumbar vertebrae.
It then descends on the surface of psoas major, under cover of the peritoneum
Divides into genital and femoral branches.
The genital branch passes through the inguinal canal, within the spermatic cord, to
supply the skin overlying the skin and fascia of the scrotum. The femoral branch
enters the thigh posterior to the inguinal ligament, lateral to the femoral artery. It
supplies an area of skin and fascia over the femoral triangle.
It may be injured during abdominal or pelvic surgery, or during inguinal hernia repairs.

Rate question:

22
23
24
25
26
27

Next question

Comment on this question

All contents of this site are 2012 E-Medical Revision Ltd

http://www.emrcs.com/question/question.php?q=0

Terms and Conditions

Privacy policy

1/1

15/05/2015

Reference ranges

Previous

Question 28 of 517

Next

Question stats

End and review

Score: 84.8%
1

11.4%

7.8%

6.6%

A. Thymus

14.1%

B. Heart

60.2%

C. Great vessels

60.2% of users answered this


question correctly

Which of the following structures is not located within the mediastinum?

D. Arch of azygos vein


E. Vertebral bodies

6
7
8
9

Search
Next question
Go

10
11

The vertebral bodies lie outside of the mediastinum, as do the lungs.

12

Mediastinum

13
14

Region between the pulmonary cavities.


It is covered by the mediastinal pleura. It does not contain the lungs.
It extends from the thoracic inlet superiorly to the diaphragm inferiorly.

15
16
17

Mediastinal regions

18

Superior mediastinum (between manubriosternal angle and T4/5)


Middle mediastinum
Posterior mediastinum
Anterior mediastinum

19-21 3 / 3
22
23
24
25

Region
Superior mediastinum

Anterior mediastinum

Middle mediastinum

Posterior mediastinum

Contents

26
27

Superior vena cava


Brachiocephalic veins
Arch of aorta
Thoracic duct
Trachea
Oesophagus
Thymus
Vagus nerve
Left recurrent laryngeal nerve
Phrenic nerve

28

Thymic remnants
Lymph nodes
Fat

Pericardium
Heart
Aortic root
Arch of azygos vein
Main bronchi

Oesophagus
Thoracic aorta
Azygos vein
Thoracic duct
Vagus nerve
Sympathetic nerve trunks
Splanchnic nerves

Rate question:

Next question

Comment on this question

All contents of this site are 2012 E-Medical Revision Ltd

http://www.emrcs.com/question/question.php?q=0

Terms and Conditions

Privacy policy

1/1

15/05/2015

Reference ranges

Previous

Question 29 of 517

Next

A 22 year old man is stabbed in the chest at the level of the junction between the sternum
and manubrium. Which structure is at greatest risk?

A. Left atrium
B. Oesophagus

Question stats

Score: 85.7%
1

10.8%

10.9%

6.4%

9.6%

62.3%

62.3% of users answered this


question correctly

C. Thyroid gland

End and review

D. Inferior vena cava

6
7
8

E. Aortic arch

Search
Go

Next question

10
11

Theme from 2009 Exam


At the level of the Angle of Louis (Manubriosternal angle), is the surface marking for the
aortic arch. The oesophagus is posteriorly located and at less risk.

12

Sternal angle

15

13
14

16

Anatomical structures at the level of the manubrium and upper sternum


Upper part of the manubrium

17
18

Left brachiocephalic vein


Brachiocephalic artery
Left common carotid
Left subclavian artery

19-21 3 / 3
22
23

Lower part of the manubrium/


manubrio-sternal angle

Rate question:

24

Costal cartilages of the 2nd ribs


Transition point between superior and
inferior mediastinum
Arch of the aorta
Tracheal bifurcation
Union of the azygos vein and superior
vena cava
The thoracic duct crosses to the midline

25
26
27
28
29

Next question

Comment on this question

All contents of this site are 2012 E-Medical Revision Ltd

http://www.emrcs.com/question/question.php?q=0

Terms and Conditions

Privacy policy

1/1

15/05/2015

Reference ranges

Question 31 of 517

Previous

Next

Question stats

End and review

Score: 83.8%
1

10.8%

7.7%

10.6%

A. Hyperplasia of the annulus fibrosus

21.4%

B. Proliferation of the nucleus pulposus

49.5%

C. Fusion of the transverse processes of the 6th and 7th cervical


vertebrae

49.5% of users answered this


question correctly

A cervical rib is due to which of the following?

D. An accessory cervical vertebra

6
7
8

E. Elongation of the transverse processes of the 7th cervical vertebra

Search
Go

Next question

10
11
12

Cervical ribs occur as a result of the elongation of the transverse process of the 7th cervical
vertebra. It is usually a fibrous band that attaches to the first thoracic rib.

13
14

Cervical ribs

15
16

0.2-0.4% incidence
Most cases present with neurological symptoms
Consist of an anomalous fibrous band that often originates from C7 and may arc
towards, but rarely reaches the sternum
Congenital cases may present around the third decade, some cases are reported to
occur following trauma
Bilateral in up to 70%
Compression of the subclavian artery may produce absent radial pulse on clinical
examination and in particular may result in a positive Adsons test (lateral flexion of the
neck towards the symptomatic side and traction of the symptomatic arm- leads to
obliteration of radial pulse)
Treatment is most commonly undertaken when there is evidence of neurovascular
compromise. A transaxillary approach is the traditional operative method for excision.

17
18
19-21 3 / 3
22
23
24
25
26
27
28
29
30
31

Image sourced from Wikipedia

3D reconstruction of a left-sided cervical rib


Rate question:

Next question

Comment on this question

All contents of this site are 2012 E-Medical Revision Ltd

http://www.emrcs.com/question/question.php?q=0

Terms and Conditions

Privacy policy

1/1

15/05/2015

Reference ranges

Previous

Question 32 of 517

Next

Question stats

End and review

Score: 84.2%
1

10.4%

11.4%

11.6%

A. Internal jugular vein

54.2%

B. Internal carotid artery

12.4%

C. Vagus nerve

54.2% of users answered this


question correctly

Which of the structures listed below is not a content of the carotid sheath?

D. Recurrent laryngeal nerve


E. Common carotid artery

6
7
8
9

Search
Next question
Go

10
11

Contents of carotid sheath:


Common carotid artery
Internal carotid artery
Internal jugular vein
Vagus nerve

12
13
14
15
16

Theme from April 2014 exam

17
18

Common carotid artery


The right common carotid artery arises at the bifurcation of the brachiocephalic trunk, the left
common carotid arises from the arch of the aorta. Both terminate at the level of the upper
border of the thyroid cartilage (the lower border of the third cervical vertebra) by dividing into
the internal and external carotid arteries.
Left common carotid artery
This vessel arises immediately to the left and slightly behind the origin of the brachiocephalic
trunk. Its thoracic portion is 2.5- 3.5 cm in length and runs superolaterally to the
sternoclavicular joint.

19-21 3 / 3
22
23
24
25
26
27
28
29

In the thorax
The vessel is in contact, from below upwards, with the trachea, left recurrent laryngeal nerve,
left margin of the oesophagus. Anteriorly the left brachiocephalic vein runs across the artery,
and the cardiac branches from the left vagus descend in front of it. These structures
together with the thymus and the anterior margins of the left lung and pleura separate the
artery from the manubrium.

30
31
32

In the neck
The artery runs superiorly deep to sternocleidomastoid and then enters the anterior triangle.
At this point it lies within the carotid sheath with the vagus nerve and the internal jugular vein.
Posteriorly the sympathetic trunk lies between the vessel and the prevertebral fascia. At the
level of C7 the vertebral artery and thoracic duct lie behind it. The anterior tubercle of C6
transverse process is prominent and the artery can be compressed against this structure (it
corresponds to the level of the cricoid).
Anteriorly at C6 the omohyoid muscle passes superficial to the artery.
Within the carotid sheath the jugular vein lies lateral to the artery.
Right common carotid artery
The right common carotid arises from the brachiocephalic artery. The right common carotid
artery corresponds with the cervical portion of the left common carotid, except that there is
no thoracic duct on the right. The oesophagus is less closely related to the right carotid than
the left.
Summary points about the carotid anatomy
Path
Passes behind the sternoclavicular joint (12% patients above this level) to the upper border
of the thyroid cartilage, to divide into the external (ECA) and internal carotid arteries (ICA).
Relations
Level of 6th cervical vertebra crossed by omohyoid
Then passes deep to the thyrohyoid, sternohyoid, sternomastoid muscles.
Passes anterior to the carotid tubercle (transverse process 6th cervical vertebra)-NB
compression here stops haemorrhage.
The inferior thyroid artery passes posterior to the common carotid artery.
Then : Left common carotid artery crossed by thoracic duct, Right common carotid
artery crossed by recurrent laryngeal nerve
http://www.emrcs.com/question/question.php?q=0

1/2

15/05/2015

Image sourced from Wikipedia

Rate question:

Next question

Comment on this question

All contents of this site are 2012 E-Medical Revision Ltd

http://www.emrcs.com/question/question.php?q=0

Terms and Conditions

Privacy policy

2/2

15/05/2015

Reference ranges

Previous

Question 33 of 517

Next

A 22 year old man is undergoing a wedge excision of his great toenail. As the surgeon
passes a needle into the area to administer local anaesthetic, the patient notices a sharp
pain. By which pathway will this sensation be conveyed to the central nervous system?

Question stats

Score: 84.6%
1

11.9%

10.1%

8.6%

7.8%

61.5%

A. Anterior corticospinal tract


61.5% of users answered this
question correctly

B. Posterior spinocerebellar tract

End and review

6
7

C. Cuneate fasciculus
8

D. Vestibulospinal tract
9

Search

E. Spinothalamic tract

Go

10
11

Next question

12
13

Spinothalamic tract- Pain and temperature


Vestibulospinal tract- Motor neuronal signals relating to posture
Cuneate fasciculus- Fine touch, pressure and proprioception
Posterior spinocerebellar tract- Proprioceptive signals to cerebellum
Anterior corticospinal tract- Conveys motor signals from precentral gyrus to motor cells
within the cord

14
15
16
17
18
19-21 3 / 3

Theme from January 2011 Exam


Pain impulses are transmitted via the spinothalamic tract

22
23

Spinothalamic tract

24
25

The spinothalamic tract transmits impulses from receptors which measure crude touch, pain
and temperature. The spinothalamic tract comprises the lateral and anterior spinothalamic
tracts, the former typically transmits pain and temperature and the latter crude touch and
pressure. Neurones transmitting these signals will typically ascend by one or two vertebral
levels in Lissaurs tract prior to decussating in the spinal cord itself. Neurones then pass
rostrally in the cord to connect at the thalamus.

26
27
28
29
30

Rate question:

Next question

31
32

Comment on this question

33

All contents of this site are 2012 E-Medical Revision Ltd

http://www.emrcs.com/question/question.php?q=0

Terms and Conditions

Privacy policy

1/1

15/05/2015

Reference ranges

Previous

Question 34 of 517

Next

A 73 year old lady is admitted with brisk rectal bleeding. Despite attempts at resuscitation the
bleeding proceeds to cause haemodynamic compromise. An upper GI endoscopy is normal.
A mesenteric angiogram is performed and a contrast blush is seen in the region of the
sigmoid colon. The radiologist decides to embolise the vessel supplying this area. At what
spinal level does it leave the aorta?

Question stats

Score: 85%
1

16.3%

14%

16.6%

45.3%

7.8%

45.3% of users answered this


question correctly

A. L2

End and review

6
7
8

B. L1
C. L4

Search

D. L3

Go

E. T10

10
11
12

Next question

13
14

Theme from 2009 Exam

15

The inferior mesenteric artery leaves the aorta at L3. It supplies the left colon and sigmoid.
Its proximal continuation to communicate with the middle colic artery is via the marginal
artery.

16
17
18
19-21 3 / 3

Levels

22

Transpyloric plane
Level of the body of L1

23
24
25

Pylorus stomach
Left kidney hilum (L1- left one!)
Right hilum of the kidney (1.5cm lower than the left)
Fundus of the gallbladder
Neck of pancreas
Duodenojejunal flexure
Superior mesenteric artery
Portal vein
Left and right colic flexure
Root of the transverse mesocolon
2nd part of the duodenum
Upper part of conus medullaris
Spleen

26
27
28
29
30
31
32
33
34

Can be identified by asking the supine patient to sit up without using their arms. The plane is
located where the lateral border of the rectus muscle crosses the costal margin.
Anatomical planes
Subcostal plane

Lowest margin of 10th costal cartilage

Intercristal plane

Level of body L4 (highest point of iliac crest)

Intertubercular plane

Level of body L5

Common level landmarks


Inferior mesenteric artery

L3

Bifurcation of aorta into common iliac arteries

L4

Formation of IVC

L5 (union of common iliac veins)

Diaphragm apertures

Rate question:

Vena cava T8
Oesophagus T10
Aortic hiatus T12

Next question

Comment on this question

http://www.emrcs.com/question/question.php?q=0

1/2

15/05/2015

All contents of this site are 2012 E-Medical Revision Ltd

http://www.emrcs.com/question/question.php?q=0

Terms and Conditions

Privacy policy

2/2

15/05/2015

Reference ranges

Question 35 of 517

Previous

Next

A 23 year old lady with troublesome axillary hyperhidrosis is undergoing a thorascopic


sympathectomy to treat the condition. Which of the following structures will need to be
divided to access the sympathetic trunk?

Question stats

Score: 81%
1

11.4%

9.7%

45.3%

11.5%

22%

A. Intercostal vein
45.3% of users answered this
question correctly

B. Intercostal artery

End and review

6
7

C. Parietal pleura
8

D. Visceral pleura
9

Search

E. None of the above

Go

10
11

Next question

12

The sympathetic chain lies posterior to the parietal pleura. During a thorascopic
sympathetomy this structure will need to be divided. The intercostal vessels lie posteriorly.
They may be damaged with troublesome bleeding but otherwise are best left alone as
deliberate division will not improve surgical access.

13
14
15
16

Sympathetic nervous system- anatomy

17
18

The cell bodies of the pre-ganglionic efferent neurones lie in the lateral horn of the grey
matter of the spinal cord in the thoraco-lumbar regions.
The pre-ganglionic efferents leave the spinal cord at levels T1-L2. These pass to the
sympathetic chain.
Lateral branches of the sympathetic chain connect it to every spinal nerve. These post
ganglionic nerves will pass to structures that receive sympathetic innervation at the
periphery.

19-21 3 / 3
22
23
24
25
26

Sympathetic chains
These lie on the vertebral column and run from the base of the skull to the coccyx.

27

Cervical
region

Lie anterior to the transverse processes of the cervical vertebrae and posterior to the
carotid sheath.

Thoracic
region

Lie anterior to the neck of the upper ribs and and lateral sides of the lower thoracic
vertebrae.They are covered by the parietal pleura

Lumbar
region

Enter by passing posterior to the medial arcuate ligament. Lie anteriorly to the
vertebrae and medial to psoas major.

28
29
30
31
32
33
34

Sympathetic ganglia

35

Superior cervical ganglion lies anterior to C2 and C3.


Middle cervical ganglion (if present) C6
Stellate ganglion- anterior to transverse process of C7, lies posterior to the subclavian
artery, vertebral artery and cervical pleura.
Thoracic ganglia are segmentally arranged.
There are usually 4 lumbar ganglia.

Clinical importance
Interruption of the head and neck supply of the sympathetic nerves will result in an
ipsilateral Horners syndrome.
For treatment of hyperhidrosis the sympathetic denervation can be achieved by
removing the second and third thoracic ganglia with their rami. Removal of T1 will
cause a Horners syndrome and is therefore not performed.
In patients with vascular disease of the lower limbs a lumbar sympathetomy may be
performed, either radiologically or (more rarely now) surgically. The ganglia of L2 and
below are disrupted. If L1 is removed then ejaculation may be compromised (and little
additional benefit conferred as the preganglionic fibres do not arise below L2.

Rate question:

Next question

Comment on this question

All contents of this site are 2012 E-Medical Revision Ltd

http://www.emrcs.com/question/question.php?q=0

Terms and Conditions

Privacy policy

1/1

15/05/2015

Reference ranges

Previous

Question 36 of 517

Next

Question stats

End and review

Score: 79.1%
1

22.1%

17.4%

45.8%

A. Subdural space

8.5%

B. Epidural space

6.1%

C. Subarachnoid space

45.8% of users answered this


question correctly

In which space is a lumbar puncture performed?

D. Extradural space
E. Intraventricular space

6
7
8
9

Search
Next question
Go

10
11

Theme from January 2013 exam


Samples of CSF are normally obtained by inserting a needle between the third and fourth
lumbar vertebrae. The tip of the needle lies in the sub arachnoid space, the spinal cord
terminates at L1 and is not at risk of injury. Clinical evidence of raised intracranial pressure is
a contraindication to lumbar puncture.

12
13
14
15

Cerebrospinal fluid

16
17

The CSF fills the space between the arachnoid mater and pia mater (covering surface of the
brain). The total volume of CSF in the brain is approximately 150ml. Approximately 500 ml is
produced by the ependymal cells in the choroid plexus (70%), or blood vessels (30%). It is
reabsorbed via the arachnoid granulations which project into the venous sinuses.

18
19-21 3 / 3
22
23

Circulation
1. Lateral ventricles (via foramen Munro)
2. 3rd ventricle
3. Cerebral aqueduct (aqueduct Sylvius)
4. 4th ventricle (via foramina of Magendie and Luschka)
5. Subarachnoid space
6. Reabsorbed into venous system via arachnoid granulations in superior sagittal sinus

24
25
26
27
28
29

Composition
30

Glucose: 50-80mg/dl
Protein: 15-40 mg/dl
Red blood cells: Nil

31

White blood cells: 0-3 cells/ mm3

33

32

34
35

Rate question:

36

Next question

Comment on this question

All contents of this site are 2012 E-Medical Revision Ltd

http://www.emrcs.com/question/question.php?q=0

Terms and Conditions

Privacy policy

1/1

15/05/2015

Reference ranges

Question 37 of 517

Previous

Next

A 21 year old man is stabbed in the antecubital fossa. A decision is made to surgically
explore the wound. At operation the surgeon dissects down onto the brachial artery. A nerve
is identified medially, which nerve is it likely to be?

Question stats

Score: 79.5%
1

10.9%

8%

9.9%

18.6%

52.7%

A. Radial
52.7% of users answered this
question correctly

B. Recurrent branch of median

End and review

6
7

C. Anterior interosseous
8

D. Ulnar
9

Search

E. Median

Go

Next question

10
11
12

Theme from September 2012 Exam

13
14
15

Median nerve

16

The median nerve is formed by the union of a lateral and medial root respectively from the
lateral (C5,6,7) and medial (C8 and T1) cords of the brachial plexus; the medial root passes
anterior to the third part of the axillary artery. The nerve descends lateral to the brachial
artery, crosses to its medial side (usually passing anterior to the artery). It passes deep to
the bicipital aponeurosis and the median cubital vein at the elbow.
It passes between the two heads of the pronator teres muscle, and runs on the deep surface
of flexor digitorum superficialis (within its fascial sheath).
Near the wrist it becomes superficial between the tendons of flexor digitorum superficialis and
flexor carpi radialis, deep to palmaris longus tendon. It passes deep to the flexor retinaculum
to enter the palm, but lies anterior to the long flexor tendons within the carpal tunnel.

17
18
19-21 3 / 3
22
23
24
25
26
27

Branches
Region

Branch

Upper
arm

No branches, although the nerve commonly communicates with the


musculocutaneous nerve

Forearm

Pronator teres
Flexor carpi radialis
Palmaris longus
Flexor digitorum superficialis
Flexor pollicis longus
Flexor digitorum profundus (only the radial half)

28
29
30
31

Distal
forearm

Palmar cutaneous branch

Hand
(Motor)

Motor supply (LOAF)

Hand
(Sensory)

32
33
34
35
36
37

Lateral 2 lumbricals
Opponens pollicis
Abductor pollicis brevis
Flexor pollicis brevis

Over thumb and lateral 2 fingers


On the palmar aspect this projects proximally, on the dorsal aspect only the
distal regions are innervated with the radial nerve providing the more proximal
cutaneous innervation.

Patterns of damage
Damage at wrist
e.g. carpal tunnel syndrome
paralysis and wasting of thenar eminence muscles and opponens pollicis (ape hand
deformity)
sensory loss to palmar aspect of lateral (radial) 2 fingers

Damage at elbow, as above plus:


unable to pronate forearm
weak wrist flexion
ulnar deviation of wrist

http://www.emrcs.com/question/question.php?q=0

1/2

15/05/2015

Anterior interosseous nerve (branch of median nerve)


leaves just below the elbow
results in loss of pronation of forearm and weakness of long flexors of thumb and index
finger

Topography of the median nerve

Image sourced from Wikipedia

Rate question:

Next question

Comment on this question

All contents of this site are 2012 E-Medical Revision Ltd

http://www.emrcs.com/question/question.php?q=0

Terms and Conditions

Privacy policy

2/2

15/05/2015

Reference ranges

Question 38 of 517

Previous

Next

A 65 year old man with long standing atrial fibrillation develops an embolus to the lower leg.
The decision is made to perform an embolectomy, utilising a trans popliteal approach. After
incising the deep fascia, which of the following structures will the surgeons encounter first on
exploring the central region of the popliteal fossa?

Question stats

Score: 80%
1

27.4%

15.9%

15.4%

34.5%

6.8%

34.5% of users answered this


question correctly

A. Popliteal vein

End and review

B. Common peroneal nerve

6
7
8

C. Popliteal artery

Search

D. Tibial nerve
E. None of the above

Go

10
11

Next question

12
13

Theme from 2009 Exam


Theme from January 2013 Exam
The tibial nerve lies superior to the vessels in the inferior aspect of the popliteal fossa. In the
upper part of the fossa the tibial nerve lies lateral to the vessels, it then passes superficial to
them to lie medially. The popliteal artery is the deepest structure in the popliteal fossa.

14
15
16
17
18

Popliteal fossa

19-21 3 / 3
22

Boundaries of the popliteal fossa


Laterally

Biceps femoris above, lateral head of gastrocnemius and plantaris below

Medially

Semimembranosus and semitendinosus above, medial head of gastrocnemius


below

23
24
25
26

Floor

Popliteal surface of the femur, posterior ligament of knee joint and popliteus muscle

Roof

Superficial and deep fascia

27
28
29

Image showing the popliteal fossa

30
31
32
33
34
35
36
37
38

Image sourced from Wikipedia

Contents
Popliteal artery and vein
Small saphenous vein
Common peroneal nerve
Tibial nerve
Posterior cutaneous nerve of the thigh
Genicular branch of the obturator nerve
Lymph nodes

Rate question:

Next question

Comment on this question

All contents of this site are 2012 E-Medical Revision Ltd

http://www.emrcs.com/question/question.php?q=0

Terms and Conditions

Privacy policy

1/1

15/05/2015

Reference ranges

Previous

Question 39 of 517

Next

A 43 year old man is undergoing a right hemicolectomy and the ileo-colic artery is ligated.
From which of the following vessels is is derived?

A. Inferior mesenteric artery


B. Superior mesenteric artery

Question stats

Score: 80.4%
1

17.7%

58.5%

9.2%

8.3%

6.3%

58.5% of users answered this


question correctly

C. Coeliac axis

End and review

D. Aorta

6
7
8

E. None of the above

Search
Next question

Go

10
11

The ileocolic artery is a branch of the SMA and supplies the right colon and terminal ileum.
The transverse colon is supplied by the middle colic artery. As veins accompany arteries in
the mesentery and are lined by lymphatics, high ligation is the norm in cancer resections.
The ileo-colic artery branches off the SMA near the duodenum.

12
13
14
15

Colon anatomy

16
17

The colon commences with the caecum. This represents the most dilated segment of the
human colon and its base (which is intraperitoneal) is marked by the convergence of teniae
coli. At this point is located the vermiform appendix. The colon continues as the ascending
colon, the posterior aspect of which is retroperitoneal. The line of demarcation between the
intra and retro peritoneal right colon is visible as a white line, in the living, and forms the line
of incision for colonic resections.

18
19-21 3 / 3
22
23
24

The ascending colon becomes the transverse colon after passing the hepatic flexure. At this
located the colon becomes wholly intra peritoneal once again. The superior aspect of the
transverse colon is the point of attachment of the transverse colon to the greater omentum.
This is an important anatomical site since division of these attachments permits entry into the
lesser sac. Separation of the greater omentum from the transverse colon is a routine
operative step in both gastric and colonic resections.

25

At the left side of the abdomen the transverse colon passes to the left upper quadrant and
makes an oblique inferior turn at the splenic flexure. Following this, the posterior aspect
becomes retroperitoneal once again.

30

At the level of approximately L4 the descending colon becomes wholly intraperitoneal and
becomes the sigmoid colon. Whilst the sigmoid is wholly intraperitoneal there are usually
attachments laterally between the sigmoid and the lateral pelvic sidewall. These small
congenital adhesions are not formal anatomical attachments but frequently require division
during surgical resections.

33

26
27
28
29

31
32

34
35
36
37

At its distal end the sigmoid passes to the midline and at the region around the sacral
promontary it becomes the upper rectum. This transition is visible macroscopically as the
point where the teniae fuse. More distally the rectum passes through the peritoneum at the
region of the peritoneal reflection and becomes extraperitoneal.

38
39

Arterial supply
Superior mesenteric artery and inferior mesenteric artery: linked by the marginal artery.
Ascending colon: ileocolic and right colic arteries
Transverse colon: middle colic artery
Descending and sigmoid colon: inferior mesenteric artery
Venous drainage
From regional veins (that accompany arteries) to superior and inferior mesenteric vein
Lymphatic drainage
Initially along nodal chains that accompany supplying arteries, then para-aortic nodes.
Embryology
Midgut- Second part of duodenum to 2/3 transverse colon
Hindgut- Distal 1/3 transverse colon to anus
Peritoneal location
The right and left colon are part intraperitoneal and part extraperitoneal. The sigmoid and
transverse colon are generally wholly intraperitoneal. This has implications for the sequelae
of perforations, which will tend to result in generalised peritonitis in the wholly intra peritoneal
segments.

http://www.emrcs.com/question/question.php?q=0

1/2

15/05/2015

Colonic relations
Region of colon

Relation

Caecum/ right colon

Right ureter, gonadal vessels

Hepatic flexure

Gallbladder (medially)

Splenic flexure

Spleen and tail of pancreas

Distal sigmoid/ upper


rectum

Left ureter

Rectum

Ureters, autonomic nerves, seminal vesicles, prostate, urethra


(distally)

Rate question:

Next question

Comment on this question

All contents of this site are 2012 E-Medical Revision Ltd

http://www.emrcs.com/question/question.php?q=0

Terms and Conditions

Privacy policy

2/2

15/05/2015

Reference ranges

Question 40 of 517

Previous

Next

A 53 year old man is undergoing a distal pancreatectomy for trauma. Which of the following
vessels is responsible for the arterial supply to the tail of the pancreas?

A. Splenic artery
B. Pancreaticoduodenal artery

Question stats

Score: 80.9%
1

59.3%

17%

9%

7.1%

7.5%

59.3% of users answered this


question correctly

C. Gastric artery

End and review

D. Hepatic artery

6
7
8

E. Superior mesenteric artery

Search
Next question

Go

10
11
12

Pancreatic head is supplied by the pancreaticoduodenal artery


Pancreatic tail is supplied by branches of the splenic artery

13
14

Theme from April 2014 Exam


There is an arterial "watershed" in the supply between the head and tail of the pancreas.
The head is supplied by the pancreaticoduodenal artery and the tail is supplied by branches
of the splenic artery.

15
16
17
18
19-21 3 / 3

Pancreas

22

The pancreas is a retroperitoneal organ and lies posterior to the stomach. It may be
accessed surgically by dividing the peritoneal reflection that connects the greater omentum
to the transverse colon. The pancreatic head sits in the curvature of the duodenum. Its tail
lies close to the hilum of the spleen, a site of potential injury during splenectomy.

23
24
25
26

Relations
Posterior to the pancreas
Pancreatic head

Inferior vena cava


Common bile duct
Right and left renal veins
Superior mesenteric vein and artery

Pancreatic neck

Superior mesenteric vein, portal vein

Pancreatic body-

Left renal vein


Crus of diaphragm
Psoas muscle
Adrenal gland
Kidney
Aorta

Pancreatic tail

Left kidney

27
28
29
30
31
32
33
34
35
36
37
38
39

Anterior to the pancreas


Pancreatic head

1st part of the duodenum


Pylorus
Gastroduodenal artery
SMA and SMV(uncinate process)

Pancreatic body

Stomach
Duodenojejunal flexure

Pancreatic tail

Splenic hilum

40

Superior to the pancreas


Coeliac trunk and its branches common hepatic artery and splenic artery
Grooves of the head of the pancreas
2nd and 3rd part of the duodenum
Arterial supply
Head: pancreaticoduodenal artery
Rest: splenic artery

Venous drainage

http://www.emrcs.com/question/question.php?q=0

1/2

15/05/2015

Head: superior mesenteric vein


Body and tail: splenic vein

Ampulla of Vater
Merge of pancreatic duct and common bile duct
Is an important landmark, halfway along the second part of the duodenum, that marks
the anatomical transition from foregut to midgut (also the site of transition between
regions supplied by coeliac trunk and SMA).

Image sourced from Wikipedia

Rate question:

Next question

Comment on this question

All contents of this site are 2012 E-Medical Revision Ltd

http://www.emrcs.com/question/question.php?q=0

Terms and Conditions

Privacy policy

2/2

15/05/2015

Reference ranges

Previous

Question 41 of 517

Next

A 43 year old lady presents with varicose veins and undergoes a saphenofemoral
disconnection, long saphenous vein stripping to the ankle and isolated hook phlebectomies.
Post operatively she notices an area of numbness superior to her ankle. What is the most
likely cause for this?

Question stats

Score: 81.3%
1

24.3%

7.1%

41.9%

9.8%

16.9%

41.9% of users answered this


question correctly

A. Sural nerve injury

End and review

B. Femoral nerve injury

6
7
8

C. Saphenous nerve injury

Search

D. Common peroneal nerve injury


E. Superficial peroneal nerve injury

Go

10
11

Next question

12
13

The sural nerve is related to the short saphenous vein. The saphenous nerve is related to
the long saphenous vein below the knee and for this reason full length stripping of the vein is
no longer advocated.

14
15
16

Saphenous vein

17
18

Long saphenous vein


This vein may be harvested for triple or quadruple bypass surgery

19-21 3 / 3
22
23

Originates at the 1st digit where the dorsal vein merges with the dorsal venous arch of
the foot
Passes anterior to the medial malleolus and runs up the medial side of the leg
At the knee, it runs over the posterior border of the medial epicondyle of the femur
bone
Then passes laterally to lie on the anterior surface of the thigh before entering an
opening in the fascia lata called the saphenous opening
It joins with the femoral vein in the region of the femoral triangle at the saphenofemoral
junction

24
25
26
27
28
29
30
31

Tributaries

32

Medial marginal
Superficial epigastric
Superficial iliac circumflex
Superficial external pudendal veins

33
34
35
36
37

Short saphenous vein


38

Originates at the 5th digit where the dorsal vein merges with the dorsal venous arch of
the foot, which attaches to the great saphenous vein.
It passes around the lateral aspect of the foot (inferior and posterior to the lateral
malleolus) and runs along the posterior aspect of the leg (with the sural nerve)
Passes between the heads of the gastrocnemius muscle, and drains into the popliteal
vein, approximately at or above the level of the knee joint.

Rate question:

39
40
41

Next question

Comment on this question

All contents of this site are 2012 E-Medical Revision Ltd

http://www.emrcs.com/question/question.php?q=0

Terms and Conditions

Privacy policy

1/1

15/05/2015

Reference ranges

Question 42 of 517

Previous

Next

Question stats

End and review

Score: 81.6%
1

14.9%

19.3%

8.4%

A. Pronator quadratus

13.8%

B. Biceps

43.5%

C. Brachioradialis

43.5% of users answered this


question correctly

Which of the following muscles does not attach to the radius?

D. Supinator
E. Brachialis

6
7
8
9

Search
Next question
Go

The brachialis muscle inserts into the ulna. The other muscles are all inserted onto the
radius.

10
11
12
13

Radius

14

The radius is one of the two long forearm bones that extends from the lateral side of the
elbow to the thumb side of the wrist. It has two expanded ends, of which the distal end is the
larger. Key points relating to its topography and relations are outlined below;

15
16
17
18

Upper end

19-21 3 / 3

Articular cartilage- covers medial > lateral side


Articulates with radial notch of the ulna by the annular ligament
Muscle attachment- biceps brachii at the tuberosity

22
23
24
25

Shaft
Muscle attachment
Upper third of the body

26
27

Supinator
Flexor digitorum superficialis
Flexor pollicis longus

28
29

Middle third of the body

Pronator teres

30

Lower quarter of the body

Pronator quadratus
Tendon of supinator longus

31
32
33

Lower end

34

Quadrilateral
Anterior surface- capsule of wrist joint
Medial surface- head of ulna
Lateral surface- ends in the styloid process
Posterior surface: 3 grooves containing:

35

1. Tendons of extensor carpi radialis longus and brevis


2. Tendon of extensor pollicis longus
3. Tendon of extensor indicis

39

36
37
38

40
41
42

Image sourced from Wikipedia

Rate question:

Next question

Comment on this question

http://www.emrcs.com/question/question.php?q=0

1/2

15/05/2015

All contents of this site are 2012 E-Medical Revision Ltd

http://www.emrcs.com/question/question.php?q=0

Terms and Conditions

Privacy policy

2/2

15/05/2015

Reference ranges

Question 43 of 517

Previous

Next

A 25 year old man is stabbed in the upper arm. The brachial artery is lacerated at the level
of the proximal humerus, and is being repaired. A nerve lying immediately lateral to the
brachial artery is also lacerated. Which of the following is the nerve most likely to be?

Question stats

Score: 80%
1

10.4%

44.4%

21%

7.5%

16.6%

A. Ulnar nerve
44.4% of users answered this
question correctly

B. Median nerve

End and review

6
7

C. Radial nerve
8

D. Intercostobrachial nerve
9

Search

E. Axillary nerve

Go

10
11

Next question

12

The brachial artery begins at the lower border of teres major and terminates in the cubital
fossa by branching into the radial and ulnar arteries. In the upper arm the median nerve lies
closest to it in the lateral position. In the cubital fossa it lies medial to it.

13
14
15
16
17
18
19-21 3 / 3
22
23
24
25
26
27
28
29
30

Image sourced from Wikipedia

31
32

Brachial artery

33
34

The brachial artery begins at the lower border of teres major as a continuation of the axillary
artery. It terminates in the cubital fossa at the level of the neck of the radius by dividing into
the radial and ulnar arteries.

35
36
37

Relations
Posterior relations include the long head of triceps with the radial nerve and profunda
vessels intervening. Anteriorly it is overlapped by the medial border of biceps.
It is crossed by the median nerve in the middle of the arm.
In the cubital fossa it is separated from the median cubital vein by the bicipital aponeurosis.
The basilic vein is in contact at the most proximal aspect of the cubital fossa and lies
medially.

38
39
40
41
42
43

Rate question:

Next question

Comment on this question

All contents of this site are 2012 E-Medical Revision Ltd

http://www.emrcs.com/question/question.php?q=0

Terms and Conditions

Privacy policy

1/1

15/05/2015

Reference ranges

Question 44 of 517

Previous

Next

Question stats

End and review

Score: 78.4%
1

15.5%

17.9%

10.5%

A. Medial to anterior to lateral

9%

B. Lateral to posterior to medial

47.1%

C. Medial to posterior to lateral

47.1% of users answered this


question correctly

What is the course of the median nerve relative to the brachial artery in the upper arm?

D. Medial to anterior to medial


E. Lateral to anterior to medial

6
7
8
9

Search
Next question
Go

10
11

Relations of median nerve to the brachial artery:


Lateral -> Anterior -> Medial

12
13
14

Theme from 2009, 2012 and 2014 Exams

15
16

The median nerve descends lateral to the brachial artery, it usually passes anterior to the
artery to lie on its medial side. It passes deep to the bicipital aponeurosis and the median
cubital vein at the elbow. It enters the forearm between the two heads of the pronator teres
muscle.

17
18
19-21 3 / 3
22
23
24
25
26
27
28

Image sourced from Wikipedia

29
30
31

Brachial artery

32

The brachial artery begins at the lower border of teres major as a continuation of the axillary
artery. It terminates in the cubital fossa at the level of the neck of the radius by dividing into
the radial and ulnar arteries.

33
34
35

Relations
Posterior relations include the long head of triceps with the radial nerve and profunda
vessels intervening. Anteriorly it is overlapped by the medial border of biceps.
It is crossed by the median nerve in the middle of the arm.
In the cubital fossa it is separated from the median cubital vein by the bicipital aponeurosis.
The basilic vein is in contact at the most proximal aspect of the cubital fossa and lies
medially.

36
37
38
39
40
41

Rate question:

42

Next question

43

Comment on this question

44

All contents of this site are 2012 E-Medical Revision Ltd

http://www.emrcs.com/question/question.php?q=0

Terms and Conditions

Privacy policy

1/1

15/05/2015

Reference ranges

Previous

Question 45 of 517

Next

Question stats

End and review

Score: 78.8%
1

8.9%

15.7%

16.4%

A. Oculomotor nerve

11.3%

B. Internal carotid artery

47.8%

C. Opthalmic nerve

47.8% of users answered this


question correctly

Which of the following is not a content of the cavernous sinus?

D. Abducens nerve
E. Optic nerve

6
7
8
9

Search
Next question
Go

10
11

Mnemonic for contents of cavernous sinus:


O TOM CAT

12

Occulomotor nerve (III)


Trochlear nerve (IV)
Ophthalmic nerve (V1)
Maxillary nerve (V2)
Carotid artery
Abducent nerve (VI)
T

14

13

15
16
17
18
19-21 3 / 3
22

OTOM=lateral wall components


CA= components within sinus

23
24
25

The optic nerve lies above and outside the cavernous sinus.

26
27

Cavernous sinus
28

The cavernous sinuses are paired and are situated on the body of the sphenoid bone. It
runs from the superior orbital fissure to the petrous temporal bone.

30
31

Relations
Medial

29

32

Lateral

33

Pituitary fossa Temporal lobe


Sphenoid sinus

34
35

Contents

36

Lateral wall components

(from top to bottom:)


Oculomotor nerve
Trochlear nerve
Ophthalmic nerve
Maxillary nerve

37
38
39
40

Contents of the sinus

(from medial to lateral:)


Internal carotid artery (and sympathetic plexus)
Abducens nerve

41
42
43

Blood supply
Ophthalmic vein, superficial cortical veins, basilar plexus of veins posteriorly.

44
45

Drains into the internal jugular vein via: the superior and inferior petrosal sinuses

Image sourced from Wikipedia

http://www.emrcs.com/question/question.php?q=0

1/2

15/05/2015

Rate question:

Next question

Comment on this question

All contents of this site are 2012 E-Medical Revision Ltd

http://www.emrcs.com/question/question.php?q=0

Terms and Conditions

Privacy policy

2/2

15/05/2015

Reference ranges

Question 46 of 517

Previous

Next

Surgical occlusion of which of these structures, will result in the greatest reduction in hepatic
blood flow?

A. Portal vein
B. Common hepatic artery

Question stats

Score: 79.2%
1

59.5%

12.9%

8%

12.3%

7.4%

59.5% of users answered this


question correctly

C. Right hepatic artery

End and review

D. Coeliac axis

6
7
8

E. Left hepatic artery

Search
Next question

Go

10
11

The portal vein transports 70% of the blood supply to the liver, while the hepatic artery
provides 30%. The portal vein contains the products of digestion. The arterial and venous
blood is dispersed by sinusoids to the central veins of the liver lobules; these drain into the
hepatic veins and then into the IVC. The caudate lobe drains directly into the IVC rather than
into other hepatic veins.

12
13
14
15
16

Liver

17
18

Structure of the liver

19-21 3 / 3

Right lobe

Supplied by right hepatic artery


Contains Couinaud segments V to VIII (-/+Sg I)

22
23
24

Left lobe

Supplied by the left hepatic artery


Contains Couinaud segments II to IV (+/- Sg1)

25
26
27

Quadrate lobe

Part of the right lobe anatomically, functionally is part of the left


Couinaud segment IV
Porta hepatis lies behind
On the right lies the gallbladder fossa
On the left lies the fossa for the umbilical vein

28
29
30
31
32

Caudate lobe

Supplied by both right and left hepatic arteries


Couinaud segment I
Lies behind the plane of the porta hepatis
Anterior and lateral to the inferior vena cava
Bile from the caudate lobe drains into both right and left hepatic ducts

33
34
35
36
37

Detailed knowledge of Couinaud segments is not required for MRCS Part A


38

Between the liver lobules are portal canals which contain the portal triad: Hepatic
Artery, Portal Vein, tributary of Bile Duct.

39
40
41

Relations of the liver

42
43

Anterior

Postero inferiorly

Diaphragm

Oesophagus

45

Xiphoid process

Stomach

46

44

Duodenum
Hepatic flexure of colon
Right kidney
Gallbladder
Inferior vena cava
Porta hepatis
Location

Postero inferior surface, it joins nearly at right angles with the left sagittal fossa,
and separates the caudate lobe behind from the quadrate lobe in front

http://www.emrcs.com/question/question.php?q=0

1/2

15/05/2015

Transmits

Common hepatic duct


Hepatic artery
Portal vein
Sympathetic and parasympathetic nerve fibres
Lymphatic drainage of the liver (and nodes)

Ligaments
Falciform
ligament

2 layer fold peritoneum from the umbilicus to anterior liver surface


Contains ligamentum teres (remnant umbilical vein)
On superior liver surface it splits into the coronary and left
triangular ligaments

Ligamentum teres

Joins the left branch of the portal vein in the porta hepatis

Ligamentum
venosum

Remnant of ductus venosus

Arterial supply
Hepatic artery

Venous
Hepatic veins
Portal vein

Nervous supply
Sympathetic and parasympathetic trunks of coeliac plexus

Rate question:

Next question

Comment on this question

All contents of this site are 2012 E-Medical Revision Ltd

http://www.emrcs.com/question/question.php?q=0

Terms and Conditions

Privacy policy

2/2

15/05/2015

Reference ranges

Question 47 of 517

Previous

Next

A 43 year old man is due to undergo an excision of the sub mandibular gland. Which of the
following incisions is the most appropriate for this procedure?

A. A transversely orientated incision 4cm below the mandible


B. A transversely orientated incision immediately inferior to the mandible

Question stats

Score: 79.6%
1

44.5%

22%

15.2%

11%

7.3%

44.5% of users answered this


question correctly

C. A vertical incision 3 cm anterior to the angle of the mandible and


extending inferiorly

End and review

6
7
8

D. A transversely orientated incision 2cm above the mandible

Search

E. A transversely orientated incision 12cm below the mandible

Go

10
11

Next question

12

Theme from 2009 Exam


To access the sub mandibular gland a transverse incision 4cm below the mandible should be
made. Incisions located higher than this may damage the marginal mandibular branch of the
facial nerve.

13
14
15
16

Anterior triangle of the neck

17
18

Boundaries

19-21 3 / 3
22

Anterior border of the Sternocleidomastoid


Lower border of mandible
Anterior midline

23
24
25

Sub triangles (divided by Digastric above and Omohyoid)

26

Muscular triangle: Neck strap muscles


Carotid triangle: Carotid sheath
Submandibular Triangle (digastric)

27
28
29
30

Contents of the anterior triangle


Digastric triangle

31

Submandibular gland
Submandibular nodes
Facial vessels
Hypoglossal nerve

32
33
34

Muscular triangle

Strap muscles
External jugular vein

35
36

Carotid triangle

Carotid sheath (Common carotid, vagus and internal jugular vein)


Ansa cervicalis

37
38
39

Nerve supply to digastric muscle

40

Anterior: Mylohyoid nerve


Posterior: Facial nerve

41
42
43
44
45
46
47

Image sourced from Wikipedia

Rate question:

Next question

Comment on this question

All contents of this site are 2012 E-Medical Revision Ltd

http://www.emrcs.com/question/question.php?q=0

Terms and Conditions

Privacy policy

1/2

15/05/2015

http://www.emrcs.com/question/question.php?q=0

2/2

15/05/2015

Reference ranges

Previous

Question 48 of 517

Next

A 5 year old boy presents with recurrent headaches. As part of his assessment he
undergoes an MRI scan of his brain. This demonstrates enlargement of the lateral and third
ventricles. Where is the most likely site of obstruction?

Question stats

Score: 80%
1

13.2%

9.9%

20.4%

49.8%

6.5%

A. Foramen of Luschka
49.8% of users answered this
question correctly

B. Foramen of Magendie

End and review

6
7

C. Foramen of Munro
8

D. Aqueduct of Sylvius
9

Search

E. None of the above

Go

10
11

Next question

12

Theme from September 2011 Exam


Theme from April 2012 Exam
Theme from September 2012 Exam
Theme from April 2013 Exam
Theme from September 2013 Exam
Theme from April 2014 exam
The CSF flows from the 3rd to the 4th ventricle via the Aqueduct of Sylvius.

13
14
15
16
17
18
19-21 3 / 3

Cerebrospinal fluid

22

The CSF fills the space between the arachnoid mater and pia mater (covering surface of the
brain). The total volume of CSF in the brain is approximately 150ml. Approximately 500 ml is
produced by the ependymal cells in the choroid plexus (70%), or blood vessels (30%). It is
reabsorbed via the arachnoid granulations which project into the venous sinuses.

23
24
25
26

Circulation
1. Lateral ventricles (via foramen Munro)
2. 3rd ventricle
3. Cerebral aqueduct (aqueduct Sylvius)
4. 4th ventricle (via foramina of Magendie and Luschka)
5. Subarachnoid space
6. Reabsorbed into venous system via arachnoid granulations in superior sagittal sinus

27
28
29
30
31
32

Composition
33

Glucose: 50-80mg/dl
Protein: 15-40 mg/dl
Red blood cells: Nil

34

White blood cells: 0-3 cells/ mm3

36

35

37
38

Rate question:

39

Next question

40
Comment on this question

41
42
43
44
45
46
47
48

All contents of this site are 2012 E-Medical Revision Ltd

http://www.emrcs.com/question/question.php?q=0

Terms and Conditions

Privacy policy

1/1

15/05/2015

Reference ranges

Question 49 of 517

Previous

Next

A 23 year old man presents with appendicitis. A decision is made to perform an


appendicectomy. The operation commences with a 5cm incision centered on McBurneys
point. Which of the following structures will be encountered first during the dissection?

Question stats

Score: 80.4%
1

64.1%

7.7%

10.1%

11.7%

6.4%

A. External oblique aponeurosis


64.1% of users answered this
question correctly

B. Internal oblique muscle

End and review

6
7

C. Transversalis fascia
8

D. Rectus sheath
9

Search

E. Peritoneum

Go

Next question

10
11
12

Theme from April 2014 exam


The external oblique will be encountered first in this location. The rectus sheath lies more
medially.
The external oblique muscle is the most superficial of the abdominal wall muscles. It
originates from the 5th to 12th ribs and passes inferomedially to insert into the linea alba,
pubic tubercle and anterior half of the iliac crest. It is innervated by the thoracoabdominal
nerves (T7-T11) and sub costal nerves.

13
14
15
16
17
18
19-21 3 / 3

Abdominal wall

22

The 2 main muscles of the abdominal wall are the rectus abdominis (anterior) and the
quadratus lumborum (posterior).
The remaining abdominal wall consists of 3 muscular layers. Each muscle passes from the
lateral aspect of the quadratus lumborum posteriorly to the lateral margin of the rectus
sheath anteriorly. Each layer is muscular posterolaterally and aponeurotic anteriorly.

23
24
25
26
27
28
29
30
31
32

Image sourced from Wikipedia

Muscles of abdominal wall

33
34

External
oblique

Lies most superficially


Originates from 5th to 12th ribs
Inserts into the anterior half of the outer aspect of the iliac crest, linea
alba and pubic tubercle
More medially and superiorly to the arcuate line, the aponeurotic layer
overlaps the rectus abdominis muscle
The lower border forms the inguinal ligament
The triangular expansion of the medial end of the inguinal ligament is the
lacunar ligament.

35
36
37
38
39
40
41

Internal
oblique

Arises from the thoracolumbar fascia, the anterior 2/3 of the iliac crest
and the lateral 2/3 of the inguinal ligament
The muscle sweeps upwards to insert into the cartilages of the lower 3
ribs
The lower fibres form an aponeurosis that runs from the tenth costal
cartilage to the body of the pubis
At its lowermost aspect it joins the fibres of the aponeurosis of
transversus abdominis to form the conjoint tendon.

42
43
44
45
46
47
48

Transversus
abdominis

Innermost muscle
Arises from the inner aspect of the costal cartilages of the lower 6 ribs ,
from the anterior 2/3 of the iliac crest and lateral 1/3 of the inguinal
ligament
Its fibres run horizontally around the abdominal wall ending in an
aponeurosis. The upper part runs posterior to the rectus abdominis. Lower
down the fibres run anteriorly only.
The rectus abdominis lies medially; running from the pubic crest and
symphysis to insert into the xiphoid process and 5th, 6th and 7th costal
cartilages. The muscles lies in a aponeurosis as described above.
Nerve supply: anterior primary rami of T7-12

http://www.emrcs.com/question/question.php?q=0

49

1/2

15/05/2015

Surgical notes
During abdominal surgery it is usually necessary to divide either the muscles or their
aponeuroses. During a midline laparotomy it is desirable to divide the aponeurosis. This will
leave the rectus sheath intact above the arcuate line and the muscles intact below it.
Straying off the midline will often lead to damage to the rectus muscles, particularly below the
arcuate line where they may often be in close proximity to each other.
Rate question:

Next question

Comment on this question

All contents of this site are 2012 E-Medical Revision Ltd

http://www.emrcs.com/question/question.php?q=0

Terms and Conditions

Privacy policy

2/2

15/05/2015

Reference ranges

Previous

Question 50 of 517

Next

A 23 year old man is undergoing an inguinal hernia repair. The surgeons mobilise the
spermatic cord and place it in a hernia ring. A small slender nerve is identified superior to the
cord. Which nerve is it most likely to be?

Question stats

Score: 80.7%
1

10.9%

8.9%

16.6%

56.7%

6.9%

A. Iliohypogastric nerve
56.7% of users answered this
question correctly

B. Pudendal nerve

End and review

6
7

C. Femoral branch of the genitofemoral nerve


8

D. Ilioinguinal nerve
9

Search

E. Obturator nerve

Go

10
11

Next question

12

Theme from April 2014 exam


The ilioinguinal nerve passes through the inguinal canal and is the nerve most commonly
identified during hernia surgery. The genitofemoral nerve splits into two branches, the genital
branch passes through the inguinal canal within the cord structures. The femoral branch of
the genitofemoral nerve enters the thigh posterior to the inguinal ligament, lateral to the
femoral artery. The iliohypogastric nerve pierces the external oblique aponeurosis above the
superficial inguinal ring.

13
14
15
16
17
18
19-21 3 / 3

Ilioinguinal nerve

22

Arises from the first lumbar ventral ramus with the iliohypogastric nerve. It passes
inferolaterally through the substance of psoas major and over the anterior surface of
quadratus lumborum. It pierces the internal oblique muscle and passes deep to the
aponeurosis of the external oblique muscle. It enters the inguinal canal and then passes
through the superficial inguinal ring to reach the skin.

23

Branches

27

24
25
26

28

To supply those muscles of the abdominal wall through which it passes.


Skin and fascia over the pubic symphysis, superomedial part of the femoral triangle,
surface of the scrotum, root and dorsum of penis or labum majus in females.

29
30
31
32

Rate question:

33

Next question

34
Comment on this question

35
36
37
38
39
40
41
42
43
44
45
46
47
48
49
50

All contents of this site are 2012 E-Medical Revision Ltd

http://www.emrcs.com/question/question.php?q=0

Terms and Conditions

Privacy policy

1/1

15/05/2015

Reference ranges

Previous

Question 51 of 517

Next

A 34 year old man undergoes excision of a sarcoma from the right buttock. During the
procedure the sciatic nerve is sacrificed. Which of the following will not occur as a result of
this process?

Question stats

Score: 79.3%
1

48.5%

15.2%

11.4%

14.6%

10.3%

A. Loss of extension at the knee joint


48.5% of users answered this
question correctly

B. Foot drop

End and review

6
7

C. Inability to extend extensor hallucis longus


8

D. Loss of sensation to the posterior aspect of the thigh


9

Search

E. Loss of sensation to the posterior aspect of the lower leg

Go

10
11

Next question

12

Extension of the knee joint is caused by the obturator and femoral nerves.

13
14

Sciatic nerve

15
16

The sciatic nerve is formed from the sacral plexus and is the largest nerve in the body. It is
the continuation of the main part of the plexus arising from ventral rami of L4 to S3. These
rami converge at the inferior border of piriformis to form the nerve itself. It passes through
the inferior part of the greater sciatic foramen and emerges beneath piriformis. Medially, lie
the inferior gluteal nerve and vessels and the pudendal nerve and vessels. It runs
inferolaterally under the cover of gluteus maximus midway between the greater trochanter
and ischial tuberosity. It receives its blood supply from the inferior gluteal artery. The nerve
provides cutaneous sensation to the skin of the foot and the leg. It also innervates the
posterior thigh muscles and the lower leg and foot muscles. The nerve splits into the tibial
and common peroneal nerves approximately half way down the posterior thigh. The tibial
nerve supplies the flexor muscles and the common peroneal nerve supplies the extensor
muscles and the abductor muscles.

17
18
19-21 3 / 3
22
23
24
25
26
27
28

Summary points
Origin

Spinal nerves L4 - S3

Articular Branches

Hip joint

29
30
31

Muscular branches in
upper leg

32

Semitendinosus
Semimembranosus
Biceps femoris
Part of adductor magnus

33
34
35
36

Cutaneous sensation

Posterior aspect of thigh


Gluteal region
Entire lower leg (except the medial aspect)

37
38
39

Terminates

At the upper part of the popliteal fossa by dividing into the tibial
and peroneal nerves

40
41
42

The nerve to the short head of the biceps femoris comes from the common peroneal
part of the sciatic and the other muscular branches arise from the tibial portion.
The tibial nerve goes on to innervate all muscles of the foot except the extensor
digitorum brevis (which is innervated by the common peroneal nerve).

43
44
45
46
47

Rate question:

48

Next question

49

Comment on this question

50
51

All contents of this site are 2012 E-Medical Revision Ltd

http://www.emrcs.com/question/question.php?q=0

Terms and Conditions

Privacy policy

1/1

15/05/2015

Reference ranges

Previous

Question 52 of 517

Next

Question stats

End and review

Score: 80%
1

14.8%

13.5%

38.4%

A. L1

18.7%

B. L2

14.5%

C. L3

38.4% of users answered this


question correctly

Where does the spinal cord terminate in neonates?

D. L4
E. L5

6
7
8
9

Search
Next question
Go

Theme from 2009 Exam


Theme from January 2013 Exam
At the 3rd month the foetus's spinal cord occupies the entire length of the vertebral canal.
The vertebral column then grows longer exceeding the growth rate of the spinal cord. This
results with the cord being at L3 at birth and L1-2 by adulthood.

10
11
12
13
14
15

Spinal cord

16
17

Located in a canal within the vertebral column that affords it structural support.
Rostrally it continues to the medulla oblongata of the brain and caudally it tapers at a
level corresponding to the L1-2 interspace (in the adult), a central structure, the filum
terminale anchors the cord to the first coccygeal vertebra.
The spinal cord is characterised by cervico-lumbar enlargements and these, broadly
speaking, are the sites which correspond to the brachial and lumbar plexuses
respectively.

18
19-21 3 / 3
22
23
24
25
26

There are some key points to note when considering the surgical anatomy of the spinal cord:

27

* During foetal growth the spinal cord becomes shorter than the spinal canal, hence the adult
site of cord termination at the L1-2 level.

28
29
30

* Due to growth of the vertebral column the spine segmental levels may not always
correspond to bony landmarks as they do in the cervical spine.

31
32

* The spinal cord is incompletely divided into two symmetrical halves by a dorsal median
sulcus and ventral median fissure. Grey matter surrounds a central canal that is
continuous rostrally with the ventricular system of the CNS.

33
34
35

* The grey matter is sub divided cytoarchitecturally into Rexeds laminae.

36

* Afferent fibres entering through the dorsal roots usually terminate near their point of entry
but may travel for varying distances in Lissauers tract. In this way they may establish
synaptic connections over several levels

37

* At the tip of the dorsal horn are afferents associated with nociceptive stimuli. The ventral
horn contains neurones that innervate skeletal muscle.

40

38
39

41
42

The key point to remember when revising CNS anatomy is to keep a clinical perspective in
mind. So it is worth classifying the ways in which the spinal cord may become injured. These
include:

43
44
45

Trauma either direct or as a result of disc protrusion


Neoplasia either by direct invasion (rare) or as a result of pathological vertebral
fracture
Inflammatory diseases such as Rheumatoid disease, or OA (formation of
osteophytes compressing nerve roots etc.
Vascular either as a result of stroke (rare in cord) or as complication of aortic
dissection
Infection historically diseases such as TB, epidural abscesses.

46
47
48
49
50
51
52

The anatomy of the cord will, to an extent dictate the clinical presentation. Some points/
conditions to remember:

Brown- Sequard syndrome-Hemisection of the cord producing ipsilateral loss of


proprioception and upper motor neurone signs, plus contralateral loss of pain and

http://www.emrcs.com/question/question.php?q=0

1/2

15/05/2015

temperature sensation. The explanation of this is that the fibres decussate at different
levels.
Lesions below L1 will tend to present with lower motor neurone signs

Rate question:

Next question

Comment on this question

All contents of this site are 2012 E-Medical Revision Ltd

http://www.emrcs.com/question/question.php?q=0

Terms and Conditions

Privacy policy

2/2

15/05/2015

Reference ranges

Question 1 of 465

Next

A 45 year old man is undergoing a low anterior resection for a carcinoma of the rectum.
Which of the following fascial structures will need to be divided to mobilise the mesorectum
from the sacrum and coccyx?

Question stats

End and review

Score: 100%

27.1%

11.9%

14.1%

38.7%

8.2%

A. Denonvilliers fascia
38.7% of users answered this
question correctly

B. Colles fascia
C. Sibsons fascia
D. Waldeyers fascia

Search

E. None of the above

Go

Next question

Fascial layers surrounding the rectum:


Anteriorly lies the fascia of Denonvilliers
Posteriorly lies Waldeyers fascia

Waldeyers fascia separates the mesorectum from the sacrum and will need to be divided.
Rectum
The rectum is approximately 12 cm long. It is a capacitance organ. It has both intra and
extraperitoneal components. The transition between the sigmoid colon is marked by the
disappearance of the tenia coli.The extra peritoneal rectum is surrounded by mesorectal fat
that also contains lymph nodes. This mesorectal fatty layer is removed surgically during
rectal cancer surgery (Total Mesorectal Excision). The fascial layers that surround the
rectum are important clinical landmarks, anteriorly lies the fascia of Denonvilliers. Posteriorly
lies Waldeyers fascia.
Extra peritoneal rectum
Posterior upper third
Posterior and lateral middle third
Whole lower third

Relations
Anteriorly (Males)

Rectovesical pouch
Bladder
Prostate
Seminal vesicles

Anteriorly (Females)

Recto-uterine pouch (Douglas)


Cervix
Vaginal wall

Posteriorly

Sacrum
Coccyx
Middle sacral artery

Laterally

Levator ani
Coccygeus

Arterial supply
Superior rectal artery
Venous drainage
Superior rectal vein
Lymphatic drainage
Mesorectal lymph nodes (superior to dentate line)
Internal iliac and then para-aortic nodes
Inguinal nodes (inferior to dentate line)

Rate question:

http://www.emrcs.com/question/question.php?q=0

Next question

1/2

15/05/2015
Comment on this question

All contents of this site are 2012 E-Medical Revision Ltd

http://www.emrcs.com/question/question.php?q=0

Terms and Conditions

Privacy policy

2/2

15/05/2015

Reference ranges

Previous

Question 2 of 465

Next

A 10 year old child has a grommet inserted for a glue ear. What type of epithelium is present
on the external aspect of the tympanic membrane?

A. Stratified squamous
B. Ciliated columnar

Question stats

End and review

Score: 100%

46.2%

16.2%

10.8%

20.2%

6.6%

1
2

46.2% of users answered this


question correctly

C. Non ciliated columnar


D. Non stratified squamous
E. None of the above

Search
Next question

Go

The external aspect of the tympanic membrane is lined by stratified squamous epithelium.
This is significant clinically in the development of middle ear infections when this type of
epithelium may migrate inside the middle ear.
Ear- anatomy
The ear is composed of three anatomically distinct regions.
External ear
Auricle is composed of elastic cartilage covered by skin. The lobule has no cartilage and
contains fat and fibrous tissue.
External auditory meatus is approximately 2.5cm long.
Lateral third of the external auditory meatus is cartilaginous and the medial two thirds is
bony.
The region is innervated by the greater auricular nerve. The auriculotemporal branch of the
trigeminal nerve supplies most the of external auditory meatus and the lateral surface of the
auricle.

Middle ear
Space between the tympanic membrane and cochlea. The aditus leads to the mastoid air
cells is the route through which middle ear infections may cause mastoiditis. Anteriorly the
eustacian tube connects the middle ear to the naso pharynx.
The tympanic membrane consists of:
Outer layer of stratified squamous epithelium.
Middle layer of fibrous tissue.
Inner layer of mucous membrane continuous with the middle ear.
The tympanic membrane is approximately 1cm in diameter.
The chorda tympani nerve passes on the medial side of the pars flaccida.
The middle ear is innervated by the glossopharyngeal nerve and pain may radiate to the
middle ear following tonsillectomy.
Ossicles
Malleus attaches to the tympanic membrane (the Umbo).
Malleus articulates with the incus (synovial joint).
Incus attaches to stapes (another synovial joint).
Internal ear
Cochlea, semi circular canals and vestibule
Organ of corti is the sense organ of hearing and is located on the inside of the cochlear duct
on the basilar membrane.
Vestibule accommodates the utricule and the saccule. These structures contain endolymph
and are surrounded by perilymph within the vestibule.
The semicircular canals lie at various angles to the petrous temporal bone. All share a
common opening into the vestibule.
Rate question:

Next question

Comment on this question

http://www.emrcs.com/question/question.php?q=0

1/2

15/05/2015

All contents of this site are 2012 E-Medical Revision Ltd

http://www.emrcs.com/question/question.php?q=0

Terms and Conditions

Privacy policy

2/2

15/05/2015

Reference ranges

Question 3 of 465

Previous

Next

A 73 year old lady is admitted with acute mesenteric ischaemia. A CT angiogram is


performed and a stenotic lesion is noted at the origin of the superior mesenteric artery. At
which of the following levels does this branch from the aorta?

Question stats

End and review

Score: 100%
1

56.1%

18.1%

12.3%

7.8%

5.7%

A. L1
56.1% of users answered this
question correctly

B. L2
C. L3
D. L4

Search

E. L5

Go

Next question

Theme from January 2012 Exam


Theme from April 2014 exam
The SMA leaves the aorta at L1. It passes under the neck of the pancreas prior to giving its
first branch the inferior pancreatico-duodenal artery.
Superior mesenteric artery

Branches off aorta at L1


Supplies small bowel from duodenum (distal to ampulla of vater) through to mid
transverse colon
Takes more oblique angle from aorta and thus more likely to recieve emboli than
coeliac axis

Relations of superior mesenteric artery


Superiorly

Neck of pancreas

Postero-inferiorly Third part of duodenum


Uncinate process
Posteriorly

Left renal vein

Right

Superior mesenteric vein

Branches of the superior mesenteric artery


Inferior pancreatico-duodenal artery
Jejunal and ileal arcades
Ileo-colic artery
Right colic artery
Middle colic artery

Overview of SMA and branches

Image sourced from Wikipedia

Rate question:

Next question

Comment on this question

All contents of this site are 2012 E-Medical Revision Ltd

http://www.emrcs.com/question/question.php?q=0

Terms and Conditions

Privacy policy

1/1

15/05/2015

Reference ranges

Previous

Question 4 of 465

Next

Question stats

End and review

Score: 100%
1

10.8%

10.9%

55.1%

A. It arises from the lateral cord of the brachial plexus

13%

B. It provides cutaneous innervation to the lateral side of the forearm

10.2%

C. If damaged, then extension of the elbow joint will be impaired

55.1% of users answered this


question correctly

The following statements relating to the musculocutaneous nerve are true except?

D. It supplies the biceps muscle


E. It runs beneath biceps

Search
Next question
Go

It supplies biceps, brachialis and coracobrachialis. If damaged then elbow flexion will be
impaired.
Musculocutaneous nerve

Branch of lateral cord of brachial plexus

Path
It penetrates the Coracobrachialis muscle
Passes obliquely between the Biceps brachii and the Brachialis to the lateral side of
the arm
Above the elbow it pierces the deep fascia lateral to the tendon of the Biceps brachii
Continues into the forearm as the lateral cutaneous nerve of the forearm

Innervates
Coracobrachialis
Biceps brachii
Brachialis

Rate question:

Next question

Comment on this question

All contents of this site are 2012 E-Medical Revision Ltd

http://www.emrcs.com/question/question.php?q=0

Terms and Conditions

Privacy policy

1/1

15/05/2015

Reference ranges

Previous

Question 5 of 465

Next

Question stats

End and review

Score: 71.4%
1

12.5%

13.9%

40.2%

A. Lesser petrosal nerve

12.2%

B. Accessory meningeal artery

21.4%

C. Maxillary nerve

40.2% of users answered this


question correctly

Which of the following structures does not pass through the foramen ovale?

D. Emissary veins
E. Otic ganglion

Search
Next question
Go

Mnemonic: OVALE
O tic ganglion
V3 (Mandibular nerve:3rd branch of trigeminal)
A ccessory meningeal artery
L esser petrosal nerve
E missary veins

Foramina of the base of the skull

Foramen

Location

Contents

Foramen
ovale

Sphenoid
bone

Otic ganglion
V3 (Mandibular nerve:3rd branch of
trigeminal)
Accessory meningeal artery
Lesser petrosal nerve
Emissary veins

Foramen
spinosum

Sphenoid
bone

Middle meningeal artery


Meningeal branch of the Mandibular nerve

Foramen
rotundum

Sphenoid
bone

Maxillary nerve (V2)

Foramen
lacerum/
carotid canal

Sphenoid
bone

Base of the medial pterygoid plate.


Internal carotid artery*
Nerve and artery of the pterygoid canal

Jugular
foramen

Temporal
bone

Anterior: inferior petrosal sinus


Intermediate: glossopharyngeal, vagus, and accessory nerves.
Posterior: sigmoid sinus (becoming the internal jugular vein) and
some meningeal branches from the occipital and ascending
pharyngeal arteries.

Foramen
magnum

Occipital
bone

Anterior and posterior spinal arteries


Vertebral arteries
Medulla oblongata

Stylomastoid
foramen

Temporal
bone

Stylomastoid artery
Facial nerve

Superior
orbital fissure

Sphenoid
bone

Oculomotor nerve (III)


trochlear nerve (IV)
lacrimal, frontal and nasociliary branches of ophthalmic nerve (V1)
abducent nerve (VI)
Superior and inferior ophthalmic vein

*= In life the foramen lacerum is occluded by a cartilagenous plug. The ICA initially passes
into the carotid canal which ascends superomedially to enter the cranial cavity through the
foramen lacerum.
Base of skull anatomical overview

http://www.emrcs.com/question/question.php?q=0

1/2

15/05/2015

Image sourced from Wikipedia

Rate question:

Next question

Comment on this question

All contents of this site are 2012 E-Medical Revision Ltd

http://www.emrcs.com/question/question.php?q=0

Terms and Conditions

Privacy policy

2/2

15/05/2015

Reference ranges

Previous

Question 6 of 465

Next

Question stats

End and review

Score: 75%
1

16.7%

13.3%

13%

A. III

12.6%

B. VII

44.4%

C. IX

44.4% of users answered this


question correctly

Which of the cranial nerves listed below is least likely to carry parasympathetic fibres?

D. X

E. II
Search
Next question
Go

The parasympathetic functions served by the cranial nerves include:


III (oculomotor)

Pupillary constriction and accommodation

VII (facial)

Lacrimal gland, submandibular and sublingual glands

IX (glossopharyngeal) Parotid
X (vagus)

Heart and abdominal viscera

The optic nerve carries no parasympathetic fibres.


The cranial preganglionic parasympathetic nerves arise from specific nuclei in the CNS.
These synapse at one of four parasympathetic ganglia; otic, pterygopalatine, ciliary and
submandibular. From these ganglia the parasympathetic nerves complete their journey to
their target tissues via CN V (trigeminal) branches (ophthalmic nerve CNV branch 1, Maxillary
nerve CN V branch2, mandibular nerve CN V branch 3)
Cranial nerves
Cranial nerve lesions
Olfactory nerve

May be injured in basal skull fractures or involved in frontal lobe tumour


extension. Loss of olfactory nerve function in relation to major CNS
pathology is seldom an isolated event and thus it is poor localiser of CNS
pathology.

Optic nerve

Problems with visual acuity may result from intra ocular disorders.
Problems with the blood supply such as amaurosis fugax may produce
temporary visual distortion. More important surgically is the pupillary
response to light. The pupillary size may be altered in a number of
disorders. Nerves involved in the resizing of the pupil connect to the
pretectal nucleus of the high midbrain, bypassing the lateral geniculate
nucleus and the primary visual cortex. From the pretectal nucleus
neurones pass to the Edinger - Westphal nucleus, motor axons from here
pass along with the oculomotor nerve. They synapse with ciliary ganglion
neurones; the parasympathetic axons from this then innervate the iris and
produce miosis. The miotic pupil is seen in disorders such as Horner's
syndrome or opiate overdose.
Mydriasis is the dilatation of the pupil in response to disease, trauma,
drugs (or the dark!). It is pathological when light fails to induce miosis. The
radial muscle is innervated by the sympathetic nervous system. Because
the parasympathetic fibres travel with the oculomotor nerve they will be
damaged by lesions affecting this nerve (e.g. cranial trauma).
The response to light shone in one eye is usually a constriction of both
pupils. This indicates intact direct and consensual light reflexes. When
the optic nerve has an afferent defect the light shining on the affected eye
will produce a diminished pupillary response in both eyes. Whereas light
shone on the unaffected eye will produce a normal pupillary response in
both eyes. This is referred to as the Marcus Gunn pupil and is seen in
conditions such as optic neuritis. In a total CN II lesion shining the light in
the affected eye will produce no response.

Oculomotor nerve

The pupillary effects are described above. In addition it supplies all ocular
muscles apart from lateral rectus and superior oblique. Thus the affected
eye will be deviated inferolaterally. Levator palpebrae superioris may also
be impaired resulting in impaired ability to open the eye.

Trochlear nerve

The eye will not be able to look down.

Trigeminal nerve

Largest cranial nerve. Exits the brainstem at the pons. Branches are
ophthalmic, maxillary and mandibular. Only the mandibular branch has
both sensory and motor fibres. Branches converge to form the trigeminal
ganglion (located in Meckels cave). It supplies the muscles of mastication
and also tensor veli palatine, mylohyoid, anterior belly of digastric and
tensor tympani. The detailed descriptions of the various sensory functions
are described in other areas of the website. The corneal reflex is important
and is elicited by applying a small tip of cotton wool to the cornea, a reflex
blink should occur if it is intact. It is mediated by: the naso ciliary branch

http://www.emrcs.com/question/question.php?q=0

1/2

15/05/2015

of the ophthalmic branch of the trigeminal (sensory component) and the


facial nerve producing the motor response. Lesions of the afferent arc will
produce bilateral absent blink and lesions of the efferent arc will result in a
unilateral absent blink.
Abducens nerve

The affected eye will have a deficit of abduction. This cranial nerve exits
the brainstem between the pons and medulla. It thus has a relatively long
intra cranial course which renders it susceptible to damage in raised intra
cranial pressure.

Facial nerve

Emerges from brainstem between pons and medulla. It controls muscles


of facial expression and taste from the anterior 2/3 of the tongue. The
nerve passes into the petrous temporal bone and into the internal auditory
meatus. It then passes through the facial canal and exits at the
stylomastoid foramen. It passes through the parotid gland and divides at
this point. It does not innervate the parotid gland. Its divisions are
considered in other parts of the website. Its motor fibres innervate
orbicularis oculi to produce the efferent arm of the corneal reflex. In
surgical practice it may be injured during parotid gland surgery or invaded
by malignancies of the gland and a lower motor neurone on the ipsilateral
side will result.

Vestibulocochlear nerve

Exits from the pons and then passes through the internal auditory
meatus. It is implicated in sensorineural hearing loss. Individuals with
sensorineural hearing loss will localise the sound in webers test to the
normal ear. Rinnes test will be reduced on the affected side but should
still work. These two tests will distinguish sensorineural hearing loss from
conductive deafness. In the latter condition webers test will localise to the
affected ear and Rinnes test will be impaired on the affected side. Surgical
lesions affecting this nerve include CNS tumours and basal skull fractures.
It may also be damaged by the administration of ototoxic drugs (of which
gentamicin is the most commonly used in surgical practice).

Glossopharyngeal
nerve

Exits the pons just above the vagus. Receives sensory fibres from
posterior 1/3 tongue, tonsils, pharynx and middle ear (otalgia may occur
following tonsillectomy). It receives visceral afferents from the carotid
bodies. It supplies parasympathetic fibres to the parotid gland via the otic
ganglion and motor function to stylopharyngeaus muscle. The sensory
function of the nerve is tested using the gag reflex.

Vagus nerve

Leaves the medulla between the olivary nucleus and the inferior cerebellar
peduncle. Passes through the jugular foramen and into the carotid sheath.
Details of the functions of the vagus nerve are covered in the website
under relevant organ sub headings.

Accessory nerve

Exists from the caudal aspect of the brainstem (multiple branches)


supplies trapezius and sternocleidomastoid muscles. The distal portion of
this nerve is most prone to injury during surgical procedures.

Hypoglossal
nerve

Emerges from the medulla at the preolivary sulcus, passes through the
hypoglossal canal. It lies on the carotid sheath and passes deep to the
posterior belly of digastric to supply muscles of the tongue (except
palatoglossus). Its location near the carotid sheath makes it vulnerable
during carotid endarterectomy surgery and damage will produce ipsilateral
defect in muscle function.

Rate question:

Next question

Comment on this question

All contents of this site are 2012 E-Medical Revision Ltd

http://www.emrcs.com/question/question.php?q=0

Terms and Conditions

Privacy policy

2/2

15/05/2015

Reference ranges

Question 7 of 465

Previous

Next

A 72 year old man is undergoing an open abdominal aortic aneurysm repair. The aneurysm
is located in a juxtarenal location and surgical access to the neck of aneurysm is difficult.
Which of the following structures may be divided to improve access?

Question stats

Score: 77.8%
1

17.3%

14.4%

44.7%

14.1%

9.5%

A. Cisterna chyli
44.7% of users answered this
question correctly

B. Transverse colon

End and review

6
7

C. Left renal vein


D. Superior mesenteric artery
Search

E. Coeliac axis

Go

Next question

The left renal vein will be stretched over the neck of the anuerysm in this location and is not
infrequently divided. This adds to the nephrotoxic insult of juxtarenal aortic surgery as a
supra renal clamp is also often applied. Deliberate division of the Cisterna Chyli will not
improve access and will result in a chyle leak. Division of the transverse colon will not help at
all and would result in a high risk of graft infection. Division of the SMA is pointless for a
juxtarenal procedure.
Abdominal aorta
Abdominal aortic topography
Origin

T12

Termination

L4

Posterior relations

L1-L4 Vertebral bodies

Anterior relations

Lesser omentum
Liver
Left renal vein
Inferior mesenteric vein
Third part of duodenum
Pancreas
Parietal peritoneum
Peritoneal cavity

Right lateral relations

Right crus of the diaphragm


Cisterna chyli
Azygos vein
IVC (becomes posterior distally)

Left lateral relations

4th part of duodenum


Duodenal-jejunal flexure
Left sympathetic trunk

The abdominal aorta

Image sourced from Wikipedia

Rate question:
http://www.emrcs.com/question/question.php?q=0

Next question

1/2

15/05/2015

Rate question:

Next question

Comment on this question

All contents of this site are 2012 E-Medical Revision Ltd

http://www.emrcs.com/question/question.php?q=0

Terms and Conditions

Privacy policy

2/2

15/05/2015

Reference ranges

Question 8 of 465

Previous

Next

An occlusion of the anterior cerebral artery may compromise the blood supply to the
following structures except:

A. Medial inferior surface of the frontal lobe


B. Corpus callosum

Question stats

Score: 70%
1

12.5%

18.4%

13.2%

12.4%

43.5%

43.5% of users answered this


question correctly

C. Medial surface of the frontal lobe

End and review

D. Olfactory bulb

6
7
8

E. Brocas area

Search
Next question

Go

Brocas area is usually supplied by branches from the middle cerebral artery.
Circle of Willis
The two internal carotid arteries and two vertebral arteries form an anastomosis known as
the Circle of Willis on the inferior surface of the brain. Each half of the circle is formed by:
1. Anterior communicating artery
2. Anterior cerebral artery
3. Internal carotid artery
4. Posterior communicating artery
5. Posterior cerebral arteries and the termination of the basilar artery
The circle and its branches supply; the corpus striatum, internal capsule, diencephalon and
midbrain.

Image sourced from Wikipedia

Vertebral arteries
Enter the cranial cavity via foramen magnum
Lie in the subarachnoid space
Ascend on anterior surface of medulla oblongata
Unite to form the basilar artery at the base of the pons

Branches:
Posterior spinal artery
Anterior spinal artery
Posterior inferior cerebellar artery

Basilar artery
Branches:
Anterior inferior cerebellar artery
http://www.emrcs.com/question/question.php?q=0

1/2

15/05/2015

Labyrinthine artery
Pontine arteries
Superior cerebellar artery
Posterior cerebral artery

Internal carotid arteries


Branches:
Posterior communicating artery
Anterior cerebral artery
Middle cerebral artery
Anterior choroid artery

Rate question:

Next question

Comment on this question

All contents of this site are 2012 E-Medical Revision Ltd

http://www.emrcs.com/question/question.php?q=0

Terms and Conditions

Privacy policy

2/2

15/05/2015

Reference ranges

Previous

Question 9 of 465

Next

Question stats

End and review

Score: 63.6%
1

12.8%

36%

10.6%

A. Submandibular ganglion

33.4%

B. Otic ganglion

7.2%

C. Ciliary ganglion

36% of users answered this


question correctly

Parasympathetic fibres innervating the parotid gland originate from which of the following?

D. Pterygopalatine ganglion
E. None of the above

6
7
8
9

Search
Next question
Go

Theme from April 2014


Secretion of saliva by the parotid gland is controlled by postsynaptic parasympathetic fibres
originating in the inferior salivatory nucleus; these leave the brain via the tympanic nerve
(branch of glossopharyngeal nerve (CN IX), travel through the tympanic plexus (located in
the middle ear), and then form the lesser petrosal nerve until reaching the otic ganglion.
After synapsing in the Otic ganglion, the postganglionic (postsynaptic) fibres travel as part of
the auriculotemporal nerve (a branch of the mandibular nerve (V3) to reach the parotid
gland.
Parotid gland
Anatomy of the parotid gland
Location

Overlying the mandibular ramus; anterior and inferior to the ear.

Salivary duct

Crosses the masseter, pierces the buccinator and drains adjacent to the
2nd upper molar tooth (Stensen's duct).

Structures passing
through the gland

Facial nerve (Mnemonic: The Zebra Buggered My Cat; Temporal


Zygomatic, Buccal, Mandibular, Cervical)
External carotid artery
Retromandibular vein
Auriculotemporal nerve

Relations

Anterior: masseter, medial pterygoid, superficial temporal and


maxillary artery, facial nerve, stylomandibular ligament
Posterior: posterior belly digastric muscle, sternocleidomastoid,
stylohyoid, internal carotid artery, mastoid process, styloid
process

Arterial supply

Branches of external carotid artery

Venous drainage

Retromandibular vein

Lymphatic
drainage

Deep cervical nodes

Nerve innervation

Parasympathetic-Secretomotor
Sympathetic-Superior cervical ganglion
Sensory- Greater auricular nerve

Parasympathetic stimulation produces a water rich, serous saliva. Sympathetic stimulation


leads to the production of a low volume, enzyme-rich saliva.
Rate question:

Next question

Comment on this question

All contents of this site are 2012 E-Medical Revision Ltd

http://www.emrcs.com/question/question.php?q=0

Terms and Conditions

Privacy policy

1/1

15/05/2015

Reference ranges

Question 10 of 465

Previous

Next

Following an oesophagogastrectomy the surgeons will anastomose the oesophageal


remnant to the stomach, which of the following is not part of the layers that comprise the
oesophageal wall?

Question stats

Score: 69.2%
1

43.6%

29.8%

10.6%

8.8%

7.2%

A. Serosa
43.6% of users answered this
question correctly

B. Adventitia

End and review

6
7

C. Muscularis propria
8

D. Submucosa
9

Search

E. Mucosa

Go

10

Next question

The oesophageal wall lacks the serosa layer

Theme from April 2010 exam


The wall lacks a serosa which can make the wall hold sutures less securely.
Oesophagus

25cm long
Starts at C6 vertebra, pierces diaphragm at T10 and ends at T11
Squamous epithelium

Constrictions of the oesophagus


Structure

Distance from incisors

Cricoid cartilage

15cm

Arch of the Aorta

22.5cm

Left principal bronchus

27cm

Diaphragmatic hiatus

40cm

Relations
Anteriorly

Trachea to T4
Recurrent laryngeal nerve
Left bronchus, Left atrium
Diaphragm

Posteriorly

Thoracic duct to left at T5


Hemiazygos to the left T8
Descending aorta
First 2 intercostal branches of aorta

Left

Thoracic duct
Left subclavian artery

Right

Azygos vein

Arterial, venous and lymphatic drainage of the oesophagus


Artery

Vein

Lymphatics

Muscularis
externa

Upper
third

Inferior
thyroid

Inferior thyroid

Deep
cervical

Striated muscle

Mid third

Aortic
branches

Azygos branches

Mediastinal

Smooth & striated


muscle

Lower
third

Left gastric

Posterior mediastinal and


coeliac

Gastric

Smooth muscle

http://www.emrcs.com/question/question.php?q=0

1/2

15/05/2015

Nerve supply
Upper half is supplied by recurrent laryngeal nerve
Lower half by oesophageal plexus (vagus)

Histology
Mucosa :Non-keratinized stratified squamous epithelium
Submucosa: glandular tissue
Muscularis externa (muscularis): composition varies. See table
Adventitia

Rate question:

Next question

Comment on this question

All contents of this site are 2012 E-Medical Revision Ltd

http://www.emrcs.com/question/question.php?q=0

Terms and Conditions

Privacy policy

2/2

15/05/2015

Reference ranges

Previous

Question 11 of 465

Next

Question stats

End and review

Score: 71.4%
1

18%

13%

22.3%

A. Filum terminale

37.4%

B. Conus medullaris

9.4%

C. Ligamentum flavum

37.4% of users answered this


question correctly

Which of the following structures suspends the spinal cord in the dural sheath?

D. Denticulate ligaments
E. Anterior longitudinal ligament

6
7
8
9

Search
Next question
Go

The spinal cord is approximately 45cm in men and 43cm in women. The denticulate ligament
is a continuation of the pia mater (innermost covering of the spinal cord) which has
intermittent lateral projections attaching the spinal cord to the dura mater.

10
11

Spinal cord

Located in a canal within the vertebral column that affords it structural support.
Rostrally it continues to the medulla oblongata of the brain and caudally it tapers at a
level corresponding to the L1-2 interspace (in the adult), a central structure, the filum
terminale anchors the cord to the first coccygeal vertebra.
The spinal cord is characterised by cervico-lumbar enlargements and these, broadly
speaking, are the sites which correspond to the brachial and lumbar plexuses
respectively.

There are some key points to note when considering the surgical anatomy of the spinal cord:
* During foetal growth the spinal cord becomes shorter than the spinal canal, hence the adult
site of cord termination at the L1-2 level.
* Due to growth of the vertebral column the spine segmental levels may not always
correspond to bony landmarks as they do in the cervical spine.
* The spinal cord is incompletely divided into two symmetrical halves by a dorsal median
sulcus and ventral median fissure. Grey matter surrounds a central canal that is
continuous rostrally with the ventricular system of the CNS.
* The grey matter is sub divided cytoarchitecturally into Rexeds laminae.
* Afferent fibres entering through the dorsal roots usually terminate near their point of entry
but may travel for varying distances in Lissauers tract. In this way they may establish
synaptic connections over several levels
* At the tip of the dorsal horn are afferents associated with nociceptive stimuli. The ventral
horn contains neurones that innervate skeletal muscle.
The key point to remember when revising CNS anatomy is to keep a clinical perspective in
mind. So it is worth classifying the ways in which the spinal cord may become injured. These
include:

Trauma either direct or as a result of disc protrusion


Neoplasia either by direct invasion (rare) or as a result of pathological vertebral
fracture
Inflammatory diseases such as Rheumatoid disease, or OA (formation of
osteophytes compressing nerve roots etc.
Vascular either as a result of stroke (rare in cord) or as complication of aortic
dissection
Infection historically diseases such as TB, epidural abscesses.

The anatomy of the cord will, to an extent dictate the clinical presentation. Some points/
conditions to remember:

Brown- Sequard syndrome-Hemisection of the cord producing ipsilateral loss of


proprioception and upper motor neurone signs, plus contralateral loss of pain and
temperature sensation. The explanation of this is that the fibres decussate at different
levels.

http://www.emrcs.com/question/question.php?q=0

1/2

15/05/2015

Lesions below L1 will tend to present with lower motor neurone signs

Rate question:

Next question

Comment on this question

All contents of this site are 2012 E-Medical Revision Ltd

http://www.emrcs.com/question/question.php?q=0

Terms and Conditions

Privacy policy

2/2

15/05/2015

Reference ranges

Previous

Question 12 of 465

Next

Question stats

End and review

Score: 75%
1

21.1%

57.3%

7.1%

A. 4th intercostal space, mid axillary line

7.5%

B. 5th intercostal space, mid axillary line

7%

C. 4th intercostal space, mid scapular line

57.3% of users answered this


question correctly

Where is the 'safe triangle' for chest drain insertion located?

D. 5th intercostal space, mid scapular line


E. 4th intercostal space, mid clavicular line

7
8
9

Search
Next question
Go

10
11

'Safe Triangle' for chest drain insertion:

12

5th intercostal space, mid axillary line

Theme from April 2012 exam


Chest drains
There are a number of different indications for chest drain insertion. In general terms large
bore chest drains are preferred for trauma and haemothorax drainage. Smaller diameter
chest drains can be used for pneumothorax or pleural effusion drainage.
Insertion can be performed either using anatomical guidance or through ultrasound
guidance. In the exam, the anatomical method is usually tested.
It is advised that chest drains are placed in the 'safe triangle'. The triangle is located in the
mid axillary line of the 5th intercostal space. It is bordered by:
Anterior edge latissimus dorsi, the lateral border of pectoralis major, a line superior to the
horizontal level of the nipple, and the apex below the axilla.
Another triangle is situated behind the scapula. It is bounded above by the trapezius, below
by the latissimus dorsi, and laterally by the vertebral border of the scapula; the floor is partly
formed by the rhomboid major. If the scapula is drawn forward by folding the arms across the
chest, and the trunk bent forward, parts of the sixth and seventh ribs and the interspace
between them become subcutaneous and available for auscultation. The space is therefore
known as the triangle of auscultation.
References
Prof Harold Ellis. The applied anatomy of chest drains insertions. British Journal of hospital
medicine 2007; (68): 44-45.
Laws D, Neville E, Duffy J. BTS guidelines for insertion of chest drains. Thorax, 2003; (58):
53-59.
Rate question:

Next question

Comment on this question

All contents of this site are 2012 E-Medical Revision Ltd

http://www.emrcs.com/question/question.php?q=0

Terms and Conditions

Privacy policy

1/1

15/05/2015

Reference ranges

Question 13 of 465

Previous

Next

Your consultant decides to perform an open inguinal hernia repair under local anaesthesia.
Which of the following dermatomal levels will require blockade?

A. T10
B. T12

Question stats

Score: 76.5%
1

15.7%

53.5%

11.3%

12.3%

7.2%

53.5% of users answered this


question correctly

C. T11

End and review

D. S1

6
7
8

E. S2

Search
Go

Next question

10
11
12

Theme from April 2012 Exam

13

Dermatomes
The common dermatomal levels and cutaneous nerves responsible for them is illustrated
below.

Image sourced from Wikipedia

Rate question:

Next question

Comment on this question

All contents of this site are 2012 E-Medical Revision Ltd

http://www.emrcs.com/question/question.php?q=0

Terms and Conditions

Privacy policy

1/1

15/05/2015

Reference ranges

Previous

Question 14 of 465

Next

A 44 year old man is undergoing a parotidectomy and the surgeon is carefully preserving the
facial nerve. Unfortunately his trainee then proceeds to divide it. Which of the following will
not be affected as a result?

Question stats

Score: 68.4%
1

37.6%

13.5%

9.7%

28%

11.2%

A. Taste sensation from anterior two thirds of the tongue


37.6% of users answered this
question correctly

B. Closing the ipsilateral eyelid

End and review

6
7

C. Raising the ipsilateral side of the lip


8

D. Ipsilateral corneal reflex


9

Search

E. None of the above

Go

Next question

10
11
12

Theme from April 2014 exam


The chorda tympani branches inside the facial canal and will therefore be unaffected by this
most unfortunate event! The corneal reflex is mediated by the opthalmic branch of the
trigeminal nerve sensing the stimulus on the cornea, lid or conjunctiva; the facial nerve
initiates the motor response of the reflex.

13
14

Facial nerve
The facial nerve is the main nerve supplying the structures of the second embryonic
branchial arch. It is predominantly an efferent nerve to the muscles of facial expression,
digastric muscle and also to many glandular structures. It contains a few afferent fibres which
originate in the cells of its genicular ganglion and are concerned with taste.
Supply - 'face, ear, taste, tear'
Face: muscles of facial expression
Ear: nerve to stapedius
Taste: supplies anterior two-thirds of tongue
Tear: parasympathetic fibres to lacrimal glands, also salivary glands

Path
Subarachnoid path
Origin: motor- pons, sensory- nervus intermedius
Pass through the petrous temporal bone into the internal auditory meatus with the
vestibulocochlear nerve. Here they combine to become the facial nerve.

Facial canal path


The canal passes superior to the vestibule of the inner ear
At the medial aspect of the middle ear, it becomes wider and contains the geniculate
ganglion.
- 3 branches:
1. greater petrosal nerve
2. nerve to stapedius
3. chorda tympani
Stylomastoid foramen
Passes through the stylomastoid foramen (tympanic cavity anterior and mastoid
antrum posteriorly)
Posterior auricular nerve and branch to posterior belly of digastric and stylohyoid
muscle

Face
Enters parotid gland and divides into 5 branches:
Temporal branch
Zygomatic branch
Buccal branch
Marginal mandibular branch
Cervical branch

http://www.emrcs.com/question/question.php?q=0

1/2

15/05/2015

Rate question:

Next question

Comment on this question

All contents of this site are 2012 E-Medical Revision Ltd

http://www.emrcs.com/question/question.php?q=0

Terms and Conditions

Privacy policy

2/2

15/05/2015

Reference ranges

Question 15 of 465

Previous

Next

A 45 year old lady develops severe back pain and on examination is found to have clinical
evidence of an L5/ S1 radiculopathy. Her symptoms deteriorate and eventually a
laminectomy is performed. During a posterior surgical approach the surgeons encounter a
tough ligamentous structure lying anterior to the spinous processes. This structure is most
likely to be the

Question stats

Score: 70%
1

9.4%

18.9%

13.3%

41.8%

16.6%

41.8% of users answered this


question correctly

A. Transverse spinal ligament

End and review

6
7
8

B. Supraspinal ligament
C. Anterior longitudinal ligament

Search

D. Ligamentum flavum

Go

E. Posterior longitudinal ligament

10
11
12

Next question

The ligamentum lies in this position, as illustrated below:

13
14
15

Image sourced from Wikipedia

Vertebral column

There are 7 cervical, 12 thoracic, 5 lumbar, and 5 sacral vertebrae.


The spinal cord segmental levels do not necessarily correspond to the vertebral
segments. For example, while the C1 cord is located at the C1 vertebra, the C8 cord is
situated at the C7 vertebra. While the T1 cord is situated at the T1 vertebra, the T12
cord is situated at the T8 vertebra. The lumbar cord is situated between T9 and T11
vertebrae. The sacral cord is situated between the T12 to L2 vertebrae.

Cervical vertebrae
The interface between the first and second vertebra is called the atlanto-axis junction. The
C3 cord contains the phrenic nucleus.
Muscle

Nerve root value

Deltoid

C5,6

Biceps

C5,6

Wrist extensors

C6-8

Triceps

C6-8

Wrist flexors

C6-T1

Hand muscles

C8-T1

Thoracic vertebrae
The thoracic vertebral segments are defined by those that have a rib. The spinal roots form
the intercostal nerves that run on the bottom side of the ribs and these nerves control the
intercostal muscles and associated dermatomes.
Lumbosacral vertebrae
Form the remainder of the segments below the vertebrae of the thorax. The lumbosacral

http://www.emrcs.com/question/question.php?q=0

1/2

15/05/2015

spinal cord, however, starts at about T9 and continues only to L2. It contains most of the
segments that innervate the hip and legs, as well as the buttocks and anal regions.
Cauda Equina
The spinal cord ends at L1-L2 vertebral level. The tip of the spinal cord is called the conus.
Below the conus, there is a spray of spinal roots that is called the cauda equina. Injuries
below L2 represent injuries to spinal roots rather than the spinal cord proper.
Rate question:

Next question

Comment on this question

All contents of this site are 2012 E-Medical Revision Ltd

http://www.emrcs.com/question/question.php?q=0

Terms and Conditions

Privacy policy

2/2

15/05/2015

Reference ranges

Question 16 of 465

Previous

Next

Question stats

End and review

Score: 72.7%
1

27.7%

11.6%

38.6%

A. Inferior opthalmic vein

11.7%

B. Abducens nerve

10.5%

C. Opthalmic artery

38.6% of users answered this


question correctly

Which of the following does not pass through the superior orbital fissure?

D. Trochlear nerve
E. Superior opthalmic vein

6
7
8
9

Search
Next question
Go

10
11

Mnemonic for the nerves passing through the supraorbital fissure:


Live Frankly To See Absolutely No Insult

12
13
14

Lacrimal
Frontal
Trochlear
Superior Division of Oculomotor
Abducens
Nasociliary
Inferior Division of Oculomotor nerve

15
16

Theme from January 2012 Exam


Theme from April 2014 exam
The opthalmic artery arises from the internal carotid immediately after it has pierced the dura
and arachnoid. It runs through the optic canal below the optic nerve and within its dural and
arachnoid sheaths. It terminates as the supratrochlear and dorsal nasal arteries.
Foramina of the base of the skull

Foramen

Location

Contents

Foramen
ovale

Sphenoid
bone

Otic ganglion
V3 (Mandibular nerve:3rd branch of
trigeminal)
Accessory meningeal artery
Lesser petrosal nerve
Emissary veins

Foramen
spinosum

Sphenoid
bone

Middle meningeal artery


Meningeal branch of the Mandibular nerve

Foramen
rotundum

Sphenoid
bone

Maxillary nerve (V2)

Foramen
lacerum/
carotid canal

Sphenoid
bone

Base of the medial pterygoid plate.


Internal carotid artery*
Nerve and artery of the pterygoid canal

Jugular
foramen

Temporal
bone

Anterior: inferior petrosal sinus


Intermediate: glossopharyngeal, vagus, and accessory nerves.
Posterior: sigmoid sinus (becoming the internal jugular vein) and
some meningeal branches from the occipital and ascending
pharyngeal arteries.

Foramen
magnum

Occipital
bone

Anterior and posterior spinal arteries


Vertebral arteries
Medulla oblongata

Stylomastoid
foramen

Temporal
bone

Stylomastoid artery
Facial nerve

Superior
orbital fissure

Sphenoid
bone

Oculomotor nerve (III)


trochlear nerve (IV)
lacrimal, frontal and nasociliary branches of ophthalmic nerve (V1)
abducent nerve (VI)
Superior and inferior ophthalmic vein

*= In life the foramen lacerum is occluded by a cartilagenous plug. The ICA initially passes
into the carotid canal which ascends superomedially to enter the cranial cavity through the

http://www.emrcs.com/question/question.php?q=0

1/2

15/05/2015

foramen lacerum.
Base of skull anatomical overview

Image sourced from Wikipedia

Rate question:

Next question

Comment on this question

All contents of this site are 2012 E-Medical Revision Ltd

http://www.emrcs.com/question/question.php?q=0

Terms and Conditions

Privacy policy

2/2

15/05/2015

Reference ranges

Previous

Question 17 of 465

Next

An 18 year old man undergoes a tonsillectomy for attacks of recurrent acute tonsillitis. Whilst
in recovery he develops a post operative haemorrhage. Which of the following vessels is the
most likely culprit?

Question stats

Score: 73.9%
1

11.8%

53.9%

14.3%

9.5%

10.5%

A. Facial vein
53.9% of users answered this
question correctly

B. External palatine vein

End and review

6
7

C. External carotid artery


8

D. Internal jugular vein


9

Search

E. None of the above

Go

10
11

Next question

12

The external palatine vein lies immediately lateral to the tonsil and if damaged may be a
cause of reactionary haemorrhage following tonsillectomy.

13

Tonsil

15

14

16

Anatomy

17

Each palatine tonsil has two surfaces, a medial surface which projects into the pharynx
and a lateral surface that is embedded in the wall of the pharynx.
They are usually 25mm tall by 15mm wide, although this varies according to age and
may be almost completely atrophied in the elderly.
Their arterial supply is from the tonsillar artery, a branch of the facial artery.
Its veins pierce the constrictor muscle to join the external palatine or facial veins. The
external palatine vein is immediately lateral to the tonsil, which may result in
haemorrhage during tonsillectomy.
Lymphatic drainage is the jugulodigastric node and the deep cervical nodes.

Tonsillitis
Usually bacterial (50%)- group A Streptococcus. Remainder viral.
May be complicated by development of abscess (quinsy). This may distort the uvula.
- Indications for tonsillectomy include recurrent acute tonsillitis, suspected malignancy,
enlargement causing sleep apnoea.
- Dissection tonsillectomy is the preferred technique with haemorrhage being the commonest
complication. Delayed otalgia may occur owing to irritation of the glossopharyngeal nerve.
Rate question:

Next question

Comment on this question

All contents of this site are 2012 E-Medical Revision Ltd

http://www.emrcs.com/question/question.php?q=0

Terms and Conditions

Privacy policy

1/1

15/05/2015

Reference ranges

Previous

Question 18 of 465

Next

A patient is found to have an ischaemic left colon. Which artery arising from the aorta at
around the level of L3 is most likely to account for this situation?

A. Superior mesenteric artery


B. Inferior mesenteric artery

Question stats

Score: 75%
1

10.8%

68%

6.9%

8.1%

6.1%

68% of users answered this


question correctly

C. Superior rectal artery

End and review

D. Ileocolic artery

6
7
8

E. Middle colic artery

Search
Go

Next question

10
11

Theme from January 2013 Exam


Theme from April 2014 exam
Only the IMA is likely to affect the left side of the colon and originate at L3.

12

Inferior mesenteric artery

15

13
14

16

The IMA is the main arterial supply of the embryonic hindgut and originates approximately 34 cm superior to the aortic bifurcation. From its aortic origin it passes immediately inferiorly
across the anterior aspect of the aorta to eventually lie on its left hand side. At the level of
the left common iliac artery it becomes the superior rectal artery.

17
18

Branches
The left colic artery arises from the IMA near its origin. More distally up to three sigmoid
arteries will exit the IMA to supply the sigmoid colon.
Rate question:

Next question

Comment on this question

All contents of this site are 2012 E-Medical Revision Ltd

http://www.emrcs.com/question/question.php?q=0

Terms and Conditions

Privacy policy

1/1

15/05/2015

Reference ranges

Previous

Question 19 of 465

Next

Question stats

End and review

Score: 72%
1

20.5%

7.7%

12.1%

A. T10

7.5%

B. T9

52.2%

C. T8

52.2% of users answered this


question correctly

At which level does the aorta perforate the diaphragm?

D. T11
E. T12

6
7
8
9

Search
Next question
Go

10
11

Memory aid:
T8 (8 letters) = vena cava
T10 (10 letters) = oesophagus
T12 (12 letters) = aortic hiatus

12
13
14
15
16

Theme from April 2012 exam


Theme from April 2014 exam

17
18

Diaphragm apertures
19

Diaphragm aperture levels


Vena cava T8
Oesophagus T10
Aortic hiatus T12
Rate question:

Next question

Comment on this question

All contents of this site are 2012 E-Medical Revision Ltd

http://www.emrcs.com/question/question.php?q=0

Terms and Conditions

Privacy policy

1/1

15/05/2015

Reference ranges

Previous

Question 20 of 465

Next

A 24 year old lady is stabbed in the buttock. Following the injury the wound is sutured in the
emergency department. Eight weeks later she attends the clinic, as she walks into the clinic
room she has a waddling gait and difficulty with thigh abduction. On examination she has
buttock muscle wasting. Which nerve has been injured?

Question stats

Score: 73.1%
1

42.8%

13.9%

12.6%

7.3%

23.4%

42.8% of users answered this


question correctly

A. Superior gluteal nerve

End and review

B. Obturator nerve

6
7
8

C. Sciatic nerve

Search

D. Femoral nerve
E. Inferior gluteal nerve

Go

10
11

Next question

12
13

Theme from April 2012 Exam


Damage to the superior gluteal nerve will result in a Trendelenburg gait.

14
15

Trendelenburg test

16
17

Injury or division of the superior gluteal nerve results in a motor deficit that consists of
weakened abduction of the thigh by gluteus medius, a disabling gluteus medius limp and a
compensatory list of the body to the weakened gluteal side. The compensation results in a
gravitational shift so that the body is supported on the unaffected limb.

18
19
20

When a person is asked to stand on one leg, the gluteus medius usually contracts as soon
as the contralateral leg leaves the floor, preventing the pelvis from dipping towards the
unsupported side. When a person with paralysis of the superior gluteal nerve is asked to
stand on one leg, the pelvis on the unsupported side descends, indicating that the gluteus
medius on the affected side is weak or non functional ( a positive Trendelenburg test).
This eponymous test also refers to a vascular investigation in which tourniquets are placed
around the upper thigh, these can help determine whether saphenofemoral incompetence is
present.
Rate question:

Next question

Comment on this question

All contents of this site are 2012 E-Medical Revision Ltd

http://www.emrcs.com/question/question.php?q=0

Terms and Conditions

Privacy policy

1/1

15/05/2015

Reference ranges

Question 21 of 465

Previous

Next

Question stats

End and review

Score: 75%
1

52.3%

19.6%

10.4%

A. L1

8.4%

B. L2

9.3%

C. T12

52.3% of users answered this


question correctly

At which level is the hilum of the left kidney located?

D. T11
E. L3

6
7
8
9

Search
Next question
Go

10
11

Remember L1 ('left one') is the level of the hilum of the left kidney

12
13

Theme from April 2012 exam


Theme from April 2014

14

Renal anatomy

16

15

17

Each kidney is about 11cm long, 5cm wide and 3cm thick. They are located in a deep gutter
alongside the projecting vertebral bodies, on the anterior surface of psoas major. In most
cases the left kidney lies approximately 1.5cm higher than the right. The upper pole of both
kidneys approximates with the 11th rib (beware pneumothorax during nephrectomy). On the
left hand side the hilum is located at the L1 vertebral level and the right kidney at level L1-2.
The lower border of the kidneys is usually alongside L3.

18
19
20
21

The table below shows the anatomical relations of the kidneys:


Relations
Relations

Right Kidney

Left Kidney

Posterior

Quadratus lumborum, diaphragm,


psoas major, transversus abdominis

Quadratus lumborum, diaphragm,


psoas major, transversus abdominis

Anterior

Hepatic flexure of colon

Stomach, Pancreatic tail

Superior

Liver, adrenal gland

Spleen, adrenal gland

Fascial covering
Each kidney and suprarenal gland is enclosed within a common layer of investing fascia,
derived from the transversalis fascia. It is divided into anterior and posterior layers (Gerotas
fascia).
Renal structure
Kidneys are surrounded by an outer cortex and an inner medulla which usually contains
between 6 and 10 pyramidal structures. The papilla marks the innermost apex of these. They
terminate at the renal pelvis, into the ureter.
Lying in a hollow within the kidney is the renal sinus. This contains:
1. Branches of the renal artery
2. Tributaries of the renal vein
3. Major and minor calyces's
4. Fat
Structures at the renal hilum
The renal vein lies most anteriorly, then renal artery (it is an end artery) and the ureter lies
most posterior.
Rate question:

Next question

Comment on this question

All contents of this site are 2012 E-Medical Revision Ltd

http://www.emrcs.com/question/question.php?q=0

Terms and Conditions

Privacy policy

1/1

15/05/2015

Reference ranges

Question 22 of 465

Previous

Next

During a radical neck dissection, division of which of the following fascial layers will expose
the ansa cervicalis?

A. Pretracheal fascia
B. Carotid sheath

Question stats

Score: 75.9%
1

32.5%

20.5%

15.7%

19.7%

11.6%

32.5% of users answered this


question correctly

C. Prevertebral fascia

End and review

D. Investing layer of fascia

6
7
8

E. Sibsons fascia

Search
Go

Next question

10
11

The ansa cervicalis lies anterior to the carotid sheath. It may be exposed by division of the
pretracheal fascia at the posterolateral aspect of the thyroid gland. The pre vertebral fascia
lies more posteriorly and division of the investing layer of fascia will not expose this nerve.

12

Ansa cervicalis

15

13
14

16
17

Superior
root

Branch of C1 anterolateral to carotid sheath

Inferior
root

Derived from C2 and C3 roots, passes posterolateral to the internal jugular vein
(may lie either deep or superficial to it)

19

Innervation

Sternohyoid
Sternothyroid
Omohyoid

21

18

20

22

The ansa cervicalis lies anterior to the carotid sheath. The nerve supply to the inferior strap
muscles enters at their inferior aspect. Therefore when dividing these muscles to expose a
large goitre, the muscles should be divided in their upper half.

Image sourced from Wikipedia

Rate question:

Next question

Comment on this question

All contents of this site are 2012 E-Medical Revision Ltd

http://www.emrcs.com/question/question.php?q=0

Terms and Conditions

Privacy policy

1/1

15/05/2015

Reference ranges

Question 1 of 443

Next

A 73 year old lady presents with symptoms of faecal incontinence. On examination she has
weak anal sphincter muscles. What are the main nerve root values of the nerves supplying
the external anal sphincter?

Question stats

End and review

Score: 100%

17.2%

14%

13.6%

11%

44.1%

A. S2,3
44.1% of users answered this
question correctly

B. L5, S1
C. S4,5
D. S5

Search

E. S2,3,4

Go

Next question

S2, 3, 4 Keeps the poo off the floor

Theme from September 2011 Exam


Theme from April 2014 exam
The external anal sphincter is innervated by the inferior rectal branch of the pudendal nerve,
this has root values of S2, 3 and the perineal branch of S4.
Anal sphincter

Internal anal sphincter composed of smooth muscle continuous with the circular
muscle of the rectum. It surrounds the upper two- thirds of the anal canal and is
supplied by sympathetic nerves.
External anal sphincter is composed of striated muscle which surrounds the internal
sphincter but extends more distally.
The nerve supply of the external anal sphincter is from the inferior rectal branch of the
pudendal nerve (S2 and S3) and the perineal branch of the S4 nerve roots.

Image showing relationship of internal and external anal sphincters

Image sourced from Wikipedia

Rate question:

Next question

Comment on this question

All contents of this site are 2012 E-Medical Revision Ltd

http://www.emrcs.com/question/question.php?q=0

Terms and Conditions

Privacy policy

1/1

15/05/2015

Reference ranges

Previous

Question 2 of 443

Next

A 22 year old falls over and lands on a shard of glass. It penetrates the palmar aspect of his
hand, immediately lateral to the pisiform bone. Which of the following structures is most likely
to be injured?

Question stats

End and review

Score: 100%

18.7%

13.6%

45.6%

11%

11%

1
2

A. Palmar cutaneous branch of the median nerve


45.6% of users answered this
question correctly

B. Lateral tendons of flexor digitorum superficialis


C. Ulnar artery
D. Flexor carpi radialis tendons

Search

E. Lateral tendons of flexor digitorum profundus

Go

Next question

The ulnar nerve and artery are at most immediate risk in this injury. This is illustrated in the
image below:

Image sourced from Wikipedia

Hand
Anatomy of the hand
Bones

Intrinsic Muscles

8 Carpal bones
5 Metacarpals
14 phalanges
7 Interossei - Supplied by ulnar nerve
3 palmar-adduct fingers
4 dorsal- abduct fingers

Intrinsic muscles

Lumbricals
Flex MCPJ and extend the IPJ.
Origin deep flexor tendon and insertion dorsal extensor hood
mechanism.
Innervation: 1st and 2nd- median nerve, 3rd and 4th- deep branch of
the ulnar nerve.

Thenar eminence

Hypothenar
eminence

Abductor pollicis brevis


Opponens pollicis
Flexor pollicis brevis

Opponens digiti minimi


Flexor digiti minimi brevis

http://www.emrcs.com/question/question.php?q=0

1/2

15/05/2015

Abductor digiti minimi

Image sourced from Wikipedia

Fascia and compartments of the palm


The fascia of the palm is continuous with the antebrachial fascia and the fascia of the
dorsum of the hand. The palmar fascia is thin over the thenar and hypothenar eminences. In
contrast the palmar fascia is relatively thick. The palmar aponeurosis covers the soft tissues
and overlies the flexor tendons. The apex of the palmar aponeurosis is continuous with the
flexor retinaculum and the palmaris longus tendon. Distally, it forms four longitudinal digital
bands that attach to the bases of the proximal phalanges, blending with the fibrous digital
sheaths.
A medial fibrous septum extends deeply from the medial border of the palmar aponeurosis to
the 5th metacarpal. Lying medial to this are the hypothenar muscles. In a similar fashion, a
lateral fibrous septum extends deeply from the lateral border of the palmar aponeurosis to
the 3rd metacarpal. The thenar compartment lies lateral to this area.
Lying between the thenar and hypothenar compartments is the central compartment. It
contains the flexor tendons and their sheaths, the lumbricals, the superficial palmar arterial
arch and the digital vessels and nerves.
The deepest muscular plane is the adductor compartment, which contains adductor pollicis.
Short muscles of the hand
These comprise the lumbricals and interossei. The four slender lumbrical muscles flex the
fingers at the metacarpophalangeal joints and extend the interphalangeal joint. The four
dorsal interossei are located between the metacarpals and the four palmar interossei lie on
the palmar surface of the metacarpals in the interosseous compartment of the hand.
Long flexor tendons and sheaths in the hand
The tendons of FDS and FDP enter the common flexor sheath deep to the flexor
retinaculum. The tendons enter the central compartment of the hand and fan out to their
respective digital synovial sheaths. Near the base of the proximal phalanx, the tendon of FDS
splits to permit the passage of FDP. The FDP tendons are attached to the margins of the
anterior aspect of the base of the distal phalanx.
The fibrous digital sheaths contain the flexor tendons and their synovial sheaths. These
extend from the heads of the metacarpals to the base of the distal phalanges.
Rate question:

Next question

Comment on this question

All contents of this site are 2012 E-Medical Revision Ltd

http://www.emrcs.com/question/question.php?q=0

Terms and Conditions

Privacy policy

2/2

15/05/2015

Reference ranges

Previous

Question 3 of 443

Next

A 72 year old man has a fall. He is found to have a fractured neck of femur and goes on to
have a left hip hemiarthroplasty. Two months post operatively he is found to have an odd
gait. When standing on his left leg his pelvis dips on the right side. There is no foot drop.
What is the cause?

Question stats

End and review

Score: 100%
1

8.1%

6.6%

18.1%

6.8%

60.4%

60.4% of users answered this


question correctly

A. Sciatic nerve damage


B. L5 radiculopathy
C. Inferior gluteal nerve damage

Search

D. Previous poliomyelitis
E. Superior gluteal nerve damage

Go

Next question

Theme from 2010 exam


Theme from April 2014 exam
This patient has a trendelenburg gait caused by damage to the superior gluteal nerve
causing weakness of the abductor muscles. Classically a patient is asked to stand on one leg
and the pelvis dips on the opposite side. The absence of a foot drop excludes the possibility
of polio or L5 radiculopathy.
Gluteal region
Gluteal muscles
Gluteus maximus: inserts to gluteal tuberosity of the femur and iliotibial tract
Gluteus medius: attach to lateral greater trochanter
Gluteus minimis: attach to anterior greater trochanter
All extend and abduct the hip

Deep lateral hip rotators


Piriformis
Gemelli
Obturator internus
Quadratus femoris

Nerves
Superior gluteal nerve (L5, S1)

Inferior gluteal nerve

Gluteus medius
Gluteus minimis
Tensor fascia lata
Gluteus maximus

Damage to the superior gluteal nerve will result in the patient developing a Trendelenberg
gait. Affected patients are unable to abduct the thigh at the hip joint. During the stance
phase, the weakened abductor muscles allow the pelvis to tilt down on the opposite side. To
compensate, the trunk lurches to the weakened side to attempt to maintain a level pelvis
throughout the gait cycle. The pelvis sags on the opposite side of the lesioned superior
gluteal nerve.
Rate question:

Next question

Comment on this question

All contents of this site are 2012 E-Medical Revision Ltd

http://www.emrcs.com/question/question.php?q=0

Terms and Conditions

Privacy policy

1/1

15/05/2015

Reference ranges

Question 4 of 443

Previous

Next

Question stats

End and review

Score: 75%
1

15.8%

28.7%

18.5%

A. Adductor longus

28.6%

B. Pectineus

8.5%

C. Psoas major

28.6% of users answered this


question correctly

Which of the following structures lies posterior to the femoral nerve in the femoral triangle?

D. Iliacus
E. None of the above

Search
Next question
Go

The iliacus lies posterior to the femoral nerve in the femoral triangle. The femoral sheath lies
anterior to the iliacus and pectineus muscles.
Femoral nerve

Root values

L2, 3, 4

Innervates

Pectineus
Sartorius
Quadriceps femoris
Vastus lateralis/medialis/intermedius

Branches

Medial cutaneous nerve of thigh


Saphenous nerve
Intermediate cutaneous nerve of thigh

Path
Penetrates psoas major and exits the pelvis by passing under the inguinal ligament to enter
the femoral triangle, lateral to the femoral artery and vein.

Image sourced from Wikipedia

Mnemonic for femoral nerve supply


(don't) M I S V Q Scan for PE
M edial cutaneous nerve of the thigh
I ntermediate cutaneous nerve of the thigh
S aphenous nerve
V astus
Q uadriceps femoris
S artorius
PE ectineus
http://www.emrcs.com/question/question.php?q=0

1/2

15/05/2015

Rate question:

Next question

Comment on this question

All contents of this site are 2012 E-Medical Revision Ltd

http://www.emrcs.com/question/question.php?q=0

Terms and Conditions

Privacy policy

2/2

16/05/2015

Reference ranges

Question 1 of 439

Next

You are assisting in an open right adrenalectomy for a large adrenal adenoma. The
consultant is distracted and you helpfully pull the adrenal into the wound to improve the view.
Unfortunately this is followed by brisk bleeding. The vessel responsible for this is most likely
to be:

Question stats

End and review

Score: 0%

6.9%

13.6%

35.9%

9.4%

34.3%

34.3% of users answered this


question correctly

A. Portal vein
B. Phrenic vein
C. Right renal vein

Search

D. Superior mesenteric vein


E. Inferior vena cava

Go

Next question

Theme from April 2014 exam


It drains directly via a very short vessel. If the sutures are not carefully tied then it may be
avulsed off the IVC. An injury best managed using a Satinsky clamp and a 6/0 prolene
suture.
Adrenal gland anatomy
Anatomy
Location

Superomedially to the upper pole of each kidney

Relationships of
the right adrenal

Diaphragm-Posteriorly, Kidney-Inferiorly, Vena Cava-Medially, Hepatorenal pouch and bare area of the liver-Anteriorly

Relationships of
the left adrenal

Crus of the diaphragm-Postero- medially, Pancreas and splenic


vessels-Inferiorly, Lesser sac and stomach-Anteriorly

Arterial supply

Superior adrenal arteries- from inferior phrenic artery, Middle adrenal


arteries - from aorta, Inferior adrenal arteries -from renal arteries

Venous drainage
of the right adrenal

Via one central vein directly into the IVC

Venous drainage
of the left adrenal

Via one central vein into the left renal vein

Rate question:

Next question

Comment on this question

All contents of this site are 2012 E-Medical Revision Ltd

http://www.emrcs.com/question/question.php?q=0

Terms and Conditions

Privacy policy

1/1

16/05/2015

Reference ranges

Previous

Question 2 of 439

Next

A 28 year old lady requires an episiotomy for a ventouse vaginal delivery. Which of the
nerves listed below will usually be anaesthetised to allow the episiotomy?

A. Femoral
B. Ilioinguinal

Question stats

End and review

Score: 50%

5.4%

10.3%

58.7%

16.7%

8.8%

1
2

58.7% of users answered this


question correctly

C. Pudendal
D. Genitofemoral
E. Sacral plexus

Search
Go

Next question

Theme from April 2013 Exam


The pudendal nerve innervates the posterior vulval area and is routinely blocked in
procedures such as episiotomy.
Pudendal nerve
The pudendal nerve arises from nerve roots S2, S3 and S4 and exits the pelvis through the
greater sciatic foramen. It re-enters the perineum through the lesser sciatic foramen. It
travels inferior to give innervation to the anal sphincters and external urethral sphincter. It
also provides cutaneous innervation to the region of perineum surrounding the anus and
posterior vulva.
Traction and compression of the pudendal nerve by the foetus in late pregnancy may result
in late onset pudendal neuropathy which may be part of the process involved in the
development of faecal incontinence.
Rate question:

Next question

Comment on this question

All contents of this site are 2012 E-Medical Revision Ltd

http://www.emrcs.com/question/question.php?q=0

Terms and Conditions

Privacy policy

1/1

16/05/2015

Reference ranges

Question 1 of 437

Next

A motorcyclist is involved in a road traffic accident. He suffers a complex humeral


shaft fracture which is plated. Post operatively he complains of an inability to
extend his fingers. Which of the following structures is most likely to have been
injured?

Question stats

End and review

Score: 100%

7.9%

67.2%

10.1%

7.5%

7.3%

67.2% of users answered this


question correctly

A. Ulnar nerve
B. Radial nerve
C. Median nerve

Search

D. Axillary nerve
E. None of the above

Go

Next question

Mnemonic for radial nerve muscles: BEST


B rachioradialis
E xtensors
S upinator
T riceps

The radial nerve is responsible for innervation of the extensor compartment of the
forearm.
Radial nerve
Continuation of posterior cord of the brachial plexus (root values C5 to T1)
Path
In the axilla: lies posterior to the axillary artery on subscapularis, latissimus
dorsi and teres major.
Enters the arm between the brachial artery and the long head of triceps
(medial to humerus).
Spirals around the posterior surface of the humerus in the groove for the
radial nerve.
At the distal third of the lateral border of the humerus it then pierces the
intermuscular septum and descends in front of the lateral epicondyle.
At the lateral epicondyle it lies deeply between brachialis and
brachioradialis where it then divides into a superficial and deep terminal
branch.
Deep branch crosses the supinator to become the posterior interosseous
nerve.

In the image below the relationships of the radial nerve can be appreciated

http://www.emrcs.com/question/question.php?q=0

1/3

16/05/2015

Image sourced from Wikipedia

Regions innervated
Motor (main
nerve)

Triceps
Anconeus
Brachioradialis
Extensor carpi radialis

Motor
(posterior
interosseous
branch)

Supinator
Extensor carpi ulnaris
Extensor digitorum
Extensor indicis
Extensor digiti minimi
Extensor pollicis longus and brevis
Abductor pollicis longus

Sensory

The area of skin supplying the proximal phalanges on the dorsal


aspect of the hand is supplied by the radial nerve (this does not
apply to the little finger and part of the ring finger)

Muscular innervation and effect of denervation


Anatomical
location

Muscle affected

Effect of paralysis

Shoulder

Long head of
triceps

Minor effects on shoulder stability in


abduction

Arm

Triceps

Loss of elbow extension

Forearm

Supinator
Brachioradialis
Extensor carpi
radialis longus
and brevis

Weakening of supination of prone


hand and elbow flexion in mid prone
position

The cutaneous sensation of the upper limb- illustrating the contribution of the
radial nerve

http://www.emrcs.com/question/question.php?q=0

2/3

16/05/2015

Image sourced from Wikipedia

Rate question:

Next question

Comment on this question

All contents of this site are 2012 E-Medical Revision Ltd

http://www.emrcs.com/question/question.php?q=0

Terms and Conditions

Privacy policy

3/3

16/05/2015

Reference ranges

Previous

Question 2 of 437

Next

An enthusiastic surgical registrar undertakes his first solo splenectomy. The


operation is far more difficult than anticipated and the registrar leaves a tube
drain to the splenic bed at the end of the procedure. Over the following 24 hours
approximately 500ml of clear fluid has entered the drain. Biochemical testing of
the fluid is most likely to reveal:

Question stats

End and review

Score: 100%

10.9%

13.2%

9.6%

55.2%

11.1%

1
2

55.2% of users answered this


question correctly

A. Elevated creatinine
B. Elevated triglycerides
C. Elevated glucagon

Search

D. Elevated amylase

Go

E. None of the above


Next question

During splenectomy the tail of the pancreas may be damaged. The pancreatic
duct will then drain into the splenic bed, amylase is the most likely biochemical
finding. Glucagon is not secreted into the pancreatic duct.
Splenic anatomy
The spleen is the largest lymphoid organ in the body. It is an intraperitoneal
organ, the peritoneal attachments condense at the hilum where the vessels enter
the spleen. Its blood supply is from the splenic artery (derived from the coeliac
axis) and the splenic vein (which is joined by the IMV and unites with the SMV).

Embryology: derived from mesenchymal tissue


Shape: clenched fist
Position: below 9th-12th ribs
Weight: 75-150g

Relations
Superiorly- diaphragm
Anteriorly- gastric impression
Posteriorly- kidney
Inferiorly- colon
Hilum: tail of pancreas and splenic vessels
Forms apex of lesser sac (containing short gastric vessels)

Rate question:

Next question

Comment on this question

All contents of this site are 2012 E-Medical Revision Ltd

http://www.emrcs.com/question/question.php?q=0

Terms and Conditions

Privacy policy

1/1

16/05/2015

Reference ranges

Previous

Question 3 of 437

Next

A 48 year old lady is undergoing an axillary node clearance for breast cancer.
Which of the structures listed below are most likely to be encountered during the
axillary dissection?

Question stats

End and review

Score: 100%
1

24.1%

36.5%

13.5%

18.5%

7.4%

A. Cords of the brachial plexus


36.5% of users answered this
question correctly

B. Thoracodorsal trunk
C. Internal mammary artery
D. Thoracoacromial artery

Search

E. None of the above

Go

Next question

Beware of damaging the thoracodorsal trunk if a latissimus dorsi flap


reconstruction is planned.

Theme from 2009 Exam


The thoracodorsal trunk runs through the nodes in the axilla. If injured it may
compromise the function and blood supply to latissimus dorsi, which is significant
if it is to be used as a flap for a reconstructive procedure.
Axilla
Boundaries of the axilla
Medially

Chest wall and Serratus anterior

Laterally

Humeral head

Floor

Subscapularis

Anterior aspect

Lateral border of Pectoralis major

Fascia

Clavipectoral fascia

Content:
Long thoracic
nerve (of Bell)

Derived from C5-C7 and passes behind the brachial plexus to


enter the axilla. It lies on the medial chest wall and supplies
serratus anterior. Its location puts it at risk during axillary
surgery and damage will lead to winging of the scapula.

Thoracodorsal
nerve and
thoracodorsal
trunk

Innervate and vascularise latissimus dorsi.

Axillary vein

Lies at the apex of the axilla, it is the continuation of the


basilic vein. Becomes the subclavian vein at the outer border
of the first rib.

Intercostobrachial
nerves

Traverse the axillary lymph nodes and are often divided during
axillary surgery. They provide cutaneous sensation to the
axillary skin.

Lymph nodes

The axilla is the main site of lymphatic drainage for the


breast.

Rate question:

http://www.emrcs.com/question/question.php?q=0

Next question

1/2

16/05/2015

Comment on this question

All contents of this site are 2012 E-Medical Revision Ltd

http://www.emrcs.com/question/question.php?q=0

Terms and Conditions

Privacy policy

2/2

16/05/2015

Reference ranges

Previous

Question 4 of 437

Next

A 53 year old lady is recovering following a difficult mastectomy and axillary nodal
clearance for carcinoma of the breast. She complains of shoulder pain and on
examination has obvious winging of the scapula. Loss of innervation to which of
the following is the most likely underlying cause?

Question stats

End and review

Score: 100%
1

19.3%

59.6%

6.4%

7.2%

7.4%

59.6% of users answered this


question correctly

A. Latissimus dorsi
B. Serratus anterior
C. Pectoralis minor

Search

D. Pectoralis major
E. Rhomboids

Go

Next question

Theme from April 2012 exam


Theme from April 2014 exam
Winging of the scapula is most commonly the result of long thoracic nerve injury
or dysfunction. Iatrogenic damage during the course of the difficult axillary
dissection is the most likely cause in this scenario. Damage to the rhomboids may
produce winging of the scapula but would be rare in the scenario given.
Long thoracic nerve

Derived from ventral rami of C5, C6, and C7 (close to their emergence from
intervertebral foramina)
It runs downward and passes either anterior or posterior to the middle
scalene muscle
It reaches upper tip of serratus anterior muscle and descends on outer
surface of this muscle, giving branches into it
Winging of Scapula occurs in long thoracic nerve injury (most common) or
from spinal accessory nerve injury (which denervates the trapezius) or a
dorsal scapular nerve injury

Rate question:

Next question

Comment on this question

All contents of this site are 2012 E-Medical Revision Ltd

http://www.emrcs.com/question/question.php?q=0

Terms and Conditions

Privacy policy

1/1

16/05/2015

Reference ranges

Previous

Question 5 of 437

Next

A 56 year old lady is referred to the colorectal clinic with symptoms of pruritus ani.
On examination a polypoidal mass is identified inferior to the dentate line. A
biopsy confirms squamous cell carcinoma. To which of the following lymph node
groups will the lesion potentially metastasise?

Question stats

End and review

Score: 100%
1

16.8%

13.2%

17%

46.8%

6.2%

46.8% of users answered this


question correctly

A. Internal iliac
B. External iliac
C. Mesorectal

Search

D. Inguinal
E. None of the above

Go

Next question

Theme from September 2011 Exam


Theme from April 2012 Exam
Theme from April 2014 exam
Lesions distal to the dentate line drain to the inguinal nodes. Occasionally this will
result in the need for a block dissection of the groin.

Rectum
The rectum is approximately 12 cm long. It is a capacitance organ. It has both
intra and extraperitoneal components. The transition between the sigmoid colon is
marked by the disappearance of the tenia coli.The extra peritoneal rectum is
surrounded by mesorectal fat that also contains lymph nodes. This mesorectal
fatty layer is removed surgically during rectal cancer surgery (Total Mesorectal
Excision). The fascial layers that surround the rectum are important clinical
landmarks, anteriorly lies the fascia of Denonvilliers. Posteriorly lies Waldeyers
fascia.
Extra peritoneal rectum
Posterior upper third
Posterior and lateral middle third
Whole lower third

Relations
Anteriorly (Males)

Rectovesical pouch
Bladder
Prostate
Seminal vesicles

Anteriorly (Females)

Recto-uterine pouch (Douglas)


Cervix
Vaginal wall

Posteriorly

Sacrum
Coccyx
Middle sacral artery

Laterally

Levator ani
Coccygeus

Arterial supply
Superior rectal artery
Venous drainage
Superior rectal vein
http://www.emrcs.com/question/question.php?q=0

1/2

16/05/2015

Lymphatic drainage
Mesorectal lymph nodes (superior to dentate line)
Internal iliac and then para-aortic nodes
Inguinal nodes (inferior to dentate line)

Rate question:

Next question

Comment on this question

All contents of this site are 2012 E-Medical Revision Ltd

http://www.emrcs.com/question/question.php?q=0

Terms and Conditions

Privacy policy

2/2

16/05/2015

Reference ranges

Previous

Question 6 of 437

Next

A 20 year old man is hit with a hammer on the right side of the head. He dies on
arrival in the emergency department. Which of these features is most likely to be
found at post mortem?

Question stats

Score: 100%
1

7%

18.2%

55.5%

12.8%

6.5%

A. Hydrocephalus
55.5% of users answered this
question correctly

B. Supra tentorial herniation

End and review

C. Laceration of the middle meningeal artery


D. Sub dural haematoma
Search

E. Posterior fossa haematoma

Go

Next question

Theme based on 2011 exam


This will account for the scenario given where there is a brief delay prior to death.
The other options are less acute and a supratentorial herniation would not occur
in this setting.
Head injury
Patients who suffer head injuries should be managed according to ATLS
principles and extra cranial injuries should be managed alongside cranial trauma.
Inadequate cardiac output will compromise CNS perfusion irrespective of the
nature of the cranial injury.
Types of traumatic brain injury
Extradural
haematoma

Bleeding into the space betw een the dura mater and the skull.
Often results from acceleration-deceleration trauma or a blow
to the side of the head. The majority of extradural haematomas
occur in the temporal region w here skull fractures cause a
rupture of the middle meningeal artery.
Features
Raised intracranial pressure
Some patients may exhibit a lucid interval

Subdural
haematoma

Bleeding into the outermost meningeal layer. Most commonly


occur around the frontal and parietal lobes. May be either
acute or chronic.
Risk factors include old age and alcoholism.
Slow er onset of symptoms than a extradural haematoma.

Subarachnoid Usually occurs spontaneously in the context of a ruptured


haemorrhage cerebral aneurysm, but may be seen in association w ith other
injuries w hen a patient has sustained a traumatic brain injury.

Pathophysiology
Primary brain injury may be focal (contusion/ haematoma) or diffuse
(diffuse axonal injury)
Diffuse axonal injury occurs as a result of mechanical shearing following
deceleration, causing disruption and tearing of axons
Intra-cranial haematomas can be extradural, subdural or intracerebral,
while contusions may occur adjacent to (coup) or contralateral (contrecoup) to the side of impact
Secondary brain injury occurs when cerebral oedema, ischaemia, infection,
tonsillar or tentorial herniation exacerbates the original injury. The normal
cerebral auto regulatory processes are disrupted following trauma
rendering the brain more susceptible to blood flow changes and hypoxia
The Cushings reflex (hypertension and bradycardia) often occurs late and
is usually a pre terminal event

http://www.emrcs.com/question/question.php?q=0

1/2

16/05/2015

Management
Where there is life threatening rising ICP such as in extra dural haematoma
and whilst theatre is prepared or transfer arranged use of IV mannitol/
frusemide may be required.
Diffuse cerebral oedema may require decompressive craniotomy
Exploratory Burr Holes have little management in modern practice except
where scanning may be unavailable and to thus facilitate creation of formal
craniotomy flap
Depressed skull fractures that are open require formal surgical reduction
and debridement, closed injuries may be managed non operatively if there
is minimal displacement.
ICP monitoring is appropriate in those who have GCS 3-8 and normal CT
scan.
ICP monitoring is mandatory in those who have GCS 3-8 and abnormal CT
scan.
Hyponatraemia is most likely to be due to syndrome of inappropriate ADH
secretion.
Minimum of cerebral perfusion pressure of 70mmHg in adults.
Minimum cerebral perfusion pressure of between 40 and 70 mmHg in
children.

Interpretation of pupillary findings in head injuries


Pupil size

Light response

Interpretation

Unilaterally
dilated

Sluggish or fixed

3rd nerve compression secondary


to tentorial herniation

Bilaterally
dilated

Sluggish or fixed

Unilaterally
dilated or equal

Cross reactive
(Marcus - Gunn)

Bilaterally
constricted

May be difficult to
assess

Unilaterally
constricted

Preserved

Rate question:

Poor CNS perfusion


Bilateral 3rd nerve palsy
Optic nerve injury

Opiates
Pontine lesions
Metabolic encephalopathy
Sympathetic pathway disruption

Next question

Comment on this question

All contents of this site are 2012 E-Medical Revision Ltd

http://www.emrcs.com/question/question.php?q=0

Terms and Conditions

Privacy policy

2/2

16/05/2015

Reference ranges

Previous

Question 7 of 437

Next

Which of the following ligaments contains the artery supplying the head of femur
in children?

A. Transverse ligament
B. Ligamentum teres

Question stats

Score: 100%
1

10.8%

54.4%

14.5%

12.7%

7.7%

54.4% of users answered this


question correctly

C. Iliofemoral ligament

End and review

6
7

D. Ischiofemoral ligament
E. Pubofemoral ligament

Search
Next question

Go

Theme from 2010 Exam


Hip joint

Head of femur articulates with acetabulum of the pelvis


Both covered by articular hyaline cartilage
The acetabulum forms at the union of the ilium, pubis, and ischium
The triradiate cartilage (Y-shaped growth plate) separates the pelvic bones
The acetabulum holds the femoral head by the acetabular labrum
Normal angle between femoral head and femoral shaft is 130o

Ligaments
Transverse ligament: joints anterior and posterior ends of the articular
cartilage
Head of femur ligament (ligamentum teres): acetabular notch to the fovea.
Contains arterial supply to head of femur in children.

Image sourced from Wikipedia

http://www.emrcs.com/question/question.php?q=0

1/2

16/05/2015

Image sourced from Wikipedia

Extracapsular ligaments
Iliofemoral ligament: inverted Y shape. Anterior iliac spine to the
trochanteric line
Pubofemoral ligament: acetabulum to lesser trochanter
Ischiofemoral ligament: posterior support. Ischium to greater trochanter.

Blood supply
Medial circumflex femoral and lateral circumflex femoral arteries (Branches of
profunda femoris). Also from the inferior gluteal artery. These form an
anastomosis and travel to up the femoral neck to supply the head.

Rate question:

Next question

Comment on this question

All contents of this site are 2012 E-Medical Revision Ltd

http://www.emrcs.com/question/question.php?q=0

Terms and Conditions

Privacy policy

2/2

16/05/2015

Reference ranges

Previous

Question 8 of 437

Next

A 72 year old man develops a hydrocele which is being surgically managed. As


part of the procedure the surgeons divide the tunica vaginalis. From which of the
following is this structure derived?

Question stats

Score: 81.8%
1

47.3%

14.3%

11.2%

21.2%

6%

47.3% of users answered this


question correctly

A. Peritoneum
B. External oblique aponeurosis

End and review

C. Internal oblique aponeurosis


8

D. Transversalis fascia
Search

E. Rectus sheath

Go

Next question

The tunica vaginalis is derived from peritoneum, it secretes the fluid that fills the
hydrocele cavity.
Scrotal and testicular anatomy
Spermatic cord
Formed by the vas deferens and is covered by the following structures:
Layer

Origin

Internal spermatic fascia

Transversalis fascia

Cremasteric fascia

From the fascial coverings of internal oblique

External spermatic fascia

External oblique aponeurosis

Contents of the cord


Vas deferens

Transmits sperm and accessory gland


secretions

Testicular artery

Branch of abdominal aorta supplies testis


and epididymis

Artery of vas deferens

Arises from inferior vesical artery

Cremasteric artery

Arises from inferior epigastric artery

Pampiniform plexus

Venous plexus, drains into right or left


testicular vein

Sympathetic nerve fibres

Lie on arteries, the parasympathetic fibres lie


on the vas

Genital branch of the


genitofemoral nerve

Supplies cremaster

Lymphatic vessels

Drain to lumbar and para-aortic nodes

Scrotum
Composed of skin and closely attached dartos fascia.
Arterial supply from the anterior and posterior scrotal arteries
Lymphatic drainage to the inguinal lymph nodes
Parietal layer of the tunica vaginalis is the innermost layer

Testes
The testes are surrounded by the tunica vaginalis (closed peritoneal sac).
http://www.emrcs.com/question/question.php?q=0

1/2

16/05/2015

The parietal layer of the tunica vaginalis adjacent to the internal spermatic
fascia.
The testicular arteries arise from the aorta immediately inferiorly to the
renal arteries.
The pampiniform plexus drains into the testicular veins, the left drains into
the left renal vein and the right into the inferior vena cava.
Lymphatic drainage is to the para-aortic nodes.

Rate question:

Next question

Comment on this question

All contents of this site are 2012 E-Medical Revision Ltd

http://www.emrcs.com/question/question.php?q=0

Terms and Conditions

Privacy policy

2/2

16/05/2015

Reference ranges

Question 9 of 437

Previous

Next

A 43 year old lady is donating her left kidney to her sister and the surgeons are
harvesting the left kidney. Which of the following structures will lie most anteriorly
at the hilum of the left kidney?

Question stats

Score: 84.6%
1

17%

51.7%

15.1%

8.9%

7.3%

A. Left renal artery


51.7% of users answered this
question correctly

B. Left renal vein

End and review

6
7

C. Left ureter
8

D. Left ovarian vein


9

Search

E. Left ovarian artery

Go

Next question

The renal veins lie most anteriorly, then artery and ureter lies posteriorly.
Renal arteries

The right renal artery is longer than the left renal artery
The renal vein/artery/pelvis enter the kidney at the hilum

Relations
Right

Anterior- IVC, right renal vein, the head of the pancreas, and the descending
part of the duodenum

Left

Anterior- left renal vein, the tail of the pancreas

Branches
The renal arteries are direct branches off the aorta (upper border of L2)
In 30% there may be accessory arteries (mainly left side). Instead of
entering the kidney at the hilum, they usually pierce the upper or lower part
of the organ.
Before reaching the hilum of the kidney, each artery divides into four or five
segmental branches (renal vein anterior and ureter posterior); which then
divide within the sinus into lobar arteries supplying each pyramid and
cortex.
Each vessel gives off some small inferior suprarenal branches to the
suprarenal gland, the ureter, and the surrounding cellular tissue and
muscles.

Rate question:

Next question

Comment on this question

All contents of this site are 2012 E-Medical Revision Ltd

http://www.emrcs.com/question/question.php?q=0

Terms and Conditions

Privacy policy

1/1

16/05/2015

Reference ranges

Previous

Question 10 of 437

Next

Question stats

End and review

Score: 85.7%
1

8.7%

36.8%

9.8%

A. Maxillary branch of the trigeminal nerve

35.8%

B. Mandibular branch of the trigeminal nerve

9%

C. C3-C4

35.8% of users answered this


question correctly

What is the sensory nerve supply to the angle of the jaw?

D. Greater auricular nerve (C2-C3)


E. Buccal branch of the facial nerve

7
8
9

Search
Next question
Go

10

The trigeminal nerve is the major sensory nerve to the face except over the angle
of the jaw. The angle of the jaw is innervated by the greater auricular nerve.
Trigeminal nerve
The trigeminal nerve is the main sensory nerve of the head. In addition to its
major sensory role, it also innervates the muscles of mastication.
Distribution of the trigeminal nerve
Sensory

Scalp
Face
Oral cavity (and teeth)
Nose and sinuses
Dura mater

Motor

Muscles of mastication
Mylohyoid
Anterior belly of digastric
Tensor tympani
Tensor palati

Autonomic connections (ganglia)

Ciliary
Sphenopalatine
Otic
Submandibular

Path
Originates at the pons
Sensory root forms the large, crescentic trigeminal ganglion within Meckel's
cave, and contains the cell bodies of incoming sensory nerve fibres. Here
the 3 branches exit.
The motor root cell bodies are in the pons and the motor fibres are
distributed via the mandibular nerve. The motor root is not part of the
trigeminal ganglion.

Branches of the trigeminal nerve


Ophthalmic nerve

Sensory only

Maxillary nerve

Sensory only

Mandibular nerve

Sensory and motor

Sensory
http://www.emrcs.com/question/question.php?q=0

1/2

16/05/2015

Ophthalmic

Exits skull via the superior orbital fissure


Sensation of: scalp and forehead, the upper eyelid, the conjunctiva
and cornea of the eye, the nose (including the tip of the nose, except
alae nasi), the nasal mucosa, the frontal sinuses, and parts of the
meninges (the dura and blood vessels).

Maxillary
nerve

Exit skull via the foramen rotundum


Sensation: lower eyelid and cheek, the nares and upper lip, the
upper teeth and gums, the nasal mucosa, the palate and roof of the
pharynx, the maxillary, ethmoid and sphenoid sinuses, and parts of
the meninges.

Mandibular
nerve

Exit skull via the foramen ovale


Sensation: lower lip, the lower teeth and gums, the chin and jaw
(except the angle of the jaw), parts of the external ear, and parts of
the meninges.

Motor
Distributed via the mandibular nerve.
The following muscles of mastication are innervated:
Masseter
Temporalis
Medial pterygoid
Lateral pterygoid

Other muscles innervated include:


Tensor veli palatini
Mylohyoid
Anterior belly of digastric
Tensor tympani

Rate question:

Next question

Comment on this question

All contents of this site are 2012 E-Medical Revision Ltd

http://www.emrcs.com/question/question.php?q=0

Terms and Conditions

Privacy policy

2/2

16/05/2015

Reference ranges

Previous

Question 11 of 437

Next

A 63 year old man is undergoing a coronary artery bypass procedure. During the
median sternotomy which structure would routinely require division?

A. Parietal pleura
B. Interclavicular ligament

Question stats

Score: 87.5%
1

26.2%

40.2%

16%

10.8%

6.8%

40.2% of users answered this


question correctly

C. Internal mammary artery

End and review

D. Brachiocephalic vein

6
7
8

E. Left vagus nerve

Search
Go

Next question

10
11

Theme from January 2011 Exam


The interclavicular ligament lies at the upper end of a median sternotomy and is
routinely divided to provide access. The pleural reflections are often encountered
and should not be intentionally divided, if they are, then a chest drain will need to
be inserted on the affected side as collections may then accumulate in the pleural
cavity. Other structures encountered include the pectoralis major muscles, again
if the incision is truly midline then these should not require formal division. The
close relationship of the brachiocephalic vein should be borne in mind and it
should be avoided, iatrogenic injury to this structure will result in considerable
haemorrhage.
Sternotomy
A median sternotomy is the commonest incision utilised to access the heart and
great vessels.
A midline incision is made from the interclavicular fossa to the xiphoid process
and the fat and subcutaneous tissues are divided to the level of the sternum. The
periosteum may be gently mobilised off the midline, although vigorous periosteal
stripping is best avoided. A bone saw is used to divide the bone itself. Posteriorly
the reflections of the parietal pleura should be identified and avoided (unless
surgery to the lung is planned). The fibrous pericardium is incised and the heart
brought into view. Bleeding from the bony edges of the cut sternum is stopped
using roller ball diathermy or bone wax. The left brachiocephalic vein is an
important posterior relation at the superior aspect of the sternotomy incision and
should be avoided. More inferior the thymic remnants may be identified. At the
inferior aspect of the incision the abdominal cavity may be entered (though this is
seldom troublesome).

Rate question:

Next question

Comment on this question

All contents of this site are 2012 E-Medical Revision Ltd

http://www.emrcs.com/question/question.php?q=0

Terms and Conditions

Privacy policy

1/1

16/05/2015

Reference ranges

Question 12 of 437

Previous

Next

Which of the following structures separates the subclavian artery from the
subclavian vein?

A. Scalenus anterior
B. Scalenus medius

Question stats

Score: 88.9%
1

57.8%

17.4%

8.9%

8.1%

7.8%

57.8% of users answered this


question correctly

C. Sternocleidomastoid

End and review

D. Pectoralis major

6
7
8

E. Pectoralis minor

Search
Next question

Go

10
11

The artery and vein are separated by scalenus anterior. This muscle runs from
the transverse processes of C3,4,5 and 6 to insert onto the scalene tubercle of
the first rib.

12

Subclavian artery
Path
The left subclavian comes directly off the arch of aorta
The right subclavian arises from the brachiocephalic artery (trunk) when it
bifurcates into the subclavian and the right common carotid artery.
From its origin, the subclavian artery travels laterally, passing between
anterior and middle scalene muscles, deep to scalenus anterior and
anterior to scalenus medius. As the subclavian artery crosses the lateral
border of the first rib, it becomes the axillary artery. At this point it is
superficial and within the subclavian triangle.

Image sourced from Wikipedia

Branches
Vertebral artery
Internal thoracic artery
Thyrocervical trunk
Costocervical trunk
Dorsal scapular artery

Rate question:

Next question

Comment on this question

http://www.emrcs.com/question/question.php?q=0

1/2

16/05/2015

All contents of this site are 2012 E-Medical Revision Ltd

http://www.emrcs.com/question/question.php?q=0

Terms and Conditions

Privacy policy

2/2

16/05/2015

Reference ranges

Previous

Question 13 of 437

Next

A 56 year old lady is due to undergo a left hemicolectomy for carcinoma of the
splenic flexure. The surgeons decide to perform a high ligation of the inferior
mesenteric vein. Into which of the following does this structure usually drain?

Question stats

Score: 90%
1

25%

21.3%

9.6%

7.6%

36.4%

A. Portal vein
36.4% of users answered this
question correctly

B. Inferior vena cava

End and review

6
7

C. Left renal vein


8

D. Left iliac vein


9

Search

E. Splenic vein

Go

10
11

Next question

12
13

Beware of ureteric injury in colonic surgery.

The inferior mesenteric vein drains into the splenic vein, this point of union lies
close to the duodenum and this surgical maneouvre is a recognised cause of
ileus.
Left colon
Position
As the left colon passes inferiorly its posterior aspect becomes
extraperitoneal, and the ureter and gonadal vessels are close posterior
relations that may become involved in disease processes
At a level of L3-4 (variable) the left colon becomes the sigmoid colon and
wholly intraperitoneal once again
The sigmoid colon is a highly mobile structure and may even lie on the right
side of the abdomen
It passes towards the midline, the taenia blend and this marks the transition
between sigmoid colon and upper rectum

Blood supply
Inferior mesenteric artery
However, the marginal artery (from the right colon) contributes, this
contribution becomes clinically significant when the IMA is divided surgically
(e.g. During AAA repair)

Rate question:

Next question

Comment on this question

All contents of this site are 2012 E-Medical Revision Ltd

http://www.emrcs.com/question/question.php?q=0

Terms and Conditions

Privacy policy

1/1

16/05/2015

Reference ranges

Previous

Question 14 of 437

Next

A man undergoes a high anterior resection for carcinoma of the upper rectum.
Which of the following vessels will require ligation?

A. Superior mesenteric artery


B. Inferior mesenteric artery

Question stats

Score: 90.5%
1

9.1%

60%

7%

11.9%

12%

60% of users answered this


question correctly

C. Coeliac axis

End and review

D. Perineal artery

6
7
8

E. Middle colic artery

Search
Next question

Go

10
11

Theme from April 2013 Exam


Theme from September 2013 Exam
The IMA is usually divided during anterior resection. Not only is this borne out of
oncological necessity but it also permits sufficient colonic mobilisation for
anastomosis.

12
13
14

Colon anatomy
The colon commences with the caecum. This represents the most dilated
segment of the human colon and its base (which is intraperitoneal) is marked by
the convergence of teniae coli. At this point is located the vermiform appendix.
The colon continues as the ascending colon, the posterior aspect of which is
retroperitoneal. The line of demarcation between the intra and retro peritoneal
right colon is visible as a white line, in the living, and forms the line of incision for
colonic resections.
The ascending colon becomes the transverse colon after passing the hepatic
flexure. At this located the colon becomes wholly intra peritoneal once again. The
superior aspect of the transverse colon is the point of attachment of the
transverse colon to the greater omentum. This is an important anatomical site
since division of these attachments permits entry into the lesser sac. Separation
of the greater omentum from the transverse colon is a routine operative step in
both gastric and colonic resections.
At the left side of the abdomen the transverse colon passes to the left upper
quadrant and makes an oblique inferior turn at the splenic flexure. Following this,
the posterior aspect becomes retroperitoneal once again.
At the level of approximately L4 the descending colon becomes wholly
intraperitoneal and becomes the sigmoid colon. Whilst the sigmoid is wholly
intraperitoneal there are usually attachments laterally between the sigmoid and
the lateral pelvic sidewall. These small congenital adhesions are not formal
anatomical attachments but frequently require division during surgical resections.
At its distal end the sigmoid passes to the midline and at the region around the
sacral promontary it becomes the upper rectum. This transition is visible
macroscopically as the point where the teniae fuse. More distally the rectum
passes through the peritoneum at the region of the peritoneal reflection and
becomes extraperitoneal.
Arterial supply
Superior mesenteric artery and inferior mesenteric artery: linked by the marginal
artery.
Ascending colon: ileocolic and right colic arteries
Transverse colon: middle colic artery
Descending and sigmoid colon: inferior mesenteric artery

http://www.emrcs.com/question/question.php?q=0

1/2

16/05/2015

Venous drainage
From regional veins (that accompany arteries) to superior and inferior mesenteric
vein
Lymphatic drainage
Initially along nodal chains that accompany supplying arteries, then para-aortic
nodes.
Embryology
Midgut- Second part of duodenum to 2/3 transverse colon
Hindgut- Distal 1/3 transverse colon to anus
Peritoneal location
The right and left colon are part intraperitoneal and part extraperitoneal. The
sigmoid and transverse colon are generally wholly intraperitoneal. This has
implications for the sequelae of perforations, which will tend to result in
generalised peritonitis in the wholly intra peritoneal segments.
Colonic relations
Region of colon

Relation

Caecum/ right colon

Right ureter, gonadal vessels

Hepatic flexure

Gallbladder (medially)

Splenic flexure

Spleen and tail of pancreas

Distal sigmoid/ upper


rectum

Left ureter

Rectum

Ureters, autonomic nerves, seminal vesicles, prostate,


urethra (distally)

Rate question:

Next question

Comment on this question

All contents of this site are 2012 E-Medical Revision Ltd

http://www.emrcs.com/question/question.php?q=0

Terms and Conditions

Privacy policy

2/2

16/05/2015

Reference ranges

Previous

Question 15 of 437

Next

A 43 year old lady is due to undergo an axillary node clearance as part of


treatment for carcinoma of the breast. Which of the following fascial layers will be
divided during the surgical approach to the axilla?

Question stats

Score: 91.3%
1

13.2%

8.7%

10%

60.2%

7.9%

A. Sibsons fascia
60.2% of users answered this
question correctly

B. Pre tracheal fascia

End and review

6
7

C. Waldayers fascia
8

D. Clavipectoral fascia
9

Search

E. None of the above

Go

Next question

10
11
12

The clavipectoral fascia is situated under the clavicular portion of pectoralis


major. It protects both the axillary vessels and nodes. During an axillary node
clearance for breast cancer the clavipectoral fascia is incised and this allows
access to the nodal stations. The nodal stations are; level 1 nodes inferior to
pectoralis minor, level 2 lie behind it and level 3 above it. During a Patey
Mastectomy surgeons divide pectoralis minor to gain access to level 3 nodes. The
use of sentinel node biopsy (and stronger assistants!) have made this procedure
far less common.

13
14
15

Axilla
Boundaries of the axilla
Medially

Chest wall and Serratus anterior

Laterally

Humeral head

Floor

Subscapularis

Anterior aspect

Lateral border of Pectoralis major

Fascia

Clavipectoral fascia

Content:
Long thoracic
nerve (of Bell)

Derived from C5-C7 and passes behind the brachial plexus to


enter the axilla. It lies on the medial chest wall and supplies
serratus anterior. Its location puts it at risk during axillary
surgery and damage will lead to winging of the scapula.

Thoracodorsal
nerve and
thoracodorsal
trunk

Innervate and vascularise latissimus dorsi.

Axillary vein

Lies at the apex of the axilla, it is the continuation of the


basilic vein. Becomes the subclavian vein at the outer border
of the first rib.

Intercostobrachial
nerves

Traverse the axillary lymph nodes and are often divided during
axillary surgery. They provide cutaneous sensation to the
axillary skin.

Lymph nodes

The axilla is the main site of lymphatic drainage for the


breast.

Rate question:

http://www.emrcs.com/question/question.php?q=0

Next question

1/2

16/05/2015

Comment on this question

All contents of this site are 2012 E-Medical Revision Ltd

http://www.emrcs.com/question/question.php?q=0

Terms and Conditions

Privacy policy

2/2

16/05/2015

Reference ranges

Previous

Question 16 of 437

Next

What are the boundaries of the 'safe triangle' for chest drain insertion?

A. Bounded by trapezius, latissimus dorsi, and laterally by the vertebral


border of the scapula
B. Bounded by latissimus dorsi, pectoralis major, line superior to the
nipple and apex at the axilla

Question stats

Score: 92%
1

10.9%

54.7%

19.8%

7.8%

6.7%

54.7% of users answered this


question correctly

C. Bounded by latissimus dorsi, serratus anterior, line superior to the


nipple and apex at the axilla
D. Bounded by trapezius, deltoid, rhomboid major and teres minor

End and review

6
7
8
9

Search

E. Bounded by trapezius, deltoid and latissimus dorsi

Go

10
11

Next question

12
13

Theme from April 2012 Exam

14

Chest drains

15
16

There are a number of different indications for chest drain insertion. In general
terms large bore chest drains are preferred for trauma and haemothorax
drainage. Smaller diameter chest drains can be used for pneumothorax or pleural
effusion drainage.
Insertion can be performed either using anatomical guidance or through
ultrasound guidance. In the exam, the anatomical method is usually tested.
It is advised that chest drains are placed in the 'safe triangle'. The triangle is
located in the mid axillary line of the 5th intercostal space. It is bordered by:
Anterior edge latissimus dorsi, the lateral border of pectoralis major, a line
superior to the horizontal level of the nipple, and the apex below the axilla.
Another triangle is situated behind the scapula. It is bounded above by the
trapezius, below by the latissimus dorsi, and laterally by the vertebral border of
the scapula; the floor is partly formed by the rhomboid major. If the scapula is
drawn forward by folding the arms across the chest, and the trunk bent forward,
parts of the sixth and seventh ribs and the interspace between them become
subcutaneous and available for auscultation. The space is therefore known as
the triangle of auscultation.
References
Prof Harold Ellis. The applied anatomy of chest drains insertions. British Journal
of hospital medicine 2007; (68): 44-45.
Laws D, Neville E, Duffy J. BTS guidelines for insertion of chest drains. Thorax,
2003; (58): 53-59.
Rate question:

Next question

Comment on this question

All contents of this site are 2012 E-Medical Revision Ltd

http://www.emrcs.com/question/question.php?q=0

Terms and Conditions

Privacy policy

1/1

16/05/2015

Reference ranges

Previous

Question 17 of 437

Next

Question stats

End and review

Score: 92.3%
1

14.2%

17.9%

19.5%

A. Transverse process of C6

15.9%

B. Transverse process of the axis

32.5%

C. Vertebral canal

32.5% of users answered this


question correctly

The vertebral artery traverses all of the following except?

D. Foramen magnum
E. Intervertebral foramen

6
7
8
9

Search
Next question
Go

10
11

The vertebral artery passes through the foramina which are located in the
transverse processes of the cervical vertebra, it does not traverse the
intervertebral foramen.

12
13
14

Vertebral artery

15
16

The vertebral artery is the first branch of the subclavian artery. Anatomically it is
divisible into 4 regions:

17

The first part runs to the foramen in the transverse process of C6. Anterior
to this part lies the vertebral and internal jugular veins. On the left side the
thoracic duct is also an anterior relation.
The second part runs superiorly through the foramina of the the transverse
processes of the upper 6 cervical vertebrae. Once it has passed through
the transverse process of the axis it then turns superolaterally to the atlas.
It is accompanied by a venous plexus and the inferior cervical sympathetic
ganglion.
The third part runs posteromedially on the lateral mass of the atlas. It
enters the sub occipital triangle, in the groove of the upper surface of the
posterior arch of the atlas. It then passes anterior to the edge of the
posterior atlanto-occipital membrane to enter the vertebral canal.
The fourth part passes through the spinal dura and arachnoid, running
superiorly and anteriorly at the lateral aspect of the medulla oblongata. At
the lower border of the pons it unites to form the basilar artery.

Rate question:

Next question

Comment on this question

All contents of this site are 2012 E-Medical Revision Ltd

http://www.emrcs.com/question/question.php?q=0

Terms and Conditions

Privacy policy

1/1

16/05/2015

Reference ranges

Previous

Question 18 of 437

Next

A 60 year old female attends the preoperative hernia clinic. She reports some
visual difficulty. On examination she is noted to have a homonymous hemianopia.
Where is the lesion most likely to be?

Question stats

Score: 85.7%
1

6.7%

15%

11.7%

26.7%

39.9%

A. Frontal lobe
39.9% of users answered this
question correctly

B. Pituitary gland

End and review

6
7

C. Parietal lobe
8

D. Optic chiasm
9

Search

E. Optic tract

Go

Next question

10
11
12

Lesions before optic chiasm:


Monocular vision loss = Optic nerve lesion
Bitemporal hemianopia = Optic chiasm lesion
Lesions after the optic chiasm:
Homonymous hemianopia = Optic tract lesion
Upper quadranopia = Temporal lobe lesion
Lower quadranopia = Parietal lobe lesion

13
14
15
16
17
18

Theme from April 2012 exam


Theme from April 2014 exam
Unfortunately we thought as surgeons we could forget about visual field defects!
However the college seem to like them. Remember a homonymous hemianopia is
indicative of an optic tract lesion. Parietal lobe lesions tend to cause inferior
quadranopias and there is a bitemporal hemianopia with optic chiasm lesion or
pituitary tumours.

Visual field defects


Theme from January 2012 exam
Theme from April 2012 exam

left homonymous hemianopia means visual field defect to the left, i.e.
Lesion of right optic tract
homonymous quadrantanopias: PITS (Parietal-Inferior, Temporal-Superior)
incongruous defects = optic tract lesion; congruous defects = optic
radiation lesion or occipital cortex

Homonymous hemianopia
Incongruous defects: lesion of optic tract
Congruous defects: lesion of optic radiation or occipital cortex
Macula sparing: lesion of occipital cortex

Homonymous quadrantanopias
Superior: lesion of temporal lobe
Inferior: lesion of parietal lobe
Mnemonic = PITS (Parietal-Inferior, Temporal-Superior)

Bitemporal hemianopia
http://www.emrcs.com/question/question.php?q=0

1/2

16/05/2015

Lesion of optic chiasm


Upper quadrant defect > lower quadrant defect = inferior chiasmal
compression, commonly a pituitary tumour
Lower quadrant defect > upper quadrant defect = superior chiasmal
compression, commonly a craniopharyngioma

Rate question:

Next question

Comment on this question

All contents of this site are 2012 E-Medical Revision Ltd

http://www.emrcs.com/question/question.php?q=0

Terms and Conditions

Privacy policy

2/2

16/05/2015

Reference ranges

Question 19 of 437

Previous

Next

A 34 year old male is being examined in the pre-operative assessment clinic. A


murmur is identified in the 4th intercostal space just next to the left side of the
sternum. From where is it most likely to have originated?

Question stats

Score: 80%
1

24.7%

10.4%

8.2%

5.4%

51.2%

A. Mitral valve
51.2% of users answered this
question correctly

B. Aortic valve

End and review

6
7

C. Pulmonary valve
8

D. Right ventricular aneurysm


9

Search

E. Tricuspid valve

Go

10
11

Next question

12

Theme from September 2012 Exam


The tricuspid valve is generally referred to being best auscultated adjacent to the
sternum. The plane of projected sound from the mitral area is best heard in the
region of the cadiac apex.

13
14
15
16

Heart sounds

17
18

Sites of auscultation

19

Valve

Site

Pulmonary valve

Left second intercostal space, at the upper sternal border

Aortic valve

Right second intercostal space, at the upper sternal border

Mitral valve

Left fifth intercostal space, just medial to mid clavicular line

Tricuspid valve

Left fifth intercostal space, at the lower left sternal border

The diagram below demonstrates where the various cardiac valves are best
heard.

Image sourced from Wikipedia

Rate question:

Next question

Comment on this question

All contents of this site are 2012 E-Medical Revision Ltd

http://www.emrcs.com/question/question.php?q=0

Terms and Conditions

Privacy policy

1/1

16/05/2015

Reference ranges

Previous

Question 20 of 437

Next

During an Ivor Lewis Oesophagectomy for carcinoma of the lower third of the
oesophagus which structure is divided to allow mobilisation of the oesophagus?

A. Vagus nerve
B. Azygos vein

Question stats

Score: 80.6%
1

19.1%

40.3%

12.2%

11.2%

17.2%

40.3% of users answered this


question correctly

C. Right inferior lobar bronchus

End and review

D. Phrenic nerve

6
7
8

E. Pericardiophrenic artery

Search
Next question

Go

10
11

Theme from April 2014 exam


The azygos vein is routinely divided during an oesophagectomy to allow
mobilisation. It arches anteriorly to insert into the SVC on the right hand side.

12

Treatment of oesophageal cancer

15

13
14

16

In general resections are not offered to those patients with distant


metastasis, and usually not to those with N2 disease.
Local nodal involvement is not in itself a contra indication to resection.
Surgical resection is the mainstay of treatment.
Neoadjuvent chemotherapy is given in most cases prior to surgery.
In situ disease may be managed by endoscopic mucosal resection,
although this is still debated.
In patients with lower third lesions an Ivor - Lewis type procedure is most
commonly performed. Very distal tumours may be suitable to a transhiatal
procedure. Which is an attractive option as the penetration of two visceral
cavities required for an Ivor- Lewis type procedure increases the morbidity
considerably.
More proximal lesions will require a total oesphagectomy (Mckeown type)
with anastomosis to the cervical oesophagus.
Patients with unresectable disease may derive benefit from local ablative
procedures, palliative chemotherapy or stent insertion.

17
18
19
20

Operative details of Ivor- Lewis procedure


Combined laparotomy and right thoracotomy

Indication
Lower and middle third oesophageal tumours

Preparation
Staging with a combination of CT chest abdomen and pelvis- if no
metastatic disease detected then patients will undergo a staging
laparoscopy to detect peritoneal disease.
If both these modalities are negative then patients will finally undergo a
PET CT scan to detect occult metastatic disease. Only in those whom no
evidence of advanced disease is detected will proceed to resection.
Patients receive a GA, double lumen endotracheal tube to allow for lung
deflation, CVP and arterial monitoring.

Procedure
A rooftop incision is made to access the stomach and duodenum.

http://www.emrcs.com/question/question.php?q=0

1/2

16/05/2015

Laparotomy To mobilize the stomach


The greater omentum is incised away from its attachment to the right
gastroepiploic vessels along the greater curvature of the stomach.
Then the short gastric vessels are ligated and detached from the greater
curvature from the spleen.
The lesser omentum is incised, preserving the right gastric artery.
The retroperitoneal attachments of the duodenum in its second and third
portions are incised, allowing the pylorus to reach the oesophageal hiatus.
Some surgeons perform a pyloroplasty at this point to facilitate gastric
emptying.
The left gastric vessels are then ligated, avoiding any injury to the common
hepatic or splenic arteries. Care must be taken to avoid inadvertently
devascularising the liver owing to variations in anatomy.

Right Thoracotomy Oesophageal resection and oesophagogastric anastomosis


Through 5th intercostal space
Dissection performed 10cm above the tumour
This may involve transection of the azygos vein.
The oesophagus is then removed with the stomach creating a gastric tube.
An anastomosis is created.

The chest is closed with underwater seal drainage and tube drains to the
abdominal cavity.
Post operatively
Patients will typically recover in ITU initially.
A nasogastric tube will have been inserted intraoperatively and must
remain in place during the early phases of recovery.
Post operatively these patients are at relatively high risk of developing
complications:

* Atelectasis- due to the effects of thoracotomy and lung collapse


* Anastomotic leakage. The risk is relatively high owing to the presence of a
relatively devascularised stomach. Often the only blood supply is from the
gastroepiploic artery as all others will have been divided. If a leak does occur then
many will attempt to manage conservatively with prolonged nasogastric tube
drainage and TPN. The reality is that up to 50% of patients developing an
anastomotic leak will not survive to discharge.
* Delayed gastric emptying (may be avoided by performing a pyloroplasty).
Rate question:

Next question

Comment on this question

All contents of this site are 2012 E-Medical Revision Ltd

http://www.emrcs.com/question/question.php?q=0

Terms and Conditions

Privacy policy

2/2

16/05/2015

Reference ranges

Question 21 of 437

Previous

Next

Question stats

End and review

Score: 78.1%
1

36.3%

18.5%

15.4%

A. Causes flexion of the thoracic spine

13.6%

B. Causes the rib cage to be pulled down

16.2%

C. Innervated by anterior primary rami of T12 and L1-3

36.3% of users answered this


question correctly

Which of the following statements relating to quadratus lumborum is false?

D. Attached to the iliac crest


E. Inserts into the 12th rib

6
7
8
9

Search
Next question
Go

10
11

Quadratus lumborum
Origin: Medial aspect of iliac crest and iliolumbar ligament
Insertion: 12th rib
Action: Pulls the rib cage inferiorly. Lateral flexion.
Nerve supply: Anterior primary rami of T12 and L1-3

12
13
14
15
16

The rectus abdominis causes flexion of the thoracic spine and therefore the
statement suggesting that quaratus lumborum does so is incorrect.

17

Abdominal wall

19

18

20

The 2 main muscles of the abdominal wall are the rectus abdominis (anterior)
and the quadratus lumborum (posterior).
The remaining abdominal wall consists of 3 muscular layers. Each muscle passes
from the lateral aspect of the quadratus lumborum posteriorly to the lateral margin
of the rectus sheath anteriorly. Each layer is muscular posterolaterally and
aponeurotic anteriorly.

21

Image sourced from Wikipedia

Muscles of abdominal wall


External
oblique

Internal
oblique

Lies most superficially


Originates from 5th to 12th ribs
Inserts into the anterior half of the outer aspect of the iliac
crest, linea alba and pubic tubercle
More medially and superiorly to the arcuate line, the
aponeurotic layer overlaps the rectus abdominis muscle
The lower border forms the inguinal ligament
The triangular expansion of the medial end of the inguinal
ligament is the lacunar ligament.

Arises from the thoracolumbar fascia, the anterior 2/3 of the


iliac crest and the lateral 2/3 of the inguinal ligament
The muscle sweeps upwards to insert into the cartilages of
the lower 3 ribs
The lower fibres form an aponeurosis that runs from the tenth
costal cartilage to the body of the pubis
At its lowermost aspect it joins the fibres of the aponeurosis
of transversus abdominis to form the conjoint tendon.

Transversus
http://www.emrcs.com/question/question.php?q=0

1/2

16/05/2015

Transversus
abdominis

Innermost muscle
Arises from the inner aspect of the costal cartilages of the
lower 6 ribs , from the anterior 2/3 of the iliac crest and lateral
1/3 of the inguinal ligament
Its fibres run horizontally around the abdominal wall ending in
an aponeurosis. The upper part runs posterior to the rectus
abdominis. Lower down the fibres run anteriorly only.
The rectus abdominis lies medially; running from the pubic
crest and symphysis to insert into the xiphoid process and
5th, 6th and 7th costal cartilages. The muscles lies in a
aponeurosis as described above.
Nerve supply: anterior primary rami of T7-12

Surgical notes
During abdominal surgery it is usually necessary to divide either the muscles or
their aponeuroses. During a midline laparotomy it is desirable to divide the
aponeurosis. This will leave the rectus sheath intact above the arcuate line and
the muscles intact below it. Straying off the midline will often lead to damage to
the rectus muscles, particularly below the arcuate line where they may often be in
close proximity to each other.
Rate question:

Next question

Comment on this question

All contents of this site are 2012 E-Medical Revision Ltd

http://www.emrcs.com/question/question.php?q=0

Terms and Conditions

Privacy policy

2/2

16/05/2015

Reference ranges

Question 22 of 437

Previous

Next

A 23 year old climber falls and fractures his humerus. The surgeons decide upon
a posterior approach to the middle third of the bone. Which of the following
nerves is at greatest risk in this approach?

Question stats

Score: 78.8%
1

8.2%

6.3%

11.2%

66.4%

7.9%

A. Ulnar
66.4% of users answered this
question correctly

B. Antebrachial

End and review

6
7

C. Musculocutaneous
8

D. Radial
9

Search

E. Intercostobrachial

Go

Next question

10
11
12

Theme from April 2012 Exam


The radial nerve wraps around the humerus and may be injured during a
posterior approach. An IM nail may be preferred as it avoids the complex
dissection needed for direct bone exposure.

13
14
15
16

Radial nerve
Continuation of posterior cord of the brachial plexus (root values C5 to T1)

17
18
19

Path

20

In the axilla: lies posterior to the axillary artery on subscapularis, latissimus


dorsi and teres major.
Enters the arm between the brachial artery and the long head of triceps
(medial to humerus).
Spirals around the posterior surface of the humerus in the groove for the
radial nerve.
At the distal third of the lateral border of the humerus it then pierces the
intermuscular septum and descends in front of the lateral epicondyle.
At the lateral epicondyle it lies deeply between brachialis and
brachioradialis where it then divides into a superficial and deep terminal
branch.
Deep branch crosses the supinator to become the posterior interosseous
nerve.

21
22

In the image below the relationships of the radial nerve can be appreciated

Image sourced from Wikipedia

Regions innervated
http://www.emrcs.com/question/question.php?q=0

1/2

16/05/2015

Motor (main
nerve)

Triceps
Anconeus
Brachioradialis
Extensor carpi radialis

Motor
(posterior
interosseous
branch)

Supinator
Extensor carpi ulnaris
Extensor digitorum
Extensor indicis
Extensor digiti minimi
Extensor pollicis longus and brevis
Abductor pollicis longus

Sensory

The area of skin supplying the proximal phalanges on the dorsal


aspect of the hand is supplied by the radial nerve (this does not
apply to the little finger and part of the ring finger)

Muscular innervation and effect of denervation


Anatomical
location

Muscle affected

Effect of paralysis

Shoulder

Long head of
triceps

Minor effects on shoulder stability in


abduction

Arm

Triceps

Loss of elbow extension

Forearm

Supinator
Brachioradialis
Extensor carpi
radialis longus
and brevis

Weakening of supination of prone


hand and elbow flexion in mid prone
position

The cutaneous sensation of the upper limb- illustrating the contribution of the
radial nerve

Image sourced from Wikipedia

Rate question:

Next question

Comment on this question

All contents of this site are 2012 E-Medical Revision Ltd

http://www.emrcs.com/question/question.php?q=0

Terms and Conditions

Privacy policy

2/2

16/05/2015

Reference ranges

Previous3 / 3

Question 23-25 of 437

Next

Question stats

End and review

Score: 82.1%

Average score for registered users:

1
2

Theme: Nerve injury


A. Median nerve

23

66.2%

24

57.9%

25

46%

B. Ulnar nerve

3
4
5

C. Radial nerve

D. Posterior interosseous nerve

Search

E. Anterior interosseous nerve

7
Go

F. Musculocutaneous nerve

8
9

G. Axillary nerve
10

H. Brachial Trunks C5-6


11

I. Brachial trunks C6-7

12

J. Brachial Trunks C8-T1

13

Please select the most likely lesion site for each scenario. Each option may be
used once, more than once or not at all.

14
15
16

23.

A 42 year old teacher is admitted with a fall. An x-ray confirms a


fracture of the surgical neck of the humerus. Which nerve is at risk?

17
18
19

Axillary nerve

20

The Axillary nerve winds around the bone at the neck of the
humerus. The axillary nerve is also at risk during shoulder
dislocation.

21
22
23-25 3 / 3

24.

A 32 year old window cleaner is admitted after falling off the roof. He
reports that he had slipped off the top of the roof and was able to
cling onto the gutter for a few seconds. The patient has Horner's
syndrome.
Brachial Trunks C8-T1
The patient has a Klumpke's paralysis involving brachial trunks C8T1. Classically there is weakness of the hand intrinsic muscles.
Involvement of T1 may cause a Horner's syndrome. It occurs as a
result of traction injuries or during delivery.

25.

A 32 year old rugby player is hit hard on the shoulder during a


rough tackle. Clinically his arm is hanging loose on the side. It is
pronated and medially rotated.
Brachial Trunks C5-6
The patient has an Erb's palsy involving brachial trunks C5-6.

Next question

Brachial plexus

Origin
Sections of the
plexus

Anterior rami of C5 to T1
Roots, trunks, divisions, cords, branches
Mnemonic:Real Teenagers Drink Cold Beer

http://www.emrcs.com/question/question.php?q=0

1/2

16/05/2015

Roots

Located in the posterior triangle


Pass between scalenus anterior and medius

Trunks

Located posterior to middle third of clavicle


Upper and middle trunks related superiorly to the
subclavian artery
Lower trunk passes over 1st rib posterior to the
subclavian artery

Divisions

Apex of axilla

Cords

Related to axillary artery

Diagram illustrating the branches of the brachial plexus

Image sourced from Wikipedia

Cutaneous sensation of the upper limb

Image sourced from Wikipedia

Rate question:

Next question

Comment on this question

All contents of this site are 2012 E-Medical Revision Ltd

http://www.emrcs.com/question/question.php?q=0

Terms and Conditions

Privacy policy

2/2

16/05/2015

Reference ranges

Question 26 of 437

Previous

Next

A 68 year old man with critical limb ischaemia is undergoing a femoro-distal


bypass graft. During mobilisation of the proximal part of the posterior tibial artery
which of the following is at greatest risk of injury?

Question stats

Score: 82.9%
1

45.1%

7.4%

14.4%

25.5%

7.5%

A. Tibial nerve
45.1% of users answered this
question correctly

B. Sciatic nerve

End and review

6
7

C. Saphenous nerve
8

D. Common peroneal nerve


9

Search

E. Medial superior genicular artery

Go

10
11

Next question

12

The tibial nerve is closely related to the posterior tibial artery. The tibial nerve
crosses the vessel posteriorly approximately 2.5cm distal to its origin. At its origin
the nerve lies medial and then lateral after it crosses the vessel as described.

13
14
15

Posterior tibial artery

16
17
18

Larger terminal branch of the popliteal artery


Terminates by dividing into the medial and lateral plantar arteries
Accompanied by two veins throughout its length
Position of the artery corresponds to a line drawn from the lower angle of
the popliteal fossa, at the level of the neck of the fibula, to a point midway
between the medial malleolus and the most prominent part of the heel

19
20
21
22
23-25 3 / 3
26

Relations of the posterior tibial artery


Proximal to distal
Anteriorly

Tibialis posterior
Flexor digitorum longus
Posterior surface of tibia and ankle joint

Posterior

Tibial nerve 2.5 cm distal to its origin


Fascia overlying the deep muscular layer
Proximal part covered by gastrocnemius and soleus
Distal part covered by skin and fascia

Rate question:

Next question

Comment on this question

All contents of this site are 2012 E-Medical Revision Ltd

http://www.emrcs.com/question/question.php?q=0

Terms and Conditions

Privacy policy

1/1

16/05/2015

Reference ranges

Previous

Question 27 of 437

Next

A 67 year old man is undergoing an angiogram for gastro intestinal bleeding. The
radiologist advances the catheter into the coeliac axis. At what spinal level does
this vessel typically arise from the aorta?

Question stats

Score: 83.7%
1

13.8%

10%

6.8%

56.9%

12.6%

A. T10
56.9% of users answered this
question correctly

B. L3

End and review

6
7

C. L4
8

D. T12
9

Search

E. None of the above

Go

10
11

Next question

12

Theme from April 2014 exam


The coeliac axis lies at T12, it takes an almost horizontal angle off the aorta. It
has three major branches.

13
14
15

Abdominal aortic branches

16
17

Branches

Level

Paired

Type

Inferior phrenic

T12 (Upper border)

Yes

Parietal

Coeliac

T12

No

Visceral

Superior mesenteric

L1

No

Visceral

Middle suprarenal

L1

Yes

Visceral

Renal

L1-L2

Yes

Visceral

Gonadal

L2

Yes

Visceral

Lumbar

L1-L4

Yes

Parietal

Inferior mesenteric

L3

No

Visceral

Median sacral

L4

No

Parietal

Common iliac

L4

Yes

Terminal

18
19
20
21
22
23-25 3 / 3

Rate question:

26
27

Next question

Comment on this question

All contents of this site are 2012 E-Medical Revision Ltd

http://www.emrcs.com/question/question.php?q=0

Terms and Conditions

Privacy policy

1/1

16/05/2015

Reference ranges

Question 28 of 437

Previous

Next

Which of the following muscles does not insert to the medial surface of the
greater trochanter?

A. Gemelli
B. Obturator internus

Question stats

Score: 81.8%
1

13.4%

14.1%

18.1%

33%

21.4%

33% of users answered this


question correctly

C. Piriformis

End and review

D. Quadratus femoris

6
7
8

E. Obturator externus

Search
Next question

Go

10
11
12

Mnemonic for muscle attachment on greater trochanter is POGO:


Piriformis
Obturator internus
Gemelli
Obturator externus

13
14
15
16
17
18

The quadratus femoris fibres pass laterally to be inserted into the quadrate
tubercle on the intertrochanteric crest of the femur. The other muscles all insert
on the trochanteric fossa lying medial to the greater trochanter.

19
20
21

Gluteal region

22
23-25 3 / 3

Gluteal muscles

26

Gluteus maximus: inserts to gluteal tuberosity of the femur and iliotibial tract
Gluteus medius: attach to lateral greater trochanter
Gluteus minimis: attach to anterior greater trochanter
All extend and abduct the hip

27
28

Deep lateral hip rotators


Piriformis
Gemelli
Obturator internus
Quadratus femoris

Nerves
Superior gluteal nerve (L5, S1)

Inferior gluteal nerve

Gluteus medius
Gluteus minimis
Tensor fascia lata
Gluteus maximus

Damage to the superior gluteal nerve will result in the patient developing a
Trendelenberg gait. Affected patients are unable to abduct the thigh at the hip
joint. During the stance phase, the weakened abductor muscles allow the pelvis to
tilt down on the opposite side. To compensate, the trunk lurches to the weakened
side to attempt to maintain a level pelvis throughout the gait cycle. The pelvis
sags on the opposite side of the lesioned superior gluteal nerve.
Rate question:

Next question

Comment on this question

http://www.emrcs.com/question/question.php?q=0

1/2

16/05/2015

All contents of this site are 2012 E-Medical Revision Ltd

http://www.emrcs.com/question/question.php?q=0

Terms and Conditions

Privacy policy

2/2

16/05/2015

Reference ranges

Previous

Question 29 of 437

Next

During a radical gastrectomy for carcinoma of the stomach the surgeons remove
the omentum. What is the main source of its blood supply?

A. Ileocolic artery
B. Superior mesenteric artery

Question stats

Score: 82.2%
1

8.1%

19.6%

55.4%

10%

6.9%

55.4% of users answered this


question correctly

C. Gastroepiploic artery

End and review

D. Middle colic artery

6
7
8

E. Inferior mesenteric artery

Search
Go

Next question

10
11
12

The vessels supplying the omentum are the omental branches of the right and left
gastro-epiploic arteries. The colonic vessels are not responsible for the arterial
supply to the omentum. The left gastro-epiploic artery is a branch of the splenic
artery and the right gastro-epiploic artery is a terminal branch of the
gastroduodenal artery.

13
14
15
16

Omentum

17
18

The omentum is divided into two parts which invest the stomach. Giving rise
to the greater and lesser omentum. The greater omentum is attached to
the inferolateral border of the stomach and houses the gastro-epiploic
arteries.
It is of variable size but is less well developed in children. This is important
as the omentum confers protection against visceral perforation (e.g.
Appendicitis).
Inferiorly between the omentum and transverse colon is one potential entry
point into the lesser sac.
Several malignant processes may involve the omentum of which ovarian
cancer is the most notable.

19
20
21
22
23-25 3 / 3
26
27
28
29

Rate question:

Next question

Comment on this question

All contents of this site are 2012 E-Medical Revision Ltd

http://www.emrcs.com/question/question.php?q=0

Terms and Conditions

Privacy policy

1/1

16/05/2015

Reference ranges

Question 30 of 437

Previous

Next

A 38 year old lady is due to undergo a parathyroidectomy for


hyperparathyroidism. At operation the inferior parathyroid gland is identified as
being enlarged. A vessel is located adjacent to the gland laterally. This vessel is
most likely to be the:

Question stats

Score: 82.6%
1

18.9%

37.1%

18.8%

16.3%

8.8%

37.1% of users answered this


question correctly

A. External carotid artery

End and review

B. Common carotid artery

6
7
8

C. Internal carotid artery

Search

D. External jugular vein


E. None of the above

Go

10
11

Next question

12
13

The common carotid artery is a lateral relation of the inferior parathyroid.

14
15

Parathyroid glands- anatomy

16
17

Four parathyroid glands


Located posterior to the thyroid gland
They lie within the pretracheal fascia

18
19
20

Embryology
The parathyroids develop from the extremities of the third and fourth pharyngeal
pouches. The parathyroids derived from the fourth pharyngeal pouch are located
more superiorly and are associated with the thyroid gland. Those derived from
the third pharyngeal pouch lie more inferiorly and may become associated with
the thymus.

21
22
23-25 3 / 3
26
27
28

Blood supply
The blood supply to the parathyroid glands is derived from the inferior and
superior thyroid arteries[1]. There is a rich anastomosis between the two vessels.
Venous drainage is into the thyroid veins.

29
30

Relations
Laterally

Common carotid

Medially

Recurrent laryngeal nerve, trachea

Anterior

Thyroid

Posterior

Pretracheal fascia

References
1.Nobori, M., et al., Blood supply of the parathyroid gland from the superior
thyroid artery. Surgery, 1994. 115(4): p. 417-23.

Rate question:

Next question

Comment on this question

All contents of this site are 2012 E-Medical Revision Ltd


http://www.emrcs.com/question/question.php?q=0

Terms and Conditions

Privacy policy
1/2

16/05/2015

http://www.emrcs.com/question/question.php?q=0

2/2

16/05/2015

Reference ranges

Question 31 of 437

Previous

Next

A 45 year old man has a long femoral line inserted to provide CVP
measurements. The catheter passes from the common iliac vein into the inferior
vena cava. At which of the following vertebral levels will this occur?

Question stats

Score: 83.3%
1

38.7%

30.8%

7.8%

13.1%

9.6%

A. L5
38.7% of users answered this
question correctly

B. L4

End and review

6
7

C. S1
8

D. L3
9

Search

E. L2

Go

Next question

10
11
12

The common iliac veins fuse with the IVC at L5.


Inferior vena cava

13
14
15
16

Origin

17

L5

18
19

Path

20

Left and right common iliac veins merge to form the IVC.
Passes right of midline
Paired segmental lumbar veins drain into the IVC throughout its length
The right gonadal vein empties directly into the cava and the left gonadal
vein generally empties into the left renal vein.
The next major veins are the renal veins and the hepatic veins
Pierces the central tendon of diaphragm at T8
Right atrium

21
22
23-25 3 / 3
26
27
28
29
30
31

Image sourced from Wikipedia

Relations
Anteriorly

Small bowel, first and third part of duodenum, head of pancreas, liver
and bile duct, right common iliac artery, right gonadal artery

Posteriorly

Right renal artery, right psoas, right sympathetic chain, coeliac


ganglion

Levels
Level

Vein

http://www.emrcs.com/question/question.php?q=0

1/2

16/05/2015

T8

Hepatic vein, inferior phrenic vein, pierces diaphragm

L1

Suprarenal veins, renal vein

L2

Gonadal vein

L1-5

Lumbar veins

L5

Common iliac vein, formation of IVC

Rate question:

Next question

Comment on this question

All contents of this site are 2012 E-Medical Revision Ltd

http://www.emrcs.com/question/question.php?q=0

Terms and Conditions

Privacy policy

2/2

16/05/2015

Reference ranges

Previous

Question 32 of 437

Next

Following a carotid endarterectomy a man notices that he has a weakness of his


tongue. Damage to which of the following nerves is the most likely explanation for
this process?

Question stats

Score: 83.7%
1

67.5%

8%

9.4%

8.5%

6.5%

A. Hypoglossal
67.5% of users answered this
question correctly

B. Accessory

End and review

6
7

C. Ansa cervicalis
8

D. Vagus
9

Search

E. Cervical plexus

Go

Next question

10
11
12

Theme from April 2013


The hypoglossal nerve innervates the tongue and is one of the structures more
commonly at risk in carotid surgery.

13
14
15

Internal carotid artery

16
17

The internal carotid artery is formed from the common carotid opposite the upper
border of the thyroid cartilage. It extends superiorly to enter the skull via the
carotid canal. From the carotid canal it then passes through the cavernous sinus,
above which it divides into the anterior and middle cerebral arteries.
Relations in the neck
Posterior

18
19
20
21
22

Longus capitis
Pre-vertebral fascia
Sympathetic chain
Superior laryngeal nerve

23-25 3 / 3
26
27
28

Medially

External carotid (near origin)


Wall of pharynx
Ascending pharyngeal artery

29
30
31

Laterally

Anteriorly

32

Internal jugular vein (moves posteriorly at entrance to skull)


Vagus nerve (most posterolaterally)

Sternocleidomastoid
Lingual and facial veins
Hypoglossal nerve

Relations in the carotid canal


Internal carotid plexus
Cochlea and middle ear cavity
Trigeminal ganglion (superiorly)
Leaves canal lies above the foramen lacerum

Path and relations in the cranial cavity


The artery bends sharply forwards in the cavernous sinus, the aducens nerve lies
close to its inferolateral aspect. The oculomotor, trochlear, opthalmic and, usually,
the maxillary nerves lie in the lateral wall of the sinus. Near the superior orbital
fissure it turns posteriorly and passes postero-medially to pierce the roof of the
cavernous sinus inferior to the optic nerve. It then passes between the optic and
oculomotor nerves to terminate below the anterior perforated substance by

http://www.emrcs.com/question/question.php?q=0

1/2

16/05/2015

dividing into the anterior and middle cerebral arteries.


Branches
Anterior and middle cerebral artery
Ophthalmic artery
Posterior communicating artery
Anterior choroid artery
Meningeal arteries
Hypophyseal arteries

Image demonstrating the internal carotid artery and its relationship to the external
carotid artery

Image sourced from Wikipedia

Rate question:

Next question

Comment on this question

All contents of this site are 2012 E-Medical Revision Ltd

http://www.emrcs.com/question/question.php?q=0

Terms and Conditions

Privacy policy

2/2

16/05/2015

Reference ranges

Previous

Question 33 of 437

Next

At which of the following levels does the inferior vena cava exit the abdominal
cavity?

A. T6
B. T7

Question stats

Score: 84.3%
1

7.2%

7.4%

13.9%

58.9%

12.6%

58.9% of users answered this


question correctly

C. T10

End and review

D. T8

6
7
8

E. T12

Search
Next question

Go

10
11
12

Theme from April 2012 exam


Theme from April 2014 exam

13
14

Levels

15
16

Transpyloric plane
Level of the body of L1

17
18

Pylorus stomach
Left kidney hilum (L1- left one!)
Right hilum of the kidney (1.5cm lower than the left)
Fundus of the gallbladder
Neck of pancreas
Duodenojejunal flexure
Superior mesenteric artery
Portal vein
Left and right colic flexure
Root of the transverse mesocolon
2nd part of the duodenum
Upper part of conus medullaris
Spleen

19
20
21
22
23-25 3 / 3
26
27
28
29
30
31

Can be identified by asking the supine patient to sit up without using their arms.
The plane is located where the lateral border of the rectus muscle crosses the
costal margin.

32
33

Anatomical planes
Subcostal plane

Lowest margin of 10th costal cartilage

Intercristal plane

Level of body L4 (highest point of iliac crest)

Intertubercular plane

Level of body L5

Common level landmarks


Inferior mesenteric artery

L3

Bifurcation of aorta into common iliac


arteries

L4

Formation of IVC

L5 (union of common iliac


veins)

Diaphragm apertures

http://www.emrcs.com/question/question.php?q=0

Vena cava T8
Oesophagus T10
Aortic hiatus T12

1/2

16/05/2015

Rate question:

Next question

Comment on this question

All contents of this site are 2012 E-Medical Revision Ltd

http://www.emrcs.com/question/question.php?q=0

Terms and Conditions

Privacy policy

2/2

16/05/2015

Reference ranges

Previous

Question 34 of 437

Next

Question stats

End and review

Score: 84.6%
1

57.3%

9.8%

11.6%

A. Popliteal artery

9.8%

B. Popliteal vein

11.5%

C. Tibial nerve

57.3% of users answered this


question correctly

Which of the following structures lies deepest in the popliteal fossa?

D. Common peroneal nerve


E. Popliteal lymph nodes

6
7
8
9

Search
Next question
Go

Theme from January 2012 exam


Theme from April 2014 exam

10
11
12
13

From superficial to deep:


The common peroneal nerve exits the popliteal fossa along the medial border of
the biceps tendon. Then the tibial nerve lies lateral to the popliteal vessels to
pass posteriorly and then medially to them. The popliteal vein lies superficial to
the popliteal artery, which is the deepest structure in the fossa.
Popliteal fossa

14
15
16
17
18
19

Boundaries of the popliteal fossa


Laterally

Biceps femoris above, lateral head of gastrocnemius and plantaris below

Medially

Semimembranosus and semitendinosus above, medial head of


gastrocnemius below

20
21
22
23-25 3 / 3
26

Floor

Popliteal surface of the femur, posterior ligament of knee joint and


popliteus muscle

Roof

Superficial and deep fascia

27
28

Image showing the popliteal fossa

29
30
31
32
33
34

Image sourced from Wikipedia

Contents
Popliteal artery and vein
Small saphenous vein
Common peroneal nerve
Tibial nerve
Posterior cutaneous nerve of the thigh
Genicular branch of the obturator nerve
Lymph nodes

http://www.emrcs.com/question/question.php?q=0

1/2

16/05/2015

Rate question:

Next question

Comment on this question

All contents of this site are 2012 E-Medical Revision Ltd

http://www.emrcs.com/question/question.php?q=0

Terms and Conditions

Privacy policy

2/2

16/05/2015

Reference ranges

Previous

Question 35 of 437

Next

Question stats

End and review

Score: 84.9%
1

62.5%

7.9%

12.7%

A. Radial

8.9%

B. Ulnar

8%

C. Axillary

62.5% of users answered this


question correctly

Which of the following nerves is responsible for innervation of the triceps muscle?

D. Median
E. None of the above

7
8
9

Search
Next question
Go

10
11

To remember nerve roots and their reflexes:

12
13

1-2 Ankle (S1-S2)


3-4 Knee (L3-L4)
5-6 Biceps (C5-C6)
7-8 Triceps (C7-C8)

14
15
16
17

The radial nerve innervates all three heads of triceps, with a separate branch to
each head.

18
19
20

Triceps

21

Origin

22

Long head- infraglenoid tubercle of the scapula.


Lateral head- dorsal surface of the humerus, lateral and
proximal to the groove of the radial nerve
Medial head- posterior surface of the humerus on the
inferomedial side of the radial groove and both of the
intermuscular septae

23-25 3 / 3
26
27
28
29

Insertion

30

Olecranon process of the ulna. Here the olecranon bursa is


between the triceps tendon and olecranon.
Some fibres insert to the deep fascia of the forearm, posterior
capsule of the elbow (preventing the capsule from being
trapped between olecranon and olecranon fossa during
extension)

31
32
33
34
35

Innervation

Radial nerve

Blood
supply

Profunda brachii artery

Action

Elbow extension. The long head can adduct the humerus and and
extend it from a flexed position

Relations

The radial nerve and profunda brachii vessels lie between the lateral
and medial heads

Rate question:

Next question

Comment on this question

All contents of this site are 2012 E-Medical Revision Ltd

http://www.emrcs.com/question/question.php?q=0

Terms and Conditions

Privacy policy

1/1

16/05/2015

Reference ranges

Question 36 of 437

Previous

Next

Which of the positions listed below best describes the location of the coeliac
autonomic plexus?

A. Anterolateral to the aorta


B. Posterolateral to the aorta

Question stats

Score: 83.3%
1

40.3%

20.3%

14.8%

16%

8.6%

40.3% of users answered this


question correctly

C. Anterolateral to the sympathetic chain

End and review

D. Anteromedial to the sympathetic chain

6
7
8

E. Posterior to L1

Search
Go

Next question

10
11
12
13

Coeliac plexus

14
15

The coeliac plexus is the largest of the autonomic plexuses. It is located on a level
of the last thoracic and first lumbar vertebrae. It surrounds the coeliac axis and
the SMA. It lies posterior to the stomach and the lesser sac. It lies anterior to the
crura of the diaphragm and the aorta. The plexus and ganglia are joined are
joined by the greater and lesser splanchnic nerves on both sides and branches
from both the vagus and phrenic nerves.

16
17
18
19
20
21
22
23-25 3 / 3
26
27
28
29
30
31
32

Image sourced from Wikipedia

33
34

Rate question:

Next question

35
36

Comment on this question

All contents of this site are 2012 E-Medical Revision Ltd

http://www.emrcs.com/question/question.php?q=0

Terms and Conditions

Privacy policy

1/1

16/05/2015

Reference ranges

Question 1 of 401

Next

An intravenous drug user develops a false aneurysm and requires emergency surgery. The
procedure is difficult and the femoral nerve is inadvertently transected. Which of the following
muscles is least likely to be affected as a result?

Question stats

End and review

Score: 100%

12.5%

10.2%

18.1%

14.5%

44.8%

A. Sartorius
44.8% of users answered this
question correctly

B. Vastus medialis
C. Pectineus
D. Quadriceps femoris

Search

E. Adductor magnus

Go

Next question

Mnemonic for femoral nerve supply


(don't) M I S V Q Scan for PE
M edial cutaneous nerve of the thigh
I ntermediate cutaneous nerve of the thigh
S aphenous nerve
V astus
Q uadriceps femoris
S artorius
PE ectineus

Adductor magnus is innervated by the obturator and sciatic nerve. The pectineus muscle is
sometimes supplied by the obturator nerve but this is variable. Since the question states
least likely, the correct answer is adductor magnus
Femoral nerve

Root values
Innervates

Branches

L2, 3, 4
Pectineus
Sartorius
Quadriceps femoris
Vastus lateralis/medialis/intermedius

Medial cutaneous nerve of thigh


Saphenous nerve
Intermediate cutaneous nerve of thigh

Path
Penetrates psoas major and exits the pelvis by passing under the inguinal ligament to enter
the femoral triangle, lateral to the femoral artery and vein.

http://www.emrcs.com/question/question.php?q=0

1/2

16/05/2015

Image sourced from Wikipedia

Mnemonic for femoral nerve supply


(don't) M I S V Q Scan for PE
M edial cutaneous nerve of the thigh
I ntermediate cutaneous nerve of the thigh
S aphenous nerve
V astus
Q uadriceps femoris
S artorius
PE ectineus

Rate question:

Next question

Comment on this question

All contents of this site are 2012 E-Medical Revision Ltd

http://www.emrcs.com/question/question.php?q=0

Terms and Conditions

Privacy policy

2/2

16/05/2015

Reference ranges

Previous

Question 2 of 401

Next

What is the nerve root value of the external urethral sphincter?

Question stats

Score: 100%

8.9%

16.1%

52.2%

A. S4

9.1%

B. S1, S2, S3

13.7%

C. S2, S3, S4

52.2% of users answered this


question correctly

D. L3, L4, L5

End and review

1
2

E. L5, S1, S2
Search
Next question
Go

Theme from April 2012 exam


Theme from April 2013 exam
Theme from April 2014 exam
The external urethral sphincter is innervated by branches of the pudendal nerve, therefore
the root values are S2, S3, S4.
Urethral anatomy
Female urethra
The female urethra is shorter and more acutely angulated than the male urethra. It is an
extra-peritoneal structure and embedded in the endopelvic fascia. The neck of the bladder is
subjected to transmitted intra-abdominal pressure and therefore deficiency in this area may
result in stress urinary incontinence. Between the layers of the urogenital diaphragm the
female urethra is surrounded by the external urethral sphincter, this is innervated by the
pudendal nerve. It ultimately lies anterior to the vaginal orifice.
Male urethra
In males the urethra is much longer and is divided into four parts.
Pre-prostatic
urethra

Extremely short and lies between the bladder and prostate gland.It has a
stellate lumen and is between 1 and 1.5cm long.Innervated by sympathetic
noradrenergic fibres, as this region is composed of striated muscles bundles
they may contract and prevent retrograde ejaculation.

Prostatic
urethra

This segment is wider than the membranous urethra and contains several
openings for the transmission of semen (at the midpoint of the urethral crest).

Membranous
urethra

Narrowest part of the urethra and surrounded by external sphincter. It traverses


the perineal membrane 2.5cm postero-inferior to the symphysis pubis.

Penile
urethra

Travels through the corpus spongiosum on the underside of the penis. It is the
longest urethral segment.It is dilated at its origin as the infrabulbar fossa and
again in the gland penis as the navicular fossa. The bulbo-urethral glands open
into the spongiose section of the urethra 2.5cm below the perineal membrane.

The urothelium is transitional in nature near to the bladder and becomes squamous more
distally.
Rate question:

Next question

Comment on this question

All contents of this site are 2012 E-Medical Revision Ltd

http://www.emrcs.com/question/question.php?q=0

Terms and Conditions

Privacy policy

1/1

16/05/2015

Reference ranges

Question 3 of 401

Previous

Next

A 45 year old man is stabbed in the abdomen and the inferior vena cava is injured. How
many functional valves does this vessel usually have?

A. 0
B. 1

Question stats

End and review

Score: 66.7%
1

46.5%

10.4%

15.3%

10.6%

17.2%

46.5% of users answered this


question correctly

C. 3
D. 2
E. 4

Search
Next question

Go

Mnemonic for the Inferior vena cava tributaries: I Like To Rise So High:
Iliacs
Lumbar
Testicular
Renal
Suprarenal
Hepatic vein

The lack of valves in the IVC is important clinically when it is cannulated during
cardiopulmonary bypass, using separate SVC and IVC catheters, such as when the right
atrium is to be opened. Note that there is a non functional valve between the right atrium and
inferior vena cava.
Inferior vena cava
Origin
L5

Path
Left and right common iliac veins merge to form the IVC.
Passes right of midline
Paired segmental lumbar veins drain into the IVC throughout its length
The right gonadal vein empties directly into the cava and the left gonadal vein
generally empties into the left renal vein.
The next major veins are the renal veins and the hepatic veins
Pierces the central tendon of diaphragm at T8
Right atrium

Image sourced from Wikipedia

Relations
Anteriorly

Small bowel, first and third part of duodenum, head of pancreas, liver and bile
duct, right common iliac artery, right gonadal artery

Posteriorly

Right renal artery, right psoas, right sympathetic chain, coeliac ganglion

Levels

http://www.emrcs.com/question/question.php?q=0

1/2

16/05/2015

Level

Vein

T8

Hepatic vein, inferior phrenic vein, pierces diaphragm

L1

Suprarenal veins, renal vein

L2

Gonadal vein

L1-5

Lumbar veins

L5

Common iliac vein, formation of IVC

Rate question:

Next question

Comment on this question

All contents of this site are 2012 E-Medical Revision Ltd

http://www.emrcs.com/question/question.php?q=0

Terms and Conditions

Privacy policy

2/2

16/05/2015

Reference ranges

Previous

Question 4 of 401

Next

Question stats

End and review

Score: 80%
1

9.7%

13.6%

52.8%

A. Posterior tibial artery

12.5%

B. Tibial nerve

11.4%

C. Tibialis anterior tendon

52.8% of users answered this


question correctly

Which of the following structures does not pass posteriorly to the medial malleolus?

D. Tendon of flexor digitorum longus


E. Tendon of flexor hallucis longus

Search
Next question
Go

Mnemonic for structures posterior to the medial malleolus:


Tom Dick And Nervous Harry
T ibialis posterior tendon
flexor Digitorum longus
A rtery
N erve
H allucis longus

Theme from April 2014 exam


Medial malleolus
The following structures pass posterior to medial malleolus (in order):

Tibialis posterior tendon


Flexor digitorum longus tendon
Posterior tibial artery
Tibial nerve
Tendon of flexor hallucis longus

Rate question:

Next question

Comment on this question

All contents of this site are 2012 E-Medical Revision Ltd

http://www.emrcs.com/question/question.php?q=0

Terms and Conditions

Privacy policy

1/1

16/05/2015

Reference ranges

Previous

Question 5 of 401

Next

Which of the following statements relating to the root of the neck is false?

A. The lung projects into the neck beyond the first rib and is constrained
by Sibson's fascia
B. The subclavian artery arches over the first rib anterior to scalenus
anterior

Question stats

End and review

Score: 83.3%
1

9.8%

44.2%

17.8%

16.4%

11.8%

44.2% of users answered this


question correctly

C. The roots and trunks of the Brachial plexus lie posterior to the
subclavian artery on the first rib
D. The roots and trunks of the Brachial plexus lie between scalenus
anterior and scalenus medius muscles

Search
Go

E. The thyrocervical trunk is a branch of the subclavian artery


Next question

The subclavian artery lies posterior to scalenus anterior, the vein lies in front. Sibson's fascia
is another name for the suprapleural membrane.
Root of the neck
Thoracic Outlet

Where the subclavian artery and vein and the brachial plexus exit the thorax and enter
the arm.
They pass over the 1st rib and under the clavicle.
The subclavian vein is the most anterior structure and is immediately anterior to
scalenus anterior and its attachment to the first rib.
Scalenus anterior has 2 parts, the subclavian artery leaves the thorax by passing over
the first rib and between these 2 portions of the muscle.
At the level of the first rib, the lower cervical nerve roots combine to form the 3 trunks
of the brachial plexus. The lowest trunk is formed by the union of C8 and T1, and this
trunk lies directly posterior to the artery and is in contact with the superior surface of
the first rib.

Thoracic outlet obstruction causes neurovascular compromise.


Rate question:

Next question

Comment on this question

All contents of this site are 2012 E-Medical Revision Ltd

http://www.emrcs.com/question/question.php?q=0

Terms and Conditions

Privacy policy

1/1

16/05/2015

Reference ranges

Previous

Question 7 of 401

Next

Question stats

End and review

Score: 88.9%
1

11.4%

50%

16.7%

A. The sino atrial node

13.5%

B. The circumflex artery

8.4%

C. The atrioventricular node

50% of users answered this


question correctly

Which of the following are not generally supplied by the right coronary artery?

D. Most of the right ventricle

6
7

E. The right atrium


Search
Next question
Go

The circumflex artery is generally a branch of the left coronary artery.


Heart anatomy
The walls of each cardiac chamber comprise:
Epicardium
Myocardium
Endocardium

Cardiac muscle is attached to the cardiac fibrous skeleton.


Relations
The heart and roots of the great vessels within the pericardial sac are related anteriorly to
the sternum, medial ends of the 3rd to 5th ribs on the left and their associated costal
cartilages. The heart and pericardial sac are situated obliquely two thirds to the left and one
third to the right of the median plane.
The pulmonary valve lies at the level of the left third costal cartilage.
The mitral valve lies at the level of the fourth costal cartilage.
Coronary sinus
This lies in the posterior part of the coronary groove and receives blood from the cardiac
veins. The great cardiac vein lies at its left and the middle and small cardiac veins lie on its
right. The smallest cardiac vein (anterior cardiac vein) drains into the right atrium directly.
Aortic sinus
Right coronary artery arises from the right aortic sinus, the left is derived from the left aortic
sinus and no vessel emerges from the posterior sinus.
Right and left ventricles
Structure

Left Ventricle

A-V Valve

Mitral (double leaflet)

Walls

Twice as thick as right

Trabeculae carnae

Much thicker and more numerous

Right coronary artery


The RCA supplies:
Right atrium
Diaphragmatic part of the left ventricle
Usually the posterior third of the interventricular septum
The sino atrial node (60% cases)
The atrio ventricular node (80% cases)

Left coronary artery


The LCA supplies:
Left atrium
Most of left ventricle
Part of the right ventricle
Anterior two thirds of the inter ventricular septum
The sino atrial node (remaining 40% cases)

http://www.emrcs.com/question/question.php?q=0

1/2

16/05/2015

Innervation of the heart


Autonomic nerve fibres from the superficial and deep cardiac plexus. These lie anterior to
the bifurcation of the trachea, posterior to the ascending aorta and superior to the
bifurcation of the pulmonary trunk. The parasympathetic supply to the heart is from
presynaptic fibres of the vagus nerves.
Valves of the heart
Mitral valve

Aortic valve

Pulmonary
valve

Tricuspid valve

2 cusps

3 cusps

3 cusps

3 cusps

First heart sound

Second heart
sound

Second heart
sound

First heart sound

1 anterior cusp

2 anterior cusps

2 anterior cusps

2 anterior cusps

Attached to chordae
tendinae

No chordae

No chordae

Attached to chordae
tendinae

Rate question:

Next question

Comment on this question

All contents of this site are 2012 E-Medical Revision Ltd

http://www.emrcs.com/question/question.php?q=0

Terms and Conditions

Privacy policy

2/2

16/05/2015

Reference ranges

Previous

Question 6 of 401

Next

A patient presents to the clinic following a surgical procedure. She complains that she is
unable to shrug her shoulder. What is the most likely underlying nerve injury?

A. Accessory nerve
B. Cervical plexus

Question stats

Score: 87.5%
1

61.2%

7.6%

7.6%

13.5%

10%

61.2% of users answered this


question correctly

C. Ansa cervicalis

End and review

D. Long thoracic nerve


E. Axillary nerve

Search
Next question

Go

Theme from April 2013 exam


Theme from April 2014 exam
The accessory nerve may be injured in operations in the posterior triangle. Injury will affect
trapezius.
Cranial nerves
Cranial nerve lesions
Olfactory nerve

May be injured in basal skull fractures or involved in frontal lobe tumour


extension. Loss of olfactory nerve function in relation to major CNS
pathology is seldom an isolated event and thus it is poor localiser of CNS
pathology.

Optic nerve

Problems with visual acuity may result from intra ocular disorders.
Problems with the blood supply such as amaurosis fugax may produce
temporary visual distortion. More important surgically is the pupillary
response to light. The pupillary size may be altered in a number of
disorders. Nerves involved in the resizing of the pupil connect to the
pretectal nucleus of the high midbrain, bypassing the lateral geniculate
nucleus and the primary visual cortex. From the pretectal nucleus
neurones pass to the Edinger - Westphal nucleus, motor axons from here
pass along with the oculomotor nerve. They synapse with ciliary ganglion
neurones; the parasympathetic axons from this then innervate the iris and
produce miosis. The miotic pupil is seen in disorders such as Horner's
syndrome or opiate overdose.
Mydriasis is the dilatation of the pupil in response to disease, trauma,
drugs (or the dark!). It is pathological when light fails to induce miosis. The
radial muscle is innervated by the sympathetic nervous system. Because
the parasympathetic fibres travel with the oculomotor nerve they will be
damaged by lesions affecting this nerve (e.g. cranial trauma).
The response to light shone in one eye is usually a constriction of both
pupils. This indicates intact direct and consensual light reflexes. When
the optic nerve has an afferent defect the light shining on the affected eye
will produce a diminished pupillary response in both eyes. Whereas light
shone on the unaffected eye will produce a normal pupillary response in
both eyes. This is referred to as the Marcus Gunn pupil and is seen in
conditions such as optic neuritis. In a total CN II lesion shining the light in
the affected eye will produce no response.

Oculomotor nerve

The pupillary effects are described above. In addition it supplies all ocular
muscles apart from lateral rectus and superior oblique. Thus the affected
eye will be deviated inferolaterally. Levator palpebrae superioris may also
be impaired resulting in impaired ability to open the eye.

Trochlear nerve

The eye will not be able to look down.

Trigeminal nerve

Largest cranial nerve. Exits the brainstem at the pons. Branches are
ophthalmic, maxillary and mandibular. Only the mandibular branch has
both sensory and motor fibres. Branches converge to form the trigeminal
ganglion (located in Meckels cave). It supplies the muscles of mastication
and also tensor veli palatine, mylohyoid, anterior belly of digastric and
tensor tympani. The detailed descriptions of the various sensory functions
are described in other areas of the website. The corneal reflex is important
and is elicited by applying a small tip of cotton wool to the cornea, a reflex
blink should occur if it is intact. It is mediated by: the naso ciliary branch
of the ophthalmic branch of the trigeminal (sensory component) and the
facial nerve producing the motor response. Lesions of the afferent arc will
produce bilateral absent blink and lesions of the efferent arc will result in a
unilateral absent blink.

Abducens nerve

The affected eye will have a deficit of abduction. This cranial nerve exits
the brainstem between the pons and medulla. It thus has a relatively long
intra cranial course which renders it susceptible to damage in raised intra
cranial pressure.

http://www.emrcs.com/question/question.php?q=0

1/2

16/05/2015

Facial nerve

Emerges from brainstem between pons and medulla. It controls muscles


of facial expression and taste from the anterior 2/3 of the tongue. The
nerve passes into the petrous temporal bone and into the internal auditory
meatus. It then passes through the facial canal and exits at the
stylomastoid foramen. It passes through the parotid gland and divides at
this point. It does not innervate the parotid gland. Its divisions are
considered in other parts of the website. Its motor fibres innervate
orbicularis oculi to produce the efferent arm of the corneal reflex. In
surgical practice it may be injured during parotid gland surgery or invaded
by malignancies of the gland and a lower motor neurone on the ipsilateral
side will result.

Vestibulocochlear nerve

Exits from the pons and then passes through the internal auditory
meatus. It is implicated in sensorineural hearing loss. Individuals with
sensorineural hearing loss will localise the sound in webers test to the
normal ear. Rinnes test will be reduced on the affected side but should
still work. These two tests will distinguish sensorineural hearing loss from
conductive deafness. In the latter condition webers test will localise to the
affected ear and Rinnes test will be impaired on the affected side. Surgical
lesions affecting this nerve include CNS tumours and basal skull fractures.
It may also be damaged by the administration of ototoxic drugs (of which
gentamicin is the most commonly used in surgical practice).

Glossopharyngeal
nerve

Exits the pons just above the vagus. Receives sensory fibres from
posterior 1/3 tongue, tonsils, pharynx and middle ear (otalgia may occur
following tonsillectomy). It receives visceral afferents from the carotid
bodies. It supplies parasympathetic fibres to the parotid gland via the otic
ganglion and motor function to stylopharyngeaus muscle. The sensory
function of the nerve is tested using the gag reflex.

Vagus nerve

Leaves the medulla between the olivary nucleus and the inferior cerebellar
peduncle. Passes through the jugular foramen and into the carotid sheath.
Details of the functions of the vagus nerve are covered in the website
under relevant organ sub headings.

Accessory nerve

Exists from the caudal aspect of the brainstem (multiple branches)


supplies trapezius and sternocleidomastoid muscles. The distal portion of
this nerve is most prone to injury during surgical procedures.

Hypoglossal
nerve

Emerges from the medulla at the preolivary sulcus, passes through the
hypoglossal canal. It lies on the carotid sheath and passes deep to the
posterior belly of digastric to supply muscles of the tongue (except
palatoglossus). Its location near the carotid sheath makes it vulnerable
during carotid endarterectomy surgery and damage will produce ipsilateral
defect in muscle function.

Rate question:

Next question

Comment on this question

All contents of this site are 2012 E-Medical Revision Ltd

http://www.emrcs.com/question/question.php?q=0

Terms and Conditions

Privacy policy

2/2

16/05/2015

Reference ranges

Previous

Question 8 of 401

Next

A 44 year old man has a malignant melanoma and is undergoing a block dissection of the
groin. The femoral triangle is being explored for intra operative bleeding. Which of the
following forms the medial border of the femoral triangle?

Question stats

Score: 72.7%
1

8.7%

8.7%

50.5%

16.3%

15.9%

A. Femoral artery
50.5% of users answered this
question correctly

B. Biceps femoris

End and review

6
7

C. Adductor longus
8

D. Sartorius
Search

E. Adductor magnus

Go

Next question

Vastus medialis forms the lateral border of the adductor canal. The sartorius muscles
forms the roof of the adductor canal.

Adductor longus forms the medial boundary of the femoral triangle (see below).
Femoral triangle anatomy
Boundaries
Superiorly

Inguinal ligament

Laterally

Sartorius

Medially

Adductor longus

Floor

Iliopsoas, adductor longus and pectineus

Roof

Fascia lata and Superficial fascia


Superficial inguinal lymph nodes (palpable below the inguinal ligament)
Great saphenous vein

Image sourced from Wikipedia

Contents
Femoral vein (medial to lateral)
Femoral artery-pulse palpated at the mid inguinal point
Femoral nerve
Deep and superficial inguinal lymph nodes
Lateral cutaneous nerve
Great saphenous vein
Femoral branch of the genitofemoral nerve

http://www.emrcs.com/question/question.php?q=0

1/2

16/05/2015

Rate question:

Next question

Comment on this question

All contents of this site are 2012 E-Medical Revision Ltd

http://www.emrcs.com/question/question.php?q=0

Terms and Conditions

Privacy policy

2/2

16/05/2015

Reference ranges

Question 9 of 401

Previous

Next

The foramen marking the termination of the adductor canal is located in which of the
following?

A. Adductor longus
B. Adductor magnus

Question stats

Score: 75%
1

20%

44.4%

9%

14.9%

11.8%

44.4% of users answered this


question correctly

C. Adductor brevis

End and review

D. Sartorius

6
7
8

E. Semimembranosus

Search
Go

Next question

The foramen marking the distal limit of the adductor canal is contained within adductor
magnus. The vessel passes through this region to enter the popliteal fossa.
Adductor canal

Also called Hunter's or subsartorial canal


Immediately distal to the apex of the femoral triangle, lying in the middle third of the
thigh. Canal terminates at the adductor hiatus.

Borders

Contents

Laterally Vastus medialis muscle

Saphenous nerve

Posteriorly Adductor longus, adductor magnus

Superficial femoral artery

Roof Sartorius

Superficial femoral vein

In the image below the sartorius muscle is removed to expose the canal contents

Image sourced from Wikipedia

Rate question:

Next question

Comment on this question

All contents of this site are 2012 E-Medical Revision Ltd

http://www.emrcs.com/question/question.php?q=0

Terms and Conditions

Privacy policy

1/1

16/05/2015

Reference ranges

Question 10 of 401

Previous

Next

Question stats

End and review

Score: 76.9%
1

48.7%

22.4%

10.2%

A. Superior thyroid artery

10.1%

B. Inferior thyroid artery

8.6%

C. Lingual artery

48.7% of users answered this


question correctly

Which of the following is the first vessel to branch from the external carotid artery?

D. Facial artery
E. Occipital artery

6
7
8
9

Search
Next question
Go

10

Mnemonic
(Order in which they branch off)Some (sup thyroid)Attendings (Ascending
Pharyngeal)Like (Lingual)Freaking (Facial)Out (Occipital)Potential (Post
auricular)Medical (Maxillary)Students (Sup temporal)

The first branch of the external carotid artery is the superior thyroid artery. The inferior
thyroid artery is derived from the thyrocervical trunk. The other branches are illustrated
below.
External carotid artery
The external carotid commences immediately lateral to the pharyngeal side wall. It ascends
and lies anterior to the internal carotid and posterior to the posterior belly of digastric and
stylohyoid. More inferiorly it is covered by sternocleidomastoid, passed by hypoglossal
nerves, lingual and facial veins.
It then pierces the fascia of the parotid gland finally dividing into its terminal branches within
the gland itself.
Surface marking of the carotid
This is an imaginary line drawn from the bifurcation of the common carotid passing behind
the angle of the jaw to a point immediately anterior to the tragus of the ear.
Branches of the external carotid artery
It has six branches, three in front, two behind and one deep.
Three in front

Superior thyroid
Lingual
Facial

Two behind

Occipital
Posterior auricular

Deep

Ascending pharyngeal

It terminates by dividing into the superficial temporal and maxillary arteries in the parotid
gland.

Image sourced from Wikipedia

http://www.emrcs.com/question/question.php?q=0

1/2

16/05/2015

Rate question:

Next question

Comment on this question

All contents of this site are 2012 E-Medical Revision Ltd

http://www.emrcs.com/question/question.php?q=0

Terms and Conditions

Privacy policy

2/2

16/05/2015

Reference ranges

Previous

Question 11 of 401

Next

A motorcyclist is injured in a road traffic accident and is not wearing a helmet. He suffers a
severe closed head injury and develops raised intracranial pressure. The first cranial nerve
to be affected by this process is likely to be:

Question stats

Score: 78.6%
1

26.4%

7.9%

9.2%

47.8%

8.8%

A. Oculomotor
47.8% of users answered this
question correctly

B. Hypoglossal

End and review

6
7

C. Motor branch of the trigeminal


8

D. Abducens
9

Search

E. Sensory branch of the trigeminal

Go

Next question

10
11

Theme from April 2014 exam


The abducens nerve (CN VI) has a long intra cranial course and is thus susceptible to raised
intra cranial pressure. It also passes over the petrous temporal bone and 6th nerve palsies
are also seen in mastoiditis.
Cranial nerves
Cranial nerve lesions
Olfactory nerve

May be injured in basal skull fractures or involved in frontal lobe tumour


extension. Loss of olfactory nerve function in relation to major CNS
pathology is seldom an isolated event and thus it is poor localiser of CNS
pathology.

Optic nerve

Problems with visual acuity may result from intra ocular disorders.
Problems with the blood supply such as amaurosis fugax may produce
temporary visual distortion. More important surgically is the pupillary
response to light. The pupillary size may be altered in a number of
disorders. Nerves involved in the resizing of the pupil connect to the
pretectal nucleus of the high midbrain, bypassing the lateral geniculate
nucleus and the primary visual cortex. From the pretectal nucleus
neurones pass to the Edinger - Westphal nucleus, motor axons from here
pass along with the oculomotor nerve. They synapse with ciliary ganglion
neurones; the parasympathetic axons from this then innervate the iris and
produce miosis. The miotic pupil is seen in disorders such as Horner's
syndrome or opiate overdose.
Mydriasis is the dilatation of the pupil in response to disease, trauma,
drugs (or the dark!). It is pathological when light fails to induce miosis. The
radial muscle is innervated by the sympathetic nervous system. Because
the parasympathetic fibres travel with the oculomotor nerve they will be
damaged by lesions affecting this nerve (e.g. cranial trauma).
The response to light shone in one eye is usually a constriction of both
pupils. This indicates intact direct and consensual light reflexes. When
the optic nerve has an afferent defect the light shining on the affected eye
will produce a diminished pupillary response in both eyes. Whereas light
shone on the unaffected eye will produce a normal pupillary response in
both eyes. This is referred to as the Marcus Gunn pupil and is seen in
conditions such as optic neuritis. In a total CN II lesion shining the light in
the affected eye will produce no response.

Oculomotor nerve

The pupillary effects are described above. In addition it supplies all ocular
muscles apart from lateral rectus and superior oblique. Thus the affected
eye will be deviated inferolaterally. Levator palpebrae superioris may also
be impaired resulting in impaired ability to open the eye.

Trochlear nerve

The eye will not be able to look down.

Trigeminal nerve

Largest cranial nerve. Exits the brainstem at the pons. Branches are
ophthalmic, maxillary and mandibular. Only the mandibular branch has
both sensory and motor fibres. Branches converge to form the trigeminal
ganglion (located in Meckels cave). It supplies the muscles of mastication
and also tensor veli palatine, mylohyoid, anterior belly of digastric and
tensor tympani. The detailed descriptions of the various sensory functions
are described in other areas of the website. The corneal reflex is important
and is elicited by applying a small tip of cotton wool to the cornea, a reflex
blink should occur if it is intact. It is mediated by: the naso ciliary branch
of the ophthalmic branch of the trigeminal (sensory component) and the
facial nerve producing the motor response. Lesions of the afferent arc will
produce bilateral absent blink and lesions of the efferent arc will result in a
unilateral absent blink.

Abducens nerve

The affected eye will have a deficit of abduction. This cranial nerve exits
the brainstem between the pons and medulla. It thus has a relatively long
intra cranial course which renders it susceptible to damage in raised intra

http://www.emrcs.com/question/question.php?q=0

1/2

16/05/2015

cranial pressure.
Facial nerve

Emerges from brainstem between pons and medulla. It controls muscles


of facial expression and taste from the anterior 2/3 of the tongue. The
nerve passes into the petrous temporal bone and into the internal auditory
meatus. It then passes through the facial canal and exits at the
stylomastoid foramen. It passes through the parotid gland and divides at
this point. It does not innervate the parotid gland. Its divisions are
considered in other parts of the website. Its motor fibres innervate
orbicularis oculi to produce the efferent arm of the corneal reflex. In
surgical practice it may be injured during parotid gland surgery or invaded
by malignancies of the gland and a lower motor neurone on the ipsilateral
side will result.

Vestibulocochlear nerve

Exits from the pons and then passes through the internal auditory
meatus. It is implicated in sensorineural hearing loss. Individuals with
sensorineural hearing loss will localise the sound in webers test to the
normal ear. Rinnes test will be reduced on the affected side but should
still work. These two tests will distinguish sensorineural hearing loss from
conductive deafness. In the latter condition webers test will localise to the
affected ear and Rinnes test will be impaired on the affected side. Surgical
lesions affecting this nerve include CNS tumours and basal skull fractures.
It may also be damaged by the administration of ototoxic drugs (of which
gentamicin is the most commonly used in surgical practice).

Glossopharyngeal
nerve

Exits the pons just above the vagus. Receives sensory fibres from
posterior 1/3 tongue, tonsils, pharynx and middle ear (otalgia may occur
following tonsillectomy). It receives visceral afferents from the carotid
bodies. It supplies parasympathetic fibres to the parotid gland via the otic
ganglion and motor function to stylopharyngeaus muscle. The sensory
function of the nerve is tested using the gag reflex.

Vagus nerve

Leaves the medulla between the olivary nucleus and the inferior cerebellar
peduncle. Passes through the jugular foramen and into the carotid sheath.
Details of the functions of the vagus nerve are covered in the website
under relevant organ sub headings.

Accessory nerve

Exists from the caudal aspect of the brainstem (multiple branches)


supplies trapezius and sternocleidomastoid muscles. The distal portion of
this nerve is most prone to injury during surgical procedures.

Hypoglossal
nerve

Emerges from the medulla at the preolivary sulcus, passes through the
hypoglossal canal. It lies on the carotid sheath and passes deep to the
posterior belly of digastric to supply muscles of the tongue (except
palatoglossus). Its location near the carotid sheath makes it vulnerable
during carotid endarterectomy surgery and damage will produce ipsilateral
defect in muscle function.

Rate question:

Next question

Comment on this question

All contents of this site are 2012 E-Medical Revision Ltd

http://www.emrcs.com/question/question.php?q=0

Terms and Conditions

Privacy policy

2/2

16/05/2015

Reference ranges

Previous

Question 12 of 401

Next

A 32 year old man is undergoing a splenectomy. Division of which of the following will be
necessary during the procedure?

A. Left crus of diaphragm


B. Short gastric vessels

Question stats

Score: 80%
1

8.5%

47.2%

15.6%

18.9%

9.8%

47.2% of users answered this


question correctly

C. Gerotas fascia

End and review

D. Splenic flexure of colon

6
7
8

E. Marginal artery

Search
Go

Next question

10
11
12

Theme from 2011 Exam


During a splenectomy the short gastric vessels which lie within the gastrosplenic ligament will
need to be divided. The splenic flexure of the colon may need to be mobilised. However, it
will almost never need to be divided, as this is watershed area that would necessitate a
formal colonic resection in the event of division.
Splenic anatomy
The spleen is the largest lymphoid organ in the body. It is an intraperitoneal organ, the
peritoneal attachments condense at the hilum where the vessels enter the spleen. Its blood
supply is from the splenic artery (derived from the coeliac axis) and the splenic vein (which is
joined by the IMV and unites with the SMV).

Embryology: derived from mesenchymal tissue


Shape: clenched fist
Position: below 9th-12th ribs
Weight: 75-150g

Relations
Superiorly- diaphragm
Anteriorly- gastric impression
Posteriorly- kidney
Inferiorly- colon
Hilum: tail of pancreas and splenic vessels
Forms apex of lesser sac (containing short gastric vessels)

Rate question:

Next question

Comment on this question

All contents of this site are 2012 E-Medical Revision Ltd

http://www.emrcs.com/question/question.php?q=0

Terms and Conditions

Privacy policy

1/1

16/05/2015

Reference ranges

Previous

Question 13 of 401

Next

A 24 year old motor cyclist is involved in a road traffic accident. He suffers a tibial fracture
which is treated with an intra medullary nail. Post operatively he develops a compartment
syndrome. Surgical decompression of the anterior compartment will relieve pressure on all of
the following muscles except?

Question stats

Score: 82.4%
1

45.1%

24.2%

14.2%

9%

7.5%

45.1% of users answered this


question correctly

A. Peroneus brevis

End and review

B. Peroneus tertius

6
7
8

C. Extensor digitorum longus

Search

D. Tibialis anterior
E. None of the above

Go

10
11

Next question

12
13

The anterior compartment contains:


Tibialis anterior
Extensor digitorum longus
Peroneus tertius
Extensor hallucis longus
Anterior tibial artery
All the muscles are innervated by the deep peroneal nerve.
Lower limb- Muscular compartments
Anterior compartment
Muscle

Nerve

Action

Tibialis anterior

Deep peroneal
nerve

Dorsiflexes ankle joint, inverts foot

Extensor digitorum
longus

Deep peroneal
nerve

Extends lateral four toes, dorsiflexes ankle


joint

Peroneus tertius

Deep peroneal
nerve

Dorsiflexes ankle, everts foot

Extensor hallucis longus

Deep peroneal
nerve

Dorsiflexes ankle joint, extends big toe

Peroneal compartment
Muscle

Nerve

Action

Peroneus longus

Superficial peroneal nerve

Everts foot, assists in plantar flexion

Peroneus brevis

Superficial peroneal nerve

Plantar flexes the ankle joint

Superficial posterior compartment


< b=""><>
b=""><>

Nerve

Action

Gastrocnemius

Tibial nerve

Plantar flexes the foot, may also flex the knee

Soleus

Tibial nerve

Plantar flexor

Deep posterior compartment


Muscle

Nerve

Action

Flexor digitorum longus

Tibial

Flexes the lateral four toes

Flexor hallucis longus

Tibial

Flexes the great toe

Tibialis posterior

Tibial

Plantar flexor, inverts the foot

Rate question:

Next question

Comment on this question

http://www.emrcs.com/question/question.php?q=0

1/2

16/05/2015
All contents of this site are 2012 E-Medical Revision Ltd

http://www.emrcs.com/question/question.php?q=0

Terms and Conditions

Privacy policy

2/2

16/05/2015

Reference ranges

Previous

Question 14 of 401

Next

A 43 year old lady underwent an attempted placement of a central line into the internal
jugular vein. Unfortunately, the doctor damaged the carotid artery and this necessitated
surgical exploration. As the surgeons incise the carotid sheath a nerve is identified lying
between the internal jugular vein and the carotid artery. Which of the following is this nerve
most likely to be?

Question stats

Score: 77.8%
1

8.8%

16.2%

7.8%

11.6%

55.6%

55.6% of users answered this


question correctly

A. Glossopharyngeal nerve

End and review

6
7
8

B. Hypoglossal nerve
C. Superior laryngeal nerve

Search

D. Recurrent laryngeal nerve

Go

E. Vagus

10
11
12

Next question

The vagus lies in the carotid sheath. The hypoglossal nerve crosses the sheath, but does
not lie within it.

13
14

Common carotid artery


The right common carotid artery arises at the bifurcation of the brachiocephalic trunk, the left
common carotid arises from the arch of the aorta. Both terminate at the level of the upper
border of the thyroid cartilage (the lower border of the third cervical vertebra) by dividing into
the internal and external carotid arteries.
Left common carotid artery
This vessel arises immediately to the left and slightly behind the origin of the brachiocephalic
trunk. Its thoracic portion is 2.5- 3.5 cm in length and runs superolaterally to the
sternoclavicular joint.
In the thorax
The vessel is in contact, from below upwards, with the trachea, left recurrent laryngeal nerve,
left margin of the oesophagus. Anteriorly the left brachiocephalic vein runs across the artery,
and the cardiac branches from the left vagus descend in front of it. These structures
together with the thymus and the anterior margins of the left lung and pleura separate the
artery from the manubrium.
In the neck
The artery runs superiorly deep to sternocleidomastoid and then enters the anterior triangle.
At this point it lies within the carotid sheath with the vagus nerve and the internal jugular vein.
Posteriorly the sympathetic trunk lies between the vessel and the prevertebral fascia. At the
level of C7 the vertebral artery and thoracic duct lie behind it. The anterior tubercle of C6
transverse process is prominent and the artery can be compressed against this structure (it
corresponds to the level of the cricoid).
Anteriorly at C6 the omohyoid muscle passes superficial to the artery.
Within the carotid sheath the jugular vein lies lateral to the artery.
Right common carotid artery
The right common carotid arises from the brachiocephalic artery. The right common carotid
artery corresponds with the cervical portion of the left common carotid, except that there is
no thoracic duct on the right. The oesophagus is less closely related to the right carotid than
the left.
Summary points about the carotid anatomy
Path
Passes behind the sternoclavicular joint (12% patients above this level) to the upper border
of the thyroid cartilage, to divide into the external (ECA) and internal carotid arteries (ICA).
Relations
Level of 6th cervical vertebra crossed by omohyoid
Then passes deep to the thyrohyoid, sternohyoid, sternomastoid muscles.
Passes anterior to the carotid tubercle (transverse process 6th cervical vertebra)-NB
compression here stops haemorrhage.
The inferior thyroid artery passes posterior to the common carotid artery.
Then : Left common carotid artery crossed by thoracic duct, Right common carotid
artery crossed by recurrent laryngeal nerve

http://www.emrcs.com/question/question.php?q=0

1/2

16/05/2015

Image sourced from Wikipedia

Rate question:

Next question

Comment on this question

All contents of this site are 2012 E-Medical Revision Ltd

http://www.emrcs.com/question/question.php?q=0

Terms and Conditions

Privacy policy

2/2

16/05/2015

Reference ranges

Previous

Question 15 of 401

Next

A patient has a chest drain insertion. There is fresh blood at the chest drain insertion area.
Which vessel has been damaged?

A. Pericardiophrenic artery
B. Intercostal vein

Question stats

Score: 73.7%
1

8%

37.1%

5.4%

5.8%

43.7%

43.7% of users answered this


question correctly

C. Right ventricle

End and review

D. Vagus artery

6
7
8

E. Intercostal artery

Search
Go

Next question

10
11
12

The intercostal vein is more superior than the artery and is thus slightly less susceptible
to injury.

13
14
15

Theme from 2009 Exam


Within the intercostal spaces there are thin, strong muscles, intercostal vessels, nerves and
lymphatics. There are 3 intercostal muscle layers corresponding to the lateral abdominal
wall; external, internal, innermost intercostals. At the mid axillary line there are thin
intracostals which is an extension of the internal intercostal muscle. In each intercostal space
lies the neurovascular bundle, comprising, from superior to inferiorly; the posterior intercostal
vein, artery and nerve, lying protected in the subcostal groove of the rib above and situated
between the second and third layer of the intercostal muscles. These blood vessels
anastomose anteriorly with the anterior intercostal vessels, which arise from the internal
thoracic artery and vein.
Chest drains
There are a number of different indications for chest drain insertion. In general terms large
bore chest drains are preferred for trauma and haemothorax drainage. Smaller diameter
chest drains can be used for pneumothorax or pleural effusion drainage.
Insertion can be performed either using anatomical guidance or through ultrasound
guidance. In the exam, the anatomical method is usually tested.
It is advised that chest drains are placed in the 'safe triangle'. The triangle is located in the
mid axillary line of the 5th intercostal space. It is bordered by:
Anterior edge latissimus dorsi, the lateral border of pectoralis major, a line superior to the
horizontal level of the nipple, and the apex below the axilla.
Another triangle is situated behind the scapula. It is bounded above by the trapezius, below
by the latissimus dorsi, and laterally by the vertebral border of the scapula; the floor is partly
formed by the rhomboid major. If the scapula is drawn forward by folding the arms across the
chest, and the trunk bent forward, parts of the sixth and seventh ribs and the interspace
between them become subcutaneous and available for auscultation. The space is therefore
known as the triangle of auscultation.
References
Prof Harold Ellis. The applied anatomy of chest drains insertions. British Journal of hospital
medicine 2007; (68): 44-45.
Laws D, Neville E, Duffy J. BTS guidelines for insertion of chest drains. Thorax, 2003; (58):
53-59.
Rate question:

Next question

Comment on this question

All contents of this site are 2012 E-Medical Revision Ltd

http://www.emrcs.com/question/question.php?q=0

Terms and Conditions

Privacy policy

1/1

16/05/2015

Reference ranges

Previous

Question 16 of 401

Next

Two teenagers are playing with an airgun when one accidentally shoots his friend in the
abdomen. He is brought to the emergency department. On examination there is a bullet entry
point immediately to the right of the rectus sheath at the level of the 1st lumbar vertebra.
Which of the following structures is most likely to be injured by the bullet?

Question stats

Score: 75%
1

18.4%

12.8%

13.1%

44.8%

10.8%

44.8% of users answered this


question correctly

A. Head of pancreas

End and review

B. Right ureter

6
7
8

C. Right adrenal gland

Search

D. Fundus of the gallbladder


E. Gastric antrum

Go

10
11

Next question

12
13

Theme from September 2011 Exam


Theme from April 2014 exam

14
15

The fundus of the gallbladder lies at this level and is the most superficially located structure.

16

Levels
Transpyloric plane
Level of the body of L1

Pylorus stomach
Left kidney hilum (L1- left one!)
Right hilum of the kidney (1.5cm lower than the left)
Fundus of the gallbladder
Neck of pancreas
Duodenojejunal flexure
Superior mesenteric artery
Portal vein
Left and right colic flexure
Root of the transverse mesocolon
2nd part of the duodenum
Upper part of conus medullaris
Spleen

Can be identified by asking the supine patient to sit up without using their arms. The plane is
located where the lateral border of the rectus muscle crosses the costal margin.
Anatomical planes
Subcostal plane

Lowest margin of 10th costal cartilage

Intercristal plane

Level of body L4 (highest point of iliac crest)

Intertubercular plane

Level of body L5

Common level landmarks


Inferior mesenteric artery

L3

Bifurcation of aorta into common iliac arteries

L4

Formation of IVC

L5 (union of common iliac veins)

Diaphragm apertures

Rate question:

Vena cava T8
Oesophagus T10
Aortic hiatus T12

Next question

Comment on this question

All contents of this site are 2012 E-Medical Revision Ltd

http://www.emrcs.com/question/question.php?q=0

Terms and Conditions

Privacy policy

1/2

16/05/2015

http://www.emrcs.com/question/question.php?q=0

2/2

16/05/2015

Reference ranges

Previous

Question 17 of 401

Next

Question stats

End and review

Score: 76.2%
1

40.6%

12.2%

10.6%

A. Subscapularis

22.5%

B. Deltoid

14%

C. Supraspinatus

40.6% of users answered this


question correctly

Which of the following muscles inserts onto the lesser tuberostiy of the the humerus?

D. Teres minor
E. Infraspinatus

6
7
8
9

Search
Next question
Go

With the exception of subscapularis which inserts into the lesser tuberosity, the muscles of
the rotator cuff insert into the greater tuberosity.

10
11
12
13

Shoulder joint

14
15

Shallow synovial ball and socket type of joint.


It is an inherently unstable joint, but is capable to a wide range of movement.
Stability is provided by muscles of the rotator cuff that pass from the scapula to insert
in the greater tuberosity (all except sub scapularis-lesser tuberosity).

16
17

Glenoid labrum
Fibrocartilaginous rim attached to the free edge of the glenoid cavity
Tendon of the long head of biceps arises from within the joint from the supraglenoid
tubercle, and is fused at this point to the labrum.
The long head of triceps attaches to the infraglenoid tubercle

Fibrous capsule
Attaches to the scapula external to the glenoid labrum and to the labrum itself
(postero-superiorly)
Attaches to the humerus at the level of the anatomical neck superiorly and the surgical
neck inferiorly
Anteriorly the capsule is in contact with the tendon of subscapularis, superiorly with the
supraspinatus tendon, and posteriorly with the tendons of infraspinatus and teres
minor. All these blend with the capsule towards their insertion.
Two defects in the fibrous capsule; superiorly for the tendon of biceps. Anteriorly there
is a defect beneath the subscapularis tendon.
The inferior extension of the capsule is closely related to the axillary nerve at the
surgical neck and this nerve is at risk in anteroinferior dislocations. It also means that
proximally sited osteomyelitis may progress to septic arthritis.

Movements and muscles


Flexion

Anterior part of deltoid


Pectoralis major
Biceps
Coracobrachialis

Extension

Posterior deltoid
Teres major
Latissimus dorsi

Adduction

Pectoralis major
Latissimus dorsi
Teres major
Coracobrachialis

Abduction

Mid deltoid
Supraspinatus

Medial rotation

Subscapularis
Anterior deltoid
Teres major
Latissimus dorsi

Lateral rotation

Posterior deltoid
Infraspinatus
Teres minor

http://www.emrcs.com/question/question.php?q=0

1/2

16/05/2015

Important anatomical relations


Anteriorly

Brachial plexus
Axillary artery and vein

Posterior

Suprascapular nerve
Suprascapular vessels

Inferior

Axillary nerve
Circumflex humeral vessels

Rate question:

Next question

Comment on this question

All contents of this site are 2012 E-Medical Revision Ltd

http://www.emrcs.com/question/question.php?q=0

Terms and Conditions

Privacy policy

2/2

16/05/2015

Reference ranges

Question 18 of 401

Previous

Next

Question stats

End and review

Score: 77.3%
1

10%

27.1%

13.9%

A. Accessory nerve

38.3%

B. Phrenic nerve

10.7%

C. Greater auricular nerve

38.3% of users answered this


question correctly

Which of the following nerves is not contained within the posterior triangle of the neck?

D. Ansa cervicalis
E. Lesser occiptal nerve

6
7
8
9

Search
Next question
Go

10
11

Theme from September 2012 Exam


Ansa cervicalis is a content of the anterior triangle of the neck.

12
13

Posterior triangle of the neck

14
15

Boundaries
Apex

Sternocleidomastoid and the Trapezius muscles at the Occipital bone

Anterior

Posterior border of the Sternocleidomastoid

Posterior

Anterior border of the Trapezius

Base

Middle third of the clavicle

16
17
18

Image sourced from Wikipedia

Contents
Nerves

Vessels

Muscles

Lymph
nodes

Accessory nerve
Phrenic nerve
Three trunks of the brachial plexus
Branches of the cervical plexus: Supraclavicular nerve, transverse cervical
nerve, great auricular nerve, lesser occipital nerve

External jugular vein


Subclavian artery

Inferior belly of omohyoid


Scalene

Supraclavicular
Occipital

Rate question:

Next question

Comment on this question

All contents of this site are 2012 E-Medical Revision Ltd

http://www.emrcs.com/question/question.php?q=0

Terms and Conditions

Privacy policy

1/1

16/05/2015

Reference ranges

Question 19 of 401

Previous

Next

A 42 year old lady is reviewed in the outpatient clinic following a routine surgical procedure.
She complains of diminished sensation at the dorso-lateral aspect of her foot. Which of the
following nerves is most likely to be affected?

Question stats

Score: 73.9%
1

46.2%

24.6%

8.9%

6.7%

13.7%

A. Sural
46.2% of users answered this
question correctly

B. Superficial peroneal

End and review

6
7

C. Deep peroneal
8

D. Medial plantar
9

Search

E. Lateral plantar

Go

Next question

10
11
12

Theme from April 2012 Exam


The sural nerve supplies the lateral aspect of the foot. It runs alongside the short saphenous
vein and may be injured in short saphenous vein surgery.

13
14
15

Foot- Cutaneous sensation

16
17

Region

Nerve

Lateral plantar

Sural

Dorsum (not 1st web space)

Superficial peroneal

1st Web space

Deep peroneal

Extremities of toes

Medial and lateral plantar nerves

Proximal plantar

Tibial

Medial plantar

Medial plantar nerve

Lateral plantar

Lateral plantar nerve

18
19

Image sourced from Wikipedia

http://www.emrcs.com/question/question.php?q=0

1/2

16/05/2015

Rate question:

Next question

Comment on this question

All contents of this site are 2012 E-Medical Revision Ltd

http://www.emrcs.com/question/question.php?q=0

Terms and Conditions

Privacy policy

2/2

16/05/2015

Reference ranges

Question 20 of 401

Previous

Next

Question stats

End and review

Score: 75%
1

9.6%

50%

10.7%

A. Sibsons fascia

22.3%

B. Denonvilliers fascia

7.3%

C. Levator ani muscle

50% of users answered this


question correctly

Which of the following anatomical planes separates the prostate from the rectum?

D. Waldeyers fascia
E. None of the above

6
7
8
9

Search
Next question
Go

Theme from April 2014 exam


The Denonvilliers fascia separates the rectum from the prostate. Waldeyers fascia separates
the rectum from the sacrum

10
11
12
13
14

Prostate gland

15

The prostate gland is approximately the shape and size of a walnut and is located inferior to
the bladder. It is separated from the rectum by Denonvilliers fascia and its blood supply is
derived from the internal iliac vessels. The internal sphincter lies at the apex of the gland and
may be damaged during prostatic surgery, affected individuals may complain of retrograde
ejaculation.

16
17
18
19
20

Summary of prostate gland


Arterial supply

Inferior vesical artery (from internal iliac)

Venous
drainage

Prostatic venous plexus (to paravertebral veins)

Lymphatic
drainage

Internal iliac nodes

Innervation

Inferior hypogastric plexus

Dimensions

Lobes

Zones

Transverse diameter (4cm)


AP diameter (2cm)
Height (3cm)

Posterior lobe: posterior to urethra


Median lobe: posterior to urethra, in between ejaculatory ducts
Lateral lobes x 2
Isthmus

Peripheral zone: subcapsular portion of posterior prostate. Most


prostate cancers are here
Central zone
Transition zone
Stroma

Relations
Anterior

Pubic symphysis
Prostatic venous plexus

Posterior

Denonvilliers fascia
Rectum
Ejaculatory ducts

Lateral

Venous plexus (lies on prostate)


Levator ani (immediately below the puboprostatic ligaments)

http://www.emrcs.com/question/question.php?q=0

1/2

16/05/2015

Image sourced from Wikipedia

Rate question:

Next question

Comment on this question

All contents of this site are 2012 E-Medical Revision Ltd

http://www.emrcs.com/question/question.php?q=0

Terms and Conditions

Privacy policy

2/2

16/05/2015

Reference ranges

Previous

Question 21 of 401

Next

A 56 year old lady is undergoing an adrenalectomy for Conns syndrome. During the
operation the surgeon damages the middle adrenal artery and haemorrhage ensues. From
which of the following structures does this vessel originate?

Question stats

Score: 76%
1

45.6%

27%

8.8%

9.5%

9%

45.6% of users answered this


question correctly

A. Aorta
B. Renal artery

End and review

C. Splenic artery
8

D. Coeliac axis
9

Search

E. Superior mesenteric artery

Go

10
11

Next question

12

The middle adrenal artery is usually a branch of the aorta, the lower adrenal artery typically
arises from the renal vessels.

13

Adrenal gland anatomy

15

14

16

Anatomy

17
18

Location

Superomedially to the upper pole of each kidney

Relationships of
the right adrenal

Diaphragm-Posteriorly, Kidney-Inferiorly, Vena Cava-Medially, Hepatorenal pouch and bare area of the liver-Anteriorly

Relationships of
the left adrenal

Crus of the diaphragm-Postero- medially, Pancreas and splenic


vessels-Inferiorly, Lesser sac and stomach-Anteriorly

Arterial supply

Superior adrenal arteries- from inferior phrenic artery, Middle adrenal


arteries - from aorta, Inferior adrenal arteries -from renal arteries

Venous drainage
of the right adrenal

Via one central vein directly into the IVC

Venous drainage
of the left adrenal

Via one central vein into the left renal vein

Rate question:

19
20
21

Next question

Comment on this question

All contents of this site are 2012 E-Medical Revision Ltd

http://www.emrcs.com/question/question.php?q=0

Terms and Conditions

Privacy policy

1/1

16/05/2015

Reference ranges

Previous

Question 22 of 401

Next

A 73 year old lady suffers a fracture at the surgical neck of the humerus. The decision is
made to operate. There are difficulties in reducing the fracture and a vessel lying posterior to
the surgical neck is injured. Which of the following is this vessel most likely to be?

Question stats

Score: 77.8%
1

24.5%

8.9%

7%

7.5%

52%

A. Axillary artery
52% of users answered this
question correctly

B. Brachial artery

End and review

6
7

C. Thoracoacromial artery
8

D. Transverse scapular artery


9

Search

E. Posterior circumflex humeral artery

Go

Next question

10
11
12

The circumflex humeral arteries lie at the surgical neck and is this scenario the posterior
circumflex is likely to be injured. The thoracoacromial and transverse scapular arteries lie
more superomedially. The posterior circumflex humeral artery is a branch of the axillary
artery.

13
14
15
16

Shoulder joint

17
18

Shallow synovial ball and socket type of joint.


It is an inherently unstable joint, but is capable to a wide range of movement.
Stability is provided by muscles of the rotator cuff that pass from the scapula to insert
in the greater tuberosity (all except sub scapularis-lesser tuberosity).

19
20
21
22

Glenoid labrum
Fibrocartilaginous rim attached to the free edge of the glenoid cavity
Tendon of the long head of biceps arises from within the joint from the supraglenoid
tubercle, and is fused at this point to the labrum.
The long head of triceps attaches to the infraglenoid tubercle

Fibrous capsule
Attaches to the scapula external to the glenoid labrum and to the labrum itself
(postero-superiorly)
Attaches to the humerus at the level of the anatomical neck superiorly and the surgical
neck inferiorly
Anteriorly the capsule is in contact with the tendon of subscapularis, superiorly with the
supraspinatus tendon, and posteriorly with the tendons of infraspinatus and teres
minor. All these blend with the capsule towards their insertion.
Two defects in the fibrous capsule; superiorly for the tendon of biceps. Anteriorly there
is a defect beneath the subscapularis tendon.
The inferior extension of the capsule is closely related to the axillary nerve at the
surgical neck and this nerve is at risk in anteroinferior dislocations. It also means that
proximally sited osteomyelitis may progress to septic arthritis.

Movements and muscles


Flexion

Anterior part of deltoid


Pectoralis major
Biceps
Coracobrachialis

Extension

Posterior deltoid
Teres major
Latissimus dorsi

Adduction

Pectoralis major
Latissimus dorsi
Teres major
Coracobrachialis

Abduction

Mid deltoid
Supraspinatus

Medial rotation

Subscapularis
Anterior deltoid
Teres major
Latissimus dorsi

http://www.emrcs.com/question/question.php?q=0

1/2

16/05/2015

Lateral rotation

Posterior deltoid
Infraspinatus
Teres minor

Important anatomical relations


Anteriorly

Brachial plexus
Axillary artery and vein

Posterior

Suprascapular nerve
Suprascapular vessels

Inferior

Axillary nerve
Circumflex humeral vessels

Rate question:

Next question

Comment on this question

All contents of this site are 2012 E-Medical Revision Ltd

http://www.emrcs.com/question/question.php?q=0

Terms and Conditions

Privacy policy

2/2

16/05/2015

Reference ranges

Previous

Question 23 of 401

Next

Which of the structures listed below lies posterior to the carotid sheath at the level of the 6th
cervical vertebra?

A. Hypoglossal nerve
B. Vagus nerve

Question stats

Score: 75%
1

19.6%

12.2%

39.3%

18.3%

10.5%

39.3% of users answered this


question correctly

C. Cervical sympathetic chain

End and review

D. Ansa cervicalis

6
7
8

E. Glossopharyngeal nerve

Search
Next question

Go

10
11

The carotid sheath is crossed anteriorly by the hypoglossal nerves and the ansa cervicalis.
The vagus lies within it. The cervical sympathetic chain lies posteriorly between the sheath
and the prevertebral fascia.

12

Common carotid artery

15

13
14

16

The right common carotid artery arises at the bifurcation of the brachiocephalic trunk, the left
common carotid arises from the arch of the aorta. Both terminate at the level of the upper
border of the thyroid cartilage (the lower border of the third cervical vertebra) by dividing into
the internal and external carotid arteries.

17
18
19
20

Left common carotid artery


This vessel arises immediately to the left and slightly behind the origin of the brachiocephalic
trunk. Its thoracic portion is 2.5- 3.5 cm in length and runs superolaterally to the
sternoclavicular joint.

21
22
23

In the thorax
The vessel is in contact, from below upwards, with the trachea, left recurrent laryngeal nerve,
left margin of the oesophagus. Anteriorly the left brachiocephalic vein runs across the artery,
and the cardiac branches from the left vagus descend in front of it. These structures
together with the thymus and the anterior margins of the left lung and pleura separate the
artery from the manubrium.
In the neck
The artery runs superiorly deep to sternocleidomastoid and then enters the anterior triangle.
At this point it lies within the carotid sheath with the vagus nerve and the internal jugular vein.
Posteriorly the sympathetic trunk lies between the vessel and the prevertebral fascia. At the
level of C7 the vertebral artery and thoracic duct lie behind it. The anterior tubercle of C6
transverse process is prominent and the artery can be compressed against this structure (it
corresponds to the level of the cricoid).
Anteriorly at C6 the omohyoid muscle passes superficial to the artery.
Within the carotid sheath the jugular vein lies lateral to the artery.
Right common carotid artery
The right common carotid arises from the brachiocephalic artery. The right common carotid
artery corresponds with the cervical portion of the left common carotid, except that there is
no thoracic duct on the right. The oesophagus is less closely related to the right carotid than
the left.
Summary points about the carotid anatomy
Path
Passes behind the sternoclavicular joint (12% patients above this level) to the upper border
of the thyroid cartilage, to divide into the external (ECA) and internal carotid arteries (ICA).
Relations
Level of 6th cervical vertebra crossed by omohyoid
Then passes deep to the thyrohyoid, sternohyoid, sternomastoid muscles.
Passes anterior to the carotid tubercle (transverse process 6th cervical vertebra)-NB
compression here stops haemorrhage.
The inferior thyroid artery passes posterior to the common carotid artery.
Then : Left common carotid artery crossed by thoracic duct, Right common carotid
artery crossed by recurrent laryngeal nerve

http://www.emrcs.com/question/question.php?q=0

1/2

16/05/2015

Image sourced from Wikipedia

Rate question:

Next question

Comment on this question

All contents of this site are 2012 E-Medical Revision Ltd

http://www.emrcs.com/question/question.php?q=0

Terms and Conditions

Privacy policy

2/2

16/05/2015

Reference ranges

Question 24 of 401

Previous

Next

A sprinter attends A&E with severe leg pain. He had forgotten to warm up and ran a 100m
sprint race. Towards the end of the race he experienced pain in the posterior aspect of his
thigh. The pain worsens, localising to the lateral aspect of the knee. The sprinter is unable to
flex the knee. What structure has been injured?

Question stats

Score: 75.9%
1

10.2%

10%

13.9%

13.5%

52.5%

52.5% of users answered this


question correctly

A. Anterior cruciate ligament

End and review

B. Posterior cruciate ligament

6
7
8

C. Semimembranosus tendon

Search

D. Semitendinosus tendon
E. Biceps femoris tendon

Go

10
11

Next question

12
13

Theme from 2009 Exam


The biceps femoris is commonly injured in sports that require explosive bending of the knee
as seen in sprinting, especially if the athlete has not warmed up first. Avulsion most
commonly occurs where the long head attaches to the ischial tuberosity. Injuries to biceps
femoris are more common than to the other hamstrings.

14
15
16
17
18

Biceps femoris

19
20

The biceps femoris is one of the hamstring group of muscles located in the posterior upper
thigh. It has two heads.

21
22

Long head

23

Origin

Ischial tuberosity

Insertion

Fibular head

Action

Knee flexion, lateral rotation tibia, extension hip

Innervation

Tibial division of sciatic nerve (L5, S1, S2)

Arterial
supply

Profunda femoris artery, inferior gluteal artery, and the superior muscular
branches of popliteal artery

24

Image demonstrating the biceps femoris muscle, with the long head outlined

Image sourced from Wikipedia

Short head
Origin

Lateral lip of linea aspera, lateral supracondylar ridge of femur

Insertion

Fibular head

Action

Knee flexion, lateral rotation tibia

Innervation

Common peroneal division of sciatic nerve (L5, S1, S2)

Arterial
supply

Profunda femoris artery, inferior gluteal artery, and the superior muscular
branches of popliteal artery

http://www.emrcs.com/question/question.php?q=0

1/2

16/05/2015

Rate question:

Next question

Comment on this question

All contents of this site are 2012 E-Medical Revision Ltd

http://www.emrcs.com/question/question.php?q=0

Terms and Conditions

Privacy policy

2/2

16/05/2015

Reference ranges

Previous

Question 25 of 401

Next

A 24 year old man falls and lands astride a manhole cover. He suffers from an injury to the
anterior bulbar urethra. Where will the extravasated urine tend to collect?

A. Lesser pelvis
B. Connective tissue of the scrotum

Question stats

Score: 77.4%
1

11.2%

42.3%

25.8%

12.6%

8.1%

42.3% of users answered this


question correctly

C. Deep perineal space

End and review

D. Ischiorectal fossa

6
7
8

E. Posterior abdominal wall

Search
Go

Next question

10
11
12

This portion of the urethra is contained between the perineal membrane and the
membranous layer of the superficial fascia. As these are densely adherent to the ischiopubic
rami, extravasated urine cannot pass posteriorly because the 2 layers are continuous
around the superficial transverse perineal muscles.

13
14
15

Lower genitourinary tract trauma

16
17
18

Most bladder injuries occur due to blunt trauma


85% associated with pelvic fractures
Easily overlooked during assessment in trauma
Up to 10% of male pelvic fractures are associated with urethral or bladder injuries

19
20
21
22

Types of injury

23
24

Urethral injury

Mainly in males
Blood at the meatus (50% cases)
There are 2 types:

25

i.Bulbar rupture
- most common
- straddle type injury e.g. bicycles
- triad signs: urinary retention, perineal haematoma, blood at
the meatus
ii. Membranous rupture
- can be extra or intraperitoneal
- commonly due to pelvic fracture
- Penile or perineal oedema/ hematoma
- PR: prostate displaced upwards (beware co-existing
retroperitoneal haematomas as they may make examination
difficult)
- Investigation: ascending urethrogram
- Management: suprapubic catheter (surgical placement, not
percutaneously)
External genitalia injuries
(i.e., the penis and the
scrotum)

Bladder injury

Rate question:

Secondary to injuries caused by penetration, blunt


trauma, continence- or sexual pleasure-enhancing
devices, and mutilation

rupture is intra or extraperitoneal


presents with haematuria or suprapubic pain
history of pelvic fracture and inability to void: always
suspect bladder or urethral injury
inability to retrieve all fluid used to irrigate the bladder
through a Foley catheter indicates bladder injury
investigation- IVU or cystogram
management: laparotomy if intraperitoneal, conservative
if extraperitoneal

Next question

Comment on this question

All contents of this site are 2012 E-Medical Revision Ltd

http://www.emrcs.com/question/question.php?q=0

Terms and Conditions

Privacy policy

1/1

16/05/2015

Reference ranges

Question 26 of 401

Previous

Next

A 73 year old man presents with symptoms of mesenteric ischaemia. As part of his diagnostic
work up a diagnostic angiogram is performed .The radiologist is attempting to cannulate the
coeliac axis from the aorta. At which of the following vertebral levels does this is usually
originate?

Question stats

Score: 78.8%
1

8.8%

10.9%

9.3%

8.6%

62.5%

62.5% of users answered this


question correctly

A. T10

End and review

B. L2

6
7
8

C. L3

Search

D. T8
E. T12

Go

10
11

Next question

12
13
14

Coeliac trunk branches:

15

Left Hand Side (LHS)

16
17

Left gastric
Hepatic
Splenic

18
19
20

The coeliac axis branches off the aorta at T12.

21
22

Coeliac axis

23

The coeliac axis has three main branches.

24
25

Left gastric
Hepatic: branches-Right Gastric, Gastroduodenal, Right Gastroepiploic, Superior
Pancreaticoduodenal, Cystic (occasionally).
Splenic: branches- Pancreatic, Short Gastric, Left Gastroepiploic

26

It occasionally gives off one of the inferior phrenic arteries.

Image sourced from Wikipedia

Relations
Anteriorly

Lesser omentum

Right

Right coeliac ganglion and caudate process of liver

Left

Left coeliac ganglion and gastric cardia

Inferiorly

Upper border of pancreas and renal vein

Rate question:

Next question

Comment on this question

All contents of this site are 2012 E-Medical Revision Ltd

http://www.emrcs.com/question/question.php?q=0

Terms and Conditions

Privacy policy

1/1

16/05/2015

Reference ranges

Question 1 of 375

Next

A 43 year old man is diagnosed as having a malignancy of the right adrenal gland. The
decision is made to resect this via an open anterior approach. Which of the following will be
most useful during the surgery?

Question stats

End and review

Score: 100%

9.2%

44.2%

22.9%

15.2%

8.5%

A. Division of the coronary ligaments of the liver


44.2% of users answered this
question correctly

B. Mobilisation of the colonic hepatic flexure


C. Division of the right renal vein
D. Division of the ligament of Trietz

Search

E. Division of the right colic artery

Go

Next question

Mobilisation of the hepatic flexure and right colon are standard steps in open adrenal
surgery from an anterior approach. Mobilisation of the liver is seldom required.
Adrenal gland anatomy
Anatomy
Location

Superomedially to the upper pole of each kidney

Relationships of
the right adrenal

Diaphragm-Posteriorly, Kidney-Inferiorly, Vena Cava-Medially, Hepatorenal pouch and bare area of the liver-Anteriorly

Relationships of
the left adrenal

Crus of the diaphragm-Postero- medially, Pancreas and splenic


vessels-Inferiorly, Lesser sac and stomach-Anteriorly

Arterial supply

Superior adrenal arteries- from inferior phrenic artery, Middle adrenal


arteries - from aorta, Inferior adrenal arteries -from renal arteries

Venous drainage
of the right adrenal

Via one central vein directly into the IVC

Venous drainage
of the left adrenal

Via one central vein into the left renal vein

Rate question:

Next question

Comment on this question

All contents of this site are 2012 E-Medical Revision Ltd

http://www.emrcs.com/question/question.php?q=0

Terms and Conditions

Privacy policy

1/1

16/05/2015

Reference ranges

Question 2 of 375

Previous

Next

A 45 year old man presents with a lipoma located posterior to the posterior border of the
sternocleidomastoid muscle, approximately 4cm superior to the middle third of the clavicle.
During surgical excision of the lesion troublesome bleeding is encountered. Which of the
following is the most likely source?

Question stats

End and review

Score: 50%

13%

46.6%

13.2%

10.5%

16.8%

1
2

46.6% of users answered this


question correctly

A. Internal jugular vein


B. External jugular vein
C. Common carotid artery

Search

D. Vertebral artery
E. Second part of the subclavian artery

Go

Next question

The external jugular vein runs obliquely in the superficial fascia of the posterior triangle. It
drains into the subclavian vein. During surgical exploration of this area the external jugular
vein may be injured and troublesome bleeding may result. The internal jugular vein and
carotid arteries are located in the anterior triangle. The third, and not the second, part of the
subclavian artery is also a content of the posterior triangle
Posterior triangle of the neck
Boundaries
Apex

Sternocleidomastoid and the Trapezius muscles at the Occipital bone

Anterior

Posterior border of the Sternocleidomastoid

Posterior

Anterior border of the Trapezius

Base

Middle third of the clavicle

Image sourced from Wikipedia

Contents
Nerves

Vessels

Muscles

Lymph
nodes

Accessory nerve
Phrenic nerve
Three trunks of the brachial plexus
Branches of the cervical plexus: Supraclavicular nerve, transverse cervical
nerve, great auricular nerve, lesser occipital nerve

External jugular vein


Subclavian artery

Inferior belly of omohyoid


Scalene

Supraclavicular
Occipital

Rate question:

http://www.emrcs.com/question/question.php?q=0

Next question

1/2

16/05/2015
Comment on this question

All contents of this site are 2012 E-Medical Revision Ltd

http://www.emrcs.com/question/question.php?q=0

Terms and Conditions

Privacy policy

2/2

16/05/2015

Reference ranges

Previous1 / 3

Question 3-5 of 375

Next

Question stats

End and review

Score: 40%

Average score for registered users:

1
2

Theme: Levels of spinal injury


A. C2

64.1%

59.5%

77.8%

3-5 1 / 3

B. C3
C. C4
D. C5

Search

E. C6

Go

F. L1
G. L2
H. L3
I. L4
J. L5
Please select the most likely spinal level for the injury described. Each option may be used
once, more than once or not at all.

3.

A 62 year old male complains of back pain. He has had a recent fall. Walking
causes pain of the left lower leg. On examination he is noted to have reduced
sensation over the knee.
You answered L4
The correct answer is L3
Sensation over the knee is equivalent to the L3 dermatome. The four nerves
involved include the infrapatellar branch of the saphenous nerve, the lateral
cutaneous nerve of the thigh, anterior cutaneous nerve of the thigh (both lateral
and medial branches).

4.

A 42 year old woman is found to have a burst fracture of the C5 vertebral body.
After a few months where would the level of injury be?
You answered C5
The correct answer is C6
A C5 burst fracture usually injures the C6 spinal cord situated at the C5
vertebrae and also the C4 spinal roots that exits the spinal column between the
C4 and C5 vertebra. Such an injury should cause a loss of sensations in C4
dermatome and weak deltoids. Due to oedema , the biceps (C5) may be initially
weak but should recover. The wrist extensors (C6), however, should remain
weak and sensation at and below C6 should be severely compromised. A
neurosurgeon would conclude that there is a burst fracture at C5 from the xrays, an initial sensory level at C4 (the first abnormal sensory dermatome) and
the partial loss of deltoids and biceps would imply a motor level at C4 (the
highest abnormal muscle level). Over time, as the patient recovers the C4 roots
and the C5 spinal cord, both the sensory level and motor level should end up at
C6. Such recovery is often attributed to 'root' recovery.

5.

A 56 year old man suddenly develops severe back pain. His pain has a radicular
pattern. On examination he is unable to extend his great toe.
L5
Extensor hallucis longus is derived from L5 and loss of EHL function is a useful
test to determine whether this level is involved.

Next question

Spinal disorders

Dorsal column lesion

Loss vibration and proprioception


Tabes dorsalis, SACD

Spinothalamic tract
http://www.emrcs.com/question/question.php?q=0

1/4

16/05/2015

Spinothalamic tract
lesion
Central cord lesion

Osteomyelitis

Infarction spinal cord

Cord compression

Brown-sequard
syndrome

Loss of pain, sensation and temperature

Flaccid paralysis of the upper limbs

Normally progressive
Staph aureus in IVDU, normally cervical region affected
Fungal infections in immunocompromised
Thoracic region affected in TB

Dorsal column signs (loss of proprioception and fine


discrimination)

UMN signs
Malignancy
Haematoma
Fracture

Hemisection of the spinal cord


Ipsilateral paralysis
Ipsilateral loss of proprioception and fine discrimination
Contralateral loss of pain and temperature

Image sourced from Wikipedia

http://www.emrcs.com/question/question.php?q=0

2/4

16/05/2015

Image sourced from Wikipedia

Dermatomes
C2 to C4 The C2 dermatome covers the occiput and the top part of the neck. C3
covers the lower part of the neck to the clavicle. C4 covers the area just below the
clavicle.
C5 to T1 Situated in the arms. C5 covers the lateral arm at and above the elbow. C6
covers the forearm and the radial (thumb) side of the hand. C7 is the middle finger, C8
is the medial aspect of the hand, and T1 covers the medial side of the forearm.
T2 to T12 The thoracic covers the axillary and chest region. T3 to T12 covers the
chest and back to the hip girdle. The nipples are situated in the middle of T4. T10 is
situated at the umbilicus. T12 ends just above the hip girdle.
L1 to L5 The cutaneous dermatome representing the hip girdle and groin area is
innervated by L1 spinal cord. L2 and 3 cover the front part of the thighs. L4 and L5
cover medial and lateral aspects of the lower leg.
S1 to S5 S1 covers the heel and the middle back of the leg. S2 covers the back of the
thighs. S3 cover the medial side of the buttocks and S4-5 covers the perineal region.
S5 is of course the lowest dermatome and represents the skin immediately at and
adjacent to the anus.

Myotomes
Upper limb
Elbow flexors/Biceps

C5

Wrist extensors

C6

Elbow extensors/Triceps

C7

Long finger flexors

C8

Small finger abductors

T1

Lower limb
Hip flexors (psoas)

L1 and L2

Knee extensors (quadriceps)

L3

Ankle dorsiflexors (tibialis anterior)

L4 and L5

Toe extensors (hallucis longus)

L5

Ankle plantar flexors (gastrocnemius)

S1

http://www.emrcs.com/question/question.php?q=0

3/4

16/05/2015

The anal sphincter is innervated by S2,3,4


Rate question:

Next question

Comment on this question

All contents of this site are 2012 E-Medical Revision Ltd

http://www.emrcs.com/question/question.php?q=0

Terms and Conditions

Privacy policy

4/4

16/05/2015

Reference ranges

Previous

Question 6 of 375

Next

The sciatic nerve lies deep to the following structures except:

Question stats

Score: 28.6%

12.5%

24.2%

18.5%

A. Gluteus maximus

28.3%

B. The femoral cutaneous nerve

16.6%

C. Long head of biceps femoris

28.3% of users answered this


question correctly

D. Gluteus medius

End and review

1
2
3-5 1 / 3
6

E. Branch of the inferior gluteal artery


Search
Next question
Go

The gluteus medius does not extend around to the sciatic nerve.
Sciatic nerve
The sciatic nerve is formed from the sacral plexus and is the largest nerve in the body. It is
the continuation of the main part of the plexus arising from ventral rami of L4 to S3. These
rami converge at the inferior border of piriformis to form the nerve itself. It passes through
the inferior part of the greater sciatic foramen and emerges beneath piriformis. Medially, lie
the inferior gluteal nerve and vessels and the pudendal nerve and vessels. It runs
inferolaterally under the cover of gluteus maximus midway between the greater trochanter
and ischial tuberosity. It receives its blood supply from the inferior gluteal artery. The nerve
provides cutaneous sensation to the skin of the foot and the leg. It also innervates the
posterior thigh muscles and the lower leg and foot muscles. The nerve splits into the tibial
and common peroneal nerves approximately half way down the posterior thigh. The tibial
nerve supplies the flexor muscles and the common peroneal nerve supplies the extensor
muscles and the abductor muscles.
Summary points
Origin

Spinal nerves L4 - S3

Articular Branches

Hip joint

Muscular branches in
upper leg

Semitendinosus
Semimembranosus
Biceps femoris
Part of adductor magnus

Cutaneous sensation

Terminates

Posterior aspect of thigh


Gluteal region
Entire lower leg (except the medial aspect)
At the upper part of the popliteal fossa by dividing into the tibial
and peroneal nerves

The nerve to the short head of the biceps femoris comes from the common peroneal
part of the sciatic and the other muscular branches arise from the tibial portion.
The tibial nerve goes on to innervate all muscles of the foot except the extensor
digitorum brevis (which is innervated by the common peroneal nerve).

Rate question:

Next question

Comment on this question

All contents of this site are 2012 E-Medical Revision Ltd

http://www.emrcs.com/question/question.php?q=0

Terms and Conditions

Privacy policy

1/1

16/05/2015

Reference ranges

Previous

Question 7 of 375

Next

Which of the following upper limb muscles is not innervated by the radial nerve?

Question stats

End and review

Score: 44.4%

16.1%

44.5%

1
2
3-5 1 / 3

12.6%

A. Extensor carpi ulnaris

11.1%

B. Abductor digiti minimi

15.8%

C. Anconeus

44.5% of users answered this


question correctly

D. Supinator
E. Brachioradialis

Search
Next question
Go

Mnemonic for radial nerve muscles: BEST


B rachioradialis
E xtensors
S upinator
T riceps

Abductor digiti minimi is innervated by the ulnar nerve.


Radial nerve
Continuation of posterior cord of the brachial plexus (root values C5 to T1)
Path
In the axilla: lies posterior to the axillary artery on subscapularis, latissimus dorsi and
teres major.
Enters the arm between the brachial artery and the long head of triceps (medial to
humerus).
Spirals around the posterior surface of the humerus in the groove for the radial nerve.
At the distal third of the lateral border of the humerus it then pierces the intermuscular
septum and descends in front of the lateral epicondyle.
At the lateral epicondyle it lies deeply between brachialis and brachioradialis where it
then divides into a superficial and deep terminal branch.
Deep branch crosses the supinator to become the posterior interosseous nerve.

In the image below the relationships of the radial nerve can be appreciated

Image sourced from Wikipedia

Regions innervated
Motor (main
nerve)

Triceps
Anconeus
Brachioradialis
Extensor carpi radialis

Motor
(posterior
interosseous
branch)

Supinator
Extensor carpi ulnaris
Extensor digitorum
Extensor indicis
Extensor digiti minimi

http://www.emrcs.com/question/question.php?q=0

1/2

16/05/2015
Extensor pollicis longus and brevis
Abductor pollicis longus
Sensory

The area of skin supplying the proximal phalanges on the dorsal aspect of
the hand is supplied by the radial nerve (this does not apply to the little
finger and part of the ring finger)

Muscular innervation and effect of denervation


Anatomical
location

Muscle affected

Effect of paralysis

Shoulder

Long head of triceps

Minor effects on shoulder stability in


abduction

Arm

Triceps

Loss of elbow extension

Forearm

Supinator
Brachioradialis
Extensor carpi radialis
longus and brevis

Weakening of supination of prone hand and


elbow flexion in mid prone position

The cutaneous sensation of the upper limb- illustrating the contribution of the radial nerve

Image sourced from Wikipedia

Rate question:

Next question

Comment on this question

All contents of this site are 2012 E-Medical Revision Ltd

http://www.emrcs.com/question/question.php?q=0

Terms and Conditions

Privacy policy

2/2

17/05/2015

Reference ranges

Question 1 of 368

Next

Question stats

Score: 100%

10.3%

6.6%

12.3%

A. Radial artery

62.8%

B. Cephalic vein

8%

C. Extensor pollicis brevis

62.8% of users answered this


question correctly

Which of the following forms the floor of the anatomical snuffbox?

D. Scaphoid bone

End and review

E. Cutaneous branch of the radial nerve


Search
Next question
Go

Theme from April 2014 exam


The scaphoid bone forms the floor of the anatomical snuffbox. The cutaneous branch of the
radial nerve is much more superficially and proximally located.
Anatomical snuffbox

Posterior border

Tendon of extensor pollicis longus

Anterior border

Tendons of extensor pollicis brevis and abductor pollicis longus

Proximal border

Styloid process of the radius

Distal border

Apex of snuffbox triangle

Floor

Trapezium and scaphoid

Content

Radial artery

Image showing the anatomical snuffbox

Image sourced from Wikipedia

Rate question:

Next question

Comment on this question

All contents of this site are 2012 E-Medical Revision Ltd

http://www.emrcs.com/question/question.php?q=0

Terms and Conditions

Privacy policy

1/1

17/05/2015

Reference ranges

Previous

Question 2 of 368

Next

During a liver resection a surgeon performs a pringles manoeuvre to control bleeding. Which
of the following structures will lie posterior to the epiploic foramen at this level?

A. Hepatic artery
B. Cystic duct

Question stats

End and review

Score: 100%

22.3%

11%

9.1%

11.2%

46.4%

1
2

46.4% of users answered this


question correctly

C. Greater omentum
D. Superior mesenteric artery
E. Inferior vena cava

Search
Next question

Go

Bleeding from liver trauma or a difficult cholecystectomy can be controlled with a


vascular clamp applied at the epiploic foramen.

The epiploic foramen has the following boundaries:


Anteriorly (in the free edge of the lesser omentum): Bile duct to the right, portal vein behind
and hepatic artery to the left.
PosteriorlyInferior vena cava
Inferiorly1st part of the duodenum
SuperiorlyCaudate process of the liver
Liver
Structure of the liver
Right lobe

Supplied by right hepatic artery


Contains Couinaud segments V to VIII (-/+Sg I)

Left lobe

Supplied by the left hepatic artery


Contains Couinaud segments II to IV (+/- Sg1)

Quadrate lobe

Part of the right lobe anatomically, functionally is part of the left


Couinaud segment IV
Porta hepatis lies behind
On the right lies the gallbladder fossa
On the left lies the fossa for the umbilical vein

Caudate lobe

Supplied by both right and left hepatic arteries


Couinaud segment I
Lies behind the plane of the porta hepatis
Anterior and lateral to the inferior vena cava
Bile from the caudate lobe drains into both right and left hepatic ducts

Detailed knowledge of Couinaud segments is not required for MRCS Part A


Between the liver lobules are portal canals which contain the portal triad: Hepatic
Artery, Portal Vein, tributary of Bile Duct.

Relations of the liver


Anterior

Postero inferiorly

Diaphragm

Oesophagus

Xiphoid process

Stomach
Duodenum
Hepatic flexure of colon
Right kidney
Gallbladder
Inferior vena cava

http://www.emrcs.com/question/question.php?q=0

1/2

17/05/2015

Porta hepatis
Location

Postero inferior surface, it joins nearly at right angles with the left sagittal fossa,
and separates the caudate lobe behind from the quadrate lobe in front

Transmits

Common hepatic duct


Hepatic artery
Portal vein
Sympathetic and parasympathetic nerve fibres
Lymphatic drainage of the liver (and nodes)

Ligaments
Falciform
ligament

2 layer fold peritoneum from the umbilicus to anterior liver surface


Contains ligamentum teres (remnant umbilical vein)
On superior liver surface it splits into the coronary and left
triangular ligaments

Ligamentum teres

Joins the left branch of the portal vein in the porta hepatis

Ligamentum
venosum

Remnant of ductus venosus

Arterial supply
Hepatic artery

Venous
Hepatic veins
Portal vein

Nervous supply
Sympathetic and parasympathetic trunks of coeliac plexus

Rate question:

Next question

Comment on this question

All contents of this site are 2012 E-Medical Revision Ltd

http://www.emrcs.com/question/question.php?q=0

Terms and Conditions

Privacy policy

2/2

17/05/2015

Reference ranges

Question 3 of 368

Previous

Next

A 72 year old lady is suspected of having a femoral hernia. At which of the following sites is it
most likely to be identifiable clinically?

A. Mid inguinal point


B. Above and medial to the pubic tubercle

Question stats

End and review

Score: 100%
1

9.8%

10.7%

60.8%

10.7%

7.9%

60.8% of users answered this


question correctly

C. Below and lateral to the pubic tubercle


D. Mid point of the inguinal ligament
E. 3 cm superomedially to the superficial inguinal ring

Search
Next question

Go

Femoral hernias exit the femoral canal below and lateral to the pubic tubercle. Femoral
hernia occur mainly in women due to their difference in pelvic anatomy. They are at high risk
of strangulation and therefore should be repaired.
Femoral canal
The femoral canal lies at the medial aspect of the femoral sheath. The femoral sheath is a
fascial tunnel containing both the femoral artery laterally and femoral vein medially. The
canal lies medial to the vein.
Borders of the femoral canal
Laterally

Femoral vein

Medially

Lacunar ligament

Anteriorly

Inguinal ligament

Posteriorly

Pectineal ligament

Image showing dissection of femoral canal

Image sourced from Wikipedia

Contents
Lymphatic vessels
Cloquet's lymph node

Physiological significance
Allows the femoral vein to expand to allow for increased venous return to the lower limbs.
Pathological significance
As a potential space, it is the site of femoral hernias. The relatively tight neck places these at
high risk of strangulation.
http://www.emrcs.com/question/question.php?q=0

1/2

17/05/2015

Rate question:

Next question

Comment on this question

All contents of this site are 2012 E-Medical Revision Ltd

http://www.emrcs.com/question/question.php?q=0

Terms and Conditions

Privacy policy

2/2

17/05/2015

Reference ranges

Previous

Question 4 of 368

Next

Which muscle is responsible for causing flexion of the distal interphalangeal joint of the ring
finger?

A. Flexor digitorum superficialis


B. Lumbricals

Question stats

End and review

Score: 100%
1

17.2%

8.6%

6.6%

58.1%

9.6%

58.1% of users answered this


question correctly

C. Palmar interossei
D. Flexor digitorum profundus
E. Flexor digiti minimi brevis

Search
Next question

Go

Theme from April 2014 exam


Flexor digitorum superficialis and flexor digitorum profundus are responsible for causing
flexion. The superficialis tendons insert on the bases of the middle phalanges; the profundus
tendons insert on the bases of the distal phalanges. Both tendons flex the wrist, MCP and
PIP joints; however, only the profundus tendons flex the DIP joints.
Hand
Anatomy of the hand
Bones

8 Carpal bones
5 Metacarpals
14 phalanges

Intrinsic Muscles

7 Interossei - Supplied by ulnar nerve


3 palmar-adduct fingers
4 dorsal- abduct fingers

Intrinsic muscles

Lumbricals
Flex MCPJ and extend the IPJ.
Origin deep flexor tendon and insertion dorsal extensor hood
mechanism.
Innervation: 1st and 2nd- median nerve, 3rd and 4th- deep branch of
the ulnar nerve.

Thenar eminence

Hypothenar
eminence

Abductor pollicis brevis


Opponens pollicis
Flexor pollicis brevis

Opponens digiti minimi


Flexor digiti minimi brevis
Abductor digiti minimi

Image sourced from Wikipedia

http://www.emrcs.com/question/question.php?q=0

1/2

17/05/2015

Fascia and compartments of the palm


The fascia of the palm is continuous with the antebrachial fascia and the fascia of the
dorsum of the hand. The palmar fascia is thin over the thenar and hypothenar eminences. In
contrast the palmar fascia is relatively thick. The palmar aponeurosis covers the soft tissues
and overlies the flexor tendons. The apex of the palmar aponeurosis is continuous with the
flexor retinaculum and the palmaris longus tendon. Distally, it forms four longitudinal digital
bands that attach to the bases of the proximal phalanges, blending with the fibrous digital
sheaths.
A medial fibrous septum extends deeply from the medial border of the palmar aponeurosis to
the 5th metacarpal. Lying medial to this are the hypothenar muscles. In a similar fashion, a
lateral fibrous septum extends deeply from the lateral border of the palmar aponeurosis to
the 3rd metacarpal. The thenar compartment lies lateral to this area.
Lying between the thenar and hypothenar compartments is the central compartment. It
contains the flexor tendons and their sheaths, the lumbricals, the superficial palmar arterial
arch and the digital vessels and nerves.
The deepest muscular plane is the adductor compartment, which contains adductor pollicis.
Short muscles of the hand
These comprise the lumbricals and interossei. The four slender lumbrical muscles flex the
fingers at the metacarpophalangeal joints and extend the interphalangeal joint. The four
dorsal interossei are located between the metacarpals and the four palmar interossei lie on
the palmar surface of the metacarpals in the interosseous compartment of the hand.
Long flexor tendons and sheaths in the hand
The tendons of FDS and FDP enter the common flexor sheath deep to the flexor
retinaculum. The tendons enter the central compartment of the hand and fan out to their
respective digital synovial sheaths. Near the base of the proximal phalanx, the tendon of FDS
splits to permit the passage of FDP. The FDP tendons are attached to the margins of the
anterior aspect of the base of the distal phalanx.
The fibrous digital sheaths contain the flexor tendons and their synovial sheaths. These
extend from the heads of the metacarpals to the base of the distal phalanges.
Rate question:

Next question

Comment on this question

All contents of this site are 2012 E-Medical Revision Ltd

http://www.emrcs.com/question/question.php?q=0

Terms and Conditions

Privacy policy

2/2

17/05/2015

Reference ranges

Previous

Question 5 of 368

Next

A 34 year old lady undergoes a thyroidectomy for Graves disease. Post operatively she
develops a tense haematoma in the neck. In which of the following fascial planes will it be
contained?

Question stats

End and review

Score: 100%
1

10.2%

7.9%

65.3%

9%

7.6%

A. Gerotas fascia
65.3% of users answered this
question correctly

B. Waldeyers fascia
C. Pretracheal fascia
D. Sibsons fascia

Search

E. Clavipectoral fascia

Go

Next question

The pretracheal fascia encloses the thyroid and is unyielding. Therefore tense haematomas
can develop.
Thyroid gland

Right and left lobes connected by isthmus


Surrounded by sheath from pretracheal layer of deep fascia
Apex: Lamina of thyroid cartilage
Base: 4th-5th tracheal ring
Pyramidal lobe: from isthmus
May be attached to foramen caecum at the base of the tongue

Relations
Anteromedially

Posterolaterally

Sternothyroid
Superior belly of omohyoid
Sternohyoid
Anterior aspect of sternocleidomastoid
Carotid sheath

Medially

Larynx
Trachea
Pharynx
Oesophagus
Cricothyroid muscle
External laryngeal nerve (near superior thyroid artery)
Recurrent laryngeal nerve (near inferior thyroid artery)

Posterior

Parathyroid glands
Anastomosis of superior and inferior thyroid arteries

Isthmus

Anteriorly: Sternothyroids, sternohyoids, anterior jugular veins


Posteriorly: 2nd, 3rd, 4th tracheal rings (attached via Ligament of
Berry)

Blood Supply
Arterial

Venous

Superior thyroid artery (1st branch of external carotid)


Inferior thyroid artery (from thyrocervical trunk)
Thyroidea ima (in 10% of population -from brachiocephalic artery or aorta)

Superior and middle thyroid veins - into the IJV


Inferior thyroid vein - into the brachiocephalic veins

Rate question:

Next question

Comment on this question

All contents of this site are 2012 E-Medical Revision Ltd

http://www.emrcs.com/question/question.php?q=0

Terms and Conditions

Privacy policy

1/2

17/05/2015

http://www.emrcs.com/question/question.php?q=0

2/2

17/05/2015

Reference ranges

Question 6 of 368

Previous

Next

A 32 year old lady complains of carpal tunnel syndrome. The carpal tunnel is explored
surgically. Which of the following structures will lie in closest proximity to the hamate bone
within the carpal tunnel?

Question stats

Score: 100%
1

11.4%

37.7%

15.2%

24.3%

11.5%

A. The tendon of abductor pollicis longus


37.7% of users answered this
question correctly

B. The tendons of flexor digitorum profundus

End and review

C. The tendons of flexor carpi radialis longus


D. Median nerve
Search

E. Radial artery

Go

Next question

The carpal tunnel contains nine flexor tendons:


Flexor digitorum profundus
Flexor digitorum superficialis
Flexor pollicis longus

The tendon of flexor digitorum profundus lies deepest in the tunnel and will thus lie nearest
to the hamate bone.
Carpal bones
The wrist is comprised of 8 carpal bones, these are arranged in two rows of 4. It is convex
from side to side posteriorly and concave anteriorly.
Diagrammatic image of carpal bones

Image sourced from Wikipedia

Key to image
A

Scaphoid

Lunate

Triquetrum

Pisiform

Trapezium

Trapezoid

Capitate

Hamate

Radius

Ulna

Metacarpals

No tendons attach to: Scaphoid, lunate, triquetrum (stabilised by ligaments)

http://www.emrcs.com/question/question.php?q=0

1/2

17/05/2015

Rate question:

Next question

Comment on this question

All contents of this site are 2012 E-Medical Revision Ltd

http://www.emrcs.com/question/question.php?q=0

Terms and Conditions

Privacy policy

2/2

17/05/2015

Reference ranges

Question 7 of 368

Previous

Next

A 45 year old man sustains a significant head injury and a craniotomy is performed. The
sigmoid sinus is bleeding profusely, into which of the following structures does it drain?

A. Internal jugular vein


B. Straight sinus

Question stats

Score: 100%
1

48.8%

13.6%

11.6%

16.5%

9.4%

48.8% of users answered this


question correctly

C. Petrosal sinus

End and review

6
7

D. Inferior sagittal sinus


E. External jugular vein

Search
Go

Next question

The sigmoid sinus is joined by the inferior petrosal sinus to drain into the internal jugular
vein.
Cranial venous sinuses
The cranial venous sinuses are located within the dura mater. They have no valves which is
important in the potential for spreading sepsis. They eventually drain into the internal jugular
vein.
They are:
Superior sagittal sinus
Inferior sagittal sinus
Straight sinus
Transverse sinus
Sigmoid sinus
Confluence of sinuses
Occipital sinus
Cavernous sinus
Topography of cranial venous sinuses

Image sourced from Wikipedia

Rate question:

Next question

Comment on this question

All contents of this site are 2012 E-Medical Revision Ltd

http://www.emrcs.com/question/question.php?q=0

Terms and Conditions

Privacy policy

1/1

17/05/2015

Reference ranges

Previous

Question 8 of 368

Next

Question stats

End and review

Score: 100%
1

8.8%

11.7%

14.6%

A. Radial

55.1%

B. Median

9.8%

C. Superficial ulnar

55.1% of users answered this


question correctly

Which nerve supplies the interossei of the fourth finger?

D. Deep ulnar
E. Posterior interosseous

6
7
8

Search
Next question
Go

Mnemonic:
PAD and DAB
Palmer interossei ADduct
Dorsal interossei ABduct

Theme from April 2013 exam


Theme from April 2014 exam
Interossei

Origin and insertion

Nerve
supply

Actions

Three palmar and four dorsal interossei occupy the spaces


between the metacarpal bones. Each palmar interossei
originates from the metacarpal of the digit on which it acts.
Each dorsal interossei comes from the surface of the adjacent
metacarpal on which it acts. As a result the dorsal interossei are
twice the size of the palmar ones.
The interossei tendons, except the first palmar, pass to one or
other side of the metacarpophalangeal joint posterior to the deep
transverse metacarpal ligament. They become inserted into the
base of the proximal phalanx and partly into the extensor hood

They are
all
innervated
by the
ulnar
nerve

Dorsal
interossei
abduct the
fingers, palmar
interossei
adduct the
fingers

Clinical notes
Along with the lumbricals the interossei flex the metacarpophalangeal joints and extend the
proximal and distal interphalangeal joints. They are responsible for fine tuning these
movements. When the interossei and lumbricals are paralysed the digits are pulled into
hyperextension by extensor digitorum and a claw hand is seen.
Rate question:

Next question

Comment on this question

All contents of this site are 2012 E-Medical Revision Ltd

http://www.emrcs.com/question/question.php?q=0

Terms and Conditions

Privacy policy

1/1

17/05/2015

Reference ranges

Previous

Question 9 of 368

Next

Question stats

End and review

Score: 100%
1

50.9%

7.5%

20.2%

A. Sphenoid bone

11.4%

B. Frontal bone

9.9%

C. Temporal bone

50.9% of users answered this


question correctly

In which of the following cranial bones does the foramen spinosum lie?

D. Occipital bone
E. Parietal bone

6
7
8
9

Search
Next question
Go

The foramen spinosum (which transmits the middle meningeal artery and vein) lies in the
sphenoid bone.
Foramina of the base of the skull

Foramen

Location

Contents

Foramen
ovale

Sphenoid
bone

Otic ganglion
V3 (Mandibular nerve:3rd branch of
trigeminal)
Accessory meningeal artery
Lesser petrosal nerve
Emissary veins

Foramen
spinosum

Sphenoid
bone

Middle meningeal artery


Meningeal branch of the Mandibular nerve

Foramen
rotundum

Sphenoid
bone

Maxillary nerve (V2)

Foramen
lacerum/
carotid canal

Sphenoid
bone

Base of the medial pterygoid plate.


Internal carotid artery*
Nerve and artery of the pterygoid canal

Jugular
foramen

Temporal
bone

Anterior: inferior petrosal sinus


Intermediate: glossopharyngeal, vagus, and accessory nerves.
Posterior: sigmoid sinus (becoming the internal jugular vein) and
some meningeal branches from the occipital and ascending
pharyngeal arteries.

Foramen
magnum

Occipital
bone

Anterior and posterior spinal arteries


Vertebral arteries
Medulla oblongata

Stylomastoid
foramen

Temporal
bone

Stylomastoid artery
Facial nerve

Superior
orbital fissure

Sphenoid
bone

Oculomotor nerve (III)


trochlear nerve (IV)
lacrimal, frontal and nasociliary branches of ophthalmic nerve (V1)
abducent nerve (VI)
Superior and inferior ophthalmic vein

*= In life the foramen lacerum is occluded by a cartilagenous plug. The ICA initially passes
into the carotid canal which ascends superomedially to enter the cranial cavity through the
foramen lacerum.
Base of skull anatomical overview

http://www.emrcs.com/question/question.php?q=0

1/2

17/05/2015
Image sourced from Wikipedia

Rate question:

Next question

Comment on this question

All contents of this site are 2012 E-Medical Revision Ltd

http://www.emrcs.com/question/question.php?q=0

Terms and Conditions

Privacy policy

2/2

17/05/2015

Reference ranges

Previous

Question 10 of 368

Next

Question stats

End and review

Score: 100%
1

10.7%

8.6%

57.9%

A. Bronchial artery

13.6%

B. Mediastinal artery

9.2%

C. Inferior thyroid artery

57.9% of users answered this


question correctly

Which of the following is not considered a major branch of the descending thoracic aorta?

D. Posterior intercostal artery


E. Oesophageal artery

6
7
8
9

Search
Next question
Go

10

The inferior thyroid artery is usually derived from the thyrocervical trunk, a branch of the
subclavian artery.
Thoracic aorta

Origin

T4

Terminates

T12

Relations

Branches

Anteriorly (from top to bottom)-root of the left lung, the pericardium, the
oesophagus, and the diaphragm
Posteriorly-vertebral column, azygos vein
Right- hemiazygos veins, thoracic duct
Left- left pleura and lung

Lateral segmental branches: Posterior intercostal arteries


Lateral visceral: Bronchial arteries supply bronchial walls and lung
excluding the alveoli
Midline branches: Oesophageal arteries

Rate question:

Next question

Comment on this question

All contents of this site are 2012 E-Medical Revision Ltd

http://www.emrcs.com/question/question.php?q=0

Terms and Conditions

Privacy policy

1/1

17/05/2015

Reference ranges

Question 11 of 368

Previous

Next

An 18 year old lady with troublesome hyperhidrosis of the hands and arms is due to undergo
a sympathectomy to treat the condition. Which of the following should the surgeons divide to
most effectively treat her condition?

Question stats

Score: 100%
1

13.7%

30.2%

22.8%

16.5%

16.7%

A. Sympathetic ganglia at T1, T2 and T3


30.2% of users answered this
question correctly

B. Sympathetic ganglia at T2 and T3

End and review

6
7

C. Sympathetic ganglia at T1 and T2


8

D. Stellate ganglion
9

Search

E. Superior cervical ganglion

Go

10
11

Next question

To treat hyperhidrosis the sympathetic ganglia at T2 and T3 should be divided. Dividing the
other structures listed would either carry a risk of Horners syndrome or be ineffective.
Sympathetic nervous system- anatomy
The cell bodies of the pre-ganglionic efferent neurones lie in the lateral horn of the grey
matter of the spinal cord in the thoraco-lumbar regions.
The pre-ganglionic efferents leave the spinal cord at levels T1-L2. These pass to the
sympathetic chain.
Lateral branches of the sympathetic chain connect it to every spinal nerve. These post
ganglionic nerves will pass to structures that receive sympathetic innervation at the
periphery.
Sympathetic chains
These lie on the vertebral column and run from the base of the skull to the coccyx.
Cervical
region

Lie anterior to the transverse processes of the cervical vertebrae and posterior to the
carotid sheath.

Thoracic
region

Lie anterior to the neck of the upper ribs and and lateral sides of the lower thoracic
vertebrae.They are covered by the parietal pleura

Lumbar
region

Enter by passing posterior to the medial arcuate ligament. Lie anteriorly to the
vertebrae and medial to psoas major.

Sympathetic ganglia
Superior cervical ganglion lies anterior to C2 and C3.
Middle cervical ganglion (if present) C6
Stellate ganglion- anterior to transverse process of C7, lies posterior to the subclavian
artery, vertebral artery and cervical pleura.
Thoracic ganglia are segmentally arranged.
There are usually 4 lumbar ganglia.

Clinical importance
Interruption of the head and neck supply of the sympathetic nerves will result in an
ipsilateral Horners syndrome.
For treatment of hyperhidrosis the sympathetic denervation can be achieved by
removing the second and third thoracic ganglia with their rami. Removal of T1 will
cause a Horners syndrome and is therefore not performed.
In patients with vascular disease of the lower limbs a lumbar sympathetomy may be
performed, either radiologically or (more rarely now) surgically. The ganglia of L2 and
below are disrupted. If L1 is removed then ejaculation may be compromised (and little
additional benefit conferred as the preganglionic fibres do not arise below L2.

Rate question:

Next question

Comment on this question

All contents of this site are 2012 E-Medical Revision Ltd

http://www.emrcs.com/question/question.php?q=0

Terms and Conditions

Privacy policy

1/1

17/05/2015

Reference ranges

Previous

Question 12 of 368

Next

A 44 year old lady is recovering following a transphenoidal hypophysectomy. Unfortunately


there is a post operative haemorrhage. Which of the following features is most likely to occur
initially?

Question stats

Score: 86.7%
1

14.3%

22.8%

45.7%

9.7%

7.6%

A. Cavernous sinus thrombosis


45.7% of users answered this
question correctly

B. Abducens nerve palsy

End and review

6
7

C. Bi-temporal homonymous hemianopia


8

D. Inferior homonymous hemianopia


9

Search

E. Central retinal vein occlusion

Go

10
11

Next question

12

Theme from April 2014 exam


The pituitary is covered by a sheath of dura and an expanding haematoma at this site may
compress the optic chiasm in the same manner as an expanding pituitary tumour.
Pituitary Gland
The pituitary gland is located within the sella turcica within the sphenoid bone in the middle
cranial fossa. It is covered by a dural fold and weighs around 0.5g. It is attached to the
hypothalamus by the infundibulum. The anterior pituitary receives hormonal stimuli from the
hypothalamus by way of the hypothalamo-pituitary portal system. It develops from a
depression in the wall of the pharynx (Rathkes pouch).
Anterior pituitary hormones
Growth hormone
Thyroid stimulating hormone
ACTH
Prolactin
LH and FSH
Melanocyte releasing hormone

Posterior pituitary hormones


Oxytocin
Anti diuretic hormone

Rate question:

Next question

Comment on this question

All contents of this site are 2012 E-Medical Revision Ltd

http://www.emrcs.com/question/question.php?q=0

Terms and Conditions

Privacy policy

1/1

17/05/2015

Reference ranges

Previous

Question 13 of 368

Next

During a right hemicolectomy the caecum is mobilised. As the bowel is retracted medially a
vessel is injured, posterior to the colon. Which of the following is the most likely vessel?

A. Right colic artery


B. Inferior vena cava

Question stats

Score: 87.5%
1

27.2%

15.2%

7.5%

16.9%

33.2%

33.2% of users answered this


question correctly

C. Aorta

End and review

D. External iliac artery

6
7
8

E. Gonadal vessels

Search
Go

Next question

10
11
12

The gonadal vessels and ureter are important posterior relations that are at risk during a
right hemicolectomy.

13

Caecum

Location

Proximal right colon below the ileocaecal valve


Intraperitoneal

Posterior relations

Psoas
Iliacus
Femoral nerve
Genitofemoral nerve
Gonadal vessels

Anterior relations

Greater omentum

Arterial supply

Ileocolic artery

Lymphatic drainage

Mesenteric nodes accompany the venous drainage

The caecum is the most distensible part of the colon and in complete large bowel
obstruction with a competent ileocaecal valve the most likely site of eventual
perforation.

Rate question:

Next question

Comment on this question

All contents of this site are 2012 E-Medical Revision Ltd

http://www.emrcs.com/question/question.php?q=0

Terms and Conditions

Privacy policy

1/1

17/05/2015

Reference ranges

Previous

Question 14 of 368

Next

A 53 year old man with a carcinoma of the lower third of the oesophagus is undergoing an
oesophagogastrectomy. As the surgeons mobilise the lower part of the oesophagus, where
are they most likely to encounter the thoracic duct?

Question stats

Score: 82.4%
1

10.3%

26.4%

22.9%

12.2%

28.2%

A. Anterior to the oesophagus


28.2% of users answered this
question correctly

B. On the left side of the oesophagus

End and review

6
7

C. On the right side of the oesophagus


8

D. Immediately anterior to the azygos vein


9

Search

E. Posterior to the oesophagus

Go

10
11

Next question

12

The thoracic duct lies posterior to the oesophagus and passes to the left at the level of the
Angle of Louis. It enters the thorax at T12 together with the aorta.

13
14

Thoracic duct

Continuation of the cisterna chyli in the abdomen.


Enters the thorax at T12
Lies posterior to the oesophagus for most of its intrathoracic course. Passes to the left
at T5.
Lymphatics draining the left side of the head and neck join the thoracic duct prior to its
insertion into the left brachiocephalic vein.
Lymphatics draining the right side of the head and neck drain via the subclavian and
jugular trunks into the right lymphatic duct and thence into the mediastinal trunk and
eventually the right brachiocephalic vein.
Its location in the thorax makes it prone to injury during oesophageal surgery. Some
surgeons administer cream to patients prior to oesophagectomy so that it is easier to
identify the cut ends of the duct.

Rate question:

Next question

Comment on this question

All contents of this site are 2012 E-Medical Revision Ltd

http://www.emrcs.com/question/question.php?q=0

Terms and Conditions

Privacy policy

1/1

17/05/2015

Reference ranges

Previous

Question 15 of 368

Next

Question stats

End and review

Score: 83.3%
1

45.7%

9.5%

17.9%

A. L4 to S3

11.5%

B. L1 to L4

15.3%

C. L3 to S1

45.7% of users answered this


question correctly

Which of the following represents the root values of the sciatic nerve?

D. S1 to S4
E. L5 to S1

6
7
8
9

Search
Next question
Go

10
11

Theme from April 2014 exam


The sciatic nerve most commonly arises from L4 to S3.

12
13

Sciatic nerve

14
15

The sciatic nerve is formed from the sacral plexus and is the largest nerve in the body. It is
the continuation of the main part of the plexus arising from ventral rami of L4 to S3. These
rami converge at the inferior border of piriformis to form the nerve itself. It passes through
the inferior part of the greater sciatic foramen and emerges beneath piriformis. Medially, lie
the inferior gluteal nerve and vessels and the pudendal nerve and vessels. It runs
inferolaterally under the cover of gluteus maximus midway between the greater trochanter
and ischial tuberosity. It receives its blood supply from the inferior gluteal artery. The nerve
provides cutaneous sensation to the skin of the foot and the leg. It also innervates the
posterior thigh muscles and the lower leg and foot muscles. The nerve splits into the tibial
and common peroneal nerves approximately half way down the posterior thigh. The tibial
nerve supplies the flexor muscles and the common peroneal nerve supplies the extensor
muscles and the abductor muscles.
Summary points
Origin

Spinal nerves L4 - S3

Articular Branches

Hip joint

Muscular branches in
upper leg

Semitendinosus
Semimembranosus
Biceps femoris
Part of adductor magnus

Cutaneous sensation

Terminates

Posterior aspect of thigh


Gluteal region
Entire lower leg (except the medial aspect)
At the upper part of the popliteal fossa by dividing into the tibial
and peroneal nerves

The nerve to the short head of the biceps femoris comes from the common peroneal
part of the sciatic and the other muscular branches arise from the tibial portion.
The tibial nerve goes on to innervate all muscles of the foot except the extensor
digitorum brevis (which is innervated by the common peroneal nerve).

Rate question:

Next question

Comment on this question

All contents of this site are 2012 E-Medical Revision Ltd

http://www.emrcs.com/question/question.php?q=0

Terms and Conditions

Privacy policy

1/1

17/05/2015

Reference ranges

Previous

Question 16 of 368

Next

Question stats

End and review

Score: 84.2%
1

13.1%

15.1%

10.5%

A. Peroneus longus

50.7%

B. Tibialis anterior

10.6%

C. Extensor hallucis longus

50.7% of users answered this


question correctly

The common peroneal nerve, or its branches, supply the following muscles except:

D. Flexor digitorum brevis


E. Extensor digitorum longus

6
7
8
9

Search
Next question
Go

10
11

Flexor digitorum is supplied by the tibial nerve.

12

Common peroneal nerve

13
14

Derived from the dorsal divisions of the sacral plexus (L4, L5, S1 and S2).

15
16

This nerve supplies the skin and fascia of the anterolateral surface of the leg and the
dorsum of the foot. It also innervates the muscles of the anterior and peroneal compartments
of the leg, extensor digitorum brevis as well as the knee, ankle and foot joints.
It is laterally placed within the sciatic nerve. From the bifurcation of the sciatic nerve it passes
inferolaterally in the lateral and proximal part of the popliteal fossa, under the cover of biceps
femoris and its tendon. To reach the posterior aspect of the fibular head. It ends by dividing
into the deep and superficial peroneal nerves at the point where it winds around the lateral
surface of the neck of the fibula in the body of peroneus longus, approximately 2cm distal to
the apex of the head of the fibula. It is palpable posterior to the head of the fibula.
Branches
In the thigh

Nerve to the short head of biceps


Articular branch (knee)

In the popliteal fossa

Lateral cutaneous nerve of the calf

Neck of fibula

Superficial and deep peroneal nerves

Rate question:

Next question

Comment on this question

All contents of this site are 2012 E-Medical Revision Ltd

http://www.emrcs.com/question/question.php?q=0

Terms and Conditions

Privacy policy

1/1

17/05/2015

Reference ranges

Question 17 of 368

Previous

Next

An 83 year old lady presents with a femoral hernia and undergoes a femoral hernia repair.
Which of the following forms the posterior wall of the femoral canal?

A. Pectineal ligament
B. Lacunar ligament

Question stats

Score: 85.7%
1

47.2%

14.7%

9.2%

20.6%

8.4%

47.2% of users answered this


question correctly

C. Inguinal ligament

End and review

D. Adductor longus

6
7
8

E. Sartorius

Search
Next question

Go

10
11
12
13

Femoral canal
The femoral canal lies at the medial aspect of the femoral sheath. The femoral sheath is a
fascial tunnel containing both the femoral artery laterally and femoral vein medially. The
canal lies medial to the vein.

14
15
16
17

Borders of the femoral canal


Laterally

Femoral vein

Medially

Lacunar ligament

Anteriorly

Inguinal ligament

Posteriorly

Pectineal ligament

Image showing dissection of femoral canal

Image sourced from Wikipedia

Contents
Lymphatic vessels
Cloquet's lymph node

Physiological significance
Allows the femoral vein to expand to allow for increased venous return to the lower limbs.
Pathological significance
As a potential space, it is the site of femoral hernias. The relatively tight neck places these at
high risk of strangulation.

http://www.emrcs.com/question/question.php?q=0

1/2

17/05/2015

Rate question:

Next question

Comment on this question

All contents of this site are 2012 E-Medical Revision Ltd

http://www.emrcs.com/question/question.php?q=0

Terms and Conditions

Privacy policy

2/2

17/05/2015

Reference ranges

Previous

Question 18 of 368

Next

A 45 year man presents with hand weakness. He is given a piece of paper to hold between
his thumb and index finger. When the paper is pulled, the patient has difficulty maintaining a
grip. Grip pressure is maintained by flexing the thumb at the interphalangeal joint. What is
the most likely nerve lesion?

Question stats

Score: 86.4%
1

11.7%

44.8%

20%

12.1%

11.5%

44.8% of users answered this


question correctly

A. Posterior interosseous nerve

End and review

B. Deep branch of ulnar nerve

6
7
8

C. Anterior interosseous nerve

Search

D. Superficial branch of the ulnar nerve


E. Radial nerve

Go

10
11

Next question

12
13

Theme from January 2012 exam


This is a description of Froment's sign, which tests for ulnar nerve palsy. It mainly tests for
the function of adductor pollicis. This is supplied by the deep branch of the ulnar nerve.
Remember the anterior interosseous branch, which innervates the flexor pollicis longus
(hence causing flexion of the thumb IP joint), branches off more proximally to the wrist.

14
15
16
17
18

Ulnar nerve
Origin
C8, T1

Supplies (no muscles in the upper arm)


Flexor carpi ulnaris
Flexor digitorum profundus
Flexor digiti minimi
Abductor digiti minimi
Opponens digiti minimi
Adductor pollicis
Interossei muscle
Third and fourth lumbricals
Palmaris brevis

Path
Posteromedial aspect of ulna to flexor compartment of forearm, then along the ulnar.
Passes beneath the flexor carpi ulnaris muscle, then superficially through the flexor
retinaculum into the palm of the hand.

http://www.emrcs.com/question/question.php?q=0

1/2

17/05/2015
Image sourced from Wikipedia

Branches
Branch

Supplies

Articular branch

Flexor carpi ulnaris


Medial half of the flexor digitorum profundus

Palmar cutaneous branch (Arises near the


middle of the forearm)

Skin on the medial part of the palm

Dorsal cutaneous branch

Dorsal surface of the medial part of the hand

Superficial branch

Cutaneous fibres to the anterior surfaces of the


medial one and one-half digits

Deep branch

Hypothenar muscles
All the interosseous muscles
Third and fourth lumbricals
Adductor pollicis
Medial head of the flexor pollicis brevis

Effects of injury
Damage at the wrist

Damage at the elbow

Wasting and paralysis of intrinsic hand muscles (claw hand)


Wasting and paralysis of hypothenar muscles
Loss of sensation medial 1 and half fingers

Radial deviation of the wrist


Clawing less in 3rd and 4th digits

Rate question:

Next question

Comment on this question

All contents of this site are 2012 E-Medical Revision Ltd

http://www.emrcs.com/question/question.php?q=0

Terms and Conditions

Privacy policy

2/2

17/05/2015

Reference ranges

Question 19 of 368

Previous

Next

Question stats

End and review

Score: 87.5%
1

16.1%

27.5%

19.6%

A. It lies deep to the prevertebral layer of deep cervical fascia

16.1%

B. Crosses posterior to the 2nd part of the subclavian artery

20.7%

C. It runs on the anterior surface of the scalene muscle

27.5% of users answered this


question correctly

Which of the following statements relating to the right phrenic nerve is false?

D. On the right side it leaves the mediastinum via the vena cava hiatus at
a level of T8

7
8

E. The right phrenic nerve passes over the right atrium

Search
Next question

Go

10
11
12
13

Phrenic nerve

14
15

Origin

16

C3,4,5
17
18

Supplies
19

Diaphragm, sensation central diaphragm and pericardium

Path
The phrenic nerve passes with the internal jugular vein across scalenus anterior. It
passes deep to prevertebral fascia of deep cervical fascia.
Left: crosses anterior to the 1st part of the subclavian artery.
Right: Anterior to scalenus anterior and crosses anterior to the 2nd part of the
subclavian artery.
On both sides, the phrenic nerve runs posterior to the subclavian vein and posterior to
the internal thoracic artery as it enters the thorax.

Right phrenic nerve


In the superior mediastinum: anterior to right vagus and laterally to superior vena cava
Middle mediastinum: right of pericardium
It passes over the right atrium to exit the diaphragm at T8

Left phrenic nerve


Passes lateral to the left subclavian artery, aortic arch and left ventricle
Passes anterior to the root of the lung
Pierces the diaphragm alone

Image showing the passage of the phrenic nerve in the neck

Image sourced from Wikipedia

Rate question:

http://www.emrcs.com/question/question.php?q=0

Next question

1/2

17/05/2015

Comment on this question

All contents of this site are 2012 E-Medical Revision Ltd

http://www.emrcs.com/question/question.php?q=0

Terms and Conditions

Privacy policy

2/2

17/05/2015

Reference ranges

Previous

Question 20 of 368

Next

Question stats

End and review

Score: 88%
1

16%

13.9%

19.3%

A. Carotid canal and internal carotid artery.

39.8%

B. Foramen ovale and mandibular nerve.

11%

C. Optic canal and ophthalmic artery.

39.8% of users answered this


question correctly

Which of the following cranial foramina pairings are incorrect?

D. Optic canal and ophthalmic nerve.


E. Foramen rotundum and maxillary nerve.

6
7
8
9

Search
Next question
Go

Question derived from 2010 and 2011 exams

10
11
12

The optic canal transmits the optic nerve. The ophthalmic nerve traverses the superior
orbital fissure.

13

Foramina of the base of the skull

15

14

16

Foramen

Location

Contents

Foramen
ovale

Sphenoid
bone

Otic ganglion
V3 (Mandibular nerve:3rd branch of
trigeminal)
Accessory meningeal artery
Lesser petrosal nerve
Emissary veins

17
18

Foramen
spinosum

Sphenoid
bone

Middle meningeal artery


Meningeal branch of the Mandibular nerve

Foramen
rotundum

Sphenoid
bone

Maxillary nerve (V2)

Foramen
lacerum/
carotid canal

Sphenoid
bone

Base of the medial pterygoid plate.


Internal carotid artery*
Nerve and artery of the pterygoid canal

Jugular
foramen

Temporal
bone

Anterior: inferior petrosal sinus


Intermediate: glossopharyngeal, vagus, and accessory nerves.
Posterior: sigmoid sinus (becoming the internal jugular vein) and
some meningeal branches from the occipital and ascending
pharyngeal arteries.

Foramen
magnum

Occipital
bone

Anterior and posterior spinal arteries


Vertebral arteries
Medulla oblongata

Stylomastoid
foramen

Temporal
bone

Stylomastoid artery
Facial nerve

Superior
orbital fissure

Sphenoid
bone

Oculomotor nerve (III)


trochlear nerve (IV)
lacrimal, frontal and nasociliary branches of ophthalmic nerve (V1)
abducent nerve (VI)
Superior and inferior ophthalmic vein

19
20

*= In life the foramen lacerum is occluded by a cartilagenous plug. The ICA initially passes
into the carotid canal which ascends superomedially to enter the cranial cavity through the
foramen lacerum.
Base of skull anatomical overview

http://www.emrcs.com/question/question.php?q=0

1/2

17/05/2015

Image sourced from Wikipedia

Rate question:

Next question

Comment on this question

All contents of this site are 2012 E-Medical Revision Ltd

http://www.emrcs.com/question/question.php?q=0

Terms and Conditions

Privacy policy

2/2

17/05/2015

Reference ranges

Question 21 of 368

Previous

Next

A 22 year old man is involved in a fight and sustains a stab wound in his upper forearm. On
examination there is a small, but deep laceration. There is an obvious loss of pincer
movement involving the thumb and index finger with minimal loss of sensation. The most
likely nerve injury is to the:

Question stats

Score: 84.6%
1

25.7%

11.4%

31.8%

6.2%

24.9%

31.8% of users answered this


question correctly

A. Ulnar nerve

End and review

B. Radial nerve

6
7
8

C. Anterior interosseous nerve

Search

D. Axillary nerve
E. Median nerve

Go

10
11

Next question

12
13

The anterior interosseous nerve is a motor branch of the median nerve just below the elbow.
When damaged it classically causes:

14
15

Pain in the forearm


Loss of pincer movement of the thumb and index finger (innervates the long flexor
muscles of flexor pollicis longus & flexor digitorum profundus of the index and middle
finger)
Minimal loss of sensation due to lack of a cutaneous branch

16
17
18
19
20
21

Median nerve
The median nerve is formed by the union of a lateral and medial root respectively from the
lateral (C5,6,7) and medial (C8 and T1) cords of the brachial plexus; the medial root passes
anterior to the third part of the axillary artery. The nerve descends lateral to the brachial
artery, crosses to its medial side (usually passing anterior to the artery). It passes deep to
the bicipital aponeurosis and the median cubital vein at the elbow.
It passes between the two heads of the pronator teres muscle, and runs on the deep surface
of flexor digitorum superficialis (within its fascial sheath).
Near the wrist it becomes superficial between the tendons of flexor digitorum superficialis and
flexor carpi radialis, deep to palmaris longus tendon. It passes deep to the flexor retinaculum
to enter the palm, but lies anterior to the long flexor tendons within the carpal tunnel.
Branches
Region

Branch

Upper
arm

No branches, although the nerve commonly communicates with the


musculocutaneous nerve

Forearm

Pronator teres
Flexor carpi radialis
Palmaris longus
Flexor digitorum superficialis
Flexor pollicis longus
Flexor digitorum profundus (only the radial half)

Distal
forearm

Palmar cutaneous branch

Hand
(Motor)

Motor supply (LOAF)

Hand
(Sensory)

Lateral 2 lumbricals
Opponens pollicis
Abductor pollicis brevis
Flexor pollicis brevis

Over thumb and lateral 2 fingers


On the palmar aspect this projects proximally, on the dorsal aspect only the
distal regions are innervated with the radial nerve providing the more proximal
cutaneous innervation.

Patterns of damage
Damage at wrist
e.g. carpal tunnel syndrome
paralysis and wasting of thenar eminence muscles and opponens pollicis (ape hand

http://www.emrcs.com/question/question.php?q=0

1/2

17/05/2015

deformity)
sensory loss to palmar aspect of lateral (radial) 2 fingers

Damage at elbow, as above plus:


unable to pronate forearm
weak wrist flexion
ulnar deviation of wrist

Anterior interosseous nerve (branch of median nerve)


leaves just below the elbow
results in loss of pronation of forearm and weakness of long flexors of thumb and index
finger

Topography of the median nerve

Image sourced from Wikipedia

Rate question:

Next question

Comment on this question

All contents of this site are 2012 E-Medical Revision Ltd

http://www.emrcs.com/question/question.php?q=0

Terms and Conditions

Privacy policy

2/2

17/05/2015

Reference ranges

Previous

Question 22 of 368

Next

A 66 year old man is undergoing a left nephro-ureterectomy. The surgeons remove the
ureter, which of the following is responsible for the blood supply to the proximal ureter?

A. Branches of the renal artery


B. External iliac artery

Question stats

Score: 85.7%
1

50.5%

9.6%

12.4%

19.1%

8.4%

50.5% of users answered this


question correctly

C. Internal iliac artery

End and review

D. Direct branches from the aorta

6
7
8

E. Common iliac artery

Search
Go

Next question

10
11

Theme from April 2014 exam


The proximal ureter is supplied by branches from the renal artery. For the other feeding
vessels - see below.

12

Ureter

15

13
14

16
17

25-35 cm long
Muscular tube lined by transitional epithelium
Surrounded by thick muscular coat. Becomes 3 muscular layers as it crosses the bony
pelvis
Retroperitoneal structure overlying transverse processes L2-L5
Lies anterior to bifurcation of iliac vessels
Blood supply is segmental; renal artery, aortic branches, gonadal branches, common
iliac and internal iliac
Lies beneath the uterine artery

Rate question:

18
19
20
21
22

Next question

Comment on this question

All contents of this site are 2012 E-Medical Revision Ltd

http://www.emrcs.com/question/question.php?q=0

Terms and Conditions

Privacy policy

1/1

17/05/2015

Reference ranges

Previous

Question 23 of 368

Next

Question stats

End and review

Score: 86.2%
1

8.7%

11.7%

11.1%

A. Peroneus brevis tendon

8.6%

B. Sural nerve

59.9%

C. Short saphenous vein

59.9% of users answered this


question correctly

Which of the following structures does not pass behind the lateral malleolus?

D. Peroneus longus tendon


E. Tibialis anterior tendon

6
7
8
9

Search
Next question
Go

10
11

Theme from April 2014 exam


Tibialis anterior tendon passes anterior to the medial malleolus.

12
13

Lateral malleolus

14
15

Structures posterior to the lateral malleolus and superficial to superior peroneal


retinaculum

16
17

Sural nerve
Short saphenous vein

18
19
20

Structures posterior to the lateral malleolus and deep to superior peroneal


retinaculum

21
22

Peroneus longus tendon


Peroneus brevis tendon

23

The calcaneofibular ligament is attached at the lateral malleolus


Rate question:

Next question

Comment on this question

All contents of this site are 2012 E-Medical Revision Ltd

http://www.emrcs.com/question/question.php?q=0

Terms and Conditions

Privacy policy

1/1

17/05/2015

Reference ranges

Previous

Question 24 of 368

Next

A 78 year old man presents with symptoms consistent with intermittent claudication. To
assess the severity of his disease you decide to measure his ankle brachial pressure index.
To do this you will identify the dorsalis pedis artery. Which of the following statements
relating to this vessel is false?

Question stats

Score: 86.7%
1

46.6%

15.3%

9.8%

15%

13.3%

46.6% of users answered this


question correctly

A. It originates from the peroneal artery

End and review

B. It is crossed by the tendon of extensor hallucis brevis

6
7
8

C. Two veins are usually closely related to it

Search

D. It passes under the inferior extensor retinaculum


E. The tendon of extensor hallucis longus lies medial to it.

Go

10
11

Next question

12
13

The dorsalis pedis artery is a direct continuation of the anterior tibial artery.

14
15

Foot- anatomy

16

Arches of the foot


The foot is conventionally considered to have two arches.
The longitudinal arch is higher on the medial than on the lateral side. The posterior
part of the calcaneum forms a posterior pillar to support the arch. The lateral part of
this structure passes via the cuboid bone and the lateral two metatarsal bones. The
medial part of this structure is more important. The head of the talus marks the summit
of this arch, located between the sustentaculum tali and the navicular bone. The
anterior pillar of the medial arch is composed of the navicular bone, the three
cuneiforms and the medial three metatarsal bones.
The transverse arch is situated on the anterior part of the tarsus and the posterior
part of the metatarsus. The cuneiforms and metatarsal bases narrow inferiorly, which
contributes to the shape of the arch.

17
18
19
20
21
22
23
24

Intertarsal joints
Sub talar joint

Formed by the cylindrical facet on the lower surface of the body of


the talus and the posterior facet on the upper surface of the
calcaneus. The facet on the talus is concave anteroposteriorly, the
other is convex. The synovial cavity of this joint does not
communicate with any other joint.

Talocalcaneonavicular
joint

The anterior part of the socket is formed by the concave articular


surface of the navicular bone, posteriorly by the upper surface of the
sustentaculum tali. The talus sits within this socket

Calcaneocuboid joint

Highest point in the lateral part of the longitudinal arch. The lower
aspect of this joint is reinforced by the long plantar and plantar
calcaneocuboid ligaments.

Transverse tarsal joint

The talocalcaneonavicular joint and the calcaneocuboid joint extend


across the tarsus in an irregular transverse plane, between the talus
and calcaneus behind and the navicular and cuboid bones in front.
This plane is termed the transverse tarsal joint.

Cuneonavicular joint

Formed between the convex anterior surface of the navicular bone


and the concave surface of the the posterior ends of the three
cuneiforms.

Intercuneiform joints

Between the three cuneiform bones.

Cuneocuboid joint

Between the circular facets on the lateral cuneiform bone and the
cuboid. This joint contributes to the tarsal part of the transverse arch.

A detailed knowledge of the joints is not required for MRCS Part A. However, the contribution
they play to the overall structure of the foot should be appreciated
Ligaments of the ankle joint and foot

http://www.emrcs.com/question/question.php?q=0

1/3

17/05/2015

Image sourced from Wikipedia

Muscles of the foot


Muscle

Origin

Insertion

Nerve
supply

Action

Abductor
hallucis

Medial side of the calcaneus,


flexor retinaculum, plantar
aponeurosis

Medial side of
the base of
the proximal
phalanx

Medial
plantar
nerve

Abducts the great


toe

Flexor
digitorum
brevis

Medial process of the


calcaneus, plantar
eponeurosis.

Via 4 tendons
into the
middle
phalanges of
the lateral 4
toes.

Medial
plantar
nerve

Flexes all the joints


of the lateral 4 toes
except for the
interphalangeal joint.

Abductor
digit
minimi

From the tubercle of the


calcaneus and from the
plantar aponeurosis

Together with
flexor digit
minimi brevis
into the lateral
side of the
base of the
proximal
phalanx of the
little toe

Lateral
plantar
nerve

Abducts the little toe


at the
metatarsophalangeal
joint

Flexor
hallucis
brevis

From the medial side of the


plantar surface of the cuboid
bone, from the adjacent part
of the lateral cuneiform bone
and from the tendon of tibialis
posterior.

Into the
proximal
phalanx of the
great toe, the
tendon
contains a
sesamoid
bone

Medial
plantar
nerve

Flexes the
metatarsophalangeal
joint of the great toe.

Adductor
hallucis

Arises from two heads. The


oblique head arises from the
sheath of the peroneus longus
tendon, and from the plantar
surfaces of the bases of the
2nd, 3rd and 4th metatarsal
bones. The transverse head
arises from the plantar
surface of the lateral 4
metatarsophalangeal joints
and from the deep transverse
metatarsal ligament.

Lateral side of
the base of
the proximal
phalanx of the
great toe.

Lateral
plantar
nerve

Adducts the great


toe towards the
second toe. Helps
maintain the
transverse arch of
the foot.

Extensor
digitorum
brevis

On the dorsal surface of the


foot from the upper surface of
the calcaneus and its
associated fascia

Via four thin


tendons which
run forward
and medially
to be inserted
into the
medial four
toes. The
lateral three
tendons join
with hoods of
extensor
digitorum
longus.

Deep
peroneal

Extend the
metatarsophalangeal
joint of the medial
four toes. It is
unable to extend the
interphalangeal joint
without the
assistance of the
lumbrical muscles.

Detailed knowledge of the foot muscles are not needed for the MRCS part A
Nerves in the foot

http://www.emrcs.com/question/question.php?q=0

2/3

17/05/2015

Lateral plantar nerve


Passes anterolaterally towards the base of the 5th metatarsal between flexor digitorum brevis
and flexor accessorius. On the medial aspect of the lateral plantar artery. At the base of the
5th metatarsal it splits into superficial and deep branches.
Medial plantar nerve
Passes forwards with the medial plantar artery under the cover of the flexor retinaculum to
the interval between abductor hallucis and flexor digitorum brevis on the sole of the foot.
Plantar arteries
Arise under the cover of the flexor retinaculum, midway between the tip of the medial
malleolus and the most prominent part of the medial side of the heel.

Medial plantar artery. Passes forwards medial to medial plantar nerve in the space
between abductor hallucis and flexor digitorum brevis.Ends by uniting with a branch of
the 1st plantar metatarsal artery.
Lateral plantar artery. Runs obliquely across the sole of the foot. It lies lateral to the
lateral plantar nerve. At the base of the 5th metatarsal bone it arches medially across
the foot on the metatarsals

Dorsalis pedis artery


This vessel is a direct continuation of the anterior tibial artery. It commences on the front of
the ankle joint and runs to the proximal end of the first metatarsal space. Here is gives off the
arcuate artery and continues forwards as the first dorsal metatarsal artery. It is accompanied
by two veins throughout its length. It is crossed by the extensor hallucis brevis
Rate question:

Next question

Comment on this question

All contents of this site are 2012 E-Medical Revision Ltd

http://www.emrcs.com/question/question.php?q=0

Terms and Conditions

Privacy policy

3/3

17/05/2015

Reference ranges

Question 25 of 368

Previous

Next

Question stats

End and review

Score: 83.9%
1

12.9%

12.2%

43.4%

A. Vagus nerve

12.2%

B. Submandibular gland

19.3%

C. Phrenic nerve

43.4% of users answered this


question correctly

Which of the following is not a content of the anterior triangle of the neck?

D. Internal jugular vein


E. Hypoglossal nerve

6
7
8
9

Search
Next question
Go

10
11

The phrenic nerve is a content of the posterior triangle. The anterior triangle contains the
carotid sheath and its contents.

12
13

Anterior triangle of the neck

14
15

Boundaries

16
17

Anterior border of the Sternocleidomastoid


Lower border of mandible
Anterior midline

18
19
20

Sub triangles (divided by Digastric above and Omohyoid)

21

Muscular triangle: Neck strap muscles


Carotid triangle: Carotid sheath
Submandibular Triangle (digastric)

22
23
24

Contents of the anterior triangle

25

Digastric triangle

Submandibular gland
Submandibular nodes
Facial vessels
Hypoglossal nerve

Muscular triangle

Strap muscles
External jugular vein

Carotid triangle

Carotid sheath (Common carotid, vagus and internal jugular vein)


Ansa cervicalis

Nerve supply to digastric muscle


Anterior: Mylohyoid nerve
Posterior: Facial nerve

Image sourced from Wikipedia

Rate question:

Next question

Comment on this question

All contents of this site are 2012 E-Medical Revision Ltd

http://www.emrcs.com/question/question.php?q=0

Terms and Conditions

Privacy policy

1/1

17/05/2015

Reference ranges

Previous

Question 26 of 368

Next

A 32 year old attends neurology clinic complaining of tingling in his hand. He has radial
deviation of his wrist and there is mild clawing of his fingers, with the 3rd and 4th digits being
relatively spared. What is the most likely lesion?

Question stats

Score: 81.3%
1

19.5%

45.5%

9.5%

12.3%

13.3%

A. Ulnar nerve damage at the wrist


45.5% of users answered this
question correctly

B. Ulnar nerve damage at the elbow

End and review

6
7

C. Radial nerve damage at the elbow


8

D. Median nerve damage at the wrist


9

Search

E. Median nerve damage at the elbow

Go

Next question

10
11
12

The ulnar paradox- the higher the lesion, the less the clawing of the fingers seen
clinically.

13
14
15

At the elbow the ulnar nerve lesion affects the flexor carpi ulnaris and flexor digitorum
profundus.

16
17
18

Ulnar nerve
19
20

Origin

21

C8, T1

22

Supplies (no muscles in the upper arm)

23
24

Flexor carpi ulnaris


Flexor digitorum profundus
Flexor digiti minimi
Abductor digiti minimi
Opponens digiti minimi
Adductor pollicis
Interossei muscle
Third and fourth lumbricals
Palmaris brevis

25
26

Path
Posteromedial aspect of ulna to flexor compartment of forearm, then along the ulnar.
Passes beneath the flexor carpi ulnaris muscle, then superficially through the flexor
retinaculum into the palm of the hand.

Image sourced from Wikipedia

http://www.emrcs.com/question/question.php?q=0

1/2

17/05/2015

Branches
Branch

Supplies

Articular branch

Flexor carpi ulnaris


Medial half of the flexor digitorum profundus

Palmar cutaneous branch (Arises near the


middle of the forearm)

Skin on the medial part of the palm

Dorsal cutaneous branch

Dorsal surface of the medial part of the hand

Superficial branch

Cutaneous fibres to the anterior surfaces of the


medial one and one-half digits

Deep branch

Hypothenar muscles
All the interosseous muscles
Third and fourth lumbricals
Adductor pollicis
Medial head of the flexor pollicis brevis

Effects of injury
Damage at the wrist

Damage at the elbow

Wasting and paralysis of intrinsic hand muscles (claw hand)


Wasting and paralysis of hypothenar muscles
Loss of sensation medial 1 and half fingers

Radial deviation of the wrist


Clawing less in 3rd and 4th digits

Rate question:

Next question

Comment on this question

All contents of this site are 2012 E-Medical Revision Ltd

http://www.emrcs.com/question/question.php?q=0

Terms and Conditions

Privacy policy

2/2

17/05/2015

Reference ranges

Previous

Question 27 of 368

Next

A 22 year old man is undergoing an endotracheal intubation. Which of the following vertebral
levels is consistent with the origin of the trachea?

A. C2
B. T1

Question stats

Score: 78.8%
1

12.2%

9.5%

40.5%

22.4%

15.5%

40.5% of users answered this


question correctly

C. C6

End and review

D. C4

6
7
8

E. C3

Search
Go

Next question

10
11
12

The trachea commences at C6. It terminates at the level of T5 (or T6 in tall subjects in deep
inspiration).

13
14

Trachea

15
16

Trachea
Location

C6 vertebra to the upper border of T5 vertebra (bifurcation)

Arterial and venous supply

Inferior thyroid arteries and the thyroid venous plexus.

Nerve

Branches of vagus, sympathetic and the recurrent nerves

17
18
19
20
21

Relations in the neck

22

Anterior(Superior to
inferior)

Posterior

23

Isthmus of the thyroid gland


Inferior thyroid veins
Arteria thyroidea ima (when that vessel exists)
Sternothyroid
Sternohyoid
Cervical fascia
Anastomosing branches between the anterior jugular
veins

24
25
26
27

Oesophagus.

Laterally

Common carotid arteries


Right and left lobes of the thyroid gland
Inferior thyroid arteries
Recurrent laryngeal nerves

Relations in the thorax


Anterior
Manubrium, the remains of the thymus, the aortic arch, left common carotid arteries,
and the deep cardiac plexus

Lateral
In the superior mediastinum, on the right side is the pleura and right vagus; on its left
side are the left recurrent nerve, the aortic arch, and the left common carotid and
subclavian arteries.

Rate question:

Next question

Comment on this question

All contents of this site are 2012 E-Medical Revision Ltd

http://www.emrcs.com/question/question.php?q=0

Terms and Conditions

Privacy policy

1/1

17/05/2015

Reference ranges

Previous

Question 28 of 368

Next

A young child undergoes a difficult craniotomy for fulminant mastoiditis and associated
abscess. During the procedure the trigeminal nerve is severely damaged within Meckels
cave. Which deficit is least likely to be present?

Question stats

Score: 74.3%
1

13.2%

16.4%

14.5%

26.2%

29.7%

A. Anaesthesia over the ipsilateral anterior aspect of the scalp


29.7% of users answered this
question correctly

B. Loss of the corneal reflex

End and review

6
7

C. Weakness of the ipsilateral masseter muscle


8

D. Anaesthesia of the anterior aspect of the tongue


9

Search

E. Anaesthesia over the entire ipsilateral side of the face

Go

Next question

10
11
12

The angle of the jaw is not innervated by sensory fibres of the trigeminal nerve and is spared
in this type of injury.
Remember the trigeminal nerve provides motor innervation to the muscles of mastication.
The close proximity of the site of injury to the motor fibres is likely to result in at least some
compromise of motor muscle function.

13
14
15
16
17

Trigeminal nerve

18

The trigeminal nerve is the main sensory nerve of the head. In addition to its major sensory
role, it also innervates the muscles of mastication.

19
20
21

Distribution of the trigeminal nerve

22

Sensory

Scalp
Face
Oral cavity (and teeth)
Nose and sinuses
Dura mater

23
24
25
26
27

Motor

Muscles of mastication
Mylohyoid
Anterior belly of digastric
Tensor tympani
Tensor palati

Autonomic connections (ganglia)

28

Ciliary
Sphenopalatine
Otic
Submandibular

Path
Originates at the pons
Sensory root forms the large, crescentic trigeminal ganglion within Meckel's cave, and
contains the cell bodies of incoming sensory nerve fibres. Here the 3 branches exit.
The motor root cell bodies are in the pons and the motor fibres are distributed via the
mandibular nerve. The motor root is not part of the trigeminal ganglion.

Branches of the trigeminal nerve


Ophthalmic nerve

Sensory only

Maxillary nerve

Sensory only

Mandibular nerve

Sensory and motor

Sensory
Ophthalmic

Exits skull via the superior orbital fissure


Sensation of: scalp and forehead, the upper eyelid, the conjunctiva and cornea of
the eye, the nose (including the tip of the nose, except alae nasi), the nasal
mucosa, the frontal sinuses, and parts of the meninges (the dura and blood
vessels).

Maxillary
nerve

Exit skull via the foramen rotundum


Sensation: lower eyelid and cheek, the nares and upper lip, the upper teeth and

http://www.emrcs.com/question/question.php?q=0

1/2

17/05/2015

gums, the nasal mucosa, the palate and roof of the pharynx, the maxillary,
ethmoid and sphenoid sinuses, and parts of the meninges.
Mandibular
nerve

Exit skull via the foramen ovale


Sensation: lower lip, the lower teeth and gums, the chin and jaw (except the
angle of the jaw), parts of the external ear, and parts of the meninges.

Motor
Distributed via the mandibular nerve.
The following muscles of mastication are innervated:
Masseter
Temporalis
Medial pterygoid
Lateral pterygoid

Other muscles innervated include:


Tensor veli palatini
Mylohyoid
Anterior belly of digastric
Tensor tympani

Rate question:

Next question

Comment on this question

All contents of this site are 2012 E-Medical Revision Ltd

http://www.emrcs.com/question/question.php?q=0

Terms and Conditions

Privacy policy

2/2

17/05/2015

Reference ranges

Previous3 / 3

Question 29-31 of 368

Next

Question stats

End and review

Score: 76.3%

Average score for registered users:

1
2

Theme: Nerve lesions


A. Iliohypogastric nerve

29

41.4%

30

63.9%

31

84.7%

B. Ilioinguinal nerve

3
4
5

C. Lateral cutaneous nerve of the thigh

D. Femoral nerve

Search

E. Saphenous nerve

7
Go

F. Genitofemoral nerve

8
9

Please select the most likely nerve implicated in the situation described. Each option may be
used once, more than once or not at all.

10
11
12
13

29.

A 42 year old woman complains of a burning pain of her anterior thigh which
worsens on walking. There is a positive tinel sign over the inguinal ligament.

14
15

Lateral cutaneous nerve of the thigh

16

The lateral cutaneous nerve supplies sensation to the anterior and lateral
aspect of the thigh. Entrapment is commonly due to intra and extra pelvic
causes. Treatment involves local anaesthetic injections.

17
18
19

30.

A 29 year old woman has had a Pfannenstiel incision. She has pain over the
inguinal ligament which radiates to the lower abdomen. There is tenderness
when the inguinal canal is compressed.

20
21
22

Ilioinguinal nerve

23
24

31.

A 22 year man is shot in the groin. On examination he has weak hip flexion,
weak knee extension, and impaired quadriceps tendon reflex, as well as
sensory deficit in the anteromedial aspect of the thigh.

25
26
27

Femoral nerve

28
29-31 3 / 3

This is a classical description of a femoral nerve injury.

Next question

Nerve lesions during surgery

A variety of different procedures carry the risk of iatrogenic nerve injury. These are important
not only from the patients perspective but also from a medicolegal standpoint.
The following operations and their associated nerve lesions are listed here:
Posterior triangle lymph node biopsy and accessory nerve lesion.
Lloyd Davies stirrups and common peroneal nerve.
Thyroidectomy and laryngeal nerve.
Anterior resection of rectum and hypogastric autonomic nerves.
Axillary node clearance; long thoracic nerve, thoracodorsal nerve and
intercostobrachial nerve.
Inguinal hernia surgery and ilioinguinal nerve.
Varicose vein surgery- sural and saphenous nerves.
Posterior approach to the hip and sciatic nerve.
Carotid endarterectomy and hypoglossal nerve.

There are many more, with sound anatomical understanding of the commonly performed
procedures the incidence of nerve lesions can be minimised. They commonly occur when
surgeons operate in an unfamiliar tissue plane or by blind placement of haemostats (not
recommended).
Rate question:

Next question

Comment on this question

http://www.emrcs.com/question/question.php?q=0

1/2

17/05/2015

All contents of this site are 2012 E-Medical Revision Ltd

http://www.emrcs.com/question/question.php?q=0

Terms and Conditions

Privacy policy

2/2

17/05/2015

Reference ranges

Previous

Question 32 of 368

Next

Question stats

End and review

Score: 77.5%
1

7.3%

12.2%

18%

A. Facial artery

44.9%

B. Lingual artery

17.6%

C. Superior thyroid artery

44.9% of users answered this


question correctly

Which of the following is not a branch of the external carotid artery?

D. Mandibular artery
E. Maxillary artery

6
7
8
9

Search
Next question
Go

10
11

External carotid artery branches mnemonic:


'Some Angry Lady Figured Out PMS'

12
13
14

Superior thyroid (superior laryngeal artery branch)


Ascending pharyngeal
Lingual
Facial (tonsillar and labial artery)
Occipital
Posterior auricular
Maxillary (inferior alveolar artery, middle meningeal artery)
Superficial temporal

15
16
17
18
19
20
21
22
23

External carotid artery


The external carotid commences immediately lateral to the pharyngeal side wall. It ascends
and lies anterior to the internal carotid and posterior to the posterior belly of digastric and
stylohyoid. More inferiorly it is covered by sternocleidomastoid, passed by hypoglossal
nerves, lingual and facial veins.
It then pierces the fascia of the parotid gland finally dividing into its terminal branches within
the gland itself.

24
25
26
27
28
29-31 3 / 3
32

Surface marking of the carotid


This is an imaginary line drawn from the bifurcation of the common carotid passing behind
the angle of the jaw to a point immediately anterior to the tragus of the ear.
Branches of the external carotid artery
It has six branches, three in front, two behind and one deep.
Three in front

Superior thyroid
Lingual
Facial

Two behind

Occipital
Posterior auricular

Deep

Ascending pharyngeal

It terminates by dividing into the superficial temporal and maxillary arteries in the parotid
gland.

http://www.emrcs.com/question/question.php?q=0

1/2

17/05/2015

Image sourced from Wikipedia

Rate question:

Next question

Comment on this question

All contents of this site are 2012 E-Medical Revision Ltd

http://www.emrcs.com/question/question.php?q=0

Terms and Conditions

Privacy policy

2/2

17/05/2015

Reference ranges

Question 33 of 368

Previous

Next

A 23 year old man is stabbed in the groin, several structures are injured and the adductor
longus muscle has been lacerated. Which of the following nerves is responsible for the
innervation of adductor longus?

Question stats

Score: 78.6%
1

16.1%

56.9%

10.1%

8.3%

8.5%

A. Femoral nerve
56.9% of users answered this
question correctly

B. Obturator nerve

End and review

6
7

C. Sciatic nerve
8

D. Common peroneal nerve


9

Search

E. Ilioinguinal nerve

Go

10
11

Next question

12

The adductors are innervated by the obturator nerve

13
14

Adductor longus

15
16

Origin

Anterior body of pubis

Insertion

Middle third of linea aspera

18

Action

Adducts and flexes the thigh, medially rotate the hip

19

Innervation

Anterior division of obturator nerve (L2, L3, L4)

17

20
21
22

The schematic image below demonstrates the relationship of the adductor muscles

23
24
25
26
27
28
29-31 3 / 3
32
33

Image sourced from Wikipedia

Rate question:

Next question

Comment on this question

All contents of this site are 2012 E-Medical Revision Ltd

http://www.emrcs.com/question/question.php?q=0

Terms and Conditions

Privacy policy

1/1

17/05/2015

Reference ranges

Question 34 of 368

Previous

Next

Which of the following statements relating to the basilar artery and its branches is false?

A. The superior cerebellar artery may be decompressed to treat trigeminal


neuralgia
B. Occlusion of the posterior cerebral artery causes contralateral loss of
the visual field

Question stats

End and review

Score: 75%
1

17.8%

18%

16.7%

35.4%

12.2%

35.4% of users answered this


question correctly

C. The oculomotor nerve lies between the superior cerebellar and


posterior cerebral arteries

6
7
8

D. The posterior inferior cerebellar artery is the largest of the cerebellar


arteries arising from the basilar artery

Search
Go

E. The labyrinthine branch is accompanied by the facial nerve

10
11

Next question

12
13

The posterior inferior cerebellar artery is the largest of the cerebellar arteries arising from
the vertebral artery. The labyrinthine artery is long and slender and may arise from the lower
part of the basilar artery. It accompanies the facial and vestibulocochlear nerves into the
internal auditory meatus. The posterior cerebellar artery is often larger than the superior
cerebellar artery and it is separated from the vessel, near it's origin, by the oculomotor
nerve. Arterial decompression is a well established therapy for trigeminal neuralgia.
Circle of Willis

14
15
16
17
18
19
20

The two internal carotid arteries and two vertebral arteries form an anastomosis known as
the Circle of Willis on the inferior surface of the brain. Each half of the circle is formed by:
1. Anterior communicating artery
2. Anterior cerebral artery
3. Internal carotid artery
4. Posterior communicating artery
5. Posterior cerebral arteries and the termination of the basilar artery

21
22
23
24
25
26

The circle and its branches supply; the corpus striatum, internal capsule, diencephalon and
midbrain.

27
28
29-31 3 / 3
32
33
34
35

Image sourced from Wikipedia

Vertebral arteries
Enter the cranial cavity via foramen magnum
Lie in the subarachnoid space
Ascend on anterior surface of medulla oblongata
Unite to form the basilar artery at the base of the pons

Branches:

http://www.emrcs.com/question/question.php?q=0

1/2

17/05/2015

Posterior spinal artery


Anterior spinal artery
Posterior inferior cerebellar artery

Basilar artery
Branches:
Anterior inferior cerebellar artery
Labyrinthine artery
Pontine arteries
Superior cerebellar artery
Posterior cerebral artery

Internal carotid arteries


Branches:
Posterior communicating artery
Anterior cerebral artery
Middle cerebral artery
Anterior choroid artery

Rate question:

Next question

Comment on this question

All contents of this site are 2012 E-Medical Revision Ltd

http://www.emrcs.com/question/question.php?q=0

Terms and Conditions

Privacy policy

2/2

17/05/2015

Reference ranges

Previous

Question 35 of 368

Next

Question stats

End and review

Score: 76.1%
1

8.1%

48.1%

9.7%

A. Semimembranosus

7.9%

B. Quadriceps femoris

26.2%

C. Biceps femoris

48.1% of users answered this


question correctly

Which of the following muscles does not recieve any innervation from the sciatic nerve?

D. Semitendinosus
E. Adductor magnus

6
7
8
9

Search
Next question
Go

10
11

The sciatic nerve is traditionally viewed as being a nerve of the posterior compartment. It is
known to contribute to the innervation of adductor magnus (although the main innervation to
this muscle is from the obturator nerve). The quadriceps femoris is nearly always innervated
by the femoral nerve.

12
13
14
15

Sciatic nerve

16

The sciatic nerve is formed from the sacral plexus and is the largest nerve in the body. It is
the continuation of the main part of the plexus arising from ventral rami of L4 to S3. These
rami converge at the inferior border of piriformis to form the nerve itself. It passes through
the inferior part of the greater sciatic foramen and emerges beneath piriformis. Medially, lie
the inferior gluteal nerve and vessels and the pudendal nerve and vessels. It runs
inferolaterally under the cover of gluteus maximus midway between the greater trochanter
and ischial tuberosity. It receives its blood supply from the inferior gluteal artery. The nerve
provides cutaneous sensation to the skin of the foot and the leg. It also innervates the
posterior thigh muscles and the lower leg and foot muscles. The nerve splits into the tibial
and common peroneal nerves approximately half way down the posterior thigh. The tibial
nerve supplies the flexor muscles and the common peroneal nerve supplies the extensor
muscles and the abductor muscles.

17
18
19
20
21
22
23
24
25
26

Summary points

27

Origin

Spinal nerves L4 - S3

Articular Branches

Hip joint

28
29-31 3 / 3
32

Muscular branches in
upper leg

Cutaneous sensation

Terminates

33

Semitendinosus
Semimembranosus
Biceps femoris
Part of adductor magnus

34
35

Posterior aspect of thigh


Gluteal region
Entire lower leg (except the medial aspect)
At the upper part of the popliteal fossa by dividing into the tibial
and peroneal nerves

The nerve to the short head of the biceps femoris comes from the common peroneal
part of the sciatic and the other muscular branches arise from the tibial portion.
The tibial nerve goes on to innervate all muscles of the foot except the extensor
digitorum brevis (which is innervated by the common peroneal nerve).

Rate question:

Next question

Comment on this question

All contents of this site are 2012 E-Medical Revision Ltd

http://www.emrcs.com/question/question.php?q=0

Terms and Conditions

Privacy policy

1/1

17/05/2015

Reference ranges

Previous

Question 36 of 368

Next

A 23 year old man is involved in a fight and is stabbed in his upper arm. The ulnar nerve is
transected. Which of the following muscles will not demonstrate compromised function as a
result?

Question stats

Score: 77.1%
1

9.9%

13%

14.9%

11.1%

51.1%

A. Flexor carpi ulnaris


51.1% of users answered this
question correctly

B. Medial half of flexor digitorum profundus

End and review

6
7

C. Palmaris brevis
8

D. Hypothenar muscles
9

Search

E. Pronator teres

Go

Next question

10
11
12

M edial lumbricals
A dductor pollicis
F lexor digitorum profundus/Flexor digiti minimi
I nterossei
A bductor digiti minimi and opponens

13
14
15
16
17

Innervates all intrinsic muscles of the hand (EXCEPT 2: thenar muscles & first two
lumbricals - supplied by median nerve)

18
19
20

Pronator teres is innervated by the median nerve. Palmaris brevis is innervated by the ulnar
nerve

21
22

Ulnar nerve

23
24

Origin
C8, T1

25
26
27

Supplies (no muscles in the upper arm)


Flexor carpi ulnaris
Flexor digitorum profundus
Flexor digiti minimi
Abductor digiti minimi
Opponens digiti minimi
Adductor pollicis
Interossei muscle
Third and fourth lumbricals
Palmaris brevis

28
29-31 3 / 3
32
33
34
35
36

Path
Posteromedial aspect of ulna to flexor compartment of forearm, then along the ulnar.
Passes beneath the flexor carpi ulnaris muscle, then superficially through the flexor
retinaculum into the palm of the hand.

http://www.emrcs.com/question/question.php?q=0

1/2

17/05/2015

Image sourced from Wikipedia

Branches
Branch

Supplies

Articular branch

Flexor carpi ulnaris


Medial half of the flexor digitorum profundus

Palmar cutaneous branch (Arises near the


middle of the forearm)

Skin on the medial part of the palm

Dorsal cutaneous branch

Dorsal surface of the medial part of the hand

Superficial branch

Cutaneous fibres to the anterior surfaces of the


medial one and one-half digits

Deep branch

Hypothenar muscles
All the interosseous muscles
Third and fourth lumbricals
Adductor pollicis
Medial head of the flexor pollicis brevis

Effects of injury
Damage at the wrist

Damage at the elbow

Wasting and paralysis of intrinsic hand muscles (claw hand)


Wasting and paralysis of hypothenar muscles
Loss of sensation medial 1 and half fingers

Radial deviation of the wrist


Clawing less in 3rd and 4th digits

Rate question:

Next question

Comment on this question

All contents of this site are 2012 E-Medical Revision Ltd

http://www.emrcs.com/question/question.php?q=0

Terms and Conditions

Privacy policy

2/2

17/05/2015

Reference ranges

Previous

Question 37 of 368

Next

Question stats

End and review

Score: 74%
1

10.6%

18.7%

14.2%

A. Extensor retinaculum

30.7%

B. Bicipital aponeurosis

25.7%

C. Biceps muscle

25.7% of users answered this


question correctly

Which of the structures listed below overlies the cephalic vein?

D. Antebrachial fascia
E. None of the above

6
7
8
9

Search
Next question
Go

10
11

The cephalic vein is superficially located in the upper limb and overlies most the fascial
planes. It pierces the coracoid membrane (continuation of the clavipectoral fascia) to
terminate in the axillary vein. It lies anterolaterally to biceps.

12
13
14

Cephalic vein

15
16

Path

17

Dorsal venous arch drains laterally into the cephalic vein


Crosses the anatomical snuffbox and travels laterally up the arm
At the antecubital fossa connected to the basilic vein by the median cubital vein
Pierces deep fascia of deltopectoral groove to join axillary vein

18
19
20
21
22

Rate question:

Next question

23

Comment on this question

24
25
26
27
28
29-31 3 / 3
32
33
34
35
36
37

All contents of this site are 2012 E-Medical Revision Ltd

http://www.emrcs.com/question/question.php?q=0

Terms and Conditions

Privacy policy

1/1

17/05/2015

Reference ranges

Previous

Question 38 of 368

Next

Question stats

End and review

Score: 75%
1

12.1%

9.5%

50.2%

A. Aortic bifurcation and L4

10.3%

B. Transpyloric plane and L1

17.8%

C. Termination of dural sac and L4

50.2% of users answered this


question correctly

Which of the following pairings are incorrect?

D. Oesophageal passage through diaphragm and T10


E. Transition between pharynx and oesophagus at C6

6
7
8
9

Search
Next question
Go

10
11

Vena cava T8 (eight letters)


Oesophagus T10 (ten letters)
Aortic hiatus T12 (twelve letters)

12
13
14
15

It terminates at S2, which is why it is safe to undertake an LP at L4/5 levels. The spinal cord
itself terminates at L1.

16
17

Levels

18
19

Transpyloric plane
Level of the body of L1

20
21
22

Pylorus stomach
Left kidney hilum (L1- left one!)
Right hilum of the kidney (1.5cm lower than the left)
Fundus of the gallbladder
Neck of pancreas
Duodenojejunal flexure
Superior mesenteric artery
Portal vein
Left and right colic flexure
Root of the transverse mesocolon
2nd part of the duodenum
Upper part of conus medullaris
Spleen

23
24
25
26
27
28
29-31 3 / 3
32
33
34
35

Can be identified by asking the supine patient to sit up without using their arms. The plane is
located where the lateral border of the rectus muscle crosses the costal margin.

36
37

Anatomical planes

38

Subcostal plane

Lowest margin of 10th costal cartilage

Intercristal plane

Level of body L4 (highest point of iliac crest)

Intertubercular plane

Level of body L5

Common level landmarks


Inferior mesenteric artery

L3

Bifurcation of aorta into common iliac arteries

L4

Formation of IVC

L5 (union of common iliac veins)

Diaphragm apertures

Rate question:

Vena cava T8
Oesophagus T10
Aortic hiatus T12

Next question

Comment on this question

All contents of this site are 2012 E-Medical Revision Ltd

http://www.emrcs.com/question/question.php?q=0

Terms and Conditions

Privacy policy

1/2

17/05/2015

http://www.emrcs.com/question/question.php?q=0

2/2

17/05/2015

Reference ranges

Previous

Question 39 of 368

Next

A 22 year old man is involved in a fight. He sustains a laceration to the posterior aspect of
his wrist. In the emergency department the wound is explored and the laceration is found to
be transversely orientated and overlies the region of the extensor retinaculum, which is
intact. Which of the following structures is least likely to be injured in this scenario?

Question stats

Score: 72.2%
1

15.7%

35.4%

17.8%

12.6%

18.6%

35.4% of users answered this


question correctly

A. Dorsal cutaneous branch of the ulnar nerve

End and review

B. Tendon of extensor indicis

6
7
8

C. Basilic vein

Search

D. Superficial branch of the radial nerve


E. Cephalic vein

Go

10
11

Next question

12
13

The extensor retinaculum attaches to the radius proximal to the styloid, thereafter it runs
obliquely and distally to wind around the ulnar styloid (but does not attach to it). The
extensor tendons lie deep to the extensor retinaculum and would therefore be less
susceptible to injury than the superficial structures.

14
15
16
17

Extensor retinaculum

18

The extensor rentinaculum is a thickening of the deep fascia that stretches across the back
of the wrist and holds the long extensor tendons in position.
Its attachments are:
The pisiform and hook of hamate medially
The end of the radius laterally

19
20
21
22
23
24

Structures related to the extensor retinaculum


Structures superficial to the
retinaculum

Basilic vein
Dorsal cutaneous branch of the ulnar nerve
Cephalic vein
Superficial branch of the radial nerve

25
26
27
28
29-31 3 / 3

Structures passing deep to the


extensor retinaculum

32

Extensor carpi ulnaris tendon


Extensor digiti minimi tendon
Extensor digitorum and extensor indicis
tendon
Extensor pollicis longus tendon
Extensor carpi radialis longus tendon
Abductor pollicis longus and extensor
pollicis brevis tendons

33
34
35
36
37
38

Beneath the extensor retinaculum fibrous septa form six compartments that contain the
extensor muscle tendons. Each compartment has its own synovial sheath.

39

The radial artery


The radial artery passes between the lateral collateral ligament of the wrist joint and the
tendons of the abductor pollicis longus and extensor pollicis brevis.
Image illustrating the topography of tendons passing under the extensor retinaculum

http://www.emrcs.com/question/question.php?q=0

1/2

17/05/2015

Image sourced from Wikipedia

Rate question:

Next question

Comment on this question

All contents of this site are 2012 E-Medical Revision Ltd

http://www.emrcs.com/question/question.php?q=0

Terms and Conditions

Privacy policy

2/2

17/05/2015

Reference ranges

Previous

Question 40 of 368

Next

Question stats

End and review

Score: 72.7%
1

7.7%

10.9%

47.7%

A. Portal vein

18.6%

B. Hepatic artery

15.1%

C. Cystic duct

47.7% of users answered this


question correctly

Which of the following is not a content of the porta hepatis?

D. Lymph nodes
E. None of the above

6
7
8
9

Search
Next question
Go

The cystic duct lies outside the porta hepatis and is an important landmark in laparoscopic
cholecystectomy. The structures in the porta hepatis are:
Portal vein
Hepatic artery
Common hepatic duct
These structures divide immediately after or within the porta hepatis to supply the functional
left and right lobes of the liver.
The porta hepatis is also surrounded by lymph nodes, that may enlarge to produce
obstructive jaundice and parasympathetic nervous fibres that travel along vessels to enter
the liver.

10
11
12
13
14
15
16
17
18
19
20

Liver

21
22

Structure of the liver

23

Right lobe

Supplied by right hepatic artery


Contains Couinaud segments V to VIII (-/+Sg I)

24
25
26

Left lobe

Supplied by the left hepatic artery


Contains Couinaud segments II to IV (+/- Sg1)

27
28
29-31 3 / 3

Quadrate lobe

Part of the right lobe anatomically, functionally is part of the left


Couinaud segment IV
Porta hepatis lies behind
On the right lies the gallbladder fossa
On the left lies the fossa for the umbilical vein

32
33
34
35
36

Caudate lobe

Supplied by both right and left hepatic arteries


Couinaud segment I
Lies behind the plane of the porta hepatis
Anterior and lateral to the inferior vena cava
Bile from the caudate lobe drains into both right and left hepatic ducts

37
38
39
40

Detailed knowledge of Couinaud segments is not required for MRCS Part A


Between the liver lobules are portal canals which contain the portal triad: Hepatic
Artery, Portal Vein, tributary of Bile Duct.

Relations of the liver


Anterior

Postero inferiorly

Diaphragm

Oesophagus

Xiphoid process

Stomach
Duodenum
Hepatic flexure of colon
Right kidney
Gallbladder
Inferior vena cava

http://www.emrcs.com/question/question.php?q=0

1/2

17/05/2015

Porta hepatis
Location

Postero inferior surface, it joins nearly at right angles with the left sagittal fossa,
and separates the caudate lobe behind from the quadrate lobe in front

Transmits

Common hepatic duct


Hepatic artery
Portal vein
Sympathetic and parasympathetic nerve fibres
Lymphatic drainage of the liver (and nodes)

Ligaments
Falciform
ligament

2 layer fold peritoneum from the umbilicus to anterior liver surface


Contains ligamentum teres (remnant umbilical vein)
On superior liver surface it splits into the coronary and left
triangular ligaments

Ligamentum teres

Joins the left branch of the portal vein in the porta hepatis

Ligamentum
venosum

Remnant of ductus venosus

Arterial supply
Hepatic artery

Venous
Hepatic veins
Portal vein

Nervous supply
Sympathetic and parasympathetic trunks of coeliac plexus

Rate question:

Next question

Comment on this question

All contents of this site are 2012 E-Medical Revision Ltd

http://www.emrcs.com/question/question.php?q=0

Terms and Conditions

Privacy policy

2/2

17/05/2015

Reference ranges

Previous

Question 41 of 368

Next

Question stats

End and review

Score: 73.7%
1

15.2%

13%

14%

A. Sternothyroid muscle

18.8%

B. Sternohyoid muscle

39%

C. Hypoglossal nerve

39% of users answered this


question correctly

Which of the following structures is not closely related to the carotid sheath?

D. Superior belly of omohyoid muscle


E. Anterior belly of digastric muscle

6
7
8
9

Search
Next question
Go

At its lower end the carotid sheath is related to sternohyoid and sternothyroid. Opposite the
cricoid cartilage the sheath is crossed by the superior belly of omohyoid. Above this level the
sheath is covered by the sternocleidomastoid muscle. Above the level of the hyoid the
vessels pass deep to the posterior belly of digastric and stylohyoid. Opposite the hyoid bone
the sheath is crossed obliquely by the hypoglossal nerve.

10
11
12
13
14
15

Common carotid artery

16
17

The right common carotid artery arises at the bifurcation of the brachiocephalic trunk, the left
common carotid arises from the arch of the aorta. Both terminate at the level of the upper
border of the thyroid cartilage (the lower border of the third cervical vertebra) by dividing into
the internal and external carotid arteries.
Left common carotid artery
This vessel arises immediately to the left and slightly behind the origin of the brachiocephalic
trunk. Its thoracic portion is 2.5- 3.5 cm in length and runs superolaterally to the
sternoclavicular joint.
In the thorax
The vessel is in contact, from below upwards, with the trachea, left recurrent laryngeal nerve,
left margin of the oesophagus. Anteriorly the left brachiocephalic vein runs across the artery,
and the cardiac branches from the left vagus descend in front of it. These structures
together with the thymus and the anterior margins of the left lung and pleura separate the
artery from the manubrium.

18
19
20
21
22
23
24
25
26
27
28
29-31 3 / 3
32

In the neck
The artery runs superiorly deep to sternocleidomastoid and then enters the anterior triangle.
At this point it lies within the carotid sheath with the vagus nerve and the internal jugular vein.
Posteriorly the sympathetic trunk lies between the vessel and the prevertebral fascia. At the
level of C7 the vertebral artery and thoracic duct lie behind it. The anterior tubercle of C6
transverse process is prominent and the artery can be compressed against this structure (it
corresponds to the level of the cricoid).
Anteriorly at C6 the omohyoid muscle passes superficial to the artery.
Within the carotid sheath the jugular vein lies lateral to the artery.

33
34
35
36
37
38
39

Right common carotid artery


The right common carotid arises from the brachiocephalic artery. The right common carotid
artery corresponds with the cervical portion of the left common carotid, except that there is
no thoracic duct on the right. The oesophagus is less closely related to the right carotid than
the left.

40
41

Summary points about the carotid anatomy


Path
Passes behind the sternoclavicular joint (12% patients above this level) to the upper border
of the thyroid cartilage, to divide into the external (ECA) and internal carotid arteries (ICA).
Relations
Level of 6th cervical vertebra crossed by omohyoid
Then passes deep to the thyrohyoid, sternohyoid, sternomastoid muscles.
Passes anterior to the carotid tubercle (transverse process 6th cervical vertebra)-NB
compression here stops haemorrhage.
The inferior thyroid artery passes posterior to the common carotid artery.
Then : Left common carotid artery crossed by thoracic duct, Right common carotid
artery crossed by recurrent laryngeal nerve

http://www.emrcs.com/question/question.php?q=0

1/2

17/05/2015

Image sourced from Wikipedia

Rate question:

Next question

Comment on this question

All contents of this site are 2012 E-Medical Revision Ltd

http://www.emrcs.com/question/question.php?q=0

Terms and Conditions

Privacy policy

2/2

17/05/2015

Reference ranges

Previous

Question 42 of 368

Next

A 21 year old develops tonsillitis. He is in considerable pain. Which of the following nerves is
responsible for the sensory innervation of the tonsillar fossa?

A. Facial nerve
B. Trigeminal nerve

Question stats

Score: 74.6%
1

8.3%

12.9%

60.5%

9.2%

9.2%

60.5% of users answered this


question correctly

C. Glossopharyngeal nerve

End and review

D. Hypoglossal nerve

6
7
8

E. Vagus

Search
Go

Next question

10
11

The glossopharyngeal nerve is the main sensory nerve for the tonsillar fossa. A lesser
contribution is made by the lesser palatine nerve. Because of this otalgia may occur following
tonsillectomy.

12

Tonsil

15

13
14

16

Anatomy

17
18

Each palatine tonsil has two surfaces, a medial surface which projects into the pharynx
and a lateral surface that is embedded in the wall of the pharynx.
They are usually 25mm tall by 15mm wide, although this varies according to age and
may be almost completely atrophied in the elderly.
Their arterial supply is from the tonsillar artery, a branch of the facial artery.
Its veins pierce the constrictor muscle to join the external palatine or facial veins. The
external palatine vein is immediately lateral to the tonsil, which may result in
haemorrhage during tonsillectomy.
Lymphatic drainage is the jugulodigastric node and the deep cervical nodes.

19
20
21
22
23
24
25
26

Tonsillitis

27

Usually bacterial (50%)- group A Streptococcus. Remainder viral.


May be complicated by development of abscess (quinsy). This may distort the uvula.

28
29-31 3 / 3

- Indications for tonsillectomy include recurrent acute tonsillitis, suspected malignancy,


enlargement causing sleep apnoea.
- Dissection tonsillectomy is the preferred technique with haemorrhage being the commonest
complication. Delayed otalgia may occur owing to irritation of the glossopharyngeal nerve.

32
33
34
35

Rate question:

Next question

36

Comment on this question

37
38
39
40
41
42

All contents of this site are 2012 E-Medical Revision Ltd

http://www.emrcs.com/question/question.php?q=0

Terms and Conditions

Privacy policy

1/1

17/05/2015

Reference ranges

Previous

Question 43 of 368

Next

A man has an incision sited than runs 8cm from the deltopectoral groove to the midline.
Which of the following is not at risk of injury?

A. Cephalic vein
B. Shoulder joint capsule

Question stats

Score: 75%
1

21.8%

29.1%

14.1%

9.1%

26%

29.1% of users answered this


question correctly

C. Axillary artery

End and review

D. Pectoralis major

6
7
8

E. Trunk of the brachial plexus

Search
Go

Next question

10
11
12

Theme from April 2012 Exam


This region will typically lie medial to the joint capsule. The diagram below illustrates the
plane that this would transect and as it can be appreciated the other structures are all at risk
of injury.

13
14
15
16
17
18
19
20
21
22
23
24
25
26
27
28
29-31 3 / 3
32
33
34
35
36
37
38

Image sourced from Wikipedia

39
40
41

Pectoralis major muscle


42

Origin

From the medial two thirds of the clavicle, manubrium and sternocostal angle

Insertion

Crest of the greater tubercle of the humerus

Nerve supply

Lateral pectoral nerve

Actions

Adductor and medial rotator of the humerus

Rate question:

43

Next question

Comment on this question

All contents of this site are 2012 E-Medical Revision Ltd

http://www.emrcs.com/question/question.php?q=0

Terms and Conditions

Privacy policy

1/1

17/05/2015

Reference ranges

Question 44 of 368

Previous

Next

A surgeon is due to perform a laparotomy for perforated duodenal ulcer. An upper midline
incision is to be performed. Which of the following structures is the incision most likely to
divide?

Question stats

Score: 75.4%
1

12.2%

7.9%

64.6%

6.4%

8.7%

A. Rectus abdominis muscle


64.6% of users answered this
question correctly

B. External oblique muscle

End and review

6
7

C. Linea alba
8

D. Internal oblique muscle


9

Search

E. None of the above

Go

Next question

10
11
12

Theme from September 2011 Exam


Upper midline abdominal incisions will involve the division of the linea alba. Division of
muscles will not usually improve access in this approach and they would not be routinely
encountered during this incision.

13
14
15
16

Abdominal incisions

17
18

Midline
incision

Commonest approach to the abdomen


Structures divided: linea alba, transversalis fascia, extraperitoneal fat,
peritoneum (avoid falciform ligament above the umbilicus)
Bladder can be accessed via an extraperitoneal approach through the
space of Retzius

19
20
21
22
23

Paramedian
incision

Parallel to the midline (about 3-4cm)


Structures divided/retracted: anterior rectus sheath, rectus (retracted),
posterior rectus sheath, transversalis fascia, extraperitoneal fat,
peritoneum
Incision is closed in layers

24
25
26
27
28

Battle

Similar location to paramedian but rectus displaced medially (and thus


denervated)
Now seldom used

29-31 3 / 3
32
33
34

Kocher's

Incision under right subcostal margin e.g. Cholecystectomy (open)

Lanz

Incision in right iliac fossa e.g. Appendicectomy

Gridiron

Oblique incision centered over McBurneys point- usually appendicectomy


(less cosmetically acceptable than Lanz

Gable

Rooftop incision

39

Pfannenstiel's

Transverse supra pubic, primarily used to access pelvic organs

40

McEvedy's

Groin incision e.g. Emergency repair strangulated femoral hernia

Rutherford
Morrison

Extraperitoneal approach to left or right lower quadrants. Gives excellent


access to iliac vessels and is the approach of choice for first time renal
transplantation.

35
36
37
38

41
42
43
44

Image sourced from Wikipedia

http://www.emrcs.com/question/question.php?q=0

1/2

17/05/2015

Rate question:

Next question

Comment on this question

All contents of this site are 2012 E-Medical Revision Ltd

http://www.emrcs.com/question/question.php?q=0

Terms and Conditions

Privacy policy

2/2

17/05/2015

Reference ranges

Question 45 of 368

Previous

Next

A 59 year old man is undergoing an extended right hemicolectomy for a carcinoma of the
splenic flexure of the colon. The surgeons divide the middle colic vein close to its origin. Into
which of the following structures does this vessel primarily drain?

Question stats

Score: 74.2%
1

48.9%

12.7%

17.2%

10.9%

10.3%

A. Superior mesenteric vein


48.9% of users answered this
question correctly

B. Portal vein

End and review

6
7

C. Inferior mesenteric vein


8

D. Inferior vena cava


9

Search

E. Ileocolic vein

Go

10
11

Next question

12

The middle colonic vein drains into the SMV, if avulsed during mobilisation then dramatic
haemorrhage can occur and be difficult to control.

13

Transverse colon

15

14

16
17

The right colon undergoes a sharp turn at the level of the hepatic flexure to become
the transverse colon.
At this point it also becomes intraperitoneal.
It is connected to the inferior border of the pancreas by the transverse mesocolon.
The greater omentum is attached to the superior aspect of the transverse colon from
which it can easily be separated. The mesentery contains the middle colic artery and
vein. The greater omentum remains attached to the transverse colon up to the splenic
flexure. At this point the colon undergoes another sharp turn.

18
19
20
21
22
23
24

Relations

25

Superior

Liver and gall-bladder, the greater curvature of the stomach, and the low er end
of the spleen

26

Inferior

Small intestine

27

Anterior

Greater omentum

28
29-31 3 / 3

Posterior From right to left w ith the descending portion of the duodenum, the head of the
pancreas, convolutions of the jejunum and ileum, spleen

32
33

Rate question:

34
Next question

35
Comment on this question

36
37
38
39
40
41
42
43
44
45

All contents of this site are 2012 E-Medical Revision Ltd

http://www.emrcs.com/question/question.php?q=0

Terms and Conditions

Privacy policy

1/1

17/05/2015

Reference ranges

Previous3 / 3

Question 46-48 of 368

Next

Question stats

End and review

Score: 75.4%

Average score for registered users:

1
2

Theme: Nerve Injury


A. Median nerve

46

55%

47

56.4%

48

54.8%

B. Ulnar nerve

3
4
5

C. Radial nerve

D. Musculocutaneous nerve

Search

E. Axillary nerve

7
Go

F. Anterior interosseous nerve

8
9

G. Posterior interosseous nerve


10

For each scenario please select the most likely underlying nerve injury. Each option may be
used once, more than once or not at all.

11
12
13
14

46.

A 10 year old boy is admitted to casualty following a fall. On examination there


is deformity and swelling of the upper arm. The ability to flex the fingers of the
affected limb is impaired. However, there is not sensory impairment. Imaging
confirms a displaced supra condylar fracture

15
16
17
18

Anterior interosseous nerve

19

Supracondylar fractures may be complicated by neurovascular compromise.


The anterior interosseous nerve is most commonly affected. It has no sensory
supply so the defect is motor alone.
47.

A well toned weight lifter attends clinic reporting weakness of his left arm. There
is weakness of flexion and supination of the forearm.

20
21
22
23
24

Musculocutaneous nerve

25

Musculocutaneous nerve compression due to entrapment of the nerve between


biceps and brachialis. Elbow flexion and supination of the arm are affected.
This is a rare isolated injury.

26
27
28

48.

An 18 year old girl sustains an Holstein-Lewis fracture. Which nerve is at risk?

29-31 3 / 3
32

Radial nerve
33

Proximal lesions affect the triceps. Also paralysis of wrist extensors and forearm
supinators occur. Reduced sensation of dorsoradial aspect of hand and dorsal
31/2 fingers. Holstein-Lewis fractures are fractures of the distal humerus with
radial nerve entrapment.

34
35
36
37

Next question

38
39
40

Brachial plexus

41
42
43

Origin

Anterior rami of C5 to T1
44

Sections of the
plexus

Roots

Trunks

Roots, trunks, divisions, cords, branches


Mnemonic:Real Teenagers Drink Cold Beer

45
46-48 3 / 3

Located in the posterior triangle


Pass between scalenus anterior and medius

Located posterior to middle third of clavicle


Upper and middle trunks related superiorly to the subclavian
artery
Lower trunk passes over 1st rib posterior to the subclavian
artery

Divisions

Apex of axilla

Cords

Related to axillary artery

http://www.emrcs.com/question/question.php?q=0

1/2

17/05/2015

Diagram illustrating the branches of the brachial plexus

Image sourced from Wikipedia

Cutaneous sensation of the upper limb

Image sourced from Wikipedia

Rate question:

Next question

Comment on this question

All contents of this site are 2012 E-Medical Revision Ltd

http://www.emrcs.com/question/question.php?q=0

Terms and Conditions

Privacy policy

2/2

17/05/2015

Reference ranges

Question 49 of 368

Previous

Next

A 23 year old man is stabbed in the chest approximately 10cm below the right nipple. In the
emergency department a abdominal ultrasound scan shows a large amount of intraperitoneal
blood. Which of the following statements relating to the likely site of injury is untrue?

Question stats

Score: 75.8%
1

14%

40.7%

12.3%

17%

16.1%

A. Part of its posterior surface is devoid of peritoneum.


40.7% of users answered this
question correctly

B. The quadrate lobe is contained within the functional right lobe.

End and review

6
7

C. Its nerve supply is from the coeliac plexus.


8

D. The hepatic flexure of the colon lies posterio-inferiorly.


9

Search

E. The right kidney is closely related posteriorly.

Go

Next question

10
11
12

The right lobe of the liver is the most likely site of injury. Therefore the answer is B as the
quadrate lobe is functionally part of the left lobe of the liver. The liver is largely covered in
peritoneum. Posteriorly there is an area devoid of peritoneum (the bare area of the liver).
The right lobe of the liver has the largest bare area (and is larger than the left lobe).

13
14
15
16

Liver

17
18

Structure of the liver

19

Right lobe

Supplied by right hepatic artery


Contains Couinaud segments V to VIII (-/+Sg I)

20
21
22

Left lobe

Supplied by the left hepatic artery


Contains Couinaud segments II to IV (+/- Sg1)

23
24
25

Quadrate lobe

Part of the right lobe anatomically, functionally is part of the left


Couinaud segment IV
Porta hepatis lies behind
On the right lies the gallbladder fossa
On the left lies the fossa for the umbilical vein

26
27
28
29-31 3 / 3
32

Caudate lobe

Supplied by both right and left hepatic arteries


Couinaud segment I
Lies behind the plane of the porta hepatis
Anterior and lateral to the inferior vena cava
Bile from the caudate lobe drains into both right and left hepatic ducts

33
34
35
36
37

Detailed knowledge of Couinaud segments is not required for MRCS Part A


38

Between the liver lobules are portal canals which contain the portal triad: Hepatic
Artery, Portal Vein, tributary of Bile Duct.

39
40
41

Relations of the liver


Anterior

Postero inferiorly

Diaphragm

Oesophagus

42
43
44

Xiphoid process

Stomach

45
46-48 3 / 3
49

Duodenum
Hepatic flexure of colon
Right kidney
Gallbladder
Inferior vena cava
Porta hepatis
Location

Postero inferior surface, it joins nearly at right angles with the left sagittal fossa,
and separates the caudate lobe behind from the quadrate lobe in front

http://www.emrcs.com/question/question.php?q=0

1/2

17/05/2015

Transmits

Common hepatic duct


Hepatic artery
Portal vein
Sympathetic and parasympathetic nerve fibres
Lymphatic drainage of the liver (and nodes)

Ligaments
Falciform
ligament

2 layer fold peritoneum from the umbilicus to anterior liver surface


Contains ligamentum teres (remnant umbilical vein)
On superior liver surface it splits into the coronary and left
triangular ligaments

Ligamentum teres

Joins the left branch of the portal vein in the porta hepatis

Ligamentum
venosum

Remnant of ductus venosus

Arterial supply
Hepatic artery

Venous
Hepatic veins
Portal vein

Nervous supply
Sympathetic and parasympathetic trunks of coeliac plexus

Rate question:

Next question

Comment on this question

All contents of this site are 2012 E-Medical Revision Ltd

http://www.emrcs.com/question/question.php?q=0

Terms and Conditions

Privacy policy

2/2

17/05/2015

Reference ranges

Previous

Question 50 of 368

Next

A 22 year old man is involved in a fight and sustains a skull fracture with an injury to the
middle meningeal artery. A craniotomy is performed, and with considerable difficulty the
haemorrhage from the middle meningeal artery is controlled by ligating it close to its origin.
What is the most likely sensory impairment that the patient may notice post operatively?

Question stats

Score: 74.6%
1

32.2%

20.8%

22.7%

12.9%

11.3%

32.2% of users answered this


question correctly

A. Parasthesia of the ipsilateral external ear

End and review

B. Loss of taste sensation from the anterior two thirds of the tongue

6
7
8

C. Parasthesia overlying the angle of the jaw

Search

D. Loss of sensation from the ipsilateral side of the tongue


E. Loss of taste from the posterior two thirds of the tongue

Go

10
11

Next question

12
13

The auriculotemporal nerve is closely related to the middle meningeal artery and may be
damaged in this scenario. The nerve supplied sensation to the external ear and outermost
part of the tympanic membrane. The angle of the jaw is innervated by C2,3 roots and would
not be affected. The tongue is supplied by the glossopharyngeal nerve.

14
15
16
17

Middle meningeal artery

18
19

Middle meningeal artery is typically the third branch of the first part of the maxillary
artery, one of the two terminal branches of the external carotid artery. After branching
off the maxillary artery in the infratemporal fossa, it runs through the foramen
spinosum to supply the dura mater (the outermost meninges) .
The middle meningeal artery is the largest of the three (paired) arteries which supply
the meninges, the others being the anterior meningeal artery and the posterior
meningeal artery.
The middle meningeal artery runs beneath the pterion. It is vulnerable to injury at this
point, where the skull is thin. Rupture of the artery may give rise to an extra dural
hematoma.
In the dry cranium, the middle meningeal, which runs within the dura mater
surrounding the brain, makes a deep indention in the calvarium.
The middle meningeal artery is intimately associated with the auriculotemporal nerve
which wraps around the artery making the two easily identifiable in the dissection of
human cadavers and also easily damaged in surgery.

20
21
22
23
24
25
26
27
28
29-31 3 / 3
32
33
34

Rate question:

35

Next question

36
Comment on this question

37
38
39
40
41
42
43
44
45
46-48 3 / 3
49
50

All contents of this site are 2012 E-Medical Revision Ltd

http://www.emrcs.com/question/question.php?q=0

Terms and Conditions

Privacy policy

1/1

17/05/2015

Reference ranges

Previous

Question 51 of 368

Next

A 72 year old man presents with haemoptysis and undergoes a bronchoscopy. The carina is
noted to be widened. At which level does the trachea bifurcate?

A. T3
B. T5

Question stats

Score: 75%
1

17.2%

51.3%

8.8%

13.2%

9.4%

51.3% of users answered this


question correctly

C. T7

End and review

D. T2

6
7
8

E. T8

Search
Go

Next question

10
11
12

The trachea bifurcates at the level of the fifth thoracic vertebra. Or the sixth in tall subjects.

13

Trachea

14
15

Trachea

16

Location

C6 vertebra to the upper border of T5 vertebra (bifurcation)

Arterial and venous supply

Inferior thyroid arteries and the thyroid venous plexus.

18

Nerve

Branches of vagus, sympathetic and the recurrent nerves

19

17

20
21

Relations in the neck

22

Anterior(Superior to
inferior)

Isthmus of the thyroid gland


Inferior thyroid veins
Arteria thyroidea ima (when that vessel exists)
Sternothyroid
Sternohyoid
Cervical fascia
Anastomosing branches between the anterior jugular
veins

23
24
25
26
27
28

Posterior

29-31 3 / 3

Oesophagus.

32

Laterally

Common carotid arteries


Right and left lobes of the thyroid gland
Inferior thyroid arteries
Recurrent laryngeal nerves

33
34
35
36
37

Relations in the thorax

38

Anterior

39

Manubrium, the remains of the thymus, the aortic arch, left common carotid arteries,
and the deep cardiac plexus

40
41
42

Lateral

43

In the superior mediastinum, on the right side is the pleura and right vagus; on its left
side are the left recurrent nerve, the aortic arch, and the left common carotid and
subclavian arteries.

44
45
46-48 3 / 3
49
50

Rate question:

Next question

51

Comment on this question

All contents of this site are 2012 E-Medical Revision Ltd

http://www.emrcs.com/question/question.php?q=0

Terms and Conditions

Privacy policy

1/1

17/05/2015

Reference ranges

Question 52 of 368

Previous

Next

A 23 year old man is injured during a game of rugby. He suffers a fracture of the distal third
of his clavicle, it is a compound fracture and there is evidence of arterial haemorrhage.
Which of the following vessels is most likely to be encountered first during subsequent
surgical exploration?

Question stats

Score: 75.7%
1

9.4%

24.2%

43.2%

12%

11.3%

43.2% of users answered this


question correctly

A. Posterior circumflex humeral artery

End and review

B. Axillary artery

6
7
8

C. Thoracoacromial artery

Search

D. Sub scapular artery


E. Lateral thoracic artery

Go

10
11

Next question

12
13

Similar theme in September 2011 Exam


The thoracoacromial artery arises from the second part of the axillary artery. It is a short,
wide trunk, which pierces the clavipectoral fascia, and ends, deep to pectoralis major by
dividing into four branches.

14
15
16
17

Thoracoacromial artery

18

The thoracoacromial artery (acromiothoracic artery; thoracic axis) is a short trunk, which
arises from the forepart of the axillary artery, its origin being generally overlapped by the
upper edge of the Pectoralis minor.

19

Projecting forward to the upper border of the Pectoralis minor, it pierces the coracoclavicular
fascia and divides into four branches: pectoral, acromial, clavicular, and deltoid.

22

20
21

23
24

Branch

Description

Pectoral
branch

Descends between the two Pectoral muscles, and is distributed to them and to the
breast, anastomosing with the intercostal branches of the internal thoracic artery
and with the lateral thoracic.

Acromial
branch

Runs laterally over the coracoid process and under the Deltoid, to which it gives
branches; it then pierces that muscle and ends on the acromion in an arterial
network formed by branches from the suprascapular, thoracoacromial, and posterior
humeral circumflex arteries.

25
26
27
28

Clavicular
branch

Runs upwards and medially to the sternoclavicular joint, supplying this articulation,
and the Subclavius.

Deltoid
branch

Arising with the acromial, it crosses over the Pectoralis minor and passes in the
same groove as the cephalic vein, between the Pectoralis major and Deltoid, and
gives branches to both muscles.

29-31 3 / 3
32
33
34
35
36
37
38
39
40

Rate question:

Next question

41

Comment on this question

42
43
44
45
46-48 3 / 3
49
50
51
52

All contents of this site are 2012 E-Medical Revision Ltd

http://www.emrcs.com/question/question.php?q=0

Terms and Conditions

Privacy policy

1/1

17/05/2015

Reference ranges

Question 53 of 368

Previous

Next

Question stats

End and review

Score: 76.1%
1

8.6%

9.4%

10%

A. It is derived from L2, L3 and L4 nerve roots

13.1%

B. It supplies sartorius

58.9%

C. It supplies quadriceps femoris

58.9% of users answered this


question correctly

The following are true of the femoral nerve, except:

D. It gives cutaneous innervations via the saphenous nerve


E. It supplies adductor longus

6
7
8
9

Search
Next question
Go

Adductor longus is supplied by the obturator nerve.

10
11
12

Femoral nerve

13
14

Root values

L2, 3, 4

15
16

Innervates

Pectineus
Sartorius
Quadriceps femoris
Vastus lateralis/medialis/intermedius

17
18
19
20

Branches

Medial cutaneous nerve of thigh


Saphenous nerve
Intermediate cutaneous nerve of thigh

21
22
23
24

Path
Penetrates psoas major and exits the pelvis by passing under the inguinal ligament to enter
the femoral triangle, lateral to the femoral artery and vein.

25
26
27
28
29-31 3 / 3
32
33
34
35
36
37
38
39
40
41
42
43
44
45
46-48 3 / 3

Image sourced from Wikipedia

49
50

Mnemonic for femoral nerve supply


(don't) M I S V Q Scan for PE
M edial cutaneous nerve of the thigh
I ntermediate cutaneous nerve of the thigh
S aphenous nerve

51
52
53

V astus
Q uadriceps femoris
S artorius
PE ectineus

http://www.emrcs.com/question/question.php?q=0

1/2

17/05/2015

Rate question:

Next question

Comment on this question

All contents of this site are 2012 E-Medical Revision Ltd

http://www.emrcs.com/question/question.php?q=0

Terms and Conditions

Privacy policy

2/2

17/05/2015

Reference ranges

Previous

Question 54 of 368

Next

Question stats

End and review

Score: 76.4%
1

50.8%

9.8%

7.6%

A. Medulla oblongata

16.9%

B. Substantia nigra

14.9%

C. Antrum of stomach

50.8% of users answered this


question correctly

Where is the vomiting centre located?

D. Pons
E. Midbrain

6
7
8
9

Search
Next question
Go

10
11

ABC's of Non- GI causes of vomiting

12
13

Acute renal failure


Brain (Increased ICP)
Cardiac (Inferior MI)
DKA
Ears (labyrinthitis)
Foreign substances (Tylenol, theo, etc)
Glaucoma
Hyperemesis Gravidarum
Infections (pyelonephritis, meningitis)

14
15
16
17
18
19
20
21
22
23

Vomiting

24

Reflex oral expulsion of gastric (and sometimes intestinal) contents - reverse peristalsis and
abdominal contraction

25
26

The vomiting centre is in part of the medulla oblongata and is triggered by receptors in
several locations:

27
28
29-31 3 / 3

Labyrinthine receptors of ear (motion sickness)


Over distention receptors of duodenum and stomach
Trigger zone of CNS - many drugs (e.g., opiates) act here
Touch receptors in throat
Sensory innervation rich, both extrinsic and intrinsic

32
33
34
35
36
37

Rate question:

Next question

38
39

Comment on this question

40
41
42
43
44
45
46-48 3 / 3
49
50
51
52
53
54

All contents of this site are 2012 E-Medical Revision Ltd

http://www.emrcs.com/question/question.php?q=0

Terms and Conditions

Privacy policy

1/1

17/05/2015

Reference ranges

Previous

Question 55 of 368

Next

Question stats

End and review

Score: 76.7%
1

25.3%

42%

11.5%

A. Glossopharyngeal

12.9%

B. Laryngeal branches of the vagus

8.2%

C. Ansa cervicalis

42% of users answered this


question correctly

Which of the following nerves conveys sensory information from the laryngeal mucosa?

D. Laryngeal branches of the trigeminal


E. None of the above

6
7
8
9

Search
Next question
Go

The laryngeal branches of the vagus supply sensory information from the larynx.

10
11
12

Larynx

13
14

The larynx lies in the anterior part of the neck at the levels of C3 to C6 vertebral bodies. The
laryngeal skeleton consists of a number of cartilagenous segments. Three of these are
paired; arytenoid, corniculate and cuneiform. Three are single; thyroid, cricoid and epiglottic.
The cricoid cartilage forms a complete ring (the only one to do so).
The laryngeal cavity extends from the laryngeal inlet to the level of the inferior border of the
cricoid cartilage.

15
16
17
18
19

Divisions of the laryngeal cavity

20

Laryngeal vestibule

Superior to the vestibular folds

Laryngeal ventricle

Lies between vestibular folds and superior to the vocal cords

Infraglottic cavity

Extends from vocal cords to inferior border of the cricoid cartilage

21
22
23
24
25

The vocal folds (true vocal cords) control sound production. The apex of each fold projects
medially into the laryngeal cavity. Each vocal fold includes:

26
27

Vocal ligament
Vocalis muscle (most medial part of thyroarytenoid muscle)

28
29-31 3 / 3

The glottis is composed of the vocal folds, processes and rima glottidis. The rima glottidis is
the narrowest potential site within the larynx, as the vocal cords may be completely opposed,
forming a complete barrier.

32
33
34

Muscles of the larynx

35

Muscle

Origin

Insertion

Innervation

Action

36

Posterior
cricoarytenoid

Posterior aspect
of lamina of
cricoid

Muscular process of
arytenoid

Recurrent
Laryngeal

Abducts vocal
fold

37

Lateral
cricoarytenoid

Arch of cricoid

Muscular process of
arytenoid

Recurrent
laryngeal

Adducts vocal
fold

40

Thyroarytenoid

Posterior aspect
of thyroid
cartilage

Muscular process of
arytenoid

Recurrent
laryngeal

Relaxes vocal
fold

42

Arytenoid
cartilage

Contralateral
arytenoid

Recurrent
laryngeal

Closure of
intercartilagenous
part of the rima
glottidis

38
39

41

43
44

Transverse
and oblique
arytenoids

Vocalis

Depression
between lamina
of thyroid
cartilage

Vocal ligament and


vocal process of
arytenoid cartilage

Anterolateral part
of cricoid

Inferior margin and


horn of thyroid
cartilage

Recurrent
laryngeal

Relaxes posterior
vocal ligament,
tenses anterior
part

45
46-48 3 / 3
49
50
51
52
53

Cricothyroid

External
laryngeal

Tenses vocal fold


54
55

Blood supply
Arterial supply is via the laryngeal arteries, branches of the superior and inferior thyroid
arteries. The superior laryngeal artery is closely related to the internal laryngeal nerve. The
inferior laryngeal artery is related to the inferior laryngeal nerve. Venous drainage is via
superior and inferior laryngeal veins, the former draining into the superior thyroid vein and
http://www.emrcs.com/question/question.php?q=0

1/2

17/05/2015

the latter draining into the middle thyroid vein, or thyroid venous plexus.
Lymphatic drainage
The vocal cords have no lymphatic drainage and this site acts as a lymphatic watershed.
Supraglottic part

Upper deep cervical nodes

Subglottic part

Prelaryngeal and pretracheal nodes and inferior deep cervical nodes

The aryepiglottic fold and vestibular folds have a dense plexus of lymphatics associated with
them and malignancies at these sites have a greater propensity for nodal metastasis.

Topography of the larynx

Image sourced from Wikipedia

Rate question:

Next question

Comment on this question

All contents of this site are 2012 E-Medical Revision Ltd

http://www.emrcs.com/question/question.php?q=0

Terms and Conditions

Privacy policy

2/2

17/05/2015

Reference ranges

Previous

Question 56 of 368

Next

Which of the following nerves passes through the greater sciatic foramen and innervates the
perineum?

A. Pudendal
B. Sciatic

Question stats

Score: 77.3%
1

67.1%

8.8%

8.5%

8.4%

7.2%

67.1% of users answered this


question correctly

C. Superior gluteal

End and review

D. Inferior gluteal

6
7
8

E. Posterior cutaneous nerve of the thigh

Search
Go

Next question

10
11
12

3 divisions of the pudendal nerve:

13

Rectal nerve
Perineal nerve
Dorsal nerve of penis/ clitoris

14
15
16

All these pass through the greater sciatic foramen.

17
18

The pudendal nerve innervates the perineum. It passes between piriformis and coccygeus
medial to the sciatic nerve.

19
20
21

Gluteal region

22

Gluteal muscles

23

Gluteus maximus: inserts to gluteal tuberosity of the femur and iliotibial tract
Gluteus medius: attach to lateral greater trochanter
Gluteus minimis: attach to anterior greater trochanter
All extend and abduct the hip

24
25
26
27
28

Deep lateral hip rotators

29-31 3 / 3

Piriformis
Gemelli
Obturator internus
Quadratus femoris

32
33
34
35

Nerves

36

Superior gluteal nerve (L5, S1)

37

Gluteus medius
Gluteus minimis
Tensor fascia lata

38
39
40

Inferior gluteal nerve

Gluteus maximus
41

Damage to the superior gluteal nerve will result in the patient developing a Trendelenberg
gait. Affected patients are unable to abduct the thigh at the hip joint. During the stance
phase, the weakened abductor muscles allow the pelvis to tilt down on the opposite side. To
compensate, the trunk lurches to the weakened side to attempt to maintain a level pelvis
throughout the gait cycle. The pelvis sags on the opposite side of the lesioned superior
gluteal nerve.

42
43
44
45
46-48 3 / 3

Rate question:

49
Next question

50
Comment on this question

51
52
53
54
55
56

All contents of this site are 2012 E-Medical Revision Ltd

http://www.emrcs.com/question/question.php?q=0

Terms and Conditions

Privacy policy

1/1

17/05/2015

Reference ranges

Question 57 of 368

Previous

Next

Question stats

End and review

Score: 77.6%
1

28%

11.1%

9.5%

A. Innervated by the deep branch of the femoral nerve

35.7%

B. Inserts at the fibula

15.7%

C. It is the shortest muscle in the body

35.7% of users answered this


question correctly

Which of the following is true in relation to the sartorius muscle?

D. Forms the Pes anserinus with Gracilis and semitendinous muscle


E. Causes extension of the knee

6
7
8
9

Search
Next question
Go

10
11

It is innervated by the superficial branch of the femoral nerve. It is a component of the pes
anserinus.

12
13

Sartorius

14
15

Longest strap muscle in the body


Most superficial muscle in the anterior compartment of the thigh

16
17
18

Origin

Anterior superior iliac spine

Insertion

Medial surface of the of the body of the tibia (upper part). It inserts anterior to
gracilis and semitendinosus

Nerve
Supply

19
20
21
22

Femoral nerve (L2,3)

23
24

Action

Flexor of the hip and knee, slight abducts the thigh and rotates it laterally
It assists with medial rotation of the tibia on the femur. For example it would
play a pivotal role in placing the right heel onto the left knee ( and vice versa)

25
26
27

Important
relations

The middle third of this muscle, and its strong underlying fascia forms the roof of
the adductor canal , in which lie the femoral vessels, the saphenous nerve and the
nerve to vastus medialis.

28
29-31 3 / 3
32
33

Rate question:

34

Next question

35
Comment on this question

36
37
38
39
40
41
42
43
44
45
46-48 3 / 3
49
50
51
52
53
54
55
56
57

All contents of this site are 2012 E-Medical Revision Ltd

http://www.emrcs.com/question/question.php?q=0

Terms and Conditions

Privacy policy

1/2

17/05/2015

http://www.emrcs.com/question/question.php?q=0

2/2

17/05/2015

Reference ranges

Previous3 / 3

Question 58-60 of 368

Next

Question stats

End and review

Score: 79.3%

Average score for registered users:

1
2

Theme: Nerve lesions


A. Sciatic nerve

58

77.7%

59

91.9%

60

77.4%

B. Peroneal nerve

3
4
5

C. Tibial Nerve

D. Obturator nerve

Search

E. Ilioinguinal nerve

7
Go

F. Femoral nerve

8
9

G. None of the above


10

Please select the most likely nerve injury for the scenario given. Each option may be used
once, more than once or not at all

11
12
13
14

58.

A 56 year old man undergoes a low anterior resection with legs in the LloydDavies position. Post operatively he complains of foot drop.

15
16

Peroneal nerve

59.

17

Positioning legs in Lloyd- Davies stirrups can carry the risk of peroneal nerve
neuropraxia if not done carefully.

18

A 23 year old man complains of severe groin pain several weeks after a difficult
inguinal hernia repair.

20

19

21
22

Ilioinguinal nerve
23

60.

The ilioinguinal nerve may have been entrapped in the mesh causing a
neuroma.

24

A 72 year old man develops a foot drop after a revision total hip replacement.

26

25

27

Sciatic nerve

28

This may be done by a number of approaches, in this scenario a posterior


approach is the most likely culprit.

29-31 3 / 3
32
33

Next question

34
35
36

Lower limb- Muscular compartments

37
38
39

Anterior compartment
Muscle

Nerve

Action

Tibialis anterior

Deep peroneal
nerve

Dorsiflexes ankle joint, inverts foot

Extensor digitorum
longus

Deep peroneal
nerve

Extends lateral four toes, dorsiflexes ankle


joint

Peroneus tertius

Deep peroneal
nerve

Dorsiflexes ankle, everts foot

Extensor hallucis longus

Deep peroneal
nerve

Dorsiflexes ankle joint, extends big toe

40
41
42
43
44
45
46-48 3 / 3
49
50
51
52

Peroneal compartment

53

Muscle

Nerve

Action

Peroneus longus

Superficial peroneal nerve

Everts foot, assists in plantar flexion

55

Peroneus brevis

Superficial peroneal nerve

Plantar flexes the ankle joint

56

54

57

Superficial posterior compartment


< b=""><>
b=""><>

Nerve

58-60 3 / 3

Action

http://www.emrcs.com/question/question.php?q=0

1/2

17/05/2015

Gastrocnemius

Tibial nerve

Plantar flexes the foot, may also flex the knee

Soleus

Tibial nerve

Plantar flexor

Deep posterior compartment


Muscle

Nerve

Action

Flexor digitorum longus

Tibial

Flexes the lateral four toes

Flexor hallucis longus

Tibial

Flexes the great toe

Tibialis posterior

Tibial

Plantar flexor, inverts the foot

Rate question:

Next question

Comment on this question

All contents of this site are 2012 E-Medical Revision Ltd

http://www.emrcs.com/question/question.php?q=0

Terms and Conditions

Privacy policy

2/2

17/05/2015

Reference ranges

Question 61 of 368

Previous

Next

A 38 year old man falls onto an outstretched hand. Following the accident he is examined in
the emergency department. On palpating his anatomical snuffbox there is tenderness noted
in the base. What is the most likely injury in this scenario?

Question stats

Score: 79.5%
1

7.8%

70.2%

8.5%

6.8%

6.6%

A. Rupture of the tendon of flexor pollicis


70.2% of users answered this
question correctly

B. Scaphoid fracture

End and review

6
7

C. Distal radius fracture


8

D. Rupture of flexor carpi ulnaris tendon


9

Search

E. None of the above

Go

Next question

10
11
12

A fall onto an outstretched hand is a common mechanism of injury for a scaphoid fracture.
This should be suspected clinically if there is tenderness in the base of the anatomical
snuffbox. A tendon rupture would not result in bony tenderness.

13
14
15

Scaphoid bone

16
17

The scaphoid has a concave articular surface for the head of the capitate and at the edge of
this is a crescentic surface for the corresponding area on the lunate.
Proximally, it has a wide convex articular surface with the radius. It has a distally sited
tubercle that can be palpated. The remaining articular surface is to the lateral side of the
tubercle. It faces laterally and is associated with the trapezium and trapezoid bones.
The narrow strip between the radial and trapezial surfaces and the tubercle gives rise to the
radial collateral carpal ligament. The tubercle receives part of the flexor retinaculum. This
area is the only part of the scaphoid that is available for the entry of blood vessels. It is
commonly fractured and avascular necrosis may result.

18
19
20
21
22
23
24
25

Scaphoid bone

26
27
28
29-31 3 / 3
32
33
34
35
36
37
38

Image sourced from Wikipedia

39
40
41

Rate question:
Comment on this question

Next question

42
43
44
45
46-48 3 / 3
49
50
51
52
53
54
55
56
57
58-60 3 / 3
61

http://www.emrcs.com/question/question.php?q=0

1/2

17/05/2015

All contents of this site are 2012 E-Medical Revision Ltd

http://www.emrcs.com/question/question.php?q=0

Terms and Conditions

Privacy policy

2/2

17/05/2015

Reference ranges

Previous

Question 62 of 368

Next

A 25 year old man sustains a severe middle cranial fossa basal skull fracture. Once he has
recovered it is noticed that he has impaired tear secretion. This is most likely to be the result
of damage to which of the following?

Question stats

Score: 80.2%
1

10.9%

28.9%

21.4%

12.2%

26.6%

A. Stellate ganglion
26.6% of users answered this
question correctly

B. Ciliary ganglion

End and review

6
7

C. Otic ganglion
8

D. Trigeminal nerve
9

Search

E. Greater petrosal nerve

Go

Next question

10
11
12

The greater petrosal nerve may be injured and carries fibres for lacrimation (see below).

13
14

Lacrimal system

15

Lacrimal gland
Consists of an orbital part and palpebral part. They are continuous posterolaterally around
the concave lateral edge of the levator palpebrae superioris muscle.
The ducts of the lacrimal gland open into the superior fornix. Those from the orbital part
penetrate the aponeurosis of levator palpebrae superioris to join those from the palpebral
part. Therefore excision of the palpebral part is functionally similar to excision of the entire
gland.

16

Blood supply
Lacrimal branch of the opthalmic artery. Venous drainage is to the superior opthalmic vein.

22

Innervation
The gland is innervated by the secretomotor paraympathetic fibres from the pterygopalatine
ganglion which in turn may reach the gland via the zygomatic or lacrimal branches of the
maxillary nerve or pass directly to the gland. The preganglionic fibres travel to the ganglion
in the greater petrosal nerve (a branch of the facial nerve at the geniculate ganglion).

17
18
19
20
21

23
24
25
26
27
28

Nasolacrimal duct
Descends from the lacrimal sac to open anteriorly in the inferior meatus of the nose.
Lacrimation reflex
Occurs in response to conjunctival irritation (or emotional events). The conjunctiva will send
signals via the opthalmic nerve. These then pass to the superior salivary centre. The
efferent signals pass via the greater petrosal nerve (parasympathetic preganglionic fibres)
and the deep petrosal nerve which carries the post ganglionic sympathetic fibres. The
parasympathetic fibres will relay in the pterygopalatine ganglion, the sympathetic fibres do
not synapse. They in turn will relay to the lacrimal apparatus.

29-31 3 / 3
32
33
34
35
36
37
38
39

Rate question:
Comment on this question

Next question

40
41
42
43
44
45
46-48 3 / 3
49
50
51
52
53
54
55
56
57
58-60 3 / 3
61

http://www.emrcs.com/question/question.php?q=0

1/2

17/05/2015
62
63

All contents of this site are 2012 E-Medical Revision Ltd

http://www.emrcs.com/question/question.php?q=0

Terms and Conditions

Privacy policy

2/2

17/05/2015

Reference ranges

Question 63 of 368

Previous

Next

Which of the following structures passes through the quadrangular space near the humeral
head?

A. Axillary artery
B. Radial nerve

Question stats

Score: 80.5%
1

24.5%

13.8%

41.5%

7.8%

12.4%

41.5% of users answered this


question correctly

C. Axillary nerve

End and review

D. Median nerve

6
7
8

E. Transverse scapular artery

Search
Next question

Go

10
11

The quadrangular space is bordered by the humerus laterally, subscapularis superiorly,


teres major inferiorly and the long head of triceps medially. It lies lateral to the triangular
space. It transmits the axillary nerve and posterior circumflex humeral artery.

12
13
14
15
16
17
18
19
20
21
22
23
24
25

Image sourced from Wikipedia

26
27

Shoulder joint

28
29-31 3 / 3

Shallow synovial ball and socket type of joint.


It is an inherently unstable joint, but is capable to a wide range of movement.
Stability is provided by muscles of the rotator cuff that pass from the scapula to insert
in the greater tuberosity (all except sub scapularis-lesser tuberosity).

32
33
34
35
36

Glenoid labrum
37

Fibrocartilaginous rim attached to the free edge of the glenoid cavity


Tendon of the long head of biceps arises from within the joint from the supraglenoid
tubercle, and is fused at this point to the labrum.
The long head of triceps attaches to the infraglenoid tubercle

38
39
40
41
42

Fibrous capsule
Attaches to the scapula external to the glenoid labrum and to the labrum itself
(postero-superiorly)
Attaches to the humerus at the level of the anatomical neck superiorly and the surgical
neck inferiorly
Anteriorly the capsule is in contact with the tendon of subscapularis, superiorly with the
supraspinatus tendon, and posteriorly with the tendons of infraspinatus and teres
minor. All these blend with the capsule towards their insertion.
Two defects in the fibrous capsule; superiorly for the tendon of biceps. Anteriorly there
is a defect beneath the subscapularis tendon.
The inferior extension of the capsule is closely related to the axillary nerve at the
surgical neck and this nerve is at risk in anteroinferior dislocations. It also means that
proximally sited osteomyelitis may progress to septic arthritis.

43
44
45
46-48 3 / 3
49
50
51
52
53
54
55

Movements and muscles


Flexion

Anterior part of deltoid


Pectoralis major
Biceps
Coracobrachialis

56
57
58-60 3 / 3
61
62

http://www.emrcs.com/question/question.php?q=0

1/2

17/05/2015

Extension

62

Posterior deltoid
Teres major
Latissimus dorsi

Adduction

Pectoralis major
Latissimus dorsi
Teres major
Coracobrachialis

Abduction

Mid deltoid
Supraspinatus

Medial rotation

Subscapularis
Anterior deltoid
Teres major
Latissimus dorsi

Lateral rotation

Posterior deltoid
Infraspinatus
Teres minor

63

Important anatomical relations


Anteriorly

Brachial plexus
Axillary artery and vein

Posterior

Suprascapular nerve
Suprascapular vessels

Inferior

Axillary nerve
Circumflex humeral vessels

Rate question:

Next question

Comment on this question

All contents of this site are 2012 E-Medical Revision Ltd

http://www.emrcs.com/question/question.php?q=0

Terms and Conditions

Privacy policy

2/2

17/05/2015

Reference ranges

Previous

Question 64 of 368

Next

Question stats

End and review

Score: 80.7%
1

14.5%

12.6%

45.3%

A. Superior orbital fissure and the oculomotor nerve

13.3%

B. Foramina rotundum and the maxillary nerve

14.2%

C. Jugular foramen and the hypoglossal nerve

45.3% of users answered this


question correctly

Which of the following pairings of foramina and their contents is not correct?

D. Foramina spinosum and the middle meningeal artery


E. Carotid canal and the internal carotid artery

6
7
8
9

Search
Next question
Go

The hypoglossal nerve passes through the hypoglossal canal.

10
11
12

Foramina of the base of the skull

13
14

Foramen

Location

Contents

Foramen
ovale

Sphenoid
bone

Otic ganglion
V3 (Mandibular nerve:3rd branch of
trigeminal)
Accessory meningeal artery
Lesser petrosal nerve
Emissary veins

15
16
17
18
19
20

Foramen
spinosum

Sphenoid
bone

Middle meningeal artery


Meningeal branch of the Mandibular nerve

21

Foramen
rotundum

Sphenoid
bone

Maxillary nerve (V2)

23

Foramen
lacerum/
carotid canal

Sphenoid
bone

Base of the medial pterygoid plate.


Internal carotid artery*
Nerve and artery of the pterygoid canal

Jugular
foramen

Temporal
bone

Anterior: inferior petrosal sinus


Intermediate: glossopharyngeal, vagus, and accessory nerves.
Posterior: sigmoid sinus (becoming the internal jugular vein) and
some meningeal branches from the occipital and ascending
pharyngeal arteries.

22

24
25
26
27
28
29-31 3 / 3
32
33

Foramen
magnum

Occipital
bone

Anterior and posterior spinal arteries


Vertebral arteries
Medulla oblongata

34
35
36

Stylomastoid
foramen

Temporal
bone

Stylomastoid artery
Facial nerve

Superior
orbital fissure

Sphenoid
bone

Oculomotor nerve (III)


trochlear nerve (IV)
lacrimal, frontal and nasociliary branches of ophthalmic nerve (V1)
abducent nerve (VI)
Superior and inferior ophthalmic vein

37
38
39
40
41
42

*= In life the foramen lacerum is occluded by a cartilagenous plug. The ICA initially passes
into the carotid canal which ascends superomedially to enter the cranial cavity through the
foramen lacerum.

43
44
45

Base of skull anatomical overview

46-48 3 / 3
49
50
51
52
53
54
55
56
57
58-60 3 / 3
61

Image sourced from Wikipedia

http://www.emrcs.com/question/question.php?q=0

1/2

17/05/2015
Image sourced from Wikipedia

62
63

Rate question:

64

Next question

Comment on this question

All contents of this site are 2012 E-Medical Revision Ltd

http://www.emrcs.com/question/question.php?q=0

Terms and Conditions

Privacy policy

2/2

17/05/2015

Reference ranges

Previous

Question 65 of 368

Next

A 55 year old man with carcinoma of the larynx is undergoing a difficult laryngectomy. The
surgeons divide the thyrocervical trunk, from which of the following vessels does this
structure most commonly originate?

Question stats

Score: 80.9%
1

59.1%

10.4%

8.6%

11.8%

10.1%

A. Subclavian artery
59.1% of users answered this
question correctly

B. Common carotid artery

End and review

6
7

C. Vertebral artery
8

D. External carotid artery


9

Search

E. Internal carotid artery

Go

Next question

10
11
12

The thyrocervical trunk is a branch of the subclavian artery. It arises from the first part
between the subclavian artery and the inner border of scalenus anterior. It branches off the
subclavian distal to the vertebral artery.

13
14
15

Root of the neck

16
17

Thoracic Outlet

18
19

Where the subclavian artery and vein and the brachial plexus exit the thorax and enter
the arm.
They pass over the 1st rib and under the clavicle.
The subclavian vein is the most anterior structure and is immediately anterior to
scalenus anterior and its attachment to the first rib.
Scalenus anterior has 2 parts, the subclavian artery leaves the thorax by passing over
the first rib and between these 2 portions of the muscle.
At the level of the first rib, the lower cervical nerve roots combine to form the 3 trunks
of the brachial plexus. The lowest trunk is formed by the union of C8 and T1, and this
trunk lies directly posterior to the artery and is in contact with the superior surface of
the first rib.

20
21
22
23
24
25
26
27
28
29-31 3 / 3

Thoracic outlet obstruction causes neurovascular compromise.

32

Rate question:

Next question

33
34

Comment on this question

35
36
37
38
39
40
41
42
43
44
45
46-48 3 / 3
49
50
51
52
53
54
55
56
57
58-60 3 / 3
61

http://www.emrcs.com/question/question.php?q=0

1/2

17/05/2015
62
63
64
65

All contents of this site are 2012 E-Medical Revision Ltd

http://www.emrcs.com/question/question.php?q=0

Terms and Conditions

Privacy policy

2/2

17/05/2015

Reference ranges

Previous

Question 66 of 368

Next

Question stats

End and review

Score: 81.1%
1

17.1%

13.3%

25.9%

A. Trachea posteriorly

31%

B. Right brachiocephalic vein

12.7%

C. Inferior thyroid vein

31% of users answered this


question correctly

The following structures are closely related to the brachiocephalic artery except:

D. Right recurrent laryngeal nerve


E. None of the above

6
7
8
9

Search
Next question
Go

There is no brachiocephalic artery on the left, however the left brachiocephalic vein lies
anteriorly to the roots of all the 3 great arteries (including the brachiocephalic artery). The
right recurrent laryngeal nerve has no relation to the brachiocephalic artery.
Brachiocephalic artery

10
11
12
13
14
15

The brachiocephalic artery is the largest branch of the aortic arch. From its aortic origin it
ascends superiorly, it initially lies anterior to the trachea and then on its right hand side. It
branches into the common carotid and right subclavian arteries at the level of the
sternoclavicular joint.

16
17
18
19

Path
Origin- apex of the midline of the aortic arch
Passes superiorly and posteriorly to the right
Divides into the right subclavian and right common carotid artery

20
21
22
23

Relations
Anterior

24

Sternohyoid
Sternothyroid
Thymic remnants
Left brachiocephalic vein
Right inferior thyroid veins

25
26
27
28
29-31 3 / 3

Posterior

Trachea
Right pleura

32
33
34

Right lateral

Right brachiocephalic vein


Superior part of SVC

35
36
37

Left lateral

Thymic remnants
Origin of left common carotid
Inferior thyroid veins
Trachea (higher level)

38
39
40
41

Branches
Normally none but may have the thyroidea ima artery

42
43
44

http://www.emrcs.com/question/question.php?q=0

1/2

17/05/2015
44
45
46-48 3 / 3
49
50
51
52
53
54
55
56
57
58-60 3 / 3
61
62
63
64

Image sourced from Wikipedia

65
66

Rate question:

Next question

Comment on this question

All contents of this site are 2012 E-Medical Revision Ltd

http://www.emrcs.com/question/question.php?q=0

Terms and Conditions

Privacy policy

2/2

17/05/2015

Reference ranges

Question 67 of 368

Previous

Next

Question stats

End and review

Score: 81.3%
1

10%

30.1%

19.1%

A. Brachioradialis

30.8%

B. Pronator teres

9.9%

C. Tendon of biceps brachii

30.1% of users answered this


question correctly

Which of the following structures separates the ulnar artery from the median nerve?

D. Flexor carpi ulnaris


E. Brachialis

6
7
8
9

Search
Next question
Go

10
11

It lies deep to pronator teres and this separates it from the median nerve.

12

Ulnar artery

13
14

Path

15

Starts: middle of antecubital fossa


Passes obliquely downward, reaching the ulnar side of the forearm at a point about
midway between the elbow and the wrist. It follows the ulnar border to the wrist,
crossing over the flexor retinaculum. It then divides into the superficial and deep volar
arches.

16
17
18
19
20

Relations
Deep to- Pronator teres, Flexor carpi radialis, Palmaris longus
Lies on- Brachialis and Flexor digitorum profundus
Superficial to the flexor retinaculum at the wrist

21
22
23
24

The median nerve is in relation with the medial side of the artery for about 2.5 cm. And then
crosses the vessel, being separated from it by the ulnar head of the Pronator teres

25
26

The ulnar nerve lies medially to the lower two-thirds of the artery

27
28

Branch

29-31 3 / 3

Anterior interosseous artery

32
33
34
35
36
37
38
39
40
41
42
43
44
45
46-48 3 / 3
49
50
51

Image sourced from Wikipedia

52
53
54

Rate question:

Next question

55
Comment on this question

56
57
58-60 3 / 3
61

http://www.emrcs.com/question/question.php?q=0

1/2

17/05/2015
62
63
64
65
66
67

All contents of this site are 2012 E-Medical Revision Ltd

http://www.emrcs.com/question/question.php?q=0

Terms and Conditions

Privacy policy

2/2

17/05/2015

Reference ranges

Previous

Question 68 of 368

Next

Question stats

End and review

Score: 81.5%
1

19.9%

8.8%

8.1%

A. Peroneus tertius

55%

B. Sartorius

8.2%

C. Adductor magnus

55% of users answered this


question correctly

Which muscle is supplied by the superficial peroneal nerve?

D. Peroneus brevis
E. Gracilis

6
7
8
9

Search
Next question
Go

10
11
12

Superficial peroneal nerve

13
14

Supplies

15

Lateral compartment of leg: peroneus longus, peroneus brevis (action: eversion and
plantar flexion)
Sensation over dorsum of the foot (except the first web space, which is innervated by
the deep peroneal nerve)

16
17
18
19
20

Path
Passes between peroneus longus and peroneus brevis along the length of the
proximal one third of the fibula
10-12 cm above the tip of the lateral malleolus, the superficial peroneal nerve pierces
the fascia
6-7 cm distal to the fibula, the superficial peroneal nerve bifurcates into intermediate
and medial dorsal cutaneous nerves

21
22
23
24
25
26
27

Rate question:

Next question

28
29-31 3 / 3

Comment on this question

32
33
34
35
36
37
38
39
40
41
42
43
44
45
46-48 3 / 3
49
50
51
52
53
54
55
56
57
58-60 3 / 3
61

http://www.emrcs.com/question/question.php?q=0

1/2

17/05/2015
62
63
64
65
66
67
68

All contents of this site are 2012 E-Medical Revision Ltd

http://www.emrcs.com/question/question.php?q=0

Terms and Conditions

Privacy policy

2/2

17/05/2015

Reference ranges

Previous

Question 69 of 368

Next

A 32 year old motorcyclist is involved in a road traffic accident. His humerus is fractured and
severely displaced. At the time of surgical repair the surgeon notes that the radial nerve has
been injured. Which of the following muscles is least likely to be affected by an injury at this
site?

Question stats

Score: 79.8%
1

9.3%

22.4%

36.3%

10.6%

21.4%

21.4% of users answered this


question correctly

A. Extensor carpi radialis brevis

End and review

B. Brachioradialis

6
7
8

C. Abductor pollicis longus

Search

D. Extensor pollicis brevis


E. None of the above

Go

10
11

Next question

12
13

Muscles supplied by the radial nerve

14
15

BEST
Brachioradialis
Extensors
Supinator
Triceps

16
17
18
19
20

The radial nerve supplies the extensor muscles, abductor pollicis longus and extensor pollicis
brevis (the latter two being innervated by the posterior interosseous branch of the radial
nerve).

21
22
23

Radial nerve

24

Continuation of posterior cord of the brachial plexus (root values C5 to T1)

25
26

Path

27

In the axilla: lies posterior to the axillary artery on subscapularis, latissimus dorsi and
teres major.
Enters the arm between the brachial artery and the long head of triceps (medial to
humerus).
Spirals around the posterior surface of the humerus in the groove for the radial nerve.
At the distal third of the lateral border of the humerus it then pierces the intermuscular
septum and descends in front of the lateral epicondyle.
At the lateral epicondyle it lies deeply between brachialis and brachioradialis where it
then divides into a superficial and deep terminal branch.
Deep branch crosses the supinator to become the posterior interosseous nerve.

28
29-31 3 / 3
32
33
34
35
36
37
38

In the image below the relationships of the radial nerve can be appreciated

39
40
41
42
43
44
45
46-48 3 / 3
49
50
51
52
53
54

Image sourced from Wikipedia

55
56

Regions innervated
Motor (main
nerve)

57
Triceps
Anconeus
Brachioradialis
Extensor carpi radialis

http://www.emrcs.com/question/question.php?q=0

58-60 3 / 3
61

1/2

17/05/2015
Extensor carpi radialis

62
63

Motor
(posterior
interosseous
branch)

Supinator
Extensor carpi ulnaris
Extensor digitorum
Extensor indicis
Extensor digiti minimi
Extensor pollicis longus and brevis
Abductor pollicis longus

64
65
66
67
68

Sensory

The area of skin supplying the proximal phalanges on the dorsal aspect of
the hand is supplied by the radial nerve (this does not apply to the little
finger and part of the ring finger)

69

Muscular innervation and effect of denervation


Anatomical
location

Muscle affected

Effect of paralysis

Shoulder

Long head of triceps

Minor effects on shoulder stability in


abduction

Arm

Triceps

Loss of elbow extension

Forearm

Supinator
Brachioradialis
Extensor carpi radialis
longus and brevis

Weakening of supination of prone hand and


elbow flexion in mid prone position

The cutaneous sensation of the upper limb- illustrating the contribution of the radial nerve

Image sourced from Wikipedia

Rate question:

Next question

Comment on this question

All contents of this site are 2012 E-Medical Revision Ltd

http://www.emrcs.com/question/question.php?q=0

Terms and Conditions

Privacy policy

2/2

17/05/2015

Reference ranges

Previous

Question 70 of 368

Next

A man develops an infection in his external auditory meatus. The infection is extremely
painful. Which of the following nerves conveys sensation from this region?

A. Occipital branch of the trigeminal nerve


B. Vestibulocochlear nerve

Question stats

Score: 80%
1

13.5%

12.5%

14.2%

45.6%

14.2%

45.6% of users answered this


question correctly

C. Facial nerve

End and review

D. Auriculotemporal nerve

6
7
8

E. Maxillary branch of the trigeminal nerve

Search
Next question

Go

10
11
12

Tensor tympania and stapedius are the only two muscles of the middle ear. Contraction
of tensor tympani will tend to dampen the vibrations produced by loud sounds, it is
innervated by a branch of the trigeminal nerve. The stapedius dampens movements of
the ossicles in response to loud sounds and is innervated by a branch of the facial
nerve.

13
14
15
16
17

The auriculotemporal nerve, which is derived from the mandibular branch of the trigeminal
nerve supplies this area.

18
19

Ear- anatomy

20
21

The ear is composed of three anatomically distinct regions.

22

External ear
Auricle is composed of elastic cartilage covered by skin. The lobule has no cartilage and
contains fat and fibrous tissue.

23

External auditory meatus is approximately 2.5cm long.


Lateral third of the external auditory meatus is cartilaginous and the medial two thirds is
bony.

26

The region is innervated by the greater auricular nerve. The auriculotemporal branch of the
trigeminal nerve supplies most the of external auditory meatus and the lateral surface of the
auricle.

24
25

27
28
29-31 3 / 3
32
33
34

Middle ear
Space between the tympanic membrane and cochlea. The aditus leads to the mastoid air
cells is the route through which middle ear infections may cause mastoiditis. Anteriorly the
eustacian tube connects the middle ear to the naso pharynx.
The tympanic membrane consists of:
Outer layer of stratified squamous epithelium.
Middle layer of fibrous tissue.
Inner layer of mucous membrane continuous with the middle ear.
The tympanic membrane is approximately 1cm in diameter.
The chorda tympani nerve passes on the medial side of the pars flaccida.
The middle ear is innervated by the glossopharyngeal nerve and pain may radiate to the
middle ear following tonsillectomy.

35
36
37
38
39
40
41
42
43
44
45
46-48 3 / 3

Ossicles
Malleus attaches to the tympanic membrane (the Umbo).
Malleus articulates with the incus (synovial joint).
Incus attaches to stapes (another synovial joint).

49
50
51
52

Internal ear
Cochlea, semi circular canals and vestibule

53
54

Organ of corti is the sense organ of hearing and is located on the inside of the cochlear duct
on the basilar membrane.
Vestibule accommodates the utricule and the saccule. These structures contain endolymph
and are surrounded by perilymph within the vestibule.

55
56
57
58-60 3 / 3
61

The semicircular canals lie at various angles to the petrous temporal bone. All share a
62

http://www.emrcs.com/question/question.php?q=0

1/2

17/05/2015
62

common opening into the vestibule.

63

Rate question:

64

Next question

65
Comment on this question

66
67
68
69
70

All contents of this site are 2012 E-Medical Revision Ltd

http://www.emrcs.com/question/question.php?q=0

Terms and Conditions

Privacy policy

2/2

17/05/2015

Reference ranges

Previous

Question 71 of 368

Next

Question stats

End and review

Score: 80.2%
1

45.6%

23.7%

10.1%

A. Flexor pollicis longus

12.5%

B. Flexor pollicis brevis

8.2%

C. Flexor digitorum superficialis

45.6% of users answered this


question correctly

Which muscle is responsible for causing flexion of the interphalangeal joint of the thumb?

D. Flexor digitorum profundus


E. Adductor pollicis

6
7
8
9

Search
Next question
Go

There are 8 muscles:


1. Two flexors (flexor pollicis brevis and flexor pollicis longus)
2. Two extensors (extensor pollicis brevis and longus)
3. Two abductors (abductor pollicis brevis and longus)
4. One adductor (adductor pollicis)
5. One muscle that opposes the thumb by rotating the CMC joint (opponens pollicis).

10
11
12
13
14
15
16

Flexor and extensor longus insert on the distal phalanx moving both the MCP and IP joints.

17
18

Hand

19

Anatomy of the hand


Bones

Intrinsic Muscles

20

8 Carpal bones
5 Metacarpals
14 phalanges
7 Interossei - Supplied by ulnar nerve

21
22
23
24
25

3 palmar-adduct fingers
4 dorsal- abduct fingers

26
27

Intrinsic muscles

Lumbricals
Flex MCPJ and extend the IPJ.
Origin deep flexor tendon and insertion dorsal extensor hood
mechanism.
Innervation: 1st and 2nd- median nerve, 3rd and 4th- deep branch of
the ulnar nerve.

28
29-31 3 / 3
32
33
34
35

Thenar eminence

Abductor pollicis brevis


Opponens pollicis
Flexor pollicis brevis

36
37
38

Hypothenar
eminence

39

Opponens digiti minimi


Flexor digiti minimi brevis
Abductor digiti minimi

40
41
42
43
44
45
46-48 3 / 3
49
50
51
52
53
54
55
56
57
58-60 3 / 3
61
62

http://www.emrcs.com/question/question.php?q=0

1/2

17/05/2015
62

Image sourced from Wikipedia

63

Fascia and compartments of the palm


The fascia of the palm is continuous with the antebrachial fascia and the fascia of the
dorsum of the hand. The palmar fascia is thin over the thenar and hypothenar eminences. In
contrast the palmar fascia is relatively thick. The palmar aponeurosis covers the soft tissues
and overlies the flexor tendons. The apex of the palmar aponeurosis is continuous with the
flexor retinaculum and the palmaris longus tendon. Distally, it forms four longitudinal digital
bands that attach to the bases of the proximal phalanges, blending with the fibrous digital
sheaths.
A medial fibrous septum extends deeply from the medial border of the palmar aponeurosis to
the 5th metacarpal. Lying medial to this are the hypothenar muscles. In a similar fashion, a
lateral fibrous septum extends deeply from the lateral border of the palmar aponeurosis to
the 3rd metacarpal. The thenar compartment lies lateral to this area.
Lying between the thenar and hypothenar compartments is the central compartment. It
contains the flexor tendons and their sheaths, the lumbricals, the superficial palmar arterial
arch and the digital vessels and nerves.
The deepest muscular plane is the adductor compartment, which contains adductor pollicis.

64
65
66
67
68
69
70
71

Short muscles of the hand


These comprise the lumbricals and interossei. The four slender lumbrical muscles flex the
fingers at the metacarpophalangeal joints and extend the interphalangeal joint. The four
dorsal interossei are located between the metacarpals and the four palmar interossei lie on
the palmar surface of the metacarpals in the interosseous compartment of the hand.
Long flexor tendons and sheaths in the hand
The tendons of FDS and FDP enter the common flexor sheath deep to the flexor
retinaculum. The tendons enter the central compartment of the hand and fan out to their
respective digital synovial sheaths. Near the base of the proximal phalanx, the tendon of FDS
splits to permit the passage of FDP. The FDP tendons are attached to the margins of the
anterior aspect of the base of the distal phalanx.
The fibrous digital sheaths contain the flexor tendons and their synovial sheaths. These
extend from the heads of the metacarpals to the base of the distal phalanges.
Rate question:

Next question

Comment on this question

All contents of this site are 2012 E-Medical Revision Ltd

http://www.emrcs.com/question/question.php?q=0

Terms and Conditions

Privacy policy

2/2

17/05/2015

Reference ranges

Question 72 of 368

Previous

Next

Which of the following structures separates the posterior cruciate ligament from the popliteal
artery?

A. Oblique popliteal ligament


B. Transverse ligament

Question stats

Score: 80.4%
1

31%

18.3%

29.6%

10.9%

10.1%

31% of users answered this


question correctly

C. Popliteus tendon

End and review

D. Biceps femoris

6
7
8

E. Semitendinosus

Search
Next question

Go

10
11

The posterior cruciate ligament is separated from the popliteal vessels at its origin by the
oblique popliteal ligament. The transverse ligament is located anteriorly.

12
13
14

Knee joint

15

The knee joint is a synovial joint, the largest and most complicated. It consists of two
condylar joints between the femure and tibia and a sellar joint between the patella and the
femur. The tibiofemoral articular surfaces are incongruent, however, this is improved by the
presence of the menisci. The degree of congruence is related to the anatomical position of
the knee joint and is greatest in full extension.

16
17
18
19
20

Knee joint compartments


Tibiofemoral

Comprise of the patella/femur joint, lateral and medial


compartments (betw een femur condyles and tibia)
Synovial membrane and cruciate ligaments partially separate the
medial and lateral compartments

Patellofemoral

Ligamentum patellae
Actions: provides joint stability in full extension

21
22
23
24
25
26
27

Fibrous capsule
The capsule of the knee joint is a complex, composite structure with contributions from
adjacent tendons.
Anterior
fibres

The capsule does not pass proximal to the patella. It blends w ith the
tendinous expansions of vastus medialis and lateralis

Posterior These fibres are vertical and run from the posterior surface of the femoral
fibres
condyles to the posterior aspect of the tibial condyle
Medial
fibres

Attach to the femoral and tibial condyles beyond their articular margins,
blending w ith the tibial collateral ligament

Lateral
fibres

Attach to the femur superior to popliteus, pass over its tendon to head of
fibula and tibial condyle

28
29-31 3 / 3
32
33
34
35
36
37
38
39

Bursae

40
Anterior

Subcutaneous prepatellar bursa; betw een patella and skin


Deep infrapatellar bursa; betw een tibia and patellar ligament
Subcutaneous infrapatellar bursa; betw een distal tibial tuberosity and
skin

41
42
43

Laterally

Bursa betw een lateral head of gastrocnemius and joint capsule


Bursa betw een fibular collateral ligament and tendon of biceps femoris
Bursa betw een fibular collateral ligament and tendon of popliteus

44
45
46-48 3 / 3
49

Medially

Bursa betw een medial head of gastrocnemius and the fibrous capsule
Bursa betw een tibial collateral ligament and tendons of sartorius, gracilis
and semitendinosus
Bursa betw een the tendon of semimembranosus and medial tibial
condyle and medial head of gastrocnemius

Posterior Highly variable and inconsistent

50
51
52
53
54

Ligaments

55

Medial collateral
ligament

Medial epicondyle femur to medial tibial condyle: valgus


stability

Lateral collateral
ligament

Lateral epicondyle femur to fibula head: varus stability

56
57
58-60 3 / 3
61
62

http://www.emrcs.com/question/question.php?q=0

1/2

17/05/2015

Anterior cruciate
ligament

Anterior tibia to lateral intercondylar notch femur: prevents


tibia sliding anteriorly

Posterior cruciate
ligament

Posterior tibia to medial intercondylar notch femur: prevents


tibia sliding posteriorly

Patellar ligament

Central band of the tendon of quadriceps femoris, extends


from patella to tibial tuberosity

62
63
64
65
66
67
68
69
70
71
72

Image sourced from Wikipedia

Image sourced from Wikipedia

Menisci
Medial and lateral menisci compensate for the incongruence of the femoral and tibial
condyles.
Composed of fibrous tissue.
Medial meniscus is attached to the tibial collateral ligament.
Lateral meniscus is attached to the loose fibres at the lateral edge of the joint and is
separate from the fibular collateral ligament. The lateral meniscus is crossed by the popliteus
tendon.
Nerve supply
The knee joint is supplied by the femoral, tibial and common peroneal divisions of the sciatic
and by a branch from the obturator nerve. Hip pathology pain may be referred to the knee.
Blood supply
Genicular branches of the femoral artery, popliteal and anterior tibial arteries all supply the
knee joint.
Rate question:

Next question

Comment on this question

All contents of this site are 2012 E-Medical Revision Ltd

http://www.emrcs.com/question/question.php?q=0

Terms and Conditions

Privacy policy

2/2

17/05/2015

Reference ranges

Previous

Question 73 of 368

Next

Question stats

End and review

Score: 78.8%
1

9.9%

7.4%

30.2%

A. 2

9.6%

B. 1

42.9%

C. 3

42.9% of users answered this


question correctly

How many compartments are there in the lower leg?

D. 5
E. 4

6
7
8
9

Search
Next question
Go

The deep compartment of the lower leg has both superficial and deep posterior layers,
together with the anterior and lateral compartments this allows for four compartments.
Decompression of the deep posterior compartment during fasciotomy may be overlooked
with significant sequelae.

10
11
12
13
14
15

Fascial compartments of the leg

16

Compartments of the thigh

17
18

Formed by septae passing from the femur to the fascia lata.


Compartment

Nerve

Anterior compartment

Femoral

Muscles

19

Blood supply

20

Iliacus
Tensor fasciae latae
Sartorius
Quadriceps femoris

Femoral artery

21
22
23
24

Medial compartment

Obturator

Adductor
longus/magnus/brevis
Gracilis
Obturator externus

Profunda femoris artery


and obturator artery

25
26
27
28

Posterior compartment
(2 layers)

Sciatic

Semimembranosus
Semitendinosus
Biceps femoris

Branches of Profunda
femoris artery

29-31 3 / 3
32
33
34
35

Compartments of the lower leg


Separated by the interosseous membrane (anterior and posterior compartments), anterior
fascial septum (separate anterior and lateral compartments) and posterior fascial septum
(separate lateral and posterior compartments)

36
37
38
39

Compartment

Anterior
compartment

Nerve

Muscles

Deep
peroneal
nerve

Tibialis anterior
Extensor digitorum longus
Extensor hallucis longus
Peroneus tertius

Blood
supply

40

Anterior
tibial
artery

42

41

43
44
45
46-48 3 / 3

Posterior
compartment

Tibial

Muscles: deep and superficial


compartments (separated by deep
transverse fascia)
Deep: Flexor hallucis longus, Flexor digitalis
longus, Tibialis posterior, Popliteus
Superficial: Gastrocnemius, Soleus,
Plantaris

Posterior
tibial

49
50
51
52
53
54

Lateral
compartment

Superficial
peroneal

Peroneus longus/brevis

Anterior
tibial

55
56
57
58-60 3 / 3

Rate question:

Next question

61
62

http://www.emrcs.com/question/question.php?q=0

1/2

17/05/2015
62

Comment on this question

63
64
65
66
67
68
69
70
71
72
73

All contents of this site are 2012 E-Medical Revision Ltd

http://www.emrcs.com/question/question.php?q=0

Terms and Conditions

Privacy policy

2/2

17/05/2015

Reference ranges

Previous

Question 74 of 368

Next

Question stats

End and review

Score: 77.2%
1

39.1%

12.7%

16%

A. Left brachiocephalic vein

11.7%

B. Intervertebral discs T4-T5

20.5%

C. Start of aortic arch

39.1% of users answered this


question correctly

Which structure is least likely to be found at the level of the sternal angle?

D. 2nd pair of costal cartilages


E. Bifurcation of the trachea into left and right bronchi

6
7
8
9

Search
Next question
Go

The left brachiocephalic vein lies posterior to the manubrium, at the level of its upper border.
The sternal angle refers to the transition between manubrium and sternum and therefore will
not include the left brachiocephalic vein.

10
11
12
13
14

Sternal angle

15

Anatomical structures at the level of the manubrium and upper sternum


Upper part of the manubrium

16
17

Left brachiocephalic vein


Brachiocephalic artery
Left common carotid
Left subclavian artery

18
19
20

Lower part of the manubrium/


manubrio-sternal angle

21

Costal cartilages of the 2nd ribs


Transition point between superior and
inferior mediastinum
Arch of the aorta
Tracheal bifurcation
Union of the azygos vein and superior
vena cava
The thoracic duct crosses to the midline

22
23
24
25
26
27
28
29-31 3 / 3

Rate question:
Comment on this question

Next question

32
33
34
35
36
37
38
39
40
41
42
43
44
45
46-48 3 / 3
49
50
51
52
53
54
55
56
57
58-60 3 / 3
61

http://www.emrcs.com/question/question.php?q=0

1/2

17/05/2015
62
63
64
65
66
67
68
69
70
71
72
73
74

All contents of this site are 2012 E-Medical Revision Ltd

http://www.emrcs.com/question/question.php?q=0

Terms and Conditions

Privacy policy

2/2

17/05/2015

Reference ranges

Previous

Question 75 of 368

Next

A 53 year old man is undergoing a left hemicolectomy for carcinoma of the descending
colon. From which embryological structure is this region of the gastrointestinal tract derived?

A. Vitellino-intestinal duct
B. Hind gut

Question stats

Score: 77.5%
1

8.2%

62.2%

13.4%

9.4%

6.8%

62.2% of users answered this


question correctly

C. Mid gut

End and review

D. Fore gut

6
7
8

E. Woolffian duct

Search
Next question

Go

10
11

The left colon is embryologically part of the hind gut. Which accounts for its separate blood
supply via the IMA.
Colon anatomy

12
13
14
15

The colon commences with the caecum. This represents the most dilated segment of the
human colon and its base (which is intraperitoneal) is marked by the convergence of teniae
coli. At this point is located the vermiform appendix. The colon continues as the ascending
colon, the posterior aspect of which is retroperitoneal. The line of demarcation between the
intra and retro peritoneal right colon is visible as a white line, in the living, and forms the line
of incision for colonic resections.
The ascending colon becomes the transverse colon after passing the hepatic flexure. At this
located the colon becomes wholly intra peritoneal once again. The superior aspect of the
transverse colon is the point of attachment of the transverse colon to the greater omentum.
This is an important anatomical site since division of these attachments permits entry into the
lesser sac. Separation of the greater omentum from the transverse colon is a routine
operative step in both gastric and colonic resections.

16
17
18
19
20
21
22
23
24
25
26

At the left side of the abdomen the transverse colon passes to the left upper quadrant and
makes an oblique inferior turn at the splenic flexure. Following this, the posterior aspect
becomes retroperitoneal once again.

27
28
29-31 3 / 3

At the level of approximately L4 the descending colon becomes wholly intraperitoneal and
becomes the sigmoid colon. Whilst the sigmoid is wholly intraperitoneal there are usually
attachments laterally between the sigmoid and the lateral pelvic sidewall. These small
congenital adhesions are not formal anatomical attachments but frequently require division
during surgical resections.

32

At its distal end the sigmoid passes to the midline and at the region around the sacral
promontary it becomes the upper rectum. This transition is visible macroscopically as the
point where the teniae fuse. More distally the rectum passes through the peritoneum at the
region of the peritoneal reflection and becomes extraperitoneal.

36

33
34
35

37
38
39

Arterial supply
Superior mesenteric artery and inferior mesenteric artery: linked by the marginal artery.
Ascending colon: ileocolic and right colic arteries
Transverse colon: middle colic artery
Descending and sigmoid colon: inferior mesenteric artery

40
41
42
43
44

Venous drainage
From regional veins (that accompany arteries) to superior and inferior mesenteric vein

45
46-48 3 / 3

Lymphatic drainage
Initially along nodal chains that accompany supplying arteries, then para-aortic nodes.

49

Embryology
Midgut- Second part of duodenum to 2/3 transverse colon
Hindgut- Distal 1/3 transverse colon to anus

51

Peritoneal location
The right and left colon are part intraperitoneal and part extraperitoneal. The sigmoid and
transverse colon are generally wholly intraperitoneal. This has implications for the sequelae
of perforations, which will tend to result in generalised peritonitis in the wholly intra peritoneal
segments.

54

50

52
53

55
56
57
58-60 3 / 3

Colonic relations

61
62

http://www.emrcs.com/question/question.php?q=0

1/2

17/05/2015
62

Region of colon

Relation

Caecum/ right colon

Right ureter, gonadal vessels

Hepatic flexure

Gallbladder (medially)

65

Splenic flexure

Spleen and tail of pancreas

66

Distal sigmoid/ upper


rectum

Left ureter

Rectum

Ureters, autonomic nerves, seminal vesicles, prostate, urethra


(distally)

63
64

67
68
69
70
71
72

Rate question:

73

Next question

74
Comment on this question

75

All contents of this site are 2012 E-Medical Revision Ltd

http://www.emrcs.com/question/question.php?q=0

Terms and Conditions

Privacy policy

2/2

17/05/2015

Reference ranges

Previous

Question 76 of 368

Next

What is the most useful test to clinically distinguish between an upper and lower motor
neurone lesion of the facial nerve?

A. Blow cheeks out


B. Loss of chin reflex

Question stats

Score: 77.9%
1

12.6%

12.7%

12.8%

53.6%

8.2%

53.6% of users answered this


question correctly

C. Close eye

End and review

D. Raise eyebrow

6
7
8

E. Open mouth against resistance

Search
Next question

Go

10
11
12

Upper motor neurone lesions of the facial nerve- Paralysis of the lower half of face.
Lower motor neurone lesion- Paralysis of the entire ipsilateral face.

13
14
15

Theme from April 2012 Exam

16
17

Upper Vs Lower motor neurone lesions - Facial nerve

18

The nucleus of the facial nerve is located in the caudal aspect of the ventrolateral pontine
tegmentum. Its axons exit the ventral pons medial to the spinal trigeminal nucleus.

19
20

Any lesion occurring within or affecting the corticobulbar tract is known as an upper motor
neuron lesion. Any lesion affecting the individual branches (temporal, zygomatic, buccal,
mandibular and cervical) is known as a lower motor neuron lesion.

21
22
23

Branches of the facial nerve leaving the facial motor nucleus (FMN) for the muscles do so via
both left and right posterior (dorsal) and anterior (ventral) routes. In other words, this means
lower motor neurons of the facial nerve can leave either from the left anterior, left posterior,
right anterior or right posterior facial motor nucleus. The temporal branch travels out from
the left and right posterior components. The inferior four branches do so via the left and right
anterior components. The left and right branches supply their respective sides of the face
(ipsilateral innervation). Accordingly, the posterior components receive motor input from both
hemispheres of the cerebral cortex (bilaterally), whereas the anterior components receive
strictly contra-lateral input. This means that the temporal branch of the facial nerve receives
motor input from both hemispheres of the cerebral cortex whereas the zygomatic, buccal,
mandibular and cervical branches receive information from only contralateral hemispheres.

24
25
26
27
28
29-31 3 / 3
32
33
34

Now, because the anterior FMN receives only contralateral cortical input whereas the
posterior receives that which is bilateral, a corticobulbar lesion (UMN lesion) occurring in the
left hemisphere would eliminate motor input to the right anterior FMN component, thus
removing signaling to the inferior four facial nerve branches, thereby paralyzing the right
mid- and lower-face. The posterior component, however, although now only receiving input
from the right hemisphere, is still able to allow the temporal branch to sufficiently innervate
the entire forehead. This means that the forehead will not be paralyzed.

35

The same mechanism applies for an upper motor neuron lesion in the right hemisphere. The
left anterior FMN component no longer receives cortical motor input due to its strict
contralateral innervation, whereas the posterior component is still sufficiently supplied by the
left hemisphere. The result is paralysis of the left mid- and lower-face with an unaffected
forehead.

41

36
37
38
39
40

42
43
44
45

On the other hand, a lower motor neuron lesion is a bit different.

46-48 3 / 3

A lesion on either the left or right side would affect both the anterior and posterior routes on
that side because of their close physical proximity to one another. So, a lesion on the left
side would inhibit muscle innervation from both the left posterior and anterior routes, thus
paralyzing the whole left side of the face (Bells Palsy). With this type of lesion, the bilateral
and contalateral inputs of the posterior and anterior routes, respectively, become irrelevant
because the lesion is below the level of the medulla and the facial motor nucleus. Whereas
at a level above the medulla a lesion occurring in one hemisphere would mean that the other
hemisphere could still sufficiently innervate the posterior facial motor nucleus, a lesion
affecting a lower motor neuron would eliminate innervation altogether because the nerves no
longer have a means to receive compensatory contralateral input at a downstream
decussation.

49
50
51
52
53
54
55
56
57
58-60 3 / 3
61

Rate question:

http://www.emrcs.com/question/question.php?q=0

Next question

1/2

17/05/2015

Rate question:

Next question

62

Comment on this question

63
64
65
66
67
68
69
70
71
72
73
74
75
76

All contents of this site are 2012 E-Medical Revision Ltd

http://www.emrcs.com/question/question.php?q=0

Terms and Conditions

Privacy policy

2/2

17/05/2015

Reference ranges

Question 79 of 368

Previous

Next

A 25 year old man undergoes an excision of a pelvic chondrosarcoma, during the operation
the obturator nerve is sacrificed. Which of the following muscles is least likely to be affected
as a result?

Question stats

Score: 76.9%
1

11.2%

15.9%

11%

52%

9.9%

A. Adductor longus
52% of users answered this
question correctly

B. Pectineus

End and review

6
7

C. Adductor magnus
8

D. Sartorius
9

Search

E. Gracilis

Go

Next question

10
11
12

Sartorius is supplied by the femoral nerve. In approximately 20% of the population, pectineus
is supplied by the accessory obturator nerve.

13

Obturator nerve

15

14

16

The obturator nerve arises from L2, L3 and L4 by branches from the ventral divisions of
each of these nerve roots. L3 forms the main contribution and the second lumbar branch is
occasionally absent. These branches unite in the substance of psoas major, descending
vertically in its posterior part to emerge from its medial border at the lateral margin of the
sacrum. It then crosses the sacroiliac joint to enter the lesser pelvis, it descends on obturator
internus to enter the obturator groove. In the lesser pelvis the nerve lies lateral to the
internal iliac vessels and ureter, and is joined by the obturator vessels lateral to the ovary or
ductus deferens.

17
18
19
20
21
22
23

Supplies

24

Medial compartment of thigh


Muscles supplied: external obturator, adductor longus, adductor brevis, adductor
magnus (not the lower part-sciatic nerve), gracilis
The cutaneous branch is often absent. When present, it passes between gracilis and
adductor longus near the middle part of the thigh, and supplies the skin and fascia of
the distal two thirds of the medial aspect.

25
26
27
28
29-31 3 / 3
32

Obturator canal

33

Connects the pelvis and thigh: contains the obturator artery, vein, nerve which divides
into anterior and posterior branches.

34
35
36

Cadaveric cross section demonstrating relationships of the obturator nerve

37
38
39
40
41
42
43
44
45
46-48 3 / 3
49
50

Image sourced from Wikipedia

51
52
53

Rate question:

Next question

54
Comment on this question

55
56
57
58-60 3 / 3
61

http://www.emrcs.com/question/question.php?q=0

1/2

17/05/2015
62
63
64
65
66
67
68
69
70
71
72
73
74
75
76
77

78
79

All contents of this site are 2012 E-Medical Revision Ltd

http://www.emrcs.com/question/question.php?q=0

Terms and Conditions

Privacy policy

2/2

17/05/2015

Reference ranges

Question 80 of 368

Previous

Next

You excitedly embark on your first laparoscopic cholecystectomy and during the operation
the anatomy of Calots triangle is more hostile than anticipated. Whilst trying to apply a
haemostatic clip you avulse the cystic artery. This is followed by brisk haemorrhage. From
which source is this most likely to originate ?

Question stats

Score: 77.1%
1

50.3%

10.1%

10.3%

7.9%

21.5%

50.3% of users answered this


question correctly

A. Right hepatic artery

End and review

B. Portal vein

6
7
8

C. Gastroduodenal artery

Search

D. Liver bed
E. Common hepatic artery

Go

10
11

Next question

12
13

The cystic artery is a branch of the right hepatic artery. There are recognised variations in
the anatomy of the blood supply to the gallbladder. However, the commonest situation is for
the cystic artery to branch from the right hepatic artery.

14
15
16

Gallbladder

17
18

Fibromuscular sac with capacity of 50ml


Columnar epithelium

19
20
21

Relations of the gallbladder


Anterior

Liver

22
23
24

Posterior

Covered by peritoneum
Transverse colon
1st part of the duodenum

25
26
27

Laterally

Right lobe of liver

Medially

Quadrate lobe of liver

28
29-31 3 / 3
32

Arterial supply
Cystic artery (branch of Right hepatic artery)

33
34
35

Venous drainage
Directly to the liver

36
37

Nerve supply
Sympathetic- mid thoracic spinal cord, Parasympathetic- anterior vagal trunk

38
39

Common bile duct

40
41

Origin

Confluence of cystic and common hepatic ducts

42
43

Relations at
origin

Medially - Hepatic artery


Posteriorly- Portal vein

44
45
46-48 3 / 3

Relations distally

Duodenum - anteriorly
Pancreas - medially and laterally
Right renal vein - posteriorly

49
50
51

Arterial supply

Branches of hepatic artery and retroduodenal branches of gastroduodenal


artery

52
53

Hepatobiliary triangle

54
55

Medially

Common hepatic duct

Inferiorly

Cystic duct

Superiorly

Inferior edge of liver

http://www.emrcs.com/question/question.php?q=0

56
57
58-60 3 / 3
61

1/2

17/05/2015

Contents

62

Cystic artery

63
64

Calots triangle has the cystic artery as the superior border.

65

Rate question:

66

Next question

67
Comment on this question

68
69
70
71
72
73
74
75
76
77

78
79
80

All contents of this site are 2012 E-Medical Revision Ltd

http://www.emrcs.com/question/question.php?q=0

Terms and Conditions

Privacy policy

2/2

17/05/2015

Reference ranges

Previous

Question 81 of 368

Next

A 43 year old man suffers a pelvic fracture which is complicated by an injury to the junction of
the membranous urethra to the bulbar urethra. In which of the following directions is the
extravasated urine most likely to pass?

Question stats

Score: 77.5%
1

18%

6.5%

8.9%

58.6%

7.9%

A. Posteriorly into extra peritoneal tissues


58.6% of users answered this
question correctly

B. Laterally into the buttocks

End and review

6
7

C. Into the abdomen


8

D. Anteriorly into the connective tissues surrounding the scrotum


9

Search

E. None of the above

Go

Next question

10
11
12

The superficial perineal pouch is a compartment bounded superficially by the superficial


perineal fascia, deep by the perineal membrane (inferior fascia of the urogenital diaphragm),
and laterally by the ischiopubic ramus. It contains the crura of the penis or clitoris, muscles,
viscera, blood vessels, nerves, the proximal part of the spongy urethra in males, and the
greater vestibular glands in females.
When urethral rupture occurs as in this case the urine will tend to pass anteriorly because
the fascial condensations will prevent lateral and posterior passage of the urine.

13
14
15
16
17
18

Urogenital triangle

19
20

The urogenital triangle is formed by the:

21

Ischiopubic inferior rami


Ischial tuberosities

22
23

A fascial sheet is attached to the sides, forming the inferior fascia of the urogenital
diaphragm.

24

It transmits the urethra in males and both the urethra and vagina in females. The
membranous urethra lies deep this structure and is surrounded by the external urethral
sphincter.

26

Superficial to the urogenital diaphragm lies the superficial perineal pouch. In males this
contains:

25

27
28
29-31 3 / 3
32
33

Bulb of penis
Crura of the penis
Superficial transverse perineal muscle
Posterior scrotal arteries
Posterior scrotal nerves

34
35
36
37

In females the internal pudendal artery branches to become the posterior labial arteries in
the superficial perineal pouch.

38
39
40

Rate question:
Comment on this question

Next question

41
42
43
44
45
46-48 3 / 3
49
50
51
52
53
54
55
56
57
58-60 3 / 3
61

http://www.emrcs.com/question/question.php?q=0

1/2

17/05/2015
62
63
64
65
66
67
68
69
70
71
72
73
74
75
76
77

78
79
80
81

All contents of this site are 2012 E-Medical Revision Ltd

http://www.emrcs.com/question/question.php?q=0

Terms and Conditions

Privacy policy

2/2

17/05/2015

Reference ranges

Previous

Question 82 of 368

Next

Question stats

End and review

Score: 77.7%
1

9.8%

12.2%

48%

A. Oculomotor nerve

17.8%

B. Abducens nerve

12.1%

C. Ophthalmic artery

48% of users answered this


question correctly

Which of the following does not pass through the superior orbital fissure?

D. Ophthalmic division of the trigeminal nerve


E. Ophthalmic veins

6
7
8
9

Search
Next question
Go

The ophthalmic artery, a branch of the internal carotid enters the orbit with the optic nerve in
the canal.

10
11
12
13

Foramina of the base of the skull

14
15

Foramen

Location

Contents

Foramen
ovale

Sphenoid
bone

Otic ganglion
V3 (Mandibular nerve:3rd branch of
trigeminal)
Accessory meningeal artery
Lesser petrosal nerve
Emissary veins

16
17
18
19
20
21

Foramen
spinosum

Sphenoid
bone

Middle meningeal artery


Meningeal branch of the Mandibular nerve

Foramen
rotundum

Sphenoid
bone

Maxillary nerve (V2)

Foramen
lacerum/
carotid canal

Sphenoid
bone

Base of the medial pterygoid plate.


Internal carotid artery*
Nerve and artery of the pterygoid canal

Jugular
foramen

Temporal
bone

22
23
24
25

Anterior: inferior petrosal sinus


Intermediate: glossopharyngeal, vagus, and accessory nerves.
Posterior: sigmoid sinus (becoming the internal jugular vein) and
some meningeal branches from the occipital and ascending
pharyngeal arteries.

26
27
28
29-31 3 / 3
32
33
34

Foramen
magnum

Occipital
bone

Anterior and posterior spinal arteries


Vertebral arteries
Medulla oblongata

Stylomastoid
foramen

Temporal
bone

Stylomastoid artery
Facial nerve

Superior
orbital fissure

Sphenoid
bone

Oculomotor nerve (III)


trochlear nerve (IV)
lacrimal, frontal and nasociliary branches of ophthalmic nerve (V1)
abducent nerve (VI)
Superior and inferior ophthalmic vein

35
36
37
38
39
40
41
42
43

*= In life the foramen lacerum is occluded by a cartilagenous plug. The ICA initially passes
into the carotid canal which ascends superomedially to enter the cranial cavity through the
foramen lacerum.

44
45
46-48 3 / 3

Base of skull anatomical overview

49
50
51
52
53
54
55
56
57
58-60 3 / 3
61
62

http://www.emrcs.com/question/question.php?q=0

1/2

17/05/2015
62

Image sourced from Wikipedia

63
64

Rate question:

65

Next question

66

Comment on this question

67
68
69
70
71
72
73
74
75
76
77

78
79
80
81
82

All contents of this site are 2012 E-Medical Revision Ltd

http://www.emrcs.com/question/question.php?q=0

Terms and Conditions

Privacy policy

2/2

17/05/2015

Reference ranges

Previous

Question 83 of 368

Next

Question stats

End and review

Score: 78.1%
1

6.8%

16.8%

50.4%

A. Popliteal nerve

14.7%

B. Superficial peroneal nerve

11.3%

C. Deep peroneal nerve

50.4% of users answered this


question correctly

Which nerve supplies the 1st web space of the foot?

D. Tibial nerve
E. Saphenous nerve

6
7
8
9

Search
Next question
Go

The first web space is innervated by the deep peroneal nerve. See diagram below:

10
11
12
13
14
15
16
17
18
19
20
21
22
23
24
25
26
27
28
29-31 3 / 3
32
33
34
35

Image sourced from Wikipedia

36
37
38

Deep peroneal nerve


39

Origin

From the common peroneal nerve, at the lateral aspect of the fibula, deep
to peroneus longus

40
41
42

Nerve root
values

L4, L5, S1, S2


43
44

Course and
relation

Pierces the anterior intermuscular septum to enter the anterior


compartment of the lower leg
Passes anteriorly down to the ankle joint, midway between the two
malleoli

45
46-48 3 / 3
49
50

Terminates
Muscles
innervated

In the dorsum of the foot

51
52

Tibialis anterior
Extensor hallucis longus
Extensor digitorum longus
Peroneus tertius
Extensor digitorum brevis

53
54
55
56

Cutaneous
innervation
Actions

Web space of the first and second toes

57
58-60 3 / 3
61

Dorsiflexion of ankle joint

http://www.emrcs.com/question/question.php?q=0

1/2

17/05/2015

Dorsiflexion of ankle joint


Extension of all toes (extensor hallucis longus and extensor
digitorum longus)
Eversion of the foot

62
63
64
65
66

After its bifurcation past the ankle joint, the lateral branch of the deep peroneal nerve
innervates the extensor digitorum brevis and the extensor hallucis brevis
The medial branch supplies the web space between the first and second digits.

67
68

Rate question:

69

Next question

70
Comment on this question

71
72
73
74
75
76
77

78
79
80
81
82
83

All contents of this site are 2012 E-Medical Revision Ltd

http://www.emrcs.com/question/question.php?q=0

Terms and Conditions

Privacy policy

2/2

17/05/2015

Reference ranges

Question 84 of 368

Previous

Next

During the course of a radical gastrectomy the surgeons detach the omentum and ligate the
right gastro-epiploic artery. From which vessel does it originate?

A. Superior mesenteric artery


B. Inferior mesenteric artery

Question stats

Score: 78.4%
1

11.3%

8.8%

18.4%

23%

38.5%

38.5% of users answered this


question correctly

C. Coeliac axis

End and review

D. Common hepatic artery

6
7
8

E. Gastroduodenal artery

Search
Next question

Go

10
11

Theme from January 2013 Exam


The gastroduodenal artery arises at the superior part of the duodenum and descends
behind it to terminate at its lower border. It terminates by dividing into the right gastroepiploic artery and the superior pancreaticoduodenal artery. The right gastro-opiploic artery
passes to the left and passes between the layers of the greater omentum to anastomose with
the left gastro-epiploic artery.

12
13
14
15
16
17

Gastroduodenal artery

18

Supplies
Pylorus, proximal part of the duodenum, and indirectly to the pancreatic head (via the
anterior and posterior superior pancreaticoduodenal arteries)

19

Path
Most commonly arises from the common hepatic artery of the coeliac trunk
Terminates by bifurcating into the right gastroepiploic artery and the superior
pancreaticoduodenal artery

22

20
21

23
24
25

Image showing stomach reflected superiorly to illustrate the relationship of the


gastroduodenal artery to the first part of the duodenum

26
27
28
29-31 3 / 3
32
33
34
35
36
37
38
39
40
41
42
43
44
45

Image sourced from Wikipedia

46-48 3 / 3
49
50

Rate question:
Comment on this question

Next question

51
52
53
54
55
56
57
58-60 3 / 3
61

http://www.emrcs.com/question/question.php?q=0

1/2

17/05/2015
62
63
64
65
66
67
68
69
70
71
72
73
74
75
76
77

78
79
80
81
82
83
84

All contents of this site are 2012 E-Medical Revision Ltd

http://www.emrcs.com/question/question.php?q=0

Terms and Conditions

Privacy policy

2/2

17/05/2015

Reference ranges

Previous

Question 85 of 368

Next

Question stats

End and review

Score: 78.6%
1

9.5%

61.1%

10.8%

A. Opponens pollicis

9.4%

B. Palmaris longus

9.2%

C. Flexor pollicis brevis

61.1% of users answered this


question correctly

Which of the following is not an intrinsic muscle of the hand?

D. Flexor digiti minimi brevis


E. Opponens digiti minimi

6
7
8
9

Search
Next question
Go

10
11

Mnemonic for intrinsic hand muscles


'A OF A OF A'

12

A bductor pollicis brevis


O pponens pollicis
F lexor pollicis brevis
A dductor pollicis (thenar muscles)
O pponens digiti minimi
F lexor digiti minimi brevis
A bductor digiti minimi (hypothenar muscles)

14

13

15
16
17
18
19
20

Palmaris longus originates in the forearm.

21
22

Hand

23

Anatomy of the hand


Bones

24

8 Carpal bones
5 Metacarpals
14 phalanges

25
26
27
28

Intrinsic Muscles

7 Interossei - Supplied by ulnar nerve


3 palmar-adduct fingers
4 dorsal- abduct fingers

29-31 3 / 3
32
33

Intrinsic muscles

Lumbricals
Flex MCPJ and extend the IPJ.
Origin deep flexor tendon and insertion dorsal extensor hood
mechanism.
Innervation: 1st and 2nd- median nerve, 3rd and 4th- deep branch of
the ulnar nerve.

34
35
36
37
38
39

Thenar eminence

Abductor pollicis brevis


Opponens pollicis
Flexor pollicis brevis

40
41
42

Hypothenar
eminence

43

Opponens digiti minimi


Flexor digiti minimi brevis
Abductor digiti minimi

44
45
46-48 3 / 3
49

http://www.emrcs.com/question/question.php?q=0

1/2

17/05/2015
49
50
51
52
53
54
55
56
57
58-60 3 / 3
61
62
63
64
65
66

Image sourced from Wikipedia

67

Fascia and compartments of the palm


The fascia of the palm is continuous with the antebrachial fascia and the fascia of the
dorsum of the hand. The palmar fascia is thin over the thenar and hypothenar eminences. In
contrast the palmar fascia is relatively thick. The palmar aponeurosis covers the soft tissues
and overlies the flexor tendons. The apex of the palmar aponeurosis is continuous with the
flexor retinaculum and the palmaris longus tendon. Distally, it forms four longitudinal digital
bands that attach to the bases of the proximal phalanges, blending with the fibrous digital
sheaths.
A medial fibrous septum extends deeply from the medial border of the palmar aponeurosis to
the 5th metacarpal. Lying medial to this are the hypothenar muscles. In a similar fashion, a
lateral fibrous septum extends deeply from the lateral border of the palmar aponeurosis to
the 3rd metacarpal. The thenar compartment lies lateral to this area.
Lying between the thenar and hypothenar compartments is the central compartment. It
contains the flexor tendons and their sheaths, the lumbricals, the superficial palmar arterial
arch and the digital vessels and nerves.
The deepest muscular plane is the adductor compartment, which contains adductor pollicis.

68
69
70
71
72
73
74
75
76
77

78
79
80

Short muscles of the hand


These comprise the lumbricals and interossei. The four slender lumbrical muscles flex the
fingers at the metacarpophalangeal joints and extend the interphalangeal joint. The four
dorsal interossei are located between the metacarpals and the four palmar interossei lie on
the palmar surface of the metacarpals in the interosseous compartment of the hand.

81
82
83
84
85

Long flexor tendons and sheaths in the hand


The tendons of FDS and FDP enter the common flexor sheath deep to the flexor
retinaculum. The tendons enter the central compartment of the hand and fan out to their
respective digital synovial sheaths. Near the base of the proximal phalanx, the tendon of FDS
splits to permit the passage of FDP. The FDP tendons are attached to the margins of the
anterior aspect of the base of the distal phalanx.
The fibrous digital sheaths contain the flexor tendons and their synovial sheaths. These
extend from the heads of the metacarpals to the base of the distal phalanges.
Rate question:

Next question

Comment on this question

All contents of this site are 2012 E-Medical Revision Ltd

http://www.emrcs.com/question/question.php?q=0

Terms and Conditions

Privacy policy

2/2

17/05/2015

Reference ranges

Previous

Question 86 of 368

Next

A man with lung cancer and bone metastasis in the thoracic spinal vertebral bodies, sustains
a pathological fracture at the level of T4. The fracture is unstable and the spinal cord is
severely compressed at this level. Which of the findings below will not be present 6 weeks
after injury?

Question stats

Score: 79%
1

14.9%

17.5%

34.7%

14.7%

18.3%

34.7% of users answered this


question correctly

A. Extensor plantar reflexes

End and review

B. Spasticity of the lower limbs

6
7
8

C. Diminished patellar tendon reflex

Search

D. Urinary incontinence
E. Sensory ataxia

Go

10
11

Next question

12
13

A thoracic cord lesion causes spastic paraperesis, hyperrflexia and extensor plantar
responses (UMN lesion), incontinence, sensory loss below the lesion and 'sensory'
ataxia.These features typically manifest several weeks later, once spinal shock (in which
areflexia predominates) has resolved.
Spinal cord

14
15
16
17
18
19

Located in a canal within the vertebral column that affords it structural support.
Rostrally it continues to the medulla oblongata of the brain and caudally it tapers at a
level corresponding to the L1-2 interspace (in the adult), a central structure, the filum
terminale anchors the cord to the first coccygeal vertebra.
The spinal cord is characterised by cervico-lumbar enlargements and these, broadly
speaking, are the sites which correspond to the brachial and lumbar plexuses
respectively.

20
21
22
23
24
25

There are some key points to note when considering the surgical anatomy of the spinal cord:

26
27

* During foetal growth the spinal cord becomes shorter than the spinal canal, hence the adult
site of cord termination at the L1-2 level.

28
29-31 3 / 3

* Due to growth of the vertebral column the spine segmental levels may not always
correspond to bony landmarks as they do in the cervical spine.

32

* The spinal cord is incompletely divided into two symmetrical halves by a dorsal median
sulcus and ventral median fissure. Grey matter surrounds a central canal that is
continuous rostrally with the ventricular system of the CNS.

34

* The grey matter is sub divided cytoarchitecturally into Rexeds laminae.

37

33

35
36

38

* Afferent fibres entering through the dorsal roots usually terminate near their point of entry
but may travel for varying distances in Lissauers tract. In this way they may establish
synaptic connections over several levels

39
40
41

* At the tip of the dorsal horn are afferents associated with nociceptive stimuli. The ventral
horn contains neurones that innervate skeletal muscle.

42
43

The key point to remember when revising CNS anatomy is to keep a clinical perspective in
mind. So it is worth classifying the ways in which the spinal cord may become injured. These
include:

44
45
46-48 3 / 3
49

Trauma either direct or as a result of disc protrusion


Neoplasia either by direct invasion (rare) or as a result of pathological vertebral
fracture
Inflammatory diseases such as Rheumatoid disease, or OA (formation of
osteophytes compressing nerve roots etc.
Vascular either as a result of stroke (rare in cord) or as complication of aortic
dissection
Infection historically diseases such as TB, epidural abscesses.

50
51
52
53
54
55
56

The anatomy of the cord will, to an extent dictate the clinical presentation. Some points/
conditions to remember:

57
58-60 3 / 3
61

http://www.emrcs.com/question/question.php?q=0

1/2

17/05/2015

Brown- Sequard syndrome-Hemisection of the cord producing ipsilateral loss of


proprioception and upper motor neurone signs, plus contralateral loss of pain and
temperature sensation. The explanation of this is that the fibres decussate at different
levels.
Lesions below L1 will tend to present with lower motor neurone signs

62
63
64
65
66
67

Rate question:

68

Next question

69

Comment on this question

70
71
72
73
74
75
76
77

78
79
80
81
82
83
84
85
86

All contents of this site are 2012 E-Medical Revision Ltd

http://www.emrcs.com/question/question.php?q=0

Terms and Conditions

Privacy policy

2/2

17/05/2015

Reference ranges

Previous

Question 87 of 368

Next

Through which of the following foramina does the genital branch of the genitofemoral nerve
exit the abdominal cavity?

A. Superficial inguinal ring


B. Sciatic notch

Question stats

Score: 78.3%
1

25.1%

9.7%

17.8%

11.9%

35.5%

35.5% of users answered this


question correctly

C. Obturator foramen

End and review

D. Femoral canal

6
7
8

E. Deep inguinal ring

Search
Next question

Go

10
11

The genitofemoral nerve divides into two branches as it approaches the inguinal ligament.
The genital branch passes anterior to the external iliac artery through the deep inguinal ring
into the inguinal canal. It communicates with the ilioinguinal nerve in the inguinal canal
(though this is seldom of clinical significance).

12
13
14
15

Genitofemoral nerve

16
17

Supplies
Small area of the upper medial thigh.

18
19

Path

20

Arises from the first and second lumbar nerves.


Passes obliquely through psoas major, and emerges from its medial border opposite
the fibrocartilage between the third and fourth lumbar vertebrae.
It then descends on the surface of psoas major, under cover of the peritoneum
Divides into genital and femoral branches.
The genital branch passes through the inguinal canal, within the spermatic cord, to
supply the skin overlying the skin and fascia of the scrotum. The femoral branch
enters the thigh posterior to the inguinal ligament, lateral to the femoral artery. It
supplies an area of skin and fascia over the femoral triangle.
It may be injured during abdominal or pelvic surgery, or during inguinal hernia repairs.

21
22
23
24
25
26
27
28
29-31 3 / 3
32

Rate question:

Next question

33
34

Comment on this question

35
36
37
38
39
40
41
42
43
44
45
46-48 3 / 3
49
50
51
52
53
54
55
56
57
58-60 3 / 3
61

http://www.emrcs.com/question/question.php?q=0

1/2

17/05/2015
62
63
64
65
66
67
68
69
70
71
72
73
74
75
76
77

78
79
80
81
82
83
84
85
86
87

All contents of this site are 2012 E-Medical Revision Ltd

http://www.emrcs.com/question/question.php?q=0

Terms and Conditions

Privacy policy

2/2

17/05/2015

Reference ranges

Previous

Question 88 of 368

Next

A 28 year old man lacerates the posterolateral aspect of his wrist with a knife in an attempted
suicide. On arrival in the emergency department the wound is inspected and found to be
located over the lateral aspect of the extensor retinaculum (which is intact). Which of the
following structures is at greatest risk of injury?

A. Superficial branch of the radial nerve

Question stats

End and review

Score: 77%
1

45.2%

18.5%

14.6%

12.8%

9%

45.2% of users answered this


question correctly

B. Radial artery

7
8

C. Dorsal branch of the ulnar nerve

Search

D. Tendon of extensor carpi radialis brevis


E. Tendon of extensor digiti minimi

Go

10
11

Next question

12
13

The superficial branch of the radial nerve passes superior to the extensor retinaculum in the
position of this laceration and is at greatest risk of injury. The dorsal branch of the ulnar
nerve and artery also pass superior to the extensor retinaculum n but are located medially.

14
15
16

Extensor retinaculum

17

The extensor rentinaculum is a thickening of the deep fascia that stretches across the back
of the wrist and holds the long extensor tendons in position.
Its attachments are:

18
19
20
21

The pisiform and hook of hamate medially


The end of the radius laterally

22
23

Structures related to the extensor retinaculum


Structures superficial to the
retinaculum

Basilic vein
Dorsal cutaneous branch of the ulnar nerve
Cephalic vein
Superficial branch of the radial nerve

24
25
26
27
28

Structures passing deep to the


extensor retinaculum

29-31 3 / 3

Extensor carpi ulnaris tendon


Extensor digiti minimi tendon
Extensor digitorum and extensor indicis
tendon
Extensor pollicis longus tendon
Extensor carpi radialis longus tendon
Abductor pollicis longus and extensor
pollicis brevis tendons

32
33
34
35
36
37
38

Beneath the extensor retinaculum fibrous septa form six compartments that contain the
extensor muscle tendons. Each compartment has its own synovial sheath.

39
40

The radial artery


The radial artery passes between the lateral collateral ligament of the wrist joint and the
tendons of the abductor pollicis longus and extensor pollicis brevis.

41
42
43

Image illustrating the topography of tendons passing under the extensor retinaculum

http://www.emrcs.com/question/question.php?q=0

44

1/2

17/05/2015

45
46-48 3 / 3
49
50
51
52
53
54
55
56
57
58-60 3 / 3
61
62
63
64
65

Image sourced from Wikipedia

66
67

Rate question:

68

Next question

69

Comment on this question

70
71
72
73
74
75
76
77

78
79
80
81
82
83
84
85
86
87
88

All contents of this site are 2012 E-Medical Revision Ltd

http://www.emrcs.com/question/question.php?q=0

Terms and Conditions

Privacy policy

2/2

17/05/2015

Reference ranges

Previous

Question 89 of 368

Next

A 43 year old man is reviewed in the clinic following a cardiac operation. A chest x-ray is
performed and a circular radio-opaque structure is noted medial to the 4th interspace on the
left. Which of the following procedures is the patient most likely to have undergone?

Question stats

Score: 77.4%
1

15%

34.9%

8.7%

7.4%

34%

A. Aortic valve replacement with metallic valve


34% of users answered this
question correctly

B. Tricuspid valve replacement with metallic valve

End and review

6
7

C. Tricuspid valve replacement with porcine valve


8

D. Pulmonary valve replacement with porcine valve


9

Search

E. Mitral valve replacement with metallic valve

Go

Next question

10
11
12

Theme from April 2012 Exam

13
14

Prosthetic heart valves on Chest X-rays

15

The aortic and mitral valves are most commonly replaced and when a metallic valve is used,
can be most readily identified on plain x-rays.
The presence of cardiac disease (such as cardiomegaly) may affect the figures quoted here.

16

Aortic
Usually located medial to the 3rd interspace on the right.

19

17
18

20
21

Mitral
Usually located medial to the 4th interspace on the left.

22
23

Tricuspid
Usually located medial to the 5th interspace on the right.

24
25

Please note that these are the sites at which an artificial valve may be located and are NOT
the sites of auscultation.
Rate question:
Comment on this question

26
27
28

Next question

29-31 3 / 3
32
33
34
35
36
37
38
39
40
41
42
43
44
45
46-48 3 / 3
49
50
51
52
53
54
55
56
57
58-60 3 / 3
61

http://www.emrcs.com/question/question.php?q=0

1/2

17/05/2015
62
63
64
65
66
67
68
69
70
71
72
73
74
75
76
77

78
79
80
81
82
83
84
85
86
87
88
89

All contents of this site are 2012 E-Medical Revision Ltd

http://www.emrcs.com/question/question.php?q=0

Terms and Conditions

Privacy policy

2/2

17/05/2015

Reference ranges

Previous

Question 90 of 368

Next

A 63 year old lady is diagnosed as having an endometrial carcinoma arising from the uterine
body. To which nodal region will the tumour initially metastasise?

A. Para aortic nodes


B. Iliac lymph nodes
C. Inguinal nodes

Question stats

End and review

Score: 77.8%
1

28.1%

41.1%

14.7%

8.1%

8%

41.1% of users answered this


question correctly

D. Pre sacral nodes

7
8

E. Mesorectal lymph nodes

Search
Next question

Go

10
11

Theme from 2011 exam


Tumours of the uterine body will tend to spread to the iliac nodes initially. Tumour expansion
crossing different nodal margins this is of considerable clinical significance, if nodal
clearance is performed during a Wertheims type hysterectomy.

12
13
14
15

Lymphatic drainage of the uterus and cervix

16
17

The uterine fundus has a lymphatic drainage that runs with the ovarian vessels and
may thus drain to the para-aortic nodes. Some drainage may also pass along the
round ligament to the inguinal nodes.
The body of the uterus drains through lymphatics contained within the broad ligament
to the iliac lymph nodes.
The cervix drains into three potential nodal stations; laterally through the broad
ligament to the external iliac nodes, along the lymphatics of the uterosacral fold to the
presacral nodes and posterolaterally along lymphatics lying alongside the uterine
vessels to the internal iliac nodes.

18
19
20
21
22
23
24
25
26

Rate question:

Next question

27
28

Comment on this question

29-31 3 / 3
32
33
34
35
36
37
38
39
40
41
42
43
44
45
46-48 3 / 3
49
50
51
52
53
54
55
56
57
58-60 3 / 3
61

http://www.emrcs.com/question/question.php?q=0

1/2

17/05/2015
62
63
64
65
66
67
68
69
70
71
72
73
74
75
76
77

78
79
80
81
82
83
84
85
86
87
88
89
90

All contents of this site are 2012 E-Medical Revision Ltd

http://www.emrcs.com/question/question.php?q=0

Terms and Conditions

Privacy policy

2/2

17/05/2015

Reference ranges

Previous

Question 91 of 368

Next

Question stats

End and review

Score: 76.6%
1

13.3%

28.7%

15.1%

A. Loss of elbow extension.

17.7%

B. Loss of extension of the interphalangeal joints.

25.2%

C. Loss of metacarpophalangeal extension.

28.7% of users answered this


question correctly

Transection of the radial nerve at the level of the axilla will result in all of the following except:

D. Loss of triceps reflex.


E. Loss of sensation overlying the first dorsal interosseous.

6
7
8
9

Search
Next question
Go

These may still extend by virtue of retained lumbrical muscle function.

10
11
12

Radial nerve

13
14

Continuation of posterior cord of the brachial plexus (root values C5 to T1)

15
16

Path
In the axilla: lies posterior to the axillary artery on subscapularis, latissimus dorsi and
teres major.
Enters the arm between the brachial artery and the long head of triceps (medial to
humerus).
Spirals around the posterior surface of the humerus in the groove for the radial nerve.
At the distal third of the lateral border of the humerus it then pierces the intermuscular
septum and descends in front of the lateral epicondyle.
At the lateral epicondyle it lies deeply between brachialis and brachioradialis where it
then divides into a superficial and deep terminal branch.
Deep branch crosses the supinator to become the posterior interosseous nerve.

17
18
19
20
21
22
23
24
25

In the image below the relationships of the radial nerve can be appreciated

26
27
28
29-31 3 / 3
32
33
34
35
36
37
38
39
40
41

Image sourced from Wikipedia

42
43

Regions innervated
Motor (main
nerve)

44
Triceps
Anconeus
Brachioradialis
Extensor carpi radialis

45
46-48 3 / 3
49
50

Motor
(posterior
interosseous
branch)

Supinator
Extensor carpi ulnaris
Extensor digitorum
Extensor indicis
Extensor digiti minimi
Extensor pollicis longus and brevis
Abductor pollicis longus

51
52
53
54
55

Sensory

The area of skin supplying the proximal phalanges on the dorsal aspect of
the hand is supplied by the radial nerve (this does not apply to the little
finger and part of the ring finger)

Muscular innervation and effect of denervation

56
57
58-60 3 / 3
61

Anatomical

Muscle affected

http://www.emrcs.com/question/question.php?q=0

Effect of paralysis

62

1/2

17/05/2015
62

location

63

Shoulder

Long head of triceps

Minor effects on shoulder stability in


abduction

Arm

Triceps

Loss of elbow extension

Forearm

Supinator
Brachioradialis
Extensor carpi radialis
longus and brevis

Weakening of supination of prone hand and


elbow flexion in mid prone position

64
65
66
67
68
69
70

The cutaneous sensation of the upper limb- illustrating the contribution of the radial nerve

71
72
73
74
75
76
77

78
79
80
81
82
83
84
85
86

Image sourced from Wikipedia

87
88
89

Rate question:

Next question

90
91

Comment on this question

All contents of this site are 2012 E-Medical Revision Ltd

http://www.emrcs.com/question/question.php?q=0

Terms and Conditions

Privacy policy

2/2

17/05/2015

Reference ranges

Previous

Question 92 of 368

Next

Question stats

End and review

Score: 76%
1

16.5%

34.5%

11.6%

A. Posterior labial arteries

21.3%

B. Pudendal nerve

16%

C. Superficial transverse perineal muscle

34.5% of users answered this


question correctly

Which of the following structures is not located in the superficial perineal space in females?

D. Greater vestibular glands


E. None of the above

6
7
8
9

Search
Next question
Go

The pudendal nerve is located in the deep perineal space and then branches to innervate
more superficial structures.

10
11
12
13

Urogenital triangle

14
15

The urogenital triangle is formed by the:

16

Ischiopubic inferior rami


Ischial tuberosities

17
18

A fascial sheet is attached to the sides, forming the inferior fascia of the urogenital
diaphragm.

19
20

It transmits the urethra in males and both the urethra and vagina in females. The
membranous urethra lies deep this structure and is surrounded by the external urethral
sphincter.

21
22
23

Superficial to the urogenital diaphragm lies the superficial perineal pouch. In males this
contains:
Bulb of penis
Crura of the penis
Superficial transverse perineal muscle
Posterior scrotal arteries
Posterior scrotal nerves

24
25
26
27
28
29-31 3 / 3
32

In females the internal pudendal artery branches to become the posterior labial arteries in
the superficial perineal pouch.

33
34
35

Rate question:
Comment on this question

Next question

36
37
38
39
40
41
42
43
44
45
46-48 3 / 3
49
50
51
52
53
54
55
56
57
58-60 3 / 3
61

http://www.emrcs.com/question/question.php?q=0

1/2

17/05/2015
62
63
64
65
66
67
68
69
70
71
72
73
74
75
76
77

78
79
80
81
82
83
84
85
86
87
88
89
90
91
92

All contents of this site are 2012 E-Medical Revision Ltd

http://www.emrcs.com/question/question.php?q=0

Terms and Conditions

Privacy policy

2/2

17/05/2015

Reference ranges

Previous

Question 94 of 368

Next

Which of the following structures does not pass behind the piriformis muscle in the greater
sciatic foramen?

A. Sciatic nerve
B. Posterior cutaneous nerve of the thigh

Question stats

Score: 75.2%
1

13.7%

17%

15.1%

43.5%

10.6%

43.5% of users answered this


question correctly

C. Inferior gluteal artery

End and review

D. Obturator nerve

6
7
8

E. None of the above

Search
Next question

Go

10
11

The obturator nerve does not pass through the greater sciatic foramen.

12
13

Greater sciatic foramen

15

Contents
Nerves

14

16

Sciatic Nerve
Superior and Inferior Gluteal Nerves
Pudendal Nerve
Posterior Femoral Cutaneous Nerve
Nerve to Quadratus Femoris
Nerve to Obturator internus

17
18
19
20
21

Vessels

Superior Gluteal Artery and vein


Inferior Gluteal Artery and vein
Internal Pudendal Artery and vein

22
23
24
25

Piriformis
The piriformis is a landmark for identifying structures passing out of the sciatic notch
Above piriformis: Superior gluteal vessels
Below piriformis: Inferior gluteal vessels, sciatic nerve (10% pass through it, <1%
above it), posterior cutaneous nerve of the thigh

26
27
28
29-31 3 / 3
32
33

Greater sciatic foramen boundaries


Anterolaterally

Greater sciatic notch of the ilium

Posteromedially

Sacrotuberous ligament

Inferior

Sacrospinous ligament and the ischial spine

Superior

Anterior sacroiliac ligament

34
35
36
37
38
39
40

The greater sciatic foramen

http://www.emrcs.com/question/question.php?q=0

41

1/2

17/05/2015
41
42
43
44
45
46-48 3 / 3
49
50
51
52
53
54
55
56
57
58-60 3 / 3
61
62
63

Image sourced from Wikipedia

64

Structures passing between both foramina (Medial to lateral)

65
66

Pudendal nerve
Internal pudendal artery
Nerve to obturator internus

67
68
69

Contents of the lesser sciatic foramen

70

Tendon of the obturator internus


Pudendal nerve
Internal pudendal artery and vein
Nerve to the obturator internus

71
72
73
74
75

Rate question:

76

Next question

77
Comment on this question

78
79
80
81
82
83
84
85
86
87
88
89
90
91
92
93
94

All contents of this site are 2012 E-Medical Revision Ltd

http://www.emrcs.com/question/question.php?q=0

Terms and Conditions

Privacy policy

2/2

17/05/2015

Reference ranges

Question 95 of 368

Previous

Next

A 56 year old man is undergoing a nephrectomy. The surgeons divide the renal artery. At
what level do these usually branch off the abdominal aorta?

A. T9
B. L2

Question stats

Score: 75.6%
1

8.2%

57.6%

14.2%

10.3%

9.7%

57.6% of users answered this


question correctly

C. L3

End and review

D. T10

6
7
8

E. L4

Search
Next question

Go

10
11
12

The renal arteries usually branch off the aorta on a level with L2.

13

Renal arteries

14
15

The right renal artery is longer than the left renal artery
The renal vein/artery/pelvis enter the kidney at the hilum

16
17
18

Relations

19

Right

Anterior- IVC, right renal vein, the head of the pancreas, and the descending part of the
duodenum

Left

Anterior- left renal vein, the tail of the pancreas

20
21
22
23

Branches

24

The renal arteries are direct branches off the aorta (upper border of L2)
In 30% there may be accessory arteries (mainly left side). Instead of entering the
kidney at the hilum, they usually pierce the upper or lower part of the organ.
Before reaching the hilum of the kidney, each artery divides into four or five segmental
branches (renal vein anterior and ureter posterior); which then divide within the sinus
into lobar arteries supplying each pyramid and cortex.
Each vessel gives off some small inferior suprarenal branches to the suprarenal gland,
the ureter, and the surrounding cellular tissue and muscles.

25
26
27
28
29-31 3 / 3
32
33
34

Rate question:

Next question

35
36

Comment on this question

37
38
39
40
41
42
43
44
45
46-48 3 / 3
49
50
51
52
53
54
55
56
57
58-60 3 / 3
61

http://www.emrcs.com/question/question.php?q=0

1/2

17/05/2015
62
63
64
65
66
67
68
69
70
71
72
73
74
75
76
77

78
79
80
81
82
83
84
85
86
87
88
89
90
91
92
93
94
95

All contents of this site are 2012 E-Medical Revision Ltd

http://www.emrcs.com/question/question.php?q=0

Terms and Conditions

Privacy policy

2/2

17/05/2015

Reference ranges

Question 96 of 368

Previous

Next

A 23 year old man is shot in the chest during a robbery. The left lung is lacerated and is
bleeding. An emergency thoracotomy is performed. The surgeons place a clamp over the
hilum of the left lung. Which of the following structures lies most anteriorly at this level?

Question stats

Score: 75.9%
1

14.4%

8.6%

11.8%

44.6%

20.7%

A. Vagus nerve
44.6% of users answered this
question correctly

B. Oesophagus

End and review

6
7

C. Descending aorta
8

D. Phrenic nerve
9

Search

E. Azygos vein

Go

Next question

10
11
12

The phrenic nerve lies anteriorly at this point. The vagus passes anteriorly and then arches
backwards immediately superior to the root of the left bronchus, giving off the recurrent
laryngeal nerve as it does so.

13
14
15

Lung anatomy

16
17

The right lung is composed of 3 lobes divided by the oblique and transverse fissures. The
left lung has two lobes divided by the oblique fissure.The apex of both lungs is approximately
4cm superior to the sterno-costal joint of the first rib. Immediately below this is a sulcus
created by the subclavian artery.
Peripheral contact points of the lung

18
19
20
21
22

Base: diaphragm
Costal surface: corresponds to the cavity of the chest
Mediastinal surface: Contacts the mediastinal pleura. Has the cardiac impression.
Above and behind this concavity is a triangular depression named the hilum, where
the structures which form the root of the lung enter and leave the viscus. These
structures are invested by pleura, which, below the hilum and behind the pericardial
impression, forms the pulmonary ligament

23
24
25
26
27
28

Right lung
Above the hilum is the azygos vein; Superior to this is the groove for the superior vena cava
and right innominate vein; behind this, and nearer the apex, is a furrow for the innominate
artery. Behind the hilum and the attachment of the pulmonary ligament is a vertical groove
for the oesophagus; In front and to the right of the lower part of the oesophageal groove is a
deep concavity for the extrapericardiac portion of the inferior vena cava.

29-31 3 / 3
32
33
34
35
36

The root of the right lung lies behind the superior vena cava and the right atrium, and below
the azygos vein.

37
38

The right main bronchus is shorter, wider and more vertical than the left main bronchus and
therefore the route taken by most foreign bodies.

39
40
41
42
43
44
45
46-48 3 / 3
49
50
51

Image sourced from Wikipedia

Left lung
Above the hilum is the furrow produced by the aortic arch, and then superiorly the groove
accommodating the left subclavian artery; Behind the hilum and pulmonary ligament is a
vertical groove produced by the descending aorta, and in front of this, near the base of the
lung, is the lower part of the oesophagus.

52
53
54
55
56
57

The root of the left lung passes under the aortic arch and in front of the descending aorta.

http://www.emrcs.com/question/question.php?q=0

58-60 3 / 3

1/2

17/05/2015

61
62
63
64
65
66
67
68
69

Image sourced from Wikipedia

70

Inferior borders of both lungs

71

6th rib in mid clavicular line


8th rib in mid axillary line
10th rib posteriorly

72
73
74

The pleura runs two ribs lower than the corresponding lung level.

75

Bronchopulmonary segments

76
77

Segment number

Right lung

Left lung

Apical

Apical

Posterior

Posterior

Anterior

Anterior

82

Lateral

Superior lingular

83

Medial

Inferior lingular

Superior (apical)

Superior (apical)

Medial basal

Medial basal

Anterior basal

Anterior basal

89

Lateral basal

Lateral basal

90

10

Posterior basal

Posterior basal

78
79
80
81

84
85
86
87
88

91
92
93
94

Rate question:

Next question

95
Comment on this question

96

All contents of this site are 2012 E-Medical Revision Ltd

http://www.emrcs.com/question/question.php?q=0

Terms and Conditions

Privacy policy

2/2

17/05/2015

Reference ranges

Question 1 of 273

Next

An 18 year old man is stabbed in the axilla during a fight. His axillary artery is lacerated and
repaired. However, the surgeon neglects to repair an associated injury to the upper trunk of
the brachial plexus. Which of the following muscles is least likely to demonstrate impaired
function as a result?

Question stats

End and review

Score: 100%

40.5%

20.8%

16.4%

12.3%

10%

40.5% of users answered this


question correctly

A. Palmar interossei
B. Infraspinatus
C. Brachialis

Search

D. Supinator brevis
E. None of the above

Go

Next question

The palmar interossei are supplied by the ulnar nerve. Which lies inferiorly and is therefore
less likely to be injured.
Brachial plexus

Origin

Anterior rami of C5 to T1

Sections of the
plexus

Roots

Trunks

Roots, trunks, divisions, cords, branches


Mnemonic:Real Teenagers Drink Cold Beer

Located in the posterior triangle


Pass between scalenus anterior and medius

Located posterior to middle third of clavicle


Upper and middle trunks related superiorly to the subclavian
artery
Lower trunk passes over 1st rib posterior to the subclavian
artery

Divisions

Apex of axilla

Cords

Related to axillary artery

Diagram illustrating the branches of the brachial plexus

Image sourced from Wikipedia

Cutaneous sensation of the upper limb

http://www.emrcs.com/question/question.php?q=0

1/2

17/05/2015

Image sourced from Wikipedia

Rate question:

Next question

Comment on this question

All contents of this site are 2012 E-Medical Revision Ltd

http://www.emrcs.com/question/question.php?q=0

Terms and Conditions

Privacy policy

2/2

17/05/2015

Reference ranges

Previous

Question 2 of 273

Next

A 22 year old man presents with appendicitis. At operation the appendix is retrocaecal and
difficult to access. Division of which of the following anatomical structures should be
undertaken?

Question stats

End and review

Score: 50%

12.6%

21.8%

7.6%

49.9%

8.1%

1
2

A. Ileocolic artery
49.9% of users answered this
question correctly

B. Mesentery of the caecum


C. Gonadal vessels
D. Lateral peritoneal attachments of the caecum

Search

E. Right colic artery

Go

Next question

The commonest appendiceal location is retrocaecal. Those struggling to find it at operation


should trace the tenia to the caecal pole where the appendix is located. If it cannot be
mobilised easily then division of the lateral caecal peritoneal attachments (as for a right
hemicolectomy) will allow caecal mobilisation and facilitate the procedure.
Appendix

Location: Base of caecum.


Up to 10cm long.
Mainly lymphoid tissue (Hence mesenteric adenitis may mimic appendicitis).
Caecal taenia coli converge at base of appendix and form a longitudinal muscle cover
over the appendix. This convergence should facilitate its identification at surgery if it is
retrocaecal and difficult to find (which it can be when people start doing
appendicectomies!)
Arterial supply: Appendicular artery (branch of the ileocolic).
It is intra peritoneal.

McBurney's point
1/3 of the way along a line drawn from the Anterior Superior Iliac Spine to the
Umbilicus

6 Positions:

Retrocaecal 74%
Pelvic 21%
Postileal
Subcaecal
Paracaecal
Preileal

Rate question:

Next question

Comment on this question

All contents of this site are 2012 E-Medical Revision Ltd

http://www.emrcs.com/question/question.php?q=0

Terms and Conditions

Privacy policy

1/1

17/05/2015

Reference ranges

Previous

Question 3 of 273

Next

Question stats

End and review

Score: 40%
1

7.2%

8.1%

15.2%

A. Teres major

57.9%

B. Pectoralis major

11.6%

C. Coracobrachialis

57.9% of users answered this


question correctly

Which of the following muscles does not adduct the shoulder?

D. Supraspinatus
E. Latissimus dorsi

Search
Next question
Go

Supraspinatus is an abductor of the shoulder.


Shoulder joint

Shallow synovial ball and socket type of joint.


It is an inherently unstable joint, but is capable to a wide range of movement.
Stability is provided by muscles of the rotator cuff that pass from the scapula to insert
in the greater tuberosity (all except sub scapularis-lesser tuberosity).

Glenoid labrum
Fibrocartilaginous rim attached to the free edge of the glenoid cavity
Tendon of the long head of biceps arises from within the joint from the supraglenoid
tubercle, and is fused at this point to the labrum.
The long head of triceps attaches to the infraglenoid tubercle

Fibrous capsule
Attaches to the scapula external to the glenoid labrum and to the labrum itself
(postero-superiorly)
Attaches to the humerus at the level of the anatomical neck superiorly and the surgical
neck inferiorly
Anteriorly the capsule is in contact with the tendon of subscapularis, superiorly with the
supraspinatus tendon, and posteriorly with the tendons of infraspinatus and teres
minor. All these blend with the capsule towards their insertion.
Two defects in the fibrous capsule; superiorly for the tendon of biceps. Anteriorly there
is a defect beneath the subscapularis tendon.
The inferior extension of the capsule is closely related to the axillary nerve at the
surgical neck and this nerve is at risk in anteroinferior dislocations. It also means that
proximally sited osteomyelitis may progress to septic arthritis.

Movements and muscles


Flexion

Anterior part of deltoid


Pectoralis major
Biceps
Coracobrachialis

Extension

Posterior deltoid
Teres major
Latissimus dorsi

Adduction

Pectoralis major
Latissimus dorsi
Teres major
Coracobrachialis

Abduction

Mid deltoid
Supraspinatus

Medial rotation

Subscapularis
Anterior deltoid
Teres major
Latissimus dorsi

Lateral rotation

Posterior deltoid
Infraspinatus
Teres minor

Important anatomical relations


http://www.emrcs.com/question/question.php?q=0

1/2

17/05/2015

Anteriorly

Brachial plexus
Axillary artery and vein

Posterior

Suprascapular nerve
Suprascapular vessels

Inferior

Axillary nerve
Circumflex humeral vessels

Rate question:

Next question

Comment on this question

All contents of this site are 2012 E-Medical Revision Ltd

http://www.emrcs.com/question/question.php?q=0

Terms and Conditions

Privacy policy

2/2

17/05/2015

Reference ranges

Previous

Question 4 of 273

Next

Question stats

End and review

Score: 57.1%
1

13.3%

11%

51.7%

A. Masseter

12.9%

B. Sternocleidomastoid

11%

C. Platysma

51.7% of users answered this


question correctly

Which of these muscles is innervated by the cervical branch of the facial nerve?

D. Geniohyoid
E. Sternothyroid

Search
Next question
Go

The cervical branch of the facial nerve innervates platysma.


Facial nerve
The facial nerve is the main nerve supplying the structures of the second embryonic
branchial arch. It is predominantly an efferent nerve to the muscles of facial expression,
digastric muscle and also to many glandular structures. It contains a few afferent fibres which
originate in the cells of its genicular ganglion and are concerned with taste.
Supply - 'face, ear, taste, tear'
Face: muscles of facial expression
Ear: nerve to stapedius
Taste: supplies anterior two-thirds of tongue
Tear: parasympathetic fibres to lacrimal glands, also salivary glands

Path
Subarachnoid path
Origin: motor- pons, sensory- nervus intermedius
Pass through the petrous temporal bone into the internal auditory meatus with the
vestibulocochlear nerve. Here they combine to become the facial nerve.

Facial canal path


The canal passes superior to the vestibule of the inner ear
At the medial aspect of the middle ear, it becomes wider and contains the geniculate
ganglion.
- 3 branches:
1. greater petrosal nerve
2. nerve to stapedius
3. chorda tympani
Stylomastoid foramen
Passes through the stylomastoid foramen (tympanic cavity anterior and mastoid
antrum posteriorly)
Posterior auricular nerve and branch to posterior belly of digastric and stylohyoid
muscle

Face
Enters parotid gland and divides into 5 branches:
Temporal branch
Zygomatic branch
Buccal branch
Marginal mandibular branch
Cervical branch

Rate question:

Next question

Comment on this question

All contents of this site are 2012 E-Medical Revision Ltd

http://www.emrcs.com/question/question.php?q=0

Terms and Conditions

Privacy policy

1/1

17/05/2015

Reference ranges

Previous

Question 5 of 273

Next

During a thyroidectomy the surgeons ligate the inferior thyroid artery. From which vessel
does this structure usually originate?

A. External carotid artery


B. Thyrocervical trunk

Question stats

End and review

Score: 44.4%
1

12.9%

53.6%

9.8%

16.5%

7.3%

53.6% of users answered this


question correctly

C. Internal carotid artery


D. Subclavian artery
E. Vertebral artery

Search
Go

Next question

The inferior thyroid artery originates from the thyrocervical trunk. This is a branch of the
subclavian artery.
Thyroid gland

Right and left lobes connected by isthmus


Surrounded by sheath from pretracheal layer of deep fascia
Apex: Lamina of thyroid cartilage
Base: 4th-5th tracheal ring
Pyramidal lobe: from isthmus
May be attached to foramen caecum at the base of the tongue

Relations
Anteromedially

Posterolaterally

Sternothyroid
Superior belly of omohyoid
Sternohyoid
Anterior aspect of sternocleidomastoid
Carotid sheath

Medially

Larynx
Trachea
Pharynx
Oesophagus
Cricothyroid muscle
External laryngeal nerve (near superior thyroid artery)
Recurrent laryngeal nerve (near inferior thyroid artery)

Posterior

Parathyroid glands
Anastomosis of superior and inferior thyroid arteries

Isthmus

Anteriorly: Sternothyroids, sternohyoids, anterior jugular veins


Posteriorly: 2nd, 3rd, 4th tracheal rings (attached via Ligament of
Berry)

Blood Supply
Arterial

Venous

Superior thyroid artery (1st branch of external carotid)


Inferior thyroid artery (from thyrocervical trunk)
Thyroidea ima (in 10% of population -from brachiocephalic artery or aorta)

Superior and middle thyroid veins - into the IJV


Inferior thyroid vein - into the brachiocephalic veins

Rate question:

Next question

Comment on this question

All contents of this site are 2012 E-Medical Revision Ltd

http://www.emrcs.com/question/question.php?q=0

Terms and Conditions

Privacy policy

1/2

17/05/2015

http://www.emrcs.com/question/question.php?q=0

2/2

17/05/2015

Reference ranges

Previous

Question 6 of 273

Next

A 56 year old man is left impotent following an abdomino-perineal excision of the colon and
rectum. What is the most likely explanation?

A. Psychosexual issues related to an end colostomy


B. Damage to the sacral venous plexus during total mesorectal excision

Question stats

Score: 54.5%
1

11.8%

26.9%

6.9%

44%

10.5%

44% of users answered this


question correctly

C. Damage to the left ureter during sigmoid mobilisation

End and review

D. Damage to the hypogastric plexus during mobilisation of the inferior


mesenteric artery
Search

E. Damage to the internal iliac artery during total mesorectal excision

Go

Next question

Autonomic nerve injury is the most common cause.


Nerve lesions during surgery
A variety of different procedures carry the risk of iatrogenic nerve injury. These are important
not only from the patients perspective but also from a medicolegal standpoint.
The following operations and their associated nerve lesions are listed here:
Posterior triangle lymph node biopsy and accessory nerve lesion.
Lloyd Davies stirrups and common peroneal nerve.
Thyroidectomy and laryngeal nerve.
Anterior resection of rectum and hypogastric autonomic nerves.
Axillary node clearance; long thoracic nerve, thoracodorsal nerve and
intercostobrachial nerve.
Inguinal hernia surgery and ilioinguinal nerve.
Varicose vein surgery- sural and saphenous nerves.
Posterior approach to the hip and sciatic nerve.
Carotid endarterectomy and hypoglossal nerve.

There are many more, with sound anatomical understanding of the commonly performed
procedures the incidence of nerve lesions can be minimised. They commonly occur when
surgeons operate in an unfamiliar tissue plane or by blind placement of haemostats (not
recommended).
Rate question:

Next question

Comment on this question

All contents of this site are 2012 E-Medical Revision Ltd

http://www.emrcs.com/question/question.php?q=0

Terms and Conditions

Privacy policy

1/1

17/05/2015

Reference ranges

Question 7 of 273

Previous

Next

A 73 year old man is due to undergo a radical prostatectomy for carcinoma of the prostate
gland. To which of the following lymph nodes will the tumour drain primarily?

A. Para aortic
B. Internal iliac

Question stats

Score: 58.3%
1

19.2%

50.2%

12.4%

11.5%

6.7%

50.2% of users answered this


question correctly

C. Superficial inguinal

End and review

6
7

D. Meso rectal
E. None of the above

Search
Next question

Go

The prostate lymphatic drainage is primarily to the internal iliac nodes and also the sacral
nodes. Although internal iliac is the first site.
Prostate gland
The prostate gland is approximately the shape and size of a walnut and is located inferior to
the bladder. It is separated from the rectum by Denonvilliers fascia and its blood supply is
derived from the internal iliac vessels. The internal sphincter lies at the apex of the gland and
may be damaged during prostatic surgery, affected individuals may complain of retrograde
ejaculation.
Summary of prostate gland
Arterial supply

Inferior vesical artery (from internal iliac)

Venous
drainage

Prostatic venous plexus (to paravertebral veins)

Lymphatic
drainage

Internal iliac nodes

Innervation

Inferior hypogastric plexus

Dimensions

Lobes

Zones

Transverse diameter (4cm)


AP diameter (2cm)
Height (3cm)

Posterior lobe: posterior to urethra


Median lobe: posterior to urethra, in between ejaculatory ducts
Lateral lobes x 2
Isthmus

Peripheral zone: subcapsular portion of posterior prostate. Most


prostate cancers are here
Central zone
Transition zone
Stroma

Relations
Anterior

Pubic symphysis
Prostatic venous plexus

Posterior

Denonvilliers fascia
Rectum
Ejaculatory ducts

Lateral

Venous plexus (lies on prostate)


Levator ani (immediately below the puboprostatic ligaments)

http://www.emrcs.com/question/question.php?q=0

1/2

17/05/2015

Image sourced from Wikipedia

Rate question:

Next question

Comment on this question

All contents of this site are 2012 E-Medical Revision Ltd

http://www.emrcs.com/question/question.php?q=0

Terms and Conditions

Privacy policy

2/2

17/05/2015

Reference ranges

Previous

Question 8 of 273

Next

Question stats

End and review

Score: 64.3%
1

10.2%

21%

22%

A. There are 7 cervical vertebrae

16.6%

B. The cervical and lumbar lordosis are secondary curves developing


after birth due to change in shape of the intervertebral discs

30.2%

Which of the following statements relating to the vertebral column is false?

30.2% of users answered this


question correctly

C. The lumbar vertebrae do not have a transverse process foramina


D. The lumbar vertebrae receive blood directly from the aorta

6
7
8

E. The spinous process is formed by the junction of the pedicles


posteriorly

Search
Go

Next question

The spinous process is formed by 2 laminae posteriorly.


Vertebral column

There are 7 cervical, 12 thoracic, 5 lumbar, and 5 sacral vertebrae.


The spinal cord segmental levels do not necessarily correspond to the vertebral
segments. For example, while the C1 cord is located at the C1 vertebra, the C8 cord is
situated at the C7 vertebra. While the T1 cord is situated at the T1 vertebra, the T12
cord is situated at the T8 vertebra. The lumbar cord is situated between T9 and T11
vertebrae. The sacral cord is situated between the T12 to L2 vertebrae.

Cervical vertebrae
The interface between the first and second vertebra is called the atlanto-axis junction. The
C3 cord contains the phrenic nucleus.
Muscle

Nerve root value

Deltoid

C5,6

Biceps

C5,6

Wrist extensors

C6-8

Triceps

C6-8

Wrist flexors

C6-T1

Hand muscles

C8-T1

Thoracic vertebrae
The thoracic vertebral segments are defined by those that have a rib. The spinal roots form
the intercostal nerves that run on the bottom side of the ribs and these nerves control the
intercostal muscles and associated dermatomes.
Lumbosacral vertebrae
Form the remainder of the segments below the vertebrae of the thorax. The lumbosacral
spinal cord, however, starts at about T9 and continues only to L2. It contains most of the
segments that innervate the hip and legs, as well as the buttocks and anal regions.
Cauda Equina
The spinal cord ends at L1-L2 vertebral level. The tip of the spinal cord is called the conus.
Below the conus, there is a spray of spinal roots that is called the cauda equina. Injuries
below L2 represent injuries to spinal roots rather than the spinal cord proper.
Rate question:

Next question

Comment on this question

All contents of this site are 2012 E-Medical Revision Ltd

http://www.emrcs.com/question/question.php?q=0

Terms and Conditions

Privacy policy

1/1

17/05/2015

Reference ranges

Previous

Question 9 of 273

Next

A 78 year old lady falls over in her nursing home and sustains a displaced intracapsular
fracture of the femoral neck. A decision is made to perform a hemi arthroplasty through a
lateral approach. Which of these vessels will be divided to facilitate access?

Question stats

Score: 66.7%
1

7.2%

19.8%

19%

11.9%

42.1%

A. Saphenous vein
42.1% of users answered this
question correctly

B. Superior gluteal artery

End and review

6
7

C. Superficial circumflex iliac artery


8

D. Profunda femoris artery


9

Search

E. Transverse branch of the lateral circumflex artery

Go

Next question

During the Hardinge style lateral approach the transverse branch of the lateral circumflex
artery is divided to gain access. The vessels and its branches are illustrated below:

Image sourced from Wikipedia

Hip joint

Head of femur articulates with acetabulum of the pelvis


Both covered by articular hyaline cartilage
The acetabulum forms at the union of the ilium, pubis, and ischium
The triradiate cartilage (Y-shaped growth plate) separates the pelvic bones
The acetabulum holds the femoral head by the acetabular labrum
Normal angle between femoral head and femoral shaft is 130o

Ligaments
Transverse ligament: joints anterior and posterior ends of the articular cartilage
Head of femur ligament (ligamentum teres): acetabular notch to the fovea. Contains
arterial supply to head of femur in children.

http://www.emrcs.com/question/question.php?q=0

1/2

17/05/2015

Image sourced from Wikipedia

Image sourced from Wikipedia

Extracapsular ligaments
Iliofemoral ligament: inverted Y shape. Anterior iliac spine to the trochanteric line
Pubofemoral ligament: acetabulum to lesser trochanter
Ischiofemoral ligament: posterior support. Ischium to greater trochanter.

Blood supply
Medial circumflex femoral and lateral circumflex femoral arteries (Branches of profunda
femoris). Also from the inferior gluteal artery. These form an anastomosis and travel to up
the femoral neck to supply the head.

Rate question:

Next question

Comment on this question

All contents of this site are 2012 E-Medical Revision Ltd

http://www.emrcs.com/question/question.php?q=0

Terms and Conditions

Privacy policy

2/2

17/05/2015

Reference ranges

Question 1 of 264

Next

A 73 year old man undergoes a sub total oesophagectomy with anastomosis of the stomach
to the cervical oesophagus. Which vessel will be primarily responsible for the arterial supply
to the oesophageal portion of the anastomosis?

Question stats

End and review

Score: 100%

14.6%

8.1%

23.4%

42.3%

11.5%

A. Superior thyroid artery


42.3% of users answered this
question correctly

B. Internal carotid artery


C. Direct branches from the thoracic aorta
D. Inferior thyroid artery

Search

E. Subclavian artery

Go

Next question

The cervical oesophagus is supplied by the inferior thyroid artery. The thoracic oesophagus
(removed in this case) is supplied by direct branches from the thoracic aorta.
Oesophagus

25cm long
Starts at C6 vertebra, pierces diaphragm at T10 and ends at T11
Squamous epithelium

Constrictions of the oesophagus


Structure

Distance from incisors

Cricoid cartilage

15cm

Arch of the Aorta

22.5cm

Left principal bronchus

27cm

Diaphragmatic hiatus

40cm

Relations
Anteriorly

Trachea to T4
Recurrent laryngeal nerve
Left bronchus, Left atrium
Diaphragm

Posteriorly

Thoracic duct to left at T5


Hemiazygos to the left T8
Descending aorta
First 2 intercostal branches of aorta

Left

Thoracic duct
Left subclavian artery

Right

Azygos vein

Arterial, venous and lymphatic drainage of the oesophagus


Artery

Vein

Lymphatics

Muscularis
externa

Upper
third

Inferior
thyroid

Inferior thyroid

Deep
cervical

Striated muscle

Mid third

Aortic
branches

Azygos branches

Mediastinal

Smooth & striated


muscle

Lower
third

Left gastric

Posterior mediastinal and


coeliac

Gastric

Smooth muscle

Nerve supply

http://www.emrcs.com/question/question.php?q=0

1/2

17/05/2015

Upper half is supplied by recurrent laryngeal nerve


Lower half by oesophageal plexus (vagus)

Histology
Mucosa :Non-keratinized stratified squamous epithelium
Submucosa: glandular tissue
Muscularis externa (muscularis): composition varies. See table
Adventitia

Rate question:

Next question

Comment on this question

All contents of this site are 2012 E-Medical Revision Ltd

http://www.emrcs.com/question/question.php?q=0

Terms and Conditions

Privacy policy

2/2

17/05/2015

Reference ranges

Question 2 of 264

Previous

Next

Which of the following structures is not closely related to the brachial artery?

Question stats

Score: 50%

20.4%

9.9%

30.6%

A. Ulnar nerve

28.3%

B. Median nerve

10.7%

C. Cephalic vein

30.6% of users answered this


question correctly

D. Long head of triceps

End and review

1
2

E. Median cubital vein


Search
Next question
Go

The cephalic vein lies superficially and on the contralateral side of the arm to the brachial
artery. The relation of the ulnar nerves and others are demonstrated in the image below:

Image sourced from Wikipedia

Brachial artery
The brachial artery begins at the lower border of teres major as a continuation of the axillary
artery. It terminates in the cubital fossa at the level of the neck of the radius by dividing into
the radial and ulnar arteries.
Relations
Posterior relations include the long head of triceps with the radial nerve and profunda
vessels intervening. Anteriorly it is overlapped by the medial border of biceps.
It is crossed by the median nerve in the middle of the arm.
In the cubital fossa it is separated from the median cubital vein by the bicipital aponeurosis.
The basilic vein is in contact at the most proximal aspect of the cubital fossa and lies
medially.
Rate question:

Next question

Comment on this question

All contents of this site are 2012 E-Medical Revision Ltd

http://www.emrcs.com/question/question.php?q=0

Terms and Conditions

Privacy policy

1/1

17/05/2015

Reference ranges

Previous3 / 3

Question 3-5 of 264

Next

Question stats

End and review

Score: 87.5%

Average score for registered users:

1
2

Theme: Anatomy of the abdominal aorta


A. Common iliac artery

73.8%

68%

60.8%

3-5 3 / 3

B. Median sacral artery


C. Left renal artery
D. Right renal artery

Search

E. Inferior mesenteric artery

Go

F. Superior mesenteric artery


G. Coeliac axis
H. Lumbar arteries
Please select the branch of the abdominal aorta that most closely matches the description
provided. Each option may be used once, more than once or not at all.

3.

An aortic branch that leaves the aorta approximately 1cm below the coeliac axis.
Superior mesenteric artery
The SMA leaves the aorta approximately 1cm below the coeliac axis. This is
usually a level of L1 to L 2. It's crossed anteriorly by the splenic vein and the
body of the pancreas. It runs downwards and forwards anterior to the uncinate
process.

4.

The most inferior single branch of the aorta.


Median sacral artery
The median sacral artery leaves the aorta a little above its bifurcation. It
descends in the midline anterior to L4 and L5.

5.

The most inferior anterior branch of the aorta.


Inferior mesenteric artery
The IMA leaves the front of of the aorta usually about 3 to 4cm superior to its
bifurcation.

Next question

Abdominal aortic branches

Branches

Level

Paired

Type

Inferior phrenic

T12 (Upper border)

Yes

Parietal

Coeliac

T12

No

Visceral

Superior mesenteric

L1

No

Visceral

Middle suprarenal

L1

Yes

Visceral

Renal

L1-L2

Yes

Visceral

Gonadal

L2

Yes

Visceral

Lumbar

L1-L4

Yes

Parietal

Inferior mesenteric

L3

No

Visceral

Median sacral

L4

No

Parietal

Common iliac

L4

Yes

Terminal

Rate question:
http://www.emrcs.com/question/question.php?q=0

Next question

1/2

17/05/2015

Comment on this question

All contents of this site are 2012 E-Medical Revision Ltd

http://www.emrcs.com/question/question.php?q=0

Terms and Conditions

Privacy policy

2/2

17/05/2015

Reference ranges

Previous

Question 6 of 264

Next

Which ligament keeps the head of the radius connected to the radial notch of the ulna?

Question stats

Score: 88.9%

55.1%

14.1%

11.6%

A. Annular (orbicular) ligament

8.6%

B. Quadrate ligament

10.5%

C. Radial collateral ligament of the elbow

55.1% of users answered this


question correctly

D. Ulnar collateral ligament

End and review

1
2
3-5 3 / 3
6

E. Radial collateral ligament


Search
Next question
Go

Theme from September 2013


The annular ligament connects the radial head to the radial notch of the ulna. This is
illustrated below:

Image sourced from Wikipedia

Radius
The radius is one of the two long forearm bones that extends from the lateral side of the
elbow to the thumb side of the wrist. It has two expanded ends, of which the distal end is the
larger. Key points relating to its topography and relations are outlined below;
Upper end
Articular cartilage- covers medial > lateral side
Articulates with radial notch of the ulna by the annular ligament
Muscle attachment- biceps brachii at the tuberosity

Shaft
Muscle attachment
Upper third of the body

Supinator
Flexor digitorum superficialis
Flexor pollicis longus

Middle third of the body

Pronator teres

Lower quarter of the body

Pronator quadratus
Tendon of supinator longus

Lower end
Quadrilateral
Anterior surface- capsule of wrist joint
Medial surface- head of ulna
Lateral surface- ends in the styloid process
Posterior surface: 3 grooves containing:
1. Tendons of extensor carpi radialis longus and brevis
2. Tendon of extensor pollicis longus
3. Tendon of extensor indicis

http://www.emrcs.com/question/question.php?q=0

1/2

17/05/2015

Image sourced from Wikipedia

Rate question:

Next question

Comment on this question

All contents of this site are 2012 E-Medical Revision Ltd

http://www.emrcs.com/question/question.php?q=0

Terms and Conditions

Privacy policy

2/2

17/05/2015

Reference ranges

Question 8 of 264

Previous

Next

Which of the following statements relating to the Cavernous Sinus is false?

Question stats

End and review

Score: 91.7%

13.9%

14.8%

1
2
3-5 3 / 3

11.2%

A. The pituitary gland lies medially

44.4%

B. The internal carotid artery passes through it

15.7%

C. The temporal lobe of the brain is a lateral relation

44.4% of users answered this


question correctly

D. The mandibular branch of the trigeminal and optic nerve lie on the
lateral wall

E. The ophthalmic veins drain into the anterior aspect of the sinus
Search
Go

Next question

The veins that drain into the sinus are important as sepsis can cause cavernous sinus
thrombosis. The maxillary branch of the trigeminal and not the mandibular branches pass
through the sinus
Cavernous sinus
The cavernous sinuses are paired and are situated on the body of the sphenoid bone. It
runs from the superior orbital fissure to the petrous temporal bone.
Relations
Medial

Lateral

Pituitary fossa Temporal lobe


Sphenoid sinus

Contents
Lateral wall components

(from top to bottom:)


Oculomotor nerve
Trochlear nerve
Ophthalmic nerve
Maxillary nerve

Contents of the sinus

(from medial to lateral:)


Internal carotid artery (and sympathetic plexus)
Abducens nerve

Blood supply
Ophthalmic vein, superficial cortical veins, basilar plexus of veins posteriorly.
Drains into the internal jugular vein via: the superior and inferior petrosal sinuses

Image sourced from Wikipedia

Rate question:

Next question

Comment on this question

All contents of this site are 2012 E-Medical Revision Ltd

http://www.emrcs.com/question/question.php?q=0

Terms and Conditions

Privacy policy

1/1

17/05/2015

Reference ranges

Question 9 of 264

Previous

Next

Which of the following is not a branch of the subclavian artery?

Question stats

End and review

Score: 92.3%

57.5%

12.6%

1
2
3-5 3 / 3

8.3%

A. Superior thyroid artery

9.5%

B. Vertebral artery

12%

C. Thyrocervical trunk

57.5% of users answered this


question correctly

D. Internal thoracic artery

8
9

E. Dorsal scapular artery


Search
Next question
Go

Mnemonic for the branches of the subclavian artery: VIT C & D


V ertebral artery
I nternal thoracic
T hyrocervical trunk
C ostalcervical trunk
D orsal scapular

Superior thyroid artery is a branch of the external carotid artery.


Subclavian artery
Path
The left subclavian comes directly off the arch of aorta
The right subclavian arises from the brachiocephalic artery (trunk) when it bifurcates
into the subclavian and the right common carotid artery.
From its origin, the subclavian artery travels laterally, passing between anterior and
middle scalene muscles, deep to scalenus anterior and anterior to scalenus medius.
As the subclavian artery crosses the lateral border of the first rib, it becomes the
axillary artery. At this point it is superficial and within the subclavian triangle.

Image sourced from Wikipedia

Branches
Vertebral artery
Internal thoracic artery
Thyrocervical trunk
Costocervical trunk
Dorsal scapular artery

Rate question:

Next question

Comment on this question

All contents of this site are 2012 E-Medical Revision Ltd

http://www.emrcs.com/question/question.php?q=0

Terms and Conditions

Privacy policy

1/1

17/05/2015

Reference ranges

Previous

Question 10 of 264

Next

During the repair of an atrial septal defect the surgeons note that blood starts to leak from
the coronary sinus. Which structure forms the largest tributary of the coronary sinus?

A. Thebesian veins
B. Great cardiac vein

Question stats

Score: 93.3%

11%

58.7%

1
2
3-5 3 / 3

11.1%

9.1%

10.1%

58.7% of users answered this


question correctly

C. Oblique vein

End and review

D. Small cardiac veins

8
9
10

E. None of the above

Search
Next question

Go

The great cardiac vein runs in the anterior interventricular groove, and is the largest tributary
of the coronary sinus. The thebesian veins drain into the heart directly.
Heart anatomy
The walls of each cardiac chamber comprise:
Epicardium
Myocardium
Endocardium

Cardiac muscle is attached to the cardiac fibrous skeleton.


Relations
The heart and roots of the great vessels within the pericardial sac are related anteriorly to
the sternum, medial ends of the 3rd to 5th ribs on the left and their associated costal
cartilages. The heart and pericardial sac are situated obliquely two thirds to the left and one
third to the right of the median plane.
The pulmonary valve lies at the level of the left third costal cartilage.
The mitral valve lies at the level of the fourth costal cartilage.
Coronary sinus
This lies in the posterior part of the coronary groove and receives blood from the cardiac
veins. The great cardiac vein lies at its left and the middle and small cardiac veins lie on its
right. The smallest cardiac vein (anterior cardiac vein) drains into the right atrium directly.
Aortic sinus
Right coronary artery arises from the right aortic sinus, the left is derived from the left aortic
sinus and no vessel emerges from the posterior sinus.
Right and left ventricles
Structure

Left Ventricle

A-V Valve

Mitral (double leaflet)

Walls

Twice as thick as right

Trabeculae carnae

Much thicker and more numerous

Right coronary artery


The RCA supplies:
Right atrium
Diaphragmatic part of the left ventricle
Usually the posterior third of the interventricular septum
The sino atrial node (60% cases)
The atrio ventricular node (80% cases)

Left coronary artery


The LCA supplies:
Left atrium
Most of left ventricle
Part of the right ventricle
Anterior two thirds of the inter ventricular septum

http://www.emrcs.com/question/question.php?q=0

1/2

17/05/2015

The sino atrial node (remaining 40% cases)

Innervation of the heart


Autonomic nerve fibres from the superficial and deep cardiac plexus. These lie anterior to
the bifurcation of the trachea, posterior to the ascending aorta and superior to the
bifurcation of the pulmonary trunk. The parasympathetic supply to the heart is from
presynaptic fibres of the vagus nerves.
Valves of the heart
Mitral valve

Aortic valve

Pulmonary
valve

Tricuspid valve

2 cusps

3 cusps

3 cusps

3 cusps

First heart sound

Second heart
sound

Second heart
sound

First heart sound

1 anterior cusp

2 anterior cusps

2 anterior cusps

2 anterior cusps

Attached to chordae
tendinae

No chordae

No chordae

Attached to chordae
tendinae

Rate question:

Next question

Comment on this question

All contents of this site are 2012 E-Medical Revision Ltd

http://www.emrcs.com/question/question.php?q=0

Terms and Conditions

Privacy policy

2/2

17/05/2015

Reference ranges

Question 11 of 264

Previous

Next

Which of the following vessels provides the greatest contribution to the arterial supply of the
breast?

A. External mammary artery


B. Thoracoacromial artery

Question stats

Score: 87.5%

10.1%

11.3%

1
2
3-5 3 / 3

56.4%

14.4%

7.8%

56.4% of users answered this


question correctly

C. Internal mammary artery

End and review

D. Lateral thoracic artery

8
9
10

E. Subclavian artery

11

Search
Go

Next question

60% of the arterial supply to the breast is derived from the internal mammary artery. The
external mammary and lateral thoracic arteries also make a significant (but lesser)
contribution. This is of importance clinically in performing reduction mammoplasty
procedures.
Breast
The breast itself lies on a layer of pectoral fascia and the following muscles:
1. Pectoralis major
2. Serratus anterior
3. External oblique
Image showing the topography of the female breast

Image sourced from Wikipedia

Breast anatomy
Nerve supply

Branches of intercostal nerves from T4-T6.

Arterial supply

Venous
drainage

Internal mammary (thoracic) artery


External mammary artery (laterally)
Anterior intercostal arteries
Thoraco-acromial artery
Superficial venous plexus to subclavian, axillary and intercostal veins.

Lymphatic
drainage

70% Axillary nodes


Internal mammary chain
Other lymphatic sites such as deep cervical and supraclavicular fossa
(later in disease)

Rate question:

Next question

Comment on this question

All contents of this site are 2012 E-Medical Revision Ltd

http://www.emrcs.com/question/question.php?q=0

Terms and Conditions

Privacy policy

1/1

17/05/2015

Reference ranges

Previous

Question 12 of 264

Next

Which of the following muscles is supplied by the external laryngeal nerve?

Question stats

End and review

Score: 88.2%

8.7%

60.4%

1
2
3-5 3 / 3

11.3%

A. Transverse arytenoid

12.2%

B. Cricothyroid

7.4%

C. Thyro-arytenoid

60.4% of users answered this


question correctly

D. Posterior crico-arytenoid

8
9
10

E. Oblique arytenoid

11

Search
Next question

12
Go

The others are all supplied by the recurrent laryngeal nerve.


Larynx
The larynx lies in the anterior part of the neck at the levels of C3 to C6 vertebral bodies. The
laryngeal skeleton consists of a number of cartilagenous segments. Three of these are
paired; arytenoid, corniculate and cuneiform. Three are single; thyroid, cricoid and epiglottic.
The cricoid cartilage forms a complete ring (the only one to do so).
The laryngeal cavity extends from the laryngeal inlet to the level of the inferior border of the
cricoid cartilage.
Divisions of the laryngeal cavity
Laryngeal vestibule

Superior to the vestibular folds

Laryngeal ventricle

Lies between vestibular folds and superior to the vocal cords

Infraglottic cavity

Extends from vocal cords to inferior border of the cricoid cartilage

The vocal folds (true vocal cords) control sound production. The apex of each fold projects
medially into the laryngeal cavity. Each vocal fold includes:
Vocal ligament
Vocalis muscle (most medial part of thyroarytenoid muscle)
The glottis is composed of the vocal folds, processes and rima glottidis. The rima glottidis is
the narrowest potential site within the larynx, as the vocal cords may be completely opposed,
forming a complete barrier.
Muscles of the larynx
Muscle

Origin

Insertion

Innervation

Action

Posterior
cricoarytenoid

Posterior aspect
of lamina of
cricoid

Muscular process of
arytenoid

Recurrent
Laryngeal

Abducts vocal
fold

Lateral
cricoarytenoid

Arch of cricoid

Muscular process of
arytenoid

Recurrent
laryngeal

Adducts vocal
fold

Thyroarytenoid

Posterior aspect
of thyroid
cartilage

Muscular process of
arytenoid

Recurrent
laryngeal

Relaxes vocal
fold

Transverse
and oblique
arytenoids

Arytenoid
cartilage

Contralateral
arytenoid

Recurrent
laryngeal

Closure of
intercartilagenous
part of the rima
glottidis

Vocalis

Depression
between lamina
of thyroid
cartilage

Vocal ligament and


vocal process of
arytenoid cartilage

Recurrent
laryngeal

Relaxes posterior
vocal ligament,
tenses anterior
part

Cricothyroid

Anterolateral part
of cricoid

Inferior margin and


horn of thyroid
cartilage

External
laryngeal

Tenses vocal fold

Blood supply
Arterial supply is via the laryngeal arteries, branches of the superior and inferior thyroid
arteries. The superior laryngeal artery is closely related to the internal laryngeal nerve. The
inferior laryngeal artery is related to the inferior laryngeal nerve. Venous drainage is via
superior and inferior laryngeal veins, the former draining into the superior thyroid vein and
http://www.emrcs.com/question/question.php?q=0

1/2

17/05/2015

the latter draining into the middle thyroid vein, or thyroid venous plexus.
Lymphatic drainage
The vocal cords have no lymphatic drainage and this site acts as a lymphatic watershed.
Supraglottic part

Upper deep cervical nodes

Subglottic part

Prelaryngeal and pretracheal nodes and inferior deep cervical nodes

The aryepiglottic fold and vestibular folds have a dense plexus of lymphatics associated with
them and malignancies at these sites have a greater propensity for nodal metastasis.

Topography of the larynx

Image sourced from Wikipedia

Rate question:

Next question

Comment on this question

All contents of this site are 2012 E-Medical Revision Ltd

http://www.emrcs.com/question/question.php?q=0

Terms and Conditions

Privacy policy

2/2

17/05/2015

Reference ranges

Previous

Question 13 of 264

Next

A 28 year old man has sustained a non salvageable testicular injury to his left testicle. The
surgeon decides to perform an orchidectomy and divides the left testicular artery. From
which of the following does this vessel originate?

Question stats

Score: 88.9%

59%

17%

1
2
3-5 3 / 3

8.4%

7.6%

8%

A. Abdominal aorta
59% of users answered this
question correctly

B. Internal iliac artery

End and review

C. Inferior epigastric artery

8
9
10

D. Inferior vesical artery

11

Search

E. External iliac artery

12
Go

Next question

13

The testicular artery is a branch of the abdominal aorta.


Scrotal and testicular anatomy
Spermatic cord
Formed by the vas deferens and is covered by the following structures:
Layer

Origin

Internal spermatic fascia

Transversalis fascia

Cremasteric fascia

From the fascial coverings of internal oblique

External spermatic fascia

External oblique aponeurosis

Contents of the cord


Vas deferens

Transmits sperm and accessory gland secretions

Testicular artery

Branch of abdominal aorta supplies testis and


epididymis

Artery of vas deferens

Arises from inferior vesical artery

Cremasteric artery

Arises from inferior epigastric artery

Pampiniform plexus

Venous plexus, drains into right or left testicular


vein

Sympathetic nerve fibres

Lie on arteries, the parasympathetic fibres lie on the


vas

Genital branch of the genitofemoral


nerve

Supplies cremaster

Lymphatic vessels

Drain to lumbar and para-aortic nodes

Scrotum
Composed of skin and closely attached dartos fascia.
Arterial supply from the anterior and posterior scrotal arteries
Lymphatic drainage to the inguinal lymph nodes
Parietal layer of the tunica vaginalis is the innermost layer

Testes
The testes are surrounded by the tunica vaginalis (closed peritoneal sac). The parietal
layer of the tunica vaginalis adjacent to the internal spermatic fascia.
The testicular arteries arise from the aorta immediately inferiorly to the renal arteries.
The pampiniform plexus drains into the testicular veins, the left drains into the left
renal vein and the right into the inferior vena cava.
Lymphatic drainage is to the para-aortic nodes.

Rate question:

Next question

Comment on this question

http://www.emrcs.com/question/question.php?q=0

1/2

17/05/2015

All contents of this site are 2012 E-Medical Revision Ltd

http://www.emrcs.com/question/question.php?q=0

Terms and Conditions

Privacy policy

2/2

17/05/2015

Reference ranges

Previous

Question 14 of 264

Next

During a carotid endarterectomy the internal carotid artery is cross clamped. Assuming that
no shunt is inserted, which of the following vessels will not have diminished or absent flow as
a result?

Question stats

Score: 84.2%

12.1%

11.1%

1
2
3-5 3 / 3

13.1%

54.1%

9.5%

A. Anterior cerebral artery


54.1% of users answered this
question correctly

B. Ophthalmic artery

End and review

C. Middle cerebral artery

8
9
10

D. Maxillary artery

11

Search

E. None of the above

12
Go

Next question

13
14

Mnemonic for branches of the cerebral portion of the internal carotid artery 'Only Press
Carotid Arteries Momentarily'
Only = Opthalmic
Press = Posterior communicating
Carotid = Choroidal
Arteries = Anterior cerebral
Momentarily = Middle cerebral

The maxillary artery is a branch of the external carotid artery.


Internal carotid artery
The internal carotid artery is formed from the common carotid opposite the upper border of
the thyroid cartilage. It extends superiorly to enter the skull via the carotid canal. From the
carotid canal it then passes through the cavernous sinus, above which it divides into the
anterior and middle cerebral arteries.
Relations in the neck
Posterior

Medially

Laterally

Anteriorly

Longus capitis
Pre-vertebral fascia
Sympathetic chain
Superior laryngeal nerve

External carotid (near origin)


Wall of pharynx
Ascending pharyngeal artery

Internal jugular vein (moves posteriorly at entrance to skull)


Vagus nerve (most posterolaterally)

Sternocleidomastoid
Lingual and facial veins
Hypoglossal nerve

Relations in the carotid canal


Internal carotid plexus
Cochlea and middle ear cavity
Trigeminal ganglion (superiorly)
Leaves canal lies above the foramen lacerum

Path and relations in the cranial cavity


The artery bends sharply forwards in the cavernous sinus, the aducens nerve lies close to its
inferolateral aspect. The oculomotor, trochlear, opthalmic and, usually, the maxillary nerves
lie in the lateral wall of the sinus. Near the superior orbital fissure it turns posteriorly and
passes postero-medially to pierce the roof of the cavernous sinus inferior to the optic nerve.
It then passes between the optic and oculomotor nerves to terminate below the anterior
perforated substance by dividing into the anterior and middle cerebral arteries.
Branches

http://www.emrcs.com/question/question.php?q=0

1/2

17/05/2015

Anterior and middle cerebral artery


Ophthalmic artery
Posterior communicating artery
Anterior choroid artery
Meningeal arteries
Hypophyseal arteries

Image demonstrating the internal carotid artery and its relationship to the external carotid
artery

Image sourced from Wikipedia

Rate question:

Next question

Comment on this question

All contents of this site are 2012 E-Medical Revision Ltd

http://www.emrcs.com/question/question.php?q=0

Terms and Conditions

Privacy policy

2/2

17/05/2015

Reference ranges

Previous

Question 15 of 264

Next

A 72 year old lady with osteoporosis falls and sustains an intracapsular femoral neck
fracture. The fracture is completely displaced. Which of the following vessels is the main
contributor to the arterial supply of the femoral head?

Question stats

Score: 80%

9.3%

7.5%

1
2
3-5 3 / 3

7.8%

68.6%

6.7%

A. Deep external pudendal artery


68.6% of users answered this
question correctly

B. Superficial femoral artery

End and review

C. External iliac artery

8
9
10

D. Circumflex femoral arteries

11

Search

E. Superficial external pudendal artery

12
Go

Next question

13
14

Theme from 2010 Exam


The vessels which form the anastomoses around the femoral head are derived from the
medial and lateral circumflex femoral arteries. These are usually derived from the profunda
femoris artery.

15

Hip joint

Head of femur articulates with acetabulum of the pelvis


Both covered by articular hyaline cartilage
The acetabulum forms at the union of the ilium, pubis, and ischium
The triradiate cartilage (Y-shaped growth plate) separates the pelvic bones
The acetabulum holds the femoral head by the acetabular labrum
Normal angle between femoral head and femoral shaft is 130o

Ligaments
Transverse ligament: joints anterior and posterior ends of the articular cartilage
Head of femur ligament (ligamentum teres): acetabular notch to the fovea. Contains
arterial supply to head of femur in children.

Image sourced from Wikipedia

Image sourced from Wikipedia

http://www.emrcs.com/question/question.php?q=0

1/2

17/05/2015

Extracapsular ligaments
Iliofemoral ligament: inverted Y shape. Anterior iliac spine to the trochanteric line
Pubofemoral ligament: acetabulum to lesser trochanter
Ischiofemoral ligament: posterior support. Ischium to greater trochanter.

Blood supply
Medial circumflex femoral and lateral circumflex femoral arteries (Branches of profunda
femoris). Also from the inferior gluteal artery. These form an anastomosis and travel to up
the femoral neck to supply the head.

Rate question:

Next question

Comment on this question

All contents of this site are 2012 E-Medical Revision Ltd

http://www.emrcs.com/question/question.php?q=0

Terms and Conditions

Privacy policy

2/2

17/05/2015

Reference ranges

Question 16 of 264

Previous

Next

A 21 year old man is hit with a hammer and sustains a depressed skull fracture at the vertex.
Which of the following sinuses is at risk in this injury?

A. Superior sagittal sinus


B. Inferior petrosal sinus

Question stats

Score: 81.8%

63.8%

7.8%

1
2
3-5 3 / 3

10%

10.3%

8.1%

63.8% of users answered this


question correctly

C. Transverse sinus

End and review

D. Inferior sagittal sinus

8
9
10

E. Straight sinus

11

Search

12
Go

Next question

13
14

Theme in September 2011 Exam

15

The superior sagittal sinus is at greatest risk in this pattern of injury. This sinus begins at the
front of the crista galli and courses backwards along the falx cerebri. It becomes continuous
with the right transverse sinus near the internal occipital protuberance.

16

Cranial venous sinuses


The cranial venous sinuses are located within the dura mater. They have no valves which is
important in the potential for spreading sepsis. They eventually drain into the internal jugular
vein.
They are:
Superior sagittal sinus
Inferior sagittal sinus
Straight sinus
Transverse sinus
Sigmoid sinus
Confluence of sinuses
Occipital sinus
Cavernous sinus
Topography of cranial venous sinuses

Image sourced from Wikipedia

Rate question:

Next question

Comment on this question

All contents of this site are 2012 E-Medical Revision Ltd

http://www.emrcs.com/question/question.php?q=0

Terms and Conditions

Privacy policy

1/1

17/05/2015

Reference ranges

Question 17 of 264

Previous

Next

A 44 year old man is stabbed in the back and the left kidney is injured. A haematoma forms,
which of the following fascial structures will contain the haematoma?

A. Waldeyers fascia
B. Sibsons fascia

Question stats

Score: 83.3%

11.3%

9.2%

1
2
3-5 3 / 3

11.2%

60.3%

8%

60.3% of users answered this


question correctly

C. Bucks fascia

End and review

D. Gerotas fascia

8
9
10

E. Denonvilliers fascia

11

Search

12
Go

Next question

13
14

Waldeyers fascia- Posterior ano-rectum


Sibsons fascia- Lung apex
Bucks fascia- Base of penis
Gerotas fascia- Surrounding kidney
Denonvilliers fascia- Between rectum and prostate

15
16
17

Renal anatomy
Each kidney is about 11cm long, 5cm wide and 3cm thick. They are located in a deep gutter
alongside the projecting vertebral bodies, on the anterior surface of psoas major. In most
cases the left kidney lies approximately 1.5cm higher than the right. The upper pole of both
kidneys approximates with the 11th rib (beware pneumothorax during nephrectomy). On the
left hand side the hilum is located at the L1 vertebral level and the right kidney at level L1-2.
The lower border of the kidneys is usually alongside L3.
The table below shows the anatomical relations of the kidneys:
Relations
Relations

Right Kidney

Left Kidney

Posterior

Quadratus lumborum, diaphragm,


psoas major, transversus abdominis

Quadratus lumborum, diaphragm,


psoas major, transversus abdominis

Anterior

Hepatic flexure of colon

Stomach, Pancreatic tail

Superior

Liver, adrenal gland

Spleen, adrenal gland

Fascial covering
Each kidney and suprarenal gland is enclosed within a common layer of investing fascia,
derived from the transversalis fascia. It is divided into anterior and posterior layers (Gerotas
fascia).
Renal structure
Kidneys are surrounded by an outer cortex and an inner medulla which usually contains
between 6 and 10 pyramidal structures. The papilla marks the innermost apex of these. They
terminate at the renal pelvis, into the ureter.
Lying in a hollow within the kidney is the renal sinus. This contains:
1. Branches of the renal artery
2. Tributaries of the renal vein
3. Major and minor calyces's
4. Fat
Structures at the renal hilum
The renal vein lies most anteriorly, then renal artery (it is an end artery) and the ureter lies
most posterior.
Rate question:

Next question

Comment on this question

All contents of this site are 2012 E-Medical Revision Ltd

http://www.emrcs.com/question/question.php?q=0

Terms and Conditions

Privacy policy

1/1

17/05/2015

Reference ranges

Question 18 of 264

Previous

Next

A baby is found to have a Klumpke's palsy post delivery. Which of the following is most likely
to be present?

A. Loss of flexors of the wrist


B. Weak elbow flexion

Question stats

Score: 84.6%

35.4%

13%

1
2
3-5 3 / 3

20.4%

14.2%

17.1%

35.4% of users answered this


question correctly

C. Pronation of the forearm

End and review

D. Adducted shoulder

8
9
10

E. Shoulder medially rotated

11

Search

12
Next question

Go

13
14

Features of Klumpkes Paralysis


Claw hand (MCP joints extended and IP joints flexed)
Loss of sensation over medial aspect of forearm and hand
Horner's syndrome
Loss of flexors of the wrist

15
16
17
18

A C8, T1 root lesion is called Klumpke's paralysis and is caused by delivery with the arm
extended.
Brachial plexus

Origin

Anterior rami of C5 to T1

Sections of the
plexus

Roots

Trunks

Roots, trunks, divisions, cords, branches


Mnemonic:Real Teenagers Drink Cold Beer

Located in the posterior triangle


Pass between scalenus anterior and medius

Located posterior to middle third of clavicle


Upper and middle trunks related superiorly to the subclavian
artery
Lower trunk passes over 1st rib posterior to the subclavian
artery

Divisions

Apex of axilla

Cords

Related to axillary artery

Diagram illustrating the branches of the brachial plexus

Image sourced from Wikipedia

Cutaneous sensation of the upper limb

http://www.emrcs.com/question/question.php?q=0

1/2

17/05/2015

Image sourced from Wikipedia

Rate question:

Next question

Comment on this question

All contents of this site are 2012 E-Medical Revision Ltd

http://www.emrcs.com/question/question.php?q=0

Terms and Conditions

Privacy policy

2/2

17/05/2015

Reference ranges

Previous

Question 19 of 264

Next

A 22 year old man undergoes a superficial parotidectomy for a pleomorphic adenoma. The
operation does not proceed well and a diathermy malfunction results in division of the buccal
branch of the facial nerve. Which of the following muscles will not demonstrate impaired
function as a result?

Question stats

Score: 81.5%

21%

31.2%

1
2
3-5 3 / 3

20%

13.8%

14%

31.2% of users answered this


question correctly

A. Zygomaticus minor

End and review

8
9

B. Mentalis
10

C. Buccinator
11

Search

D. Levator anguli oris

12

E. Risorius

Go

13
Next question

14
15

Buccal branch supplies


Zygomaticus minor

Elevates upper lip

Risorius

Aids smile

16
17
18
19

Pulls corner of mouth backward and compresses cheek


Buccinator
Levator anguli oris

Pulls angles of mouth upward and toward midline

Orbicularis

Closes and tightens lips together

Nasalis

Flares nostrils and compresses nostrils

Facial nerve
The facial nerve is the main nerve supplying the structures of the second embryonic
branchial arch. It is predominantly an efferent nerve to the muscles of facial expression,
digastric muscle and also to many glandular structures. It contains a few afferent fibres which
originate in the cells of its genicular ganglion and are concerned with taste.
Supply - 'face, ear, taste, tear'
Face: muscles of facial expression
Ear: nerve to stapedius
Taste: supplies anterior two-thirds of tongue
Tear: parasympathetic fibres to lacrimal glands, also salivary glands

Path
Subarachnoid path
Origin: motor- pons, sensory- nervus intermedius
Pass through the petrous temporal bone into the internal auditory meatus with the
vestibulocochlear nerve. Here they combine to become the facial nerve.

Facial canal path


The canal passes superior to the vestibule of the inner ear
At the medial aspect of the middle ear, it becomes wider and contains the geniculate
ganglion.
- 3 branches:
1. greater petrosal nerve
2. nerve to stapedius
3. chorda tympani
Stylomastoid foramen
Passes through the stylomastoid foramen (tympanic cavity anterior and mastoid
antrum posteriorly)
Posterior auricular nerve and branch to posterior belly of digastric and stylohyoid
muscle

Face

http://www.emrcs.com/question/question.php?q=0

1/2

17/05/2015

Enters parotid gland and divides into 5 branches:


Temporal branch
Zygomatic branch
Buccal branch
Marginal mandibular branch
Cervical branch

Rate question:

Next question

Comment on this question

All contents of this site are 2012 E-Medical Revision Ltd

http://www.emrcs.com/question/question.php?q=0

Terms and Conditions

Privacy policy

2/2

17/05/2015

Reference ranges

Previous

Question 20 of 264

Next

At which of the following vertebral body levels does the common carotid artery typically
bifurcate into the external and internal carotid arteries?

A. C4
B. C2

Question stats

Score: 82.8%

47.2%

12.9%

1
2
3-5 3 / 3

7.8%

23.9%

8.1%

47.2% of users answered this


question correctly

C. C1

End and review

D. C6

8
9
10

E. C7

11

Search

12
Next question

It terminates at the upper border of the thyroid cartilege, Which is usually located at C4.

Go

13
14
15

Common carotid artery

16
17

The right common carotid artery arises at the bifurcation of the brachiocephalic trunk, the left
common carotid arises from the arch of the aorta. Both terminate at the level of the upper
border of the thyroid cartilage (the lower border of the third cervical vertebra) by dividing into
the internal and external carotid arteries.

18
19
20

Left common carotid artery


This vessel arises immediately to the left and slightly behind the origin of the brachiocephalic
trunk. Its thoracic portion is 2.5- 3.5 cm in length and runs superolaterally to the
sternoclavicular joint.
In the thorax
The vessel is in contact, from below upwards, with the trachea, left recurrent laryngeal nerve,
left margin of the oesophagus. Anteriorly the left brachiocephalic vein runs across the artery,
and the cardiac branches from the left vagus descend in front of it. These structures
together with the thymus and the anterior margins of the left lung and pleura separate the
artery from the manubrium.
In the neck
The artery runs superiorly deep to sternocleidomastoid and then enters the anterior triangle.
At this point it lies within the carotid sheath with the vagus nerve and the internal jugular vein.
Posteriorly the sympathetic trunk lies between the vessel and the prevertebral fascia. At the
level of C7 the vertebral artery and thoracic duct lie behind it. The anterior tubercle of C6
transverse process is prominent and the artery can be compressed against this structure (it
corresponds to the level of the cricoid).
Anteriorly at C6 the omohyoid muscle passes superficial to the artery.
Within the carotid sheath the jugular vein lies lateral to the artery.
Right common carotid artery
The right common carotid arises from the brachiocephalic artery. The right common carotid
artery corresponds with the cervical portion of the left common carotid, except that there is
no thoracic duct on the right. The oesophagus is less closely related to the right carotid than
the left.
Summary points about the carotid anatomy
Path
Passes behind the sternoclavicular joint (12% patients above this level) to the upper border
of the thyroid cartilage, to divide into the external (ECA) and internal carotid arteries (ICA).
Relations
Level of 6th cervical vertebra crossed by omohyoid
Then passes deep to the thyrohyoid, sternohyoid, sternomastoid muscles.
Passes anterior to the carotid tubercle (transverse process 6th cervical vertebra)-NB
compression here stops haemorrhage.
The inferior thyroid artery passes posterior to the common carotid artery.
Then : Left common carotid artery crossed by thoracic duct, Right common carotid
artery crossed by recurrent laryngeal nerve

http://www.emrcs.com/question/question.php?q=0

1/2

17/05/2015

Image sourced from Wikipedia

Rate question:

Next question

Comment on this question

All contents of this site are 2012 E-Medical Revision Ltd

http://www.emrcs.com/question/question.php?q=0

Terms and Conditions

Privacy policy

2/2

17/05/2015

Reference ranges

Previous

Question 21 of 264

Next

A man is stabbed in the chest to the right of the manubriosternal angle. Which structure is
least likely to be injured in this case?

A. Right pleura
B. The trachea

Question stats

Score: 77.4%

13.3%

18.9%

1
2
3-5 3 / 3

12.8%

32.4%

22.6%

32.4% of users answered this


question correctly

C. Right phrenic nerve

End and review

D. Right recurrent laryngeal nerve

8
9
10

E. Brachiocephalic vein

11

Search

12
Go

Next question

13
14

The right recurrent laryngeal nerve branches off the right vagus more proximally and arches
posteriorly round the subclavian artery. So of the structures given it is the least likely to be
injured.

15
16
17

Mediastinum

18
19

Region between the pulmonary cavities.


It is covered by the mediastinal pleura. It does not contain the lungs.
It extends from the thoracic inlet superiorly to the diaphragm inferiorly.

20
21

Mediastinal regions
Superior mediastinum (between manubriosternal angle and T4/5)
Middle mediastinum
Posterior mediastinum
Anterior mediastinum

Region
Superior mediastinum

Anterior mediastinum

Middle mediastinum

Posterior mediastinum

Contents
Superior vena cava
Brachiocephalic veins
Arch of aorta
Thoracic duct
Trachea
Oesophagus
Thymus
Vagus nerve
Left recurrent laryngeal nerve
Phrenic nerve

Thymic remnants
Lymph nodes
Fat

Pericardium
Heart
Aortic root
Arch of azygos vein
Main bronchi

Oesophagus
Thoracic aorta
Azygos vein
Thoracic duct
Vagus nerve
Sympathetic nerve trunks
Splanchnic nerves

Rate question:

Next question

Comment on this question

All contents of this site are 2012 E-Medical Revision Ltd

http://www.emrcs.com/question/question.php?q=0

Terms and Conditions

Privacy policy

1/2

17/05/2015

http://www.emrcs.com/question/question.php?q=0

2/2

17/05/2015

Reference ranges

Previous

Question 22 of 264

Next

An 18 year old man is stabbed in the neck and has to undergo repair of a laceration to the
internal carotid artery. Post operatively he is noted to have a Horners syndrome. Which of
the following will not be present?

Question stats

Score: 78.8%

23.7%

29.5%

1
2
3-5 3 / 3

13.8%

8.3%

24.8%

A. Apparent enopthalmos
29.5% of users answered this
question correctly

B. Loss of sweating on the entire ipsilateral side of the face

End and review

C. Constricted pupil

8
9
10

D. Mild ptosis

11

Search

E. Normal sympathetic activity in the torso

12
Go

13

Next question

14
15

The anhidrosis will be mild as this is a distal lesion and at worst only a very limited area of the
ipsilateral face will be anhidrotic.

16
17

Horners syndrome

18
19

Horners syndrome, clinical features:

20

Ptosis
Miosis
Enopthalmos
Anhydrosis

21
22

Primarily a disorder of the sympathetic nervous system. Extent of symptoms depends upon
the anatomical site of the lesion.
Proximal lesions occur along the hypothalamospinal tract
Distal lesions are usually post ganglionic e.g. at level of internal carotid artery or beyond.

Rate question:

Next question

Comment on this question

All contents of this site are 2012 E-Medical Revision Ltd

http://www.emrcs.com/question/question.php?q=0

Terms and Conditions

Privacy policy

1/1

17/05/2015

Reference ranges

Question 23 of 264

Previous

Next

Which of the following types of epithelium lines the lumenal surface of the normal
oesophagus?

A. Non keratinised stratified squamous epithelium


B. Ciliated columnar epithelium

Question stats

Score: 80%

55.4%

11%

1
2
3-5 3 / 3

12.8%

14%

6.9%

55.4% of users answered this


question correctly

C. Keratinised stratified squamous epithelium

End and review

D. Non ciliated columnar epithelium

8
9
10

E. None of the above

11

Search

12
Next question

The oesphagus is lined by non keratinised stratified squamous epithelium. Changes to


glandular type epithelium occur as part of metaplastic processes in reflux.

Go

13
14
15
16

Oesophagus

17
18

25cm long
Starts at C6 vertebra, pierces diaphragm at T10 and ends at T11
Squamous epithelium

19
20
21
22

Constrictions of the oesophagus


Structure

Distance from incisors

Cricoid cartilage

15cm

Arch of the Aorta

22.5cm

Left principal bronchus

27cm

Diaphragmatic hiatus

40cm

23

Relations
Anteriorly

Trachea to T4
Recurrent laryngeal nerve
Left bronchus, Left atrium
Diaphragm

Posteriorly

Thoracic duct to left at T5


Hemiazygos to the left T8
Descending aorta
First 2 intercostal branches of aorta

Left

Thoracic duct
Left subclavian artery

Right

Azygos vein

Arterial, venous and lymphatic drainage of the oesophagus


Artery

Vein

Lymphatics

Muscularis
externa

Upper
third

Inferior
thyroid

Inferior thyroid

Deep
cervical

Striated muscle

Mid third

Aortic
branches

Azygos branches

Mediastinal

Smooth & striated


muscle

Lower
third

Left gastric

Posterior mediastinal and


coeliac

Gastric

Smooth muscle

Nerve supply
Upper half is supplied by recurrent laryngeal nerve

http://www.emrcs.com/question/question.php?q=0

1/2

17/05/2015

Lower half by oesophageal plexus (vagus)

Histology
Mucosa :Non-keratinized stratified squamous epithelium
Submucosa: glandular tissue
Muscularis externa (muscularis): composition varies. See table
Adventitia

Rate question:

Next question

Comment on this question

All contents of this site are 2012 E-Medical Revision Ltd

http://www.emrcs.com/question/question.php?q=0

Terms and Conditions

Privacy policy

2/2

17/05/2015

Reference ranges

Question 24 of 264

Previous

Next

A 23 year old man is stabbed in the neck, in the region between the omohyoid and digastric
muscles, the injury is explored surgically. At operation a nerve injury is identified immediately
superior to the lingual artery as is branches off the external carotid artery. Which of the
following is the most likely result of this injury?

Question stats

Score: 81.1%

51.6%

19.4%

1
2
3-5 3 / 3

7.9%

12.2%

9%

51.6% of users answered this


question correctly

A. Paralysis of the ipsilateral side of the tongue

End and review

8
9

B. Abduction of the ipsilateral vocal cord


10

C. Winging of the scapula


11

Search

D. Paralysis of the ipsilateral hemi diaphragm

12

E. Inability to abduct the shoulder

Go

13
Next question

14
15

The hypoglossal nerve runs anterior to the external carotid, above the lingual arterial
branch. If damaged then ipsilateral paralysis of the genioglossus, hyoglossus and
styloglossus muscles will occur. If the patient is asked to protrude their tongue then it will
tend to point to the affected side.

16
17
18
19

Anterior triangle of the neck

20
21

Boundaries

22
23

Anterior border of the Sternocleidomastoid


Lower border of mandible
Anterior midline

24

Sub triangles (divided by Digastric above and Omohyoid)


Muscular triangle: Neck strap muscles
Carotid triangle: Carotid sheath
Submandibular Triangle (digastric)

Contents of the anterior triangle


Digastric triangle

Submandibular gland
Submandibular nodes
Facial vessels
Hypoglossal nerve

Muscular triangle

Strap muscles
External jugular vein

Carotid triangle

Carotid sheath (Common carotid, vagus and internal jugular vein)


Ansa cervicalis

Nerve supply to digastric muscle


Anterior: Mylohyoid nerve
Posterior: Facial nerve

Image sourced from Wikipedia

Rate question:

Next question

Comment on this question

http://www.emrcs.com/question/question.php?q=0

1/2

17/05/2015

All contents of this site are 2012 E-Medical Revision Ltd

http://www.emrcs.com/question/question.php?q=0

Terms and Conditions

Privacy policy

2/2

17/05/2015

Reference ranges

Previous

Question 25 of 264

Next

Which of the following structures is not directly related to the right adrenal gland?

Question stats

End and review

Score: 81.6%

13%

22.7%

1
2
3-5 3 / 3

34.2%

A. Diaphragm posteriorly

17.7%

B. Bare area of the liver anteriorly

12.4%

C. Right renal vein

34.2% of users answered this


question correctly

D. Inferior vena cava

8
9
10

E. Hepato-renal pouch

11

Search
Next question

12
Go

13

The right renal vein is very short and lies more inferiorly.

14
15

Adrenal gland anatomy

16

Anatomy

17
18

Location

Superomedially to the upper pole of each kidney

Relationships of
the right adrenal

Diaphragm-Posteriorly, Kidney-Inferiorly, Vena Cava-Medially, Hepatorenal pouch and bare area of the liver-Anteriorly

20

Relationships of
the left adrenal

Crus of the diaphragm-Postero- medially, Pancreas and splenic


vessels-Inferiorly, Lesser sac and stomach-Anteriorly

22

Superior adrenal arteries- from inferior phrenic artery, Middle adrenal


arteries - from aorta, Inferior adrenal arteries -from renal arteries

24

Arterial supply
Venous drainage
of the right adrenal

Via one central vein directly into the IVC

Venous drainage
of the left adrenal

Via one central vein into the left renal vein

Rate question:

19

21

23

25

Next question

Comment on this question

All contents of this site are 2012 E-Medical Revision Ltd

http://www.emrcs.com/question/question.php?q=0

Terms and Conditions

Privacy policy

1/1

17/05/2015

Reference ranges

Previous

Question 26 of 264

Next

With respect to the basilic vein, which statement is false?

A. Its deep anatomical location makes it unsuitable for use as an


arteriovenous access site in fistula surgery
B. It originates from the dorsal venous network on the hand

Question stats

Score: 82.1%

41.7%

13%

1
2
3-5 3 / 3

10.5%

13.4%

21.5%

41.7% of users answered this


question correctly

C. It travels up the medial aspect of the forearm

End and review

D. Halfway between the shoulder and the elbow it lies deep to muscle

8
9
10

E. It joins the brachial vein to form the axillary vein

11

Search

12
Go

Next question

13
14

It is used in arteriovenous fistula surgery during a procedure known as a basilic vein


transposition.

15
16

Basilic vein

17

The basilic and cephalic veins both provide the main pathways of venous drainage for the
arm and hand. It is continuous with the palmar venous arch distally and the axillary vein
proximally.

18

Path

21

19
20

22

Originates on the medial side of the dorsal venous network of the hand, and passes
up the forearm and arm.
Most of its course is superficial.
Near the region anterior to the cubital fossa the vein joins the cephalic vein.
Midway up the humerus the basilic vein passes deep under the muscles.
At the lower border of the teres major muscle, the anterior and posterior circumflex
humeral veins feed into it.
It is often joined by the medial brachial vein before draining into the axillary vein.

Rate question:

23
24
25
26

Next question

Comment on this question

All contents of this site are 2012 E-Medical Revision Ltd

http://www.emrcs.com/question/question.php?q=0

Terms and Conditions

Privacy policy

1/1

17/05/2015

Reference ranges

Question 27 of 264

Previous

Next

Mobilisation of the left lobe of the liver will facilitate surgical access to which of the following?

Question stats

End and review

Score: 82.9%

38.1%

18%

1
2
3-5 3 / 3

11.9%

A. Abdominal oesophagus

10.2%

B. Duodenum

21.8%

C. Right colic flexure

38.1% of users answered this


question correctly

D. Right kidney

8
9
10

E. Pylorus of stomach

11

Search
Next question

12
Go

13

The fundus of the stomach is a posterior relation. The pylorus lies more inferolaterally.
During a total gastrectomy division of the ligaments holding the left lobe of the liver will
facilitate access to the proximal stomach and abdominal oesophagus. This manoeuvre is
seldom beneficial during a distal gastrectomy.

14
15
16
17

Liver

18
19

Structure of the liver

20

Right lobe

Supplied by right hepatic artery


Contains Couinaud segments V to VIII (-/+Sg I)

21
22

Left lobe

Supplied by the left hepatic artery


Contains Couinaud segments II to IV (+/- Sg1)

23
24
25

Quadrate lobe

Part of the right lobe anatomically, functionally is part of the left


Couinaud segment IV
Porta hepatis lies behind
On the right lies the gallbladder fossa
On the left lies the fossa for the umbilical vein

Caudate lobe

26
27

Supplied by both right and left hepatic arteries


Couinaud segment I
Lies behind the plane of the porta hepatis
Anterior and lateral to the inferior vena cava
Bile from the caudate lobe drains into both right and left hepatic ducts

Detailed knowledge of Couinaud segments is not required for MRCS Part A


Between the liver lobules are portal canals which contain the portal triad: Hepatic
Artery, Portal Vein, tributary of Bile Duct.

Relations of the liver


Anterior

Postero inferiorly

Diaphragm

Oesophagus

Xiphoid process

Stomach
Duodenum
Hepatic flexure of colon
Right kidney
Gallbladder
Inferior vena cava

Porta hepatis
Location

Transmits

Postero inferior surface, it joins nearly at right angles with the left sagittal fossa,
and separates the caudate lobe behind from the quadrate lobe in front
Common hepatic duct

http://www.emrcs.com/question/question.php?q=0

1/2

17/05/2015

Hepatic artery
Portal vein
Sympathetic and parasympathetic nerve fibres
Lymphatic drainage of the liver (and nodes)

Ligaments
Falciform
ligament

2 layer fold peritoneum from the umbilicus to anterior liver surface


Contains ligamentum teres (remnant umbilical vein)
On superior liver surface it splits into the coronary and left
triangular ligaments

Ligamentum teres

Joins the left branch of the portal vein in the porta hepatis

Ligamentum
venosum

Remnant of ductus venosus

Arterial supply
Hepatic artery

Venous
Hepatic veins
Portal vein

Nervous supply
Sympathetic and parasympathetic trunks of coeliac plexus

Rate question:

Next question

Comment on this question

All contents of this site are 2012 E-Medical Revision Ltd

http://www.emrcs.com/question/question.php?q=0

Terms and Conditions

Privacy policy

2/2

17/05/2015

Reference ranges

Previous

Question 28 of 264

Next

The following statements relating to the ankle joint are true except?

Question stats

End and review

Score: 83.7%

9.1%

44.2%

1
2
3-5 3 / 3

9.8%

A. Three groups of ligaments provide mechanical stability

18.2%

B. The sural nerve lies medial to the Achilles tendon at its point of
insertion

18.6%

44.2% of users answered this


question correctly

C. Eversion of the foot occurs at the sub talar joint


D. The flexor hallucis longus tendon is the most posterior structure at the
medial malleolus

8
9
10
11

Search

E. The saphenous nerve crosses the ankle joint.

12
Go

13

Next question

14

The sural nerve lies behind the distal fibula. Inversion and eversion are sub talar
movements. The structures passing behind the medial malleolus from anterior to posterior
include: tibialis posterior, flexor digitorum longus, posterior tibia vein, posterior tibial artery,
nerve, flexor hallucis longus.

15
16
17
18
19

Ankle joint
20
21

The ankle joint is a synovial joint composed of the tibia and fibula superiorly and the talus
inferiorly.

22
23

Ligaments of the ankle joint

24

Deltoid ligament (medially)


Lateral collateral ligament
Talofibular ligaments (both anteriorly and posteriorly)

25
26
27

The calcaneofibular ligament is separate from the fibrous capsule of the joint. The two
talofibular ligaments are fused with it.

28

The components of the syndesmosis are


Antero-inferior tibiofibular ligament
Postero-inferior tibiofibular ligament
Inferior transverse tibiofibular ligament
Interosseous ligament

Movements at the ankle joint


Plantar flexion (55 degrees)
Dorsiflexion (35 degrees)
Inversion and eversion movements occur at the level of the sub talar joint

Nerve supply
Branches of deep peroneal and tibial nerves.
References
Golano P et al. Anatomy of the ankle ligaments: a pictorial essay. Knee Surg Sports
Traumatol Arthrosc. 2010 May;18(5):557-69
Rate question:

Next question

Comment on this question

All contents of this site are 2012 E-Medical Revision Ltd

http://www.emrcs.com/question/question.php?q=0

Terms and Conditions

Privacy policy

1/1

17/05/2015

Reference ranges

Question 29 of 264

Previous

Next

The oesophagus is constricted at the following levels apart from:

Question stats

End and review

Score: 80%

17.8%

15.4%

1
2
3-5 3 / 3

32.1%

A. Cricoid cartilage

18.3%

B. Arch of the aorta

16.4%

C. Lower oesophageal sphincter

32.1% of users answered this


question correctly

D. Left main stem bronchus

8
9
10

E. Diaphragmatic hiatus

11

Search
Next question

12
Go

13

The oesophagus is not constricted at the level of the lower oesophageal sphincter.

14
15

Oesophagus

16
17

25cm long
Starts at C6 vertebra, pierces diaphragm at T10 and ends at T11
Squamous epithelium

18
19
20

Constrictions of the oesophagus

21

Structure

Distance from incisors

22

Cricoid cartilage

15cm

23

Arch of the Aorta

22.5cm

Left principal bronchus

27cm

Diaphragmatic hiatus

40cm

24
25
26
27
28
29

Relations
Anteriorly

Trachea to T4
Recurrent laryngeal nerve
Left bronchus, Left atrium
Diaphragm

Posteriorly

Thoracic duct to left at T5


Hemiazygos to the left T8
Descending aorta
First 2 intercostal branches of aorta

Left

Thoracic duct
Left subclavian artery

Right

Azygos vein

Arterial, venous and lymphatic drainage of the oesophagus


Artery

Vein

Lymphatics

Muscularis
externa

Upper
third

Inferior
thyroid

Inferior thyroid

Deep
cervical

Striated muscle

Mid third

Aortic
branches

Azygos branches

Mediastinal

Smooth & striated


muscle

Lower
third

Left gastric

Posterior mediastinal and


coeliac

Gastric

Smooth muscle

Nerve supply
Upper half is supplied by recurrent laryngeal nerve
Lower half by oesophageal plexus (vagus)

http://www.emrcs.com/question/question.php?q=0

1/2

17/05/2015

Histology
Mucosa :Non-keratinized stratified squamous epithelium
Submucosa: glandular tissue
Muscularis externa (muscularis): composition varies. See table
Adventitia

Rate question:

Next question

Comment on this question

All contents of this site are 2012 E-Medical Revision Ltd

http://www.emrcs.com/question/question.php?q=0

Terms and Conditions

Privacy policy

2/2

17/05/2015

Reference ranges

Question 30 of 264

Previous

Next

A 19 year old man is playing rugby when he suddenly notices a severe pain at the
posterolateral aspect of his right thigh. Which of the following muscle groups is most likely to
have been injured?

Question stats

Score: 80.9%

11.6%

12.9%

1
2
3-5 3 / 3

56.2%

11.2%

8%

A. Semimembranosus
56.2% of users answered this
question correctly

B. Semitendinosus

End and review

C. Long head of biceps femoris

8
9
10

D. Gastrocnemius

11

Search

E. Soleus

12
Go

Next question

13
14

Theme from April 2012 Exam


The biceps femoris is the laterally located hamstring muscle. The semitendinosus and
semimembranosus are located medially. Rupture of gastrocnemius and soleus may occur but
is less common.

15
16
17
18

Biceps femoris

19
20

The biceps femoris is one of the hamstring group of muscles located in the posterior upper
thigh. It has two heads.

21
22

Long head

23

Origin

Ischial tuberosity

24

Insertion

Fibular head

25

Action

Knee flexion, lateral rotation tibia, extension hip

Innervation

Tibial division of sciatic nerve (L5, S1, S2)

Arterial
supply

Profunda femoris artery, inferior gluteal artery, and the superior muscular
branches of popliteal artery

26
27
28
29
30

Image demonstrating the biceps femoris muscle, with the long head outlined

Image sourced from Wikipedia

Short head
Origin

Lateral lip of linea aspera, lateral supracondylar ridge of femur

Insertion

Fibular head

Action

Knee flexion, lateral rotation tibia

Innervation

Common peroneal division of sciatic nerve (L5, S1, S2)

Arterial
supply

Profunda femoris artery, inferior gluteal artery, and the superior muscular
branches of popliteal artery

Rate question:

Next question

Comment on this question

http://www.emrcs.com/question/question.php?q=0

1/2

17/05/2015

All contents of this site are 2012 E-Medical Revision Ltd

http://www.emrcs.com/question/question.php?q=0

Terms and Conditions

Privacy policy

2/2

17/05/2015

Reference ranges

Previous

Question 31 of 264

Next

Which of the following is a branch of the third part of the axillary artery?

Question stats

End and review

Score: 81.6%

7%

10.9%

1
2
3-5 3 / 3

12.1%

A. Superior thoracic

15.7%

B. Lateral thoracic

54.2%

C. Dorsal scapular

54.2% of users answered this


question correctly

D. Thoracoacromial

8
9
10

E. Posterior circumflex humeral

11

Search
Next question

12
Go

13

The other branches include:

14

Subscapular
Anterior circumflex humeral

15
16
17
18

Axilla

19
20

Boundaries of the axilla


Medially

Chest wall and Serratus anterior

Laterally

Humeral head

Floor

Subscapularis

Anterior aspect

Lateral border of Pectoralis major

Fascia

Clavipectoral fascia

21
22
23
24
25
26
27
28

Content:
Long thoracic
nerve (of Bell)

29

Derived from C5-C7 and passes behind the brachial plexus to enter the
axilla. It lies on the medial chest wall and supplies serratus anterior. Its
location puts it at risk during axillary surgery and damage will lead to
winging of the scapula.

Thoracodorsal
nerve and
thoracodorsal
trunk

Innervate and vascularise latissimus dorsi.

Axillary vein

Lies at the apex of the axilla, it is the continuation of the basilic vein.
Becomes the subclavian vein at the outer border of the first rib.

Intercostobrachial
nerves

Traverse the axillary lymph nodes and are often divided during axillary
surgery. They provide cutaneous sensation to the axillary skin.

Lymph nodes

The axilla is the main site of lymphatic drainage for the breast.

Rate question:

30
31

Next question

Comment on this question

All contents of this site are 2012 E-Medical Revision Ltd

http://www.emrcs.com/question/question.php?q=0

Terms and Conditions

Privacy policy

1/1

17/05/2015

Reference ranges

Previous

Question 32 of 264

Next

Which of the following structures separates the intervertebral disks from the spinal cord?

Question stats

End and review

Score: 82%

15.1%

38.1%

1
2
3-5 3 / 3

8.6%

A. Anterior longitudinal ligament

11.8%

B. Posterior longitudinal ligament

26.4%

C. Supraspinous ligament

38.1% of users answered this


question correctly

D. Interspinous ligament

8
9
10

E. Ligamentum flavum

11

Search
Next question

12
Go

13

The posterior longitudinal ligament overlies the posterior aspect of the vertebral bodies. It
also overlies the posterior aspect of the intervertebral disks.

14
15

Intervertebral discs

16
17
18

Consist of an outer annulus fibrosus and an inner nucleus pulposus.


The anulus fibrosus consists of several layers of fibrocartilage.
The nucleus pulposus contains loose fibres suspended in a mucoprotein gel with the
consistency of jelly. The nucleus of the disc acts as a shock absorber.
Pressure on the disc causes posterior protrusion of the nucleus pulposus. Most
commonly in the lumbrosacral and lower cervical areas.
The discs are separated by hyaline cartilage.
There is one disc between each pair of vertebrae, except for C1/2 and the
sacrococcygeal vertebrae.

19
20
21
22
23
24
25
26

Rate question:

27

Next question

28
Comment on this question

29
30
31
32

All contents of this site are 2012 E-Medical Revision Ltd

http://www.emrcs.com/question/question.php?q=0

Terms and Conditions

Privacy policy

1/1

17/05/2015

Reference ranges

Previous

Question 33 of 264

Next

At what level does the aorta bifurcate into the left and right common iliac arteries?

Question stats

End and review

Score: 82.7%

7.5%

8.7%

1
2
3-5 3 / 3

10%

A. L1

63.3%

B. L2

10.6%

C. L3

63.3% of users answered this


question correctly

D. L4

8
9
10

E. L5

11

Search
Next question

12
Go

13

Theme from September 2013 Exam


The aorta typically bifurcates at L4. This level is usually fairly constant and is often tested in
the exam.

14
15
16

Levels

17
18

Transpyloric plane
Level of the body of L1

19
20
21

Pylorus stomach
Left kidney hilum (L1- left one!)
Right hilum of the kidney (1.5cm lower than the left)
Fundus of the gallbladder
Neck of pancreas
Duodenojejunal flexure
Superior mesenteric artery
Portal vein
Left and right colic flexure
Root of the transverse mesocolon
2nd part of the duodenum
Upper part of conus medullaris
Spleen

22
23
24
25
26
27
28
29
30
31
32

Can be identified by asking the supine patient to sit up without using their arms. The plane is
located where the lateral border of the rectus muscle crosses the costal margin.

33

Anatomical planes
Subcostal plane

Lowest margin of 10th costal cartilage

Intercristal plane

Level of body L4 (highest point of iliac crest)

Intertubercular plane

Level of body L5

Common level landmarks


Inferior mesenteric artery

L3

Bifurcation of aorta into common iliac arteries

L4

Formation of IVC

L5 (union of common iliac veins)

Diaphragm apertures

Rate question:

Vena cava T8
Oesophagus T10
Aortic hiatus T12

Next question

Comment on this question

All contents of this site are 2012 E-Medical Revision Ltd

http://www.emrcs.com/question/question.php?q=0

Terms and Conditions

Privacy policy

1/1

17/05/2015

Reference ranges

Previous

Question 34 of 264

Next

A 23 year old man is due to undergo a mitral valve repair for mitral regurgitation. Which of
the following is a feature of the mitral valve?

A. Its closure is marked by the first heart sound


B. It has two anterior cusps

Question stats

Score: 83.3%

49.5%

14.2%

2
3-5 3 / 3

8.6%

17%

10.7%

D. The chordae tendinae anchor the valve directly to the wall of the left
ventricle

8
9
10
11

Search

E. It is best auscultated in the left third interspace

49.5% of users answered this


question correctly

C. The chordae tendinae attach to the anterior cusps only

End and review

12
Go

Next question

13
14

The mitral valve is best auscultated over the cardiac apex, where its closure marks the first
heart sound. It has only two cusps. These are attached to chordae tendinae which
themselves are linked to the wall of the ventricle by the papillary muscles.

15
16
17
18

Heart anatomy

19

The walls of each cardiac chamber comprise:


Epicardium
Myocardium
Endocardium

20
21
22
23

Cardiac muscle is attached to the cardiac fibrous skeleton.

24
25

Relations
The heart and roots of the great vessels within the pericardial sac are related anteriorly to
the sternum, medial ends of the 3rd to 5th ribs on the left and their associated costal
cartilages. The heart and pericardial sac are situated obliquely two thirds to the left and one
third to the right of the median plane.
The pulmonary valve lies at the level of the left third costal cartilage.
The mitral valve lies at the level of the fourth costal cartilage.

26
27
28
29
30
31
32

Coronary sinus
This lies in the posterior part of the coronary groove and receives blood from the cardiac
veins. The great cardiac vein lies at its left and the middle and small cardiac veins lie on its
right. The smallest cardiac vein (anterior cardiac vein) drains into the right atrium directly.

33
34

Aortic sinus
Right coronary artery arises from the right aortic sinus, the left is derived from the left aortic
sinus and no vessel emerges from the posterior sinus.
Right and left ventricles
Structure

Left Ventricle

A-V Valve

Mitral (double leaflet)

Walls

Twice as thick as right

Trabeculae carnae

Much thicker and more numerous

Right coronary artery


The RCA supplies:
Right atrium
Diaphragmatic part of the left ventricle
Usually the posterior third of the interventricular septum
The sino atrial node (60% cases)
The atrio ventricular node (80% cases)

Left coronary artery


The LCA supplies:
Left atrium
Most of left ventricle

http://www.emrcs.com/question/question.php?q=0

1/2

17/05/2015

Part of the right ventricle


Anterior two thirds of the inter ventricular septum
The sino atrial node (remaining 40% cases)

Innervation of the heart


Autonomic nerve fibres from the superficial and deep cardiac plexus. These lie anterior to
the bifurcation of the trachea, posterior to the ascending aorta and superior to the
bifurcation of the pulmonary trunk. The parasympathetic supply to the heart is from
presynaptic fibres of the vagus nerves.
Valves of the heart
Mitral valve

Aortic valve

Pulmonary
valve

Tricuspid valve

2 cusps

3 cusps

3 cusps

3 cusps

First heart sound

Second heart
sound

Second heart
sound

First heart sound

1 anterior cusp

2 anterior cusps

2 anterior cusps

2 anterior cusps

Attached to chordae
tendinae

No chordae

No chordae

Attached to chordae
tendinae

Rate question:

Next question

Comment on this question

All contents of this site are 2012 E-Medical Revision Ltd

http://www.emrcs.com/question/question.php?q=0

Terms and Conditions

Privacy policy

2/2

17/05/2015

Reference ranges

Previous3 / 3

Question 35-37 of 264

Next

Question stats

End and review

Score: 85%

Average score for registered users:

1
2

Theme: Nerve lesions


A. Intercostobrachial

35

73.2%

36

67.6%

37

48.3%

3-5 3 / 3
6

B. Median

C. Axillary

D. Radial

Search

E. Ulnar

Go

10

F. Musculocutaneous

11

G. Brachial plexus upper cord

12

H. Brachial plexus lower cord

13

Please select the most likely nerve injury for the scenarios given. Each option may be used
once, more than once or not at all.

14
15
16
17

35.

A 23 year old rugby player sustains a Smiths Fracture. On examination


opposition of the thumb is markedly weakened.

18
19

Median

20

This high velocity injury can often produce significant angulation and
displacement. Both of these may impair the function of the median nerve with
loss of function of the muscles of the thenar eminence
36.

A 45 year old lady recovering from a mastectomy and axillary node clearance
notices that sensation in her armpit is impaired.

21
22
23
24
25
26

Intercostobrachial

27

The intercostobrachial nerves are frequently injured during axillary dissection.


These nerves traverse the axilla and supply cutaneous sensation.

28
29

37.

An 8 year old boy falls onto an outstretched hand and sustains a


supracondylar fracture. In addition to a weak radial pulse the child is noted to
have loss of pronation of the affected hand.

30
31
32

Median

33
34

This is a common injury in children. In this case the angulation and


displacement have resulted in median nerve injury.

35-37 3 / 3

Next question

Brachial plexus

Origin
Sections of the
plexus

Roots

Trunks

Anterior rami of C5 to T1
Roots, trunks, divisions, cords, branches
Mnemonic:Real Teenagers Drink Cold Beer

Located in the posterior triangle


Pass between scalenus anterior and medius

Located posterior to middle third of clavicle


Upper and middle trunks related superiorly to the subclavian
artery
Lower trunk passes over 1st rib posterior to the subclavian
artery

Divisions

Apex of axilla

Cords

Related to axillary artery

http://www.emrcs.com/question/question.php?q=0

1/2

17/05/2015

Diagram illustrating the branches of the brachial plexus

Image sourced from Wikipedia

Cutaneous sensation of the upper limb

Image sourced from Wikipedia

Rate question:

Next question

Comment on this question

All contents of this site are 2012 E-Medical Revision Ltd

http://www.emrcs.com/question/question.php?q=0

Terms and Conditions

Privacy policy

2/2

17/05/2015

Reference ranges

Previous

Question 38 of 264

Next

A 23 year old lady with sialolithiasis of the submandibular gland is undergoing excision of the
gland. Which of the following nerves is at risk as the duct is mobilised?

A. Lingual nerve
B. Buccal nerve

Question stats

Score: 85.5%

52.8%

10.8%

1
2
3-5 3 / 3

15.4%

13.8%

7.1%

52.8% of users answered this


question correctly

C. Facial nerve

End and review

D. Glossopharyngeal

8
9
10

E. Vagus

11

Search

12
Go

Next question

13
14

The lingual nerve wraps around Whartons duct. The lingual nerve provides sensory supply
to the anterior 2/3 of the tongue.

15
16

Submandibular gland

17
18

Relations of the submandibular gland


Superficial

Deep

19

Platysma, deep fascia and mandible


Submandibular lymph nodes
Facial vein (facial artery near mandible)
Marginal mandibular nerve
Cervical branch of the facial nerve

20
21
22
23

Facial artery (inferior to the mandible)


Mylohoid muscle
Sub mandibular duct
Hyoglossus muscle
Lingual nerve
Submandibular ganglion
Hypoglossal nerve

24
25
26
27
28
29

Submandibular duct (Wharton's duct)

30

Opens lateral to the lingual frenulum on the anterior floor of mouth.


5 cm length
Lingual nerve wraps around Wharton's duct. As the duct passes forwards it crosses
medial to the nerve to lie above it and then crosses back, lateral to it, to reach a
position below the nerve.

31
32
33
34
35-37 3 / 3

Innervation

38

Sympathetic innervation- Derived from superior cervical ganglion


Parasympathetic innervation- Submandibular ganglion via lingual nerve

Arterial supply
Branch of the facial artery. The facial artery passes through the gland to groove its deep
surface. It then emerges onto the face by passing between the gland and the mandible.
Venous drainage
Anterior facial vein (lies deep to the Marginal Mandibular nerve)
Lymphatic drainage
Deep cervical and jugular chains of nodes
Rate question:

Next question

Comment on this question

All contents of this site are 2012 E-Medical Revision Ltd

http://www.emrcs.com/question/question.php?q=0

Terms and Conditions

Privacy policy

1/1

17/05/2015

Reference ranges

Previous

Question 39 of 264

Next

Which of the following is true in connection with the phrenic nerves?

Question stats

End and review

Score: 82.8%

44.2%

19.3%

1
2
3-5 3 / 3

10.1%

A. They both lie anterior to the hilum of the lungs

13.8%

B. They are derived from spinal roots C 2,3,4

12.5%

C. They pierce the diaphragm at the level of T7

44.2% of users answered this


question correctly

D. They consist of motor fibres only

8
9
10

E. None of the above

11

Search
Next question

12
Go

13

C3,4,5
Keeps the diaphragm alive!

14
15
16

They both lie anterior to the hilum of the lung. The phrenic nerves have both motor and
sensory functions. For this reason sub diaphragmatic pathology may cause referred pain to
the shoulder.

17
18
19

Phrenic nerve

20
21

Origin
C3,4,5

22
23
24

Supplies
Diaphragm, sensation central diaphragm and pericardium

25
26
27
28

Path
The phrenic nerve passes with the internal jugular vein across scalenus anterior. It
passes deep to prevertebral fascia of deep cervical fascia.
Left: crosses anterior to the 1st part of the subclavian artery.
Right: Anterior to scalenus anterior and crosses anterior to the 2nd part of the
subclavian artery.
On both sides, the phrenic nerve runs posterior to the subclavian vein and posterior to
the internal thoracic artery as it enters the thorax.

29
30
31
32
33
34
35-37 3 / 3

Right phrenic nerve


In the superior mediastinum: anterior to right vagus and laterally to superior vena cava
Middle mediastinum: right of pericardium
It passes over the right atrium to exit the diaphragm at T8

38
39

Left phrenic nerve


Passes lateral to the left subclavian artery, aortic arch and left ventricle
Passes anterior to the root of the lung
Pierces the diaphragm alone

Image showing the passage of the phrenic nerve in the neck

http://www.emrcs.com/question/question.php?q=0

1/2

17/05/2015

Image sourced from Wikipedia

Rate question:

Next question

Comment on this question

All contents of this site are 2012 E-Medical Revision Ltd

http://www.emrcs.com/question/question.php?q=0

Terms and Conditions

Privacy policy

2/2

17/05/2015

Reference ranges

Question 40 of 264

Previous

Next

A 32 year old man presents with an inguinal hernia and undergoes an open surgical repair.
The surgeons decide to place a mesh on the posterior wall of the inguinal canal to complete
the repair, which of the following structures will lie posterior to the mesh?

Question stats

Score: 83.1%

57.1%

12.3%

1
2
3-5 3 / 3

9.9%

10.2%

10.5%

A. Transversalis fascia
57.1% of users answered this
question correctly

B. External oblique

End and review

C. Rectus abdominis

8
9
10

D. Obturator nerve

11

Search

E. None of the above

12
Go

Next question

13
14
15

Inguinal canal walls: 'MALT: 2M, 2A, 2L, 2T':

16

Starting from superior, moving around in order to posterior:


Superior wall (roof): 2 Muscles:Internal oblique, transversus abdominis
Anterior wall: 2 Aponeuroses: Aponeurosis of external oblique, Aponeurosis of internal
oblique
Lower wall (floor): 2 Ligaments: Inguinal Ligament, Lacunar Ligament Posterior wall: 2Ts:
Transversalis fascia, Conjoint Tendon

17
18
19
20
21
22

This is actually quite a straightforward question. It is simply asking for the structure that forms
the posterior wall of the inguinal canal. This is composed of the transversalis fascia, the
conjoint tendon and more laterally the deep inguinal ring.

23
24
25

Inguinal canal

26
27

Location

28

Above the inguinal ligament


The inguinal canal is 4cm long
The superficial ring is located anterior to the pubic tubercle
The deep ring is located approximately 1.5-2cm above the half way point between the
anterior superior iliac spine and the pubic tubercle

29
30
31
32
33

Boundaries of the inguinal canal

34
35-37 3 / 3

Floor

External oblique aponeurosis


Inguinal ligament
Lacunar ligament

38
39
40

Roof

Internal oblique
Transversus abdominis

Anterior wall
Posterior wall

Laterally

Medially

External oblique aponeurosis


Transversalis fascia
Conjoint tendon

Internal ring
Fibres of internal oblique

External ring
Conjoint tendon

Contents
Males

Spermatic cord and ilioinguinal


nerve

As it passes through the canal the spermatic


cord has 3 coverings:
External spermatic fascia from external
oblique aponeurosis
Cremasteric fascia
Internal spermatic fascia

http://www.emrcs.com/question/question.php?q=0

1/2

17/05/2015

Females

Round ligament of uterus and


ilioinguinal nerve

Related anatomy of the inguinal region


The boundaries of Hesselbachs triangle are commonly tested and illustrated below:

Image sourced from Wikipedia

The image below demonstrates the close relationship of the vessels to the lower limb with the
inguinal canal. A fact to be borne in mind when repairing hernial defects in this region.

Image sourced from Wikipedia

Rate question:

Next question

Comment on this question

All contents of this site are 2012 E-Medical Revision Ltd

http://www.emrcs.com/question/question.php?q=0

Terms and Conditions

Privacy policy

2/2

17/05/2015

Reference ranges

Previous

Question 41 of 264

Next

A 22 year old man is involved in a fight and is stabbed in the posterior aspect of his right leg.
The knife passes into the popliteal fossa. He sustains an injury to his tibial nerve. Which
muscle is least likely to be compromised as a result?

Question stats

Score: 83.6%

11.8%

9.3%

1
2
3-5 3 / 3

11%

14.6%

53.3%

A. Tibialis posterior
53.3% of users answered this
question correctly

B. Flexor hallucis longus

End and review

C. Flexor digitorum brevis

8
9
10

D. Soleus

11

Search

E. Peroneus tertius

12
Go

13

Next question

14
15

Peroneus tertius is innervated by the deep peroneal nerve.

16

Tibial nerve

17
18

Begins at the upper border of the popliteal fossa and is a branch of the sciatic nerve.

19
20

Root values: L4, L5, S1, S2, S3

21
22

Muscles innervated

23

Popliteus
Gastrocnemius
Soleus
Plantaris
Tibialis posterior
Flexor hallucis longus
Flexor digitorum brevis

24
25
26
27
28
29
30

Terminates by dividing into the medial and lateral plantar nerves.

31

Rate question:

Next question

32
33

Comment on this question

34
35-37 3 / 3
38
39
40
41

All contents of this site are 2012 E-Medical Revision Ltd

http://www.emrcs.com/question/question.php?q=0

Terms and Conditions

Privacy policy

1/1

17/05/2015

Reference ranges

Question 42 of 264

Previous

Next

Which of the following overlies the outer muscular layer of the intrathoracic oesophagus?

Question stats

End and review

Score: 82.4%

22.5%

10.5%

1
2
3-5 3 / 3

19%

A. Serosa

36.8%

B. Meissners plexus

11.1%

C. Auerbach's plexus

36.8% of users answered this


question correctly

D. Loose connective tissue

8
9
10

E. None of the above

11

Search
Next question

12
Go

13

The oesophagus has no serosal covering and hence holds sutures poorly. The Auerbach's
and Meissner's nerve plexuses lie in between the longitudinal and circular muscle layers and
submucosally. The sub mucosal location of the Meissner's nerve plexus facilitates its sensory
role.

14
15
16
17

Oesophagus

18
19

25cm long
Starts at C6 vertebra, pierces diaphragm at T10 and ends at T11
Squamous epithelium

20
21
22
23

Constrictions of the oesophagus


Structure

Distance from incisors

Cricoid cartilage

15cm

Arch of the Aorta

22.5cm

Left principal bronchus

27cm

Diaphragmatic hiatus

40cm

24
25
26
27
28
29
30
31

Relations

32

Anteriorly

33

Trachea to T4
Recurrent laryngeal nerve
Left bronchus, Left atrium
Diaphragm

34
35-37 3 / 3
38
39

Posteriorly

Thoracic duct to left at T5


Hemiazygos to the left T8
Descending aorta
First 2 intercostal branches of aorta

40
41
42

Left

Thoracic duct
Left subclavian artery

Right

Azygos vein

Arterial, venous and lymphatic drainage of the oesophagus


Artery

Vein

Lymphatics

Muscularis
externa

Upper
third

Inferior
thyroid

Inferior thyroid

Deep
cervical

Striated muscle

Mid third

Aortic
branches

Azygos branches

Mediastinal

Smooth & striated


muscle

Lower
third

Left gastric

Posterior mediastinal and


coeliac

Gastric

Smooth muscle

Nerve supply

http://www.emrcs.com/question/question.php?q=0

1/2

17/05/2015

Upper half is supplied by recurrent laryngeal nerve


Lower half by oesophageal plexus (vagus)

Histology
Mucosa :Non-keratinized stratified squamous epithelium
Submucosa: glandular tissue
Muscularis externa (muscularis): composition varies. See table
Adventitia

Rate question:

Next question

Comment on this question

All contents of this site are 2012 E-Medical Revision Ltd

http://www.emrcs.com/question/question.php?q=0

Terms and Conditions

Privacy policy

2/2

17/05/2015

Reference ranges

Previous

Question 43 of 264

Next

Which nerve lies medially on the thyroid gland, in the groove between the oesophagus and
trachea?

A. Vagus nerve
B. External laryngeal nerve

Question stats

Score: 82.9%

20.2%

8.3%

1
2
3-5 3 / 3

50.7%

10.7%

10.1%

50.7% of users answered this


question correctly

C. Recurrent laryngeal nerve

End and review

D. Ansa cervicalis

8
9
10

E. Phrenic nerve

11

Search

12
Go

Next question

13
14

The recurrent laryngeal nerve may be injured at this site during ligation of the inferior thyroid
artery.

15
16

Thyroid gland

17
18

Right and left lobes connected by isthmus


Surrounded by sheath from pretracheal layer of deep fascia
Apex: Lamina of thyroid cartilage
Base: 4th-5th tracheal ring
Pyramidal lobe: from isthmus
May be attached to foramen caecum at the base of the tongue

19
20
21
22
23
24

Relations
25

Anteromedially

26

Sternothyroid
Superior belly of omohyoid
Sternohyoid
Anterior aspect of sternocleidomastoid

27
28
29

Posterolaterally

Carotid sheath

30
31

Medially

Larynx
Trachea
Pharynx
Oesophagus
Cricothyroid muscle
External laryngeal nerve (near superior thyroid artery)
Recurrent laryngeal nerve (near inferior thyroid artery)

32
33
34
35-37 3 / 3
38
39

Posterior

Parathyroid glands
Anastomosis of superior and inferior thyroid arteries

40
41
42

Isthmus

Anteriorly: Sternothyroids, sternohyoids, anterior jugular veins


Posteriorly: 2nd, 3rd, 4th tracheal rings (attached via Ligament of
Berry)

43

Blood Supply
Arterial

Venous

Superior thyroid artery (1st branch of external carotid)


Inferior thyroid artery (from thyrocervical trunk)
Thyroidea ima (in 10% of population -from brachiocephalic artery or aorta)

Superior and middle thyroid veins - into the IJV


Inferior thyroid vein - into the brachiocephalic veins

Rate question:

Next question

Comment on this question

All contents of this site are 2012 E-Medical Revision Ltd

http://www.emrcs.com/question/question.php?q=0

Terms and Conditions

Privacy policy

1/2

17/05/2015

http://www.emrcs.com/question/question.php?q=0

2/2

17/05/2015

Reference ranges

Previous

Question 45 of 264

Next

Which of the following nerve roots contribute nerve fibres to the ansa cervicalis?

Question stats

End and review

Score: 83.8%

10.1%

50.9%

1
2
3-5 3 / 3

13.1%

A. C1 only

11.5%

B. C1, C2 and C3

14.3%

C. C2, C3 and C6

50.9% of users answered this


question correctly

D. C2, C4 and C5

8
9
10

E. C4, C5 and C6

11

Search
Next question

12
Go

13

Ansa cervicalis muscles:

14
15

GHost THought SOmeone Stupid Shot Irene


GenioHyoid
ThyroidHyoid
Superior Omohyoid
SternoThyroid
SternoHyoid
Inferior Omohyoid

16
17
18
19
20
21
22

The ansa cervicalis is composed of a superior and inferior root, derived from C1, C2 and C3.
The superior root arises where the nerve crosses the internal carotid artery. It descends
anterior to the carotid sheath in the anterior triangle. It is joined in the region of the mid neck
by the inferior root. The inferior root may pass either superficially or deep to the internal
jugular vein.

23
24
25
26
27

Ansa cervicalis
28

Superior
root

Branch of C1 anterolateral to carotid sheath

Inferior
root

Derived from C2 and C3 roots, passes posterolateral to the internal jugular vein
(may lie either deep or superficial to it)

Innervation

Sternohyoid
Sternothyroid
Omohyoid

29
30
31
32
33
34
35-37 3 / 3
38

The ansa cervicalis lies anterior to the carotid sheath. The nerve supply to the inferior strap
muscles enters at their inferior aspect. Therefore when dividing these muscles to expose a
large goitre, the muscles should be divided in their upper half.

39
40
41
42
43
44
45

Image sourced from Wikipedia

http://www.emrcs.com/question/question.php?q=0

1/2

17/05/2015

Rate question:

Next question

Comment on this question

All contents of this site are 2012 E-Medical Revision Ltd

http://www.emrcs.com/question/question.php?q=0

Terms and Conditions

Privacy policy

2/2

17/05/2015

Reference ranges

Previous

Question 46 of 264

Next

The anterior interosseous nerve is a branch of which of the following?

Question stats

End and review

Score: 84.2%

16.8%

11.6%

1
2
3-5 3 / 3

8.4%

A. Ulnar nerve

11.9%

B. Superficial branch of the radial nerve

51.4%

C. Axillary nerve

51.4% of users answered this


question correctly

D. Deep branch of the radial nerve

8
9
10

E. Median nerve

11

Search
Next question

12
Go

13
14
15

Anterior interosseous nerve

16

The anterior interosseous nerve (volar interosseous nerve) is a branch of the median nerve
that supplies the deep muscles on the front of the forearm, except the ulnar half of the flexor
digitorum profundus.

17
18
19

It accompanies the anterior interosseous artery along the anterior of the interosseous
membrane of the forearm, in the interval between the flexor pollicis longus and flexor
digitorum profundus, supplying the whole of the former and (most commonly) the radial half
of the latter, and ending below in the pronator quadratus and wrist joint.

20
21
22
23

Innervation
The anterior interosseous nerve classically innervates 2.5 muscles:

24
25
26

Flexor pollicis longus


Pronator quadratus
The radial half of flexor digitorum profundus (the lateral two out of the four tendons).

27
28
29

These muscles are in the deep level of the anterior compartment of the forearm.

30
31

Rate question:

Next question

32

Comment on this question

33
34
35-37 3 / 3
38
39
40
41
42
43
44
45
46

All contents of this site are 2012 E-Medical Revision Ltd

http://www.emrcs.com/question/question.php?q=0

Terms and Conditions

Privacy policy

1/1

17/05/2015

Reference ranges

Previous

Question 47 of 264

Next

At which of the following anatomical locations does the common peroneal nerve bifurcate into
the superficial and deep peroneal nerves?

A. Immediately anterior to the linea aspera


B. At the lateral aspect of the neck of the fibula

Question stats

Score: 84.6%

8.4%

52.4%

1
2
3-5 3 / 3

11.1%

16.9%

11.2%

52.4% of users answered this


question correctly

C. Within the substance of tibialis anterior muscle

End and review

D. At the inferomedial aspect of the popliteal fossa

8
9
10

E. Under the medial head of gastrocnemius

11

Search

12
Go

Next question

13
14

The common peroneal nerve bifurcates at the neck of the fibula (where it is most likely to be
injured).

15
16

Common peroneal nerve

17
18

Derived from the dorsal divisions of the sacral plexus (L4, L5, S1 and S2).

19

This nerve supplies the skin and fascia of the anterolateral surface of the leg and the
dorsum of the foot. It also innervates the muscles of the anterior and peroneal compartments
of the leg, extensor digitorum brevis as well as the knee, ankle and foot joints.

20
21
22

It is laterally placed within the sciatic nerve. From the bifurcation of the sciatic nerve it passes
inferolaterally in the lateral and proximal part of the popliteal fossa, under the cover of biceps
femoris and its tendon. To reach the posterior aspect of the fibular head. It ends by dividing
into the deep and superficial peroneal nerves at the point where it winds around the lateral
surface of the neck of the fibula in the body of peroneus longus, approximately 2cm distal to
the apex of the head of the fibula. It is palpable posterior to the head of the fibula.

23
24
25
26
27
28

Branches
Nerve to the short head of biceps
Articular branch (knee)

29

In the popliteal fossa

Lateral cutaneous nerve of the calf

31

Neck of fibula

Superficial and deep peroneal nerves

In the thigh

30

32
33
34
35-37 3 / 3

Rate question:

Next question

38
39

Comment on this question

40
41
42
43
44
45
46
47

All contents of this site are 2012 E-Medical Revision Ltd

http://www.emrcs.com/question/question.php?q=0

Terms and Conditions

Privacy policy

1/1

17/05/2015

Reference ranges

Previous

Question 48 of 264

Next

A 48 year old motor cyclist sustains a complex lower limb fracture in a motor accident. For a
time the popliteal artery is occluded and eventually repaired. Subsequently he develops a
compartment syndrome and the anterior and superficial posterior compartments of the lower
leg are decompressed. Unfortunately, the operating surgeon neglects to decompress the
deep posterior compartment. Which of the following muscles is least likely to be affected as a
result?

Question stats

End and review

Score: 82.5%

10.7%

47.7%

1
2
3-5 3 / 3

16%

12.7%

12.9%

47.7% of users answered this


question correctly

8
9
10

A. Flexor digitorum longus


B. Plantaris

11

Search

C. Tibialis posterior

12
Go

D. Flexor hallucis longus

13

E. None of the above

14
15
Next question

16
17

Muscles of the deep posterior compartment:


Tibialis posterior
Flexor hallucis longus
Flexor digitorum longus
Popliteus

18
19
20
21
22
23

The plantaris muscle lies within the superficial posterior compartment of the lower leg.

24
25

Lower limb- Muscular compartments


26
27

Anterior compartment
Muscle

Nerve

Action

Tibialis anterior

Deep peroneal
nerve

Dorsiflexes ankle joint, inverts foot

Extensor digitorum
longus

Deep peroneal
nerve

Extends lateral four toes, dorsiflexes ankle


joint

Peroneus tertius

Deep peroneal
nerve

Dorsiflexes ankle, everts foot

28
29
30
31
32
33
34
35-37 3 / 3

Extensor hallucis longus

Deep peroneal
nerve

Dorsiflexes ankle joint, extends big toe

38
39
40

Peroneal compartment

41

Muscle

Nerve

Action

Peroneus longus

Superficial peroneal nerve

Everts foot, assists in plantar flexion

Peroneus brevis

Superficial peroneal nerve

Plantar flexes the ankle joint

42
43
44
45

Superficial posterior compartment


< b=""><>
b=""><>

Nerve

Action

Gastrocnemius

Tibial nerve

Plantar flexes the foot, may also flex the knee

Soleus

Tibial nerve

Plantar flexor

46
47
48

Deep posterior compartment


Muscle

Nerve

Action

Flexor digitorum longus

Tibial

Flexes the lateral four toes

Flexor hallucis longus

Tibial

Flexes the great toe

Tibialis posterior

Tibial

Plantar flexor, inverts the foot

http://www.emrcs.com/question/question.php?q=0

1/2

17/05/2015

Rate question:

Next question

Comment on this question

All contents of this site are 2012 E-Medical Revision Ltd

http://www.emrcs.com/question/question.php?q=0

Terms and Conditions

Privacy policy

2/2

17/05/2015

Reference ranges

Previous

Question 49 of 264

Next

A 23 year old lady is undergoing a trendelenberg procedure for varicose veins. During the
dissection of the saphenofemoral junction, which of the structures listed below is most liable
to injury?

Question stats

Score: 82.9%

12.4%

19.2%

1
2
3-5 3 / 3

22.4%

19.3%

26.8%

A. Superficial circumflex iliac artery


26.8% of users answered this
question correctly

B. Superficial circumflex iliac vein

End and review

C. Femoral artery

8
9
10

D. Femoral nerve

11

Search

E. Deep external pudendal artery

12
Go

13

Next question

14
15

Theme from September 2011 exam

16

The deep external pudendal artery runs under the long saphenous vein close to its origin
and may be injured. It is at greatest risk of injury during the flush ligation of the
saphenofemoral junction. Provided an injury is identified and vessel ligated, injury is seldom
associated with any serious adverse sequelae.

17
18
19
20

Saphenous vein

21
22

Long saphenous vein


This vein may be harvested for triple or quadruple bypass surgery

23
24

Originates at the 1st digit where the dorsal vein merges with the dorsal venous arch of
the foot
Passes anterior to the medial malleolus and runs up the medial side of the leg
At the knee, it runs over the posterior border of the medial epicondyle of the femur
bone
Then passes laterally to lie on the anterior surface of the thigh before entering an
opening in the fascia lata called the saphenous opening
It joins with the femoral vein in the region of the femoral triangle at the saphenofemoral
junction

25
26
27
28
29
30
31
32

Tributaries

33
34

Medial marginal
Superficial epigastric
Superficial iliac circumflex
Superficial external pudendal veins

35-37 3 / 3
38
39
40
41

Short saphenous vein

42

Originates at the 5th digit where the dorsal vein merges with the dorsal venous arch of
the foot, which attaches to the great saphenous vein.
It passes around the lateral aspect of the foot (inferior and posterior to the lateral
malleolus) and runs along the posterior aspect of the leg (with the sural nerve)
Passes between the heads of the gastrocnemius muscle, and drains into the popliteal
vein, approximately at or above the level of the knee joint.

43
44
45
46
47
48

Rate question:

49

Next question

Comment on this question

All contents of this site are 2012 E-Medical Revision Ltd

http://www.emrcs.com/question/question.php?q=0

Terms and Conditions

Privacy policy

1/1

17/05/2015

Reference ranges

Previous

Question 50 of 264

Next

Considering the pituitary gland, which of the following is false?

Question stats

End and review

Score: 81%

7.2%

15.6%

1
2
3-5 3 / 3

16.3%

A. The anterior pituitary secretes thyroid stimulating hormone

39.3%

B. The anterior pituitary develops from Rathkes pouch

21.5%

C. Patients with craniopharyngioma may develop bi temporal hemianopia

39.3% of users answered this


question correctly

D. The pituitary is in direct contact with the optic chiasm

8
9
10

E. The posterior pituitary secretes oxytocin via a positive feedback loop

11

Search
Next question

12
Go

13

Although the optic chiasm is closely related to the pituitary, and craniopharyngiomas may
compress this structure leading to bitemporal hemianopia, it is separated from the chiasm
itself by a dural fold.

14
15
16

Pituitary Gland

17
18

The pituitary gland is located within the sella turcica within the sphenoid bone in the middle
cranial fossa. It is covered by a dural fold and weighs around 0.5g. It is attached to the
hypothalamus by the infundibulum. The anterior pituitary receives hormonal stimuli from the
hypothalamus by way of the hypothalamo-pituitary portal system. It develops from a
depression in the wall of the pharynx (Rathkes pouch).

19
20
21
22

Anterior pituitary hormones

23
24

Growth hormone
Thyroid stimulating hormone
ACTH
Prolactin
LH and FSH
Melanocyte releasing hormone

25
26
27
28
29
30

Posterior pituitary hormones


31

Oxytocin
Anti diuretic hormone

32
33
34
35-37 3 / 3

Rate question:

Next question

38
39

Comment on this question

40
41
42
43
44
45
46
47
48
49
50

All contents of this site are 2012 E-Medical Revision Ltd

http://www.emrcs.com/question/question.php?q=0

Terms and Conditions

Privacy policy

1/1

17/05/2015

Reference ranges

Previous

Question 51 of 264

Next

A 24 year old man is involved in a fight and his face is cut with a knife. The wound lies
immediately anterior to the tragus of the ear and extends anteriorly. The wound is surgically
explored and the laceration is found to be mainly superficial. It extends slightly more deeply
immediately inferior to the main trunk of the facial nerve. Bleeding is observed, from which of
the following is it most likely to originate?

Question stats

Score: 81.4%

12.3%

40.2%

1
2
3-5 3 / 3

8.3%

30.1%

9.1%

40.2% of users answered this


question correctly

A. External carotid artery

End and review

8
9
10

B. Retromandibular vein
C. Occipital artery

11

Search

12

D. Maxillary artery

Go

13

E. Ascending pharyngeal artery

14
Next question

15
16

The retromandibular vein lies slightly more deeply than the facial nerve in the parotid gland.
It is formed from the maxillary and superficial temporal vein.

17
18

Retromandibular vein

19
20
21

Formed by a union of the maxillary vein and superficial temporal vein


It descends through the parotid gland and bifurcates within it
The anterior division passes forwards to join the facial vein, the posterior division is
one of the tributaries of the external jugular vein

22
23
24
25

Rate question:

26

Next question

27
Comment on this question

28
29
30
31
32
33
34
35-37 3 / 3
38
39
40
41
42
43
44
45
46
47
48
49
50
51

All contents of this site are 2012 E-Medical Revision Ltd

http://www.emrcs.com/question/question.php?q=0

Terms and Conditions

Privacy policy

1/1

17/05/2015

Reference ranges

Question 52 of 264

Previous

Next

A 52 year female post hysterectomy attends clinic. She reports pain and reduced sensation
over the medial aspect of her thigh. Clinically thigh adduction is weak. What is the most likely
nerve injury?

Question stats

Score: 79.5%

70.8%

5.7%

1
2
3-5 3 / 3

8.5%

7.6%

7.4%

A. Obturator nerve
70.8% of users answered this
question correctly

B. Sciatic nerve

End and review

C. Femoral nerve

8
9
10

D. L3 cord compression

11

Search

E. Deep peroneal nerve

12
Go

13

Next question

14
15

The obturator nerve supplies sensation to the medial aspect of the thigh and causes
adduction and internal rotation of the thigh.
Injury occurs during pelvic or abdominal surgery.
L3 cord compression is unlikely.

16
17
18

Obturator nerve

19
20

The obturator nerve arises from L2, L3 and L4 by branches from the ventral divisions of
each of these nerve roots. L3 forms the main contribution and the second lumbar branch is
occasionally absent. These branches unite in the substance of psoas major, descending
vertically in its posterior part to emerge from its medial border at the lateral margin of the
sacrum. It then crosses the sacroiliac joint to enter the lesser pelvis, it descends on obturator
internus to enter the obturator groove. In the lesser pelvis the nerve lies lateral to the
internal iliac vessels and ureter, and is joined by the obturator vessels lateral to the ovary or
ductus deferens.

21
22
23
24
25
26
27

Supplies

28

Medial compartment of thigh


Muscles supplied: external obturator, adductor longus, adductor brevis, adductor
magnus (not the lower part-sciatic nerve), gracilis
The cutaneous branch is often absent. When present, it passes between gracilis and
adductor longus near the middle part of the thigh, and supplies the skin and fascia of
the distal two thirds of the medial aspect.

29
30
31
32
33
34

Obturator canal

35-37 3 / 3

Connects the pelvis and thigh: contains the obturator artery, vein, nerve which divides
into anterior and posterior branches.

38
39
40

Cadaveric cross section demonstrating relationships of the obturator nerve


41
42
43
44
45
46
47
48
49
50
51
52

Image sourced from Wikipedia

Rate question:

Next question

Comment on this question

All contents of this site are 2012 E-Medical Revision Ltd

http://www.emrcs.com/question/question.php?q=0

Terms and Conditions

Privacy policy

1/1

17/05/2015

Reference ranges

Question 53 of 264

Previous

Next

An ENT surgeon is performing a radical neck dissection. She wishes to fully expose the
external carotid artery. To do so she inserts a self retaining retractor close to its origin.
Which of the following structures lies posterolaterally to the external carotid at this point?

Question stats

Score: 77.8%

25.7%

44.8%

1
2
3-5 3 / 3

9.5%

9.7%

10.3%

A. Superior thyroid artery


44.8% of users answered this
question correctly

B. Internal carotid artery

End and review

C. Lingual artery

8
9
10

D. Facial artery

11

Search

E. None of the above

12
Go

13

Next question

14

The internal carotid artery lies posterolaterally to the external carotid artery at it's origin from
the common carotid. The superior thyroid, lingual and facial arteries all arise from its anterior
surface.

15

External carotid artery

18

16
17

19

The external carotid commences immediately lateral to the pharyngeal side wall. It ascends
and lies anterior to the internal carotid and posterior to the posterior belly of digastric and
stylohyoid. More inferiorly it is covered by sternocleidomastoid, passed by hypoglossal
nerves, lingual and facial veins.
It then pierces the fascia of the parotid gland finally dividing into its terminal branches within
the gland itself.

20
21
22
23
24
25

Surface marking of the carotid


This is an imaginary line drawn from the bifurcation of the common carotid passing behind
the angle of the jaw to a point immediately anterior to the tragus of the ear.

26
27
28

Branches of the external carotid artery


It has six branches, three in front, two behind and one deep.
Three in front

29

Superior thyroid
Lingual
Facial

30
31
32

Two behind

Occipital
Posterior auricular

Deep

Ascending pharyngeal

33
34
35-37 3 / 3
38

It terminates by dividing into the superficial temporal and maxillary arteries in the parotid
gland.

39
40
41
42
43
44
45
46
47
48
49
50
51
52
53

Image sourced from Wikipedia

Rate question:

Next question

Comment on this question

All contents of this site are 2012 E-Medical Revision Ltd

http://www.emrcs.com/question/question.php?q=0

Terms and Conditions

Privacy policy

1/2

17/05/2015
All contents of this site are 2012 E-Medical Revision Ltd

http://www.emrcs.com/question/question.php?q=0

Terms and Conditions

Privacy policy

2/2

17/05/2015

Reference ranges

Previous

Question 54 of 264

Next

In Froment's test which muscle function is tested?

Question stats

End and review

Score: 76.1%

13.7%

16.3%

1
2
3-5 3 / 3

9.5%

A. Flexor pollicis longus

40.4%

B. Abductor pollicis longus

20%

C. Abductor pollicis brevis

40.4% of users answered this


question correctly

D. Adductor pollicis

8
9
10

E. Opponens pollicis

11

Search
Next question

12
Go

13
14
15

Nerve signs

16

Froment's sign

17
18

Assess for ulnar nerve palsy


Adductor pollicis muscle function tested
Hold a piece of paper between their thumb and index finger. The object is then pulled
away. If ulnar nerve palsy, unable to hold the paper and will flex the flexor pollicis
longus to compensate (flexion of thumb at interphalangeal joint).

19
20
21
22
23

Phalen's test

24
25

Assess carpal tunnel syndrome


More sensitive than Tinel's sign
Hold wrist in maximum flexion and the test is positive if there is numbness in the
median nerve distribution.

26
27
28
29
30

Tinel's sign

31
32

Assess for carpal tunnel syndrome


Tap the median nerve at the wrist and the test is positive if there is tingling/electric-like
sensations over the distribution of the median nerve.

33
34
35-37 3 / 3
38

Rate question:

Next question

39
40

Comment on this question

41
42
43
44
45
46
47
48
49
50
51
52
53
54

All contents of this site are 2012 E-Medical Revision Ltd

http://www.emrcs.com/question/question.php?q=0

Terms and Conditions

Privacy policy

1/1

17/05/2015

Reference ranges

Previous

Question 55 of 264

Next

A 22 year old man is involved in a fight outside a nightclub. He is stabbed in the back, on the
left side, approximately 3cm below the 12th rib in the mid scapular line. The structure most
likely to be injured first as a result is the:

Question stats

Score: 76.6%

17.4%

53.3%

1
2
3-5 3 / 3

10%

13%

6.2%

A. Spleen
53.3% of users answered this
question correctly

B. Left kidney

End and review

C. Left adrenal gland

8
9
10

D. Left ureter

11

Search

E. None of the above

12
Go

13

Next question

14
15

The left kidney lies in this location and is the most likely structure to be injured. The Spleen
lies more superiorly, and the left adrenal and ureter are unlikely to be injured in isolation.

16
17

Levels

18
19

Transpyloric plane
Level of the body of L1

20
21

Pylorus stomach
Left kidney hilum (L1- left one!)
Right hilum of the kidney (1.5cm lower than the left)
Fundus of the gallbladder
Neck of pancreas
Duodenojejunal flexure
Superior mesenteric artery
Portal vein
Left and right colic flexure
Root of the transverse mesocolon
2nd part of the duodenum
Upper part of conus medullaris
Spleen

22
23
24
25
26
27
28
29
30
31
32

Can be identified by asking the supine patient to sit up without using their arms. The plane is
located where the lateral border of the rectus muscle crosses the costal margin.

33
34
35-37 3 / 3

Anatomical planes

38

Subcostal plane

Lowest margin of 10th costal cartilage

Intercristal plane

Level of body L4 (highest point of iliac crest)

40

Intertubercular plane

Level of body L5

41

39

42

Common level landmarks

43
44

Inferior mesenteric artery

L3

Bifurcation of aorta into common iliac arteries

L4

Formation of IVC

L5 (union of common iliac veins)

45
46

Diaphragm apertures

47
48

Vena cava T8
Oesophagus T10
Aortic hiatus T12

49
50
51
52

Rate question:

53

Next question

54
Comment on this question

55

All contents of this site are 2012 E-Medical Revision Ltd

http://www.emrcs.com/question/question.php?q=0

Terms and Conditions

Privacy policy

1/1

17/05/2015

Reference ranges

Previous3 / 3

Question 56-58 of 264

Next

Question stats

End and review

Score: 78%

Average score for registered users:

1
2

Theme: Cutaneous innervation


A. Ulnar nerve

56

83.8%

57

55.7%

58

81.1%

3-5 3 / 3
6

B. Fifth cervical spinal segment

C. Radial nerve

D. Musculocutaneous nerve

Search

E. Median nerve

Go

F. None of these

10
11

Please select the source of innervation for the region described. Each option may be used
once, more than once or not at all.

12
13
14
15

56.

The skin on the palmar aspect of the thumb

16
17

Median nerve

18

The median nerve supplies cutaneous sensation to this region.


See diagram below

19
20

57.

The nail bed of the index finger

21
22

Median nerve

23
24

58.

The skin overlying the medial aspect of the palm

25
26

Ulnar nerve

27

This area is innervated by the ulnar nerve.


28
29
Next question

30
31
32

Brachial plexus

33
34

Origin

Anterior rami of C5 to T1

35-37 3 / 3
38

Sections of the
plexus

Roots, trunks, divisions, cords, branches


Mnemonic:Real Teenagers Drink Cold Beer

39
40
41

Roots

Trunks

Located in the posterior triangle


Pass between scalenus anterior and medius

42
43
44

Located posterior to middle third of clavicle


Upper and middle trunks related superiorly to the subclavian
artery
Lower trunk passes over 1st rib posterior to the subclavian
artery

45
46
47
48

Divisions

Apex of axilla

Cords

Related to axillary artery

49
50
51
52

Diagram illustrating the branches of the brachial plexus


53

http://www.emrcs.com/question/question.php?q=0

1/2

17/05/2015
54
55
56-58 3 / 3

Image sourced from Wikipedia

Cutaneous sensation of the upper limb

Image sourced from Wikipedia

Rate question:

Next question

Comment on this question

All contents of this site are 2012 E-Medical Revision Ltd

http://www.emrcs.com/question/question.php?q=0

Terms and Conditions

Privacy policy

2/2

17/05/2015

Reference ranges

Previous

Question 59 of 264

Next

With regard to the sciatic nerve which statement is false?

Question stats

End and review

Score: 77.2%

12.2%

47.3%

1
2
3-5 3 / 3

7%

A. It is derived from L4 to S3

11.9%

B. It contains nerve roots from the posterior division of the lumbosacral


plexus only

21.7%

47.3% of users answered this


question correctly

C. It divides to give the tibial nerve and common peroneal nerve


D. It provides cutaneous sensation to the posterior aspect of the thigh

8
9
10

E. It provides cutaneous sensation to the entire lower leg with the


exception of its medial aspect

11

Search

12
Go

13

Next question

14

It is derived from both anterior and posterior divisions of the lumbosacral plexus. The sciatic
nerve is the longest and widest nerve in the human body. It is particularly susceptible to
trauma in the posterior approach to the hip.

15
16
17
18

Sciatic nerve

19

The sciatic nerve is formed from the sacral plexus and is the largest nerve in the body. It is
the continuation of the main part of the plexus arising from ventral rami of L4 to S3. These
rami converge at the inferior border of piriformis to form the nerve itself. It passes through
the inferior part of the greater sciatic foramen and emerges beneath piriformis. Medially, lie
the inferior gluteal nerve and vessels and the pudendal nerve and vessels. It runs
inferolaterally under the cover of gluteus maximus midway between the greater trochanter
and ischial tuberosity. It receives its blood supply from the inferior gluteal artery. The nerve
provides cutaneous sensation to the skin of the foot and the leg. It also innervates the
posterior thigh muscles and the lower leg and foot muscles. The nerve splits into the tibial
and common peroneal nerves approximately half way down the posterior thigh. The tibial
nerve supplies the flexor muscles and the common peroneal nerve supplies the extensor
muscles and the abductor muscles.

20
21
22
23
24
25
26
27
28
29

Summary points

30

Origin

Spinal nerves L4 - S3

Articular Branches

Hip joint

31
32
33

Muscular branches in
upper leg

Semitendinosus
Semimembranosus
Biceps femoris
Part of adductor magnus

34
35-37 3 / 3
38
39

Cutaneous sensation

40

Posterior aspect of thigh


Gluteal region
Entire lower leg (except the medial aspect)

41
42

Terminates

43

At the upper part of the popliteal fossa by dividing into the tibial
and peroneal nerves

44
45
46

The nerve to the short head of the biceps femoris comes from the common peroneal
part of the sciatic and the other muscular branches arise from the tibial portion.
The tibial nerve goes on to innervate all muscles of the foot except the extensor
digitorum brevis (which is innervated by the common peroneal nerve).

47
48
49
50
51

Rate question:

Next question

52

Comment on this question

53
54
55
56-58 3 / 3
59

All contents of this site are 2012 E-Medical Revision Ltd

http://www.emrcs.com/question/question.php?q=0

Terms and Conditions

Privacy policy

1/2

17/05/2015

http://www.emrcs.com/question/question.php?q=0

2/2

17/05/2015

Reference ranges

Question 60 of 264

Previous

Next

A 28 year old man has a pleomorphic adenoma and the decision is made to resect this
surgically. Which of the following structures is least likely to be encountered during surgical
resection of the parotid gland?

Question stats

Score: 77.7%

26.6%

11.6%

1
2
3-5 3 / 3

16.9%

31.9%

13%

A. External carotid artery


31.9% of users answered this
question correctly

B. Retromandibular vein

End and review

C. Auriculotemporal nerve

8
9
10

D. Mandibular nerve

11

Search

E. Zygomatic branch of the facial nerve

12
Go

Next question

13
14
15

Structures passing through the parotid gland

16

Facial nerve and branches


External carotid artery (and its branches; the maxillary and superficial temporal)
Retromandibular vein
Auriculotemporal nerve

17
18
19
20
21

The mandibular nerve is well separated from the parotid gland.


The maxillary vein joins to the superficial temporal vein and they form the retromandibular
vein which then runs through the parotid gland.
The auriculotemporal nerve runs through the gland. Following a parotidectomy this nerve
may be damaged and during neuronal regrowth may then attach to sweat glands in this
region. This can then cause gustatory sweating (Freys Syndrome).

22
23
24
25
26

Parotid gland

27
28

Anatomy of the parotid gland

29

Location

Overlying the mandibular ramus; anterior and inferior to the ear.

Salivary duct

Crosses the masseter, pierces the buccinator and drains adjacent to the
2nd upper molar tooth (Stensen's duct).

30
31
32
33

Structures passing
through the gland

Facial nerve (Mnemonic: The Zebra Buggered My Cat; Temporal


Zygomatic, Buccal, Mandibular, Cervical)
External carotid artery
Retromandibular vein
Auriculotemporal nerve

34
35-37 3 / 3
38
39
40

Relations

Anterior: masseter, medial pterygoid, superficial temporal and


maxillary artery, facial nerve, stylomandibular ligament
Posterior: posterior belly digastric muscle, sternocleidomastoid,
stylohyoid, internal carotid artery, mastoid process, styloid
process

41
42
43
44

Arterial supply

Branches of external carotid artery

Venous drainage

Retromandibular vein

Lymphatic
drainage

Deep cervical nodes

45
46
47
48
49

Nerve innervation

50

Parasympathetic-Secretomotor
Sympathetic-Superior cervical ganglion
Sensory- Greater auricular nerve

51
52
53

Parasympathetic stimulation produces a water rich, serous saliva. Sympathetic stimulation


leads to the production of a low volume, enzyme-rich saliva.

54
55
56-58 3 / 3

Rate question:
Comment on this question

http://www.emrcs.com/question/question.php?q=0

Next question

59
60

1/2

17/05/2015
All contents of this site are 2012 E-Medical Revision Ltd

http://www.emrcs.com/question/question.php?q=0

Terms and Conditions

Privacy policy

2/2

17/05/2015

Reference ranges

Question 61 of 264

Previous

Next

A 23 year old man is undergoing a hernia repair and the mesh is to be sutured to the
inguinal ligament. From which of the following does the inguinal ligament arise?

A. Transversus abdominis fascia


B. Internal oblique

Question stats

Score: 76.9%

15.4%

9.9%

1
2
3-5 3 / 3

9.7%

7.8%

57.1%

57.1% of users answered this


question correctly

C. Rectus sheath

End and review

D. Rectus abdominis muscle

8
9
10

E. External oblique aponeurosis

11

Search

12
Next question

The inguinal ligament is formed by the external oblique aponeurosis. It runs from the pubic
tubercle to the anterior superior iliac spine.

Go

13
14
15
16

Abdominal wall

17

The 2 main muscles of the abdominal wall are the rectus abdominis (anterior) and the
quadratus lumborum (posterior).
The remaining abdominal wall consists of 3 muscular layers. Each muscle passes from the
lateral aspect of the quadratus lumborum posteriorly to the lateral margin of the rectus
sheath anteriorly. Each layer is muscular posterolaterally and aponeurotic anteriorly.

18
19
20
21
22
23
24
25
26
27

Image sourced from Wikipedia

Muscles of abdominal wall


External
oblique

Lies most superficially


Originates from 5th to 12th ribs
Inserts into the anterior half of the outer aspect of the iliac crest, linea
alba and pubic tubercle
More medially and superiorly to the arcuate line, the aponeurotic layer
overlaps the rectus abdominis muscle
The lower border forms the inguinal ligament
The triangular expansion of the medial end of the inguinal ligament is the
lacunar ligament.

28
29
30
31
32
33
34
35-37 3 / 3
38
39

Internal
oblique

Arises from the thoracolumbar fascia, the anterior 2/3 of the iliac crest
and the lateral 2/3 of the inguinal ligament
The muscle sweeps upwards to insert into the cartilages of the lower 3
ribs
The lower fibres form an aponeurosis that runs from the tenth costal
cartilage to the body of the pubis
At its lowermost aspect it joins the fibres of the aponeurosis of
transversus abdominis to form the conjoint tendon.

40
41
42
43
44
45

Transversus
abdominis

46

Innermost muscle
Arises from the inner aspect of the costal cartilages of the lower 6 ribs ,
from the anterior 2/3 of the iliac crest and lateral 1/3 of the inguinal
ligament
Its fibres run horizontally around the abdominal wall ending in an
aponeurosis. The upper part runs posterior to the rectus abdominis. Lower
down the fibres run anteriorly only.
The rectus abdominis lies medially; running from the pubic crest and
symphysis to insert into the xiphoid process and 5th, 6th and 7th costal
cartilages. The muscles lies in a aponeurosis as described above.
Nerve supply: anterior primary rami of T7-12

47
48
49
50
51
52
53
54
55

Surgical notes
During abdominal surgery it is usually necessary to divide either the muscles or their
aponeuroses. During a midline laparotomy it is desirable to divide the aponeurosis. This will
leave the rectus sheath intact above the arcuate line and the muscles intact below it.
Straying off the midline will often lead to damage to the rectus muscles, particularly below the
arcuate line where they may often be in close proximity to each other.

http://www.emrcs.com/question/question.php?q=0

56-58 3 / 3
59
60
61

1/2

17/05/2015

Rate question:

Next question

Comment on this question

All contents of this site are 2012 E-Medical Revision Ltd

http://www.emrcs.com/question/question.php?q=0

Terms and Conditions

Privacy policy

2/2

17/05/2015

Reference ranges

Previous

Question 62 of 264

Next

A 56 year old man is undergoing a carotid endarterectomy. The internal carotid artery is
mobilised. How many branches does this vessel give off in the neck?

A. 0
B. 1

Question stats

Score: 77.1%

54.6%

7.8%

1
2
3-5 3 / 3

9.7%

14.8%

13.1%

54.6% of users answered this


question correctly

C. 2

End and review

D. 3

8
9
10

E. 6

11

Search

12
Next question

The internal carotid does not have any branches in the neck.

Go

13
14
15

Internal carotid artery

16
17

The internal carotid artery is formed from the common carotid opposite the upper border of
the thyroid cartilage. It extends superiorly to enter the skull via the carotid canal. From the
carotid canal it then passes through the cavernous sinus, above which it divides into the
anterior and middle cerebral arteries.
Relations in the neck
Posterior

18
19
20
21
22

Longus capitis
Pre-vertebral fascia
Sympathetic chain
Superior laryngeal nerve

23
24
25
26

Medially

External carotid (near origin)


Wall of pharynx
Ascending pharyngeal artery

27
28
29

Laterally

30

Internal jugular vein (moves posteriorly at entrance to skull)


Vagus nerve (most posterolaterally)

31
32

Anteriorly

Sternocleidomastoid
Lingual and facial veins
Hypoglossal nerve

33
34
35-37 3 / 3
38

Relations in the carotid canal


Internal carotid plexus
Cochlea and middle ear cavity
Trigeminal ganglion (superiorly)
Leaves canal lies above the foramen lacerum

39
40
41
42
43
44

Path and relations in the cranial cavity


The artery bends sharply forwards in the cavernous sinus, the aducens nerve lies close to its
inferolateral aspect. The oculomotor, trochlear, opthalmic and, usually, the maxillary nerves
lie in the lateral wall of the sinus. Near the superior orbital fissure it turns posteriorly and
passes postero-medially to pierce the roof of the cavernous sinus inferior to the optic nerve.
It then passes between the optic and oculomotor nerves to terminate below the anterior
perforated substance by dividing into the anterior and middle cerebral arteries.

45
46
47
48
49
50

Branches
Anterior and middle cerebral artery
Ophthalmic artery
Posterior communicating artery
Anterior choroid artery
Meningeal arteries
Hypophyseal arteries

51
52
53
54
55
56-58 3 / 3
59

Image demonstrating the internal carotid artery and its relationship to the external carotid
artery

http://www.emrcs.com/question/question.php?q=0

60
61

1/2

17/05/2015
61
62

Image sourced from Wikipedia

Rate question:

Next question

Comment on this question

All contents of this site are 2012 E-Medical Revision Ltd

http://www.emrcs.com/question/question.php?q=0

Terms and Conditions

Privacy policy

2/2

17/05/2015

Reference ranges

Question 63 of 264

Previous

Next

Which of the following is a content of the adductor canal?

Question stats

End and review

Score: 77.6%

34.8%

9.6%

1
2
3-5 3 / 3

20.7%

A. Saphenous nerve

18.6%

B. Sural nerve

16.3%

C. Femoral nerve

34.8% of users answered this


question correctly

D. Profunda branch of the femoral artery

8
9
10

E. Saphenous vein

11

Search
Next question

12
Go

13

It contains the saphenous nerve and the superficial branch of the femoral artery.

14
15

Adductor canal

16

Also called Hunter's or subsartorial canal


Immediately distal to the apex of the femoral triangle, lying in the middle third of the
thigh. Canal terminates at the adductor hiatus.

17
18
19
20

Borders

Contents

Laterally Vastus medialis muscle

Saphenous nerve

Posteriorly Adductor longus, adductor magnus

Superficial femoral artery

Roof Sartorius

Superficial femoral vein

21
22
23
24
25
26

In the image below the sartorius muscle is removed to expose the canal contents

27
28
29
30
31
32
33
34
35-37 3 / 3
38
39
40
41
42
43
44
45
46
47
48

Image sourced from Wikipedia

49
50

Rate question:

Next question

51
52

Comment on this question

53
54
55
56-58 3 / 3
59
60
61

http://www.emrcs.com/question/question.php?q=0

1/2

17/05/2015
62
63

All contents of this site are 2012 E-Medical Revision Ltd

http://www.emrcs.com/question/question.php?q=0

Terms and Conditions

Privacy policy

2/2

17/05/2015

Reference ranges

Previous

Question 64 of 264

Next

A 56 year old man is undergoing a high anterior resection. Which of the following structures
is at greatest risk of injury in this procedure?

A. Superior mesenteric artery


B. Left ureter

Question stats

Score: 78%

10.8%

58.5%

1
2
3-5 3 / 3

11.1%

10.8%

8.8%

58.5% of users answered this


question correctly

C. External iliac vein

End and review

D. External iliac artery

8
9
10

E. Inferior vena cava

11

Search

12
Next question

A careless surgeon may damage all of these structures. However, the structure at greatest
risk and most frequently encountered is the left ureter.

Go

13
14
15
16

Colon anatomy
The colon commences with the caecum. This represents the most dilated segment of the
human colon and its base (which is intraperitoneal) is marked by the convergence of teniae
coli. At this point is located the vermiform appendix. The colon continues as the ascending
colon, the posterior aspect of which is retroperitoneal. The line of demarcation between the
intra and retro peritoneal right colon is visible as a white line, in the living, and forms the line
of incision for colonic resections.

17
18
19
20
21
22
23

The ascending colon becomes the transverse colon after passing the hepatic flexure. At this
located the colon becomes wholly intra peritoneal once again. The superior aspect of the
transverse colon is the point of attachment of the transverse colon to the greater omentum.
This is an important anatomical site since division of these attachments permits entry into the
lesser sac. Separation of the greater omentum from the transverse colon is a routine
operative step in both gastric and colonic resections.

24
25
26
27
28

At the left side of the abdomen the transverse colon passes to the left upper quadrant and
makes an oblique inferior turn at the splenic flexure. Following this, the posterior aspect
becomes retroperitoneal once again.

29
30
31

At the level of approximately L4 the descending colon becomes wholly intraperitoneal and
becomes the sigmoid colon. Whilst the sigmoid is wholly intraperitoneal there are usually
attachments laterally between the sigmoid and the lateral pelvic sidewall. These small
congenital adhesions are not formal anatomical attachments but frequently require division
during surgical resections.

32
33
34
35-37 3 / 3
38

At its distal end the sigmoid passes to the midline and at the region around the sacral
promontary it becomes the upper rectum. This transition is visible macroscopically as the
point where the teniae fuse. More distally the rectum passes through the peritoneum at the
region of the peritoneal reflection and becomes extraperitoneal.
Arterial supply
Superior mesenteric artery and inferior mesenteric artery: linked by the marginal artery.
Ascending colon: ileocolic and right colic arteries
Transverse colon: middle colic artery
Descending and sigmoid colon: inferior mesenteric artery

39
40
41
42
43
44
45
46

Venous drainage
From regional veins (that accompany arteries) to superior and inferior mesenteric vein

47
48

Lymphatic drainage
Initially along nodal chains that accompany supplying arteries, then para-aortic nodes.

49

Embryology
Midgut- Second part of duodenum to 2/3 transverse colon
Hindgut- Distal 1/3 transverse colon to anus

51

Peritoneal location
The right and left colon are part intraperitoneal and part extraperitoneal. The sigmoid and
transverse colon are generally wholly intraperitoneal. This has implications for the sequelae
of perforations, which will tend to result in generalised peritonitis in the wholly intra peritoneal
segments.

54

50

52
53

55
56-58 3 / 3
59
60

Colonic relations

61
62

http://www.emrcs.com/question/question.php?q=0

1/2

17/05/2015
62

Region of colon

Relation

Caecum/ right colon

Right ureter, gonadal vessels

Hepatic flexure

Gallbladder (medially)

Splenic flexure

Spleen and tail of pancreas

Distal sigmoid/ upper


rectum

Left ureter

Rectum

Ureters, autonomic nerves, seminal vesicles, prostate, urethra


(distally)

63
64

Rate question:

Next question

Comment on this question

All contents of this site are 2012 E-Medical Revision Ltd

http://www.emrcs.com/question/question.php?q=0

Terms and Conditions

Privacy policy

2/2

17/05/2015

Reference ranges

Question 65 of 264

Previous

Next

From which of these foraminae does the opthalmic branch of the trigeminal nerve exit the
skull?

A. Foramen ovale
B. Foramen rotundum

Question stats

Score: 78.2%

11.3%

10.7%

1
2
3-5 3 / 3

9.2%

61.5%

7.3%

61.5% of users answered this


question correctly

C. Foramen spinosum

End and review

D. Superior orbital fissure

8
9
10

E. Foramen magnum

11

Search

12
Next question

Go

13
14

Mnemonic:

15

Standing Room Only -Exit of branches of trigeminal nerve from the skull

16
17

V1 -Superior orbital fissure


V2 -foramen Rotundum
V3 -foramen Ovale

18
19
20

The opthalmic branch of the trigeminal nerve exits the skull through the superior orbital
fissure.

21
22
23

Trigeminal nerve

24

The trigeminal nerve is the main sensory nerve of the head. In addition to its major sensory
role, it also innervates the muscles of mastication.

25
26
27

Distribution of the trigeminal nerve


Sensory

28

Scalp
Face
Oral cavity (and teeth)
Nose and sinuses
Dura mater

29
30
31
32
33

Motor

Muscles of mastication
Mylohyoid
Anterior belly of digastric
Tensor tympani
Tensor palati

34
35-37 3 / 3
38
39
40

Autonomic connections (ganglia)

Ciliary
Sphenopalatine
Otic
Submandibular

41
42
43
44
45

Path

46

Originates at the pons


Sensory root forms the large, crescentic trigeminal ganglion within Meckel's cave, and
contains the cell bodies of incoming sensory nerve fibres. Here the 3 branches exit.
The motor root cell bodies are in the pons and the motor fibres are distributed via the
mandibular nerve. The motor root is not part of the trigeminal ganglion.

47
48
49
50
51

Branches of the trigeminal nerve


Ophthalmic nerve

Sensory only

Maxillary nerve

Sensory only

52
53
54

Mandibular nerve

Sensory and motor

55
56-58 3 / 3
59
60

Sensory
Ophthalmic

Exits skull via the superior orbital fissure

http://www.emrcs.com/question/question.php?q=0

61

1/2

17/05/2015

Sensation of: scalp and forehead, the upper eyelid, the conjunctiva and cornea of
the eye, the nose (including the tip of the nose, except alae nasi), the nasal
mucosa, the frontal sinuses, and parts of the meninges (the dura and blood
vessels).

62

Maxillary
nerve

Exit skull via the foramen rotundum


Sensation: lower eyelid and cheek, the nares and upper lip, the upper teeth and
gums, the nasal mucosa, the palate and roof of the pharynx, the maxillary,
ethmoid and sphenoid sinuses, and parts of the meninges.

65

Mandibular
nerve

Exit skull via the foramen ovale


Sensation: lower lip, the lower teeth and gums, the chin and jaw (except the
angle of the jaw), parts of the external ear, and parts of the meninges.

63
64

Motor
Distributed via the mandibular nerve.
The following muscles of mastication are innervated:
Masseter
Temporalis
Medial pterygoid
Lateral pterygoid

Other muscles innervated include:


Tensor veli palatini
Mylohyoid
Anterior belly of digastric
Tensor tympani

Rate question:

Next question

Comment on this question

All contents of this site are 2012 E-Medical Revision Ltd

http://www.emrcs.com/question/question.php?q=0

Terms and Conditions

Privacy policy

2/2

17/05/2015

Reference ranges

Previous

Question 66 of 264

Next

A 56 year old lady with metastatic breast cancer develops an oestolytic deposit in the
proximal femur. One morning whilst getting out of bed she notices severe groin pain. X-rays
show that the lesser trochanter has been avulsed. Which muscle is the most likely culprit?

Question stats

Score: 78.6%

10.6%

43.1%

1
2
3-5 3 / 3

18.2%

12.5%

15.7%

A. Vastus lateralis
43.1% of users answered this
question correctly

B. Psoas major

End and review

C. Piriformis

8
9
10

D. Gluteus maximus

11

Search

E. Gluteus medius

12
Go

Next question

13
14

The psoas major inserts into the lesser trochanter and contracts when raising the trunk from
the supine position. When oestolytic lesions are present in the femur the lesser trochanter
may be avulsed.

15

Psoas Muscle

18

16
17

19

Origin
The deep part originates from the transverse processes of the five lumbar vertebrae, the
superficial part originates from T12 and the first 4 lumbar vertebrae.

20
21
22

Insertion
Lesser trochanter of the femur.

23
24

Innervation
Anterior rami of L1 to L3.

25

Action
Flexion and external rotation of the hip. Bilateral contraction can raise the trunk from the
supine position.

27

26

28
29
30

Rate question:

Next question

31
32

Comment on this question

33
34
35-37 3 / 3
38
39
40
41
42
43
44
45
46
47
48
49
50
51
52
53
54
55
56-58 3 / 3
59
60
61

http://www.emrcs.com/question/question.php?q=0

1/2

17/05/2015
62
63
64
65
66

All contents of this site are 2012 E-Medical Revision Ltd

http://www.emrcs.com/question/question.php?q=0

Terms and Conditions

Privacy policy

2/2

17/05/2015

Reference ranges

Previous

Question 67 of 264

Next

Which of the following nerves is responsible for the motor innervation of the
sternocleidomastoid muscle?

A. Ansa cervicalis
B. Accessory nerve

Question stats

Score: 78.9%

12.4%

61%

1
2
3-5 3 / 3

9.3%

8.8%

8.4%

61% of users answered this


question correctly

C. Hypoglossal nerve

End and review

D. Facial nerve

8
9
10

E. Vagus nerve

11

Search

12
Next question

Theme from January 2013 Exam


The motor supply to the sternocleidomastoid is from the accessory nerve. The ansa
cervicalis supplies sensory information from the muscle.

Go

13
14
15
16
17

Sternocleidomastoid

18
19

Anatomy
Origin

Insertion

Innervation

Rounded tendon attached to upper manubrium sterni and muscular head


attached to medial third of the clavicle

20

Mastoid process of the temporal bone and lateral area of the superior nuchal line
of the occipital bone

22

Spinal part of accessory nerve and anterior rami of C2 and C3 (proprioception)

24

21

23

25

Action

Both: extend the head at atlanto-occipital joint and flex the cervical
vertebral column. Accessory muscles of inspiration.
Single: lateral flexion of neck, rotates head so face looks upward to the
opposite side

26
27
28
29
30

Sternocleidomastoid divides the anterior and posterior triangles of the neck.

31

Rate question:
Comment on this question

Next question

32
33
34
35-37 3 / 3
38
39
40
41
42
43
44
45
46
47
48
49
50
51
52
53
54
55
56-58 3 / 3
59
60
61

http://www.emrcs.com/question/question.php?q=0

1/2

17/05/2015
62
63
64
65
66
67

All contents of this site are 2012 E-Medical Revision Ltd

http://www.emrcs.com/question/question.php?q=0

Terms and Conditions

Privacy policy

2/2

17/05/2015

Reference ranges

Previous

Question 68 of 264

Next

A 42 year old lady undergoes a difficult cholecystectomy and significant bleeding is


occurring. The surgeons place a vascular clamp transversely across the anterior border of
the epiploic foramen. Which of the following structures will be occluded in this manoeuvre?

Question stats

Score: 77.6%

20.9%

9.5%

1
2
3-5 3 / 3

12.5%

47.1%

10%

A. Cystic artery
47.1% of users answered this
question correctly

B. Cystic duct

End and review

C. Left gastric artery

8
9
10

D. Portal vein

11

Search

E. None of the above

12
Go

Next question

13
14
15

The portal vein, hepatic artery and common bile duct are occluded.

16

Epiploic Foramen

17
18

The epiploic foramen has the following boundaries:

19

Anteriorly (in the free edge of the


lesser omentum)

Bile duct to the right, portal vein behind and hepatic


artery to the left.

Posteriorly

Inferior vena cava

Inferiorly

1st part of the duodenum

23

Superiorly

Caudate process of the liver

24

20
21
22

25

During liver surgery bleeding may be controlled using a Pringles manoeuvre, this involves
placing a vascular clamp across the anterior aspect of the epiploic foramen. Thereby
occluding:
Common bile duct
Hepatic artery
Portal vein

26
27
28
29
30
31
32

Rate question:

33
Next question

34
Comment on this question

35-37 3 / 3
38
39
40
41
42
43
44
45
46
47
48
49
50
51
52
53
54
55
56-58 3 / 3
59
60
61

http://www.emrcs.com/question/question.php?q=0

1/2

17/05/2015
62
63
64
65
66
67
68

All contents of this site are 2012 E-Medical Revision Ltd

http://www.emrcs.com/question/question.php?q=0

Terms and Conditions

Privacy policy

2/2

17/05/2015

Reference ranges

Previous

Question 69 of 264

Next

A 34 year old man is injured by farm machinery and sustains a laceration at the superolateral
aspect of the popliteal fossa. The medial aspect of biceps femoris is lacerated. Which of the
following underlying structures is at greatest risk of injury?

Question stats

Score: 78%

8.2%

13.3%

1
2
3-5 3 / 3

10.2%

13.7%

54.6%

A. Gracilis
54.6% of users answered this
question correctly

B. Sural nerve

End and review

C. Nerve to semimembranosus

8
9
10

D. Popliteal artery

11

Search

E. Common peroneal nerve

12
Go

Next question

13
14

The common peroneal nerve lies under the medial aspect of biceps femoris and is therefore
at greatest risk of injury. The tibial nerve may also be damaged in such an injury (but is not
listed here). The sural nerve branches off more inferiorly.

15

Common peroneal nerve

18

16
17

19

Derived from the dorsal divisions of the sacral plexus (L4, L5, S1 and S2).

20

This nerve supplies the skin and fascia of the anterolateral surface of the leg and the
dorsum of the foot. It also innervates the muscles of the anterior and peroneal compartments
of the leg, extensor digitorum brevis as well as the knee, ankle and foot joints.

21

It is laterally placed within the sciatic nerve. From the bifurcation of the sciatic nerve it passes
inferolaterally in the lateral and proximal part of the popliteal fossa, under the cover of biceps
femoris and its tendon. To reach the posterior aspect of the fibular head. It ends by dividing
into the deep and superficial peroneal nerves at the point where it winds around the lateral
surface of the neck of the fibula in the body of peroneus longus, approximately 2cm distal to
the apex of the head of the fibula. It is palpable posterior to the head of the fibula.

24

22
23

25
26
27
28
29

Branches
30

In the thigh

Nerve to the short head of biceps


Articular branch (knee)

In the popliteal fossa

Lateral cutaneous nerve of the calf

Neck of fibula

Superficial and deep peroneal nerves

31
32
33
34
35-37 3 / 3
38

Rate question:

Next question

39
40

Comment on this question

41
42
43
44
45
46
47
48
49
50
51
52
53
54
55
56-58 3 / 3
59
60
61

http://www.emrcs.com/question/question.php?q=0

1/2

17/05/2015
62
63
64
65
66
67
68
69

All contents of this site are 2012 E-Medical Revision Ltd

http://www.emrcs.com/question/question.php?q=0

Terms and Conditions

Privacy policy

2/2

17/05/2015

Reference ranges

Previous

Question 70 of 264

Next

A 56 year old lady undergoes a Hartmans style resection of the sigmoid colon, with ligation of
the vessels close to the colon. Which of the following vessels will be responsible to supplying
the rectal stump directly?

Question stats

Score: 76.7%

6.4%

8.5%

1
2
3-5 3 / 3

49.1%

23.6%

12.4%

A. Superior mesenteric artery


49.1% of users answered this
question correctly

B. Middle colic artery

End and review

C. Superior rectal artery

8
9
10

D. Inferior mesenteric artery

11

Search

E. External iliac artery

12
Go

Next question

13
14

This question is addressing the blood supply to the rectum. Which is supplied by the superior
rectal artery. High ligation of the IMA may compromise this structure. However, the question
states that during the Hartmans procedure the vessels were ligated close to the bowel.
Implying that the superior rectal was preserved.

15
16
17
18

Rectum

19
20

The rectum is approximately 12 cm long. It is a capacitance organ. It has both intra and
extraperitoneal components. The transition between the sigmoid colon is marked by the
disappearance of the tenia coli.The extra peritoneal rectum is surrounded by mesorectal fat
that also contains lymph nodes. This mesorectal fatty layer is removed surgically during
rectal cancer surgery (Total Mesorectal Excision). The fascial layers that surround the
rectum are important clinical landmarks, anteriorly lies the fascia of Denonvilliers. Posteriorly
lies Waldeyers fascia.

21
22
23
24
25
26

Extra peritoneal rectum


27

Posterior upper third


Posterior and lateral middle third
Whole lower third

28
29
30
31

Relations
Anteriorly (Males)

32

Rectovesical pouch
Bladder
Prostate
Seminal vesicles

33
34
35-37 3 / 3

Anteriorly (Females)

Recto-uterine pouch (Douglas)


Cervix
Vaginal wall

38
39
40

Posteriorly

Laterally

Sacrum
Coccyx
Middle sacral artery

41
42
43

Levator ani
Coccygeus

44
45

Arterial supply
Superior rectal artery

46
47

Venous drainage
Superior rectal vein

48

Lymphatic drainage

50

49

51

Mesorectal lymph nodes (superior to dentate line)


Internal iliac and then para-aortic nodes
Inguinal nodes (inferior to dentate line)

52
53
54
55

Rate question:

Next question

56-58 3 / 3
59

Comment on this question

60
61

http://www.emrcs.com/question/question.php?q=0

1/2

17/05/2015
62
63
64
65
66
67
68
69
70

All contents of this site are 2012 E-Medical Revision Ltd

http://www.emrcs.com/question/question.php?q=0

Terms and Conditions

Privacy policy

2/2

17/05/2015

Reference ranges

Previous

Question 72 of 264

Next

Which option is false in relation to the trigeminal nerve?

Question stats

End and review

Score: 77.4%

9.4%

46.6%

1
2
3-5 3 / 3

9.9%

A. The nerve originates at the pons

17.9%

B. The posterior scalp is supplied by the trigeminal nerve

16.2%

C. The maxillary nerve exits via the foramen rotundum

46.6% of users answered this


question correctly

D. The maxillary nerve is purely sensory

8
9
10

E. The motor root is not in the trigeminal ganglion

11

Search
Next question

12
Go

13

The posterior scalp is supplied by C2-C3.

14
15

Trigeminal nerve

16

The trigeminal nerve is the main sensory nerve of the head. In addition to its major sensory
role, it also innervates the muscles of mastication.

17
18
19

Distribution of the trigeminal nerve

20

Sensory

Scalp
Face
Oral cavity (and teeth)
Nose and sinuses
Dura mater

21
22
23
24

Motor

Muscles of mastication
Mylohyoid
Anterior belly of digastric
Tensor tympani
Tensor palati

25
26
27
28
29

Autonomic connections (ganglia)

Ciliary
Sphenopalatine
Otic
Submandibular

30
31
32
33
34
35-37 3 / 3

Path

38

Originates at the pons


Sensory root forms the large, crescentic trigeminal ganglion within Meckel's cave, and
contains the cell bodies of incoming sensory nerve fibres. Here the 3 branches exit.
The motor root cell bodies are in the pons and the motor fibres are distributed via the
mandibular nerve. The motor root is not part of the trigeminal ganglion.

39
40
41
42
43

Branches of the trigeminal nerve

44

Ophthalmic nerve

Sensory only

45

Maxillary nerve

Sensory only

46

Mandibular nerve

Sensory and motor

47
48
49

Sensory

50

Ophthalmic

Maxillary
nerve

Mandibular
nerve

Exits skull via the superior orbital fissure


Sensation of: scalp and forehead, the upper eyelid, the conjunctiva and cornea of
the eye, the nose (including the tip of the nose, except alae nasi), the nasal
mucosa, the frontal sinuses, and parts of the meninges (the dura and blood
vessels).
Exit skull via the foramen rotundum
Sensation: lower eyelid and cheek, the nares and upper lip, the upper teeth and
gums, the nasal mucosa, the palate and roof of the pharynx, the maxillary,
ethmoid and sphenoid sinuses, and parts of the meninges.
Exit skull via the foramen ovale
Sensation: lower lip, the lower teeth and gums, the chin and jaw (except the
angle of the jaw), parts of the external ear, and parts of the meninges.

51
52
53
54
55
56-58 3 / 3
59
60
61
62

http://www.emrcs.com/question/question.php?q=0

1/2

17/05/2015
62
63

Motor
Distributed via the mandibular nerve.
The following muscles of mastication are innervated:

64
65

Masseter
Temporalis
Medial pterygoid
Lateral pterygoid

66
67
68
69
70

Other muscles innervated include:

71

Tensor veli palatini


Mylohyoid
Anterior belly of digastric
Tensor tympani

Rate question:

72

Next question

Comment on this question

All contents of this site are 2012 E-Medical Revision Ltd

http://www.emrcs.com/question/question.php?q=0

Terms and Conditions

Privacy policy

2/2

17/05/2015

Reference ranges

Question 73 of 264

Previous

Next

A 45 year old man is undergoing a lymph node biopsy from the posterior triangle of his neck.
Which structure forms the posterior border of this region?

A. Trapezius muscle
B. Diagastric muscle

Question stats

Score: 77.8%

68.1%

8.2%

1
2
3-5 3 / 3

7.1%

6.9%

9.8%

68.1% of users answered this


question correctly

C. External jugular vein

End and review

D. Omohyoid muscle

8
9
10

E. Sternocleidomastoid muscle

11

Search

12
Next question

Go

13
14

The borders are described below.

15

Posterior triangle of the neck

16
17

Boundaries
18

Apex

Sternocleidomastoid and the Trapezius muscles at the Occipital bone

Anterior

Posterior border of the Sternocleidomastoid

Posterior

Anterior border of the Trapezius

Base

Middle third of the clavicle

19
20
21
22
23
24
25
26
27
28
29
30
31
32
33
34

Image sourced from Wikipedia

35-37 3 / 3
38

Contents
Nerves

39

Accessory nerve
Phrenic nerve
Three trunks of the brachial plexus
Branches of the cervical plexus: Supraclavicular nerve, transverse cervical
nerve, great auricular nerve, lesser occipital nerve

40
41
42
43

Vessels

44

External jugular vein


Subclavian artery

45
46

Muscles

47

Inferior belly of omohyoid


Scalene

48
49

Lymph
nodes

50

Supraclavicular
Occipital

51
52
53

Rate question:

Next question

54
55

Comment on this question

56-58 3 / 3
59
60
61

http://www.emrcs.com/question/question.php?q=0

1/2

17/05/2015
62
63
64
65
66
67
68
69
70
71
72
73

All contents of this site are 2012 E-Medical Revision Ltd

http://www.emrcs.com/question/question.php?q=0

Terms and Conditions

Privacy policy

2/2

17/05/2015

Reference ranges

Previous

Question 74 of 264

Next

On inspecting the caecum, which of the following structures is most likely to be identified at
the point at which all the tenia coli converge?

A. Gonadal vessels
B. Appendix base

Question stats

Score: 78.1%

8.3%

58.7%

1
2
3-5 3 / 3

10.1%

14.8%

8.1%

58.7% of users answered this


question correctly

C. Appendix tip

End and review

D. Ileocaecal valve

8
9
10

E. Ileocolic artery

11

Search

12
Next question

Go

13
14

The tenia coli converge at the base of the appendix.

15

Caecum

16
17

Location

18

Proximal right colon below the ileocaecal valve


Intraperitoneal

19
20

Posterior relations

21

Psoas
Iliacus
Femoral nerve
Genitofemoral nerve
Gonadal vessels

22
23
24
25

Anterior relations

Greater omentum

Arterial supply

Ileocolic artery

Lymphatic drainage

Mesenteric nodes accompany the venous drainage

26
27
28
29
30

The caecum is the most distensible part of the colon and in complete large bowel
obstruction with a competent ileocaecal valve the most likely site of eventual
perforation.

31
32
33
34
35-37 3 / 3

Rate question:
Comment on this question

Next question

38
39
40
41
42
43
44
45
46
47
48
49
50
51
52
53
54
55
56-58 3 / 3
59
60
61

http://www.emrcs.com/question/question.php?q=0

1/2

17/05/2015
62
63
64
65
66
67
68
69
70
71
72
73
74

All contents of this site are 2012 E-Medical Revision Ltd

http://www.emrcs.com/question/question.php?q=0

Terms and Conditions

Privacy policy

2/2

17/05/2015

Reference ranges

Previous

Question 75 of 264

Next

A 42 year old lady has had an axillary node clearance for breast malignancy. Post
operatively she reports weakness of the shoulder. She is unable to push herself forwards
from a wall with the right arm and the scapula is pushed out medially from the chest wall.
What is the most likely nerve injury?

Question stats

Score: 78.5%

8.2%

8.2%

1
2
3-5 3 / 3

7.3%

67.8%

8.5%

67.8% of users answered this


question correctly

A. C5, C6

End and review

8
9

B. C8, T1
10

C. Axillary nerve
11

Search

D. Long thoracic nerve

12

E. Spinal accessory nerve

Go

13
Next question

14
15

Theme from January 2012 and 2009 Exam

16

The patient has a winged scapula caused by damage to the long thoracic nerve (C5,6,7)
during surgery. The long thoracic nerve innervates serratus anterior. Serratus anterior
causes pushing out of the scapula during a punch.
NB winging of the scapular laterally may indicate trapezius muscle weakness. Innervated by
the spinal accessory nerve.

17
18
19
20
21
22

Long thoracic nerve

23
24

Derived from ventral rami of C5, C6, and C7 (close to their emergence from
intervertebral foramina)
It runs downward and passes either anterior or posterior to the middle scalene muscle
It reaches upper tip of serratus anterior muscle and descends on outer surface of this
muscle, giving branches into it
Winging of Scapula occurs in long thoracic nerve injury (most common) or from spinal
accessory nerve injury (which denervates the trapezius) or a dorsal scapular nerve
injury

25
26
27
28
29
30
31
32

Rate question:

Next question

33
34

Comment on this question

35-37 3 / 3
38
39
40
41
42
43
44
45
46
47
48
49
50
51
52
53
54
55
56-58 3 / 3
59
60
61

http://www.emrcs.com/question/question.php?q=0

1/2

17/05/2015
62
63
64
65
66
67
68
69
70
71
72
73
74
75

All contents of this site are 2012 E-Medical Revision Ltd

http://www.emrcs.com/question/question.php?q=0

Terms and Conditions

Privacy policy

2/2

17/05/2015

Reference ranges

Question 76 of 264

Previous

Next

A 36 year old male is admitted for elective surgery for a lymph node biopsy in the
supraclavicular region. Post operatively the patient has difficulty shrugging his left shoulder.
What is the most likely reason?

Question stats

Score: 78.8%

7%

11.3%

1
2
3-5 3 / 3

8.4%

8%

65.3%

A. Phrenic nerve lesion


65.3% of users answered this
question correctly

B. Axillary nerve lesion

End and review

C. C5, C6 root lesion

8
9
10

D. C8, T1 root lesion

11

Search

E. Accessory nerve lesion

12
Go

Next question

13
14

Theme from September 2011 Exam


Theme from September 2013 Exam
The accessory nerve lies in the posterior triangle and may be injured in this region. Apart
from problems with shrugging the shoulder, he may also have difficulty lifting his arm above
his head.

15
16
17
18
19

Posterior triangle of the neck

20
21

Boundaries

22

Apex

Sternocleidomastoid and the Trapezius muscles at the Occipital bone

Anterior

Posterior border of the Sternocleidomastoid

24

Posterior

Anterior border of the Trapezius

25

Base

Middle third of the clavicle

23

26
27
28
29
30
31
32
33
34
35-37 3 / 3
38
39
40

Image sourced from Wikipedia

41

Contents
Nerves

42

Accessory nerve
Phrenic nerve
Three trunks of the brachial plexus
Branches of the cervical plexus: Supraclavicular nerve, transverse cervical
nerve, great auricular nerve, lesser occipital nerve

43
44
45
46
47

Vessels

48

External jugular vein


Subclavian artery

49
50

Muscles

51

Inferior belly of omohyoid


Scalene

52
53

Lymph
nodes

54

Supraclavicular
Occipital

55
56-58 3 / 3
59

Rate question:

Next question

60
61

Comment on this question

http://www.emrcs.com/question/question.php?q=0

1/2

17/05/2015
Comment on this question

62
63
64
65
66
67
68
69
70
71
72
73
74
75
76

All contents of this site are 2012 E-Medical Revision Ltd

http://www.emrcs.com/question/question.php?q=0

Terms and Conditions

Privacy policy

2/2

17/05/2015

Reference ranges

Question 77 of 264

Previous

Next

How many fissures are present within the right lung?

Question stats

End and review

Score: 78.2%

18.3%

22.8%

1
2
3-5 3 / 3

41.4%

A. One

9%

B. Three

8.4%

C. Two

41.4% of users answered this


question correctly

D. Four

8
9
10

E. Five

11

Search
Next question

12
Go

13

The right lung has an oblique and horizontal fissure. The upper oblique fissure separates the
inferior from the middle and upper lobes. The short horizontal fissure separates the superior
and middle lobes.

14
15
16

Lung anatomy
The right lung is composed of 3 lobes divided by the oblique and transverse fissures. The
left lung has two lobes divided by the oblique fissure.The apex of both lungs is approximately
4cm superior to the sterno-costal joint of the first rib. Immediately below this is a sulcus
created by the subclavian artery.

17
18
19
20
21

Peripheral contact points of the lung


Base: diaphragm
Costal surface: corresponds to the cavity of the chest
Mediastinal surface: Contacts the mediastinal pleura. Has the cardiac impression.
Above and behind this concavity is a triangular depression named the hilum, where
the structures which form the root of the lung enter and leave the viscus. These
structures are invested by pleura, which, below the hilum and behind the pericardial
impression, forms the pulmonary ligament

22
23
24
25
26
27
28
29

Right lung
Above the hilum is the azygos vein; Superior to this is the groove for the superior vena cava
and right innominate vein; behind this, and nearer the apex, is a furrow for the innominate
artery. Behind the hilum and the attachment of the pulmonary ligament is a vertical groove
for the oesophagus; In front and to the right of the lower part of the oesophageal groove is a
deep concavity for the extrapericardiac portion of the inferior vena cava.

30
31
32
33
34

The root of the right lung lies behind the superior vena cava and the right atrium, and below
the azygos vein.

35-37 3 / 3
38
39

The right main bronchus is shorter, wider and more vertical than the left main bronchus and
therefore the route taken by most foreign bodies.

40
41
42
43
44
45
46
47
48
49

Image sourced from Wikipedia

50
51

Left lung
Above the hilum is the furrow produced by the aortic arch, and then superiorly the groove
accommodating the left subclavian artery; Behind the hilum and pulmonary ligament is a
vertical groove produced by the descending aorta, and in front of this, near the base of the
lung, is the lower part of the oesophagus.
The root of the left lung passes under the aortic arch and in front of the descending aorta.

52
53
54
55
56-58 3 / 3
59

http://www.emrcs.com/question/question.php?q=0

1/2

17/05/2015
59
60
61
62
63
64
65
66
67
Image sourced from Wikipedia

68
69

Inferior borders of both lungs

70

6th rib in mid clavicular line


8th rib in mid axillary line
10th rib posteriorly

71
72
73

The pleura runs two ribs lower than the corresponding lung level.

74

Bronchopulmonary segments

75

Segment number

Right lung

Left lung

76

Apical

Apical

77

Posterior

Posterior

Anterior

Anterior

Lateral

Superior lingular

Medial

Inferior lingular

Superior (apical)

Superior (apical)

Medial basal

Medial basal

Anterior basal

Anterior basal

Lateral basal

Lateral basal

10

Posterior basal

Posterior basal

Rate question:

Next question

Comment on this question

All contents of this site are 2012 E-Medical Revision Ltd

http://www.emrcs.com/question/question.php?q=0

Terms and Conditions

Privacy policy

2/2

17/05/2015

Reference ranges

Previous

Question 78 of 264

Next

Which of the following muscles is supplied by the musculocutaneous nerve?

Question stats

End and review

Score: 78.4%

63.1%

9.4%

1
2
3-5 3 / 3

8.8%

A. Brachialis

9.3%

B. Latissimus dorsi

9.4%

C. Flexor carpi ulnaris

63.1% of users answered this


question correctly

D. Teres minor

8
9
10

E. Triceps

11

Search
Next question

12
Go

13
14

Mnemonic

15

Muscles innervated by the musculocutaneous nerve BBC:

16
17

Biceps brachii
Brachialis
Coracobrachialis

18
19
20
21
22

Musculocutaneous nerve

23
24

Branch of lateral cord of brachial plexus

25
26

Path

27

It penetrates the Coracobrachialis muscle


Passes obliquely between the Biceps brachii and the Brachialis to the lateral side of
the arm
Above the elbow it pierces the deep fascia lateral to the tendon of the Biceps brachii
Continues into the forearm as the lateral cutaneous nerve of the forearm

28
29
30
31
32

Innervates

33

Coracobrachialis
Biceps brachii
Brachialis

34
35-37 3 / 3
38
39
40

Rate question:
Comment on this question

Next question

41
42
43
44
45
46
47
48
49
50
51
52
53
54
55
56-58 3 / 3
59
60
61

http://www.emrcs.com/question/question.php?q=0

1/2

17/05/2015
62
63
64
65
66
67
68
69
70
71
72
73
74
75
76
77
78

All contents of this site are 2012 E-Medical Revision Ltd

http://www.emrcs.com/question/question.php?q=0

Terms and Conditions

Privacy policy

2/2

17/05/2015

Reference ranges

Question 79 of 264

Previous

Next

Which of the following statements relating to the posterior cerebral artery is false?

Question stats

End and review

Score: 77.2%

7%

13.8%

1
2
3-5 3 / 3

15.9%

A. It supplies the visual cortex

51.7%

B. It is closely related to the 3rd cranial nerve

11.6%

C. It is a branch of the basilar artery

51.7% of users answered this


question correctly

D. It is connected to the circle of Willis via the superior cerebellar artery

8
9
10

E. When occluded may result in contralateral loss of field of vision

11

Search
Next question

12
Go

13

The posterior cerebral arteries are formed by the bifurcation of the basilar artery and is
connected to the circle of Willis via the posterior communicating artery.

14
15

The posterior cerebral arteries supply the occipital lobe and part of the temporal lobe.
Circle of Willis

16
17
18

The two internal carotid arteries and two vertebral arteries form an anastomosis known as
the Circle of Willis on the inferior surface of the brain. Each half of the circle is formed by:
1. Anterior communicating artery
2. Anterior cerebral artery
3. Internal carotid artery
4. Posterior communicating artery
5. Posterior cerebral arteries and the termination of the basilar artery

19
20
21
22
23
24

The circle and its branches supply; the corpus striatum, internal capsule, diencephalon and
midbrain.

25
26
27
28
29
30
31
32
33
34
35-37 3 / 3
38
39
40
41
42
43
44
45
46

Image sourced from Wikipedia

47
48

Vertebral arteries
Enter the cranial cavity via foramen magnum
Lie in the subarachnoid space
Ascend on anterior surface of medulla oblongata
Unite to form the basilar artery at the base of the pons

49
50
51
52
53

Branches:

54
55

Posterior spinal artery


Anterior spinal artery
Posterior inferior cerebellar artery

56-58 3 / 3
59
60

Basilar artery

61
62

http://www.emrcs.com/question/question.php?q=0

1/2

17/05/2015
62

Branches:

63

Anterior inferior cerebellar artery


Labyrinthine artery
Pontine arteries
Superior cerebellar artery
Posterior cerebral artery

64
65
66
67
68

Internal carotid arteries


Branches:

69
70

Posterior communicating artery


Anterior cerebral artery
Middle cerebral artery
Anterior choroid artery

71
72
73
74
75

Rate question:

Next question

76
77

Comment on this question

78
79

All contents of this site are 2012 E-Medical Revision Ltd

http://www.emrcs.com/question/question.php?q=0

Terms and Conditions

Privacy policy

2/2

17/05/2015

Reference ranges

Question 80 of 264

Previous

Next

An elderly lady falls and lands on her hip. On examination her hip is tender to palpation and
x-rays are taken. There are concerns that she may have an intertrochanteric fracture. What
is the normal angle between the femoral neck and the femoral shaft?

A. 90o

Question stats

Score: 77.5%

8.6%

20%

1
2
3-5 3 / 3

9.7%

53.4%

8.3%

53.4% of users answered this


question correctly

B. 105o

End and review

C. 80o

8
9
10

D. 130o

11

Search

12

E. 180o

Go

13
Next question

14
15

The normal angle between the femoral head and shaft is 130o. Changes to this angle may
occur as a result of disease or pathology and should be investigated.

16

Hip joint

18

17

19

Head of femur articulates with acetabulum of the pelvis


Both covered by articular hyaline cartilage
The acetabulum forms at the union of the ilium, pubis, and ischium
The triradiate cartilage (Y-shaped growth plate) separates the pelvic bones
The acetabulum holds the femoral head by the acetabular labrum
Normal angle between femoral head and femoral shaft is 130o

20
21
22
23
24
25
26

Ligaments

27

Transverse ligament: joints anterior and posterior ends of the articular cartilage
Head of femur ligament (ligamentum teres): acetabular notch to the fovea. Contains
arterial supply to head of femur in children.

28
29
30
31
32
33
34
35-37 3 / 3
38
39
40
41
42
43

Image sourced from Wikipedia

44
45
46
47
48
49
50
51
52
53
54
55
56-58 3 / 3
59
60

Image sourced from Wikipedia

http://www.emrcs.com/question/question.php?q=0

61

1/2

17/05/2015
62

Extracapsular ligaments

63

Iliofemoral ligament: inverted Y shape. Anterior iliac spine to the trochanteric line
Pubofemoral ligament: acetabulum to lesser trochanter
Ischiofemoral ligament: posterior support. Ischium to greater trochanter.

64
65
66
67

Blood supply
Medial circumflex femoral and lateral circumflex femoral arteries (Branches of profunda
femoris). Also from the inferior gluteal artery. These form an anastomosis and travel to up
the femoral neck to supply the head.

68
69
70
71

Rate question:

72

Next question

73
Comment on this question

74
75
76
77
78
79
80

All contents of this site are 2012 E-Medical Revision Ltd

http://www.emrcs.com/question/question.php?q=0

Terms and Conditions

Privacy policy

2/2

17/05/2015

Reference ranges

Previous

Question 81 of 264

Next

A 17 year old male presents to the clinic. He complains of difficulty using his left hand. It has
been a persistent problem since he sustained a distal humerus fracture as a child. On
examination there is diminished sensation overlying the hypothenar eminence and medial
one and half fingers. What is the most likely nerve lesion?

Question stats

Score: 77.7%

9.2%

6.2%

1
2
3-5 3 / 3

64.3%

13.2%

7%

64.3% of users answered this


question correctly

A. Anterior interosseous nerve

End and review

8
9

B. Posterior interosseous nerve


10

C. Ulnar nerve
11

Search

D. Median nerve

12

E. Radial nerve

Go

13
Next question

14
15

Theme from April 2012 Exam


This sensory deficit pattern is most consistent with ulnar nerve injury.

16
17
18
19
20
21
22
23
24
25
26
27
28
29
30
31
32
33
34
35-37 3 / 3

Image sourced from Wikipedia

38
39

Ulnar nerve

40
41

Origin
C8, T1

42
43
44

Supplies (no muscles in the upper arm)


Flexor carpi ulnaris
Flexor digitorum profundus
Flexor digiti minimi
Abductor digiti minimi
Opponens digiti minimi
Adductor pollicis
Interossei muscle
Third and fourth lumbricals
Palmaris brevis

45
46
47
48
49
50
51
52
53
54

Path

55

Posteromedial aspect of ulna to flexor compartment of forearm, then along the ulnar.
Passes beneath the flexor carpi ulnaris muscle, then superficially through the flexor
retinaculum into the palm of the hand.

56-58 3 / 3
59
60
61

http://www.emrcs.com/question/question.php?q=0

1/2

17/05/2015
61
62
63
64
65
66
67
68
69
70
71
72
73
74
75
76
77
78
Image sourced from Wikipedia

79
80

Branches

81

Branch

Supplies

Articular branch

Flexor carpi ulnaris


Medial half of the flexor digitorum profundus

Palmar cutaneous branch (Arises near the


middle of the forearm)

Skin on the medial part of the palm

Dorsal cutaneous branch

Dorsal surface of the medial part of the hand

Superficial branch

Cutaneous fibres to the anterior surfaces of the


medial one and one-half digits

Deep branch

Hypothenar muscles
All the interosseous muscles
Third and fourth lumbricals
Adductor pollicis
Medial head of the flexor pollicis brevis

Effects of injury
Damage at the wrist

Damage at the elbow

Wasting and paralysis of intrinsic hand muscles (claw hand)


Wasting and paralysis of hypothenar muscles
Loss of sensation medial 1 and half fingers

Radial deviation of the wrist


Clawing less in 3rd and 4th digits

Rate question:

Next question

Comment on this question

All contents of this site are 2012 E-Medical Revision Ltd

http://www.emrcs.com/question/question.php?q=0

Terms and Conditions

Privacy policy

2/2

17/05/2015

Reference ranges

Question 82 of 264

Previous

Next

An 18 year old athlete attends orthopaedic clinic reporting pain and swelling over the medial
aspect of the knee joint. The pain occurs when climbing the stairs, but is not present when
walking on flat ground. Clinically there is pain over the medial, proximal tibia and the
McMurray test is negative. What is the most likely cause of this patient's symptoms?

Question stats

Score: 77.9%

16.3%

14.4%

1
2
3-5 3 / 3

26.3%

36.4%

6.5%

36.4% of users answered this


question correctly

A. Anterior cruciate ligament tear

End and review

8
9

B. Prepatellar bursitis
10

C. Medial meniscus injury


11

Search

D. Pes Anserinus Bursitis

12

E. Fracture of tibia

Go

13
Next question

14
15
16

Pes anserinus: GOOSE'S FOOT

17

Combination of sartorius, gracilis and semitendinous tendons inserting into the


anteromedial proximal tibia.

18
19
20

Pes Anserinus Bursitis is common in sportsmen due to overuse injuries. The main sign is of
pain in the medial proximal tibia. As the McMurray test is negative, medial meniscal injury is
excluded.

21
22
23

Sartorius

24
25

Longest strap muscle in the body


Most superficial muscle in the anterior compartment of the thigh

26
27
28

Origin

Anterior superior iliac spine

Insertion

Medial surface of the of the body of the tibia (upper part). It inserts anterior to
gracilis and semitendinosus

Nerve
Supply

29
30
31
32

Femoral nerve (L2,3)

33
34

Action

Flexor of the hip and knee, slight abducts the thigh and rotates it laterally
It assists with medial rotation of the tibia on the femur. For example it would
play a pivotal role in placing the right heel onto the left knee ( and vice versa)

35-37 3 / 3
38
39

Important
relations

The middle third of this muscle, and its strong underlying fascia forms the roof of
the adductor canal , in which lie the femoral vessels, the saphenous nerve and the
nerve to vastus medialis.

40
41
42
43

Rate question:

44
Next question

45
Comment on this question

46
47
48
49
50
51
52
53
54
55
56-58 3 / 3
59
60
61

http://www.emrcs.com/question/question.php?q=0

1/2

17/05/2015
62
63
64
65
66
67
68
69
70
71
72
73
74
75
76
77
78
79
80
81
82

All contents of this site are 2012 E-Medical Revision Ltd

http://www.emrcs.com/question/question.php?q=0

Terms and Conditions

Privacy policy

2/2

17/05/2015

Reference ranges

Question 83 of 264

Previous

Next

Which of the following structures lies most posteriorly at the porta hepatis?

Question stats

End and review

Score: 78.2%

9.1%

12.5%

1
2
3-5 3 / 3

9.2%

A. Cystic artery

53.9%

B. Common hepatic artery

15.2%

C. Left hepatic artery

53.9% of users answered this


question correctly

D. Portal vein

8
9
10

E. Common bile duct

11

Search
Next question

12
Go

13

The portal vein is the most posterior structure at the porta hepatis.The common bile duct is a
continuation of the common hepatic duct and is formed by the union of the common hepatic
duct and the cystic duct.

14
15
16

Liver

17
18

Structure of the liver

19

Right lobe

Supplied by right hepatic artery


Contains Couinaud segments V to VIII (-/+Sg I)

20
21
22

Left lobe

Supplied by the left hepatic artery


Contains Couinaud segments II to IV (+/- Sg1)

23
24
25

Quadrate lobe

Part of the right lobe anatomically, functionally is part of the left


Couinaud segment IV
Porta hepatis lies behind
On the right lies the gallbladder fossa
On the left lies the fossa for the umbilical vein

26
27
28
29

Caudate lobe

Supplied by both right and left hepatic arteries


Couinaud segment I
Lies behind the plane of the porta hepatis
Anterior and lateral to the inferior vena cava
Bile from the caudate lobe drains into both right and left hepatic ducts

30
31
32
33
34

Detailed knowledge of Couinaud segments is not required for MRCS Part A

35-37 3 / 3
38

Between the liver lobules are portal canals which contain the portal triad: Hepatic
Artery, Portal Vein, tributary of Bile Duct.

39
40
41

Relations of the liver

42
43

Anterior

Postero inferiorly

Diaphragm

Oesophagus

45

Xiphoid process

Stomach

46

44

47

Duodenum
48

Hepatic flexure of colon


Right kidney

49
50
51

Gallbladder
Inferior vena cava

52
53
54

Porta hepatis
Location

Transmits

55

Postero inferior surface, it joins nearly at right angles with the left sagittal fossa,
and separates the caudate lobe behind from the quadrate lobe in front

56-58 3 / 3
59
60

Common hepatic duct


Hepatic artery

http://www.emrcs.com/question/question.php?q=0

61

1/2

17/05/2015
62

Portal vein
Sympathetic and parasympathetic nerve fibres
Lymphatic drainage of the liver (and nodes)

63
64
65

Ligaments

66

Falciform
ligament

67

2 layer fold peritoneum from the umbilicus to anterior liver surface


Contains ligamentum teres (remnant umbilical vein)
On superior liver surface it splits into the coronary and left
triangular ligaments

68
69
70

Ligamentum teres

Joins the left branch of the portal vein in the porta hepatis

71

Ligamentum
venosum

Remnant of ductus venosus

72
73
74

Arterial supply

75
76

Hepatic artery

77
78

Venous

79

Hepatic veins
Portal vein

80
81
82

Nervous supply

83

Sympathetic and parasympathetic trunks of coeliac plexus

Rate question:

Next question

Comment on this question

All contents of this site are 2012 E-Medical Revision Ltd

http://www.emrcs.com/question/question.php?q=0

Terms and Conditions

Privacy policy

2/2

17/05/2015

Reference ranges

Question 84 of 264

Previous

Next

A 76 year old man is undergoing an abdominal aortic aneurysm repair. The surgeons
occlude the aorta with two clamps, the inferior clamp being placed at the point of aortic
bifurcation. Which of the following vertebral bodies will lie posterior to the clamp at this level?

Question stats

Score: 78.5%

7.6%

7.4%

1
2
3-5 3 / 3

68.5%

9.1%

7.4%

A. L1
68.5% of users answered this
question correctly

B. T10

End and review

C. L4

8
9
10

D. L5

11

Search

E. L2

12
Go

Next question

13
14
15

Theme from September 2013 Exam


The aorta bifurcates at L4. An important landmark that is tested frequently.

16
17

Abdominal aorta

18
19

Abdominal aortic topography


Origin

T12

Termination

L4

Posterior relations

L1-L4 Vertebral bodies

Anterior relations

Lesser omentum
Liver
Left renal vein
Inferior mesenteric vein
Third part of duodenum
Pancreas
Parietal peritoneum
Peritoneal cavity

20
21
22

Right lateral relations

23
24
25
26
27
28
29
30

Right crus of the diaphragm


Cisterna chyli
Azygos vein
IVC (becomes posterior distally)

31
32
33

Left lateral relations

4th part of duodenum


Duodenal-jejunal flexure
Left sympathetic trunk

34
35-37 3 / 3
38

The abdominal aorta

39
40
41
42
43
44
45
46
47
48
49
50
51
52
53

Image sourced from Wikipedia

54
55
56-58 3 / 3

Rate question:
Comment on this question

Next question

59
60
61

http://www.emrcs.com/question/question.php?q=0

1/2

17/05/2015
62
63
64
65
66
67
68
69
70
71
72
73
74
75
76
77
78
79
80
81
82
83
84

All contents of this site are 2012 E-Medical Revision Ltd

http://www.emrcs.com/question/question.php?q=0

Terms and Conditions

Privacy policy

2/2

17/05/2015

Reference ranges

Previous

Question 85 of 264

Next

Which of the following statements relating to the greater omentum is false?

Question stats

End and review

Score: 78.6%

15.5%

49%

1
2
3-5 3 / 3

16%

A. It is less well developed in children under 5.

10.2%

B. It has no relationship to the lesser sac.

9.4%

C. It contains the gastroepiploic arteries.

49% of users answered this


question correctly

D. Has an attachment to the transverse colon.

8
9
10

E. It may be a site of metastatic disease in ovarian cancer.

11

Search
Next question

12
Go

13

It is connected with the lesser sac and the transverse colon. This plane is entered when
performing a colonic resection. It is a common site of metastasis in many visceral
malignancies.

14
15
16

Omentum

17
18

The omentum is divided into two parts which invest the stomach. Giving rise to the
greater and lesser omentum. The greater omentum is attached to the inferolateral
border of the stomach and houses the gastro-epiploic arteries.
It is of variable size but is less well developed in children. This is important as the
omentum confers protection against visceral perforation (e.g. Appendicitis).
Inferiorly between the omentum and transverse colon is one potential entry point into
the lesser sac.
Several malignant processes may involve the omentum of which ovarian cancer is the
most notable.

19
20
21
22
23
24
25
26
27

Rate question:
Comment on this question

Next question

28
29
30
31
32
33
34
35-37 3 / 3
38
39
40
41
42
43
44
45
46
47
48
49
50
51
52
53
54
55
56-58 3 / 3
59
60
61

http://www.emrcs.com/question/question.php?q=0

1/2

17/05/2015
62
63
64
65
66
67
68
69
70
71
72
73
74
75
76
77
78
79
80
81
82
83
84
85

All contents of this site are 2012 E-Medical Revision Ltd

http://www.emrcs.com/question/question.php?q=0

Terms and Conditions

Privacy policy

2/2

17/05/2015

Reference ranges

Question 1 of 179

Next

A 48 year old man with newly diagnosed hypertension is found to have a


phaeochromocytoma of the left adrenal gland and is due to undergo a laparoscopic left
adrenalectomy. Which of the following structures is not directly related to the left adrenal
gland?

Question stats

End and review

Score: 100%

12.9%

48.3%

10.9%

11.5%

16.5%

48.3% of users answered this


question correctly

A. Crus of the diaphragm


B. Lesser curvature of the stomach
C. Kidney

Search

D. Pancreas
E. Splenic artery

Go

Next question

The left adrenal gland is slightly larger than the right. It is crescent in shape and its concavity
is adapted to the medial border of the upper part of the left kidney. The upper area is
covered by peritoneum of the omental bursa which separates it from the cardia of the
stomach. The lower area is in contact with the pancreas and splenic artery and is not
covered by peritoneum. On the anterior surface is a hilum from which the suprarenal vein
emerges. The lateral aspect rests on the kidney. The medial is small and is on the left crus of
the diaphragm.
Adrenal gland anatomy
Anatomy
Location

Superomedially to the upper pole of each kidney

Relationships of
the right adrenal

Diaphragm-Posteriorly, Kidney-Inferiorly, Vena Cava-Medially, Hepatorenal pouch and bare area of the liver-Anteriorly

Relationships of
the left adrenal

Crus of the diaphragm-Postero- medially, Pancreas and splenic


vessels-Inferiorly, Lesser sac and stomach-Anteriorly

Arterial supply

Superior adrenal arteries- from inferior phrenic artery, Middle adrenal


arteries - from aorta, Inferior adrenal arteries -from renal arteries

Venous drainage
of the right adrenal

Via one central vein directly into the IVC

Venous drainage
of the left adrenal

Via one central vein into the left renal vein

Rate question:

Next question

Comment on this question

All contents of this site are 2012 E-Medical Revision Ltd

http://www.emrcs.com/question/question.php?q=0

Terms and Conditions

Privacy policy

1/1

17/05/2015

Reference ranges

Question 2 of 179

Previous

Next

Which of the following nerves innervates the long head of the biceps femoris muscle?

Question stats

Score: 100%

12.9%

36.2%

10.5%

A. Inferior gluteal nerve

24.1%

B. Tibial division of sciatic nerve

16.3%

C. Superior gluteal nerve

36.2% of users answered this


question correctly

D. Common peroneal division of sciatic nerve

End and review

1
2

E. Obturator nerve
Search
Next question
Go

The short head of biceps femoris, which may occasionally be absent, is innervated by the
common peroneal component of the sciatic nerve. The long head is innervated by the tibial
division of the sciatic nerve.
Biceps femoris
The biceps femoris is one of the hamstring group of muscles located in the posterior upper
thigh. It has two heads.
Long head
Origin

Ischial tuberosity

Insertion

Fibular head

Action

Knee flexion, lateral rotation tibia, extension hip

Innervation

Tibial division of sciatic nerve (L5, S1, S2)

Arterial
supply

Profunda femoris artery, inferior gluteal artery, and the superior muscular
branches of popliteal artery

Image demonstrating the biceps femoris muscle, with the long head outlined

Image sourced from Wikipedia

Short head
Origin

Lateral lip of linea aspera, lateral supracondylar ridge of femur

Insertion

Fibular head

Action

Knee flexion, lateral rotation tibia

Innervation

Common peroneal division of sciatic nerve (L5, S1, S2)

Arterial
supply

Profunda femoris artery, inferior gluteal artery, and the superior muscular
branches of popliteal artery

Rate question:

Next question

Comment on this question

All contents of this site are 2012 E-Medical Revision Ltd

http://www.emrcs.com/question/question.php?q=0

Terms and Conditions

Privacy policy

1/2

17/05/2015

http://www.emrcs.com/question/question.php?q=0

2/2

17/05/2015

Reference ranges

Previous

Question 3 of 179

Next

A 72 year old male with end stage critical ischaemia is undergoing an axillo-femoral bypass.
What structure is not closely related to the axillary artery?

A. Posterior cord of the brachial plexus


B. Scalenus anterior muscle

Question stats

End and review

Score: 100%
1

13.8%

44.3%

15.5%

8.4%

17.9%

44.3% of users answered this


question correctly

C. Pectoralis minor muscle


D. Axillary vein
E. Lateral cord of the brachial plexus

Search
Go

Next question

The axillary artery is the continuation of the subclavian artery. It is surrounded by the cords
of the brachial plexus (from which they are named). The axillary vein runs alongside the
axillary artery throughout its length.
Axilla
Boundaries of the axilla
Medially

Chest wall and Serratus anterior

Laterally

Humeral head

Floor

Subscapularis

Anterior aspect

Lateral border of Pectoralis major

Fascia

Clavipectoral fascia

Content:
Long thoracic
nerve (of Bell)

Derived from C5-C7 and passes behind the brachial plexus to enter the
axilla. It lies on the medial chest wall and supplies serratus anterior. Its
location puts it at risk during axillary surgery and damage will lead to
winging of the scapula.

Thoracodorsal
nerve and
thoracodorsal
trunk

Innervate and vascularise latissimus dorsi.

Axillary vein

Lies at the apex of the axilla, it is the continuation of the basilic vein.
Becomes the subclavian vein at the outer border of the first rib.

Intercostobrachial
nerves

Traverse the axillary lymph nodes and are often divided during axillary
surgery. They provide cutaneous sensation to the axillary skin.

Lymph nodes

The axilla is the main site of lymphatic drainage for the breast.

Rate question:

Next question

Comment on this question

All contents of this site are 2012 E-Medical Revision Ltd

http://www.emrcs.com/question/question.php?q=0

Terms and Conditions

Privacy policy

1/1

17/05/2015

Reference ranges

Question 4 of 179

Previous

Next

A 28 year old man is shot in the right chest and develops a right haemothorax necessitating
a thoracotomy. The surgeons decide to place a vascular clamp across the hilum of the right
lung. Which of the following structures will lie most anteriorly at this point?

Question stats

End and review

Score: 100%
1

9%

51.5%

10%

16%

13.5%

A. Thoracic duct
51.5% of users answered this
question correctly

B. Phrenic nerve
C. Vagus nerve
D. Pulmonary artery

Search

E. Pulmonary vein

Go

Next question

The phrenic nerve lies anteriorly at the root of the right lung.
Lung anatomy
The right lung is composed of 3 lobes divided by the oblique and transverse fissures. The
left lung has two lobes divided by the oblique fissure.The apex of both lungs is approximately
4cm superior to the sterno-costal joint of the first rib. Immediately below this is a sulcus
created by the subclavian artery.
Peripheral contact points of the lung
Base: diaphragm
Costal surface: corresponds to the cavity of the chest
Mediastinal surface: Contacts the mediastinal pleura. Has the cardiac impression.
Above and behind this concavity is a triangular depression named the hilum, where
the structures which form the root of the lung enter and leave the viscus. These
structures are invested by pleura, which, below the hilum and behind the pericardial
impression, forms the pulmonary ligament

Right lung
Above the hilum is the azygos vein; Superior to this is the groove for the superior vena cava
and right innominate vein; behind this, and nearer the apex, is a furrow for the innominate
artery. Behind the hilum and the attachment of the pulmonary ligament is a vertical groove
for the oesophagus; In front and to the right of the lower part of the oesophageal groove is a
deep concavity for the extrapericardiac portion of the inferior vena cava.
The root of the right lung lies behind the superior vena cava and the right atrium, and below
the azygos vein.
The right main bronchus is shorter, wider and more vertical than the left main bronchus and
therefore the route taken by most foreign bodies.

Image sourced from Wikipedia

Left lung
Above the hilum is the furrow produced by the aortic arch, and then superiorly the groove
accommodating the left subclavian artery; Behind the hilum and pulmonary ligament is a
vertical groove produced by the descending aorta, and in front of this, near the base of the
lung, is the lower part of the oesophagus.
The root of the left lung passes under the aortic arch and in front of the descending aorta.

http://www.emrcs.com/question/question.php?q=0

1/2

17/05/2015

Image sourced from Wikipedia

Inferior borders of both lungs


6th rib in mid clavicular line
8th rib in mid axillary line
10th rib posteriorly
The pleura runs two ribs lower than the corresponding lung level.
Bronchopulmonary segments
Segment number

Right lung

Left lung

Apical

Apical

Posterior

Posterior

Anterior

Anterior

Lateral

Superior lingular

Medial

Inferior lingular

Superior (apical)

Superior (apical)

Medial basal

Medial basal

Anterior basal

Anterior basal

Lateral basal

Lateral basal

10

Posterior basal

Posterior basal

Rate question:

Next question

Comment on this question

All contents of this site are 2012 E-Medical Revision Ltd

http://www.emrcs.com/question/question.php?q=0

Terms and Conditions

Privacy policy

2/2

17/05/2015

Reference ranges

Previous

Question 5 of 179

Next

An 18 year old boy is undergoing an appendicectomy for appendicitis. At which of the


following locations is the appendix most likely to be found?

A. Pre ileal
B. Pelvic

Question stats

End and review

Score: 85.7%
1

8.2%

8.7%

66.8%

9.3%

7%

66.8% of users answered this


question correctly

C. Retrocaecal
D. Post ileal
E. None of the above

Search
Go

Next question

Most appendixes lie in the retrocaecal position. If a retrocaecal appendix is difficult to remove
then mobilisation of the right colon significantly improves access.
Appendix

Location: Base of caecum.


Up to 10cm long.
Mainly lymphoid tissue (Hence mesenteric adenitis may mimic appendicitis).
Caecal taenia coli converge at base of appendix and form a longitudinal muscle cover
over the appendix. This convergence should facilitate its identification at surgery if it is
retrocaecal and difficult to find (which it can be when people start doing
appendicectomies!)
Arterial supply: Appendicular artery (branch of the ileocolic).
It is intra peritoneal.

McBurney's point
1/3 of the way along a line drawn from the Anterior Superior Iliac Spine to the
Umbilicus

6 Positions:

Retrocaecal 74%
Pelvic 21%
Postileal
Subcaecal
Paracaecal
Preileal

Rate question:

Next question

Comment on this question

All contents of this site are 2012 E-Medical Revision Ltd

http://www.emrcs.com/question/question.php?q=0

Terms and Conditions

Privacy policy

1/1

17/05/2015

Reference ranges

Previous

Question 6 of 179

Next

A 56 year old man is undergoing a pancreatectomy for carcinoma. During resection of the
gland which of the following structures will the surgeon not encounter posterior to the
pancreas itself?

Question stats

Score: 87.5%
1

14.6%

18%

18.4%

14.6%

34.3%

A. Left crus of the diaphragm


34.3% of users answered this
question correctly

B. Superior mesenteric vein

End and review

C. Common bile duct


D. Portal vein
Search

E. Gastroduodenal artery

Go

Next question

Theme from 2010 Exam


The gastroduodenal artery lies anterior to the pancreas.
Pancreas
The pancreas is a retroperitoneal organ and lies posterior to the stomach. It may be
accessed surgically by dividing the peritoneal reflection that connects the greater omentum
to the transverse colon. The pancreatic head sits in the curvature of the duodenum. Its tail
lies close to the hilum of the spleen, a site of potential injury during splenectomy.
Relations
Posterior to the pancreas
Pancreatic head

Inferior vena cava


Common bile duct
Right and left renal veins
Superior mesenteric vein and artery

Pancreatic neck

Superior mesenteric vein, portal vein

Pancreatic body-

Left renal vein


Crus of diaphragm
Psoas muscle
Adrenal gland
Kidney
Aorta

Pancreatic tail

Left kidney

Anterior to the pancreas


Pancreatic head

1st part of the duodenum


Pylorus
Gastroduodenal artery
SMA and SMV(uncinate process)

Pancreatic body

Stomach
Duodenojejunal flexure

Pancreatic tail

Splenic hilum

Superior to the pancreas


Coeliac trunk and its branches common hepatic artery and splenic artery
Grooves of the head of the pancreas
2nd and 3rd part of the duodenum
Arterial supply
Head: pancreaticoduodenal artery
Rest: splenic artery

Venous drainage
Head: superior mesenteric vein
Body and tail: splenic vein

Ampulla of Vater

http://www.emrcs.com/question/question.php?q=0

1/2

17/05/2015

Merge of pancreatic duct and common bile duct


Is an important landmark, halfway along the second part of the duodenum, that marks
the anatomical transition from foregut to midgut (also the site of transition between
regions supplied by coeliac trunk and SMA).

Image sourced from Wikipedia

Rate question:

Next question

Comment on this question

All contents of this site are 2012 E-Medical Revision Ltd

http://www.emrcs.com/question/question.php?q=0

Terms and Conditions

Privacy policy

2/2

17/05/2015

Reference ranges

Previous

Question 7 of 179

Next

Question stats

End and review

Score: 90%
1

10.3%

39.4%

12.9%

A. All metatarsals

19.9%

B. 5th metatarsal

17.4%

C. Calcaneum

39.4% of users answered this


question correctly

Which of the following bones is related to the cuboid's distal articular surface?

D. Medial cuneiform

6
7

E. 3rd metatarsal
Search
Next question
Go

Theme from April 2012 Exam


The cuboid is located at the lateral aspect of the foot between the calcaneus posteriorly and
the 4th and 5th metatarsals distally.
Foot- anatomy
Arches of the foot
The foot is conventionally considered to have two arches.
The longitudinal arch is higher on the medial than on the lateral side. The posterior
part of the calcaneum forms a posterior pillar to support the arch. The lateral part of
this structure passes via the cuboid bone and the lateral two metatarsal bones. The
medial part of this structure is more important. The head of the talus marks the summit
of this arch, located between the sustentaculum tali and the navicular bone. The
anterior pillar of the medial arch is composed of the navicular bone, the three
cuneiforms and the medial three metatarsal bones.
The transverse arch is situated on the anterior part of the tarsus and the posterior
part of the metatarsus. The cuneiforms and metatarsal bases narrow inferiorly, which
contributes to the shape of the arch.

Intertarsal joints
Sub talar joint

Formed by the cylindrical facet on the lower surface of the body of


the talus and the posterior facet on the upper surface of the
calcaneus. The facet on the talus is concave anteroposteriorly, the
other is convex. The synovial cavity of this joint does not
communicate with any other joint.

Talocalcaneonavicular
joint

The anterior part of the socket is formed by the concave articular


surface of the navicular bone, posteriorly by the upper surface of the
sustentaculum tali. The talus sits within this socket

Calcaneocuboid joint

Highest point in the lateral part of the longitudinal arch. The lower
aspect of this joint is reinforced by the long plantar and plantar
calcaneocuboid ligaments.

Transverse tarsal joint

The talocalcaneonavicular joint and the calcaneocuboid joint extend


across the tarsus in an irregular transverse plane, between the talus
and calcaneus behind and the navicular and cuboid bones in front.
This plane is termed the transverse tarsal joint.

Cuneonavicular joint

Formed between the convex anterior surface of the navicular bone


and the concave surface of the the posterior ends of the three
cuneiforms.

Intercuneiform joints

Between the three cuneiform bones.

Cuneocuboid joint

Between the circular facets on the lateral cuneiform bone and the
cuboid. This joint contributes to the tarsal part of the transverse arch.

A detailed knowledge of the joints is not required for MRCS Part A. However, the contribution
they play to the overall structure of the foot should be appreciated
Ligaments of the ankle joint and foot

http://www.emrcs.com/question/question.php?q=0

1/3

17/05/2015

Image sourced from Wikipedia

Muscles of the foot


Muscle

Origin

Insertion

Nerve
supply

Action

Abductor
hallucis

Medial side of the calcaneus,


flexor retinaculum, plantar
aponeurosis

Medial side of
the base of
the proximal
phalanx

Medial
plantar
nerve

Abducts the great


toe

Flexor
digitorum
brevis

Medial process of the


calcaneus, plantar
eponeurosis.

Via 4 tendons
into the
middle
phalanges of
the lateral 4
toes.

Medial
plantar
nerve

Flexes all the joints


of the lateral 4 toes
except for the
interphalangeal joint.

Abductor
digit
minimi

From the tubercle of the


calcaneus and from the
plantar aponeurosis

Together with
flexor digit
minimi brevis
into the lateral
side of the
base of the
proximal
phalanx of the
little toe

Lateral
plantar
nerve

Abducts the little toe


at the
metatarsophalangeal
joint

Flexor
hallucis
brevis

From the medial side of the


plantar surface of the cuboid
bone, from the adjacent part
of the lateral cuneiform bone
and from the tendon of tibialis
posterior.

Into the
proximal
phalanx of the
great toe, the
tendon
contains a
sesamoid
bone

Medial
plantar
nerve

Flexes the
metatarsophalangeal
joint of the great toe.

Adductor
hallucis

Arises from two heads. The


oblique head arises from the
sheath of the peroneus longus
tendon, and from the plantar
surfaces of the bases of the
2nd, 3rd and 4th metatarsal
bones. The transverse head
arises from the plantar
surface of the lateral 4
metatarsophalangeal joints
and from the deep transverse
metatarsal ligament.

Lateral side of
the base of
the proximal
phalanx of the
great toe.

Lateral
plantar
nerve

Adducts the great


toe towards the
second toe. Helps
maintain the
transverse arch of
the foot.

Extensor
digitorum
brevis

On the dorsal surface of the


foot from the upper surface of
the calcaneus and its
associated fascia

Via four thin


tendons which
run forward
and medially
to be inserted
into the
medial four
toes. The
lateral three
tendons join
with hoods of
extensor
digitorum
longus.

Deep
peroneal

Extend the
metatarsophalangeal
joint of the medial
four toes. It is
unable to extend the
interphalangeal joint
without the
assistance of the
lumbrical muscles.

Detailed knowledge of the foot muscles are not needed for the MRCS part A
Nerves in the foot

http://www.emrcs.com/question/question.php?q=0

2/3

17/05/2015

Lateral plantar nerve


Passes anterolaterally towards the base of the 5th metatarsal between flexor digitorum brevis
and flexor accessorius. On the medial aspect of the lateral plantar artery. At the base of the
5th metatarsal it splits into superficial and deep branches.
Medial plantar nerve
Passes forwards with the medial plantar artery under the cover of the flexor retinaculum to
the interval between abductor hallucis and flexor digitorum brevis on the sole of the foot.
Plantar arteries
Arise under the cover of the flexor retinaculum, midway between the tip of the medial
malleolus and the most prominent part of the medial side of the heel.

Medial plantar artery. Passes forwards medial to medial plantar nerve in the space
between abductor hallucis and flexor digitorum brevis.Ends by uniting with a branch of
the 1st plantar metatarsal artery.
Lateral plantar artery. Runs obliquely across the sole of the foot. It lies lateral to the
lateral plantar nerve. At the base of the 5th metatarsal bone it arches medially across
the foot on the metatarsals

Dorsalis pedis artery


This vessel is a direct continuation of the anterior tibial artery. It commences on the front of
the ankle joint and runs to the proximal end of the first metatarsal space. Here is gives off the
arcuate artery and continues forwards as the first dorsal metatarsal artery. It is accompanied
by two veins throughout its length. It is crossed by the extensor hallucis brevis
Rate question:

Next question

Comment on this question

All contents of this site are 2012 E-Medical Revision Ltd

http://www.emrcs.com/question/question.php?q=0

Terms and Conditions

Privacy policy

3/3

17/05/2015

Reference ranges

Previous

Question 8 of 179

Next

Question stats

End and review

Score: 91.7%
1

42.1%

17.9%

22.6%

A. Septum transversum

10.1%

B. Pleuroperitoneal folds

7.3%

C. Diaphragmatic crura

42.1% of users answered this


question correctly

From which structure is the central tendon of the diaphragm derived?

D. Dorsal mesocardium
E. Oropharyngeal membrane

6
7
8

Search
Next question
Go

The septum transversum is a thick ridge of mesodermal tissue in the developing embryo that
separates the thoracic and abdominal cavities and forms the central tendon of the
diaphragm.
Embryology of the diaphragm and diaphragmatic hernia
Embryology
The diaphragm is formed between the 5th and 7th weeks of gestation through the
progressive fusion of the septum transversum, pleuroperitoneal folds and via lateral
muscular ingrowth. The muscular origins of the diaphragm are somites located in cervical
segments 3 to 5, which accounts for the long path taken by the phrenic nerve. The
components contribute to the following diaphragmatic segments:
Septum transversum - Central tendon
Pleuroperitoneal membranes - Parietal membranes surrounding viscera
Cervical somites C5 to C7 - Muscular component of the diaphragm

Diaphragmatic hernia
Type of hernia

Features

Morgagni

Anteriorly located
Minimal compromise on lung development
Minimal signs on antenatal ultrasound
Usually present later
Usually good prognosis

Bochdalek hernia

Posteriorly located
Larger defect
Often diagnosed antenatally
Associated with pulmonary hypoplasia
Poor prognosis

The posterior hernias of Bochdalek are the most common type and if not diagnosed
antenatally will typically present soon after birth with respiratory distress. The classical
finding is that of a scaphoid abdomen on clinical examination because of herniation of the
abdominal contents into the chest. Bochdalek hernias are associated with a number of
chromosomal abnormalities such as Trisomy 21 and 18. Infants have considerable
respiratory distress due to hypoplasia of the developing lung. Historically this was considered
to be due to direct compression of the lung by herniated viscera. This view over simplifies the
situation and the pulmonary hypoplasia occurs concomitantly with the hernial development,
rather than as a direct result of it. The pulmonary hypoplasia is associated with pulmonary
hypertension and abnormalities of pulmonary vasculature. The pulmonary hypertension
renders infants at risk of right to left shunting (resulting in progressive and worsening
hypoxia).
Diagnostic work up of these infants includes chest x-rays/ abdominal ultrasound scans and
cardiac echo.
Surgery forms the mainstay of treatment and both thoracic and abdominal approaches may
be utilised. Following reduction of the hernial contents a careful search needs to be made for
a hernial sac as failure to recognise and correct this will result in a high recurrence rate.
Smaller defects may be primarily closed, larger defects may require a patch to close the
defect. Malrotation of the viscera is a recognised association and may require surgical
correct at the same procedure (favoring an abdominal approach).
The mortality rate is 50-75% and is related to the degree of lung compromise and age at
presentation (considerably better in infants >24 hours old).
Rate question:

Next question

Comment on this question

http://www.emrcs.com/question/question.php?q=0

1/2

17/05/2015

All contents of this site are 2012 E-Medical Revision Ltd

http://www.emrcs.com/question/question.php?q=0

Terms and Conditions

Privacy policy

2/2

17/05/2015

Reference ranges

Previous

Question 9 of 179

Next

Question stats

End and review

Score: 92.3%
1

8.8%

16.8%

56.2%

A. Immediately lateral to the foramen caecum

10%

B. Floor of mouth

8.2%

C. Opposite the second molar tooth

56.2% of users answered this


question correctly

Where does Stensens duct primarily open?

D. Opposite the fifth molar tooth


E. Into the post nasal space

6
7
8
9

Search
Next question
Go

Stensens duct conveys secretions from the parotid gland and these enter the oral cavity at
the level of the second molar tooth.
Parotid gland
Anatomy of the parotid gland
Location

Overlying the mandibular ramus; anterior and inferior to the ear.

Salivary duct

Crosses the masseter, pierces the buccinator and drains adjacent to the
2nd upper molar tooth (Stensen's duct).

Structures passing
through the gland

Facial nerve (Mnemonic: The Zebra Buggered My Cat; Temporal


Zygomatic, Buccal, Mandibular, Cervical)
External carotid artery
Retromandibular vein
Auriculotemporal nerve

Relations

Anterior: masseter, medial pterygoid, superficial temporal and


maxillary artery, facial nerve, stylomandibular ligament
Posterior: posterior belly digastric muscle, sternocleidomastoid,
stylohyoid, internal carotid artery, mastoid process, styloid
process

Arterial supply

Branches of external carotid artery

Venous drainage

Retromandibular vein

Lymphatic
drainage

Deep cervical nodes

Nerve innervation

Parasympathetic-Secretomotor
Sympathetic-Superior cervical ganglion
Sensory- Greater auricular nerve

Parasympathetic stimulation produces a water rich, serous saliva. Sympathetic stimulation


leads to the production of a low volume, enzyme-rich saliva.
Rate question:

Next question

Comment on this question

All contents of this site are 2012 E-Medical Revision Ltd

http://www.emrcs.com/question/question.php?q=0

Terms and Conditions

Privacy policy

1/1

17/05/2015

Reference ranges

Question 10 of 179

Previous

Next

Which of the following nerves is responsible for the innervation of the posterior belly of the
digastric muscle?

A. Facial nerve
B. Hypoglossal nerve

Question stats

Score: 92.9%
1

39.7%

17.7%

12.2%

19.1%

11.3%

39.7% of users answered this


question correctly

C. Trigeminal nerve

End and review

D. Ansa cervicalis

6
7
8

E. Mylohoid nerve

Search
Go

Next question

10

The posterior belly of digastric is innervated by the facial nerve and the anterior belly by the
mylohoid nerve.
Anterior triangle of the neck

Boundaries
Anterior border of the Sternocleidomastoid
Lower border of mandible
Anterior midline
Sub triangles (divided by Digastric above and Omohyoid)
Muscular triangle: Neck strap muscles
Carotid triangle: Carotid sheath
Submandibular Triangle (digastric)

Contents of the anterior triangle


Digastric triangle

Submandibular gland
Submandibular nodes
Facial vessels
Hypoglossal nerve

Muscular triangle

Strap muscles
External jugular vein

Carotid triangle

Carotid sheath (Common carotid, vagus and internal jugular vein)


Ansa cervicalis

Nerve supply to digastric muscle


Anterior: Mylohyoid nerve
Posterior: Facial nerve

Image sourced from Wikipedia

Rate question:

Next question

Comment on this question

All contents of this site are 2012 E-Medical Revision Ltd

http://www.emrcs.com/question/question.php?q=0

Terms and Conditions

Privacy policy

1/1

18/05/2015

Reference ranges

Question 1 of 169

Next

Question stats

Score: 100%

11.3%

9.4%

61.4%

A. Triquetrum

9.6%

B. Lunate

8.2%

C. Pisiform

61.4% of users answered this


question correctly

Which of the following carpal bones is a sesamoid bone in the tendon of flexor carpi ulnaris?

D. Scaphoid

End and review

E. Capitate
Search
Next question
Go

This small bone has a single articular facet. It projects from the triquetral bone at the ulnar
aspect of the wrist where most regard it as a sesamoid bine lying within the tendon of flexor
carpi ulnaris.
Carpal bones
The wrist is comprised of 8 carpal bones, these are arranged in two rows of 4. It is convex
from side to side posteriorly and concave anteriorly.
Diagrammatic image of carpal bones

Image sourced from Wikipedia

Key to image
A

Scaphoid

Lunate

Triquetrum

Pisiform

Trapezium

Trapezoid

Capitate

Hamate

Radius

Ulna

Metacarpals

No tendons attach to: Scaphoid, lunate, triquetrum (stabilised by ligaments)

Rate question:

Next question

Comment on this question

All contents of this site are 2012 E-Medical Revision Ltd

http://www.emrcs.com/question/question.php?q=0

Terms and Conditions

Privacy policy

1/2

18/05/2015

http://www.emrcs.com/question/question.php?q=0

2/2

18/05/2015

Reference ranges

Question 2 of 169

Previous

Next

A 70 year old man falls and fractures his scaphoid bone. The fracture is displaced and the
decision is made to insert a screw to fix the fracture. Which of the following structures lies
directly medial to the scaphoid?

Question stats

End and review

Score: 100%

64.7%

7.6%

9.6%

10.2%

7.9%

1
2

A. Lunate
64.7% of users answered this
question correctly

B. Pisiform
C. Trapezoid
D. Trapezium

Search

E. None of the above

Go

Next question

The lunate lies medially in the anatomical plane. Fractures of the scaphoid that are
associated with high velocity injuries may cause associated lunate dislocation.
Scaphoid bone
The scaphoid has a concave articular surface for the head of the capitate and at the edge of
this is a crescentic surface for the corresponding area on the lunate.
Proximally, it has a wide convex articular surface with the radius. It has a distally sited
tubercle that can be palpated. The remaining articular surface is to the lateral side of the
tubercle. It faces laterally and is associated with the trapezium and trapezoid bones.
The narrow strip between the radial and trapezial surfaces and the tubercle gives rise to the
radial collateral carpal ligament. The tubercle receives part of the flexor retinaculum. This
area is the only part of the scaphoid that is available for the entry of blood vessels. It is
commonly fractured and avascular necrosis may result.
Scaphoid bone

Image sourced from Wikipedia

Rate question:

Next question

Comment on this question

All contents of this site are 2012 E-Medical Revision Ltd

http://www.emrcs.com/question/question.php?q=0

Terms and Conditions

Privacy policy

1/1

18/05/2015

Reference ranges

Question 3 of 169

Previous

Next

A 55 year old man is admitted with a brisk haematemesis. He is taken to the endoscopy
department and an upper GI endoscopy is performed by the gastroenterologist. He identifies
an ulcer on the posterior duodenal wall and spends an eternity trying to control the bleeding
with all the latest haemostatic techniques. He eventually asks the surgeons for help. A
laparotomy and anterior duodenotomy are performed, as the surgeon opens the duodenum
a vessel is spurting blood into the duodenal lumen. From which of the following does this
vessel arise?

Question stats

End and review

Score: 66.7%
1

16.7%

41.2%

20.9%

10.5%

10.7%

41.2% of users answered this


question correctly

A. Left gastric artery


Search

B. Common hepatic artery

Go

C. Right hepatic artery


D. Superior mesenteric artery
E. Splenic artery
Next question

The vessel will be the gastroduodenal artery, this arises from the common hepatic artery.
Gastroduodenal artery
Supplies
Pylorus, proximal part of the duodenum, and indirectly to the pancreatic head (via the
anterior and posterior superior pancreaticoduodenal arteries)
Path
Most commonly arises from the common hepatic artery of the coeliac trunk
Terminates by bifurcating into the right gastroepiploic artery and the superior
pancreaticoduodenal artery
Image showing stomach reflected superiorly to illustrate the relationship of the
gastroduodenal artery to the first part of the duodenum

Image sourced from Wikipedia

Rate question:

Next question

Comment on this question

All contents of this site are 2012 E-Medical Revision Ltd

http://www.emrcs.com/question/question.php?q=0

Terms and Conditions

Privacy policy

1/1

18/05/2015

Reference ranges

Previous

Question 4 of 169

Next

A 73 year old lady is hit by a car. She suffers a complex fracture of the distal aspect of her
humerus with associated injury to the radial nerve. Which of the following movements will be
most impaired as a result?

Question stats

End and review

Score: 75%
1

20%

9%

6.8%

58.2%

6%

A. Elbow extension
58.2% of users answered this
question correctly

B. Elbow flexion
C. Shoulder abduction
D. Wrist extension

Search

E. None of the above

Go

Next question

The triceps will not be affected so elbow extension will be preserved. Loss of wrist extension
will be the most obvious effect.
Radial nerve
Continuation of posterior cord of the brachial plexus (root values C5 to T1)
Path
In the axilla: lies posterior to the axillary artery on subscapularis, latissimus dorsi and
teres major.
Enters the arm between the brachial artery and the long head of triceps (medial to
humerus).
Spirals around the posterior surface of the humerus in the groove for the radial nerve.
At the distal third of the lateral border of the humerus it then pierces the intermuscular
septum and descends in front of the lateral epicondyle.
At the lateral epicondyle it lies deeply between brachialis and brachioradialis where it
then divides into a superficial and deep terminal branch.
Deep branch crosses the supinator to become the posterior interosseous nerve.

In the image below the relationships of the radial nerve can be appreciated

Image sourced from Wikipedia

Regions innervated
Motor (main
nerve)

Triceps
Anconeus
Brachioradialis
Extensor carpi radialis

Motor
(posterior
interosseous
branch)

Supinator
Extensor carpi ulnaris
Extensor digitorum
Extensor indicis
Extensor digiti minimi
Extensor pollicis longus and brevis
Abductor pollicis longus

Sensory

The area of skin supplying the proximal phalanges on the dorsal aspect of
the hand is supplied by the radial nerve (this does not apply to the little
finger and part of the ring finger)

http://www.emrcs.com/question/question.php?q=0

1/2

18/05/2015

Muscular innervation and effect of denervation


Anatomical
location

Muscle affected

Effect of paralysis

Shoulder

Long head of triceps

Minor effects on shoulder stability in


abduction

Arm

Triceps

Loss of elbow extension

Forearm

Supinator
Brachioradialis
Extensor carpi radialis
longus and brevis

Weakening of supination of prone hand and


elbow flexion in mid prone position

The cutaneous sensation of the upper limb- illustrating the contribution of the radial nerve

Image sourced from Wikipedia

Rate question:

Next question

Comment on this question

All contents of this site are 2012 E-Medical Revision Ltd

http://www.emrcs.com/question/question.php?q=0

Terms and Conditions

Privacy policy

2/2

18/05/2015

Reference ranges

Question 5 of 169

Previous

Next

Question stats

End and review

Score: 80%
1

13.6%

9.3%

8.4%

A. Pyramidalis

61.4%

B. Superior epigastric artery

7.2%

C. Inferior epigastric vein

61.4% of users answered this


question correctly

Which of the following is not a content of the rectus sheath?

D. Internal iliac artery


E. Rectus abdominis

Search
Next question
Go

The rectus sheath also contains:


superior epigastric vein
inferior epigastric artery
Abdominal wall
The 2 main muscles of the abdominal wall are the rectus abdominis (anterior) and the
quadratus lumborum (posterior).
The remaining abdominal wall consists of 3 muscular layers. Each muscle passes from the
lateral aspect of the quadratus lumborum posteriorly to the lateral margin of the rectus
sheath anteriorly. Each layer is muscular posterolaterally and aponeurotic anteriorly.

Image sourced from Wikipedia

Muscles of abdominal wall


External
oblique

Internal
oblique

Transversus
abdominis

Lies most superficially


Originates from 5th to 12th ribs
Inserts into the anterior half of the outer aspect of the iliac crest, linea
alba and pubic tubercle
More medially and superiorly to the arcuate line, the aponeurotic layer
overlaps the rectus abdominis muscle
The lower border forms the inguinal ligament
The triangular expansion of the medial end of the inguinal ligament is the
lacunar ligament.

Arises from the thoracolumbar fascia, the anterior 2/3 of the iliac crest
and the lateral 2/3 of the inguinal ligament
The muscle sweeps upwards to insert into the cartilages of the lower 3
ribs
The lower fibres form an aponeurosis that runs from the tenth costal
cartilage to the body of the pubis
At its lowermost aspect it joins the fibres of the aponeurosis of
transversus abdominis to form the conjoint tendon.

Innermost muscle
Arises from the inner aspect of the costal cartilages of the lower 6 ribs ,
from the anterior 2/3 of the iliac crest and lateral 1/3 of the inguinal
ligament
Its fibres run horizontally around the abdominal wall ending in an
aponeurosis. The upper part runs posterior to the rectus abdominis. Lower
down the fibres run anteriorly only.
The rectus abdominis lies medially; running from the pubic crest and
symphysis to insert into the xiphoid process and 5th, 6th and 7th costal
cartilages. The muscles lies in a aponeurosis as described above.
Nerve supply: anterior primary rami of T7-12

Surgical notes
During abdominal surgery it is usually necessary to divide either the muscles or their
aponeuroses. During a midline laparotomy it is desirable to divide the aponeurosis. This will
leave the rectus sheath intact above the arcuate line and the muscles intact below it.
Straying off the midline will often lead to damage to the rectus muscles, particularly below the
arcuate line where they may often be in close proximity to each other.

http://www.emrcs.com/question/question.php?q=0

1/2

18/05/2015

Rate question:

Next question

Comment on this question

All contents of this site are 2012 E-Medical Revision Ltd

http://www.emrcs.com/question/question.php?q=0

Terms and Conditions

Privacy policy

2/2

18/05/2015

Reference ranges

Question 6 of 169

Previous

Next

Question stats

End and review

Score: 83.3%
1

47.3%

7.4%

12.9%

A. Superior mesenteric vein

16.1%

B. Right common iliac

16.3%

C. Right hepatic vein

47.3% of users answered this


question correctly

Which of the following vessels does not drain directly into the inferior vena cava?

D. Left hepatic vein

E. Right testicular vein


Search
Next question
Go

The superior mesenteric vein drains into the portal vein. The right and left hepatic veins
drain into it directly, this can account for major bleeding in more extensive liver shearing type
injuries.
Inferior vena cava
Origin
L5

Path
Left and right common iliac veins merge to form the IVC.
Passes right of midline
Paired segmental lumbar veins drain into the IVC throughout its length
The right gonadal vein empties directly into the cava and the left gonadal vein
generally empties into the left renal vein.
The next major veins are the renal veins and the hepatic veins
Pierces the central tendon of diaphragm at T8
Right atrium

Image sourced from Wikipedia

Relations
Anteriorly

Small bowel, first and third part of duodenum, head of pancreas, liver and bile
duct, right common iliac artery, right gonadal artery

Posteriorly

Right renal artery, right psoas, right sympathetic chain, coeliac ganglion

Levels
Level

Vein

T8

Hepatic vein, inferior phrenic vein, pierces diaphragm

L1

Suprarenal veins, renal vein

L2

Gonadal vein

L1-5

Lumbar veins

L5

Common iliac vein, formation of IVC

http://www.emrcs.com/question/question.php?q=0

1/2

18/05/2015

Rate question:

Next question

Comment on this question

All contents of this site are 2012 E-Medical Revision Ltd

http://www.emrcs.com/question/question.php?q=0

Terms and Conditions

Privacy policy

2/2

18/05/2015

Reference ranges

Previous

Question 7 of 169

Next

A 17 year old male has a suspected testicular torsion and the scrotum is to be explored
surgically. The surgeon incises the skin and then the dartos muscle. What is the next tissue
layer that will be encountered during the dissection?

Question stats

Score: 71.4%
1

9%

20.4%

22%

41.3%

7.3%

A. Visceral layer of the tunica vaginalis


41.3% of users answered this
question correctly

B. Cremasteric fascia

End and review

6
7

C. Parietal layer of the tunica vaginalis


D. External spermatic fascia
Search

E. Internal spermatic fascia

Go

Next question

The layers that will be encountered are (in order):


1. Skin
2. Dartos fascia and muscle
3. External spermatic fascia
4. Cremasteric muscle and fascia
5. Internal spermatic fascia
6. Parietal layer of the tunica vaginalis
The layers of the spermatic cord and scrotum are a popular topic in the MRCS exam.
Scrotal and testicular anatomy
Spermatic cord
Formed by the vas deferens and is covered by the following structures:
Layer

Origin

Internal spermatic fascia

Transversalis fascia

Cremasteric fascia

From the fascial coverings of internal oblique

External spermatic fascia

External oblique aponeurosis

Contents of the cord


Vas deferens

Transmits sperm and accessory gland secretions

Testicular artery

Branch of abdominal aorta supplies testis and


epididymis

Artery of vas deferens

Arises from inferior vesical artery

Cremasteric artery

Arises from inferior epigastric artery

Pampiniform plexus

Venous plexus, drains into right or left testicular


vein

Sympathetic nerve fibres

Lie on arteries, the parasympathetic fibres lie on the


vas

Genital branch of the genitofemoral


nerve

Supplies cremaster

Lymphatic vessels

Drain to lumbar and para-aortic nodes

Scrotum
Composed of skin and closely attached dartos fascia.
Arterial supply from the anterior and posterior scrotal arteries
Lymphatic drainage to the inguinal lymph nodes
Parietal layer of the tunica vaginalis is the innermost layer

Testes
The testes are surrounded by the tunica vaginalis (closed peritoneal sac). The parietal
layer of the tunica vaginalis adjacent to the internal spermatic fascia.
The testicular arteries arise from the aorta immediately inferiorly to the renal arteries.
The pampiniform plexus drains into the testicular veins, the left drains into the left
renal vein and the right into the inferior vena cava.

http://www.emrcs.com/question/question.php?q=0

1/2

18/05/2015

Lymphatic drainage is to the para-aortic nodes.

Rate question:

Next question

Comment on this question

All contents of this site are 2012 E-Medical Revision Ltd

http://www.emrcs.com/question/question.php?q=0

Terms and Conditions

Privacy policy

2/2

18/05/2015

Reference ranges

Question 8 of 169

Previous

Next

A 25 year old man is stabbed in the groin and the area, which lies within the femoral triangle
is explored. Which structure forms the lateral wall of the femoral triangle?

A. Adductor longus
B. Pectineus

Question stats

Score: 77.8%
1

15.4%

9.8%

9%

57.9%

7.8%

57.9% of users answered this


question correctly

C. Adductor magnus

End and review

D. Sartorius

6
7
8

E. Conjoint tendon

Search
Next question

Go

The sartorius forms the lateral wall of the femoral triangle (see below).
Femoral triangle anatomy
Boundaries
Superiorly

Inguinal ligament

Laterally

Sartorius

Medially

Adductor longus

Floor

Iliopsoas, adductor longus and pectineus

Roof

Fascia lata and Superficial fascia


Superficial inguinal lymph nodes (palpable below the inguinal ligament)
Great saphenous vein

Image sourced from Wikipedia

Contents
Femoral vein (medial to lateral)
Femoral artery-pulse palpated at the mid inguinal point
Femoral nerve
Deep and superficial inguinal lymph nodes
Lateral cutaneous nerve
Great saphenous vein
Femoral branch of the genitofemoral nerve

Rate question:

Next question

Comment on this question

http://www.emrcs.com/question/question.php?q=0

1/2

18/05/2015

All contents of this site are 2012 E-Medical Revision Ltd

http://www.emrcs.com/question/question.php?q=0

Terms and Conditions

Privacy policy

2/2

18/05/2015

Reference ranges

Previous

Question 9 of 169

Next

An 18 year old man develops a severe spreading sepsis of the hand. The palm is explored
surgically and the flexor digiti minimi brevis muscle is mobilised to facilitate drainage of the
infection. Which of the following structures is not closely related to this muscle?

Question stats

Score: 70%
1

14%

54.1%

10.9%

12.1%

8.9%

A. The hook of hamate


54.1% of users answered this
question correctly

B. Median nerve

End and review

6
7

C. Superficial palmar arterial arch


8

D. Digital nerves arising from the ulnar nerve


9

Search

E. None of the above

Go

Next question

The flexor digiti minimi brevis originates from the Hamate, on its under- surface lie the ulnar
contribution to the superficial palmar arterial arch and digital nerves derived from the ulnar
nerve. The median nerve overlies the flexor tendons.
Hand
Anatomy of the hand
Bones

8 Carpal bones
5 Metacarpals
14 phalanges

Intrinsic Muscles

7 Interossei - Supplied by ulnar nerve


3 palmar-adduct fingers
4 dorsal- abduct fingers

Intrinsic muscles

Lumbricals
Flex MCPJ and extend the IPJ.
Origin deep flexor tendon and insertion dorsal extensor hood
mechanism.
Innervation: 1st and 2nd- median nerve, 3rd and 4th- deep branch of
the ulnar nerve.

Thenar eminence

Hypothenar
eminence

Abductor pollicis brevis


Opponens pollicis
Flexor pollicis brevis

Opponens digiti minimi


Flexor digiti minimi brevis
Abductor digiti minimi

Image sourced from Wikipedia

Fascia and compartments of the palm

http://www.emrcs.com/question/question.php?q=0

1/2

18/05/2015

The fascia of the palm is continuous with the antebrachial fascia and the fascia of the
dorsum of the hand. The palmar fascia is thin over the thenar and hypothenar eminences. In
contrast the palmar fascia is relatively thick. The palmar aponeurosis covers the soft tissues
and overlies the flexor tendons. The apex of the palmar aponeurosis is continuous with the
flexor retinaculum and the palmaris longus tendon. Distally, it forms four longitudinal digital
bands that attach to the bases of the proximal phalanges, blending with the fibrous digital
sheaths.
A medial fibrous septum extends deeply from the medial border of the palmar aponeurosis to
the 5th metacarpal. Lying medial to this are the hypothenar muscles. In a similar fashion, a
lateral fibrous septum extends deeply from the lateral border of the palmar aponeurosis to
the 3rd metacarpal. The thenar compartment lies lateral to this area.
Lying between the thenar and hypothenar compartments is the central compartment. It
contains the flexor tendons and their sheaths, the lumbricals, the superficial palmar arterial
arch and the digital vessels and nerves.
The deepest muscular plane is the adductor compartment, which contains adductor pollicis.
Short muscles of the hand
These comprise the lumbricals and interossei. The four slender lumbrical muscles flex the
fingers at the metacarpophalangeal joints and extend the interphalangeal joint. The four
dorsal interossei are located between the metacarpals and the four palmar interossei lie on
the palmar surface of the metacarpals in the interosseous compartment of the hand.
Long flexor tendons and sheaths in the hand
The tendons of FDS and FDP enter the common flexor sheath deep to the flexor
retinaculum. The tendons enter the central compartment of the hand and fan out to their
respective digital synovial sheaths. Near the base of the proximal phalanx, the tendon of FDS
splits to permit the passage of FDP. The FDP tendons are attached to the margins of the
anterior aspect of the base of the distal phalanx.
The fibrous digital sheaths contain the flexor tendons and their synovial sheaths. These
extend from the heads of the metacarpals to the base of the distal phalanges.
Rate question:

Next question

Comment on this question

All contents of this site are 2012 E-Medical Revision Ltd

http://www.emrcs.com/question/question.php?q=0

Terms and Conditions

Privacy policy

2/2

18/05/2015

Reference ranges

Question 10 of 169

Previous

Next

A 19 year old man undergoes an open inguinal hernia repair. The cord is mobilised and the
deep inguinal ring identified. Which of the following structures forms its lateral wall?

A. External oblique aponeurosis


B. Transversalis fascia

Question stats

Score: 75%
1

26.7%

33.8%

17.8%

14.5%

7.2%

33.8% of users answered this


question correctly

C. Conjoint tendon

End and review

D. Inferior epigastric artery

6
7
8

E. Inferior epigastric vein

Search
Next question

Go

10

The transversalis fascia forms the superolateral edge of the deep inguinal ring. The
epigastric vessels form its inferomedial wall.
Inguinal canal
Location
Above the inguinal ligament
The inguinal canal is 4cm long
The superficial ring is located anterior to the pubic tubercle
The deep ring is located approximately 1.5-2cm above the half way point between the
anterior superior iliac spine and the pubic tubercle

Boundaries of the inguinal canal


Floor

External oblique aponeurosis


Inguinal ligament
Lacunar ligament

Roof

Internal oblique
Transversus abdominis

Anterior wall
Posterior wall

Laterally

Medially

External oblique aponeurosis


Transversalis fascia
Conjoint tendon

Internal ring
Fibres of internal oblique

External ring
Conjoint tendon

Contents
Males

Spermatic cord and ilioinguinal


nerve

As it passes through the canal the spermatic


cord has 3 coverings:
External spermatic fascia from external
oblique aponeurosis
Cremasteric fascia
Internal spermatic fascia

Females

Round ligament of uterus and


ilioinguinal nerve

Related anatomy of the inguinal region


The boundaries of Hesselbachs triangle are commonly tested and illustrated below:

http://www.emrcs.com/question/question.php?q=0

1/2

18/05/2015

Image sourced from Wikipedia

The image below demonstrates the close relationship of the vessels to the lower limb with the
inguinal canal. A fact to be borne in mind when repairing hernial defects in this region.

Image sourced from Wikipedia

Rate question:

Next question

Comment on this question

All contents of this site are 2012 E-Medical Revision Ltd

http://www.emrcs.com/question/question.php?q=0

Terms and Conditions

Privacy policy

2/2

18/05/2015

Reference ranges

Previous

Question 11 of 169

Next

A 22 year old man develops an infection in the pulp of his little finger. What is the most
proximal site to which this infection may migrate?

A. The metacarpophalangeal joint


B. The distal interphalangeal joint

Question stats

Score: 76.9%
1

13.5%

23.7%

13.2%

33.7%

16%

33.7% of users answered this


question correctly

C. The proximal interphalangeal joint

End and review

D. Proximal to the flexor retinaculum

6
7
8

E. Immediately distal to the carpal tunnel

Search
Next question

Go

10
11

The 5th tendon sheath extends from the little finger to the proximal aspect of the carpal
tunnel. This carries a significant risk of allowing infections to migrate proximally.

Hand
Anatomy of the hand
Bones

8 Carpal bones
5 Metacarpals
14 phalanges

Intrinsic Muscles

7 Interossei - Supplied by ulnar nerve


3 palmar-adduct fingers
4 dorsal- abduct fingers

Intrinsic muscles

Lumbricals
Flex MCPJ and extend the IPJ.
Origin deep flexor tendon and insertion dorsal extensor hood
mechanism.
Innervation: 1st and 2nd- median nerve, 3rd and 4th- deep branch of
the ulnar nerve.

Thenar eminence

Hypothenar
eminence

Abductor pollicis brevis


Opponens pollicis
Flexor pollicis brevis

Opponens digiti minimi


Flexor digiti minimi brevis
Abductor digiti minimi

Image sourced from Wikipedia

Fascia and compartments of the palm


The fascia of the palm is continuous with the antebrachial fascia and the fascia of the

http://www.emrcs.com/question/question.php?q=0

1/2

18/05/2015

dorsum of the hand. The palmar fascia is thin over the thenar and hypothenar eminences. In
contrast the palmar fascia is relatively thick. The palmar aponeurosis covers the soft tissues
and overlies the flexor tendons. The apex of the palmar aponeurosis is continuous with the
flexor retinaculum and the palmaris longus tendon. Distally, it forms four longitudinal digital
bands that attach to the bases of the proximal phalanges, blending with the fibrous digital
sheaths.
A medial fibrous septum extends deeply from the medial border of the palmar aponeurosis to
the 5th metacarpal. Lying medial to this are the hypothenar muscles. In a similar fashion, a
lateral fibrous septum extends deeply from the lateral border of the palmar aponeurosis to
the 3rd metacarpal. The thenar compartment lies lateral to this area.
Lying between the thenar and hypothenar compartments is the central compartment. It
contains the flexor tendons and their sheaths, the lumbricals, the superficial palmar arterial
arch and the digital vessels and nerves.
The deepest muscular plane is the adductor compartment, which contains adductor pollicis.
Short muscles of the hand
These comprise the lumbricals and interossei. The four slender lumbrical muscles flex the
fingers at the metacarpophalangeal joints and extend the interphalangeal joint. The four
dorsal interossei are located between the metacarpals and the four palmar interossei lie on
the palmar surface of the metacarpals in the interosseous compartment of the hand.
Long flexor tendons and sheaths in the hand
The tendons of FDS and FDP enter the common flexor sheath deep to the flexor
retinaculum. The tendons enter the central compartment of the hand and fan out to their
respective digital synovial sheaths. Near the base of the proximal phalanx, the tendon of FDS
splits to permit the passage of FDP. The FDP tendons are attached to the margins of the
anterior aspect of the base of the distal phalanx.
The fibrous digital sheaths contain the flexor tendons and their synovial sheaths. These
extend from the heads of the metacarpals to the base of the distal phalanges.
Rate question:

Next question

Comment on this question

All contents of this site are 2012 E-Medical Revision Ltd

http://www.emrcs.com/question/question.php?q=0

Terms and Conditions

Privacy policy

2/2

18/05/2015

Reference ranges

Question 12 of 169

Previous

Next

Question stats

End and review

Score: 78.6%
1

14.8%

9.8%

11.3%

A. Adductor pollicis

52.9%

B. Hypothenar muscles

11.3%

C. All the interosseous muscles

52.9% of users answered this


question correctly

Which of the following muscles is not innervated by the deep branch of the ulnar nerve?

D. Opponens pollicis
E. Third and fourth lumbricals

6
7
8
9

Search
Next question
Go

10
11
12

Ulnar nerve
Origin
C8, T1

Supplies (no muscles in the upper arm)


Flexor carpi ulnaris
Flexor digitorum profundus
Flexor digiti minimi
Abductor digiti minimi
Opponens digiti minimi
Adductor pollicis
Interossei muscle
Third and fourth lumbricals
Palmaris brevis

Path
Posteromedial aspect of ulna to flexor compartment of forearm, then along the ulnar.
Passes beneath the flexor carpi ulnaris muscle, then superficially through the flexor
retinaculum into the palm of the hand.

Image sourced from Wikipedia

Branches
Branch

Supplies

Articular branch

Flexor carpi ulnaris


Medial half of the flexor digitorum profundus

http://www.emrcs.com/question/question.php?q=0

1/2

18/05/2015

Palmar cutaneous branch (Arises near the


middle of the forearm)

Skin on the medial part of the palm

Dorsal cutaneous branch

Dorsal surface of the medial part of the hand

Superficial branch

Cutaneous fibres to the anterior surfaces of the


medial one and one-half digits

Deep branch

Hypothenar muscles
All the interosseous muscles
Third and fourth lumbricals
Adductor pollicis
Medial head of the flexor pollicis brevis

Effects of injury
Damage at the wrist

Damage at the elbow

Wasting and paralysis of intrinsic hand muscles (claw hand)


Wasting and paralysis of hypothenar muscles
Loss of sensation medial 1 and half fingers

Radial deviation of the wrist


Clawing less in 3rd and 4th digits

Rate question:

Next question

Comment on this question

All contents of this site are 2012 E-Medical Revision Ltd

http://www.emrcs.com/question/question.php?q=0

Terms and Conditions

Privacy policy

2/2

18/05/2015

Reference ranges

Previous

Question 13 of 169

Next

During an inguinal hernia repair the surgeon identifies a small nerve whilst mobilising the
cord structures at the level of the superficial inguinal ring. Which nerve is this most likely to
be?

Question stats

Score: 80%
1

6.3%

10.8%

67.9%

7.5%

7.6%

A. Subcostal
67.9% of users answered this
question correctly

B. Iliohypogastric

End and review

6
7

C. Ilioinguinal
8

D. Obturator
9

Search

E. Pudendal

Go

10
11

Next question

12
13

Ilioinguinal nerve entrapment may be a cause of neuropathic pain following inguinal


hernia surgery.

The ilioinguinal nerve passes through the superfical inguinal ring and is routinely
encountered when exploring the inguinal canal during hernia surgery. The iliohypogastric
nerve pierces the aponeurosis of the external oblique muscle superior to the superficial
inguinal ring.
Ilioinguinal nerve
Arises from the first lumbar ventral ramus with the iliohypogastric nerve. It passes
inferolaterally through the substance of psoas major and over the anterior surface of
quadratus lumborum. It pierces the internal oblique muscle and passes deep to the
aponeurosis of the external oblique muscle. It enters the inguinal canal and then passes
through the superficial inguinal ring to reach the skin.
Branches
To supply those muscles of the abdominal wall through which it passes.
Skin and fascia over the pubic symphysis, superomedial part of the femoral triangle,
surface of the scrotum, root and dorsum of penis or labum majus in females.

Rate question:

Next question

Comment on this question

All contents of this site are 2012 E-Medical Revision Ltd

http://www.emrcs.com/question/question.php?q=0

Terms and Conditions

Privacy policy

1/1

25/05/2015

Reference ranges

Question 1 of 156

Next

Question stats

Score: 0%

16%

38%

17.8%

A. Posterolateral aspect of the lateral femoral condyle

18.7%

B. Posteromedial aspect of the lateral femoral condyle

9.5%

C. Posterolateral aspect of the medial femoral condyle

38% of users answered this


question correctly

Onto which of the following structures does the anterior cruciate ligament insert?

D. Posteromedial aspect of the medial femoral condyle

End and review

E. None of the above


Search
Next question
Go

The anterior cruciate ligament is attached to the anterior intercondylar area of the tibia. Is
then passes posterolaterally to insert into the posteromedial aspect of the lateral femoral
condyle.
Knee joint
The knee joint is a synovial joint, the largest and most complicated. It consists of two
condylar joints between the femure and tibia and a sellar joint between the patella and the
femur. The tibiofemoral articular surfaces are incongruent, however, this is improved by the
presence of the menisci. The degree of congruence is related to the anatomical position of
the knee joint and is greatest in full extension.
Knee joint compartments
Tibiofemoral

Comprise of the patella/femur joint, lateral and medial


compartments (betw een femur condyles and tibia)
Synovial membrane and cruciate ligaments partially separate the
medial and lateral compartments

Patellofemoral

Ligamentum patellae
Actions: provides joint stability in full extension

Fibrous capsule
The capsule of the knee joint is a complex, composite structure with contributions from
adjacent tendons.
Anterior
fibres

The capsule does not pass proximal to the patella. It blends w ith the
tendinous expansions of vastus medialis and lateralis

Posterior These fibres are vertical and run from the posterior surface of the femoral
fibres
condyles to the posterior aspect of the tibial condyle
Medial
fibres

Attach to the femoral and tibial condyles beyond their articular margins,
blending w ith the tibial collateral ligament

Lateral
fibres

Attach to the femur superior to popliteus, pass over its tendon to head of
fibula and tibial condyle

Bursae
Anterior

Subcutaneous prepatellar bursa; betw een patella and skin


Deep infrapatellar bursa; betw een tibia and patellar ligament
Subcutaneous infrapatellar bursa; betw een distal tibial tuberosity and
skin

Laterally

Bursa betw een lateral head of gastrocnemius and joint capsule


Bursa betw een fibular collateral ligament and tendon of biceps femoris
Bursa betw een fibular collateral ligament and tendon of popliteus

Medially

Bursa betw een medial head of gastrocnemius and the fibrous capsule
Bursa betw een tibial collateral ligament and tendons of sartorius, gracilis
and semitendinosus
Bursa betw een the tendon of semimembranosus and medial tibial
condyle and medial head of gastrocnemius

Posterior Highly variable and inconsistent

Ligaments
Medial collateral
ligament

Medial epicondyle femur to medial tibial condyle: valgus


stability

Lateral collateral
ligament

Lateral epicondyle femur to fibula head: varus stability

http://www.emrcs.com/question/question.php?q=0

1/2

25/05/2015

Anterior cruciate
ligament

Anterior tibia to lateral intercondylar notch femur: prevents


tibia sliding anteriorly

Posterior cruciate
ligament

Posterior tibia to medial intercondylar notch femur: prevents


tibia sliding posteriorly

Patellar ligament

Central band of the tendon of quadriceps femoris, extends


from patella to tibial tuberosity

Image sourced from Wikipedia

Image sourced from Wikipedia

Menisci
Medial and lateral menisci compensate for the incongruence of the femoral and tibial
condyles.
Composed of fibrous tissue.
Medial meniscus is attached to the tibial collateral ligament.
Lateral meniscus is attached to the loose fibres at the lateral edge of the joint and is
separate from the fibular collateral ligament. The lateral meniscus is crossed by the popliteus
tendon.
Nerve supply
The knee joint is supplied by the femoral, tibial and common peroneal divisions of the sciatic
and by a branch from the obturator nerve. Hip pathology pain may be referred to the knee.
Blood supply
Genicular branches of the femoral artery, popliteal and anterior tibial arteries all supply the
knee joint.
Rate question:

Next question

Comment on this question

All contents of this site are 2012 E-Medical Revision Ltd

http://www.emrcs.com/question/question.php?q=0

Terms and Conditions

Privacy policy

2/2

25/05/2015

Reference ranges

Previous

Question 2 of 156

Next

During an arch aortogram the brachiocephalic artery is entered with an angiography


catheter. The radiologist continues to advance the catheter. Into which of the following
vessels is it likely to enter?

Question stats

End and review

Score: 0%

15.7%

8%

56.9%

9.6%

9.9%

1
2

A. Left subclavian artery


56.9% of users answered this
question correctly

B. Left axillary artery


C. Right subclavian artery
D. Right axillary artery

Search

E. None of the above

Go

Next question

The axillary artery is a branch of the subclavian artery and although developmental
anomalies may occur they are rare. The catheter may also enter the right carotid. There is
no brachiocephalic artery on the left side.
Brachiocephalic artery
The brachiocephalic artery is the largest branch of the aortic arch. From its aortic origin it
ascends superiorly, it initially lies anterior to the trachea and then on its right hand side. It
branches into the common carotid and right subclavian arteries at the level of the
sternoclavicular joint.
Path
Origin- apex of the midline of the aortic arch
Passes superiorly and posteriorly to the right
Divides into the right subclavian and right common carotid artery
Relations
Anterior

Posterior

Right lateral

Left lateral

Sternohyoid
Sternothyroid
Thymic remnants
Left brachiocephalic vein
Right inferior thyroid veins

Trachea
Right pleura

Right brachiocephalic vein


Superior part of SVC

Thymic remnants
Origin of left common carotid
Inferior thyroid veins
Trachea (higher level)

Branches
Normally none but may have the thyroidea ima artery

http://www.emrcs.com/question/question.php?q=0

1/2

25/05/2015

Image sourced from Wikipedia

Rate question:

Next question

Comment on this question

All contents of this site are 2012 E-Medical Revision Ltd

http://www.emrcs.com/question/question.php?q=0

Terms and Conditions

Privacy policy

2/2

25/05/2015

Reference ranges

Previous

Question 3 of 156

Next

Which of the following structures lie between the lateral and medial heads of the triceps
muscle?

A. Radial nerve
B. Median nerve

Question stats

End and review

Score: 33.3%
1

57.4%

10.8%

10.4%

11.4%

9.9%

57.4% of users answered this


question correctly

C. Ulnar nerve
D. Axillary nerve
E. Medial cutaneous nerve of the forearm

Search
Go

Next question

The radial nerve runs in its groove on between the two heads. The ulnar nerve lies anterior
to the medial head. The axillary nerve passes through the quadrangular space. This lies
superior to lateral head of the triceps muscle and thus the lateral border of the quadrangular
space is the humerus. Therefore the correct answer is the radial nerve.
Triceps

Origin

Insertion

Long head- infraglenoid tubercle of the scapula.


Lateral head- dorsal surface of the humerus, lateral and proximal to the
groove of the radial nerve
Medial head- posterior surface of the humerus on the inferomedial side of
the radial groove and both of the intermuscular septae

Olecranon process of the ulna. Here the olecranon bursa is between the
triceps tendon and olecranon.
Some fibres insert to the deep fascia of the forearm, posterior capsule of
the elbow (preventing the capsule from being trapped between olecranon
and olecranon fossa during extension)

Innervation

Radial nerve

Blood
supply

Profunda brachii artery

Action

Elbow extension. The long head can adduct the humerus and and extend it from
a flexed position

Relations

The radial nerve and profunda brachii vessels lie between the lateral and medial
heads

Rate question:

Next question

Comment on this question

All contents of this site are 2012 E-Medical Revision Ltd

http://www.emrcs.com/question/question.php?q=0

Terms and Conditions

Privacy policy

1/1

25/05/2015

Reference ranges

Previous

Question 4 of 156

Next

Into which of the following structures does the superior part of the fibrous capsule of the
shoulder joint insert?

A. The surgical neck of the humerus


B. The body of the humerus

Question stats

End and review

Score: 50%
1

19.6%

6.9%

14.5%

18.5%

40.5%

40.5% of users answered this


question correctly

C. The bicipital groove


D. Immediately distal to the greater tuberosity
E. The anatomical neck of the humerus

Search
Next question

Go

The shoulder joint is a shallow joint, hence its great mobility. However, this comes at the
expense of stability. The fibrous capsule attaches to the anatomical neck superiorly and the
surgical neck inferiorly
Shoulder joint

Shallow synovial ball and socket type of joint.


It is an inherently unstable joint, but is capable to a wide range of movement.
Stability is provided by muscles of the rotator cuff that pass from the scapula to insert
in the greater tuberosity (all except sub scapularis-lesser tuberosity).

Glenoid labrum
Fibrocartilaginous rim attached to the free edge of the glenoid cavity
Tendon of the long head of biceps arises from within the joint from the supraglenoid
tubercle, and is fused at this point to the labrum.
The long head of triceps attaches to the infraglenoid tubercle

Fibrous capsule
Attaches to the scapula external to the glenoid labrum and to the labrum itself
(postero-superiorly)
Attaches to the humerus at the level of the anatomical neck superiorly and the surgical
neck inferiorly
Anteriorly the capsule is in contact with the tendon of subscapularis, superiorly with the
supraspinatus tendon, and posteriorly with the tendons of infraspinatus and teres
minor. All these blend with the capsule towards their insertion.
Two defects in the fibrous capsule; superiorly for the tendon of biceps. Anteriorly there
is a defect beneath the subscapularis tendon.
The inferior extension of the capsule is closely related to the axillary nerve at the
surgical neck and this nerve is at risk in anteroinferior dislocations. It also means that
proximally sited osteomyelitis may progress to septic arthritis.

Movements and muscles


Flexion

Anterior part of deltoid


Pectoralis major
Biceps
Coracobrachialis

Extension

Posterior deltoid
Teres major
Latissimus dorsi

Adduction

Pectoralis major
Latissimus dorsi
Teres major
Coracobrachialis

Abduction

Mid deltoid
Supraspinatus

Medial rotation

Subscapularis
Anterior deltoid
Teres major
Latissimus dorsi

Lateral rotation

Posterior deltoid
Infraspinatus

http://www.emrcs.com/question/question.php?q=0

1/2

25/05/2015

Teres minor
Important anatomical relations
Anteriorly

Brachial plexus
Axillary artery and vein

Posterior

Suprascapular nerve
Suprascapular vessels

Inferior

Axillary nerve
Circumflex humeral vessels

Rate question:

Next question

Comment on this question

All contents of this site are 2012 E-Medical Revision Ltd

http://www.emrcs.com/question/question.php?q=0

Terms and Conditions

Privacy policy

2/2

25/05/2015

Reference ranges

Previous

Question 4 of 156

Next

Into which of the following structures does the superior part of the fibrous capsule of the
shoulder joint insert?

A. The surgical neck of the humerus


B. The body of the humerus

Question stats

End and review

Score: 50%
1

19.6%

6.9%

14.5%

18.5%

40.5%

40.5% of users answered this


question correctly

C. The bicipital groove


D. Immediately distal to the greater tuberosity
E. The anatomical neck of the humerus

Search
Next question

Go

The shoulder joint is a shallow joint, hence its great mobility. However, this comes at the
expense of stability. The fibrous capsule attaches to the anatomical neck superiorly and the
surgical neck inferiorly
Shoulder joint

Shallow synovial ball and socket type of joint.


It is an inherently unstable joint, but is capable to a wide range of movement.
Stability is provided by muscles of the rotator cuff that pass from the scapula to insert
in the greater tuberosity (all except sub scapularis-lesser tuberosity).

Glenoid labrum
Fibrocartilaginous rim attached to the free edge of the glenoid cavity
Tendon of the long head of biceps arises from within the joint from the supraglenoid
tubercle, and is fused at this point to the labrum.
The long head of triceps attaches to the infraglenoid tubercle

Fibrous capsule
Attaches to the scapula external to the glenoid labrum and to the labrum itself
(postero-superiorly)
Attaches to the humerus at the level of the anatomical neck superiorly and the surgical
neck inferiorly
Anteriorly the capsule is in contact with the tendon of subscapularis, superiorly with the
supraspinatus tendon, and posteriorly with the tendons of infraspinatus and teres
minor. All these blend with the capsule towards their insertion.
Two defects in the fibrous capsule; superiorly for the tendon of biceps. Anteriorly there
is a defect beneath the subscapularis tendon.
The inferior extension of the capsule is closely related to the axillary nerve at the
surgical neck and this nerve is at risk in anteroinferior dislocations. It also means that
proximally sited osteomyelitis may progress to septic arthritis.

Movements and muscles


Flexion

Anterior part of deltoid


Pectoralis major
Biceps
Coracobrachialis

Extension

Posterior deltoid
Teres major
Latissimus dorsi

Adduction

Pectoralis major
Latissimus dorsi
Teres major
Coracobrachialis

Abduction

Mid deltoid
Supraspinatus

Medial rotation

Subscapularis
Anterior deltoid
Teres major
Latissimus dorsi

Lateral rotation

Posterior deltoid
Infraspinatus

http://www.emrcs.com/question/question.php?q=0

1/2

25/05/2015

Teres minor
Important anatomical relations
Anteriorly

Brachial plexus
Axillary artery and vein

Posterior

Suprascapular nerve
Suprascapular vessels

Inferior

Axillary nerve
Circumflex humeral vessels

Rate question:

Next question

Comment on this question

All contents of this site are 2012 E-Medical Revision Ltd

http://www.emrcs.com/question/question.php?q=0

Terms and Conditions

Privacy policy

2/2

25/05/2015

Reference ranges

Previous

Question 5 of 156

Next

A 34 year old lady presents with symptoms of faecal incontinence. Ten years previously she
gave birth to a child by normal vaginal delivery. Injury to which of the following nerves is most
likely to account for this process?

Question stats

End and review

Score: 60%
1

9%

7.3%

65.5%

11%

7.2%

A. Genitofemoral
65.5% of users answered this
question correctly

B. Ilioinguinal
C. Pudendal
D. Hypogastric autonomic nerve

Search

E. Obturator

Go

Next question

S2,3,4 keeps the poo up off the floor - POOdendal nerve

Theme from April 2012 Exam


Damage to the pudendal nerve is classically associated with faecal incontinence and it is for
this reason that sacral neuromodulation is a popular treatment for the condition. Injury to the
hypogastric autonomic nerves is an aetiological factor in the development of constipation.
Pudendal nerve
The pudendal nerve arises from nerve roots S2, S3 and S4 and exits the pelvis through the
greater sciatic foramen. It re-enters the perineum through the lesser sciatic foramen. It
travels inferior to give innervation to the anal sphincters and external urethral sphincter. It
also provides cutaneous innervation to the region of perineum surrounding the anus and
posterior vulva.
Traction and compression of the pudendal nerve by the foetus in late pregnancy may result
in late onset pudendal neuropathy which may be part of the process involved in the
development of faecal incontinence.
Rate question:

Next question

Comment on this question

All contents of this site are 2012 E-Medical Revision Ltd

http://www.emrcs.com/question/question.php?q=0

Terms and Conditions

Privacy policy

1/1

25/05/2015

Reference ranges

Previous

Question 6 of 156

Next

During a difficult thyroidectomy haemorrhage is noted from the thyroidea ima vessel. From
which structure does this vessel usually arise?

A. External carotid artery


B. Internal carotid artery

Question stats

Score: 66.7%
1

15.7%

9.7%

51.4%

6.8%

16.4%

51.4% of users answered this


question correctly

C. Brachiocephalic artery

End and review

D. Axillary artery
E. Superior thyroid artery

Search
Next question

Go

Rhyme isthmus location:


Rings 2,3,4 make the isthmus floor

This accessory vessel which usually lies at the inferior aspect of the gland is derived either
from the brachiocephalic artery or the arch of the aorta.
Thyroid gland

Right and left lobes connected by isthmus


Surrounded by sheath from pretracheal layer of deep fascia
Apex: Lamina of thyroid cartilage
Base: 4th-5th tracheal ring
Pyramidal lobe: from isthmus
May be attached to foramen caecum at the base of the tongue

Relations
Anteromedially

Posterolaterally
Medially

Posterior

Isthmus

Sternothyroid
Superior belly of omohyoid
Sternohyoid
Anterior aspect of sternocleidomastoid
Carotid sheath
Larynx
Trachea
Pharynx
Oesophagus
Cricothyroid muscle
External laryngeal nerve (near superior thyroid artery)
Recurrent laryngeal nerve (near inferior thyroid artery)

Parathyroid glands
Anastomosis of superior and inferior thyroid arteries

Anteriorly: Sternothyroids, sternohyoids, anterior jugular veins


Posteriorly: 2nd, 3rd, 4th tracheal rings (attached via Ligament of
Berry)

Blood Supply
Arterial

Venous

Superior thyroid artery (1st branch of external carotid)


Inferior thyroid artery (from thyrocervical trunk)
Thyroidea ima (in 10% of population -from brachiocephalic artery or aorta)

Superior and middle thyroid veins - into the IJV


Inferior thyroid vein - into the brachiocephalic veins

Rate question:
http://www.emrcs.com/question/question.php?q=0

Next question

1/2

25/05/2015

Comment on this question

All contents of this site are 2012 E-Medical Revision Ltd

http://www.emrcs.com/question/question.php?q=0

Terms and Conditions

Privacy policy

2/2

25/05/2015

Reference ranges

Previous

Question 7 of 156

Next

A 49 year old man undergoes a low anterior resection for cancer. He is assessed in the
outpatient clinic post operatively. His wounds are well healed. However, he complains of
impotence. Which of the following best explains this problem?

Question stats

Score: 57.1%
1

7.4%

8.9%

46.9%

9.4%

27.4%

A. Sciatic nerve injury


46.9% of users answered this
question correctly

B. Damage to the internal iliac artery

End and review

6
7

C. Damage to the nervi erigentes


D. Damage to the vas
Search

E. Damage to the genitofemoral nerve

Go

Next question

Theme from 2012 Exam


The penis takes autonomic nerves from the nervi erigentes that lie near the seminal vesicles.
These may be compromised by direct surgical trauma (such as use of diathermy in this area)
and also by radiotherapy that is used in these patients pre operatively. The result is that up
to 50% of patients may develop impotence following rectal cancer surgery.
Penile erection
Physiology of erection
Autonomic

Somatic
nerves

Sympathetic nerves originate from T11-L2 and parasympathetic nerves from


S2-4 join to form pelvic plexus.
Parasympathetic discharge causes erection, sympathetic discharge
causes ejaculation and detumescence.
Supplied by dorsal penile and pudendal nerves. Efferent signals are relayed from
Onufs nucleus (S2-4) to innervate ischiocavernosus and bulbocavernosus
muscles.

Autonomic discharge to the penis will trigger the veno-occlusive mechanism which triggers
the flow of arterial blood into the penile sinusoidal spaces. As the inflow increases the
increased volume in this space will secondarily lead to compression of the subtunical venous
plexus with reduced venous return. During the detumesence phase the arteriolar constriction
will reduce arterial inflow and thereby allow venous return to normalise.
Priapism
Prolonged unwanted erection, in the absence of sexual desire, lasting more than 4 hours.
Classification of priaprism
Low flow priaprism

Due to veno-occlusion (high intracavernosal pressures).


Most common type
Often painful
Often low cavernosal flow
If present for >4 hours requires emergency treatment

High flow priaprism

Due to unregulated arterial blood flow.


Usually presents as semi rigid painless erection

Recurrent priaprism

Typically seen in sickle cell disease, most commonly of high flow type.

Causes
Intracavernosal drug therapies (e.g. for erectile dysfunction>
Blood disorders such as leukaemia and sickle cell disease
Neurogenic disorders such as spinal cord transection
Trauma to penis resulting in arterio-venous malformations

Tests
Exclude sickle cell/ leukaemia
Consider blood sampling from cavernosa to determine whether high or low flow (low
flow is often hypoxic)

Management
http://www.emrcs.com/question/question.php?q=0

1/2

25/05/2015

Ice packs/ cold showers


If due to low flow then blood may be aspirated from copora or try intracavernosal alpha
adrenergic agonists.
Delayed therapy of low flow priaprism may result in erectile dysfunction.

Rate question:

Next question

Comment on this question

All contents of this site are 2012 E-Medical Revision Ltd

http://www.emrcs.com/question/question.php?q=0

Terms and Conditions

Privacy policy

2/2

25/05/2015

Reference ranges

Previous

Question 8 of 156

Next

The cephalic vein pierces the clavipectoral fascia to terminate in which of the veins listed
below?

A. External jugular
B. Axillary

Question stats

Score: 50%
1

8.3%

62.3%

8.4%

7.8%

13.2%

62.3% of users answered this


question correctly

C. Internal jugular

End and review

D. Azygos

6
7
8

E. Brachial

Search
Go

Next question

Cephalic vein
Path
Dorsal venous arch drains laterally into the cephalic vein
Crosses the anatomical snuffbox and travels laterally up the arm
At the antecubital fossa connected to the basilic vein by the median cubital vein
Pierces deep fascia of deltopectoral groove to join axillary vein

Rate question:

Next question

Comment on this question

All contents of this site are 2012 E-Medical Revision Ltd

http://www.emrcs.com/question/question.php?q=0

Terms and Conditions

Privacy policy

1/1

25/05/2015

Reference ranges

Previous

Question 9 of 156

Next

A 43 year old lady develops a cerebello-pontine angle lesion. Which of the nerves listed
below is likely to be affected first?

A. CN X
B. CN III

Question stats

Score: 44.4%
1

10.7%

21.4%

35.4%

19.4%

13.1%

35.4% of users answered this


question correctly

C. CN V

End and review

D. CN IX

6
7
8

E. CN XII

Search
Next question

Go

Theme from September 2013 Exam


The most likely lesion to occur in the cerebello-pontine angle is an acoustic neuroma.
The trigeminal nerve has a broad base and involvement of at least part of this nerve is the
most likely initial finding. The defect may be subtle such as loss of the ipsilateral corneal
reflex. Ipsilateral hearing loss will also occur. Untreated, progressive lesions, may ultimately
affect may cranial nerve roots in this region.
Cranial nerves
Cranial nerve lesions
Olfactory nerve

May be injured in basal skull fractures or involved in frontal lobe tumour


extension. Loss of olfactory nerve function in relation to major CNS
pathology is seldom an isolated event and thus it is poor localiser of CNS
pathology.

Optic nerve

Problems with visual acuity may result from intra ocular disorders.
Problems with the blood supply such as amaurosis fugax may produce
temporary visual distortion. More important surgically is the pupillary
response to light. The pupillary size may be altered in a number of
disorders. Nerves involved in the resizing of the pupil connect to the
pretectal nucleus of the high midbrain, bypassing the lateral geniculate
nucleus and the primary visual cortex. From the pretectal nucleus
neurones pass to the Edinger - Westphal nucleus, motor axons from here
pass along with the oculomotor nerve. They synapse with ciliary ganglion
neurones; the parasympathetic axons from this then innervate the iris and
produce miosis. The miotic pupil is seen in disorders such as Horner's
syndrome or opiate overdose.
Mydriasis is the dilatation of the pupil in response to disease, trauma,
drugs (or the dark!). It is pathological when light fails to induce miosis. The
radial muscle is innervated by the sympathetic nervous system. Because
the parasympathetic fibres travel with the oculomotor nerve they will be
damaged by lesions affecting this nerve (e.g. cranial trauma).
The response to light shone in one eye is usually a constriction of both
pupils. This indicates intact direct and consensual light reflexes. When
the optic nerve has an afferent defect the light shining on the affected eye
will produce a diminished pupillary response in both eyes. Whereas light
shone on the unaffected eye will produce a normal pupillary response in
both eyes. This is referred to as the Marcus Gunn pupil and is seen in
conditions such as optic neuritis. In a total CN II lesion shining the light in
the affected eye will produce no response.

Oculomotor nerve

The pupillary effects are described above. In addition it supplies all ocular
muscles apart from lateral rectus and superior oblique. Thus the affected
eye will be deviated inferolaterally. Levator palpebrae superioris may also
be impaired resulting in impaired ability to open the eye.

Trochlear nerve

The eye will not be able to look down.

Trigeminal nerve

Largest cranial nerve. Exits the brainstem at the pons. Branches are
ophthalmic, maxillary and mandibular. Only the mandibular branch has
both sensory and motor fibres. Branches converge to form the trigeminal
ganglion (located in Meckels cave). It supplies the muscles of mastication
and also tensor veli palatine, mylohyoid, anterior belly of digastric and
tensor tympani. The detailed descriptions of the various sensory functions
are described in other areas of the website. The corneal reflex is important
and is elicited by applying a small tip of cotton wool to the cornea, a reflex
blink should occur if it is intact. It is mediated by: the naso ciliary branch
of the ophthalmic branch of the trigeminal (sensory component) and the
facial nerve producing the motor response. Lesions of the afferent arc will
produce bilateral absent blink and lesions of the efferent arc will result in a
unilateral absent blink.

Abducens nerve

The affected eye will have a deficit of abduction. This cranial nerve exits
the brainstem between the pons and medulla. It thus has a relatively long

http://www.emrcs.com/question/question.php?q=0

1/2

25/05/2015

intra cranial course which renders it susceptible to damage in raised intra


cranial pressure.
Facial nerve

Emerges from brainstem between pons and medulla. It controls muscles


of facial expression and taste from the anterior 2/3 of the tongue. The
nerve passes into the petrous temporal bone and into the internal auditory
meatus. It then passes through the facial canal and exits at the
stylomastoid foramen. It passes through the parotid gland and divides at
this point. It does not innervate the parotid gland. Its divisions are
considered in other parts of the website. Its motor fibres innervate
orbicularis oculi to produce the efferent arm of the corneal reflex. In
surgical practice it may be injured during parotid gland surgery or invaded
by malignancies of the gland and a lower motor neurone on the ipsilateral
side will result.

Vestibulocochlear nerve

Exits from the pons and then passes through the internal auditory
meatus. It is implicated in sensorineural hearing loss. Individuals with
sensorineural hearing loss will localise the sound in webers test to the
normal ear. Rinnes test will be reduced on the affected side but should
still work. These two tests will distinguish sensorineural hearing loss from
conductive deafness. In the latter condition webers test will localise to the
affected ear and Rinnes test will be impaired on the affected side. Surgical
lesions affecting this nerve include CNS tumours and basal skull fractures.
It may also be damaged by the administration of ototoxic drugs (of which
gentamicin is the most commonly used in surgical practice).

Glossopharyngeal
nerve

Exits the pons just above the vagus. Receives sensory fibres from
posterior 1/3 tongue, tonsils, pharynx and middle ear (otalgia may occur
following tonsillectomy). It receives visceral afferents from the carotid
bodies. It supplies parasympathetic fibres to the parotid gland via the otic
ganglion and motor function to stylopharyngeaus muscle. The sensory
function of the nerve is tested using the gag reflex.

Vagus nerve

Leaves the medulla between the olivary nucleus and the inferior cerebellar
peduncle. Passes through the jugular foramen and into the carotid sheath.
Details of the functions of the vagus nerve are covered in the website
under relevant organ sub headings.

Accessory nerve

Exists from the caudal aspect of the brainstem (multiple branches)


supplies trapezius and sternocleidomastoid muscles. The distal portion of
this nerve is most prone to injury during surgical procedures.

Hypoglossal
nerve

Emerges from the medulla at the preolivary sulcus, passes through the
hypoglossal canal. It lies on the carotid sheath and passes deep to the
posterior belly of digastric to supply muscles of the tongue (except
palatoglossus). Its location near the carotid sheath makes it vulnerable
during carotid endarterectomy surgery and damage will produce ipsilateral
defect in muscle function.

Rate question:

Next question

Comment on this question

All contents of this site are 2012 E-Medical Revision Ltd

http://www.emrcs.com/question/question.php?q=0

Terms and Conditions

Privacy policy

2/2

25/05/2015

Reference ranges

Previous

Question 10 of 156

Next

Question stats

End and review

Score: 40%
1

7.9%

17.3%

7.6%

A. Inferior mesenteric artery

59.6%

B. Inferior phrenic artery

7.6%

C. Superior mesenteric artery

59.6% of users answered this


question correctly

Which of the following is not a branch of the abdominal aorta?

D. Superior phrenic artery


E. Renal artery

6
7
8
9

Search
Next question
Go

10

Mnemonic for the Descending abdominal aorta branches from diaphragm to iliacs:
'Prostitutes Cause Sagging Swollen Red Testicles [in men] Living In Sin':
Phrenic [inferior]
Celiac
Superior mesenteric
Suprarenal [middle]
Renal
Testicular ['in men' only]
Lumbars
Inferior mesenteric
Sacral

The superior phrenic artery branches from the aorta in the thorax.
Abdominal aortic branches

Branches

Level

Paired

Type

Inferior phrenic

T12 (Upper border)

Yes

Parietal

Coeliac

T12

No

Visceral

Superior mesenteric

L1

No

Visceral

Middle suprarenal

L1

Yes

Visceral

Renal

L1-L2

Yes

Visceral

Gonadal

L2

Yes

Visceral

Lumbar

L1-L4

Yes

Parietal

Inferior mesenteric

L3

No

Visceral

Median sacral

L4

No

Parietal

Common iliac

L4

Yes

Terminal

Rate question:

Next question

Comment on this question

All contents of this site are 2012 E-Medical Revision Ltd

http://www.emrcs.com/question/question.php?q=0

Terms and Conditions

Privacy policy

1/1

26/05/2015

Reference ranges

Question 1 of 146

Next

A 40 year old lady presents with varicose veins, these are found to originate from the short
saphenous vein. As the vein is mobilised close to its origin which structure is at greatest risk
of injury?

Question stats

End and review

Score: 100%

7.6%

59%

14.7%

10.6%

8%

A. Sciatic nerve
59% of users answered this
question correctly

B. Sural nerve
C. Common peroneal nerve
D. Tibial nerve

Search

E. Popliteal artery

Go

Next question

The sural nerve is closely related and damage to this structure is a major cause of litigation.
The other structures may all be injured but the risks are lower.
Popliteal fossa
Boundaries of the popliteal fossa
Laterally

Biceps femoris above, lateral head of gastrocnemius and plantaris below

Medially

Semimembranosus and semitendinosus above, medial head of gastrocnemius


below

Floor

Popliteal surface of the femur, posterior ligament of knee joint and popliteus muscle

Roof

Superficial and deep fascia

Image showing the popliteal fossa

Image sourced from Wikipedia

Contents
Popliteal artery and vein
Small saphenous vein
Common peroneal nerve
Tibial nerve
Posterior cutaneous nerve of the thigh
Genicular branch of the obturator nerve
Lymph nodes

Rate question:

Next question

Comment on this question

All contents of this site are 2012 E-Medical Revision Ltd

http://www.emrcs.com/question/question.php?q=0

Terms and Conditions

Privacy policy

1/1

26/05/2015

Reference ranges

Previous

Question 2 of 146

Next

A 23 year old man is admitted with a suspected ureteric colic. A KUB style x-ray is obtained.
In which of the following locations is the stone most likely to be visualised?

A. The tips of the transverse processes between L2 and L5


B. The tips of transverse processes between T10-L1

Question stats

End and review

Score: 100%

51.5%

11.9%

18.4%

7.7%

10.5%

1
2

51.5% of users answered this


question correctly

C. At the crest of the ilium


D. Over the S3 foramina
E. Over the sacrococcygeal joint

Search
Go

Next question

The ureter lies anterior to L2 to L5 and stones may be visualised at these points, they may
also be identified over the sacro-iliac joints.
Ureter

25-35 cm long
Muscular tube lined by transitional epithelium
Surrounded by thick muscular coat. Becomes 3 muscular layers as it crosses the bony
pelvis
Retroperitoneal structure overlying transverse processes L2-L5
Lies anterior to bifurcation of iliac vessels
Blood supply is segmental; renal artery, aortic branches, gonadal branches, common
iliac and internal iliac
Lies beneath the uterine artery

Rate question:

Next question

Comment on this question

All contents of this site are 2012 E-Medical Revision Ltd

http://www.emrcs.com/question/question.php?q=0

Terms and Conditions

Privacy policy

1/1

26/05/2015

Reference ranges

Previous

Question 3 of 146

Next

A 72 year old man with non reconstructible arterial disease is undergoing an above knee
amputation. The posterior compartment muscles are divided. Which of the following muscles
does not lie in the posterior compartment of the thigh?

Question stats

End and review

Score: 100%
1

9.3%

67.7%

7.2%

7.3%

8.5%

A. Biceps femoris
67.7% of users answered this
question correctly

B. Quadriceps femoris
C. Semitendinosus
D. Semimembranosus

Search

E. None of the above

Go

Next question

The quadriceps femoris lies in the anterior compartment.


Fascial compartments of the leg
Compartments of the thigh
Formed by septae passing from the femur to the fascia lata.
Compartment

Nerve

Anterior compartment

Femoral

Medial compartment

Obturator

Posterior compartment
(2 layers)

Sciatic

Muscles
Iliacus
Tensor fasciae latae
Sartorius
Quadriceps femoris

Adductor
longus/magnus/brevis
Gracilis
Obturator externus

Semimembranosus
Semitendinosus
Biceps femoris

Blood supply
Femoral artery

Profunda femoris artery


and obturator artery

Branches of Profunda
femoris artery

Compartments of the lower leg


Separated by the interosseous membrane (anterior and posterior compartments), anterior
fascial septum (separate anterior and lateral compartments) and posterior fascial septum
(separate lateral and posterior compartments)
Compartment

Nerve

Anterior
compartment

Deep
peroneal
nerve

Posterior
compartment

Tibial

Lateral
compartment

Superficial
peroneal

Muscles

Rate question:

Blood
supply
Anterior
tibial
artery

Tibialis anterior
Extensor digitorum longus
Extensor hallucis longus
Peroneus tertius

Muscles: deep and superficial


compartments (separated by deep
transverse fascia)
Deep: Flexor hallucis longus, Flexor digitalis
longus, Tibialis posterior, Popliteus
Superficial: Gastrocnemius, Soleus,
Plantaris

Peroneus longus/brevis

Posterior
tibial

Anterior
tibial

Next question

Comment on this question

http://www.emrcs.com/question/question.php?q=0

1/2

26/05/2015

All contents of this site are 2012 E-Medical Revision Ltd

http://www.emrcs.com/question/question.php?q=0

Terms and Conditions

Privacy policy

2/2

26/05/2015

Reference ranges

Previous

Question 4 of 146

Next

A woman develops winging of the scapula following a Patey mastectomy. What is the most
likely cause?

A. Division of pectoralis minor to access level 3 axillary nodes


B. Damage to the brachial plexus during axillary dissection

Question stats

End and review

Score: 100%
1

7.5%

7%

70.2%

7.2%

8.1%

70.2% of users answered this


question correctly

C. Damage to the long thoracic nerve during axillary dissection


D. Division of the thoracodorsal trunk during axillary dissection
E. Damage to the thoracodorsal trunk during axillary dissection

Search
Go

Next question

Theme from January 2012 exam


The serratus anterior muscle is supplied by the long thoracic nerve which runs along the
surface of serratus anterior and is liable to injury during nodal dissection. Although pectoralis
minor is divided during a Patey mastectomy (now seldom performed) it is rare for this alone
to produce winging of the scapula.
Long thoracic nerve

Derived from ventral rami of C5, C6, and C7 (close to their emergence from
intervertebral foramina)
It runs downward and passes either anterior or posterior to the middle scalene muscle
It reaches upper tip of serratus anterior muscle and descends on outer surface of this
muscle, giving branches into it
Winging of Scapula occurs in long thoracic nerve injury (most common) or from spinal
accessory nerve injury (which denervates the trapezius) or a dorsal scapular nerve
injury

Rate question:

Next question

Comment on this question

All contents of this site are 2012 E-Medical Revision Ltd

http://www.emrcs.com/question/question.php?q=0

Terms and Conditions

Privacy policy

1/1

26/05/2015

Reference ranges

Previous

Question 5 of 146

Next

Question stats

End and review

Score: 100%
1

19.8%

7.8%

50.1%

A. In the pelvis

8.5%

B. On the contralateral side

13.8%

C. In its usual position

50.1% of users answered this


question correctly

In a patient with an ectopic kidney where is the adrenal gland most likely to be located?

D. Superior to the spleen


E. It will be absent

Search
Next question
Go

Because the kidney is present, rather than absent, the adrenal will usual develop and in the
normal location.
Adrenal gland embryology
First detected at 6 weeks' gestation, the adrenal cortex is derived from the mesoderm of the
posterior abdominal wall. Steroid secretion from the fetal cortex begins shortly thereafter.
Adult-type zona glomerulosa and fasciculata are detected in fetal life but make up only a
small proportion of the gland, and the zona reticularis is not present at all. The fetal cortex
predominates throughout fetal life. The adrenal medulla is of ectodermal origin, arising from
neural crest cells that migrate to the medial aspect of the developing cortex.
The fetal adrenal gland is relatively large. At 4 months' gestation, it is 4 times the size of the
kidney; however, at birth, it is a third of the size of the kidney. This occurs because of the
rapid regression of the fetal cortex at birth. It disappears almost completely by age 1 year; by
age 4-5 years, the permanent adult-type adrenal cortex has fully developed.
Anatomic anomalies of the adrenal gland may occur. Because the development of the
adrenals is closely associated with that of the kidneys, agenesis of an adrenal gland is
usually associated with ipsilateral agenesis of the kidney, and fused adrenal glands (whereby
the 2 glands join across the midline posterior to the aorta) are also associated with a fused
kidney.
Adrenal hypoplasia occurs in the following 2 forms: (1) hypoplasia or absence of the fetal
cortex with a poorly formed medulla and (2) disorganized fetal cortex and medulla with no
permanent cortex present. Adrenal heterotopia describes a normal adrenal gland in an
abnormal location, such as within the renal or hepatic capsules. Accessory adrenal tissue
(adrenal rests), which is usually comprised only of cortex but seen combined with medulla in
some cases, is most commonly located in the broad ligament or spermatic cord but can be
found anywhere within the abdomen. Even intracranial adrenal rests have been reported
Rate question:

Next question

Comment on this question

All contents of this site are 2012 E-Medical Revision Ltd

http://www.emrcs.com/question/question.php?q=0

Terms and Conditions

Privacy policy

1/1

26/05/2015

Reference ranges

Question 6 of 146

Previous

Next

Question stats

End and review

Score: 83.3%
1

12%

13.4%

37.2%

A. Soleus posteriorly

14.8%

B. Tibial nerve medially

22.6%

C. Deep peroneal nerve laterally

37.2% of users answered this


question correctly

Which of the following structures is not closely related to the posterior tibial artery?

D. Flexor hallucis longus postero-inferiorly

E. Popliteus
Search
Next question
Go

The deep peroneal nerve lies in the anterior compartment. The tibial nerve lies medially. At
its termination it lies deep to the flexor retinaculum.
Posterior tibial artery

Larger terminal branch of the popliteal artery


Terminates by dividing into the medial and lateral plantar arteries
Accompanied by two veins throughout its length
Position of the artery corresponds to a line drawn from the lower angle of the popliteal
fossa, at the level of the neck of the fibula, to a point midway between the medial
malleolus and the most prominent part of the heel

Relations of the posterior tibial artery


Proximal to distal
Anteriorly

Tibialis posterior
Flexor digitorum longus
Posterior surface of tibia and ankle joint

Posterior

Tibial nerve 2.5 cm distal to its origin


Fascia overlying the deep muscular layer
Proximal part covered by gastrocnemius and soleus
Distal part covered by skin and fascia

Rate question:

Next question

Comment on this question

All contents of this site are 2012 E-Medical Revision Ltd

http://www.emrcs.com/question/question.php?q=0

Terms and Conditions

Privacy policy

1/1

26/05/2015

Reference ranges

Previous

Question 7 of 146

Next

Question stats

End and review

Score: 71.4%
1

9%

40.9%

29.3%

A. Lunate bone

9.2%

B. Scaphoid bone

11.6%

C. Ulnar nerve

29.3% of users answered this


question correctly

Which of the following is not closely related to the capitate bone?

D. Hamate bone

6
7

E. Trapezoid bone
Search
Next question
Go

The ulnar nerve and artery lie adjacent to the pisiform bone. The capitate bone articulates
with the lunate, scaphoid, hamate and trapezoid bones, which are therefore closely related to
it.
Capitate bone
This is the largest of the carpal bones. It is centrally placed with a rounded head set into the
cavities of the lunate and scaphoid bones. Flatter articular surfaces are present for the
hamate medially and the trapezoid laterally. Distally the bone articulates predominantly with
the middle metacarpal.
Rate question:

Next question

Comment on this question

All contents of this site are 2012 E-Medical Revision Ltd

http://www.emrcs.com/question/question.php?q=0

Terms and Conditions

Privacy policy

1/1

26/05/2015

Reference ranges

Question 8 of 146

Previous

Next

Which of the following statements relating to the tympanic membrane is false?

A. The umbo marks the point of attachment of the handle of the malleus to
the tympanic membrane
B. The lateral aspect of the tympanic membrane is lined by stratified
squamous epithelium

Question stats

Score: 75%

E. The tympanic membrane is approximately 1cm in diameter

14.4%

12.9%

38%

22%

12.7%

38% of users answered this


question correctly

C. The chorda tympani nerve runs medial to the pars tensa


D. The medial aspect of the tympanic membrane is lined by mucous
membrane

End and review

6
7
8

Search
Go

Next question

The chorda tympani runs medially to the pars flaccida. The relationship is shown from the
medial aspect in the dissection below.

Image sourced from Wikipedia

Ear- anatomy
The ear is composed of three anatomically distinct regions.
External ear
Auricle is composed of elastic cartilage covered by skin. The lobule has no cartilage and
contains fat and fibrous tissue.
External auditory meatus is approximately 2.5cm long.
Lateral third of the external auditory meatus is cartilaginous and the medial two thirds is
bony.
The region is innervated by the greater auricular nerve. The auriculotemporal branch of the
trigeminal nerve supplies most the of external auditory meatus and the lateral surface of the
auricle.

Middle ear
Space between the tympanic membrane and cochlea. The aditus leads to the mastoid air
cells is the route through which middle ear infections may cause mastoiditis. Anteriorly the
eustacian tube connects the middle ear to the naso pharynx.
The tympanic membrane consists of:
Outer layer of stratified squamous epithelium.
Middle layer of fibrous tissue.
Inner layer of mucous membrane continuous with the middle ear.
The tympanic membrane is approximately 1cm in diameter.
The chorda tympani nerve passes on the medial side of the pars flaccida.
The middle ear is innervated by the glossopharyngeal nerve and pain may radiate to the
middle ear following tonsillectomy.
Ossicles

http://www.emrcs.com/question/question.php?q=0

1/2

26/05/2015

Malleus attaches to the tympanic membrane (the Umbo).


Malleus articulates with the incus (synovial joint).
Incus attaches to stapes (another synovial joint).
Internal ear
Cochlea, semi circular canals and vestibule
Organ of corti is the sense organ of hearing and is located on the inside of the cochlear duct
on the basilar membrane.
Vestibule accommodates the utricule and the saccule. These structures contain endolymph
and are surrounded by perilymph within the vestibule.
The semicircular canals lie at various angles to the petrous temporal bone. All share a
common opening into the vestibule.
Rate question:

Next question

Comment on this question

All contents of this site are 2012 E-Medical Revision Ltd

http://www.emrcs.com/question/question.php?q=0

Terms and Conditions

Privacy policy

2/2

26/05/2015

Reference ranges

Previous

Question 9 of 146

Next

Question stats

End and review

Score: 60%
1

7.9%

9.5%

21.8%

A. Lateral rotation of the arm

47.2%

B. Adduction of the arm at the glenohumeral joint

13.6%

C. Protraction of the scapula

47.2% of users answered this


question correctly

An injury to the spinal accessory nerve will affect which of the following movements?

D. Upward rotation of the scapula


E. Depression of the scapula

6
7
8
9

Search
Next question
Go

The spinal accessory nerve innervates trapezius. The entire muscle will retract the scapula.
However, its upper and lower fibres act together to upwardly rotate it.
Shoulder joint

Shallow synovial ball and socket type of joint.


It is an inherently unstable joint, but is capable to a wide range of movement.
Stability is provided by muscles of the rotator cuff that pass from the scapula to insert
in the greater tuberosity (all except sub scapularis-lesser tuberosity).

Glenoid labrum
Fibrocartilaginous rim attached to the free edge of the glenoid cavity
Tendon of the long head of biceps arises from within the joint from the supraglenoid
tubercle, and is fused at this point to the labrum.
The long head of triceps attaches to the infraglenoid tubercle

Fibrous capsule
Attaches to the scapula external to the glenoid labrum and to the labrum itself
(postero-superiorly)
Attaches to the humerus at the level of the anatomical neck superiorly and the surgical
neck inferiorly
Anteriorly the capsule is in contact with the tendon of subscapularis, superiorly with the
supraspinatus tendon, and posteriorly with the tendons of infraspinatus and teres
minor. All these blend with the capsule towards their insertion.
Two defects in the fibrous capsule; superiorly for the tendon of biceps. Anteriorly there
is a defect beneath the subscapularis tendon.
The inferior extension of the capsule is closely related to the axillary nerve at the
surgical neck and this nerve is at risk in anteroinferior dislocations. It also means that
proximally sited osteomyelitis may progress to septic arthritis.

Movements and muscles


Flexion

Anterior part of deltoid


Pectoralis major
Biceps
Coracobrachialis

Extension

Posterior deltoid
Teres major
Latissimus dorsi

Adduction

Pectoralis major
Latissimus dorsi
Teres major
Coracobrachialis

Abduction

Mid deltoid
Supraspinatus

Medial rotation

Subscapularis
Anterior deltoid
Teres major
Latissimus dorsi

Lateral rotation

Posterior deltoid
Infraspinatus
Teres minor

http://www.emrcs.com/question/question.php?q=0

1/2

26/05/2015

Important anatomical relations


Anteriorly

Brachial plexus
Axillary artery and vein

Posterior

Suprascapular nerve
Suprascapular vessels

Inferior

Axillary nerve
Circumflex humeral vessels

Rate question:

Next question

Comment on this question

All contents of this site are 2012 E-Medical Revision Ltd

http://www.emrcs.com/question/question.php?q=0

Terms and Conditions

Privacy policy

2/2

26/05/2015

Reference ranges

Previous

Question 10 of 146

Next

Question stats

End and review

Score: 54.5%
1

13.3%

23.1%

41.2%

A. Main bronchi

9.8%

B. Arch of the azygos vein

12.5%

C. Thoracic duct

41.2% of users answered this


question correctly

Which of the following is not contained within the middle mediastinum?

D. Pericardium
E. Aortic root

6
7
8
9

Search
Next question
Go

10

The thoracic duct lies within the posterior and superior mediastinum.
Mediastinum
Region between the pulmonary cavities.
It is covered by the mediastinal pleura. It does not contain the lungs.
It extends from the thoracic inlet superiorly to the diaphragm inferiorly.
Mediastinal regions
Superior mediastinum (between manubriosternal angle and T4/5)
Middle mediastinum
Posterior mediastinum
Anterior mediastinum

Region
Superior mediastinum

Anterior mediastinum

Middle mediastinum

Posterior mediastinum

Contents
Superior vena cava
Brachiocephalic veins
Arch of aorta
Thoracic duct
Trachea
Oesophagus
Thymus
Vagus nerve
Left recurrent laryngeal nerve
Phrenic nerve

Thymic remnants
Lymph nodes
Fat

Pericardium
Heart
Aortic root
Arch of azygos vein
Main bronchi

Oesophagus
Thoracic aorta
Azygos vein
Thoracic duct
Vagus nerve
Sympathetic nerve trunks
Splanchnic nerves

Rate question:

Next question

Comment on this question

All contents of this site are 2012 E-Medical Revision Ltd

http://www.emrcs.com/question/question.php?q=0

Terms and Conditions

Privacy policy

1/1

26/05/2015

Reference ranges

Question 11 of 146

Previous

Next

A 55 year old man is due to undergo a radical prostatectomy for carcinoma of the prostate
gland. Which of the following vessels directly supplies the prostate?

A. External iliac artery


B. Common iliac artery

Question stats

Score: 46.2%
1

8.1%

7%

22.4%

56%

6.6%

56% of users answered this


question correctly

C. Internal iliac artery

End and review

D. Inferior vesical artery

6
7
8

E. None of the above

Search
Next question

Go

10
11

The arterial supply to the prostate gland is from the inferior vesical artery, it is a branch of
the prostatovesical artery. The prostatovesical artery usually arises from the internal
pudendal and inferior gluteal arterial branches of the internal iliac artery.
Prostate gland
The prostate gland is approximately the shape and size of a walnut and is located inferior to
the bladder. It is separated from the rectum by Denonvilliers fascia and its blood supply is
derived from the internal iliac vessels. The internal sphincter lies at the apex of the gland and
may be damaged during prostatic surgery, affected individuals may complain of retrograde
ejaculation.
Summary of prostate gland
Arterial supply

Inferior vesical artery (from internal iliac)

Venous
drainage

Prostatic venous plexus (to paravertebral veins)

Lymphatic
drainage

Internal iliac nodes

Innervation

Inferior hypogastric plexus

Dimensions

Lobes

Zones

Transverse diameter (4cm)


AP diameter (2cm)
Height (3cm)

Posterior lobe: posterior to urethra


Median lobe: posterior to urethra, in between ejaculatory ducts
Lateral lobes x 2
Isthmus

Peripheral zone: subcapsular portion of posterior prostate. Most


prostate cancers are here
Central zone
Transition zone
Stroma

Relations
Anterior

Pubic symphysis
Prostatic venous plexus

Posterior

Denonvilliers fascia
Rectum
Ejaculatory ducts

Lateral

Venous plexus (lies on prostate)


Levator ani (immediately below the puboprostatic ligaments)

http://www.emrcs.com/question/question.php?q=0

1/2

26/05/2015

Image sourced from Wikipedia

Rate question:

Next question

Comment on this question

All contents of this site are 2012 E-Medical Revision Ltd

http://www.emrcs.com/question/question.php?q=0

Terms and Conditions

Privacy policy

2/2

26/05/2015

Reference ranges

Previous

Question 12 of 146

Next

Question stats

End and review

Score: 42.9%
1

17%

33.5%

18%

A. Superior cardiac nerve

7.2%

B. Right vagus nerve

24.3%

C. Left vagus nerve

24.3% of users answered this


question correctly

Which nerve directly innervates the sinoatrial node?

D. Inferior cardiac nerve


E. None of the above

6
7
8
9

Search
Next question
Go

10
11

Theme from September 2011 Exam


Theme from September 2012 Exam
No single one of the above nerves is responsible for direct cardiac innervation (which those
who have handled the heart surgically will appreciate).
The heart receives its nerves from the superficial and deep cardiac plexuses. The cardiac
plexuses send small branches to the heart along the major vessels, continuing with the right
and left coronary arteries. The vagal efferent fibres emerge from the brainstem in the roots
of the vagus and accessory nerves, and run to ganglia in the cardiac plexuses and within the
heart itself.

12

The background vagal discharge serves to limit heart rate, and loss of this background vagal
tone accounts for the higher resting heart rate seen following cardiac transplant.
Sinoatrial node

Located in the wall of the right atrium in the upper part of the sulcus terminalis from
which it extends anteriorly over the opening of the superior vena cava.
In most cases it is supplied by the right coronary artery.
It has a complicated nerve supply from the cardiac nerve plexus that takes both
sympathetic and parasympathetic fibres that run alongside the main vessels.

Rate question:

Next question

Comment on this question

All contents of this site are 2012 E-Medical Revision Ltd

http://www.emrcs.com/question/question.php?q=0

Terms and Conditions

Privacy policy

1/1

26/05/2015

Reference ranges

Previous

Question 13 of 146

Next

A 30 year old man presents with back pain and the surgeon tests the ankle reflex. Which of
the following nerve roots are tested in this manoeuvre?

A. S3 and S4
B. L4 and L5

Question stats

Score: 46.7%
1

10.6%

19.3%

11.4%

52.3%

6.4%

52.3% of users answered this


question correctly

C. L3 and L4

End and review

D. S1 and S2

6
7
8

E. S4 only

Search
Go

Next question

10
11
12

Theme from April 2012 Exam

13

Ankle reflex
The ankle reflex is elicited by tapping the Achilles tendon with a tendon hammer. It tests the
S1 and S2 nerve roots. It is typically delayed in L5 and S1 disk prolapses.
Rate question:

Next question

Comment on this question

All contents of this site are 2012 E-Medical Revision Ltd

http://www.emrcs.com/question/question.php?q=0

Terms and Conditions

Privacy policy

1/1

26/05/2015

Reference ranges

Previous

Question 14 of 146

Next

Question stats

End and review

Score: 52.9%
1

16.7%

12.3%

9.2%

A. Superior gluteal nerve

9.9%

B. Sciatic nerve

52%

C. Inferior gluteal artery

52% of users answered this


question correctly

Which of the following structures is not closely related to the piriformis muscle?

D. Inferior gluteal nerve


E. Medial femoral circumflex artery

6
7
8
9

Search
Next question
Go

10
11

Nerve supply of lateral hip rotators

12
13

Piriformis: ventral rami S1, S2


Obturator internus: nerve to obturator internus
Superior gemellus: nerve to obturator internus
Inferior gemellus: nerve to quadratus femoris
Quadrator femoris: nerve to quadrator femoris

14

The piriformis muscle is an important anatomical landmark in the gluteal region. The following
structures are closely related:
Sciatic nerve
Inferior gluteal artery and nerve
Superior gluteal artery and nerve

The medial femoral circumflex artery runs deep to quadratus femoris.


Gluteal region
Gluteal muscles
Gluteus maximus: inserts to gluteal tuberosity of the femur and iliotibial tract
Gluteus medius: attach to lateral greater trochanter
Gluteus minimis: attach to anterior greater trochanter
All extend and abduct the hip

Deep lateral hip rotators


Piriformis
Gemelli
Obturator internus
Quadratus femoris

Nerves
Superior gluteal nerve (L5, S1)

Inferior gluteal nerve

Gluteus medius
Gluteus minimis
Tensor fascia lata
Gluteus maximus

Damage to the superior gluteal nerve will result in the patient developing a Trendelenberg
gait. Affected patients are unable to abduct the thigh at the hip joint. During the stance
phase, the weakened abductor muscles allow the pelvis to tilt down on the opposite side. To
compensate, the trunk lurches to the weakened side to attempt to maintain a level pelvis
throughout the gait cycle. The pelvis sags on the opposite side of the lesioned superior
gluteal nerve.
Rate question:

Next question

Comment on this question

All contents of this site are 2012 E-Medical Revision Ltd

http://www.emrcs.com/question/question.php?q=0

Terms and Conditions

Privacy policy

1/1

26/05/2015

Reference ranges

Previous

Question 15 of 146

Next

An 18 year old male presents to casualty with a depressed skull fracture. This is managed
surgically. Over the next few days he complains of double vision on walking down stairs and
reading. On examination the left eye faces downwards and medially. Which of the nerves
listed below is most likely to be responsible?

Question stats

Score: 50%
1

6.7%

18.2%

29.9%

38.8%

6.5%

38.8% of users answered this


question correctly

A. Facial

End and review

B. Oculomotor

6
7
8

C. Abducens

Search

D. Trochlear
E. Trigeminal nerve

Go

10
11

Next question

12
13

Theme from September 2012 Exam


The trochlear nerve has a relatively long intracranial course and this makes it vulnerable to
injury in head trauma. Head trauma is the commonest cause of an acute fourth nerve palsy.
A 4th nerve palsy is the commonest cause of a vertical diplopia. The diplopia is at its worst
when the eye looks medially which it usually does as part of the accommodation reflex when
walking down stairs.

14
15

Disorders of the oculomotor system

Nerve

Oculomotor
nerve

Trochlear
nerve

Abducens
nerve

Path

Nerve palsy
features
Large nucleus at the midbrain
Fibres pass through the red nucleus and the
pyramidal tract; through the cavernous sinus into
the orbit

Longest intracranial course


Only nerve to exit the dorsal aspect of brainstem
Nucleus at midbrain, passes between the posterior
cerebral and superior cerebellar arteries, through
the cavernous sinus into the orbit
Nucleus lies in the mid pons

Rate question:

Ptosis
Eye down and out
Unable to move the
eye superiorly,
inferiorly, medially
Pupil fixed and
dilated
Vertical diplopia
(diplopia on
descending the
stairs)
Unable to look down
and in
Convergence of eyes
in primary position
Lateral diplopia
towards side of
lesion
Eye deviates
medially

Next question

Comment on this question

All contents of this site are 2012 E-Medical Revision Ltd

http://www.emrcs.com/question/question.php?q=0

Terms and Conditions

Privacy policy

1/1

26/05/2015

Reference ranges

Previous

Question 16 of 146

Next

A 77 year old man with symptoms of intermittent claudication is due to have his ankle
brachial pressure indices measured. The dorsalis pedis artery is impalpable. Which of the
following tendinous structures lies medial to it, that may facilitate its identification?

Question stats

Score: 55%
1

12.1%

7.6%

62.1%

7.9%

10.3%

A. Extensor digitorum longus tendon


62.1% of users answered this
question correctly

B. Peroneus tertius tendon

End and review

6
7

C. Extensor hallucis longus tendon


8

D. Extensor digitorum brevis tendon


9

Search

E. Flexor digitorum longus tendon

Go

Next question

10
11
12

The extensor hallucis longus tendon lies medial to the dorsalis pedis artery.

13
14

Foot- anatomy

15

Arches of the foot


The foot is conventionally considered to have two arches.

16

The longitudinal arch is higher on the medial than on the lateral side. The posterior
part of the calcaneum forms a posterior pillar to support the arch. The lateral part of
this structure passes via the cuboid bone and the lateral two metatarsal bones. The
medial part of this structure is more important. The head of the talus marks the summit
of this arch, located between the sustentaculum tali and the navicular bone. The
anterior pillar of the medial arch is composed of the navicular bone, the three
cuneiforms and the medial three metatarsal bones.
The transverse arch is situated on the anterior part of the tarsus and the posterior
part of the metatarsus. The cuneiforms and metatarsal bases narrow inferiorly, which
contributes to the shape of the arch.

Intertarsal joints
Sub talar joint

Formed by the cylindrical facet on the lower surface of the body of


the talus and the posterior facet on the upper surface of the
calcaneus. The facet on the talus is concave anteroposteriorly, the
other is convex. The synovial cavity of this joint does not
communicate with any other joint.

Talocalcaneonavicular
joint

The anterior part of the socket is formed by the concave articular


surface of the navicular bone, posteriorly by the upper surface of the
sustentaculum tali. The talus sits within this socket

Calcaneocuboid joint

Highest point in the lateral part of the longitudinal arch. The lower
aspect of this joint is reinforced by the long plantar and plantar
calcaneocuboid ligaments.

Transverse tarsal joint

The talocalcaneonavicular joint and the calcaneocuboid joint extend


across the tarsus in an irregular transverse plane, between the talus
and calcaneus behind and the navicular and cuboid bones in front.
This plane is termed the transverse tarsal joint.

Cuneonavicular joint

Formed between the convex anterior surface of the navicular bone


and the concave surface of the the posterior ends of the three
cuneiforms.

Intercuneiform joints

Between the three cuneiform bones.

Cuneocuboid joint

Between the circular facets on the lateral cuneiform bone and the
cuboid. This joint contributes to the tarsal part of the transverse arch.

A detailed knowledge of the joints is not required for MRCS Part A. However, the contribution
they play to the overall structure of the foot should be appreciated
Ligaments of the ankle joint and foot

http://www.emrcs.com/question/question.php?q=0

1/3

26/05/2015

Image sourced from Wikipedia

Muscles of the foot


Muscle

Origin

Insertion

Nerve
supply

Action

Abductor
hallucis

Medial side of the calcaneus,


flexor retinaculum, plantar
aponeurosis

Medial side of
the base of
the proximal
phalanx

Medial
plantar
nerve

Abducts the great


toe

Flexor
digitorum
brevis

Medial process of the


calcaneus, plantar
eponeurosis.

Via 4 tendons
into the
middle
phalanges of
the lateral 4
toes.

Medial
plantar
nerve

Flexes all the joints


of the lateral 4 toes
except for the
interphalangeal joint.

Abductor
digit
minimi

From the tubercle of the


calcaneus and from the
plantar aponeurosis

Together with
flexor digit
minimi brevis
into the lateral
side of the
base of the
proximal
phalanx of the
little toe

Lateral
plantar
nerve

Abducts the little toe


at the
metatarsophalangeal
joint

Flexor
hallucis
brevis

From the medial side of the


plantar surface of the cuboid
bone, from the adjacent part
of the lateral cuneiform bone
and from the tendon of tibialis
posterior.

Into the
proximal
phalanx of the
great toe, the
tendon
contains a
sesamoid
bone

Medial
plantar
nerve

Flexes the
metatarsophalangeal
joint of the great toe.

Adductor
hallucis

Arises from two heads. The


oblique head arises from the
sheath of the peroneus longus
tendon, and from the plantar
surfaces of the bases of the
2nd, 3rd and 4th metatarsal
bones. The transverse head
arises from the plantar
surface of the lateral 4
metatarsophalangeal joints
and from the deep transverse
metatarsal ligament.

Lateral side of
the base of
the proximal
phalanx of the
great toe.

Lateral
plantar
nerve

Adducts the great


toe towards the
second toe. Helps
maintain the
transverse arch of
the foot.

Extensor
digitorum
brevis

On the dorsal surface of the


foot from the upper surface of
the calcaneus and its
associated fascia

Via four thin


tendons which
run forward
and medially
to be inserted
into the
medial four
toes. The
lateral three
tendons join
with hoods of
extensor
digitorum
longus.

Deep
peroneal

Extend the
metatarsophalangeal
joint of the medial
four toes. It is
unable to extend the
interphalangeal joint
without the
assistance of the
lumbrical muscles.

Detailed knowledge of the foot muscles are not needed for the MRCS part A
Nerves in the foot

http://www.emrcs.com/question/question.php?q=0

2/3

26/05/2015

Lateral plantar nerve


Passes anterolaterally towards the base of the 5th metatarsal between flexor digitorum brevis
and flexor accessorius. On the medial aspect of the lateral plantar artery. At the base of the
5th metatarsal it splits into superficial and deep branches.
Medial plantar nerve
Passes forwards with the medial plantar artery under the cover of the flexor retinaculum to
the interval between abductor hallucis and flexor digitorum brevis on the sole of the foot.
Plantar arteries
Arise under the cover of the flexor retinaculum, midway between the tip of the medial
malleolus and the most prominent part of the medial side of the heel.

Medial plantar artery. Passes forwards medial to medial plantar nerve in the space
between abductor hallucis and flexor digitorum brevis.Ends by uniting with a branch of
the 1st plantar metatarsal artery.
Lateral plantar artery. Runs obliquely across the sole of the foot. It lies lateral to the
lateral plantar nerve. At the base of the 5th metatarsal bone it arches medially across
the foot on the metatarsals

Dorsalis pedis artery


This vessel is a direct continuation of the anterior tibial artery. It commences on the front of
the ankle joint and runs to the proximal end of the first metatarsal space. Here is gives off the
arcuate artery and continues forwards as the first dorsal metatarsal artery. It is accompanied
by two veins throughout its length. It is crossed by the extensor hallucis brevis
Rate question:

Next question

Comment on this question

All contents of this site are 2012 E-Medical Revision Ltd

http://www.emrcs.com/question/question.php?q=0

Terms and Conditions

Privacy policy

3/3

26/05/2015

Reference ranges

Question 17 of 146

Previous

Next

A 23 year old man falls over whilst intoxicated and a shard of glass transects his median
nerve at the proximal border of the flexor retinaculum. His tendons escape injury. Which of
the following features is least likely to be present?

Question stats

Score: 57.1%
1

12.2%

39%

13.9%

22.7%

12.1%

A. Weakness of thumb abduction


39% of users answered this
question correctly

B. Loss of sensation on the dorsal aspect of the thenar eminence

End and review

6
7

C. Loss of power of opponens pollicis


8

D. Adduction and lateral rotation of the thumb at rest


9

Search

E. Loss of power of abductor pollicis brevis

Go

Next question

10
11
12

The median nerve may be injured proximal to the flexor retinaculum. This will result in loss of
abductor pollicis brevis, flexor pollicis brevis, opponens pollicis and the first and second
lumbricals. When the patient is asked to close the hand slowly there is a lag of the index and
middle fingers reflecting the impaired lumbrical muscle function. The sensory changes are
minor and do not extend to the dorsal aspect of the thenar eminence.
Abductor pollicis longus will contribute to thumb abduction (and is innervated by the posterior
interosseous nerve) and therefore abduction will be weaker than prior to the injury.

13
14
15
16
17

Median nerve
The median nerve is formed by the union of a lateral and medial root respectively from the
lateral (C5,6,7) and medial (C8 and T1) cords of the brachial plexus; the medial root passes
anterior to the third part of the axillary artery. The nerve descends lateral to the brachial
artery, crosses to its medial side (usually passing anterior to the artery). It passes deep to
the bicipital aponeurosis and the median cubital vein at the elbow.
It passes between the two heads of the pronator teres muscle, and runs on the deep surface
of flexor digitorum superficialis (within its fascial sheath).
Near the wrist it becomes superficial between the tendons of flexor digitorum superficialis and
flexor carpi radialis, deep to palmaris longus tendon. It passes deep to the flexor retinaculum
to enter the palm, but lies anterior to the long flexor tendons within the carpal tunnel.
Branches
Region

Branch

Upper
arm

No branches, although the nerve commonly communicates with the


musculocutaneous nerve

Forearm

Pronator teres
Flexor carpi radialis
Palmaris longus
Flexor digitorum superficialis
Flexor pollicis longus
Flexor digitorum profundus (only the radial half)

Distal
forearm

Palmar cutaneous branch

Hand
(Motor)

Motor supply (LOAF)

Hand
(Sensory)

Lateral 2 lumbricals
Opponens pollicis
Abductor pollicis brevis
Flexor pollicis brevis

Over thumb and lateral 2 fingers


On the palmar aspect this projects proximally, on the dorsal aspect only the
distal regions are innervated with the radial nerve providing the more proximal
cutaneous innervation.

Patterns of damage
Damage at wrist
e.g. carpal tunnel syndrome
paralysis and wasting of thenar eminence muscles and opponens pollicis (ape hand
deformity)
sensory loss to palmar aspect of lateral (radial) 2 fingers

http://www.emrcs.com/question/question.php?q=0

1/2

26/05/2015

Damage at elbow, as above plus:


unable to pronate forearm
weak wrist flexion
ulnar deviation of wrist

Anterior interosseous nerve (branch of median nerve)


leaves just below the elbow
results in loss of pronation of forearm and weakness of long flexors of thumb and index
finger

Topography of the median nerve

Image sourced from Wikipedia

Rate question:

Next question

Comment on this question

All contents of this site are 2012 E-Medical Revision Ltd

http://www.emrcs.com/question/question.php?q=0

Terms and Conditions

Privacy policy

2/2

26/05/2015

Reference ranges

Previous

Question 18 of 146

Next

Question stats

End and review

Score: 59.1%
1

7.7%

10.2%

65.5%

A. Transverse arytenoid

7.5%

B. Posterior crico-arytenoid

9%

C. Cricothyroid

65.5% of users answered this


question correctly

The following muscles are supplied by the recurrent laryngeal nerve except:

D. Oblique arytenoid
E. Thyroarytenoid

7
8
9

Search
Next question
Go

10
11

Innervates: all intrinsic larynx muscles (excluding cricothyroid)

12
13

The external branch of the superior laryngeal nerve innervates the cricothyroid muscle.

14
15

Recurrent laryngeal nerve


16
17

Branch of the vagus nerve

18

Path
Right
Arises anterior to the subclavian artery and ascends obliquely next to the trachea,
behind the common carotid artery
It is either anterior or posterior to the inferior thyroid artery

Left
Arises left to the arch of the aorta
Winds below the aorta
Ascends along the side of the trachea

Then both

Pass in a groove between the trachea and oesophagus


Enters the larynx behind the articulation between the thyroid cartilage and cricoid
Distributed to larynx muscles

Branches to

Cardiac plexus
Mucous membrane and muscular coat of the oesophagus and trachea

Innervates

Intrinsic larynx muscles (excluding cricothyroid)

http://www.emrcs.com/question/question.php?q=0

1/2

26/05/2015

Image sourced from Wikipedia

Rate question:

Next question

Comment on this question

All contents of this site are 2012 E-Medical Revision Ltd

http://www.emrcs.com/question/question.php?q=0

Terms and Conditions

Privacy policy

2/2

26/05/2015

Reference ranges

Previous

Question 19 of 146

Next

Question stats

End and review

Score: 60.9%
1

18.1%

11.3%

9.4%

A. Uranchus

53.1%

B. Cloaca

8%

C. Vitello-intestinal duct

53.1% of users answered this


question correctly

From which embryological structure is the ureter derived?

D. Mesonephric duct
E. None of the above

7
8
9

Search
Next question
Go

10
11

The ureter develops from the mesonephric duct.

12

Ureter

13
14
15

25-35 cm long
Muscular tube lined by transitional epithelium
Surrounded by thick muscular coat. Becomes 3 muscular layers as it crosses the bony
pelvis
Retroperitoneal structure overlying transverse processes L2-L5
Lies anterior to bifurcation of iliac vessels
Blood supply is segmental; renal artery, aortic branches, gonadal branches, common
iliac and internal iliac
Lies beneath the uterine artery

Rate question:

16
17
18
19

Next question

Comment on this question

All contents of this site are 2012 E-Medical Revision Ltd

http://www.emrcs.com/question/question.php?q=0

Terms and Conditions

Privacy policy

1/1

26/05/2015

Reference ranges

Previous

Question 20 of 146

Next

A 16 year old boy is hit by a car and sustains a blow to the right side of his head. He is
initially conscious but on arrival in the emergency department is comatose. On examination
his right pupil is fixed and dilated. The neurosurgeons plan immediate surgery. What type of
initial approach should be made?

Question stats

Score: 62.5%
1

16.5%

56%

9%

12.2%

6.3%

56% of users answered this


question correctly

A. Left parieto-temporal craniotomy

End and review

B. Right parieto-temporal craniotomy

6
7
8

C. Posterior fossa craniotomy

Search

D. Left parieto-temporal burr holes


E. None of the above

Go

10
11

Next question

12
13

Theme from April 2012 Exam


A unilateral dilated pupil is a classic sign of transtentorial herniation. The medial aspect of
the temporal lobe (uncus) herniates across the tentorium and causes pressure on the
ipsilateral oculomotor nerve, interrupting parasympathetic input to the eye and resulting in a
dilated pupil. In addition the brainstem is compressed. As the ipsilateral oculomotor nerve is
being compressed, craniotomy (rather than Burr Holes) should be made on the ipsilateral
side.

14
15
16
17
18
19

Head injury

20

Patients who suffer head injuries should be managed according to ATLS principles and extra
cranial injuries should be managed alongside cranial trauma. Inadequate cardiac output will
compromise CNS perfusion irrespective of the nature of the cranial injury.
Types of traumatic brain injury
Extradural
haematoma

Bleeding into the space betw een the dura mater and the skull. Often
results from acceleration-deceleration trauma or a blow to the side of the
head. The majority of extradural haematomas occur in the temporal region
w here skull fractures cause a rupture of the middle meningeal artery.
Features
Raised intracranial pressure
Some patients may exhibit a lucid interval

Subdural
haematoma

Bleeding into the outermost meningeal layer. Most commonly occur around
the frontal and parietal lobes. May be either acute or chronic.
Risk factors include old age and alcoholism.
Slow er onset of symptoms than a extradural haematoma.

Subarachnoid Usually occurs spontaneously in the context of a ruptured cerebral


haemorrhage aneurysm, but may be seen in association w ith other injuries w hen a
patient has sustained a traumatic brain injury.

Pathophysiology
Primary brain injury may be focal (contusion/ haematoma) or diffuse (diffuse axonal
injury)
Diffuse axonal injury occurs as a result of mechanical shearing following deceleration,
causing disruption and tearing of axons
Intra-cranial haematomas can be extradural, subdural or intracerebral, while
contusions may occur adjacent to (coup) or contralateral (contre-coup) to the side of
impact
Secondary brain injury occurs when cerebral oedema, ischaemia, infection, tonsillar or
tentorial herniation exacerbates the original injury. The normal cerebral auto
regulatory processes are disrupted following trauma rendering the brain more
susceptible to blood flow changes and hypoxia
The Cushings reflex (hypertension and bradycardia) often occurs late and is usually a
pre terminal event

Management
Where there is life threatening rising ICP such as in extra dural haematoma and whilst
theatre is prepared or transfer arranged use of IV mannitol/ frusemide may be
required.
Diffuse cerebral oedema may require decompressive craniotomy
Exploratory Burr Holes have little management in modern practice except where
scanning may be unavailable and to thus facilitate creation of formal craniotomy flap

http://www.emrcs.com/question/question.php?q=0

1/2

26/05/2015

Depressed skull fractures that are open require formal surgical reduction and
debridement, closed injuries may be managed non operatively if there is minimal
displacement.
ICP monitoring is appropriate in those who have GCS 3-8 and normal CT scan.
ICP monitoring is mandatory in those who have GCS 3-8 and abnormal CT scan.
Hyponatraemia is most likely to be due to syndrome of inappropriate ADH secretion.
Minimum of cerebral perfusion pressure of 70mmHg in adults.
Minimum cerebral perfusion pressure of between 40 and 70 mmHg in children.

Interpretation of pupillary findings in head injuries


Pupil size

Light response

Interpretation

Unilaterally
dilated

Sluggish or fixed

3rd nerve compression secondary to


tentorial herniation

Bilaterally dilated

Sluggish or fixed

Unilaterally
dilated or equal

Cross reactive
(Marcus - Gunn)

Bilaterally
constricted

May be difficult to
assess

Unilaterally
constricted

Preserved

Rate question:

Poor CNS perfusion


Bilateral 3rd nerve palsy
Optic nerve injury

Opiates
Pontine lesions
Metabolic encephalopathy
Sympathetic pathway disruption

Next question

Comment on this question

All contents of this site are 2012 E-Medical Revision Ltd

http://www.emrcs.com/question/question.php?q=0

Terms and Conditions

Privacy policy

2/2

26/05/2015

Reference ranges

Question 21 of 146

Previous

Next

Question stats

End and review

Score: 64%
1

11.9%

12.5%

50.1%

A. The brachial plexus passes anterior to the middle scalene muscle

16.3%

B. The phrenic nerve lies anterior to the anterior scalene muscle

9.1%

C. The subclavian artery passes posterior to the middle scalene

50.1% of users answered this


question correctly

Which of the following relationship descriptions regarding the scalene muscles is incorrect?

D. The subclavian vein lies anterior to the anterior scalene muscle at the
level of the first rib

6
7
8

E. The anterior scalene inserts into the first rib

Search
Go

Next question

10
11
12

The subclavian artery passes anterior to the middle scalene.

13

Scalene muscles

14
15

The 3 paired muscles are:

16

Scalenus anterior: Elevate 1st rib and laterally flex the neck to same side
Scalenus medius: Same action as scalenus anterior
Scalenus posterior: Elevate 2nd rib and tilt neck to opposite side

17
18
19

Innervation Spinal nerves C4-6

20

Origin

Transverse processes C2 to C7

21

Insertion

First and second ribs

Important
relations

The brachial plexus and subclavian artery pass betw een the anterior
and middle scalenes through a space called the scalene
hiatus/fissure.
The subclavian vein and phrenic nerve pass anteriorly to the anterior
scalene as it crosses over the first rib.

Image sourced from Wikipedia

Thoracic outlet syndrome


The scalenes are at risk of adhering to the fascia surrounding the brachial plexus or
shortening causing compression of the brachial plexus when it passes between the clavicle
and 1st rib causing thoracic outlet syndrome.
Rate question:

Next question

Comment on this question

All contents of this site are 2012 E-Medical Revision Ltd

http://www.emrcs.com/question/question.php?q=0

Terms and Conditions

Privacy policy

1/1

26/05/2015

Reference ranges

Question 22 of 146

Previous

Next

A 56 year old man is having a long venous line inserted via the femoral vein into the right
atrium for CVP measurements. The catheter is advanced through the IVC. At which of the
following levels does this vessel enter the thorax?

Question stats

Score: 65.4%
1

6.3%

13%

7.4%

66.1%

7.2%

A. L2
66.1% of users answered this
question correctly

B. T10

End and review

6
7

C. L1
8

D. T8
9

Search

E. T6

Go

Next question

10
11
12

Theme from 2010 Exam


Theme from September 2012 Exam
The IVC passes through the diaphragm at T8.

13
14
15

Inferior vena cava

16
17

Origin

18

L5

19
20

Path

21

Left and right common iliac veins merge to form the IVC.
Passes right of midline
Paired segmental lumbar veins drain into the IVC throughout its length
The right gonadal vein empties directly into the cava and the left gonadal vein
generally empties into the left renal vein.
The next major veins are the renal veins and the hepatic veins
Pierces the central tendon of diaphragm at T8
Right atrium

22

Image sourced from Wikipedia

Relations
Anteriorly

Small bowel, first and third part of duodenum, head of pancreas, liver and bile
duct, right common iliac artery, right gonadal artery

Posteriorly

Right renal artery, right psoas, right sympathetic chain, coeliac ganglion

Levels
Level

Vein

T8

Hepatic vein, inferior phrenic vein, pierces diaphragm

L1

Suprarenal veins, renal vein

L2

Gonadal vein

L1-5

Lumbar veins

L5

Common iliac vein, formation of IVC

http://www.emrcs.com/question/question.php?q=0

1/2

26/05/2015

Rate question:

Next question

Comment on this question

All contents of this site are 2012 E-Medical Revision Ltd

http://www.emrcs.com/question/question.php?q=0

Terms and Conditions

Privacy policy

2/2

26/05/2015

Reference ranges

Question 23 of 146

Previous

Next

A 23 year old man falls and injures his hand. There are concerns that he may have a
scaphoid fracture as there is tenderness in his anatomical snuffbox on clinical examination.
Which of the following forms the posterior border of this structure?

Question stats

Score: 63%
1

7.2%

15.5%

17.5%

18.6%

41.2%

A. Basilic vein
41.2% of users answered this
question correctly

B. Radial artery

End and review

6
7

C. Extensor pollicis brevis


8

D. Abductor pollicis longus


9

Search

E. Extensor pollicis longus

Go

10
11

Next question

12

Theme from 2009 Exam


Theme from September 2012 Exam
Its boundaries are extensor pollicis longus, medially (posterior border) and laterally (anterior
border) by the tendons of abductor pollicis longus and extensor pollicis brevis.

13
14
15
16

Anatomical snuffbox

17
18

Posterior border

Tendon of extensor pollicis longus

Anterior border

Tendons of extensor pollicis brevis and abductor pollicis longus

Proximal border

Styloid process of the radius

Distal border

Apex of snuffbox triangle

Floor

Trapezium and scaphoid

Content

Radial artery

19
20
21
22
23

Image showing the anatomical snuffbox

Image sourced from Wikipedia

Rate question:

Next question

Comment on this question

All contents of this site are 2012 E-Medical Revision Ltd

http://www.emrcs.com/question/question.php?q=0

Terms and Conditions

Privacy policy

1/1

26/05/2015

Reference ranges

Previous

Question 24 of 146

Next

Question stats

End and review

Score: 64.3%
1

40.6%

13.8%

11.1%

A. Sharpeys fibres

25.7%

B. Peripheral lamellae

8.9%

C. Elastic fibres

40.6% of users answered this


question correctly

Which of the following structures attaches periosteum to bone?

D. Fibrolamellar bundles
E. Purkinje fibres

6
7
8
9

Search
Next question
Go

10
11

Periosteum is attached to bone by strong collagenous fibers called Sharpey's fibres, which
extend to the outer circumferential and interstitial lamellae. It also provides an attachment for
muscles and tendons.

12
13
14

Periosteum

15
16

Periosteum is a membrane that covers the outer surface of all bones, except at the joints of
long bones. Endosteum lines the inner surface of all bones.

17
18

Periosteum consists of dense irregular connective tissue. Periosteum is divided into an outer
"fibrous layer" and inner "cambium layer" (or "osteogenic layer"). The fibrous layer contains
fibroblasts, while the cambium layer contains progenitor cells that develop into osteoblasts.
These osteoblasts are responsible for increasing the width of a long bone and the overall
size of the other bone types. After a bone fracture the progenitor cells develop into
osteoblasts and chondroblasts, which are essential to the healing process.

19
20
21
22
23

As opposed to osseous tissue, periosteum has nociceptive nerve endings, making it very
sensitive to manipulation. It also provides nourishment by providing the blood supply.
Periosteum is attached to bone by strong collagenous fibers called Sharpey's fibres, which
extend to the outer circumferential and interstitial lamellae. It also provides an attachment for
muscles and tendons.

24

Periosteum that covers the outer surface of the bones of the skull is known as "pericranium"
except when in reference to the layers of the scalp.
Rate question:

Next question

Comment on this question

All contents of this site are 2012 E-Medical Revision Ltd

http://www.emrcs.com/question/question.php?q=0

Terms and Conditions

Privacy policy

1/1

26/05/2015

Reference ranges

Previous

Question 25 of 146

Next

A 62 year old man is undergoing a left hemicolectomy for carcinoma of the descending
colon. The registrar commences mobilisation of the left colon by pulling downwards and
medially. Blood soon appears in the left paracolic gutter. The most likely source of bleeding
is the:

Question stats

Score: 62.1%
1

22.6%

16.9%

33.9%

17.3%

9.4%

33.9% of users answered this


question correctly

A. Marginal artery

End and review

B. Left testicular artery

6
7
8

C. Spleen

Search

D. Left renal vein


E. None of the above

Go

10
11

Next question

12
13

The spleen is commonly torn by traction injuries in colonic surgery. The other structures are
associated with bleeding during colonic surgery but would not manifest themselves as blood
in the paracolic gutter prior to incision of the paracolonic peritoneal edge.

14
15
16

Left colon

17
18

Position

19

As the left colon passes inferiorly its posterior aspect becomes extraperitoneal, and
the ureter and gonadal vessels are close posterior relations that may become involved
in disease processes
At a level of L3-4 (variable) the left colon becomes the sigmoid colon and wholly
intraperitoneal once again
The sigmoid colon is a highly mobile structure and may even lie on the right side of the
abdomen
It passes towards the midline, the taenia blend and this marks the transition between
sigmoid colon and upper rectum

20
21
22
23
24
25

Blood supply
Inferior mesenteric artery
However, the marginal artery (from the right colon) contributes, this contribution
becomes clinically significant when the IMA is divided surgically (e.g. During AAA
repair)

Rate question:

Next question

Comment on this question

All contents of this site are 2012 E-Medical Revision Ltd

http://www.emrcs.com/question/question.php?q=0

Terms and Conditions

Privacy policy

1/1

26/05/2015

Reference ranges

Previous

Question 26 of 146

Next

A man is undergoing excision of a sub mandibular gland. As the gland is mobilised, a vessel
is injured lying between the gland and the mandible. Which of the following is this vessel
most likely to be?

Question stats

Score: 60%
1

42.7%

5.9%

7.4%

35.5%

8.5%

A. Lingual artery
35.5% of users answered this
question correctly

B. Occipital artery

End and review

6
7

C. Superior thyroid artery


8

D. Facial artery
9

Search

E. External jugular vein

Go

10
11

Next question

12
13

The high salivary viscosity of submandibular gland secretions favors stone formation.
Most stones are radio-opaque.

14
15

The facial artery lies between the gland and mandible and is often ligated during excision of
the gland. The lingual artery may be encountered but this is usually later in the operative
process as Whartons duct is mobilised.

16

Submandibular gland

19

17
18

20

Relations of the submandibular gland


Superficial

Deep

21

Platysma, deep fascia and mandible


Submandibular lymph nodes
Facial vein (facial artery near mandible)
Marginal mandibular nerve
Cervical branch of the facial nerve

22

Facial artery (inferior to the mandible)


Mylohoid muscle
Sub mandibular duct
Hyoglossus muscle
Lingual nerve
Submandibular ganglion
Hypoglossal nerve

26

23
24
25

Submandibular duct (Wharton's duct)


Opens lateral to the lingual frenulum on the anterior floor of mouth.
5 cm length
Lingual nerve wraps around Wharton's duct. As the duct passes forwards it crosses
medial to the nerve to lie above it and then crosses back, lateral to it, to reach a
position below the nerve.

Innervation
Sympathetic innervation- Derived from superior cervical ganglion
Parasympathetic innervation- Submandibular ganglion via lingual nerve

Arterial supply
Branch of the facial artery. The facial artery passes through the gland to groove its deep
surface. It then emerges onto the face by passing between the gland and the mandible.
Venous drainage
Anterior facial vein (lies deep to the Marginal Mandibular nerve)
Lymphatic drainage
Deep cervical and jugular chains of nodes
Rate question:

Next question

Comment on this question

All contents of this site are 2012 E-Medical Revision Ltd

http://www.emrcs.com/question/question.php?q=0

Terms and Conditions

Privacy policy

1/1

26/05/2015

Reference ranges

Previous2 / 3

Question 27-29 of 146

Next

Question stats

End and review

Score: 60.6%

Average score for registered users:

1
2

Theme: Cranial nerves


A. Facial

27

58.3%

28

40.6%

29

72%

B. Trigeminal

3
4
5

C. Vagus

D. Hypoglossal

Search

E. Glossopharyngeal

7
Go

For each of the following functions please select the most likely responsible cranial nerve.
Each option may be used once, more than once or not at all.

8
9
10
11
12

27.

Supplies the motor fibres of styloglossus.

13
14

You answered Glossopharyngeal

15

The correct answer is Hypoglossal


16

28.

The hypoglossal nerve supplies motor innervation to all extrinsic and intrinsic
muscles of the tongue.

17

Provides sensation to the anterior two thirds of the tongue.

19

18

20

Trigeminal

29.

21

Taste to the anterior two thirds of the tongue is supplied by the facial nerve,
the trigeminal supplies general sensation.

22

Supplies general sensation to the posterior third of the tongue.

24

23

25

Glossopharyngeal

26

The glossopharyngeal nerve supplies general sensation to the posterior third


of the tongue and contributes to the gag reflex.

27-29 2 / 3

Next question

Cranial nerves

Cranial nerve lesions


Olfactory nerve

May be injured in basal skull fractures or involved in frontal lobe tumour


extension. Loss of olfactory nerve function in relation to major CNS
pathology is seldom an isolated event and thus it is poor localiser of CNS
pathology.

Optic nerve

Problems with visual acuity may result from intra ocular disorders.
Problems with the blood supply such as amaurosis fugax may produce
temporary visual distortion. More important surgically is the pupillary
response to light. The pupillary size may be altered in a number of
disorders. Nerves involved in the resizing of the pupil connect to the
pretectal nucleus of the high midbrain, bypassing the lateral geniculate
nucleus and the primary visual cortex. From the pretectal nucleus
neurones pass to the Edinger - Westphal nucleus, motor axons from here
pass along with the oculomotor nerve. They synapse with ciliary ganglion
neurones; the parasympathetic axons from this then innervate the iris and
produce miosis. The miotic pupil is seen in disorders such as Horner's
syndrome or opiate overdose.
Mydriasis is the dilatation of the pupil in response to disease, trauma,
drugs (or the dark!). It is pathological when light fails to induce miosis. The
radial muscle is innervated by the sympathetic nervous system. Because
the parasympathetic fibres travel with the oculomotor nerve they will be
damaged by lesions affecting this nerve (e.g. cranial trauma).
The response to light shone in one eye is usually a constriction of both
pupils. This indicates intact direct and consensual light reflexes. When
the optic nerve has an afferent defect the light shining on the affected eye
will produce a diminished pupillary response in both eyes. Whereas light
shone on the unaffected eye will produce a normal pupillary response in
both eyes. This is referred to as the Marcus Gunn pupil and is seen in
conditions such as optic neuritis. In a total CN II lesion shining the light in
the affected eye will produce no response.

http://www.emrcs.com/question/question.php?q=0

1/2

26/05/2015

Oculomotor nerve

The pupillary effects are described above. In addition it supplies all ocular
muscles apart from lateral rectus and superior oblique. Thus the affected
eye will be deviated inferolaterally. Levator palpebrae superioris may also
be impaired resulting in impaired ability to open the eye.

Trochlear nerve

The eye will not be able to look down.

Trigeminal nerve

Largest cranial nerve. Exits the brainstem at the pons. Branches are
ophthalmic, maxillary and mandibular. Only the mandibular branch has
both sensory and motor fibres. Branches converge to form the trigeminal
ganglion (located in Meckels cave). It supplies the muscles of mastication
and also tensor veli palatine, mylohyoid, anterior belly of digastric and
tensor tympani. The detailed descriptions of the various sensory functions
are described in other areas of the website. The corneal reflex is important
and is elicited by applying a small tip of cotton wool to the cornea, a reflex
blink should occur if it is intact. It is mediated by: the naso ciliary branch
of the ophthalmic branch of the trigeminal (sensory component) and the
facial nerve producing the motor response. Lesions of the afferent arc will
produce bilateral absent blink and lesions of the efferent arc will result in a
unilateral absent blink.

Abducens nerve

The affected eye will have a deficit of abduction. This cranial nerve exits
the brainstem between the pons and medulla. It thus has a relatively long
intra cranial course which renders it susceptible to damage in raised intra
cranial pressure.

Facial nerve

Emerges from brainstem between pons and medulla. It controls muscles


of facial expression and taste from the anterior 2/3 of the tongue. The
nerve passes into the petrous temporal bone and into the internal auditory
meatus. It then passes through the facial canal and exits at the
stylomastoid foramen. It passes through the parotid gland and divides at
this point. It does not innervate the parotid gland. Its divisions are
considered in other parts of the website. Its motor fibres innervate
orbicularis oculi to produce the efferent arm of the corneal reflex. In
surgical practice it may be injured during parotid gland surgery or invaded
by malignancies of the gland and a lower motor neurone on the ipsilateral
side will result.

Vestibulocochlear nerve

Exits from the pons and then passes through the internal auditory
meatus. It is implicated in sensorineural hearing loss. Individuals with
sensorineural hearing loss will localise the sound in webers test to the
normal ear. Rinnes test will be reduced on the affected side but should
still work. These two tests will distinguish sensorineural hearing loss from
conductive deafness. In the latter condition webers test will localise to the
affected ear and Rinnes test will be impaired on the affected side. Surgical
lesions affecting this nerve include CNS tumours and basal skull fractures.
It may also be damaged by the administration of ototoxic drugs (of which
gentamicin is the most commonly used in surgical practice).

Glossopharyngeal
nerve

Exits the pons just above the vagus. Receives sensory fibres from
posterior 1/3 tongue, tonsils, pharynx and middle ear (otalgia may occur
following tonsillectomy). It receives visceral afferents from the carotid
bodies. It supplies parasympathetic fibres to the parotid gland via the otic
ganglion and motor function to stylopharyngeaus muscle. The sensory
function of the nerve is tested using the gag reflex.

Vagus nerve

Leaves the medulla between the olivary nucleus and the inferior cerebellar
peduncle. Passes through the jugular foramen and into the carotid sheath.
Details of the functions of the vagus nerve are covered in the website
under relevant organ sub headings.

Accessory nerve

Exists from the caudal aspect of the brainstem (multiple branches)


supplies trapezius and sternocleidomastoid muscles. The distal portion of
this nerve is most prone to injury during surgical procedures.

Hypoglossal
nerve

Emerges from the medulla at the preolivary sulcus, passes through the
hypoglossal canal. It lies on the carotid sheath and passes deep to the
posterior belly of digastric to supply muscles of the tongue (except
palatoglossus). Its location near the carotid sheath makes it vulnerable
during carotid endarterectomy surgery and damage will produce ipsilateral
defect in muscle function.

Rate question:

Next question

Comment on this question

All contents of this site are 2012 E-Medical Revision Ltd

http://www.emrcs.com/question/question.php?q=0

Terms and Conditions

Privacy policy

2/2

26/05/2015

Reference ranges

Previous

Question 30 of 146

Next

Question stats

End and review

Score: 61.8%
1

7.7%

10.1%

7.7%

A. Sartorius

64.3%

B. Quadratus femoris

10.2%

C. Semimembranosus

64.3% of users answered this


question correctly

The integrity of which muscle is assessed by the Trendelenburg test?

D. Gluteus medius
E. Piriformis

6
7
8
9

Search
Next question
Go

10
11

Theme from 2011 Exam

12

Trendelenburg test

13
14

Injury or division of the superior gluteal nerve results in a motor deficit that consists of
weakened abduction of the thigh by gluteus medius, a disabling gluteus medius limp and a
compensatory list of the body to the weakened gluteal side. The compensation results in a
gravitational shift so that the body is supported on the unaffected limb.

15
16
17
18

When a person is asked to stand on one leg, the gluteus medius usually contracts as soon
as the contralateral leg leaves the floor, preventing the pelvis from dipping towards the
unsupported side. When a person with paralysis of the superior gluteal nerve is asked to
stand on one leg, the pelvis on the unsupported side descends, indicating that the gluteus
medius on the affected side is weak or non functional ( a positive Trendelenburg test).

19
20
21
22

This eponymous test also refers to a vascular investigation in which tourniquets are placed
around the upper thigh, these can help determine whether saphenofemoral incompetence is
present.

23
24
25

Rate question:

26

Next question

27-29 2 / 3
Comment on this question

30

All contents of this site are 2012 E-Medical Revision Ltd

http://www.emrcs.com/question/question.php?q=0

Terms and Conditions

Privacy policy

1/1

26/05/2015

Reference ranges

Previous

Question 31 of 146

Next

A 52 year old female renal patient needs a femoral catheter to allow for haemodialysis.
Which of the structures listed below is least likely to be encountered during its insertion?

A. Great saphenous vein


B. Deep circumflex iliac artery

Question stats

Score: 60%
1

14.8%

43.6%

14.9%

8.6%

18.2%

43.6% of users answered this


question correctly

C. Superficial circumflex iliac artery

End and review

D. Femoral vein

6
7
8

E. Femoral branch of the genitofemoral nerve

Search
Next question

Go

10
11

Femoral access catheters are typically inserted in the region of the femoral triangle.
Therefore the physician may encounter the femoral, vein, nerve, branches of the femoral
artery and tributaries of the femoral vein. The deep circumflex iliac artery arises above the
inguinal ligament and is therefore less likely to be encountered than the superficial circumflex
iliac artery which arises below the inguinal ligament.

12
13
14
15
16

Femoral triangle anatomy

17
18

Boundaries

19

Superiorly

Inguinal ligament

Laterally

Sartorius

21

Medially

Adductor longus

22

Floor

Iliopsoas, adductor longus and pectineus

20

23

Roof

Fascia lata and Superficial fascia


Superficial inguinal lymph nodes (palpable below the inguinal ligament)
Great saphenous vein

24
25
26
27-29 2 / 3
30
31

Image sourced from Wikipedia

Contents
Femoral vein (medial to lateral)
Femoral artery-pulse palpated at the mid inguinal point
Femoral nerve
Deep and superficial inguinal lymph nodes
Lateral cutaneous nerve
Great saphenous vein
Femoral branch of the genitofemoral nerve

http://www.emrcs.com/question/question.php?q=0

1/2

26/05/2015

Rate question:

Next question

Comment on this question

All contents of this site are 2012 E-Medical Revision Ltd

http://www.emrcs.com/question/question.php?q=0

Terms and Conditions

Privacy policy

2/2

26/05/2015

Reference ranges

Question 32 of 146

Previous

Next

A 53 year old man with a chronically infected right kidney is due to undergo a nephrectomy.
Which of the following structures would be encountered first during a posterior approach to
the hilum of the right kidney?

Question stats

Score: 61.1%
1

17.5%

42.3%

17%

14.8%

8.6%

A. Right renal artery


42.3% of users answered this
question correctly

B. Ureter

End and review

6
7

C. Right renal vein


8

D. Inferior vena cava


9

Search

E. Right testicular vein

Go

10
11

Next question

12

The ureter is the most posterior structure at the hilum of the right kidney and would therefore
be encountered first during a posterior approach.

13

Renal arteries

15

14

16
17

The right renal artery is longer than the left renal artery
The renal vein/artery/pelvis enter the kidney at the hilum

18
19
20

Relations
Right

Left

Anterior- IVC, right renal vein, the head of the pancreas, and the descending part of the
duodenum

21

Anterior- left renal vein, the tail of the pancreas

23

22

24
25

Branches

26

The renal arteries are direct branches off the aorta (upper border of L2)
In 30% there may be accessory arteries (mainly left side). Instead of entering the
kidney at the hilum, they usually pierce the upper or lower part of the organ.
Before reaching the hilum of the kidney, each artery divides into four or five segmental
branches (renal vein anterior and ureter posterior); which then divide within the sinus
into lobar arteries supplying each pyramid and cortex.
Each vessel gives off some small inferior suprarenal branches to the suprarenal gland,
the ureter, and the surrounding cellular tissue and muscles.

Rate question:

27-29 2 / 3
30
31
32

Next question

Comment on this question

All contents of this site are 2012 E-Medical Revision Ltd

http://www.emrcs.com/question/question.php?q=0

Terms and Conditions

Privacy policy

1/1

26/05/2015

Reference ranges

Question 33 of 146

Previous

Next

A 28 year old man is stabbed outside a nightclub in the upper arm. The median nerve is
transected. Which of the following muscles will demonstrate impaired function as a result?

A. Palmaris brevis
B. Second and third interossei

Question stats

Score: 62.2%
1

12.5%

11%

16%

23.9%

36.5%

36.5% of users answered this


question correctly

C. Adductor pollicis

End and review

D. Abductor pollicis longus

6
7
8

E. Abductor pollicis brevis

Search
Next question

Go

10
11
12

Palmaris brevis - Ulnar nerve


Palmar interossei- Ulnar nerve
Adductor pollicis - Ulnar nerve
Abductor pollicis longus - Posterior interosseous nerve
Abductor pollicis brevis - Median nerve

13
14
15
16
17

The median nerve innervates all the short muscles of the thumb except the adductor and the
deep head of the short flexor. Palmaris and the interossei are innervated by the ulnar nerve.

18
19
20

Median nerve

21

The median nerve is formed by the union of a lateral and medial root respectively from the
lateral (C5,6,7) and medial (C8 and T1) cords of the brachial plexus; the medial root passes
anterior to the third part of the axillary artery. The nerve descends lateral to the brachial
artery, crosses to its medial side (usually passing anterior to the artery). It passes deep to
the bicipital aponeurosis and the median cubital vein at the elbow.
It passes between the two heads of the pronator teres muscle, and runs on the deep surface
of flexor digitorum superficialis (within its fascial sheath).
Near the wrist it becomes superficial between the tendons of flexor digitorum superficialis and
flexor carpi radialis, deep to palmaris longus tendon. It passes deep to the flexor retinaculum
to enter the palm, but lies anterior to the long flexor tendons within the carpal tunnel.

23
24
25
26
27-29 2 / 3
30
31
32

Branches
Region

Branch

Upper
arm

No branches, although the nerve commonly communicates with the


musculocutaneous nerve

Forearm

Pronator teres
Flexor carpi radialis
Palmaris longus
Flexor digitorum superficialis
Flexor pollicis longus
Flexor digitorum profundus (only the radial half)

Distal
forearm

Palmar cutaneous branch

Hand
(Motor)

Motor supply (LOAF)

Hand
(Sensory)

22

33

Lateral 2 lumbricals
Opponens pollicis
Abductor pollicis brevis
Flexor pollicis brevis

Over thumb and lateral 2 fingers


On the palmar aspect this projects proximally, on the dorsal aspect only the
distal regions are innervated with the radial nerve providing the more proximal
cutaneous innervation.

Patterns of damage
Damage at wrist
e.g. carpal tunnel syndrome
paralysis and wasting of thenar eminence muscles and opponens pollicis (ape hand
deformity)
sensory loss to palmar aspect of lateral (radial) 2 fingers

http://www.emrcs.com/question/question.php?q=0

1/2

26/05/2015

Damage at elbow, as above plus:


unable to pronate forearm
weak wrist flexion
ulnar deviation of wrist

Anterior interosseous nerve (branch of median nerve)


leaves just below the elbow
results in loss of pronation of forearm and weakness of long flexors of thumb and index
finger

Topography of the median nerve

Image sourced from Wikipedia

Rate question:

Next question

Comment on this question

All contents of this site are 2012 E-Medical Revision Ltd

http://www.emrcs.com/question/question.php?q=0

Terms and Conditions

Privacy policy

2/2

26/05/2015

Reference ranges

Previous

Question 34 of 146

Next

A 22 year old man sustains a blow to the side of his head with a baseball bat during a fight.
He is initially conscious. However, he subsequently loses consciousness and then dies. Post
mortem examination shows an extradural haematoma. The most likely culprit vessel is a
branch of which of the following?

Question stats

Score: 63.2%
1

29.2%

14.1%

8.9%

41%

6.8%

41% of users answered this


question correctly

A. Middle cerebral artery

End and review

B. Internal carotid artery

6
7
8

C. Anterior cerebral artery

Search

D. Maxillary artery
E. Mandibular artery

Go

10
11

Next question

12
13

The middle meningeal artery is the most likely source of the extradural haematoma in this
setting. It is a branch of the maxillary artery. The middle cerebral artery does not give rise to
the middle meningeal artery. Note that the question is asking for the vessel which gives rise
to the middle meningeal artery ("the likely culprit vessel is a branch of which of the following")

14
15
16
17

Middle meningeal artery

18
19

Middle meningeal artery is typically the third branch of the first part of the maxillary
artery, one of the two terminal branches of the external carotid artery. After branching
off the maxillary artery in the infratemporal fossa, it runs through the foramen
spinosum to supply the dura mater (the outermost meninges) .
The middle meningeal artery is the largest of the three (paired) arteries which supply
the meninges, the others being the anterior meningeal artery and the posterior
meningeal artery.
The middle meningeal artery runs beneath the pterion. It is vulnerable to injury at this
point, where the skull is thin. Rupture of the artery may give rise to an extra dural
hematoma.
In the dry cranium, the middle meningeal, which runs within the dura mater
surrounding the brain, makes a deep indention in the calvarium.
The middle meningeal artery is intimately associated with the auriculotemporal nerve
which wraps around the artery making the two easily identifiable in the dissection of
human cadavers and also easily damaged in surgery.

20
21
22
23
24
25
26
27-29 2 / 3
30
31
32
33
34

Rate question:

Next question

Comment on this question

All contents of this site are 2012 E-Medical Revision Ltd

http://www.emrcs.com/question/question.php?q=0

Terms and Conditions

Privacy policy

1/1

26/05/2015

Reference ranges

Question 35 of 146

Previous

Next

A 72 year old man with carcinoma of the lung is undergoing a left pneumonectomy. The left
main bronchus is divided. Which of the following thoracic vertebrae lies posterior to this
structure?

Question stats

Score: 64.1%
1

22.9%

12.7%

48.1%

8.7%

7.6%

A. T3
48.1% of users answered this
question correctly

B. T7

End and review

6
7

C. T6
8

D. T10
9

Search

E. T1

Go

Next question

10
11
12

The left main bronchus lies at T6. Topographical anatomy of the thorax is important as it
helps surgeons to predict the likely structures to be injured in trauma scenarios (so popular
with examiners)

13
14
15

Lung anatomy

16
17

The right lung is composed of 3 lobes divided by the oblique and transverse fissures. The
left lung has two lobes divided by the oblique fissure.The apex of both lungs is approximately
4cm superior to the sterno-costal joint of the first rib. Immediately below this is a sulcus
created by the subclavian artery.
Peripheral contact points of the lung

18
19
20
21
22

Base: diaphragm
Costal surface: corresponds to the cavity of the chest
Mediastinal surface: Contacts the mediastinal pleura. Has the cardiac impression.
Above and behind this concavity is a triangular depression named the hilum, where
the structures which form the root of the lung enter and leave the viscus. These
structures are invested by pleura, which, below the hilum and behind the pericardial
impression, forms the pulmonary ligament

23
24
25
26
27-29 2 / 3
30

Right lung
Above the hilum is the azygos vein; Superior to this is the groove for the superior vena cava
and right innominate vein; behind this, and nearer the apex, is a furrow for the innominate
artery. Behind the hilum and the attachment of the pulmonary ligament is a vertical groove
for the oesophagus; In front and to the right of the lower part of the oesophageal groove is a
deep concavity for the extrapericardiac portion of the inferior vena cava.

31
32
33
34
35

The root of the right lung lies behind the superior vena cava and the right atrium, and below
the azygos vein.
The right main bronchus is shorter, wider and more vertical than the left main bronchus and
therefore the route taken by most foreign bodies.

Image sourced from Wikipedia

Left lung
Above the hilum is the furrow produced by the aortic arch, and then superiorly the groove
accommodating the left subclavian artery; Behind the hilum and pulmonary ligament is a
vertical groove produced by the descending aorta, and in front of this, near the base of the
lung, is the lower part of the oesophagus.
The root of the left lung passes under the aortic arch and in front of the descending aorta.

http://www.emrcs.com/question/question.php?q=0

1/2

26/05/2015

Image sourced from Wikipedia

Inferior borders of both lungs


6th rib in mid clavicular line
8th rib in mid axillary line
10th rib posteriorly
The pleura runs two ribs lower than the corresponding lung level.
Bronchopulmonary segments
Segment number

Right lung

Left lung

Apical

Apical

Posterior

Posterior

Anterior

Anterior

Lateral

Superior lingular

Medial

Inferior lingular

Superior (apical)

Superior (apical)

Medial basal

Medial basal

Anterior basal

Anterior basal

Lateral basal

Lateral basal

10

Posterior basal

Posterior basal

Rate question:

Next question

Comment on this question

All contents of this site are 2012 E-Medical Revision Ltd

http://www.emrcs.com/question/question.php?q=0

Terms and Conditions

Privacy policy

2/2

26/05/2015

Reference ranges

Question 36 of 146

Previous

Next

Question stats

End and review

Score: 65%
1

11.9%

12.9%

29.9%

A. Preprostatic part

36.8%

B. Prostatic part

8.5%

C. Membranous part

36.8% of users answered this


question correctly

Which of the following regions of the male urethra is entirely surrounded by Bucks fascia?

D. Spongiose part
E. None of the above

6
7
8
9

Search
Next question
Go

10
11

Theme from 2010 Exam


Bucks fascia is a layer of deep fascia that covers the penis it is continuous with the external
spermatic fascia and the penile suspensory ligament. The membranous part of the urethra
may partially pass through Bucks fascia as it passes into the penis. However, the spongiose
part of the urethra is contained wholly within Bucks fascia.

12
13
14
15

Image of penile cross section


Bucks fascia corresponds to the layer of deep fascia

16
17
18
19
20
21
22
23

Image sourced from Wikipedia

24
25
26

Urethral anatomy

27-29 2 / 3

Female urethra
The female urethra is shorter and more acutely angulated than the male urethra. It is an
extra-peritoneal structure and embedded in the endopelvic fascia. The neck of the bladder is
subjected to transmitted intra-abdominal pressure and therefore deficiency in this area may
result in stress urinary incontinence. Between the layers of the urogenital diaphragm the
female urethra is surrounded by the external urethral sphincter, this is innervated by the
pudendal nerve. It ultimately lies anterior to the vaginal orifice.

30

Male urethra
In males the urethra is much longer and is divided into four parts.

36

Pre-prostatic
urethra

Extremely short and lies between the bladder and prostate gland.It has a
stellate lumen and is between 1 and 1.5cm long.Innervated by sympathetic
noradrenergic fibres, as this region is composed of striated muscles bundles
they may contract and prevent retrograde ejaculation.

Prostatic
urethra

This segment is wider than the membranous urethra and contains several
openings for the transmission of semen (at the midpoint of the urethral crest).

Membranous
urethra

Narrowest part of the urethra and surrounded by external sphincter. It traverses


the perineal membrane 2.5cm postero-inferior to the symphysis pubis.

Penile
urethra

Travels through the corpus spongiosum on the underside of the penis. It is the
longest urethral segment.It is dilated at its origin as the infrabulbar fossa and
again in the gland penis as the navicular fossa. The bulbo-urethral glands open
into the spongiose section of the urethra 2.5cm below the perineal membrane.

31
32
33
34
35

The urothelium is transitional in nature near to the bladder and becomes squamous more
distally.
Rate question:

Next question

Comment on this question

All contents of this site are 2012 E-Medical Revision Ltd

http://www.emrcs.com/question/question.php?q=0

Terms and Conditions

Privacy policy

1/1

26/05/2015

Reference ranges

Question 37 of 146

Previous

Next

Question stats

End and review

Score: 65.9%
1

9.8%

30.1%

8.1%

A. It is the largest synovial joint in the body

36.9%

B. When the knee is fully extended all ligaments of the knee joint are taut

15.1%

C. Rupture of the anterior cruciate ligament may result in haemarthrosis

36.9% of users answered this


question correctly

Which of the following statements relating to the knee joint is false?

D. The posterior aspect of the patella is extrasynovial


E. The joint is innervated by the femoral, sciatic and obturator nerves

7
8
9

Search
Next question
Go

The posterior aspect is intrasynovial and the knee itself comprises the largest synovial joint
in the body. It may swell considerably following trauma such as ACL injury. Which may be
extremely painful owing to rich innervation from femoral, sciatic and ( a smaller) contribution
from the obturator nerve. During full extension all ligaments are taut and the knee is locked.

10
11
12
13
14
15

Knee joint

16

The knee joint is a synovial joint, the largest and most complicated. It consists of two
condylar joints between the femure and tibia and a sellar joint between the patella and the
femur. The tibiofemoral articular surfaces are incongruent, however, this is improved by the
presence of the menisci. The degree of congruence is related to the anatomical position of
the knee joint and is greatest in full extension.
Knee joint compartments

17
18
19
20
21
22

Tibiofemoral

Comprise of the patella/femur joint, lateral and medial


compartments (betw een femur condyles and tibia)
Synovial membrane and cruciate ligaments partially separate the
medial and lateral compartments

23
24
25

Patellofemoral

Ligamentum patellae
Actions: provides joint stability in full extension

26
27-29 2 / 3
30

Fibrous capsule
The capsule of the knee joint is a complex, composite structure with contributions from
adjacent tendons.
Anterior
fibres

The capsule does not pass proximal to the patella. It blends w ith the
tendinous expansions of vastus medialis and lateralis

Posterior These fibres are vertical and run from the posterior surface of the femoral
fibres
condyles to the posterior aspect of the tibial condyle

31
32
33
34
35
36

Medial
fibres

Attach to the femoral and tibial condyles beyond their articular margins,
blending w ith the tibial collateral ligament

Lateral
fibres

Attach to the femur superior to popliteus, pass over its tendon to head of
fibula and tibial condyle

37

Bursae
Anterior

Subcutaneous prepatellar bursa; betw een patella and skin


Deep infrapatellar bursa; betw een tibia and patellar ligament
Subcutaneous infrapatellar bursa; betw een distal tibial tuberosity and
skin

Laterally

Bursa betw een lateral head of gastrocnemius and joint capsule


Bursa betw een fibular collateral ligament and tendon of biceps femoris
Bursa betw een fibular collateral ligament and tendon of popliteus

Medially

Bursa betw een medial head of gastrocnemius and the fibrous capsule
Bursa betw een tibial collateral ligament and tendons of sartorius, gracilis
and semitendinosus
Bursa betw een the tendon of semimembranosus and medial tibial
condyle and medial head of gastrocnemius

Posterior Highly variable and inconsistent

Ligaments
Medial collateral
ligament

Medial epicondyle femur to medial tibial condyle: valgus


stability

Lateral collateral
ligament

Lateral epicondyle femur to fibula head: varus stability

http://www.emrcs.com/question/question.php?q=0

1/2

26/05/2015

Anterior cruciate
ligament

Anterior tibia to lateral intercondylar notch femur: prevents


tibia sliding anteriorly

Posterior cruciate
ligament

Posterior tibia to medial intercondylar notch femur: prevents


tibia sliding posteriorly

Patellar ligament

Central band of the tendon of quadriceps femoris, extends


from patella to tibial tuberosity

Image sourced from Wikipedia

Image sourced from Wikipedia

Menisci
Medial and lateral menisci compensate for the incongruence of the femoral and tibial
condyles.
Composed of fibrous tissue.
Medial meniscus is attached to the tibial collateral ligament.
Lateral meniscus is attached to the loose fibres at the lateral edge of the joint and is
separate from the fibular collateral ligament. The lateral meniscus is crossed by the popliteus
tendon.
Nerve supply
The knee joint is supplied by the femoral, tibial and common peroneal divisions of the sciatic
and by a branch from the obturator nerve. Hip pathology pain may be referred to the knee.
Blood supply
Genicular branches of the femoral artery, popliteal and anterior tibial arteries all supply the
knee joint.
Rate question:

Next question

Comment on this question

All contents of this site are 2012 E-Medical Revision Ltd

http://www.emrcs.com/question/question.php?q=0

Terms and Conditions

Privacy policy

2/2

26/05/2015

Reference ranges

Previous

Question 38 of 146

Next

A 48 year old lady is undergoing a left sided adrenalectomy for an adrenal adenoma. The
superior adrenal artery is injured and starts to bleed, from which of the following does this
vessel arise?

Question stats

Score: 64.3%
1

16.9%

42.1%

23.5%

11.3%

6.2%

A. Left renal artery


42.1% of users answered this
question correctly

B. Inferior phrenic artery

End and review

6
7

C. Aorta
8

D. Splenic
9

Search

E. None of the above

Go

10
11

Next question

12

The superior adrenal artery is a branch of the inferior phrenic artery.

13
14

Adrenal gland anatomy

15
16

Anatomy

17

Location

Superomedially to the upper pole of each kidney

Relationships of
the right adrenal

Diaphragm-Posteriorly, Kidney-Inferiorly, Vena Cava-Medially, Hepatorenal pouch and bare area of the liver-Anteriorly

Relationships of
the left adrenal

Crus of the diaphragm-Postero- medially, Pancreas and splenic


vessels-Inferiorly, Lesser sac and stomach-Anteriorly

Arterial supply

Superior adrenal arteries- from inferior phrenic artery, Middle adrenal


arteries - from aorta, Inferior adrenal arteries -from renal arteries

Venous drainage
of the right adrenal

18
19
20
21
22
23
24
25

Via one central vein directly into the IVC

26
27-29 2 / 3

Venous drainage
of the left adrenal

Via one central vein into the left renal vein

30
31
32

Rate question:

33

Next question

34
Comment on this question

35
36
37
38

All contents of this site are 2012 E-Medical Revision Ltd

http://www.emrcs.com/question/question.php?q=0

Terms and Conditions

Privacy policy

1/1

26/05/2015

Reference ranges

Previous

Question 39 of 146

Next

Question stats

End and review

Score: 65.1%
1

11.1%

15.3%

8.2%

A. Superior gluteal artery

52.7%

B. Internal pudendal vessels

12.7%

C. Sciatic nerve

52.7% of users answered this


question correctly

Which of the following does not exit the pelvis through the greater sciatic foramen?

D. Obturator nerve
E. Inferior gluteal nerve

6
7
8
9

Search
Next question
Go

The obturator nerve exits through the obturator foramen.

10
11
12

Greater sciatic foramen

13
14

Contents
Nerves

15

Sciatic Nerve
Superior and Inferior Gluteal Nerves
Pudendal Nerve
Posterior Femoral Cutaneous Nerve
Nerve to Quadratus Femoris
Nerve to Obturator internus

16
17
18
19
20

Vessels

21

Superior Gluteal Artery and vein


Inferior Gluteal Artery and vein
Internal Pudendal Artery and vein

22
23
24

Piriformis
The piriformis is a landmark for identifying structures passing out of the sciatic notch
Above piriformis: Superior gluteal vessels
Below piriformis: Inferior gluteal vessels, sciatic nerve (10% pass through it, <1%
above it), posterior cutaneous nerve of the thigh

25
26
27-29 2 / 3
30
31
32

Greater sciatic foramen boundaries

33

Anterolaterally

Greater sciatic notch of the ilium

Posteromedially

Sacrotuberous ligament

Inferior

Sacrospinous ligament and the ischial spine

37

Superior

Anterior sacroiliac ligament

38

34
35
36

39

The greater sciatic foramen

http://www.emrcs.com/question/question.php?q=0

1/2

26/05/2015
Image sourced from Wikipedia

Structures passing between both foramina (Medial to lateral)


Pudendal nerve
Internal pudendal artery
Nerve to obturator internus

Contents of the lesser sciatic foramen


Tendon of the obturator internus
Pudendal nerve
Internal pudendal artery and vein
Nerve to the obturator internus

Rate question:

Next question

Comment on this question

All contents of this site are 2012 E-Medical Revision Ltd

http://www.emrcs.com/question/question.php?q=0

Terms and Conditions

Privacy policy

2/2

26/05/2015

Reference ranges

Previous

Question 40 of 146

Next

Question stats

End and review

Score: 65.9%
1

12.3%

40.7%

17.2%

A. The hypoglossal canal transmits the hypoglossal nerve

17.2%

B. The foramen spinosum is at the base of the medial pterygoid plate.

12.6%

C. The jugular foramen transmits the accessory nerve

40.7% of users answered this


question correctly

Which statement is false about the foramina of the skull?

D. The foramen lacerum is located in the sphenoid bone


E. The stylomastoid foramen transmits the facial nerve

6
7
8
9

Search
Next question
Go

10
11
12

Foramina of the base of the skull

13
14

Foramen

Location

Contents

Foramen
ovale

Sphenoid
bone

Otic ganglion
V3 (Mandibular nerve:3rd branch of
trigeminal)
Accessory meningeal artery
Lesser petrosal nerve
Emissary veins

15
16
17
18
19
20

Foramen
spinosum

Sphenoid
bone

Middle meningeal artery


Meningeal branch of the Mandibular nerve

21

Foramen
rotundum

Sphenoid
bone

Maxillary nerve (V2)

23

Foramen
lacerum/
carotid canal

Sphenoid
bone

Base of the medial pterygoid plate.


Internal carotid artery*
Nerve and artery of the pterygoid canal

22

24
25
26
27-29 2 / 3

Jugular
foramen

Temporal
bone

Anterior: inferior petrosal sinus


Intermediate: glossopharyngeal, vagus, and accessory nerves.
Posterior: sigmoid sinus (becoming the internal jugular vein) and
some meningeal branches from the occipital and ascending
pharyngeal arteries.

30
31
32
33

Foramen
magnum

Occipital
bone

Anterior and posterior spinal arteries


Vertebral arteries
Medulla oblongata

34
35
36

Stylomastoid
foramen

Temporal
bone

Stylomastoid artery
Facial nerve

Superior
orbital fissure

Sphenoid
bone

Oculomotor nerve (III)


trochlear nerve (IV)
lacrimal, frontal and nasociliary branches of ophthalmic nerve (V1)
abducent nerve (VI)
Superior and inferior ophthalmic vein

37
38
39
40

*= In life the foramen lacerum is occluded by a cartilagenous plug. The ICA initially passes
into the carotid canal which ascends superomedially to enter the cranial cavity through the
foramen lacerum.
Base of skull anatomical overview

Image sourced from Wikipedia

http://www.emrcs.com/question/question.php?q=0

1/2

26/05/2015

Rate question:

Next question

Comment on this question

All contents of this site are 2012 E-Medical Revision Ltd

http://www.emrcs.com/question/question.php?q=0

Terms and Conditions

Privacy policy

2/2

26/05/2015

Reference ranges

Question 41 of 146

Previous

Next

An 80 year old lady with a caecal carcinoma is undergoing a right hemicolectomy performed
through a transverse incision. The procedure is difficult and the incision is extended medially
by dividing the rectus sheath. Brisk arterial haemorrhage ensues. From which of the following
does the damaged vessel originate?

Question stats

Score: 66.7%
1

13.1%

39.2%

15.4%

11.3%

21%

39.2% of users answered this


question correctly

A. Internal iliac artery

End and review

B. External iliac artery

6
7
8

C. Superior vesical artery

Search

D. Inferior vesical artery


E. None of the above

Go

10
11

Next question

12
13

The vessel damaged is the epigastric artery. This originates from the external iliac artery
(see below).

14
15

Epigastric artery

16
17

The inferior epigastric artery arises from the external iliac artery immediately above the
inguinal ligament. It then passes along the medial margin of the deep inguinal ring. From
here it continues superiorly to lie behind the rectus abdominis muscle.

18
19
20

This is illustrated below:

21
22
23
24
25
26
27-29 2 / 3
30
31
32
33
34
35
36
37

Image sourced from Wikipedia

38
39

Rate question:

40
Next question

41
Comment on this question

All contents of this site are 2012 E-Medical Revision Ltd

http://www.emrcs.com/question/question.php?q=0

Terms and Conditions

Privacy policy

1/1

26/05/2015

Reference ranges

Question 42 of 146

Previous

Next

A 73 year old man has a large abdominal aortic aneurysm. During a laparotomy for planned
surgical repair the surgeons find the aneurysm is far more proximally located and lies near
the origin of the SMA. During the dissection a vessel lying transversely across the aorta is
injured. What is this vessel most likely to be?

Question stats

Score: 67.4%
1

50.8%

19.7%

9.4%

9.4%

10.6%

50.8% of users answered this


question correctly

A. Left renal vein

End and review

B. Right renal vein

6
7
8

C. Inferior mesenteric artery

Search

D. Ileocolic artery
E. Middle colic artery

Go

10
11

Next question

12
13

Theme from April 2012 Exam


The left renal vein runs across the surface of the aorta and may require deliberate ligation
during juxtarenal aneurysm repair.

14
15
16

Abdominal aorta

17
18

Abdominal aortic topography

19

Origin

T12

Termination

L4

21

Posterior relations

L1-L4 Vertebral bodies

22

Anterior relations

Lesser omentum
Liver
Left renal vein
Inferior mesenteric vein
Third part of duodenum
Pancreas
Parietal peritoneum
Peritoneal cavity

20

23
24
25
26
27-29 2 / 3
30
31

Right lateral relations

Left lateral relations

Right crus of the diaphragm


Cisterna chyli
Azygos vein
IVC (becomes posterior distally)

32
33
34
35

4th part of duodenum


Duodenal-jejunal flexure
Left sympathetic trunk

36
37
38

The abdominal aorta

39
40
41
42

Image sourced from Wikipedia

Rate question:

Next question

Comment on this question

http://www.emrcs.com/question/question.php?q=0

1/2

26/05/2015

All contents of this site are 2012 E-Medical Revision Ltd

http://www.emrcs.com/question/question.php?q=0

Terms and Conditions

Privacy policy

2/2

26/05/2015

Reference ranges

Question 1 of 104

Next

Question stats

Score: 100%

13.2%

11.3%

9.4%

A. Thoracodorsal nerve

11%

B. Axillary nerve

55%

C. Radial nerve

55% of users answered this


question correctly

Which of the following is not a branch of the posterior cord of the brachial plexus?

D. Lower subscapular nerve

End and review

E. Musculocutaneous nerve
Search
Next question
Go

Mnemonic branches off the posterior cord


S ubscapular (upper and lower)
T horacodorsal
A xillary
R adial

The musculocutaneous nerve is a branch off the lateral cord.


Brachial plexus

Origin

Anterior rami of C5 to T1

Sections of the
plexus

Roots

Trunks

Roots, trunks, divisions, cords, branches


Mnemonic:Real Teenagers Drink Cold Beer

Located in the posterior triangle


Pass between scalenus anterior and medius

Located posterior to middle third of clavicle


Upper and middle trunks related superiorly to the subclavian
artery
Lower trunk passes over 1st rib posterior to the subclavian
artery

Divisions

Apex of axilla

Cords

Related to axillary artery

Diagram illustrating the branches of the brachial plexus

Image sourced from Wikipedia

Cutaneous sensation of the upper limb

http://www.emrcs.com/question/question.php?q=0

1/2

26/05/2015

Image sourced from Wikipedia

Rate question:

Next question

Comment on this question

All contents of this site are 2012 E-Medical Revision Ltd

http://www.emrcs.com/question/question.php?q=0

Terms and Conditions

Privacy policy

2/2

26/05/2015

Reference ranges

Previous

Question 3 of 104

Next

A 73 year old man presents with a tumour at the tip of his tongue. To which of the following
regions will the tumour initially metastasise?

A. Sub mental nodes


B. Ipsilateral deep cervical nodes

Question stats

End and review

Score: 66.7%
1

53.6%

17.7%

7.3%

13.6%

7.9%

53.6% of users answered this


question correctly

C. Tonsil
D. Ipsilateral superficial cervical nodes
E. Contralateral deep cervical nodes

Search
Go

Next question

Lymphatic drainage of the tongue

The lymphatic drainage of the anterior two thirds of the tongue shows only minimal
communication of lymphatics across the midline, so metastasis to the ipsilateral nodes
is usual.
The lymphatic drainage of the posterior third of the tongue have communicating
networks, as a result early bilateral nodal metastases are more common in this area.
Lymphatics from the tip of the tongue usually pass to the sub mental nodes and from
there to the deep cervical nodes.
Lymphatics from the mid portion of the tongue usually drain to the submandibular
nodes and then to the deep cervical nodes. Mid tongue tumours that are laterally
located will usually drain to the ipsilateral deep cervical nodes, those from more central
regions may have bilateral deep cervical nodal involvement.

Rate question:

Next question

Comment on this question

All contents of this site are 2012 E-Medical Revision Ltd

http://www.emrcs.com/question/question.php?q=0

Terms and Conditions

Privacy policy

1/1

26/05/2015

Reference ranges

Question 4 of 104

Previous

Next

A 78 year old man is undergoing a femoro-popliteal bypass graft. The operation is not
progressing well and the surgeon is complaining of poor access. Retraction of which of the
following structures will improve access to the femoral artery in the groin?

Question stats

End and review

Score: 50%
1

11.8%

19.9%

12.4%

10%

45.9%

A. Quadriceps
45.9% of users answered this
question correctly

B. Adductor longus
C. Adductor magnus
D. Pectineus

Search

E. Sartorius

Go

Next question

At the lower border of the femoral triangle the femoral artery passes under the sartorius
muscle. This can be retracted to improve access.
Femoral triangle anatomy
Boundaries
Superiorly

Inguinal ligament

Laterally

Sartorius

Medially

Adductor longus

Floor

Iliopsoas, adductor longus and pectineus

Roof

Fascia lata and Superficial fascia


Superficial inguinal lymph nodes (palpable below the inguinal ligament)
Great saphenous vein

Image sourced from Wikipedia

Contents
Femoral vein (medial to lateral)
Femoral artery-pulse palpated at the mid inguinal point
Femoral nerve
Deep and superficial inguinal lymph nodes
Lateral cutaneous nerve
Great saphenous vein
Femoral branch of the genitofemoral nerve

Rate question:

Next question

Comment on this question

http://www.emrcs.com/question/question.php?q=0

1/2

26/05/2015

All contents of this site are 2012 E-Medical Revision Ltd

http://www.emrcs.com/question/question.php?q=0

Terms and Conditions

Privacy policy

2/2

26/05/2015

Reference ranges

Previous

Question 5 of 104

Next

A builder falls off a ladder whilst laying roof tiles. He sustains a burst fracture of L3. The MRI
scan shows complete nerve root transection at this level, as a result of the injury. Which
clinical sign will not be present initially?

Question stats

End and review

Score: 60%
1

17.4%

38.2%

7.9%

16.1%

20.4%

A. Flaccid paralysis of the legs


38.2% of users answered this
question correctly

B. Extensor plantar response


C. Sensory loss in the legs
D. Incontinence

Search

E. Areflexia

Go

Next question

In lower motor neuron lesions everything is reduced

The main purpose of this question is to differentiate the features of an UMN lesion and a
LMN lesion. The features of a LMN lesion include:
Flaccid paralysis of muscles supplied
Atrophy of muscles supplied.
Loss of reflexes of muscles supplied.
Muscles fasciculation
For lesions below L1 LMN signs will occur. Hence in an L3 lesion, there will be loss of the
patella reflex but there will be no extensor plantar reflex.
Spinal cord

Located in a canal within the vertebral column that affords it structural support.
Rostrally it continues to the medulla oblongata of the brain and caudally it tapers at a
level corresponding to the L1-2 interspace (in the adult), a central structure, the filum
terminale anchors the cord to the first coccygeal vertebra.
The spinal cord is characterised by cervico-lumbar enlargements and these, broadly
speaking, are the sites which correspond to the brachial and lumbar plexuses
respectively.

There are some key points to note when considering the surgical anatomy of the spinal cord:
* During foetal growth the spinal cord becomes shorter than the spinal canal, hence the adult
site of cord termination at the L1-2 level.
* Due to growth of the vertebral column the spine segmental levels may not always
correspond to bony landmarks as they do in the cervical spine.
* The spinal cord is incompletely divided into two symmetrical halves by a dorsal median
sulcus and ventral median fissure. Grey matter surrounds a central canal that is
continuous rostrally with the ventricular system of the CNS.
* The grey matter is sub divided cytoarchitecturally into Rexeds laminae.
* Afferent fibres entering through the dorsal roots usually terminate near their point of entry
but may travel for varying distances in Lissauers tract. In this way they may establish
synaptic connections over several levels
* At the tip of the dorsal horn are afferents associated with nociceptive stimuli. The ventral
horn contains neurones that innervate skeletal muscle.
The key point to remember when revising CNS anatomy is to keep a clinical perspective in
mind. So it is worth classifying the ways in which the spinal cord may become injured. These
include:

Trauma either direct or as a result of disc protrusion


Neoplasia either by direct invasion (rare) or as a result of pathological vertebral
fracture
Inflammatory diseases such as Rheumatoid disease, or OA (formation of
osteophytes compressing nerve roots etc.

http://www.emrcs.com/question/question.php?q=0

1/2

26/05/2015

Vascular either as a result of stroke (rare in cord) or as complication of aortic


dissection
Infection historically diseases such as TB, epidural abscesses.

The anatomy of the cord will, to an extent dictate the clinical presentation. Some points/
conditions to remember:

Brown- Sequard syndrome-Hemisection of the cord producing ipsilateral loss of


proprioception and upper motor neurone signs, plus contralateral loss of pain and
temperature sensation. The explanation of this is that the fibres decussate at different
levels.
Lesions below L1 will tend to present with lower motor neurone signs

Rate question:

Next question

Comment on this question

All contents of this site are 2012 E-Medical Revision Ltd

http://www.emrcs.com/question/question.php?q=0

Terms and Conditions

Privacy policy

2/2

26/05/2015

Reference ranges

Previous

Question 6 of 104

Next

A 56 year old machinist has his arm entrapped in a steel grinder and is brought to the
emergency department. On examination, he is unable to extend his metacarpophalangeal
joints and abduct his shoulder. He has weakness of his elbow and wrist. What has been
injured?

Question stats

Score: 66.7%
1

8.5%

8.8%

11.7%

13.4%

57.6%

57.6% of users answered this


question correctly

A. Ulnar nerve

End and review

B. Axillary nerve
C. Medial cord of brachial plexus
Search

D. Lateral cord of brachial plexus


E. Posterior cord of brachial plexus

Go

Next question

The posterior cord gives rise to:


Radial nerve ((innervates the triceps, brachioradialis, wrist extensors, and finger
extensors)
Axillary nerve (innervates deltoid and teres minor)
Upper subscapular nerve (innervates subscapularis)
Lower subscapular nerve (innervates teres major and subscapularis)
Thoracodorsal nerve (innervates latissimus dorsi)

Theme from September 2012 exam


This is a description of a posterior cord lesion. Remember that the posterior cord gives rise
to the axillary and radial nerve.

Cords of the brachial plexus


The brachial plexus cords are described according to their relationship with the axillary
artery. The cords pass over the 1st rib near to the dome of the lung and pass beneath the
clavicle immediately posterior to the subclavian artery.
Lateral cord
Anterior divisions of the upper and middle trunks form the lateral cord
Origin of the lateral pectoral nerve (C5, C6, C7)
Medial cord
Anterior division of the lower trunk forms the medial cord
Origin of the medial pectoral nerve (C8, T1), the medial brachial cutaneous nerve
(T1), and the medial antebrachial cutaneous nerve (C8, T1)
Posterior cord
Formed by the posterior divisions of the 3 trunks (C5-T1)
Origin of the upper and lower subscapular nerves (C7, C8 and C5, C6, respectively)
and the thoracodorsal nerve to the latissimus dorsi (also known as the middle
subscapular nerve, C6, C7, C8), axillary and radial nerve

Rate question:

Next question

Comment on this question

All contents of this site are 2012 E-Medical Revision Ltd

http://www.emrcs.com/question/question.php?q=0

Terms and Conditions

Privacy policy

1/1

26/05/2015

Reference ranges

Previous

Question 7 of 104

Next

A 66 year old man with peripheral vascular disease is undergoing a below knee amputation.
In which of the lower leg compartments does peroneus brevis lie?

A. Lateral compartment
B. Anterior compartment
C. Superficial posterior compartment

Question stats

End and review

Score: 71.4%
1

58.1%

12.4%

9.2%

13.2%

7%

58.1% of users answered this


question correctly

6
7

D. Deep posterior compartment


E. None of the above

Search
Next question

Go

The interosseous membrane separates the anterior and posterior compartments. The deep
and superficial compartments are separated by the deep transverse fascia. The peroneus
brevis is part of the lateral compartment.
Fascial compartments of the leg
Compartments of the thigh
Formed by septae passing from the femur to the fascia lata.
Compartment

Nerve

Anterior compartment

Femoral

Medial compartment

Obturator

Posterior compartment
(2 layers)

Sciatic

Muscles
Iliacus
Tensor fasciae latae
Sartorius
Quadriceps femoris

Adductor
longus/magnus/brevis
Gracilis
Obturator externus

Semimembranosus
Semitendinosus
Biceps femoris

Blood supply
Femoral artery

Profunda femoris artery


and obturator artery

Branches of Profunda
femoris artery

Compartments of the lower leg


Separated by the interosseous membrane (anterior and posterior compartments), anterior
fascial septum (separate anterior and lateral compartments) and posterior fascial septum
(separate lateral and posterior compartments)
Compartment

Nerve

Anterior
compartment

Deep
peroneal
nerve

Posterior
compartment

Tibial

Lateral
compartment

Superficial
peroneal

Muscles

Rate question:

http://www.emrcs.com/question/question.php?q=0

Blood
supply
Anterior
tibial
artery

Tibialis anterior
Extensor digitorum longus
Extensor hallucis longus
Peroneus tertius

Muscles: deep and superficial


compartments (separated by deep
transverse fascia)
Deep: Flexor hallucis longus, Flexor digitalis
longus, Tibialis posterior, Popliteus
Superficial: Gastrocnemius, Soleus,
Plantaris

Peroneus longus/brevis

Posterior
tibial

Anterior
tibial

Next question

1/2

26/05/2015
Comment on this question

All contents of this site are 2012 E-Medical Revision Ltd

http://www.emrcs.com/question/question.php?q=0

Terms and Conditions

Privacy policy

2/2

26/05/2015

Reference ranges

Previous

Question 8 of 104

Next

A 70 year old man is due to undergo an arterial bypass procedure for claudication and foot
ulceration. The anterior tibial artery will form the site of the distal arterial anastomosis. Which
of the following structures is not closely related to it?

Question stats

Score: 75%
1

9.1%

15.3%

54%

10.6%

11%

A. Interosseous membrane
54% of users answered this
question correctly

B. Deep peroneal nerve

End and review

6
7

C. Tibialis posterior
8

D. Extensor hallucis longus


Search

E. Dorsalis pedis artery

Go

Next question

As an artery of the anterior compartment, the anterior tibial artery is closely related to tibialis
anterior.
Anterior tibial artery

Begins opposite the distal border of popliteus


Terminates in front of the ankle, continuing as the dorsalis pedis artery
As it descends it lies on the interosseous membrane, distal part of the tibia and front of
the ankle joint
Passes between the tendons of extensor digitorum and extensor hallucis longus
distally
It is related to the deep peroneal nerve, it lies anterior to the middle third of the vessel
and lateral to it in the lower third

Rate question:

Next question

Comment on this question

All contents of this site are 2012 E-Medical Revision Ltd

http://www.emrcs.com/question/question.php?q=0

Terms and Conditions

Privacy policy

1/1

26/05/2015

Reference ranges

Previous

Question 9 of 104

Next

Question stats

End and review

Score: 77.8%
1

18.5%

22.2%

9.3%

A. Obturator internus

11.8%

B. Quadratus femoris

38.3%

C. Gemellus inferior

38.3% of users answered this


question correctly

Which of the muscles below does not cause lateral rotation of the hip?

D. Piriformis
E. Pectineus

6
7
8
9

Search
Next question
Go

Mnemonic lateral hip rotators: P-GO-GO-Q (top to bottom)


Piriformis
Gemellus superior
Obturator internus
Gemellus inferior
Obturator externus
Quadratus femoris

Pectineus adducts and medially rotates the femur.


Hip joint

Head of femur articulates with acetabulum of the pelvis


Both covered by articular hyaline cartilage
The acetabulum forms at the union of the ilium, pubis, and ischium
The triradiate cartilage (Y-shaped growth plate) separates the pelvic bones
The acetabulum holds the femoral head by the acetabular labrum
Normal angle between femoral head and femoral shaft is 130o

Ligaments
Transverse ligament: joints anterior and posterior ends of the articular cartilage
Head of femur ligament (ligamentum teres): acetabular notch to the fovea. Contains
arterial supply to head of femur in children.

Image sourced from Wikipedia

http://www.emrcs.com/question/question.php?q=0

1/2

26/05/2015

Image sourced from Wikipedia

Extracapsular ligaments
Iliofemoral ligament: inverted Y shape. Anterior iliac spine to the trochanteric line
Pubofemoral ligament: acetabulum to lesser trochanter
Ischiofemoral ligament: posterior support. Ischium to greater trochanter.

Blood supply
Medial circumflex femoral and lateral circumflex femoral arteries (Branches of profunda
femoris). Also from the inferior gluteal artery. These form an anastomosis and travel to up
the femoral neck to supply the head.

Rate question:

Next question

Comment on this question

All contents of this site are 2012 E-Medical Revision Ltd

http://www.emrcs.com/question/question.php?q=0

Terms and Conditions

Privacy policy

2/2

26/05/2015

Reference ranges

Previous

Question 9 of 104

Next

Question stats

End and review

Score: 77.8%
1

18.5%

22.2%

9.3%

A. Obturator internus

11.8%

B. Quadratus femoris

38.3%

C. Gemellus inferior

38.3% of users answered this


question correctly

Which of the muscles below does not cause lateral rotation of the hip?

D. Piriformis
E. Pectineus

6
7
8
9

Search
Next question
Go

Mnemonic lateral hip rotators: P-GO-GO-Q (top to bottom)


Piriformis
Gemellus superior
Obturator internus
Gemellus inferior
Obturator externus
Quadratus femoris

Pectineus adducts and medially rotates the femur.


Hip joint

Head of femur articulates with acetabulum of the pelvis


Both covered by articular hyaline cartilage
The acetabulum forms at the union of the ilium, pubis, and ischium
The triradiate cartilage (Y-shaped growth plate) separates the pelvic bones
The acetabulum holds the femoral head by the acetabular labrum
Normal angle between femoral head and femoral shaft is 130o

Ligaments
Transverse ligament: joints anterior and posterior ends of the articular cartilage
Head of femur ligament (ligamentum teres): acetabular notch to the fovea. Contains
arterial supply to head of femur in children.

Image sourced from Wikipedia

http://www.emrcs.com/question/question.php?q=0

1/2

26/05/2015

Image sourced from Wikipedia

Extracapsular ligaments
Iliofemoral ligament: inverted Y shape. Anterior iliac spine to the trochanteric line
Pubofemoral ligament: acetabulum to lesser trochanter
Ischiofemoral ligament: posterior support. Ischium to greater trochanter.

Blood supply
Medial circumflex femoral and lateral circumflex femoral arteries (Branches of profunda
femoris). Also from the inferior gluteal artery. These form an anastomosis and travel to up
the femoral neck to supply the head.

Rate question:

Next question

Comment on this question

All contents of this site are 2012 E-Medical Revision Ltd

http://www.emrcs.com/question/question.php?q=0

Terms and Conditions

Privacy policy

2/2

26/05/2015

Reference ranges

Question 10 of 104

Previous

Next

Question stats

End and review

Score: 80%
1

5.8%

19.9%

16.1%

A. Spinal accessory nerve

11.5%

B. Phrenic nerve

46.8%

C. External jugular vein

46.8% of users answered this


question correctly

Which of the following is not a content of the posterior triangle of the neck?

D. Occipital lymph nodes


E. Internal jugular vein

6
7
8
9

Search
Next question
Go

10

The IJV does not lie in the posterior triangle. However, the terminal branches of the external
jugular vein do.
Posterior triangle of the neck
Boundaries
Apex

Sternocleidomastoid and the Trapezius muscles at the Occipital bone

Anterior

Posterior border of the Sternocleidomastoid

Posterior

Anterior border of the Trapezius

Base

Middle third of the clavicle

Image sourced from Wikipedia

Contents
Nerves

Vessels

Muscles

Lymph
nodes

Accessory nerve
Phrenic nerve
Three trunks of the brachial plexus
Branches of the cervical plexus: Supraclavicular nerve, transverse cervical
nerve, great auricular nerve, lesser occipital nerve

External jugular vein


Subclavian artery

Inferior belly of omohyoid


Scalene

Supraclavicular
Occipital

Rate question:

Next question

Comment on this question

All contents of this site are 2012 E-Medical Revision Ltd

http://www.emrcs.com/question/question.php?q=0

Terms and Conditions

Privacy policy

1/1

26/05/2015

Reference ranges

Previous

Question 11 of 104

Next

Question stats

End and review

Score: 81.8%
1

10.6%

11.6%

7.7%

A. Maxillary nerve

58.5%

B. Buccal nerve

11.6%

C. Zygomatic nerve

58.5% of users answered this


question correctly

Which nerve is at risk during submandibular gland excision?

D. Marginal mandibular nerve


E. Cervical nerve

6
7
8
9

Search
Next question
Go

10
11

The marginal mandibular nerve lies deep to platysma. It supplies the depressor anguli oris
and the depressor labii inferioris. If injured it may lead to facial asymmetry and dribbling.
Submandibular gland
Relations of the submandibular gland
Superficial

Platysma, deep fascia and mandible


Submandibular lymph nodes
Facial vein (facial artery near mandible)
Marginal mandibular nerve
Cervical branch of the facial nerve

Deep

Facial artery (inferior to the mandible)


Mylohoid muscle
Sub mandibular duct
Hyoglossus muscle
Lingual nerve
Submandibular ganglion
Hypoglossal nerve

Submandibular duct (Wharton's duct)


Opens lateral to the lingual frenulum on the anterior floor of mouth.
5 cm length
Lingual nerve wraps around Wharton's duct. As the duct passes forwards it crosses
medial to the nerve to lie above it and then crosses back, lateral to it, to reach a
position below the nerve.

Innervation
Sympathetic innervation- Derived from superior cervical ganglion
Parasympathetic innervation- Submandibular ganglion via lingual nerve

Arterial supply
Branch of the facial artery. The facial artery passes through the gland to groove its deep
surface. It then emerges onto the face by passing between the gland and the mandible.
Venous drainage
Anterior facial vein (lies deep to the Marginal Mandibular nerve)
Lymphatic drainage
Deep cervical and jugular chains of nodes
Rate question:

Next question

Comment on this question

All contents of this site are 2012 E-Medical Revision Ltd

http://www.emrcs.com/question/question.php?q=0

Terms and Conditions

Privacy policy

1/1

26/05/2015

Reference ranges

Previous

Question 12 of 104

Next

In a patient with a carcinoma of the distal sigmoid colon, what is the most likely source of its
blood supply?

A. Ileocolic artery
B. External iliac artery

Question stats

Score: 83.3%
1

8%

7.2%

8.2%

7.9%

68.6%

68.6% of users answered this


question correctly

C. Internal iliac artery

End and review

D. Superior mesenteric artery

6
7
8

E. Inferior mesenteric artery

Search
Go

Next question

10
11
12

Theme from September 2013 Exam


During a high anterior resection of such tumours, the inferior mesenteric artery is ligated.
Note that the branches (mainly middle rectal branch) of the internal iliac artery are important
in maintaining vascularity of the rectal stump and hence the integrity of the anastomoses.
Rectum
The rectum is approximately 12 cm long. It is a capacitance organ. It has both intra and
extraperitoneal components. The transition between the sigmoid colon is marked by the
disappearance of the tenia coli.The extra peritoneal rectum is surrounded by mesorectal fat
that also contains lymph nodes. This mesorectal fatty layer is removed surgically during
rectal cancer surgery (Total Mesorectal Excision). The fascial layers that surround the
rectum are important clinical landmarks, anteriorly lies the fascia of Denonvilliers. Posteriorly
lies Waldeyers fascia.
Extra peritoneal rectum
Posterior upper third
Posterior and lateral middle third
Whole lower third

Relations
Anteriorly (Males)

Rectovesical pouch
Bladder
Prostate
Seminal vesicles

Anteriorly (Females)

Recto-uterine pouch (Douglas)


Cervix
Vaginal wall

Posteriorly

Sacrum
Coccyx
Middle sacral artery

Laterally

Levator ani
Coccygeus

Arterial supply
Superior rectal artery
Venous drainage
Superior rectal vein
Lymphatic drainage
Mesorectal lymph nodes (superior to dentate line)
Internal iliac and then para-aortic nodes
Inguinal nodes (inferior to dentate line)

Rate question:

Next question

Comment on this question

All contents of this site are 2012 E-Medical Revision Ltd

http://www.emrcs.com/question/question.php?q=0

Terms and Conditions

Privacy policy

1/1

26/05/2015

Reference ranges

Previous

Question 13 of 104

Next

Question stats

End and review

Score: 76.9%
1

34.7%

8.2%

8.8%

A. Internal acoustic meatus

40.4%

B. Foramen lacerum

7.8%

C. Foramen spinosum

34.7% of users answered this


question correctly

Which of these openings transmits the facial nerve into the temporal bone?

D. Stylomastoid foramen
E. Jugular foramen

6
7
8
9

Search
Next question
Go

10
11

It enters the temporal bone through the internal acoustic meatus and exits through the
stylomastoid foramen.

12
13

Facial nerve
The facial nerve is the main nerve supplying the structures of the second embryonic
branchial arch. It is predominantly an efferent nerve to the muscles of facial expression,
digastric muscle and also to many glandular structures. It contains a few afferent fibres which
originate in the cells of its genicular ganglion and are concerned with taste.
Supply - 'face, ear, taste, tear'
Face: muscles of facial expression
Ear: nerve to stapedius
Taste: supplies anterior two-thirds of tongue
Tear: parasympathetic fibres to lacrimal glands, also salivary glands

Path
Subarachnoid path
Origin: motor- pons, sensory- nervus intermedius
Pass through the petrous temporal bone into the internal auditory meatus with the
vestibulocochlear nerve. Here they combine to become the facial nerve.

Facial canal path


The canal passes superior to the vestibule of the inner ear
At the medial aspect of the middle ear, it becomes wider and contains the geniculate
ganglion.
- 3 branches:
1. greater petrosal nerve
2. nerve to stapedius
3. chorda tympani
Stylomastoid foramen
Passes through the stylomastoid foramen (tympanic cavity anterior and mastoid
antrum posteriorly)
Posterior auricular nerve and branch to posterior belly of digastric and stylohyoid
muscle

Face
Enters parotid gland and divides into 5 branches:
Temporal branch
Zygomatic branch
Buccal branch
Marginal mandibular branch
Cervical branch

Rate question:

Next question

Comment on this question

All contents of this site are 2012 E-Medical Revision Ltd

http://www.emrcs.com/question/question.php?q=0

Terms and Conditions

Privacy policy

1/2

26/05/2015

http://www.emrcs.com/question/question.php?q=0

2/2

26/05/2015

Reference ranges

Previous

Question 14 of 104

Next

A motor cyclist is involved in a road traffic accident causing severe right shoulder injuries. He
is found to have an adducted, medially rotated shoulder. The elbow is fully extended and the
forearm pronated. Which is the most likely diagnosis?

Question stats

Score: 78.6%
1

24%

50.6%

8.3%

7.3%

9.8%

A. C8, T1 root lesion


50.6% of users answered this
question correctly

B. C5, C6 root lesion

End and review

6
7

C. Radial nerve lesion


8

D. Ulnar nerve lesion


9

Search

E. Axillary nerve lesion

Go

Next question

10
11
12
13

Erbs Palsy C5, C6 lesion


The features include:

14

Waiter's tip position


Loss of shoulder abduction (deltoid and supraspinatus paralysis)
Loss of external rotation of the shoulder (paralysis of infraspinatus and teres
major)
Loss of elbow flexion (paralysis of biceps, brachialis and brachioradialis)
Loss of forearm supination (paralysis of Biceps)

The motorcyclist has had an Erb's palsy (C5, C6 root lesion). This is commonly known to be
associated with birth injury when a baby has a shoulder dystocia.

Brachial plexus

Origin
Sections of the
plexus

Roots

Trunks

Anterior rami of C5 to T1
Roots, trunks, divisions, cords, branches
Mnemonic:Real Teenagers Drink Cold Beer

Located in the posterior triangle


Pass between scalenus anterior and medius

Located posterior to middle third of clavicle


Upper and middle trunks related superiorly to the subclavian
artery
Lower trunk passes over 1st rib posterior to the subclavian
artery

Divisions

Apex of axilla

Cords

Related to axillary artery

Diagram illustrating the branches of the brachial plexus

http://www.emrcs.com/question/question.php?q=0

1/2

26/05/2015
Image sourced from Wikipedia

Cutaneous sensation of the upper limb

Image sourced from Wikipedia

Rate question:

Next question

Comment on this question

All contents of this site are 2012 E-Medical Revision Ltd

http://www.emrcs.com/question/question.php?q=0

Terms and Conditions

Privacy policy

2/2

26/05/2015

Reference ranges

Previous

Question 15 of 104

Next

A patient is due to undergo a right hemicolectomy for a carcinoma of the caecum. Which of
the following vessels will require high ligation to provide optimal oncological control?

A. Middle colic artery


B. Inferior mesenteric artery
C. Superior mesenteric artery

Question stats

End and review

Score: 80%
1

14.1%

9.9%

26.1%

42.9%

7%

42.9% of users answered this


question correctly

D. Ileo-colic artery

7
8

E. None of the above

Search
Go

Next question

10
11
12

The ileo - colic artery supplies the caecum and would require high ligation during a right
hemicolectomy. The middle colic artery should generally be preserved when resecting a
caecal lesion.
This question is essentially asking you to name the vessel supplying the caecum. The SMA
does not directly supply the caecum, it is the ileocolic artery which does this.

13
14
15

Caecum

Location

Proximal right colon below the ileocaecal valve


Intraperitoneal

Posterior relations

Psoas
Iliacus
Femoral nerve
Genitofemoral nerve
Gonadal vessels

Anterior relations

Greater omentum

Arterial supply

Ileocolic artery

Lymphatic drainage

Mesenteric nodes accompany the venous drainage

The caecum is the most distensible part of the colon and in complete large bowel
obstruction with a competent ileocaecal valve the most likely site of eventual
perforation.

Rate question:

Next question

Comment on this question

All contents of this site are 2012 E-Medical Revision Ltd

http://www.emrcs.com/question/question.php?q=0

Terms and Conditions

Privacy policy

1/1

26/05/2015

Reference ranges

Previous

Question 17 of 104

Next

A 72 year old man is undergoing a repair of an abdominal aortic aneurysm. The aorta is
cross clamped both proximally and distally. The proximal clamp is applied immediately inferior
to the renal arteries. Both common iliac arteries are clamped distally. A longitudinal
aortotomy is performed. After evacuating the contents of the aneurysm sac a significant
amount of ongoing bleeding is encountered. This is most likely to originate from:

Question stats

Score: 76.5%
1

9.4%

20.3%

7.6%

15.2%

47.6%

47.6% of users answered this


question correctly

A. The coeliac axis

End and review

6
7
8

B. Testicular artery
C. Splenic artery

Search

D. Superior mesenteric artery

Go

E. Lumbar arteries

10
11
12

Next question

13
14

The lumbar arteries are posteriorly sited and are a common cause of back bleeding during
aortic surgery. The other vessels cited all exit the aorta in the regions that have been cross
clamped.

15
16
17

Abdominal aortic branches

Branches

Level

Paired

Type

Inferior phrenic

T12 (Upper border)

Yes

Parietal

Coeliac

T12

No

Visceral

Superior mesenteric

L1

No

Visceral

Middle suprarenal

L1

Yes

Visceral

Renal

L1-L2

Yes

Visceral

Gonadal

L2

Yes

Visceral

Lumbar

L1-L4

Yes

Parietal

Inferior mesenteric

L3

No

Visceral

Median sacral

L4

No

Parietal

Common iliac

L4

Yes

Terminal

Rate question:

Next question

Comment on this question

All contents of this site are 2012 E-Medical Revision Ltd

http://www.emrcs.com/question/question.php?q=0

Terms and Conditions

Privacy policy

1/1

26/05/2015

Reference ranges

Question 18 of 104

Previous

Next

Question stats

End and review

Score: 77.8%
1

8.4%

8%

13.3%

A. It is supplied by the femoral nerve

25.8%

B. It forms the lateral boundary of the femoral triangle

44.5%

C. The middle third forms the roof of the adductor canal

44.5% of users answered this


question correctly

Which of the following statements relating to sartorius is untrue?

D. It is a flexor of the hip and knee


E. It inserts into the medial femoral condyle

6
7
8
9

Search
Next question
Go

10
11

It inserts into the medial aspect of the upper part of the tibia.

12

Sartorius

13
14
15

Longest strap muscle in the body


Most superficial muscle in the anterior compartment of the thigh

16
17
18

Origin

Anterior superior iliac spine

Insertion

Medial surface of the of the body of the tibia (upper part). It inserts anterior to
gracilis and semitendinosus

Nerve
Supply

Femoral nerve (L2,3)

Action

Important
relations

Flexor of the hip and knee, slight abducts the thigh and rotates it laterally
It assists with medial rotation of the tibia on the femur. For example it would
play a pivotal role in placing the right heel onto the left knee ( and vice versa)
The middle third of this muscle, and its strong underlying fascia forms the roof of
the adductor canal , in which lie the femoral vessels, the saphenous nerve and the
nerve to vastus medialis.

Rate question:

Next question

Comment on this question

All contents of this site are 2012 E-Medical Revision Ltd

http://www.emrcs.com/question/question.php?q=0

Terms and Conditions

Privacy policy

1/1

26/05/2015

Reference ranges

Previous

Question 19 of 104

Next

A 63 year old man undergoes a radical cystectomy for carcinoma of the bladder. During the
procedure there is considerable venous bleeding. What is the primary site of venous
drainage of the urinary bladder?

Question stats

Score: 78.9%
1

49.5%

25.4%

9.8%

8%

7.4%

A. Vesicoprostatic venous plexus


49.5% of users answered this
question correctly

B. Internal iliac vein

End and review

6
7

C. External iliac vein


8

D. Gonadal vein
9

Search

E. Common iliac vein

Go

10
11

Next question

12

The urinary bladder has a rich venous plexus surrounding it, this drains subsequently into
the internal iliac vein. The vesicoprostatic plexus may be a site of considerable venous
bleeding during cystectomy.

13
14
15

Bladder

16
17

The empty bladder is contained within the pelvic cavity. It is usually a three sided pyramid.
The apex of the bladder points forwards towards the symphysis pubis and the base lies
immediately anterior to the rectum or vagina. Continuous with the apex is the median
umbilical ligament, during development this was the site of the uranchus.
The inferior aspect of the bladder is retroperitoneal and the superior aspect covered by
peritoneum. As the bladder distends it will tend to separate the peritoneum from the fascia of
transversalis. For this reason a bladder that is distended due to acute urinary retention may
be approached with a suprapubic catheter that avoids entry into the peritoneal cavity.
The trigone is the least mobile part of the bladder and forms the site of the ureteric orifices
and internal urethral orifice. In the empty bladder the ureteric orifices are approximately 23cm apart, this distance may increase to 5cm in the distended bladder.

18
19

Arterial supply
The superior and inferior vesical arteries provide the main blood supply to the bladder.
These are branches of the internal iliac artery.
Venous drainage
In males the bladder is drained by the vesicoprostatic venous plexus. In females the bladder
is drained by the vesicouterine venous plexus. In both sexes this venous plexus will ultimately
drain to the internal iliac veins.
Lymphatic drainage
Lymphatic drainage is predominantly to the external iliac nodes, internal iliac and obturator
nodes also form sites of bladder lymphatic drainage.
Innervation
Parasympathetic nerve fibres innervate the bladder from the pelvic splanchnic nerves.
Sympathetic nerve fibres are derived from L1 and L2 via the hypogastric nerve plexuses.
The parasympathetic nerve fibres will typically cause detrusor muscle contraction and result
in voiding. The muscle of the trigone is innervated by the sympathetic nervous system. The
external urethral sphincter is under conscious control. During bladder filling the rate of firing
of nerve impulses to the detrusor muscle is low and receptive relaxation occurs. At higher
volumes and increased intra vesical pressures the rate of neuronal firing will increase and
eventually voiding will occur.
Rate question:

Next question

Comment on this question

All contents of this site are 2012 E-Medical Revision Ltd

http://www.emrcs.com/question/question.php?q=0

Terms and Conditions

Privacy policy

1/1

26/05/2015

Reference ranges

Previous

Question 20 of 104

Next

A man sustains a laceration between the base of the little finger and wrist. Several weeks
after the injury there is loss of thumb adduction power. Which nerve is most likely to have
been injured?

Question stats

Score: 75%
1

18.4%

49%

14.2%

8.2%

10.1%

A. Superficial ulnar nerve


49% of users answered this
question correctly

B. Deep ulnar nerve

End and review

6
7

C. Median nerve
8

D. Radial nerve
9

Search

E. Recurrent branch of median nerve

Go

10
11

Next question

12

Theme from 2009 Exam


Theme from April 2014 Exam

13

Ulnar nerve injury at wrist

15

14

16

Branches of the ulnar nerve in the wrist and hand


At the wrist the ulnar nerve divides into superficial and deep branches. The superficial
branch lies deep to the palmaris brevis. It divides into two; to produce digital nerves, which
innervate the skin of the medial third of the palm and the palmar surface of one and a half
fingers.
The deep branch arises from the nerve on the flexor retinaculum lateral to the pisiform bone.
It passes posteriorly between the abductor and short flexor of the little finger supplying them,
and supplying and piercing the opponens digiti minimi near its origin from the flexor
retinaculum, turns laterally over the distal surface of the Hook of the Hamate bone. It
eventually passes between the two heads of adductor pollicis with the deep palmar arch and
ends in the first dorsal interosseous muscle. In the palm the deep branch also innervates the
lumbricals and interosseous muscles.
Rate question:

17
18
19
20

Next question

Comment on this question

All contents of this site are 2012 E-Medical Revision Ltd

http://www.emrcs.com/question/question.php?q=0

Terms and Conditions

Privacy policy

1/1

26/05/2015

Reference ranges

Previous

Question 21 of 104

Next

A 60 year old female is undergoing a Whipples procedure for adenocarcinoma of the


pancreas. As the surgeons begin to mobilise the pancreatic head they identify a large vessel
passing inferiorly over the anterior aspect of the uncinate process. What is it likely to be?

Question stats

Score: 76.2%
1

47.8%

13%

10.1%

9.5%

19.6%

A. Superior mesenteric artery


47.8% of users answered this
question correctly

B. Coeliac axis

End and review

6
7

C. Inferior mesenteric artery


8

D. Aorta
9

Search

E. Left gastric artery

Go

Next question

10
11
12

Theme from January 2012 Exam


The superior mesenteric artery arises from the aorta and passes anterior to the lower part of
the pancreas. Invasion of this structure is a relative contra indication to resectional surgery.

13
14
15

Pancreas

16
17

The pancreas is a retroperitoneal organ and lies posterior to the stomach. It may be
accessed surgically by dividing the peritoneal reflection that connects the greater omentum
to the transverse colon. The pancreatic head sits in the curvature of the duodenum. Its tail
lies close to the hilum of the spleen, a site of potential injury during splenectomy.
Relations
Posterior to the pancreas
Pancreatic head

Inferior vena cava


Common bile duct
Right and left renal veins
Superior mesenteric vein and artery

Pancreatic neck

Superior mesenteric vein, portal vein

Pancreatic body-

Left renal vein


Crus of diaphragm
Psoas muscle
Adrenal gland
Kidney
Aorta

Pancreatic tail

Left kidney

18
19
20
21

Anterior to the pancreas


Pancreatic head

1st part of the duodenum


Pylorus
Gastroduodenal artery
SMA and SMV(uncinate process)

Pancreatic body

Stomach
Duodenojejunal flexure

Pancreatic tail

Splenic hilum

Superior to the pancreas


Coeliac trunk and its branches common hepatic artery and splenic artery
Grooves of the head of the pancreas
2nd and 3rd part of the duodenum
Arterial supply
Head: pancreaticoduodenal artery
Rest: splenic artery

Venous drainage
Head: superior mesenteric vein
Body and tail: splenic vein

Ampulla of Vater
http://www.emrcs.com/question/question.php?q=0

1/2

26/05/2015

Merge of pancreatic duct and common bile duct


Is an important landmark, halfway along the second part of the duodenum, that marks
the anatomical transition from foregut to midgut (also the site of transition between
regions supplied by coeliac trunk and SMA).

Image sourced from Wikipedia

Rate question:

Next question

Comment on this question

All contents of this site are 2012 E-Medical Revision Ltd

http://www.emrcs.com/question/question.php?q=0

Terms and Conditions

Privacy policy

2/2

27/05/2015

Reference ranges

Question 1 of 83

Next

A 23 year old man has a cannula inserted into his cephalic vein. Through which structure
does the cephalic vein pass?

A. Interosseous membrane
B. Triceps

Question stats

End and review

Score: 100%

12.6%

9.2%

7.8%

54.1%

16.4%

54.1% of users answered this


question correctly

C. Pectoralis major
D. Clavipectoral fascia
E. Tendon of biceps

Search
Go

Next question

The cephalic vein is a favored vessel for arteriovenous fistula formation and should be
preserved in patients with end stage renal failure

The cephalic vein penetrates the calvipectoral fascia (but not the pectoralis major) prior to
terminating in the axillary vein.
Cephalic vein
Path
Dorsal venous arch drains laterally into the cephalic vein
Crosses the anatomical snuffbox and travels laterally up the arm
At the antecubital fossa connected to the basilic vein by the median cubital vein
Pierces deep fascia of deltopectoral groove to join axillary vein

Rate question:

Next question

Comment on this question

All contents of this site are 2012 E-Medical Revision Ltd

http://www.emrcs.com/question/question.php?q=0

Terms and Conditions

Privacy policy

1/1

27/05/2015

Reference ranges

Previous

Question 2 of 83

Next

An 18 year old man is undergoing an orchidectomy via a scrotal approach. The surgeons
mobilise the spermatic cord. From which of the following is the outermost layer of this
structure derived?

Question stats

End and review

Score: 100%

9.4%

57.3%

14.3%

7.2%

11.7%

1
2

A. Internal oblique aponeurosis


57.3% of users answered this
question correctly

B. External oblique aponeurosis


C. Transversalis fascia
D. Rectus sheath

Search

E. Campers fascia

Go

Next question

The outermost covering of the spermatic cord is derived from the external oblique
aponeurosis.This layer is added as the cord passes through the superficial inguinal ring.
Scrotal and testicular anatomy
Spermatic cord
Formed by the vas deferens and is covered by the following structures:
Layer

Origin

Internal spermatic fascia

Transversalis fascia

Cremasteric fascia

From the fascial coverings of internal oblique

External spermatic fascia

External oblique aponeurosis

Contents of the cord


Vas deferens

Transmits sperm and accessory gland secretions

Testicular artery

Branch of abdominal aorta supplies testis and


epididymis

Artery of vas deferens

Arises from inferior vesical artery

Cremasteric artery

Arises from inferior epigastric artery

Pampiniform plexus

Venous plexus, drains into right or left testicular


vein

Sympathetic nerve fibres

Lie on arteries, the parasympathetic fibres lie on the


vas

Genital branch of the genitofemoral


nerve

Supplies cremaster

Lymphatic vessels

Drain to lumbar and para-aortic nodes

Scrotum
Composed of skin and closely attached dartos fascia.
Arterial supply from the anterior and posterior scrotal arteries
Lymphatic drainage to the inguinal lymph nodes
Parietal layer of the tunica vaginalis is the innermost layer

Testes
The testes are surrounded by the tunica vaginalis (closed peritoneal sac). The parietal
layer of the tunica vaginalis adjacent to the internal spermatic fascia.
The testicular arteries arise from the aorta immediately inferiorly to the renal arteries.
The pampiniform plexus drains into the testicular veins, the left drains into the left
renal vein and the right into the inferior vena cava.
Lymphatic drainage is to the para-aortic nodes.

Rate question:

Next question

Comment on this question

http://www.emrcs.com/question/question.php?q=0

1/2

27/05/2015

All contents of this site are 2012 E-Medical Revision Ltd

http://www.emrcs.com/question/question.php?q=0

Terms and Conditions

Privacy policy

2/2

27/05/2015

Reference ranges

Question 3 of 83

Previous

Next

Question stats

End and review

Score: 100%
1

10.7%

8.6%

7.5%

A. Trapezium

66.5%

B. Triquetrum

6.6%

C. Trapezoid

66.5% of users answered this


question correctly

Which of the following is not a carpal bone?

D. Trapezius
E. Lunate

Search
Next question
Go

Mnemonic for the Carpal Bones


Sally Likes To Play The Tiny Chrome Harmonica
She Looks Too Pretty Try To Catch Her
Scared Lovers Try Positions That They Can't Handle

Trapezius is a muscle of the back.

Carpal bones
The wrist is comprised of 8 carpal bones, these are arranged in two rows of 4. It is convex
from side to side posteriorly and concave anteriorly.
Diagrammatic image of carpal bones

Image sourced from Wikipedia

Key to image
A

Scaphoid

Lunate

Triquetrum

Pisiform

Trapezium

Trapezoid

Capitate

Hamate

Radius

Ulna

Metacarpals

No tendons attach to: Scaphoid, lunate, triquetrum (stabilised by ligaments)

http://www.emrcs.com/question/question.php?q=0

1/2

27/05/2015

Rate question:

Next question

Comment on this question

All contents of this site are 2012 E-Medical Revision Ltd

http://www.emrcs.com/question/question.php?q=0

Terms and Conditions

Privacy policy

2/2

27/05/2015

Reference ranges

Question 4 of 83

Previous

Next

A 53 year old male presents with a carcinoma of the transverse colon. Which of the following
structures should be ligated close to their origin to maximise clearance of the tumour?

A. Superior mesenteric artery


B. Inferior mesenteric artery

Question stats

End and review

Score: 75%
1

15.1%

8.9%

59.2%

10.5%

6.3%

59.2% of users answered this


question correctly

C. Middle colic artery


D. Ileo-colic artery
E. Superior rectal artery

Search
Go

Next question

The middle colic artery supplies the transverse colon and requires high ligation during
cancer resections. It is a branch of the superior mesenteric artery.
Transverse colon

The right colon undergoes a sharp turn at the level of the hepatic flexure to become
the transverse colon.
At this point it also becomes intraperitoneal.
It is connected to the inferior border of the pancreas by the transverse mesocolon.
The greater omentum is attached to the superior aspect of the transverse colon from
which it can easily be separated. The mesentery contains the middle colic artery and
vein. The greater omentum remains attached to the transverse colon up to the splenic
flexure. At this point the colon undergoes another sharp turn.

Relations
Superior

Liver and gall-bladder, the greater curvature of the stomach, and the low er end
of the spleen

Inferior

Small intestine

Anterior

Greater omentum

Posterior From right to left w ith the descending portion of the duodenum, the head of the
pancreas, convolutions of the jejunum and ileum, spleen

Rate question:

Next question

Comment on this question

All contents of this site are 2012 E-Medical Revision Ltd

http://www.emrcs.com/question/question.php?q=0

Terms and Conditions

Privacy policy

1/1

27/05/2015

Reference ranges

Question 6 of 83

Previous

Next

Question stats

End and review

Score: 83.3%
1

9.2%

16.8%

8.2%

A. Psoas major

23.8%

B. 12th rib

42%

C. Quadratus lumborum

42% of users answered this


question correctly

Which of the following structures does not lie posterior to the right kidney?

D. Medial arcuate ligament

E. 10th rib
Search
Next question
Go

Theme from April 2012 Exam


The 8th and10th ribs lie more superiorly. The 12th rib is a closer relation posteriorly.
Quadratus lumborum runs between the posterior part of the iliac crest, iliolumbar ligament
and the transverse processes of the lower lumbar vertebrae to the medial part of the lower
border of the last rib and transverse process of the upper lumbar vertebrae. In these last two
locations it is posterior to the kidney.
Renal anatomy
Each kidney is about 11cm long, 5cm wide and 3cm thick. They are located in a deep gutter
alongside the projecting vertebral bodies, on the anterior surface of psoas major. In most
cases the left kidney lies approximately 1.5cm higher than the right. The upper pole of both
kidneys approximates with the 11th rib (beware pneumothorax during nephrectomy). On the
left hand side the hilum is located at the L1 vertebral level and the right kidney at level L1-2.
The lower border of the kidneys is usually alongside L3.
The table below shows the anatomical relations of the kidneys:
Relations
Relations

Right Kidney

Left Kidney

Posterior

Quadratus lumborum, diaphragm,


psoas major, transversus abdominis

Quadratus lumborum, diaphragm,


psoas major, transversus abdominis

Anterior

Hepatic flexure of colon

Stomach, Pancreatic tail

Superior

Liver, adrenal gland

Spleen, adrenal gland

Fascial covering
Each kidney and suprarenal gland is enclosed within a common layer of investing fascia,
derived from the transversalis fascia. It is divided into anterior and posterior layers (Gerotas
fascia).
Renal structure
Kidneys are surrounded by an outer cortex and an inner medulla which usually contains
between 6 and 10 pyramidal structures. The papilla marks the innermost apex of these. They
terminate at the renal pelvis, into the ureter.
Lying in a hollow within the kidney is the renal sinus. This contains:
1. Branches of the renal artery
2. Tributaries of the renal vein
3. Major and minor calyces's
4. Fat
Structures at the renal hilum
The renal vein lies most anteriorly, then renal artery (it is an end artery) and the ureter lies
most posterior.
Rate question:

Next question

Comment on this question

All contents of this site are 2012 E-Medical Revision Ltd

http://www.emrcs.com/question/question.php?q=0

Terms and Conditions

Privacy policy

1/1

27/05/2015

Reference ranges

Question 7 of 83

Previous

Next

A 56 year old man is undergoing a radical nephrectomy via a posterior approach. Which of
the following structures is most likely to be encountered during the operative approach?

A. 8th rib
B. 10th rib

Question stats

Score: 85.7%
1

7.1%

11.4%

7.2%

67.1%

7.2%

67.1% of users answered this


question correctly

C. 6th rib

End and review

6
7

D. 12th rib
E. 9th rib

Search
Go

Next question

The 11th and 12th ribs lie posterior to the kidneys and may be encountered during a
posterior approach. A pneumothorax is a recognised complication of this type of surgery.
Renal anatomy
Each kidney is about 11cm long, 5cm wide and 3cm thick. They are located in a deep gutter
alongside the projecting vertebral bodies, on the anterior surface of psoas major. In most
cases the left kidney lies approximately 1.5cm higher than the right. The upper pole of both
kidneys approximates with the 11th rib (beware pneumothorax during nephrectomy). On the
left hand side the hilum is located at the L1 vertebral level and the right kidney at level L1-2.
The lower border of the kidneys is usually alongside L3.
The table below shows the anatomical relations of the kidneys:
Relations
Relations

Right Kidney

Left Kidney

Posterior

Quadratus lumborum, diaphragm,


psoas major, transversus abdominis

Quadratus lumborum, diaphragm,


psoas major, transversus abdominis

Anterior

Hepatic flexure of colon

Stomach, Pancreatic tail

Superior

Liver, adrenal gland

Spleen, adrenal gland

Fascial covering
Each kidney and suprarenal gland is enclosed within a common layer of investing fascia,
derived from the transversalis fascia. It is divided into anterior and posterior layers (Gerotas
fascia).
Renal structure
Kidneys are surrounded by an outer cortex and an inner medulla which usually contains
between 6 and 10 pyramidal structures. The papilla marks the innermost apex of these. They
terminate at the renal pelvis, into the ureter.
Lying in a hollow within the kidney is the renal sinus. This contains:
1. Branches of the renal artery
2. Tributaries of the renal vein
3. Major and minor calyces's
4. Fat
Structures at the renal hilum
The renal vein lies most anteriorly, then renal artery (it is an end artery) and the ureter lies
most posterior.
Rate question:

Next question

Comment on this question

All contents of this site are 2012 E-Medical Revision Ltd

http://www.emrcs.com/question/question.php?q=0

Terms and Conditions

Privacy policy

1/1

27/05/2015

Reference ranges

Question 8 of 83

Previous

Next

A 73 year old lady presents with a femoral hernia. Which of the following structures forms the
lateral wall of the femoral canal?

A. Pubic tubercle
B. Femoral vein

Question stats

Score: 87.5%
1

7.7%

50%

9.2%

16.3%

16.9%

50% of users answered this


question correctly

C. Femoral artery

End and review

D. Conjoint tendon

6
7
8

E. Femoral nerve

Search
Next question

Go

The canal exists to allow for the physiological expansion of the femoral vein, which lies lateral
to it.
Femoral canal
The femoral canal lies at the medial aspect of the femoral sheath. The femoral sheath is a
fascial tunnel containing both the femoral artery laterally and femoral vein medially. The
canal lies medial to the vein.
Borders of the femoral canal
Laterally

Femoral vein

Medially

Lacunar ligament

Anteriorly

Inguinal ligament

Posteriorly

Pectineal ligament

Image showing dissection of femoral canal

Image sourced from Wikipedia

Contents
Lymphatic vessels
Cloquet's lymph node

Physiological significance
Allows the femoral vein to expand to allow for increased venous return to the lower limbs.
Pathological significance
As a potential space, it is the site of femoral hernias. The relatively tight neck places these at
high risk of strangulation.

http://www.emrcs.com/question/question.php?q=0

1/2

27/05/2015

Rate question:

Next question

Comment on this question

All contents of this site are 2012 E-Medical Revision Ltd

http://www.emrcs.com/question/question.php?q=0

Terms and Conditions

Privacy policy

2/2

27/05/2015

Reference ranges

Previous

Question 9 of 83

Next

Question stats

End and review

Score: 77.8%
1

11.3%

9.6%

7.4%

A. Subscapularis

7.3%

B. Teres minor

64.4%

C. Supraspinatus

64.4% of users answered this


question correctly

Which of the following is not a muscle of the rotator cuff?

D. Infraspinatus
E. Deltoid

6
7
8
9

Search
Next question
Go

Deltoid may abduct the shoulder and is not a rotator cuff muscle.
Muscles of the rotator cuff

Muscle

Innervation

Supraspinatus muscle

Suprascapular nerve

Infraspinatus muscle

Suprascapular nerve

Teres minor muscle

Axillary nerve

Subscapularis muscle

Superior and inferior subscapular nerves

Rate question:

Next question

Comment on this question

All contents of this site are 2012 E-Medical Revision Ltd

http://www.emrcs.com/question/question.php?q=0

Terms and Conditions

Privacy policy

1/1

27/05/2015

Reference ranges

Previous

Question 10 of 83

Next

Question stats

End and review

Score: 80%
1

66.1%

7.6%

8.2%

A. Peroneus brevis

8.1%

B. Flexor digitorum longus

9.9%

C. Soleus

66.1% of users answered this


question correctly

Which of the following muscles is not within the posterior compartment of the lower leg?

D. Popliteus
E. Flexor hallucis longus

6
7
8
9

Search
Next question
Go

10

Theme from 2007 Exam


Peroneus brevis lies in the lateral compartment.
Fascial compartments of the leg
Compartments of the thigh
Formed by septae passing from the femur to the fascia lata.
Compartment

Nerve

Anterior compartment

Femoral

Medial compartment

Obturator

Posterior compartment
(2 layers)

Sciatic

Muscles
Iliacus
Tensor fasciae latae
Sartorius
Quadriceps femoris

Adductor
longus/magnus/brevis
Gracilis
Obturator externus

Semimembranosus
Semitendinosus
Biceps femoris

Blood supply
Femoral artery

Profunda femoris artery


and obturator artery

Branches of Profunda
femoris artery

Compartments of the lower leg


Separated by the interosseous membrane (anterior and posterior compartments), anterior
fascial septum (separate anterior and lateral compartments) and posterior fascial septum
(separate lateral and posterior compartments)
Compartment

Nerve

Anterior
compartment

Deep
peroneal
nerve

Posterior
compartment

Tibial

Lateral
compartment

Superficial
peroneal

Muscles

Rate question:

Blood
supply
Anterior
tibial
artery

Tibialis anterior
Extensor digitorum longus
Extensor hallucis longus
Peroneus tertius

Muscles: deep and superficial


compartments (separated by deep
transverse fascia)
Deep: Flexor hallucis longus, Flexor digitalis
longus, Tibialis posterior, Popliteus
Superficial: Gastrocnemius, Soleus,
Plantaris

Peroneus longus/brevis

Posterior
tibial

Anterior
tibial

Next question

Comment on this question

http://www.emrcs.com/question/question.php?q=0

1/2

27/05/2015

All contents of this site are 2012 E-Medical Revision Ltd

http://www.emrcs.com/question/question.php?q=0

Terms and Conditions

Privacy policy

2/2

27/05/2015

Reference ranges

Previous

Question 11 of 83

Next

Question stats

End and review

Score: 72.7%
1

9.9%

10.4%

47.3%

A. One

22.1%

B. Two

10.3%

C. Three

47.3% of users answered this


question correctly

How many unpaired branches leave the abdominal aorta to supply the abdominal viscera?

D. Four
E. Five

6
7
8
9

Search
Next question
Go

10
11

There are three unpaired branches to the abdominal viscera. These include the coeliac axis,
the SMA and IMA. Branches to the adrenals, renal arteries and gonadal vessels are paired.
The fourth unpaired branch of the abdominal aorta, the median sacral artery, does not
directly supply the abdominal viscera.
Abdominal aortic branches

Branches

Level

Paired

Type

Inferior phrenic

T12 (Upper border)

Yes

Parietal

Coeliac

T12

No

Visceral

Superior mesenteric

L1

No

Visceral

Middle suprarenal

L1

Yes

Visceral

Renal

L1-L2

Yes

Visceral

Gonadal

L2

Yes

Visceral

Lumbar

L1-L4

Yes

Parietal

Inferior mesenteric

L3

No

Visceral

Median sacral

L4

No

Parietal

Common iliac

L4

Yes

Terminal

Rate question:

Next question

Comment on this question

All contents of this site are 2012 E-Medical Revision Ltd

http://www.emrcs.com/question/question.php?q=0

Terms and Conditions

Privacy policy

1/1

27/05/2015

Reference ranges

Previous

Question 12 of 83

Next

A 34 year old man with a submandibular gland stone is undergoing excision of the
submandibular gland. The incision is sited transversely approximately 4cm below the
mandible. After incising the skin, platysma and deep fascia which of the following structures
is most likely to be encountered.

Question stats

Score: 66.7%
1

27.5%

28%

21.4%

15.6%

7.5%

28% of users answered this


question correctly

A. Facial artery

End and review

B. Facial vein

6
7
8

C. Lingual nerve

Search

D. Hypoglossal nerve
E. Glossopharyngeal nerve

Go

10
11

Next question

12

When approaching the submandibular gland the facial vein and submandibular lymph nodes
are the most superficially encountered structures. Each sub mandibular gland has a
superficial and deep part, separated by the mylohyoid muscle. The facial artery passes in a
groove on the superficial aspect of the gland. It then emerges onto the surface of the face by
passing between the gland and the mandible. The facial vein is encountered first in this
surgical approach because the incision is made 4cm below the mandible (to avoid injury to
the marginal mandibular nerve).
Submandibular gland
Relations of the submandibular gland
Superficial

Platysma, deep fascia and mandible


Submandibular lymph nodes
Facial vein (facial artery near mandible)
Marginal mandibular nerve
Cervical branch of the facial nerve

Deep

Facial artery (inferior to the mandible)


Mylohoid muscle
Sub mandibular duct
Hyoglossus muscle
Lingual nerve
Submandibular ganglion
Hypoglossal nerve

Submandibular duct (Wharton's duct)


Opens lateral to the lingual frenulum on the anterior floor of mouth.
5 cm length
Lingual nerve wraps around Wharton's duct. As the duct passes forwards it crosses
medial to the nerve to lie above it and then crosses back, lateral to it, to reach a
position below the nerve.

Innervation
Sympathetic innervation- Derived from superior cervical ganglion
Parasympathetic innervation- Submandibular ganglion via lingual nerve

Arterial supply
Branch of the facial artery. The facial artery passes through the gland to groove its deep
surface. It then emerges onto the face by passing between the gland and the mandible.
Venous drainage
Anterior facial vein (lies deep to the Marginal Mandibular nerve)
Lymphatic drainage
Deep cervical and jugular chains of nodes
Rate question:

Next question

Comment on this question

All contents of this site are 2012 E-Medical Revision Ltd

http://www.emrcs.com/question/question.php?q=0

Terms and Conditions

Privacy policy

1/1

27/05/2015

Reference ranges

Question 13 of 83

Previous

Next

Question stats

End and review

Score: 69.2%
1

25.3%

15.6%

30.5%

8.9%

A. Does not include the middle cerebral artery

19.8%

B. Asymmetry of the circle of willis is a risk factor for the development of


intracranial aneurysms

30.5% of users answered this


question correctly

You are working as an anatomy demonstrator and the medical students decide to test your
knowledge on the Circle of Willis. Which of the following comments is false?

C. Majority of blood passing through the vessels mix together

6
7
8

D. Includes the anterior communicating arteries

Search

E. The circle surrounds the stalk of the pituitary gland

Go

Next question

10
11
12

There is minimum mixing of blood passing through the vessels.

13

Circle of Willis
The two internal carotid arteries and two vertebral arteries form an anastomosis known as
the Circle of Willis on the inferior surface of the brain. Each half of the circle is formed by:
1. Anterior communicating artery
2. Anterior cerebral artery
3. Internal carotid artery
4. Posterior communicating artery
5. Posterior cerebral arteries and the termination of the basilar artery
The circle and its branches supply; the corpus striatum, internal capsule, diencephalon and
midbrain.

Image sourced from Wikipedia

Vertebral arteries
Enter the cranial cavity via foramen magnum
Lie in the subarachnoid space
Ascend on anterior surface of medulla oblongata
Unite to form the basilar artery at the base of the pons

Branches:
Posterior spinal artery
Anterior spinal artery
Posterior inferior cerebellar artery

Basilar artery
Branches:

http://www.emrcs.com/question/question.php?q=0

1/2

27/05/2015

Anterior inferior cerebellar artery


Labyrinthine artery
Pontine arteries
Superior cerebellar artery
Posterior cerebral artery

Internal carotid arteries


Branches:
Posterior communicating artery
Anterior cerebral artery
Middle cerebral artery
Anterior choroid artery

Rate question:

Next question

Comment on this question

All contents of this site are 2012 E-Medical Revision Ltd

http://www.emrcs.com/question/question.php?q=0

Terms and Conditions

Privacy policy

2/2

27/05/2015

Reference ranges

Previous3 / 3

Question 14-16 of 83

Next

Question stats

End and review

Score: 75%

Average score for registered users:

1
2

Theme: Surgical nerve lesions


A. Recurrent laryngeal nerve

14

79.2%

15

90.6%

16

67.1%

B. Accessory nerve

3
4
5

C. Hypoglossal nerve

D. Vagus nerve

Search

E. Common peroneal nerve

7
Go

F. Tibial nerve

8
9

G. Long saphenous nerve


10

H. Phrenic nerve
11

I. Thoracodorsal nerve

12

Please choose which of the listed nerves is at greatest risk for the procedures given. Each
option may be used once, more than once or not at all.

14.

13
14-16 3 / 3

A 64 year old man has a suspected lymphoma and lymph node biopsy from the
posterolateral aspect of the right neck is planned.
Accessory nerve
The accessory nerve has a superficial course and is easily injured. It lies under
platysma and may be divided during the early part of the procedure.

15.

A 43 year old women is due to undergo an axillary node clearance following a


positive sentinel node biopsy.
Thoracodorsal nerve
The long thoracic nerve is also at risk. This nerve traverses the level 2 axillary
nodes to supply latissimus dorsi and may be divided or damaged with
diathermy.

16.

A 53 year old man is to undergo a thyroidectomy.


Recurrent laryngeal nerve
Recurrent laryngeal nerve injury may complicate thyroid surgery in up to 2% of
cases.

Next question

Nerve lesions during surgery

A variety of different procedures carry the risk of iatrogenic nerve injury. These are important
not only from the patients perspective but also from a medicolegal standpoint.
The following operations and their associated nerve lesions are listed here:
Posterior triangle lymph node biopsy and accessory nerve lesion.
Lloyd Davies stirrups and common peroneal nerve.
Thyroidectomy and laryngeal nerve.
Anterior resection of rectum and hypogastric autonomic nerves.
Axillary node clearance; long thoracic nerve, thoracodorsal nerve and
intercostobrachial nerve.
Inguinal hernia surgery and ilioinguinal nerve.
Varicose vein surgery- sural and saphenous nerves.
Posterior approach to the hip and sciatic nerve.
Carotid endarterectomy and hypoglossal nerve.

There are many more, with sound anatomical understanding of the commonly performed
procedures the incidence of nerve lesions can be minimised. They commonly occur when
surgeons operate in an unfamiliar tissue plane or by blind placement of haemostats (not
recommended).

http://www.emrcs.com/question/question.php?q=0

1/2

27/05/2015

Rate question:

Next question

Comment on this question

All contents of this site are 2012 E-Medical Revision Ltd

http://www.emrcs.com/question/question.php?q=0

Terms and Conditions

Privacy policy

2/2

27/05/2015

Reference ranges

Previous

Question 17 of 83

Next

A 45 year old man presents with bilateral inguinal hernias. The surgical team decide to repair
these laparoscopically through an extraperitoneal approach. Through an infraumbilical
incision the surgeons displace the inferior aspect of the rectus abdominis muscle anteriorly
and place a prosthetic mesh into the area to repair the hernias. Which structure will lie
posterior to the mesh?

Question stats

Score: 76.5%
1

39.7%

17.9%

18.7%

16%

7.8%

39.7% of users answered this


question correctly

A. Peritoneum

End and review

6
7
8

B. Internal oblique aponeurosis


C. External oblique aponeurosis

Search

D. Posterior aspect of the rectus sheath

Go

E. Bucks fascia

10
11
12

Next question

13
14-16 3 / 3

During a TEP repair of inguinal hernia the only structure to lie posterior to the mesh is
peritoneum. The question is really only asking which structure lies posterior to the rectus
abdominis muscle. Since this region is below the arcuate line the transversalis fascia and
peritoneum lie posterior to it. Bucks fascia lies in the penis.

17

Rectus abdominis muscle


The rectus sheath is formed by the aponeuroses of the lateral abdominal wall muscles. The
rectus sheath has a composition that varies according to anatomical level.
1. Above the costal margin the anterior sheath is composed of external oblique aponeurosis,
the costal cartilages are posterior to it.
2. From the costal margin to the arcuate line, the anterior rectus sheath is composed of
external oblique aponeurosis and the anterior part of the internal oblique aponeurosis. The
posterior part of the internal oblique aponeurosis and transversus abdominis form the
posterior rectus sheath.
3. Below the arcuate line the aponeuroses of all the abdominal muscles lie in anterior aspect
of the rectus sheath. Posteriorly lies the transversalis fascia and peritoneum.
The arcuate line is the point at which the inferior epigastric vessels enter the rectus sheath.
Rate question:

Next question

Comment on this question

All contents of this site are 2012 E-Medical Revision Ltd

http://www.emrcs.com/question/question.php?q=0

Terms and Conditions

Privacy policy

1/1

27/05/2015

Reference ranges

Previous

Question 18 of 83

Next

A 20 year old man undergoes an open appendicectomy performed via a lanz incision. This
surgeon places the incision on a level of the anterior superior iliac spine in an attempt to
improve cosmesis. During the procedure the appendix is found to be retrocaecal and the
incision is extended laterally. Which of the following nerves is at greatest risk of injury?

Question stats

Score: 77.8%
1

13.4%

49.2%

8.3%

22.5%

6.5%

49.2% of users answered this


question correctly

A. Genitofemoral

End and review

B. Ilioinguinal

6
7
8

C. Obturator

Search

D. Lateral femoral cutaneous


E. Femoral

Go

10
11

Next question

12
13

Theme from April 2012 Exam

14-16 3 / 3
17

Ilioinguinal nerve

18

Arises from the first lumbar ventral ramus with the iliohypogastric nerve. It passes
inferolaterally through the substance of psoas major and over the anterior surface of
quadratus lumborum. It pierces the internal oblique muscle and passes deep to the
aponeurosis of the external oblique muscle. It enters the inguinal canal and then passes
through the superficial inguinal ring to reach the skin.
Branches
To supply those muscles of the abdominal wall through which it passes.
Skin and fascia over the pubic symphysis, superomedial part of the femoral triangle,
surface of the scrotum, root and dorsum of penis or labum majus in females.

Rate question:

Next question

Comment on this question

All contents of this site are 2012 E-Medical Revision Ltd

http://www.emrcs.com/question/question.php?q=0

Terms and Conditions

Privacy policy

1/1

27/05/2015

Reference ranges

Question 19 of 83

Previous

Next

The femoral nerve is transected by a rather careless surgeon during a botched femoropopliteal bypass operation. Which of the following actions will be impaired?

A. Extension of the great toe


B. Adduction of the thigh

Question stats

Score: 78.9%
1

8.9%

9.7%

16.3%

58.9%

6.2%

58.9% of users answered this


question correctly

C. Flexion of the knee joint

End and review

D. Extension of the knee joint

6
7
8

E. Eversion of the foot

Search
Next question

Go

10
11

The femoral nerve supplies the quadriceps muscle which is responsible for extension at the
knee joint.

12
13
14-16 3 / 3

Femoral nerve

17
18

Root values

L2, 3, 4

Innervates

19

Pectineus
Sartorius
Quadriceps femoris
Vastus lateralis/medialis/intermedius

Branches

Medial cutaneous nerve of thigh


Saphenous nerve
Intermediate cutaneous nerve of thigh

Path
Penetrates psoas major and exits the pelvis by passing under the inguinal ligament to enter
the femoral triangle, lateral to the femoral artery and vein.

Image sourced from Wikipedia

Mnemonic for femoral nerve supply


(don't) M I S V Q Scan for PE
M edial cutaneous nerve of the thigh
I ntermediate cutaneous nerve of the thigh
S aphenous nerve
V astus
Q uadriceps femoris
S artorius

http://www.emrcs.com/question/question.php?q=0

1/2

27/05/2015

PE ectineus

Rate question:

Next question

Comment on this question

All contents of this site are 2012 E-Medical Revision Ltd

http://www.emrcs.com/question/question.php?q=0

Terms and Conditions

Privacy policy

2/2

27/05/2015

Reference ranges

Previous

Question 20 of 83

Next

Question stats

End and review

Score: 80%
1

21.8%

13.3%

39%

A. Greater petrosal nerve

14.9%

B. Nerve to stapedius

11.1%

C. Auriculotemporal

39% of users answered this


question correctly

Which of the following is not a direct branch of the facial nerve?

D. Chorda tympani
E. Buccal

6
7
8
9

Search
Next question
Go

10
11

The auriculotemporal nerve is a direct branch of the mandibular nerve.


Other branches of the mandibular nerve include:
Lingual
Inferior Alveolar
N. To the Mylohyoid
Mental

12
13
14-16 3 / 3
17
18

Facial nerve

19
20

The facial nerve is the main nerve supplying the structures of the second embryonic
branchial arch. It is predominantly an efferent nerve to the muscles of facial expression,
digastric muscle and also to many glandular structures. It contains a few afferent fibres which
originate in the cells of its genicular ganglion and are concerned with taste.
Supply - 'face, ear, taste, tear'
Face: muscles of facial expression
Ear: nerve to stapedius
Taste: supplies anterior two-thirds of tongue
Tear: parasympathetic fibres to lacrimal glands, also salivary glands

Path
Subarachnoid path
Origin: motor- pons, sensory- nervus intermedius
Pass through the petrous temporal bone into the internal auditory meatus with the
vestibulocochlear nerve. Here they combine to become the facial nerve.

Facial canal path


The canal passes superior to the vestibule of the inner ear
At the medial aspect of the middle ear, it becomes wider and contains the geniculate
ganglion.
- 3 branches:
1. greater petrosal nerve
2. nerve to stapedius
3. chorda tympani
Stylomastoid foramen
Passes through the stylomastoid foramen (tympanic cavity anterior and mastoid
antrum posteriorly)
Posterior auricular nerve and branch to posterior belly of digastric and stylohyoid
muscle

Face
Enters parotid gland and divides into 5 branches:
Temporal branch
Zygomatic branch
Buccal branch
Marginal mandibular branch
Cervical branch

Rate question:

http://www.emrcs.com/question/question.php?q=0

Next question

1/2

27/05/2015
Comment on this question

All contents of this site are 2012 E-Medical Revision Ltd

http://www.emrcs.com/question/question.php?q=0

Terms and Conditions

Privacy policy

2/2

27/05/2015

Reference ranges

Question 21 of 83

Previous

Next

A 32 year old man is stabbed in the neck and the inferior trunk of his brachial plexus is
injured. Which of the modalities listed below is least likely to be affected?

A. Initiating abduction of the shoulder


B. Abduction of the fingers

Question stats

Score: 81%
1

47.1%

10.7%

17.8%

11.8%

12.7%

47.1% of users answered this


question correctly

C. Flexion of the little finger

End and review

D. Sensation on the palmar aspect of the little finger

6
7
8

E. Gripping a screwdriver

Search
Next question

Go

10
11
12

Inferior trunk of brachial plexus.


C8 and T1 roots
Contributes to ulnar nerve and part of median nerve

13
14-16 3 / 3
17
18

Theme from September 2012 Exam


The inferior trunk of the brachial plexus is rarely injured. Nerve roots C8 and T1 are the main
contributors to this trunk. Therefore an injury to this site will most consistently affect the ulnar
nerve. The inferior trunk also contributes to the median nerve by way of the posterior division
and therefore some impairment of grip is almost inevitable.

19
20
21

Brachial plexus

Origin

Anterior rami of C5 to T1

Sections of the
plexus

Roots

Trunks

Roots, trunks, divisions, cords, branches


Mnemonic:Real Teenagers Drink Cold Beer

Located in the posterior triangle


Pass between scalenus anterior and medius

Located posterior to middle third of clavicle


Upper and middle trunks related superiorly to the subclavian
artery
Lower trunk passes over 1st rib posterior to the subclavian
artery

Divisions

Apex of axilla

Cords

Related to axillary artery

Diagram illustrating the branches of the brachial plexus

Image sourced from Wikipedia

Cutaneous sensation of the upper limb

http://www.emrcs.com/question/question.php?q=0

1/2

27/05/2015

Image sourced from Wikipedia

Rate question:

Next question

Comment on this question

All contents of this site are 2012 E-Medical Revision Ltd

http://www.emrcs.com/question/question.php?q=0

Terms and Conditions

Privacy policy

2/2

27/05/2015

Reference ranges

Previous

Question 22 of 83

Next

Question stats

End and review

Score: 81.8%
1

36.7%

25.3%

17.5%

12.1%

A. Ventral and dorsal endodermal outgrowths of the duodenum

8.4%

B. Ventral and dorsal outgrowths of mesenchymal tissue from the


posterior abdominal wall

36.7% of users answered this


question correctly

During embryological development, which of the following represent the correct origin of the
pancreas?

C. Ventral and dorsal outgrowths of the vitellointestinal duct

6
7
8

D. Ventral and dorsal biliary tract diverticulae

Search

E. Buds from the inferior aspect of the caudate lobe

Go

Next question

10
11
12

The pancreas develops from a ventral and dorsal endodermal outgrowth of the duodenum.
The ventral arises close to, or in common with the hepatic diverticulum, and the larger, dorsal
outgrowth arises slightly cranial to the ventral extending into the mesoduodenum and
mesogastrium. When the buds eventually fuse the duct of the ventral rudiment becomes the
main pancreatic duct.

13
14-16 3 / 3
17
18
19

Pancreas

20

The pancreas is a retroperitoneal organ and lies posterior to the stomach. It may be
accessed surgically by dividing the peritoneal reflection that connects the greater omentum
to the transverse colon. The pancreatic head sits in the curvature of the duodenum. Its tail
lies close to the hilum of the spleen, a site of potential injury during splenectomy.

21
22

Relations
Posterior to the pancreas
Pancreatic head

Inferior vena cava


Common bile duct
Right and left renal veins
Superior mesenteric vein and artery

Pancreatic neck

Superior mesenteric vein, portal vein

Pancreatic body-

Left renal vein


Crus of diaphragm
Psoas muscle
Adrenal gland
Kidney
Aorta

Pancreatic tail

Left kidney

Anterior to the pancreas


Pancreatic head

1st part of the duodenum


Pylorus
Gastroduodenal artery
SMA and SMV(uncinate process)

Pancreatic body

Stomach
Duodenojejunal flexure

Pancreatic tail

Splenic hilum

Superior to the pancreas


Coeliac trunk and its branches common hepatic artery and splenic artery
Grooves of the head of the pancreas
2nd and 3rd part of the duodenum
Arterial supply
Head: pancreaticoduodenal artery
Rest: splenic artery

Venous drainage
Head: superior mesenteric vein
Body and tail: splenic vein

http://www.emrcs.com/question/question.php?q=0

1/2

27/05/2015

Ampulla of Vater
Merge of pancreatic duct and common bile duct
Is an important landmark, halfway along the second part of the duodenum, that marks
the anatomical transition from foregut to midgut (also the site of transition between
regions supplied by coeliac trunk and SMA).

Image sourced from Wikipedia

Rate question:

Next question

Comment on this question

All contents of this site are 2012 E-Medical Revision Ltd

http://www.emrcs.com/question/question.php?q=0

Terms and Conditions

Privacy policy

2/2

27/05/2015

Reference ranges

Question 23 of 83

Previous

Next

Question stats

End and review

Score: 78.3%
1

14.7%

12.3%

13.1%

A. Ansa cervicalis

42.3%

B. Vagus nerve

17.6%

C. Anterior jugular vein

42.3% of users answered this


question correctly

Which of the following is not a content of the anterior triangle of the neck?

D. Transverse cervical nerve


E. Hypoglossal nerve

6
7
8
9

Search
Next question
Go

10
11

The transverse cervical nerve lies within the posterior triangle. The anterior jugular vein is
formed in the submental region and descends in the superficial fascia near the median
plane. It passes inferior to enter the suprasternal space, it is linked to the contralateral
anterior jugular vein by the jugular venous arch.

12
13
14-16 3 / 3
17

Anterior triangle of the neck

18
19

Boundaries

20

Anterior border of the Sternocleidomastoid


Lower border of mandible
Anterior midline

21
22
23

Sub triangles (divided by Digastric above and Omohyoid)


Muscular triangle: Neck strap muscles
Carotid triangle: Carotid sheath
Submandibular Triangle (digastric)

Contents of the anterior triangle


Digastric triangle

Submandibular gland
Submandibular nodes
Facial vessels
Hypoglossal nerve

Muscular triangle

Strap muscles
External jugular vein

Carotid triangle

Carotid sheath (Common carotid, vagus and internal jugular vein)


Ansa cervicalis

Nerve supply to digastric muscle


Anterior: Mylohyoid nerve
Posterior: Facial nerve

Image sourced from Wikipedia

Rate question:

Next question

Comment on this question

All contents of this site are 2012 E-Medical Revision Ltd

http://www.emrcs.com/question/question.php?q=0

Terms and Conditions

Privacy policy

1/1

27/05/2015

Reference ranges

Previous

Question 24 of 83

Next

A 23 year old man presents with delayed diagnosis of appendicitis. The appendix is
retrocaecal and has perforated causing a psoas abscess. Into which structure does the
psoas major muscle insert?

Question stats

Score: 79.2%
1

17.9%

12.1%

51.1%

12.4%

6.4%

A. Greater trochanter of the femur


51.1% of users answered this
question correctly

B. Linea aspera of the femur

End and review

6
7

C. Lesser trochanter of the femur


8

D. Iliac crest
9

Search

E. None of the above

Go

10
11

Next question

12

Theme based on 2011 exam


The psoas major inserts into the lesser trochanter.

13
14-16 3 / 3
17

Psoas Muscle

18
19

Origin
The deep part originates from the transverse processes of the five lumbar vertebrae, the
superficial part originates from T12 and the first 4 lumbar vertebrae.

20
21

Insertion
Lesser trochanter of the femur.

22

Innervation
Anterior rami of L1 to L3.

24

23

Action
Flexion and external rotation of the hip. Bilateral contraction can raise the trunk from the
supine position.

Rate question:

Next question

Comment on this question

All contents of this site are 2012 E-Medical Revision Ltd

http://www.emrcs.com/question/question.php?q=0

Terms and Conditions

Privacy policy

1/1

27/05/2015

Reference ranges

Previous

Question 25 of 83

Next

A 63 year old man is due to undergo a splenectomy. Which splenic structure lies most
posteriorly?

A. Gastrosplenic ligament
B. Splenic vein

Question stats

Score: 80%
1

13.5%

14%

14.5%

9.1%

48.9%

48.9% of users answered this


question correctly

C. Splenic artery

End and review

D. Splenic notch

6
7
8

E. Lienorenal ligament

Search
Go

Next question

10
11
12

Theme from 2011 Exam


The lienorenal ligament lies most posteriorly. The antero-lateral connection is via the
phrenicocolic ligament. Anteriorly the gastro splenic ligament. These structures condense
around the vessels at the splenic hilum.

13
14-16 3 / 3
17
18

Splenic anatomy

19

The spleen is the largest lymphoid organ in the body. It is an intraperitoneal organ, the
peritoneal attachments condense at the hilum where the vessels enter the spleen. Its blood
supply is from the splenic artery (derived from the coeliac axis) and the splenic vein (which is
joined by the IMV and unites with the SMV).

20
21
22
23
24

Embryology: derived from mesenchymal tissue


Shape: clenched fist
Position: below 9th-12th ribs
Weight: 75-150g

25

Relations
Superiorly- diaphragm
Anteriorly- gastric impression
Posteriorly- kidney
Inferiorly- colon
Hilum: tail of pancreas and splenic vessels
Forms apex of lesser sac (containing short gastric vessels)

Rate question:

Next question

Comment on this question

All contents of this site are 2012 E-Medical Revision Ltd

http://www.emrcs.com/question/question.php?q=0

Terms and Conditions

Privacy policy

1/1

27/05/2015

Reference ranges

Question 26 of 83

Previous

Next

Which of these statements relating to the external carotid is false?

A. It ends by bifurcating into the superficial temporal and ascending


pharyngeal artery
B. Its first branch is the superior thyroid artery
C. The superior thyroid, lingual and facial arteries all arise from its anterior
surface

Question stats

End and review

Score: 80.8%
1

51.3%

9.9%

11.3%

12.1%

15.4%

51.3% of users answered this


question correctly

6
7
8

D. The ascending pharyngeal artery is a medial branch

Search

E. Initially it lies anteromedial to the internal carotid

Go

Next question

10
11
12

It terminates by dividing into the superficial temporal and maxillary branches. The external
carotid has eight branches, 3 from its anterior surface ; thyroid, lingual and facial. The
pharyngeal artery is a medial branch. The posterior auricular and occipital are posterior
branches.

13
14-16 3 / 3
17
18

External carotid artery

19

The external carotid commences immediately lateral to the pharyngeal side wall. It ascends
and lies anterior to the internal carotid and posterior to the posterior belly of digastric and
stylohyoid. More inferiorly it is covered by sternocleidomastoid, passed by hypoglossal
nerves, lingual and facial veins.
It then pierces the fascia of the parotid gland finally dividing into its terminal branches within
the gland itself.

20
21
22
23
24
25

Surface marking of the carotid


This is an imaginary line drawn from the bifurcation of the common carotid passing behind
the angle of the jaw to a point immediately anterior to the tragus of the ear.

26

Branches of the external carotid artery


It has six branches, three in front, two behind and one deep.
Three in front

Superior thyroid
Lingual
Facial

Two behind

Occipital
Posterior auricular

Deep

Ascending pharyngeal

It terminates by dividing into the superficial temporal and maxillary arteries in the parotid
gland.

Image sourced from Wikipedia

Rate question:

Next question

Comment on this question

http://www.emrcs.com/question/question.php?q=0

1/2

27/05/2015

All contents of this site are 2012 E-Medical Revision Ltd

http://www.emrcs.com/question/question.php?q=0

Terms and Conditions

Privacy policy

2/2

27/05/2015

Reference ranges

Previous

Question 27 of 83

Next

Question stats

End and review

Score: 81.5%
1

7.4%

52%

11.7%

A. The hilum contains the splenic vessels.

14.9%

B. The spleen is derived from endodermal tissue.

13.9%

C. The white pulp has immune function.

52% of users answered this


question correctly

Which of the following statements about the spleen is false?

D. The colon lies inferiorly.


E. Weighs 150g.

6
7
8
9

Search
Next question
Go

10
11

1,3,5,7,9,11 (odd numbers up to 11)


The spleen is: 1 inch thick, 3 inches wide, 5 inches long, weighs 7oz (150-200g), lies
between the 9th and 11th ribs

12
13
14-16 3 / 3
17

Most of the gut is derived endodermally except for the spleen which is from mesenchymal
tissue.

18
19
20

Spleen

21

The spleen is located in the left upper quadrant of the abdomen and its size can vary
depending upon the amount of blood it contains. The typical adult spleen is 12.5cm long and
7.5cm wide. The usual weight of the adult spleen is 150g.
The exact position of the spleen can vary with respiratory activity, posture and the state of
surrounding viscera. It usually lies obliquely with its long axis aligned to the 9th, 10th and
11th ribs. It is separated from these ribs by both diaphragm and pleural cavity. The normal
spleen is not palpable.

22
23
24
25
26
27

The shape of the spleen is influenced by the state of the colon and stomach. Gastric
distension will cause the spleen to resemble the shape of an orange segment. Colonic
distension will cause it to become more tetrahedral.
The spleen is almost entirely covered by peritoneum, which adheres firmly to its capsule.
Recesses of the greater sac separate it from the stomach and kidney. It develops from the
upper dorsal mesogastrium, remaining connected to the posterior abdominal wall and
stomach by two folds of peritoneum; the lienorenal ligament and gastrosplenic ligament. The
lienorenal ligament is derived from peritoneum where the wall of the general peritoneum
meets the omental bursa between the left kidney and spleen; the splenic vessels lie in its
layers. The gastrosplenic ligament also has two layers, formed by the meeting of the walls of
the greater sac and omental bursa between spleen and stomach, the short gastric and left
gastroepiploic branches of the splenic artery pass in its layers. Laterally, the spleen is in
contact with the phrenicocolic ligament.

Relations
Superiorly

Diaphragm

Anteriorly

Gastric impression

Posteriorly Kidney
Inferiorly

Colon

Hilum

Tail of pancreas and splenic vessels (splenic artery divides here, branches
pass to the w hite pulp transporting plasma)

Contents
White
pulp

Immune function. Contains central trabecular artery. The germinal centres are
supplied by arterioles called penicilliary radicles.

Red
pulp

Filters abnormal red blood cells.

Function
Filtration of abnormal blood cells and foreign bodies such as bacteria.
Immunity: IgM. Production of properdin, and tuftsin which help target fungi and bacteria
for phagocytosis.
Haematopoiesis: up to 5th month gestation or in haematological disorders.
Pooling: storage of 40% platelets.
Iron reutilisation

http://www.emrcs.com/question/question.php?q=0

1/2

27/05/2015

Storage monocytes

Disorders of the spleen


Massive splenomegaly
Myelofibrosis
Chronic myeloid leukaemia
Visceral leishmaniasis (kala-azar)
Malaria
Gaucher's syndrome

Other causes (as above plus)


Portal hypertension e.g. secondary to cirrhosis
Lymphoproliferative disease e.g. CLL, Hodgkin's
Haemolytic anaemia
Infection: hepatitis, glandular fever
Infective endocarditis
Sickle-cell*, thalassaemia
Rheumatoid arthritis (Felty's syndrome)

*the majority of adult patients with sickle-cell will have an atrophied spleen due to repeated
infarction
Rate question:

Next question

Comment on this question

All contents of this site are 2012 E-Medical Revision Ltd

http://www.emrcs.com/question/question.php?q=0

Terms and Conditions

Privacy policy

2/2

27/05/2015

Reference ranges

Previous

Question 29 of 83

Next

Question stats

End and review

Score: 82.8%
1

4.8%

22.9%

43.2%

A. Vertebral

19.1%

B. External jugular

10%

C. Internal jugular

43.2% of users answered this


question correctly

Into which of the following veins does the middle thyroid vein drain?

D. Subclavian
E. Anterior jugular

6
7
8
9

Search
Next question
Go

10
11

It drains to the internal jugular vein. Which is one of the reasons why it bleeds so copiously if
a ligature slips.

12
13

Thyroid gland

14-16 3 / 3
17
18

Right and left lobes connected by isthmus


Surrounded by sheath from pretracheal layer of deep fascia
Apex: Lamina of thyroid cartilage
Base: 4th-5th tracheal ring
Pyramidal lobe: from isthmus
May be attached to foramen caecum at the base of the tongue

19
20
21
22
23

Relations

24

Anteromedially

25

Sternothyroid
Superior belly of omohyoid
Sternohyoid
Anterior aspect of sternocleidomastoid

26
27
28

Posterolaterally

29

Carotid sheath

Medially

Larynx
Trachea
Pharynx
Oesophagus
Cricothyroid muscle
External laryngeal nerve (near superior thyroid artery)
Recurrent laryngeal nerve (near inferior thyroid artery)

Posterior

Parathyroid glands
Anastomosis of superior and inferior thyroid arteries

Isthmus

Anteriorly: Sternothyroids, sternohyoids, anterior jugular veins


Posteriorly: 2nd, 3rd, 4th tracheal rings (attached via Ligament of
Berry)

Blood Supply
Arterial

Venous

Superior thyroid artery (1st branch of external carotid)


Inferior thyroid artery (from thyrocervical trunk)
Thyroidea ima (in 10% of population -from brachiocephalic artery or aorta)

Superior and middle thyroid veins - into the IJV


Inferior thyroid vein - into the brachiocephalic veins

Rate question:

Next question

Comment on this question

All contents of this site are 2012 E-Medical Revision Ltd

http://www.emrcs.com/question/question.php?q=0

Terms and Conditions

Privacy policy

1/1

27/05/2015

Reference ranges

Previous

Question 30 of 83

Next

Question stats

End and review

Score: 83.3%
1

8.6%

8.6%

7.9%

A. Hilum left kidney

61.5%

B. Superior mesenteric artery

13.4%

C. Fundus of the gallbladder

61.5% of users answered this


question correctly

Which of the following structures is not at the level of the transpyloric plane?

D. Cardioesophageal junction
E. Root of transverse mesocolon

6
7
8
9

Search
Next question
Go

10
11

Cardiooesophageal junction level = T11

12
13
14-16 3 / 3

A knowledge of this anatomic level is commonly tested.


The oesophagus extends from C6 (the lower border of the cricoid cartilage) to T11 at the
cardioesophageal junction. Note that in the neonate the oesophagus extends from C4 or C5
to T9.

17
18
19

Levels

20
21

Transpyloric plane
Level of the body of L1

22
23
24

Pylorus stomach
Left kidney hilum (L1- left one!)
Right hilum of the kidney (1.5cm lower than the left)
Fundus of the gallbladder
Neck of pancreas
Duodenojejunal flexure
Superior mesenteric artery
Portal vein
Left and right colic flexure
Root of the transverse mesocolon
2nd part of the duodenum
Upper part of conus medullaris
Spleen

25
26
27
28
29
30

Can be identified by asking the supine patient to sit up without using their arms. The plane is
located where the lateral border of the rectus muscle crosses the costal margin.
Anatomical planes
Subcostal plane

Lowest margin of 10th costal cartilage

Intercristal plane

Level of body L4 (highest point of iliac crest)

Intertubercular plane

Level of body L5

Common level landmarks


Inferior mesenteric artery

L3

Bifurcation of aorta into common iliac arteries

L4

Formation of IVC

L5 (union of common iliac veins)

Diaphragm apertures

Rate question:

Vena cava T8
Oesophagus T10
Aortic hiatus T12

Next question

Comment on this question

All contents of this site are 2012 E-Medical Revision Ltd

http://www.emrcs.com/question/question.php?q=0

Terms and Conditions

Privacy policy

1/2

27/05/2015

http://www.emrcs.com/question/question.php?q=0

2/2

27/05/2015

Reference ranges

Previous

Question 31 of 83

Next

A 62 year old man presents with arm weakness. On examination he has a weakness of elbow
extension and loss of sensation on the dorsal aspect of the first digit. What is the site of the
most likely underlying defect?

Question stats

Score: 83.9%
1

5.6%

9%

8%

68.3%

9%

68.3% of users answered this


question correctly

A. Axillary nerve
B. Median nerve

End and review

C. Ulnar nerve
8

D. Radial nerve
9

Search

E. Musculocutaneous nerve

Go

Next question

10
11
12

Theme from April 2012 Exam


The long head of the triceps muscle may be innervated by the axillary nerve and therefore
complete loss of triceps muscles function may not be present even with proximally sited
nerve lesions.

13
14-16 3 / 3
17
18

Radial nerve

19
20

Continuation of posterior cord of the brachial plexus (root values C5 to T1)


21
22

Path
In the axilla: lies posterior to the axillary artery on subscapularis, latissimus dorsi and
teres major.
Enters the arm between the brachial artery and the long head of triceps (medial to
humerus).
Spirals around the posterior surface of the humerus in the groove for the radial nerve.
At the distal third of the lateral border of the humerus it then pierces the intermuscular
septum and descends in front of the lateral epicondyle.
At the lateral epicondyle it lies deeply between brachialis and brachioradialis where it
then divides into a superficial and deep terminal branch.
Deep branch crosses the supinator to become the posterior interosseous nerve.

23
24
25
26
27
28
29
30
31

In the image below the relationships of the radial nerve can be appreciated

Image sourced from Wikipedia

Regions innervated
Motor (main
nerve)

Triceps
Anconeus
Brachioradialis
Extensor carpi radialis

Motor
(posterior
interosseous
branch)

Supinator
Extensor carpi ulnaris
Extensor digitorum
Extensor indicis
Extensor digiti minimi
Extensor pollicis longus and brevis
Abductor pollicis longus

Sensory

The area of skin supplying the proximal phalanges on the dorsal aspect of

http://www.emrcs.com/question/question.php?q=0

1/2

27/05/2015
the hand is supplied by the radial nerve (this does not apply to the little
finger and part of the ring finger)

Muscular innervation and effect of denervation


Anatomical
location

Muscle affected

Effect of paralysis

Shoulder

Long head of triceps

Minor effects on shoulder stability in


abduction

Arm

Triceps

Loss of elbow extension

Forearm

Supinator
Brachioradialis
Extensor carpi radialis
longus and brevis

Weakening of supination of prone hand and


elbow flexion in mid prone position

The cutaneous sensation of the upper limb- illustrating the contribution of the radial nerve

Image sourced from Wikipedia

Rate question:

Next question

Comment on this question

All contents of this site are 2012 E-Medical Revision Ltd

http://www.emrcs.com/question/question.php?q=0

Terms and Conditions

Privacy policy

2/2

27/05/2015

Reference ranges

Previous

Question 32 of 83

Next

Question stats

End and review

Score: 84.4%
1

20.1%

12.9%

48.3%

A. Coracoid process

9.9%

B. Acromion

8.7%

C. Infraglenoid tubercle

48.3% of users answered this


question correctly

From which of the following structures does the long head of the triceps muscle arise?

D. Coraco-acromial ligament
E. Coraco-humeral ligament

6
7
8
9

Search
Next question
Go

10
11

The long head arises from the infraglenoid tubercle. The fleshy lateral and medial heads are
attached to the posterior aspect of the humerus between the insertion of the teres minor and
the olecranon fossa.

12
13
14-16 3 / 3

Triceps

17
18

Origin

Insertion

19

Long head- infraglenoid tubercle of the scapula.


Lateral head- dorsal surface of the humerus, lateral and proximal to the
groove of the radial nerve
Medial head- posterior surface of the humerus on the inferomedial side of
the radial groove and both of the intermuscular septae

20
21
22
23

Olecranon process of the ulna. Here the olecranon bursa is between the
triceps tendon and olecranon.
Some fibres insert to the deep fascia of the forearm, posterior capsule of
the elbow (preventing the capsule from being trapped between olecranon
and olecranon fossa during extension)

24
25
26
27
28

Innervation

Radial nerve

Blood
supply

Profunda brachii artery

Action

Elbow extension. The long head can adduct the humerus and and extend it from
a flexed position

Relations

The radial nerve and profunda brachii vessels lie between the lateral and medial
heads

29
30
31

Rate question:

32

Next question

Comment on this question

All contents of this site are 2012 E-Medical Revision Ltd

http://www.emrcs.com/question/question.php?q=0

Terms and Conditions

Privacy policy

1/1

27/05/2015

Reference ranges

Previous

Question 33 of 83

Next

A 45 year old man is undergoing a left hemicolectomy. As the surgeons mobilise the left
colon they identify a tubular structure lying at the inferior aspect of psoas major. What is it
most likely to be?

Question stats

Score: 84.8%
1

65.5%

7.6%

9.2%

9.5%

8.2%

A. Left ureter
65.5% of users answered this
question correctly

B. Left common iliac vein

End and review

6
7

C. Left common iliac artery


8

D. Left external iliac artery


9

Search

E. Left external iliac vein

Go

10
11

Next question

12

The left ureter lies posterior to the left colon. The sigmoid colon and upper rectum may be
more closely related to the iliac vessels. These are not typically found above L4.

13
14-16 3 / 3
17

Ureter

18
19

25-35 cm long
Muscular tube lined by transitional epithelium
Surrounded by thick muscular coat. Becomes 3 muscular layers as it crosses the bony
pelvis
Retroperitoneal structure overlying transverse processes L2-L5
Lies anterior to bifurcation of iliac vessels
Blood supply is segmental; renal artery, aortic branches, gonadal branches, common
iliac and internal iliac
Lies beneath the uterine artery

20
21
22
23
24
25
26
27
28

Rate question:

Next question

29

Comment on this question

30
31
32
33

All contents of this site are 2012 E-Medical Revision Ltd

http://www.emrcs.com/question/question.php?q=0

Terms and Conditions

Privacy policy

1/1

27/05/2015

Reference ranges

Previous

Question 34 of 83

Next

Question stats

End and review

Score: 85.3%
1

12.8%

19.6%

42.5%

A. Medial pterygoid

12.1%

B. Mylohyoid

13%

C. Stylohyoid

42.5% of users answered this


question correctly

Which muscle is not innervated by the trigeminal nerve?

D. Masseter
E. Temporalis

6
7
8
9

Search
Next question
Go

10
11

Stylohyoid is innervated by the facial nerve.

12

Trigeminal nerve

13
14-16 3 / 3

The trigeminal nerve is the main sensory nerve of the head. In addition to its major sensory
role, it also innervates the muscles of mastication.

17
18
19

Distribution of the trigeminal nerve

20

Sensory

Scalp
Face
Oral cavity (and teeth)
Nose and sinuses
Dura mater

21
22
23
24

Motor

Muscles of mastication
Mylohyoid
Anterior belly of digastric
Tensor tympani
Tensor palati

25
26
27
28
29

Autonomic connections (ganglia)

Ciliary
Sphenopalatine
Otic
Submandibular

30
31
32
33
34

Path
Originates at the pons
Sensory root forms the large, crescentic trigeminal ganglion within Meckel's cave, and
contains the cell bodies of incoming sensory nerve fibres. Here the 3 branches exit.
The motor root cell bodies are in the pons and the motor fibres are distributed via the
mandibular nerve. The motor root is not part of the trigeminal ganglion.

Branches of the trigeminal nerve


Ophthalmic nerve

Sensory only

Maxillary nerve

Sensory only

Mandibular nerve

Sensory and motor

Sensory
Ophthalmic

Exits skull via the superior orbital fissure


Sensation of: scalp and forehead, the upper eyelid, the conjunctiva and cornea of
the eye, the nose (including the tip of the nose, except alae nasi), the nasal
mucosa, the frontal sinuses, and parts of the meninges (the dura and blood
vessels).

Maxillary
nerve

Exit skull via the foramen rotundum


Sensation: lower eyelid and cheek, the nares and upper lip, the upper teeth and
gums, the nasal mucosa, the palate and roof of the pharynx, the maxillary,
ethmoid and sphenoid sinuses, and parts of the meninges.

Mandibular
nerve

Exit skull via the foramen ovale


Sensation: lower lip, the lower teeth and gums, the chin and jaw (except the
angle of the jaw), parts of the external ear, and parts of the meninges.

http://www.emrcs.com/question/question.php?q=0

1/2

27/05/2015

Motor
Distributed via the mandibular nerve.
The following muscles of mastication are innervated:
Masseter
Temporalis
Medial pterygoid
Lateral pterygoid

Other muscles innervated include:


Tensor veli palatini
Mylohyoid
Anterior belly of digastric
Tensor tympani

Rate question:

Next question

Comment on this question

All contents of this site are 2012 E-Medical Revision Ltd

http://www.emrcs.com/question/question.php?q=0

Terms and Conditions

Privacy policy

2/2

27/05/2015

Reference ranges

Previous

Question 35 of 83

Next

A 42 year old woman is due to undergo a left nephroureterectomy for a transitional cell
carcinoma involving the ureter. Which of the following structures is not related to the left
ureter?

Question stats

Score: 82.9%
1

32.9%

14.5%

16%

21.7%

15%

A. Round ligament of the uterus


32.9% of users answered this
question correctly

B. Internal iliac artery

End and review

6
7

C. Ovarian artery
8

D. Peritoneum
9

Search

E. Sigmoid mesocolon

Go

10
11

Next question

12

The ureter is not related to the round ligament of the uterus, it is related to the broad
ligament and is within 1.5cm of the supravaginal part of the cervix.

13
14-16 3 / 3
17

Ureter

18
19

25-35 cm long
Muscular tube lined by transitional epithelium
Surrounded by thick muscular coat. Becomes 3 muscular layers as it crosses the bony
pelvis
Retroperitoneal structure overlying transverse processes L2-L5
Lies anterior to bifurcation of iliac vessels
Blood supply is segmental; renal artery, aortic branches, gonadal branches, common
iliac and internal iliac
Lies beneath the uterine artery

20
21
22
23
24
25
26
27
28

Rate question:

Next question

29

Comment on this question

30
31
32
33
34
35

All contents of this site are 2012 E-Medical Revision Ltd

http://www.emrcs.com/question/question.php?q=0

Terms and Conditions

Privacy policy

1/1

27/05/2015

Reference ranges

Previous

Question 36 of 83

Next

Question stats

End and review

Score: 80.6%
1

11.5%

9.7%

54.1%

A. Vertebral artery

13.1%

B. Subscapular artery

11.6%

C. Thyroidea ima artery

54.1% of users answered this


question correctly

Which of the following most commonly arises from the brachiocephalic artery?

D. Left Subclavian artery


E. None of the above

6
7
8
9

Search
Next question
Go

Other occasional branches include the thymic and bronchial branch.

10
11
12

Brachiocephalic artery

13
14-16 3 / 3

The brachiocephalic artery is the largest branch of the aortic arch. From its aortic origin it
ascends superiorly, it initially lies anterior to the trachea and then on its right hand side. It
branches into the common carotid and right subclavian arteries at the level of the
sternoclavicular joint.

17
18
19
20

Path
Origin- apex of the midline of the aortic arch
Passes superiorly and posteriorly to the right
Divides into the right subclavian and right common carotid artery

22
23
24

Relations
Anterior

21

25

Sternohyoid
Sternothyroid
Thymic remnants
Left brachiocephalic vein
Right inferior thyroid veins

26
27
28
29

Posterior

30

Trachea
Right pleura

31
32

Right lateral

33

Right brachiocephalic vein


Superior part of SVC

34
35

Left lateral

Thymic remnants
Origin of left common carotid
Inferior thyroid veins
Trachea (higher level)

36

Branches
Normally none but may have the thyroidea ima artery

http://www.emrcs.com/question/question.php?q=0

1/2

27/05/2015

Image sourced from Wikipedia

Rate question:

Next question

Comment on this question

All contents of this site are 2012 E-Medical Revision Ltd

http://www.emrcs.com/question/question.php?q=0

Terms and Conditions

Privacy policy

2/2

27/05/2015

Reference ranges

Question 37 of 83

Previous

Next

A 28 year old man is undergoing an appendicectomy. The external oblique aponeurosis is


incised and the underlying muscle split in the line of its fibres. At the medial edge of the
wound is a tough fibrous structure. Entry to this structure will most likely encounter which of
the following?

Question stats

Score: 81.1%
1

12.3%

41.4%

14.1%

17.2%

15%

41.4% of users answered this


question correctly

A. Internal oblique

End and review

B. Rectus abdominis

6
7
8

C. Transversus abdominis

Search

D. Linea alba
E. Peritoneum

Go

10
11

Next question

12
13

This structure will be the rectus sheath and when entered the rectus abdominis muscle will
be encountered.

14-16 3 / 3
17
18

Abdominal incisions

19
20

Midline
incision

Commonest approach to the abdomen


Structures divided: linea alba, transversalis fascia, extraperitoneal fat,
peritoneum (avoid falciform ligament above the umbilicus)
Bladder can be accessed via an extraperitoneal approach through the
space of Retzius

21
22
23
24

Paramedian
incision

Parallel to the midline (about 3-4cm)


Structures divided/retracted: anterior rectus sheath, rectus (retracted),
posterior rectus sheath, transversalis fascia, extraperitoneal fat,
peritoneum
Incision is closed in layers

25
26
27
28
29

Battle

Similar location to paramedian but rectus displaced medially (and thus


denervated)
Now seldom used

30
31
32

Kocher's

Incision under right subcostal margin e.g. Cholecystectomy (open)

Lanz

Incision in right iliac fossa e.g. Appendicectomy

Gridiron

Oblique incision centered over McBurneys point- usually appendicectomy


(less cosmetically acceptable than Lanz

33
34

Gable

Rooftop incision

Pfannenstiel's

Transverse supra pubic, primarily used to access pelvic organs

McEvedy's

Groin incision e.g. Emergency repair strangulated femoral hernia

Rutherford
Morrison

Extraperitoneal approach to left or right lower quadrants. Gives excellent


access to iliac vessels and is the approach of choice for first time renal
transplantation.

35
36
37

Image sourced from Wikipedia

http://www.emrcs.com/question/question.php?q=0

1/2

27/05/2015

Rate question:

Next question

Comment on this question

All contents of this site are 2012 E-Medical Revision Ltd

http://www.emrcs.com/question/question.php?q=0

Terms and Conditions

Privacy policy

2/2

27/05/2015

Reference ranges

Question 38 of 83

Previous

Next

A 35 year old man presents to the surgical clinic with a suspected direct inguinal hernia.
These will pass through Hesselbach's triangle. Which of the following forms the medial edge
of this structure?

Question stats

Score: 81.6%
1

9.7%

22.1%

52%

9.8%

6.3%

A. External oblique aponeurosis


52% of users answered this
question correctly

B. Inferior epigastric artery

End and review

6
7

C. Rectus abdominis muscle


8

D. Inferior epigastric vein


9

Search

E. Obturator nerve

Go

10
11

Next question

12

Direct inguinal hernias pass through Hesselbachs triangle (although this is of minimal clinical
significance!). Its medial boundary is the rectus muscle.

13
14-16 3 / 3
17

Hesselbach's triangle

18
19

Direct hernias pass through Hesselbachs triangle.

20

Superolaterally

Epigastric vessels

21

Medially

Lateral edge of rectus muscle

22

Inferiorly

Inguinal ligament

23
24
25

The boundaries of Hesselbachs triangle are commonly tested and illustrated below
26
27
28
29
30
31
32
33
34
35
36
37
38

Image sourced from Wikipedia

Rate question:

Next question

Comment on this question

All contents of this site are 2012 E-Medical Revision Ltd

http://www.emrcs.com/question/question.php?q=0

Terms and Conditions

Privacy policy

1/1

27/05/2015

Reference ranges

Question 39 of 83

Previous

Next

Question stats

End and review

Score: 82.1%
1

12.5%

46.7%

12.6%

A. Sternohyoid

17.8%

B. Mylohyoid

10.5%

C. Omohyoid

46.7% of users answered this


question correctly

Which of the following muscles is not innervated by the ansa cervicalis?

D. Sternothyroid
E. None of the above

6
7
8
9

Search
Next question
Go

10
11

Ansa cervicalis muscles:

12
13

GHost THought SOmeone Stupid Shot Irene

14-16 3 / 3
17

GenioHyoid
ThyroidHyoid
Superior Omohyoid
SternoThyroid
SternoHyoid
Inferior Omohyoid

18
19
20
21
22

Mylohyoid is innervated by the mylohyoid branch of the inferior alveolar nerve.

23
24

Ansa cervicalis
25
26

Superior
root

Branch of C1 anterolateral to carotid sheath

Inferior
root

Derived from C2 and C3 roots, passes posterolateral to the internal jugular vein
(may lie either deep or superficial to it)

Innervation

Sternohyoid
Sternothyroid
Omohyoid

27
28
29
30
31
32
33

The ansa cervicalis lies anterior to the carotid sheath. The nerve supply to the inferior strap
muscles enters at their inferior aspect. Therefore when dividing these muscles to expose a
large goitre, the muscles should be divided in their upper half.

34
35
36
37
38
39

Image sourced from Wikipedia

Rate question:

Next question

Comment on this question

http://www.emrcs.com/question/question.php?q=0

1/2

27/05/2015

All contents of this site are 2012 E-Medical Revision Ltd

http://www.emrcs.com/question/question.php?q=0

Terms and Conditions

Privacy policy

2/2

27/05/2015

Reference ranges

Question 40 of 83

Previous

Next

Question stats

End and review

Score: 82.5%
1

11.3%

45.4%

17.7%

12.3%

A. The internal mammary artery provides the majority of its arterial supply

13.2%

B. Nipple retraction may occur as a result of tumour infiltration of the


clavipectoral fascia

45.4% of users answered this


question correctly

A 58 year old lady presents with a mass in the upper outer quadrant of the right breast.
Which of the following statements relating to the breast is untrue?

C. The internal mammary artery is a branch of the subclavian artery

Search
Go

Next question

7
8

D. Up to 70% of lymphatic drainage is to the ipsilateral axillary nodes


E. None of the above

10
11
12

Both skin dimpling and nipple retraction are features of breast malignancy. However, they
usually occur as a result of tumour infiltration of the breast ligaments and ducts respectively.
The clavipectoral fascia encases the axillary contents. The lymphatic drainage of the breast
is to the axilla and also to the internal mammary chain. The breast is well vascularised and
the internal mammary artery is a branch of the subclavian artery.

13
14-16 3 / 3
17
18
19

Breast

20
21

The breast itself lies on a layer of pectoral fascia and the following muscles:
1. Pectoralis major
2. Serratus anterior
3. External oblique

22
23
24

Image showing the topography of the female breast

25
26
27
28
29
30
31
32
33
34
35
36
37
38

Image sourced from Wikipedia

39
40

Breast anatomy
Nerve supply

Branches of intercostal nerves from T4-T6.

Arterial supply

Venous
drainage

Internal mammary (thoracic) artery


External mammary artery (laterally)
Anterior intercostal arteries
Thoraco-acromial artery
Superficial venous plexus to subclavian, axillary and intercostal veins.

Lymphatic
drainage

70% Axillary nodes


Internal mammary chain
Other lymphatic sites such as deep cervical and supraclavicular fossa
(later in disease)

Rate question:

Next question

Comment on this question

http://www.emrcs.com/question/question.php?q=0

1/2

27/05/2015

All contents of this site are 2012 E-Medical Revision Ltd

http://www.emrcs.com/question/question.php?q=0

Terms and Conditions

Privacy policy

2/2

27/05/2015

Reference ranges

Previous

Question 41 of 83

Next

Question stats

End and review

Score: 82.9%
1

29.8%

8.6%

9.9%

A. Gonads

8.5%

B. Tail of pancreas

43.2%

C. Greater omentum

43.2% of users answered this


question correctly

Where are accessory spleens not found?

D. Splenorenal ligament
E. Ureter

6
7
8
9

Search
Next question
Go

Accessory spleens

10
11
12

- 10% population
- 1 cm size
- locations: hilum of the spleen, tail of the pancreas, along the splenic vessels, in the
gastrosplenic ligament, the splenorenal ligament, the walls of the stomach or intestines, the
greater omentum, the mesentery, the gonads

13
14-16 3 / 3
17
18
19

Spleen

20

The spleen is located in the left upper quadrant of the abdomen and its size can vary
depending upon the amount of blood it contains. The typical adult spleen is 12.5cm long and
7.5cm wide. The usual weight of the adult spleen is 150g.
The exact position of the spleen can vary with respiratory activity, posture and the state of
surrounding viscera. It usually lies obliquely with its long axis aligned to the 9th, 10th and
11th ribs. It is separated from these ribs by both diaphragm and pleural cavity. The normal
spleen is not palpable.

21

The shape of the spleen is influenced by the state of the colon and stomach. Gastric
distension will cause the spleen to resemble the shape of an orange segment. Colonic
distension will cause it to become more tetrahedral.

27

The spleen is almost entirely covered by peritoneum, which adheres firmly to its capsule.
Recesses of the greater sac separate it from the stomach and kidney. It develops from the
upper dorsal mesogastrium, remaining connected to the posterior abdominal wall and
stomach by two folds of peritoneum; the lienorenal ligament and gastrosplenic ligament. The
lienorenal ligament is derived from peritoneum where the wall of the general peritoneum
meets the omental bursa between the left kidney and spleen; the splenic vessels lie in its
layers. The gastrosplenic ligament also has two layers, formed by the meeting of the walls of
the greater sac and omental bursa between spleen and stomach, the short gastric and left
gastroepiploic branches of the splenic artery pass in its layers. Laterally, the spleen is in
contact with the phrenicocolic ligament.

30

22
23
24
25
26

28
29

31
32
33
34
35
36
37
38
39

Relations
Superiorly

Diaphragm

40

Anteriorly

Gastric impression

41

Posteriorly Kidney
Inferiorly

Colon

Hilum

Tail of pancreas and splenic vessels (splenic artery divides here, branches
pass to the w hite pulp transporting plasma)

Contents
White
pulp

Immune function. Contains central trabecular artery. The germinal centres are
supplied by arterioles called penicilliary radicles.

Red
pulp

Filters abnormal red blood cells.

Function
Filtration of abnormal blood cells and foreign bodies such as bacteria.
Immunity: IgM. Production of properdin, and tuftsin which help target fungi and bacteria
for phagocytosis.
Haematopoiesis: up to 5th month gestation or in haematological disorders.
Pooling: storage of 40% platelets.
Iron reutilisation
Storage monocytes

http://www.emrcs.com/question/question.php?q=0

1/2

27/05/2015

Disorders of the spleen


Massive splenomegaly
Myelofibrosis
Chronic myeloid leukaemia
Visceral leishmaniasis (kala-azar)
Malaria
Gaucher's syndrome

Other causes (as above plus)


Portal hypertension e.g. secondary to cirrhosis
Lymphoproliferative disease e.g. CLL, Hodgkin's
Haemolytic anaemia
Infection: hepatitis, glandular fever
Infective endocarditis
Sickle-cell*, thalassaemia
Rheumatoid arthritis (Felty's syndrome)

*the majority of adult patients with sickle-cell will have an atrophied spleen due to repeated
infarction
Rate question:

Next question

Comment on this question

All contents of this site are 2012 E-Medical Revision Ltd

http://www.emrcs.com/question/question.php?q=0

Terms and Conditions

Privacy policy

2/2

28/05/2015

Reference ranges

2/3

Question 1-3 of 42

Next

Question stats
Average score for registered users:

Theme: Nerve injury


A. Median nerve

64%

48.2%

79.9%

End and review

Score: 66.7%
1-3 2 / 3

B. Ulnar nerve
C. Radial nerve
D. Anterior interosseous nerve

Search

E. Posterior interosseous nerve

Go

F. Axillary nerve
G. Musculocutaneous nerve
Please select the nerve at risk of injury in each scenario. Each option may be used once,
more than once or not at all.

1.

A 43 year old typist presents with pain at the dorsal aspect of the upper part of
her forearm. She also complains of weakness when extending her fingers. On
examination triceps and supinator are both functioning normally. There is
weakness of most of the extensor muscles. However, there is no sensory deficit.
Posterior interosseous nerve
The radial nerve may become entrapped in the "arcade of Frohse" which is a
superficial part of the supinator muscle which overlies the posterior interosseous
nerve. This nerve is entirely muscular and articular in its distribution. It passes
postero-inferiorly and gives branches to extensor carpi radialis brevis and
supinator. It enters supinator and curves around the lateral and posterior
surfaces of the radius. On emerging from the supinator the posterior
interosseous nerve lies between the superficial extensor muscles and the
lowermost fibres of supinator. It then gives branches to the extensors.

2.

A 28 year teacher reports difficulty with writing. There is no sensory loss. She is
known to have an aberrant Gantzer muscle.
Anterior interosseous nerve
Anterior interosseous lesions occur due to fracture, or rarely due to
compression. The Gantzer muscle is an aberrant accessory of the flexor pollicis
longus and is a risk factor for anterior interosseous nerve compression.
Remember loss of pincer grip and normal sensation indicates an interosseous
nerve lesion.

3.

A 35 year tennis player attends reporting tingling down his arm. He says that his
'funny bone' was hit very hard by a tennis ball. There is weakness of abduction
and adduction of his extended fingers.
You answered Axillary nerve
The correct answer is Ulnar nerve
Theme from September 2012 exam
The ulnar nerve arises from the medial cord of the brachial plexus (C8, T1 and
contribution from C7). The nerve descends between the axillary artery and vein,
posterior to the cutaneous nerve of the forearm and then lies anterior to triceps
on the medial side of the brachial artery. In the distal half of the arm it passes
through the medial intermuscular septum, and continues between this structure
and the medial head of triceps to enter the forearm between the medial
epicondyle of the humerus and the olecranon. It may be injured at this site in this
scenario.

Next question

Brachial plexus

Origin
Sections of the
plexus

Anterior rami of C5 to T1
Roots, trunks, divisions, cords, branches
Mnemonic:Real Teenagers Drink Cold Beer

http://www.emrcs.com/question/question.php?q=0

1/2

28/05/2015

Roots

Located in the posterior triangle


Pass between scalenus anterior and medius

Trunks

Located posterior to middle third of clavicle


Upper and middle trunks related superiorly to the subclavian
artery
Lower trunk passes over 1st rib posterior to the subclavian
artery

Divisions

Apex of axilla

Cords

Related to axillary artery

Diagram illustrating the branches of the brachial plexus

Image sourced from Wikipedia

Cutaneous sensation of the upper limb

Image sourced from Wikipedia

Rate question:

Next question

Comment on this question

All contents of this site are 2012 E-Medical Revision Ltd

http://www.emrcs.com/question/question.php?q=0

Terms and Conditions

Privacy policy

2/2

28/05/2015

Reference ranges

Question 4 of 42

Previous

Next

A 72 year old man is undergoing a left pneumonectomy for carcinoma of the bronchus. As
the surgeons approach the root of the lung, which structure will lie most anteriorly (in the
anatomical plane)?

Question stats

End and review

Score: 75%
1-3 2 / 3

11.4%

57.1%

9.9%

11.7%

9.8%

A. Vagus nerve
57.1% of users answered this
question correctly

B. Phrenic nerve
C. Bronchus
D. Pulmonary vein

Search

E. Pulmonary artery

Go

Next question

The phrenic nerve is the most anteriorly located structure in the lung root. The vagus nerve
lies most posteriorly.
Lung anatomy
The right lung is composed of 3 lobes divided by the oblique and transverse fissures. The
left lung has two lobes divided by the oblique fissure.The apex of both lungs is approximately
4cm superior to the sterno-costal joint of the first rib. Immediately below this is a sulcus
created by the subclavian artery.
Peripheral contact points of the lung
Base: diaphragm
Costal surface: corresponds to the cavity of the chest
Mediastinal surface: Contacts the mediastinal pleura. Has the cardiac impression.
Above and behind this concavity is a triangular depression named the hilum, where
the structures which form the root of the lung enter and leave the viscus. These
structures are invested by pleura, which, below the hilum and behind the pericardial
impression, forms the pulmonary ligament

Right lung
Above the hilum is the azygos vein; Superior to this is the groove for the superior vena cava
and right innominate vein; behind this, and nearer the apex, is a furrow for the innominate
artery. Behind the hilum and the attachment of the pulmonary ligament is a vertical groove
for the oesophagus; In front and to the right of the lower part of the oesophageal groove is a
deep concavity for the extrapericardiac portion of the inferior vena cava.
The root of the right lung lies behind the superior vena cava and the right atrium, and below
the azygos vein.
The right main bronchus is shorter, wider and more vertical than the left main bronchus and
therefore the route taken by most foreign bodies.

Image sourced from Wikipedia

Left lung
Above the hilum is the furrow produced by the aortic arch, and then superiorly the groove
accommodating the left subclavian artery; Behind the hilum and pulmonary ligament is a
vertical groove produced by the descending aorta, and in front of this, near the base of the
lung, is the lower part of the oesophagus.
The root of the left lung passes under the aortic arch and in front of the descending aorta.

http://www.emrcs.com/question/question.php?q=0

1/2

28/05/2015

Image sourced from Wikipedia

Inferior borders of both lungs


6th rib in mid clavicular line
8th rib in mid axillary line
10th rib posteriorly
The pleura runs two ribs lower than the corresponding lung level.
Bronchopulmonary segments
Segment number

Right lung

Left lung

Apical

Apical

Posterior

Posterior

Anterior

Anterior

Lateral

Superior lingular

Medial

Inferior lingular

Superior (apical)

Superior (apical)

Medial basal

Medial basal

Anterior basal

Anterior basal

Lateral basal

Lateral basal

10

Posterior basal

Posterior basal

Rate question:

Next question

Comment on this question

All contents of this site are 2012 E-Medical Revision Ltd

http://www.emrcs.com/question/question.php?q=0

Terms and Conditions

Privacy policy

2/2

28/05/2015

Reference ranges

Previous

Question 5 of 42

Next

A 56 year old man is undergoing an anterior resection for a carcinoma of the rectum. Which
of the structures below is least likely to be encountered during the mobilisation of the anterior
rectum?

Question stats

End and review

Score: 80%
1-3 2 / 3

19.9%

42.3%

10.7%

7.8%

19.3%

4
5

A. Denonvilliers' fascia
42.3% of users answered this
question correctly

B. Middle sacral artery


C. Bladder
D. Rectovesical pouch

Search

E. Seminal vesicles

Go

Next question

With the exception of the middle sacral artery all of the other structures lie anterior to the
rectum. They may all be palpated during digital rectal examination.
Rectum
The rectum is approximately 12 cm long. It is a capacitance organ. It has both intra and
extraperitoneal components. The transition between the sigmoid colon is marked by the
disappearance of the tenia coli.The extra peritoneal rectum is surrounded by mesorectal fat
that also contains lymph nodes. This mesorectal fatty layer is removed surgically during
rectal cancer surgery (Total Mesorectal Excision). The fascial layers that surround the
rectum are important clinical landmarks, anteriorly lies the fascia of Denonvilliers. Posteriorly
lies Waldeyers fascia.
Extra peritoneal rectum
Posterior upper third
Posterior and lateral middle third
Whole lower third

Relations
Anteriorly (Males)

Rectovesical pouch
Bladder
Prostate
Seminal vesicles

Anteriorly (Females)

Recto-uterine pouch (Douglas)


Cervix
Vaginal wall

Posteriorly

Sacrum
Coccyx
Middle sacral artery

Laterally

Levator ani
Coccygeus

Arterial supply
Superior rectal artery
Venous drainage
Superior rectal vein
Lymphatic drainage
Mesorectal lymph nodes (superior to dentate line)
Internal iliac and then para-aortic nodes
Inguinal nodes (inferior to dentate line)

Rate question:

Next question

Comment on this question

All contents of this site are 2012 E-Medical Revision Ltd

http://www.emrcs.com/question/question.php?q=0

Terms and Conditions

Privacy policy

1/1

28/05/2015

Reference ranges

Previous

Question 6 of 42

Next

In relation to the middle cranial fossa, which of the following statements relating to the
foramina is incorrect?

A. The foramen rotundum transmits the maxillary nerve


B. The foramen lacerum is closely related to the internal carotid artery
C. The foramen spinosum lies posterolateral to the foramen ovale

Question stats

End and review

Score: 83.3%
1-3 2 / 3

8.8%

11%

13.5%

54.9%

11.8%

4
5
6

54.9% of users answered this


question correctly

D. The foramen ovale transmits the middle meningeal artery


E. The foramen rotundum lies anteromedial to the foramen ovale

Search
Next question

Go

Theme addressed in 2010 and 2011 exam


The foramen spinosum transmits the middle meningeal artery. The foramen ovale transmits
the mandibular nerve. As the foramina weaken the bone, a fracture at this site is not
uncommon.
Foramina of the base of the skull

Foramen

Location

Contents

Foramen
ovale

Sphenoid
bone

Otic ganglion
V3 (Mandibular nerve:3rd branch of
trigeminal)
Accessory meningeal artery
Lesser petrosal nerve
Emissary veins

Foramen
spinosum

Sphenoid
bone

Middle meningeal artery


Meningeal branch of the Mandibular nerve

Foramen
rotundum

Sphenoid
bone

Maxillary nerve (V2)

Foramen
lacerum/
carotid canal

Sphenoid
bone

Base of the medial pterygoid plate.


Internal carotid artery*
Nerve and artery of the pterygoid canal

Jugular
foramen

Temporal
bone

Anterior: inferior petrosal sinus


Intermediate: glossopharyngeal, vagus, and accessory nerves.
Posterior: sigmoid sinus (becoming the internal jugular vein) and
some meningeal branches from the occipital and ascending
pharyngeal arteries.

Foramen
magnum

Occipital
bone

Anterior and posterior spinal arteries


Vertebral arteries
Medulla oblongata

Stylomastoid
foramen

Temporal
bone

Stylomastoid artery
Facial nerve

Superior
orbital fissure

Sphenoid
bone

Oculomotor nerve (III)


trochlear nerve (IV)
lacrimal, frontal and nasociliary branches of ophthalmic nerve (V1)
abducent nerve (VI)
Superior and inferior ophthalmic vein

*= In life the foramen lacerum is occluded by a cartilagenous plug. The ICA initially passes
into the carotid canal which ascends superomedially to enter the cranial cavity through the
foramen lacerum.
Base of skull anatomical overview

http://www.emrcs.com/question/question.php?q=0

1/2

28/05/2015

Image sourced from Wikipedia

Rate question:

Next question

Comment on this question

All contents of this site are 2012 E-Medical Revision Ltd

http://www.emrcs.com/question/question.php?q=0

Terms and Conditions

Privacy policy

2/2

28/05/2015

Reference ranges

Previous

Question 7 of 42

Next

During an operation for varicose veins the surgeons are mobilising the long saphenous vein.
Near its point of entry to the femoral vein an artery is injured and bleeding is encountered.
From where is the bleeding most likely to originate?

Question stats

End and review

Score: 71.4%
1-3 2 / 3

19.1%

19.7%

21.8%

10.2%

29.2%

4
5

A. Femoral artery
29.2% of users answered this
question correctly

B. Profunda femoris artery


C. Superficial circumflex iliac artery
D. Superficial epigastric artery

Search

E. Deep external pudendal artery

Go

Next question

Theme from 2011 Exam


The deep external pudendal artery is a branch of the SFA and it runs medially under the
long saphenous vein near its point of union with the femoral vein. The superficial external
pudendal artery lies superior to the SFJ. Neither vessel is functionally important and if injured
they are best ligated.
Femoral triangle anatomy
Boundaries
Superiorly

Inguinal ligament

Laterally

Sartorius

Medially

Adductor longus

Floor

Iliopsoas, adductor longus and pectineus

Roof

Fascia lata and Superficial fascia


Superficial inguinal lymph nodes (palpable below the inguinal ligament)
Great saphenous vein

Image sourced from Wikipedia

Contents
Femoral vein (medial to lateral)
Femoral artery-pulse palpated at the mid inguinal point
Femoral nerve
Deep and superficial inguinal lymph nodes
Lateral cutaneous nerve
Great saphenous vein
Femoral branch of the genitofemoral nerve

http://www.emrcs.com/question/question.php?q=0

1/2

28/05/2015

Rate question:

Next question

Comment on this question

All contents of this site are 2012 E-Medical Revision Ltd

http://www.emrcs.com/question/question.php?q=0

Terms and Conditions

Privacy policy

2/2

28/05/2015

Reference ranges

Previous

Question 8 of 42

Next

A 78 year old man is lifting a heavy object when a feels a pain in his forearm and is unable to
continue. He has a swelling over his upper forearm. An MRI scan shows a small cuff of
tendon still attached to the radial tuberosity consistent with a recent tear. Which of the
following muscles has been injured?

Question stats

Score: 62.5%
1-3 2 / 3

11.8%

10.6%

9.5%

24.8%

43.3%

43.3% of users answered this


question correctly

A. Pronator teres

End and review

4
5

B. Supinator
C. Aconeus
Search

D. Brachioradialis
E. Biceps brachii

Go

Next question

Biceps inserts into the radial tuberosity. Distal injuries of this muscle are rare but are
reported and are clinically more important than more proximal ruptures.
Radius
The radius is one of the two long forearm bones that extends from the lateral side of the
elbow to the thumb side of the wrist. It has two expanded ends, of which the distal end is the
larger. Key points relating to its topography and relations are outlined below;
Upper end
Articular cartilage- covers medial > lateral side
Articulates with radial notch of the ulna by the annular ligament
Muscle attachment- biceps brachii at the tuberosity

Shaft
Muscle attachment
Upper third of the body

Supinator
Flexor digitorum superficialis
Flexor pollicis longus

Middle third of the body

Pronator teres

Lower quarter of the body

Pronator quadratus
Tendon of supinator longus

Lower end
Quadrilateral
Anterior surface- capsule of wrist joint
Medial surface- head of ulna
Lateral surface- ends in the styloid process
Posterior surface: 3 grooves containing:
1. Tendons of extensor carpi radialis longus and brevis
2. Tendon of extensor pollicis longus
3. Tendon of extensor indicis

Image sourced from Wikipedia

http://www.emrcs.com/question/question.php?q=0

1/2

28/05/2015

Rate question:

Next question

Comment on this question

All contents of this site are 2012 E-Medical Revision Ltd

http://www.emrcs.com/question/question.php?q=0

Terms and Conditions

Privacy policy

2/2

28/05/2015

Reference ranges

Previous

Question 9 of 42

Next

Question stats

End and review

Score: 66.7%
1-3 2 / 3

9.5%

17.9%

24.1%

A. First pharyngeal arch

14.6%

B. Second pharyngeal arch

33.9%

C. Fourth pharyngeal arch

33.9% of users answered this


question correctly

What is embryological origin of the pulmonary artery?

D. Fifth pharyngeal arch

4
5

8
9

E. Sixth pharyngeal arch


Search
Next question
Go

Theme from September 2011 Exam


Theme from September 2012 Exam
The proximal part of the sixth right pharyngeal arch gives origin to the right pulmonary artery.
The distal part gives origin to the left pulmonary artery and the ductus arteriosus.
Pharyngeal arches
These develop during the fourth week of embryonic growth from a series of mesodermal
outpouchings of the developing pharynx.
They develop and fuse in the ventral midline. Pharyngeal pouches form on the endodermal
side between the arches.
There are 6 pharyngeal arches, the fifth does not contribute any useful structures and often
fuses with the sixth arch.
Pharyngeal arches
Pharyngeal
arch

Muscular
contributions

Skeletal
contributions

Endocrine

Artery

Nerve

First

Muscles of
mastication
Anterior belly of
digastric
Mylohyoid
Tensor tympanic
Tensor veli
palatini

Maxilla
Meckels
cartilage
Incus
Malleus

n/a

Maxillary
External
carotid

Mandibular

Second

Buccinator
Platysma
Muscles of facial
expression
Stylohyoid
Posterior belly of
digastric
Stapedius

Stapes
Styloid
process
Lesser horn
and upper
body of hyoid

n/a

Inferior
branch of
superior
thyroid
artery
Stapedial
artery

Facial

Third

Stylopharyngeus

Greater horn
and lower part
of hyoid

Thymus
Inferior
parathyroids

Common
and
internal
carotid

Glossopharyngeal

Fourth

Cricothyroid
All intrinsic
muscles of the
soft palate

Thyroid and
epiglottic
cartilages

Superior
parathyroids

Rightsubclavian
artery,
Left-aortic
arch

Vagus

Sixth

All intrinsic
muscles of the
larynx (except
cricothyroid)

Cricoid,
arytenoid and
corniculate
cartilages

n/a

Right Pulmonary
artery,
LeftPulmonary
artery and
ductus
arteriosus

Vagus and
recurrent
laryngeal nerve

Rate question:

Next question

Comment on this question

All contents of this site are 2012 E-Medical Revision Ltd

http://www.emrcs.com/question/question.php?q=0

Terms and Conditions

Privacy policy

1/2

28/05/2015

http://www.emrcs.com/question/question.php?q=0

2/2

28/05/2015

Reference ranges

Question 10 of 42

Previous

Next

A 53 year old lady presents with pain and discomfort in her hand. She works as a typist and
notices that the pain is worst when she is working. She also suffers symptoms at night. Her
little finger is less affected by the pain. Which of the nerves listed below is most likely to be
affected?

Question stats

Score: 70%
1-3 2 / 3

8.1%

59.9%

9.8%

12.8%

9.5%

59.9% of users answered this


question correctly

A. Radial

End and review

4
5

8
9

B. Median
10

C. Ulnar
Search

D. Anterior interosseous nerve


E. Posterior interosseous nerve

Go

Next question

Motor supply: LOAF


L ateral 2 lumbricals
O pponens pollicis
A bductor pollicisbrevis
F lexor pollicis brevis

Theme from April 2012 Exam


The most likely diagnosis here is carpal tunnel syndrome, the median nerve is compressed in
the wrist and symptoms usually affect the fingers and wrist either at night or when the hand is
being used (e.g. as a typist).
Median nerve
The median nerve is formed by the union of a lateral and medial root respectively from the
lateral (C5,6,7) and medial (C8 and T1) cords of the brachial plexus; the medial root passes
anterior to the third part of the axillary artery. The nerve descends lateral to the brachial
artery, crosses to its medial side (usually passing anterior to the artery). It passes deep to
the bicipital aponeurosis and the median cubital vein at the elbow.
It passes between the two heads of the pronator teres muscle, and runs on the deep surface
of flexor digitorum superficialis (within its fascial sheath).
Near the wrist it becomes superficial between the tendons of flexor digitorum superficialis and
flexor carpi radialis, deep to palmaris longus tendon. It passes deep to the flexor retinaculum
to enter the palm, but lies anterior to the long flexor tendons within the carpal tunnel.
Branches
Region

Branch

Upper
arm

No branches, although the nerve commonly communicates with the


musculocutaneous nerve

Forearm

Pronator teres
Flexor carpi radialis
Palmaris longus
Flexor digitorum superficialis
Flexor pollicis longus
Flexor digitorum profundus (only the radial half)

Distal
forearm

Palmar cutaneous branch

Hand
(Motor)

Motor supply (LOAF)

Hand
(Sensory)

Lateral 2 lumbricals
Opponens pollicis
Abductor pollicis brevis
Flexor pollicis brevis

Over thumb and lateral 2 fingers


On the palmar aspect this projects proximally, on the dorsal aspect only the
distal regions are innervated with the radial nerve providing the more proximal
cutaneous innervation.

Patterns of damage
Damage at wrist

http://www.emrcs.com/question/question.php?q=0

1/2

28/05/2015

e.g. carpal tunnel syndrome


paralysis and wasting of thenar eminence muscles and opponens pollicis (ape hand
deformity)
sensory loss to palmar aspect of lateral (radial) 2 fingers

Damage at elbow, as above plus:


unable to pronate forearm
weak wrist flexion
ulnar deviation of wrist

Anterior interosseous nerve (branch of median nerve)


leaves just below the elbow
results in loss of pronation of forearm and weakness of long flexors of thumb and index
finger

Topography of the median nerve

Image sourced from Wikipedia

Rate question:

Next question

Comment on this question

All contents of this site are 2012 E-Medical Revision Ltd

http://www.emrcs.com/question/question.php?q=0

Terms and Conditions

Privacy policy

2/2

28/05/2015

Reference ranges

Previous

Question 11 of 42

Next

Question stats

End and review

Score: 72.7%
1-3 2 / 3

48.6%

16.5%

15.2%

A. Serratus anterior

11.9%

B. Latissimus dorsi

7.8%

C. Pectoralis major

48.6% of users answered this


question correctly

Which of the following muscles lies medial to the long thoracic nerve?

D. Pectoralis minor

4
5

8
9
10

E. None of the above

11

Search
Next question
Go

Theme from 2009 Exam


Long thoracic nerve

Derived from ventral rami of C5, C6, and C7 (close to their emergence from
intervertebral foramina)
It runs downward and passes either anterior or posterior to the middle scalene muscle
It reaches upper tip of serratus anterior muscle and descends on outer surface of this
muscle, giving branches into it
Winging of Scapula occurs in long thoracic nerve injury (most common) or from spinal
accessory nerve injury (which denervates the trapezius) or a dorsal scapular nerve
injury

Rate question:

Next question

Comment on this question

All contents of this site are 2012 E-Medical Revision Ltd

http://www.emrcs.com/question/question.php?q=0

Terms and Conditions

Privacy policy

1/1

28/05/2015

Reference ranges

Previous

Question 12 of 42

Next

The thebesian veins contribute to the venous drainage of the heart. Into which of the
following structures do they primarily drain?

A. Great cardiac vein


B. Atrium

Question stats

Score: 75%
1-3 2 / 3

30.8%

41.7%

9.9%

8.7%

8.9%

41.7% of users answered this


question correctly

C. Superior vena cava

End and review

D. Oblique vein

4
5

8
9
10

E. Small cardiac vein

11

Search

12
Next question

Go

The thebesian veins are numerous small veins running over the surface of the heart they
drain into the heart itself. Usually this is to the atrium directly.
Heart anatomy
The walls of each cardiac chamber comprise:
Epicardium
Myocardium
Endocardium

Cardiac muscle is attached to the cardiac fibrous skeleton.


Relations
The heart and roots of the great vessels within the pericardial sac are related anteriorly to
the sternum, medial ends of the 3rd to 5th ribs on the left and their associated costal
cartilages. The heart and pericardial sac are situated obliquely two thirds to the left and one
third to the right of the median plane.
The pulmonary valve lies at the level of the left third costal cartilage.
The mitral valve lies at the level of the fourth costal cartilage.
Coronary sinus
This lies in the posterior part of the coronary groove and receives blood from the cardiac
veins. The great cardiac vein lies at its left and the middle and small cardiac veins lie on its
right. The smallest cardiac vein (anterior cardiac vein) drains into the right atrium directly.
Aortic sinus
Right coronary artery arises from the right aortic sinus, the left is derived from the left aortic
sinus and no vessel emerges from the posterior sinus.
Right and left ventricles
Structure

Left Ventricle

A-V Valve

Mitral (double leaflet)

Walls

Twice as thick as right

Trabeculae carnae

Much thicker and more numerous

Right coronary artery


The RCA supplies:
Right atrium
Diaphragmatic part of the left ventricle
Usually the posterior third of the interventricular septum
The sino atrial node (60% cases)
The atrio ventricular node (80% cases)

Left coronary artery


The LCA supplies:
Left atrium
Most of left ventricle
Part of the right ventricle
Anterior two thirds of the inter ventricular septum

http://www.emrcs.com/question/question.php?q=0

1/2

28/05/2015

The sino atrial node (remaining 40% cases)

Innervation of the heart


Autonomic nerve fibres from the superficial and deep cardiac plexus. These lie anterior to
the bifurcation of the trachea, posterior to the ascending aorta and superior to the
bifurcation of the pulmonary trunk. The parasympathetic supply to the heart is from
presynaptic fibres of the vagus nerves.
Valves of the heart
Mitral valve

Aortic valve

Pulmonary
valve

Tricuspid valve

2 cusps

3 cusps

3 cusps

3 cusps

First heart sound

Second heart
sound

Second heart
sound

First heart sound

1 anterior cusp

2 anterior cusps

2 anterior cusps

2 anterior cusps

Attached to chordae
tendinae

No chordae

No chordae

Attached to chordae
tendinae

Rate question:

Next question

Comment on this question

All contents of this site are 2012 E-Medical Revision Ltd

http://www.emrcs.com/question/question.php?q=0

Terms and Conditions

Privacy policy

2/2

28/05/2015

Reference ranges

Previous

Question 13 of 42

Next

Which of the following is not contained within the deep posterior compartment of the lower
leg?

A. Tibialis posterior muscle


B. Posterior tibial artery

Question stats

Score: 76.9%
1-3 2 / 3

8.5%

8.6%

12.4%

58.1%

12.4%

58.1% of users answered this


question correctly

C. Tibial nerve

End and review

D. Sural nerve

4
5

8
9
10

E. Flexor hallucis longus

11

Search

12
Go

Next question

13

The deep posterior compartment lies anterior to soleus. The sural nerve is superficially sited
and therefore not contained within it.
Lower limb- Muscular compartments
Anterior compartment
Muscle

Nerve

Action

Tibialis anterior

Deep peroneal
nerve

Dorsiflexes ankle joint, inverts foot

Extensor digitorum
longus

Deep peroneal
nerve

Extends lateral four toes, dorsiflexes ankle


joint

Peroneus tertius

Deep peroneal
nerve

Dorsiflexes ankle, everts foot

Extensor hallucis longus

Deep peroneal
nerve

Dorsiflexes ankle joint, extends big toe

Peroneal compartment
Muscle

Nerve

Action

Peroneus longus

Superficial peroneal nerve

Everts foot, assists in plantar flexion

Peroneus brevis

Superficial peroneal nerve

Plantar flexes the ankle joint

Superficial posterior compartment


< b=""><>
b=""><>

Nerve

Action

Gastrocnemius

Tibial nerve

Plantar flexes the foot, may also flex the knee

Soleus

Tibial nerve

Plantar flexor

Deep posterior compartment


Muscle

Nerve

Action

Flexor digitorum longus

Tibial

Flexes the lateral four toes

Flexor hallucis longus

Tibial

Flexes the great toe

Tibialis posterior

Tibial

Plantar flexor, inverts the foot

Rate question:

Next question

Comment on this question

All contents of this site are 2012 E-Medical Revision Ltd

http://www.emrcs.com/question/question.php?q=0

Terms and Conditions

Privacy policy

1/1

28/05/2015

Reference ranges

Previous

Question 14 of 42

Next

When performing minor surgery in the scalp, which of the following regions is considered a
danger area as regards spread of infection into the CNS?

A. Aponeurosis epicranialis
B. Skin

Question stats

Score: 78.6%
1-3 2 / 3

20.2%

7.6%

22.1%

12.8%

37.2%

37.2% of users answered this


question correctly

C. Pericranium

End and review

D. Connective tissue

4
5

8
9
10

E. Loose areolar tissue

11

Search

12
Next question

This area is most dangerous as infections can spread easily. The emissary veins that drain
this area may allow sepsis to spread to the cranial cavity.

Go

13
14

Head injury
Patients who suffer head injuries should be managed according to ATLS principles and extra
cranial injuries should be managed alongside cranial trauma. Inadequate cardiac output will
compromise CNS perfusion irrespective of the nature of the cranial injury.
Types of traumatic brain injury
Extradural
haematoma

Bleeding into the space betw een the dura mater and the skull. Often
results from acceleration-deceleration trauma or a blow to the side of the
head. The majority of extradural haematomas occur in the temporal region
w here skull fractures cause a rupture of the middle meningeal artery.
Features
Raised intracranial pressure
Some patients may exhibit a lucid interval

Subdural
haematoma

Bleeding into the outermost meningeal layer. Most commonly occur around
the frontal and parietal lobes. May be either acute or chronic.
Risk factors include old age and alcoholism.
Slow er onset of symptoms than a extradural haematoma.

Subarachnoid Usually occurs spontaneously in the context of a ruptured cerebral


haemorrhage aneurysm, but may be seen in association w ith other injuries w hen a
patient has sustained a traumatic brain injury.

Pathophysiology
Primary brain injury may be focal (contusion/ haematoma) or diffuse (diffuse axonal
injury)
Diffuse axonal injury occurs as a result of mechanical shearing following deceleration,
causing disruption and tearing of axons
Intra-cranial haematomas can be extradural, subdural or intracerebral, while
contusions may occur adjacent to (coup) or contralateral (contre-coup) to the side of
impact
Secondary brain injury occurs when cerebral oedema, ischaemia, infection, tonsillar or
tentorial herniation exacerbates the original injury. The normal cerebral auto
regulatory processes are disrupted following trauma rendering the brain more
susceptible to blood flow changes and hypoxia
The Cushings reflex (hypertension and bradycardia) often occurs late and is usually a
pre terminal event

Management
Where there is life threatening rising ICP such as in extra dural haematoma and whilst
theatre is prepared or transfer arranged use of IV mannitol/ frusemide may be
required.
Diffuse cerebral oedema may require decompressive craniotomy
Exploratory Burr Holes have little management in modern practice except where
scanning may be unavailable and to thus facilitate creation of formal craniotomy flap
Depressed skull fractures that are open require formal surgical reduction and
debridement, closed injuries may be managed non operatively if there is minimal
displacement.
ICP monitoring is appropriate in those who have GCS 3-8 and normal CT scan.
ICP monitoring is mandatory in those who have GCS 3-8 and abnormal CT scan.
Hyponatraemia is most likely to be due to syndrome of inappropriate ADH secretion.
Minimum of cerebral perfusion pressure of 70mmHg in adults.

http://www.emrcs.com/question/question.php?q=0

1/2

28/05/2015

Minimum cerebral perfusion pressure of between 40 and 70 mmHg in children.

Interpretation of pupillary findings in head injuries


Pupil size

Light response

Interpretation

Unilaterally
dilated

Sluggish or fixed

3rd nerve compression secondary to


tentorial herniation

Bilaterally dilated

Sluggish or fixed

Unilaterally
dilated or equal

Cross reactive
(Marcus - Gunn)

Bilaterally
constricted

May be difficult to
assess

Unilaterally
constricted

Preserved

Rate question:

Poor CNS perfusion


Bilateral 3rd nerve palsy
Optic nerve injury

Opiates
Pontine lesions
Metabolic encephalopathy
Sympathetic pathway disruption

Next question

Comment on this question

All contents of this site are 2012 E-Medical Revision Ltd

http://www.emrcs.com/question/question.php?q=0

Terms and Conditions

Privacy policy

2/2

28/05/2015

Reference ranges

Question 15 of 42

Previous

Next

Which of the following structures are at risk of direct injury following a fracture dislocation of
the femoral condyles?

A. Popliteal artery
B. Sciatic nerve

Question stats

Score: 80%
1-3 2 / 3

48.4%

12.8%

10.4%

10.7%

17.7%

48.4% of users answered this


question correctly

C. Plantaris muscle

End and review

D. Tibial artery

4
5

8
9
10

E. Tibial nerve

11

Search

12
Go

Next question

13
14

The heads of gastrocnemius will contract to pull the fracture segment posteriorly. The
popliteal artery lies against the bone and may be damaged or compressed.

15

Popliteal fossa
Boundaries of the popliteal fossa
Laterally

Biceps femoris above, lateral head of gastrocnemius and plantaris below

Medially

Semimembranosus and semitendinosus above, medial head of gastrocnemius


below

Floor

Popliteal surface of the femur, posterior ligament of knee joint and popliteus muscle

Roof

Superficial and deep fascia

Image showing the popliteal fossa

Image sourced from Wikipedia

Contents
Popliteal artery and vein
Small saphenous vein
Common peroneal nerve
Tibial nerve
Posterior cutaneous nerve of the thigh
Genicular branch of the obturator nerve
Lymph nodes

Rate question:

Next question

Comment on this question

All contents of this site are 2012 E-Medical Revision Ltd

http://www.emrcs.com/question/question.php?q=0

Terms and Conditions

Privacy policy

1/1

28/05/2015

Reference ranges

Question 16 of 42

Previous

Next

A 25 year old man is being catheterised, prior to a surgical procedure. As the catheter enters
the prostatic urethra which of the following changes will occur?

A. Resistance will increase significantly


B. Resistance will increase slightly

Question stats

Score: 81.3%
1-3 2 / 3

16.6%

29.5%

10.4%

36.4%

7.1%

36.4% of users answered this


question correctly

C. It will lie horizontally

End and review

D. Resistance will decrease

4
5

8
9
10

E. It will deviate laterally

11

Search

12
Next question

Theme from September 2011 Exam


The prostatic urethra is much wider than the membranous urethra and therefore resistance
will decrease. The prostatic urethra is inclined vertically.

Go

13
14
15
16

Prostate gland
The prostate gland is approximately the shape and size of a walnut and is located inferior to
the bladder. It is separated from the rectum by Denonvilliers fascia and its blood supply is
derived from the internal iliac vessels. The internal sphincter lies at the apex of the gland and
may be damaged during prostatic surgery, affected individuals may complain of retrograde
ejaculation.
Summary of prostate gland
Arterial supply

Inferior vesical artery (from internal iliac)

Venous
drainage

Prostatic venous plexus (to paravertebral veins)

Lymphatic
drainage

Internal iliac nodes

Innervation

Inferior hypogastric plexus

Dimensions

Lobes

Zones

Transverse diameter (4cm)


AP diameter (2cm)
Height (3cm)

Posterior lobe: posterior to urethra


Median lobe: posterior to urethra, in between ejaculatory ducts
Lateral lobes x 2
Isthmus

Peripheral zone: subcapsular portion of posterior prostate. Most


prostate cancers are here
Central zone
Transition zone
Stroma

Relations
Anterior

Pubic symphysis
Prostatic venous plexus

Posterior

Denonvilliers fascia
Rectum
Ejaculatory ducts

Lateral

Venous plexus (lies on prostate)


Levator ani (immediately below the puboprostatic ligaments)

http://www.emrcs.com/question/question.php?q=0

1/2

28/05/2015

Image sourced from Wikipedia

Rate question:

Next question

Comment on this question

All contents of this site are 2012 E-Medical Revision Ltd

http://www.emrcs.com/question/question.php?q=0

Terms and Conditions

Privacy policy

2/2

28/05/2015

Reference ranges

Question 17 of 42

Previous

Next

A 24 year female is admitted to A&E with tingling of her hand after a fall. She is found to have
a fracture of the medial epicondyle. What is the most likely nerve lesion?

A. Ulnar nerve
B. Radial nerve

Question stats

Score: 82.4%
1-3 2 / 3

65.2%

8.8%

13.4%

6.1%

6.5%

65.2% of users answered this


question correctly

C. Median nerve

End and review

D. Axillary nerve

4
5

8
9
10

E. Cutaneous nerve

11

Search

12
Next question

The radial nerve is located near the lateral epicondyle.

Go

13
14
15

Ulnar nerve

16
17

Origin
C8, T1

Supplies (no muscles in the upper arm)


Flexor carpi ulnaris
Flexor digitorum profundus
Flexor digiti minimi
Abductor digiti minimi
Opponens digiti minimi
Adductor pollicis
Interossei muscle
Third and fourth lumbricals
Palmaris brevis

Path
Posteromedial aspect of ulna to flexor compartment of forearm, then along the ulnar.
Passes beneath the flexor carpi ulnaris muscle, then superficially through the flexor
retinaculum into the palm of the hand.

Image sourced from Wikipedia

Branches
Branch

Supplies

Articular branch

Flexor carpi ulnaris


Medial half of the flexor digitorum profundus

http://www.emrcs.com/question/question.php?q=0

1/2

28/05/2015

Palmar cutaneous branch (Arises near the


middle of the forearm)

Skin on the medial part of the palm

Dorsal cutaneous branch

Dorsal surface of the medial part of the hand

Superficial branch

Cutaneous fibres to the anterior surfaces of the


medial one and one-half digits

Deep branch

Hypothenar muscles
All the interosseous muscles
Third and fourth lumbricals
Adductor pollicis
Medial head of the flexor pollicis brevis

Effects of injury
Damage at the wrist

Damage at the elbow

Wasting and paralysis of intrinsic hand muscles (claw hand)


Wasting and paralysis of hypothenar muscles
Loss of sensation medial 1 and half fingers

Radial deviation of the wrist


Clawing less in 3rd and 4th digits

Rate question:

Next question

Comment on this question

All contents of this site are 2012 E-Medical Revision Ltd

http://www.emrcs.com/question/question.php?q=0

Terms and Conditions

Privacy policy

2/2

28/05/2015

Reference ranges

Previous

Question 18 of 42

Next

During a gangland gunfight a man is shot in the chest. The bullet passes through the
posterior mediastinum (from left to right). Which of the following structures is least likely to be
injured

Question stats

Score: 77.8%
1-3 2 / 3

16.6%

10.3%

15.7%

13.8%

43.6%

A. Thoracic duct
43.6% of users answered this
question correctly

B. Oesophagus

End and review

C. Vagus nerve

4
5

8
9
10

D. Descending thoracic aorta

11

Search

E. Arch of the azygos vein

12
Go

13

Next question

14
15

The arch of the azygos vein lies in the middle mediastinum.

16

Mediastinum

17
18

Region between the pulmonary cavities.


It is covered by the mediastinal pleura. It does not contain the lungs.
It extends from the thoracic inlet superiorly to the diaphragm inferiorly.
Mediastinal regions
Superior mediastinum (between manubriosternal angle and T4/5)
Middle mediastinum
Posterior mediastinum
Anterior mediastinum

Region
Superior mediastinum

Anterior mediastinum

Middle mediastinum

Posterior mediastinum

Contents
Superior vena cava
Brachiocephalic veins
Arch of aorta
Thoracic duct
Trachea
Oesophagus
Thymus
Vagus nerve
Left recurrent laryngeal nerve
Phrenic nerve

Thymic remnants
Lymph nodes
Fat

Pericardium
Heart
Aortic root
Arch of azygos vein
Main bronchi

Oesophagus
Thoracic aorta
Azygos vein
Thoracic duct
Vagus nerve
Sympathetic nerve trunks
Splanchnic nerves

Rate question:

Next question

Comment on this question

All contents of this site are 2012 E-Medical Revision Ltd

http://www.emrcs.com/question/question.php?q=0

Terms and Conditions

Privacy policy

1/1

28/05/2015

Reference ranges

Previous

Question 19 of 42

Next

Question stats

End and review

Score: 78.9%
1-3 2 / 3

12.9%

13.8%

19.8%

A. Piriform recess

19.7%

B. Rima vestibuli

33.8%

C. Vestibule

33.8% of users answered this


question correctly

The space between the vocal cords is referred to as which of the following?

D. Glottis

4
5

8
9
10

E. Rima glottidis

11

Search
Next question

12
Go

13

The rima glottidis is the narrowest part of the laryngeal cavity.

14
15

Larynx

16

The larynx lies in the anterior part of the neck at the levels of C3 to C6 vertebral bodies. The
laryngeal skeleton consists of a number of cartilagenous segments. Three of these are
paired; arytenoid, corniculate and cuneiform. Three are single; thyroid, cricoid and epiglottic.
The cricoid cartilage forms a complete ring (the only one to do so).
The laryngeal cavity extends from the laryngeal inlet to the level of the inferior border of the
cricoid cartilage.

17
18
19

Divisions of the laryngeal cavity


Laryngeal vestibule

Superior to the vestibular folds

Laryngeal ventricle

Lies between vestibular folds and superior to the vocal cords

Infraglottic cavity

Extends from vocal cords to inferior border of the cricoid cartilage

The vocal folds (true vocal cords) control sound production. The apex of each fold projects
medially into the laryngeal cavity. Each vocal fold includes:
Vocal ligament
Vocalis muscle (most medial part of thyroarytenoid muscle)
The glottis is composed of the vocal folds, processes and rima glottidis. The rima glottidis is
the narrowest potential site within the larynx, as the vocal cords may be completely opposed,
forming a complete barrier.
Muscles of the larynx
Muscle

Origin

Insertion

Innervation

Action

Posterior
cricoarytenoid

Posterior aspect
of lamina of
cricoid

Muscular process of
arytenoid

Recurrent
Laryngeal

Abducts vocal
fold

Lateral
cricoarytenoid

Arch of cricoid

Muscular process of
arytenoid

Recurrent
laryngeal

Adducts vocal
fold

Thyroarytenoid

Posterior aspect
of thyroid
cartilage

Muscular process of
arytenoid

Recurrent
laryngeal

Relaxes vocal
fold

Transverse
and oblique
arytenoids

Arytenoid
cartilage

Contralateral
arytenoid

Recurrent
laryngeal

Closure of
intercartilagenous
part of the rima
glottidis

Vocalis

Depression
between lamina
of thyroid
cartilage

Vocal ligament and


vocal process of
arytenoid cartilage

Recurrent
laryngeal

Relaxes posterior
vocal ligament,
tenses anterior
part

Cricothyroid

Anterolateral part
of cricoid

Inferior margin and


horn of thyroid
cartilage

External
laryngeal

Tenses vocal fold

Blood supply
Arterial supply is via the laryngeal arteries, branches of the superior and inferior thyroid
arteries. The superior laryngeal artery is closely related to the internal laryngeal nerve. The
inferior laryngeal artery is related to the inferior laryngeal nerve. Venous drainage is via
superior and inferior laryngeal veins, the former draining into the superior thyroid vein and
http://www.emrcs.com/question/question.php?q=0

1/2

28/05/2015

the latter draining into the middle thyroid vein, or thyroid venous plexus.
Lymphatic drainage
The vocal cords have no lymphatic drainage and this site acts as a lymphatic watershed.
Supraglottic part

Upper deep cervical nodes

Subglottic part

Prelaryngeal and pretracheal nodes and inferior deep cervical nodes

The aryepiglottic fold and vestibular folds have a dense plexus of lymphatics associated with
them and malignancies at these sites have a greater propensity for nodal metastasis.

Topography of the larynx

Image sourced from Wikipedia

Rate question:

Next question

Comment on this question

All contents of this site are 2012 E-Medical Revision Ltd

http://www.emrcs.com/question/question.php?q=0

Terms and Conditions

Privacy policy

2/2

28/05/2015

Reference ranges

Previous

Question 20 of 42

Next

A 78 year old man develops a carcinoma of the scrotum. To which of the following lymph
node groups may the tumour initially metastasise?

A. Para aortic
B. Obturator

Question stats

Score: 80%
1-3 2 / 3

17.4%

5.5%

63.1%

7.6%

6.4%

63.1% of users answered this


question correctly

C. Inguinal

End and review

D. Meso rectal

4
5

8
9
10

E. None of the above

11

Search

12
Next question

Go

13
14

The scrotum is drained by the inguinal nodes.

15

Scrotal and testicular anatomy

16
17

Spermatic cord
Formed by the vas deferens and is covered by the following structures:

18
19

Layer

Origin

Internal spermatic fascia

Transversalis fascia

Cremasteric fascia

From the fascial coverings of internal oblique

External spermatic fascia

External oblique aponeurosis

20

Contents of the cord


Vas deferens

Transmits sperm and accessory gland secretions

Testicular artery

Branch of abdominal aorta supplies testis and


epididymis

Artery of vas deferens

Arises from inferior vesical artery

Cremasteric artery

Arises from inferior epigastric artery

Pampiniform plexus

Venous plexus, drains into right or left testicular


vein

Sympathetic nerve fibres

Lie on arteries, the parasympathetic fibres lie on the


vas

Genital branch of the genitofemoral


nerve

Supplies cremaster

Lymphatic vessels

Drain to lumbar and para-aortic nodes

Scrotum
Composed of skin and closely attached dartos fascia.
Arterial supply from the anterior and posterior scrotal arteries
Lymphatic drainage to the inguinal lymph nodes
Parietal layer of the tunica vaginalis is the innermost layer

Testes
The testes are surrounded by the tunica vaginalis (closed peritoneal sac). The parietal
layer of the tunica vaginalis adjacent to the internal spermatic fascia.
The testicular arteries arise from the aorta immediately inferiorly to the renal arteries.
The pampiniform plexus drains into the testicular veins, the left drains into the left
renal vein and the right into the inferior vena cava.
Lymphatic drainage is to the para-aortic nodes.

Rate question:

Next question

Comment on this question

http://www.emrcs.com/question/question.php?q=0

1/2

28/05/2015
All contents of this site are 2012 E-Medical Revision Ltd

http://www.emrcs.com/question/question.php?q=0

Terms and Conditions

Privacy policy

2/2

28/05/2015

Reference ranges

Previous

Question 21 of 42

Next

A 63 year old man is undergoing an upper GI endoscopy for dysphagia. At 33 cm (from the
incisors) a malignant looking stricture is encountered. The endoscopist attempts a balloon
dilatation.Unfortunately the tumour splits through the oesophageal wall. Into which region will
the oesophageal contents now drain?

Question stats

Score: 81%
1-3 2 / 3

11%

53.9%

15.2%

8.5%

11.4%

53.9% of users answered this


question correctly

A. Superior mediastinum

End and review

4
5

8
9

B. Posterior mediastinum
10

C. Middle mediastinum
11

Search

D. Anterior mediastinum

12

E. Peritoneal cavity

Go

13
14

Next question

15

At this position the oesophagus is still likely to be intrathoracic and located in the posterior
mediastinum.

16

Mediastinum

18

17

19

Region between the pulmonary cavities.


It is covered by the mediastinal pleura. It does not contain the lungs.
It extends from the thoracic inlet superiorly to the diaphragm inferiorly.

20
21

Mediastinal regions
Superior mediastinum (between manubriosternal angle and T4/5)
Middle mediastinum
Posterior mediastinum
Anterior mediastinum

Region
Superior mediastinum

Anterior mediastinum

Middle mediastinum

Posterior mediastinum

Contents
Superior vena cava
Brachiocephalic veins
Arch of aorta
Thoracic duct
Trachea
Oesophagus
Thymus
Vagus nerve
Left recurrent laryngeal nerve
Phrenic nerve

Thymic remnants
Lymph nodes
Fat

Pericardium
Heart
Aortic root
Arch of azygos vein
Main bronchi

Oesophagus
Thoracic aorta
Azygos vein
Thoracic duct
Vagus nerve
Sympathetic nerve trunks
Splanchnic nerves

Rate question:

Next question

Comment on this question

All contents of this site are 2012 E-Medical Revision Ltd

http://www.emrcs.com/question/question.php?q=0

Terms and Conditions

Privacy policy

1/2

28/05/2015

http://www.emrcs.com/question/question.php?q=0

2/2

28/05/2015

Reference ranges

Previous

Question 22 of 42

Next

During a tricuspid valve repair the right atrium is opened, following establishment of
cardiopulmonary bypass. Which of the following structures do not lie within the right atrium?

A. Crista terminalis
B. Tricuspid valve

Question stats

Score: 81.8%
1-3 2 / 3

15.2%

11.3%

9.2%

33.7%

30.6%

33.7% of users answered this


question correctly

C. Fossa ovalis

End and review

D. Trabeculae carnae

4
5

8
9
10

E. Musculi pectinati

11

Search

12
Next question

Go

13
14

Structures within the right atrium:


Musculi pectinati
Crista terminalis
Opening of the coronary sinus
Fossa ovalis

15
16
17
18
19
20

The trabeculae carnae are located in the right ventricle.

21
22

Heart anatomy
The walls of each cardiac chamber comprise:
Epicardium
Myocardium
Endocardium

Cardiac muscle is attached to the cardiac fibrous skeleton.


Relations
The heart and roots of the great vessels within the pericardial sac are related anteriorly to
the sternum, medial ends of the 3rd to 5th ribs on the left and their associated costal
cartilages. The heart and pericardial sac are situated obliquely two thirds to the left and one
third to the right of the median plane.
The pulmonary valve lies at the level of the left third costal cartilage.
The mitral valve lies at the level of the fourth costal cartilage.
Coronary sinus
This lies in the posterior part of the coronary groove and receives blood from the cardiac
veins. The great cardiac vein lies at its left and the middle and small cardiac veins lie on its
right. The smallest cardiac vein (anterior cardiac vein) drains into the right atrium directly.
Aortic sinus
Right coronary artery arises from the right aortic sinus, the left is derived from the left aortic
sinus and no vessel emerges from the posterior sinus.
Right and left ventricles
Structure

Left Ventricle

A-V Valve

Mitral (double leaflet)

Walls

Twice as thick as right

Trabeculae carnae

Much thicker and more numerous

Right coronary artery


The RCA supplies:
Right atrium
Diaphragmatic part of the left ventricle
Usually the posterior third of the interventricular septum
The sino atrial node (60% cases)
The atrio ventricular node (80% cases)

http://www.emrcs.com/question/question.php?q=0

1/2

28/05/2015

Left coronary artery


The LCA supplies:
Left atrium
Most of left ventricle
Part of the right ventricle
Anterior two thirds of the inter ventricular septum
The sino atrial node (remaining 40% cases)

Innervation of the heart


Autonomic nerve fibres from the superficial and deep cardiac plexus. These lie anterior to
the bifurcation of the trachea, posterior to the ascending aorta and superior to the
bifurcation of the pulmonary trunk. The parasympathetic supply to the heart is from
presynaptic fibres of the vagus nerves.
Valves of the heart
Mitral valve

Aortic valve

Pulmonary
valve

Tricuspid valve

2 cusps

3 cusps

3 cusps

3 cusps

First heart sound

Second heart
sound

Second heart
sound

First heart sound

1 anterior cusp

2 anterior cusps

2 anterior cusps

2 anterior cusps

Attached to chordae
tendinae

No chordae

No chordae

Attached to chordae
tendinae

Rate question:

Next question

Comment on this question

All contents of this site are 2012 E-Medical Revision Ltd

http://www.emrcs.com/question/question.php?q=0

Terms and Conditions

Privacy policy

2/2

28/05/2015

Reference ranges

Previous

Question 23 of 42

Next

Question stats

End and review

Score: 82.6%
1-3 2 / 3

10.8%

17%

15.7%

A. Internal jugular vein

44.5%

B. External jugular vein

12.1%

C. Anterior temporal diploic vein

44.5% of users answered this


question correctly

Which of the following is a recognised tributary of the retromandibular vein?

D. Maxillary vein

4
5

8
9
10

E. Inferior opthalmic vein

11

Search
Next question

12
Go

13

The retromandibular vein is formed from the union of the maxillary and superficial temporal
veins.

14
15

Retromandibular vein

16
17
18

Formed by a union of the maxillary vein and superficial temporal vein


It descends through the parotid gland and bifurcates within it
The anterior division passes forwards to join the facial vein, the posterior division is
one of the tributaries of the external jugular vein

19
20
21
22

Rate question:

23

Next question

Comment on this question

All contents of this site are 2012 E-Medical Revision Ltd

http://www.emrcs.com/question/question.php?q=0

Terms and Conditions

Privacy policy

1/1

28/05/2015

Reference ranges

Question 24 of 42

Previous

Next

An 22 year old soldier is shot in the abdomen and amongst his various injuries is a major
disruption to the abdominal aorta. There is torrential haemorrhage and the surgeons decide
to control the aorta by placement of a vascular clamp immediately inferior to the diaphragm.
Which of the following vessels may be injured in this maneouvre?

Question stats

Score: 83.3%
1-3 2 / 3

55.5%

11.1%

9.7%

9.7%

14%

55.5% of users answered this


question correctly

A. Inferior phrenic arteries

End and review

4
5

8
9

B. Superior phrenic arteries


10

C. Splenic artery
11

Search

D. Renal arteries

12

E. Superior mesenteric artery

Go

13
Next question

14
15

As the first branches of the abdominal aorta the inferior phrenic arteries are at greatest risk.
The superior phrenic arteries lie in the thorax. The potential space at the level of the
diaphragmatic hiatus is a potentially useful site for aortic occlusion. However, leaving the
clamp applied for more than about 10 -15 minutes usually leads to poor outcomes.

16
17
18
19

Abdominal aorta

20
21

Abdominal aortic topography


Origin

T12

Termination

L4

Posterior relations

L1-L4 Vertebral bodies

Anterior relations

Lesser omentum
Liver
Left renal vein
Inferior mesenteric vein
Third part of duodenum
Pancreas
Parietal peritoneum
Peritoneal cavity

Right lateral relations

Right crus of the diaphragm


Cisterna chyli
Azygos vein
IVC (becomes posterior distally)

Left lateral relations

4th part of duodenum


Duodenal-jejunal flexure
Left sympathetic trunk

22
23
24

The abdominal aorta

Image sourced from Wikipedia

Rate question:

http://www.emrcs.com/question/question.php?q=0

Next question

1/2

28/05/2015
Comment on this question

All contents of this site are 2012 E-Medical Revision Ltd

http://www.emrcs.com/question/question.php?q=0

Terms and Conditions

Privacy policy

2/2

28/05/2015

Reference ranges

Question 25 of 42

Previous

Next

Question stats

End and review

Score: 80%
1-3 2 / 3

10.8%

6.8%

9%

A. The fundus is usually intra peritoneal

8.6%

B. Arterial supply is from the cystic artery

64.8%

C. The cystic artery is usually located in Calots triangle

64.8% of users answered this


question correctly

Which of the following statements relating to the gallbladder is untrue?

D. Calots triangle may rarely contain an aberrant hepatic artery

4
5

8
9
10

E. Cholecystokinin causes relaxation of the gallbladder

11

Search
Next question

12
Go

13

CCK causes gallbladder contraction.

14
15

Gallbladder

16

Fibromuscular sac with capacity of 50ml


Columnar epithelium

17
18
19

Relations of the gallbladder


Anterior

20
21

Liver

22

Posterior

Covered by peritoneum
Transverse colon
1st part of the duodenum

23
24
25

Laterally

Right lobe of liver

Medially

Quadrate lobe of liver

Arterial supply
Cystic artery (branch of Right hepatic artery)
Venous drainage
Directly to the liver
Nerve supply
Sympathetic- mid thoracic spinal cord, Parasympathetic- anterior vagal trunk
Common bile duct
Origin

Confluence of cystic and common hepatic ducts

Relations at
origin

Medially - Hepatic artery


Posteriorly- Portal vein

Relations distally

Arterial supply

Duodenum - anteriorly
Pancreas - medially and laterally
Right renal vein - posteriorly
Branches of hepatic artery and retroduodenal branches of gastroduodenal
artery

Hepatobiliary triangle
Medially

Common hepatic duct

Inferiorly

Cystic duct

Superiorly

Inferior edge of liver

Contents

Cystic artery

Calots triangle has the cystic artery as the superior border.

http://www.emrcs.com/question/question.php?q=0

1/2

28/05/2015

Rate question:

Next question

Comment on this question

All contents of this site are 2012 E-Medical Revision Ltd

http://www.emrcs.com/question/question.php?q=0

Terms and Conditions

Privacy policy

2/2

28/05/2015

Reference ranges

Previous

Question 26 of 42

Next

Which of the following nerves is the primary source of innervation to the anterior scrotal
skin?

A. Iliohypogastric nerve
B. Pudendal nerve

Question stats

Score: 80.8%
1-3 2 / 3

16.2%

22.1%

39%

15.8%

6.9%

39% of users answered this


question correctly

C. Ilioinguinal nerve

End and review

D. Femoral branch of the genitofemoral nerve

4
5

8
9
10

E. Obturator nerve

11

Search

12
Go

Next question

13
14

Theme from April 2012 Exam


The pudendal nerve may innervate the posterior skin of the scrotum. The anterior
innervation of the scrotum is primarily provided by the ilioinguinal nerve. The genital branch
of the genitofemoral nerve provides a smaller contribution.

15
16
17
18

Scrotal sensation

19

The scrotum is innervated by the ilioinguinal nerve and the pudendal nerve. The ilioinguinal
nerve arises from L1 and pierces the internal oblique muscle. It eventually passes through
the superficial inguinal ring to innervate the anterior skin of the scrotum.

20
21
22

The pudendal nerve is the principal nerve of the perineum. It arises in the pelvis from 3
nerve roots. It passes through both greater and lesser sciatic foramina to enter the perineal
region. The perineal branches pass anteromedially and divide into posterior scrotal
branches. The posterior scrotal branches pass superficially to supply the skin and fascia of
the perineum. It cross communicates with the inferior rectal nerve.
Rate question:

23
24
25
26

Next question

Comment on this question

All contents of this site are 2012 E-Medical Revision Ltd

http://www.emrcs.com/question/question.php?q=0

Terms and Conditions

Privacy policy

1/1

28/05/2015

Reference ranges

Question 27 of 42

Previous

Next

Question stats

End and review

Score: 77.8%
1-3 2 / 3

11.2%

50.2%

16.9%

A. Pectineal ligament

11.9%

B. Deep inguinal ring

9.8%

C. Cremaster muscle and fascia

50.2% of users answered this


question correctly

The transversalis fascia contributes to which of the following?

D. Inguinal ligament

4
5

8
9
10

E. External spermatic fascia

11

Search
Next question

12
Go

13

The internal spermatic fascia (derived from transversalis fascia) invests:


Ducuts deferens
Testicular vessels

14
15
16
17

The principal outpouching of the transversalis fascia is the internal spermatic fascia. The
mouth of the outpouching is the deep inguinal ring.

18
19

Abdominal wall

20
21

The 2 main muscles of the abdominal wall are the rectus abdominis (anterior) and the
quadratus lumborum (posterior).
The remaining abdominal wall consists of 3 muscular layers. Each muscle passes from the
lateral aspect of the quadratus lumborum posteriorly to the lateral margin of the rectus
sheath anteriorly. Each layer is muscular posterolaterally and aponeurotic anteriorly.

22
23
24
25
26
27

Image sourced from Wikipedia

Muscles of abdominal wall


External
oblique

Internal
oblique

Transversus
abdominis

Lies most superficially


Originates from 5th to 12th ribs
Inserts into the anterior half of the outer aspect of the iliac crest, linea
alba and pubic tubercle
More medially and superiorly to the arcuate line, the aponeurotic layer
overlaps the rectus abdominis muscle
The lower border forms the inguinal ligament
The triangular expansion of the medial end of the inguinal ligament is the
lacunar ligament.

Arises from the thoracolumbar fascia, the anterior 2/3 of the iliac crest
and the lateral 2/3 of the inguinal ligament
The muscle sweeps upwards to insert into the cartilages of the lower 3
ribs
The lower fibres form an aponeurosis that runs from the tenth costal
cartilage to the body of the pubis
At its lowermost aspect it joins the fibres of the aponeurosis of
transversus abdominis to form the conjoint tendon.

Innermost muscle
Arises from the inner aspect of the costal cartilages of the lower 6 ribs ,
from the anterior 2/3 of the iliac crest and lateral 1/3 of the inguinal
ligament
Its fibres run horizontally around the abdominal wall ending in an
aponeurosis. The upper part runs posterior to the rectus abdominis. Lower
down the fibres run anteriorly only.
The rectus abdominis lies medially; running from the pubic crest and
symphysis to insert into the xiphoid process and 5th, 6th and 7th costal
cartilages. The muscles lies in a aponeurosis as described above.
Nerve supply: anterior primary rami of T7-12

Surgical notes
During abdominal surgery it is usually necessary to divide either the muscles or their
http://www.emrcs.com/question/question.php?q=0

1/2

28/05/2015

aponeuroses. During a midline laparotomy it is desirable to divide the aponeurosis. This will
leave the rectus sheath intact above the arcuate line and the muscles intact below it.
Straying off the midline will often lead to damage to the rectus muscles, particularly below the
arcuate line where they may often be in close proximity to each other.
Rate question:

Next question

Comment on this question

All contents of this site are 2012 E-Medical Revision Ltd

http://www.emrcs.com/question/question.php?q=0

Terms and Conditions

Privacy policy

2/2

28/05/2015

Reference ranges

Question 28 of 42

Previous

Next

A 63 year old man is undergoing a right pneumonectomy for carcinoma of the bronchus. As
the surgeons approach the root of the lung, which structure will lie most posteriorly (in the
anatomical plane)?

Question stats

Score: 78.6%
1-3 2 / 3

10.9%

13.8%

52.2%

11.9%

11.1%

A. Phrenic nerve
52.2% of users answered this
question correctly

B. Main bronchus

End and review

C. Vagus nerve

4
5

8
9
10

D. Pulmonary vein

11

Search

E. Pulmonary artery

12
Go

Next question

13
14

The vagus nerve is the most posteriorly located structure at the lung root. The phrenic nerve
lies most anteriorly.
Lung anatomy

15
16
17
18

The right lung is composed of 3 lobes divided by the oblique and transverse fissures. The
left lung has two lobes divided by the oblique fissure.The apex of both lungs is approximately
4cm superior to the sterno-costal joint of the first rib. Immediately below this is a sulcus
created by the subclavian artery.

19
20
21
22

Peripheral contact points of the lung


Base: diaphragm
Costal surface: corresponds to the cavity of the chest
Mediastinal surface: Contacts the mediastinal pleura. Has the cardiac impression.
Above and behind this concavity is a triangular depression named the hilum, where
the structures which form the root of the lung enter and leave the viscus. These
structures are invested by pleura, which, below the hilum and behind the pericardial
impression, forms the pulmonary ligament

23
24
25
26
27
28

Right lung
Above the hilum is the azygos vein; Superior to this is the groove for the superior vena cava
and right innominate vein; behind this, and nearer the apex, is a furrow for the innominate
artery. Behind the hilum and the attachment of the pulmonary ligament is a vertical groove
for the oesophagus; In front and to the right of the lower part of the oesophageal groove is a
deep concavity for the extrapericardiac portion of the inferior vena cava.
The root of the right lung lies behind the superior vena cava and the right atrium, and below
the azygos vein.
The right main bronchus is shorter, wider and more vertical than the left main bronchus and
therefore the route taken by most foreign bodies.

Image sourced from Wikipedia

Left lung
Above the hilum is the furrow produced by the aortic arch, and then superiorly the groove
accommodating the left subclavian artery; Behind the hilum and pulmonary ligament is a
vertical groove produced by the descending aorta, and in front of this, near the base of the
lung, is the lower part of the oesophagus.
The root of the left lung passes under the aortic arch and in front of the descending aorta.

http://www.emrcs.com/question/question.php?q=0

1/2

28/05/2015

Image sourced from Wikipedia

Inferior borders of both lungs


6th rib in mid clavicular line
8th rib in mid axillary line
10th rib posteriorly
The pleura runs two ribs lower than the corresponding lung level.
Bronchopulmonary segments
Segment number

Right lung

Left lung

Apical

Apical

Posterior

Posterior

Anterior

Anterior

Lateral

Superior lingular

Medial

Inferior lingular

Superior (apical)

Superior (apical)

Medial basal

Medial basal

Anterior basal

Anterior basal

Lateral basal

Lateral basal

10

Posterior basal

Posterior basal

Rate question:

Next question

Comment on this question

All contents of this site are 2012 E-Medical Revision Ltd

http://www.emrcs.com/question/question.php?q=0

Terms and Conditions

Privacy policy

2/2

28/05/2015

Reference ranges

Question 29 of 42

Previous

Next

A 43 year old lady is undergoing an axillary node clearance for breast cancer. The nodal
disease is bulky. During clearance of the level 3 nodes there is suddenly brisk haemorrhage.
The most likely vessel responsible is:

Question stats

Score: 75.9%
1-3 2 / 3

33.6%

9.4%

29%

14.3%

13.8%

A. Thoracoacromial artery
33.6% of users answered this
question correctly

B. Cephalic vein

End and review

C. Thoracodorsal trunk

4
5

8
9
10

D. Internal mammary artery

11

Search

E. Posterior circumflex humeral artery

12
Go

13

Next question

14
15

The thoracoacromial artery pierces the pectoralis major and gives off branches within this
space. The level 3 axillary nodes lie between pectoralis major and minor.Although the
thoracodorsal trunk may be injured during an axillary dissection it does not lie within the level
3 nodes.

16
17
18

Thoracoacromial artery

19
20

The thoracoacromial artery (acromiothoracic artery; thoracic axis) is a short trunk, which
arises from the forepart of the axillary artery, its origin being generally overlapped by the
upper edge of the Pectoralis minor.

21
22
23

Projecting forward to the upper border of the Pectoralis minor, it pierces the coracoclavicular
fascia and divides into four branches: pectoral, acromial, clavicular, and deltoid.

24
25
26

Branch

Description

Pectoral
branch

Descends between the two Pectoral muscles, and is distributed to them and to the
breast, anastomosing with the intercostal branches of the internal thoracic artery
and with the lateral thoracic.

27

Acromial
branch

Runs laterally over the coracoid process and under the Deltoid, to which it gives
branches; it then pierces that muscle and ends on the acromion in an arterial
network formed by branches from the suprascapular, thoracoacromial, and posterior
humeral circumflex arteries.

Clavicular
branch

Runs upwards and medially to the sternoclavicular joint, supplying this articulation,
and the Subclavius.

Deltoid
branch

Arising with the acromial, it crosses over the Pectoralis minor and passes in the
same groove as the cephalic vein, between the Pectoralis major and Deltoid, and
gives branches to both muscles.

Rate question:

28
29

Next question

Comment on this question

All contents of this site are 2012 E-Medical Revision Ltd

http://www.emrcs.com/question/question.php?q=0

Terms and Conditions

Privacy policy

1/1

28/05/2015

Reference ranges

Previous

Question 30 of 42

Next

A 73 year old lady with long standing atrial fibrillation develops a cold and pulseless white
arm. A brachial embolus is suspected and a brachial embolectomy is performed. Which of
the following structures is at greatest risk of injury during this procedure?

Question stats

Score: 76.7%
1-3 2 / 3

14.5%

9.9%

10.2%

57.6%

7.8%

A. Radial nerve
57.6% of users answered this
question correctly

B. Cephalic vein

End and review

C. Ulnar nerve

4
5

8
9
10

D. Median nerve

11

Search

E. None of the above

12
Go

13

Next question

14

The median nerve lies close to the brachial artery in the antecubital fossa. This is the usual
site of surgical access to the brachial artery for an embolectomy procedure. The median
nerve may be damaged during clumsy application of vascular clamps to the artery.

15

Brachial artery

18

16
17

19

The brachial artery begins at the lower border of teres major as a continuation of the axillary
artery. It terminates in the cubital fossa at the level of the neck of the radius by dividing into
the radial and ulnar arteries.

20
21
22

Relations
Posterior relations include the long head of triceps with the radial nerve and profunda
vessels intervening. Anteriorly it is overlapped by the medial border of biceps.
It is crossed by the median nerve in the middle of the arm.
In the cubital fossa it is separated from the median cubital vein by the bicipital aponeurosis.
The basilic vein is in contact at the most proximal aspect of the cubital fossa and lies
medially.

23
24
25
26
27
28

Rate question:

29

Next question

30
Comment on this question

All contents of this site are 2012 E-Medical Revision Ltd

http://www.emrcs.com/question/question.php?q=0

Terms and Conditions

Privacy policy

1/1

28/05/2015

Reference ranges

Previous

Question 31 of 42

Next

A 73 year old lady is admitted with right iliac fossa pain. A plain abdominal x-ray is taken and
the caecal diameter measured. Which of the following caecal diameters is pathological?

A. 4cm
B. 5cm

Question stats

Score: 77.4%
1-3 2 / 3

8.6%

7.7%

12.4%

14.3%

57%

57% of users answered this


question correctly

C. 6cm

End and review

D. 7cm

4
5

8
9
10

E. 10cm

11

Search

12
Next question

8 cm is still within normal limits. However, caecal diameters of 9 and 10 are pathological and
should prompt further investigation.

Go

13
14
15
16

Right colon
Ileocaecal valve

17
18
19

Entry point of the terminal ileum to the caecum


An important colonoscopic landmark
The ileocaecal valve is not always competent and this may allow partial decompression
of an obstructed colon

20
21
22
23

Appendix
At the base of the caecum the taenia coalesce to mark the base of the appendix
This is a reliable way of locating the appendix surgically and is a constant landmark
The appendix has a small mesentery (the mesoappendix) and in this runs the
appendiceal artery, a branch of the ileocolic artery.

24
25
26
27
28

The posterior aspect of the right colon is extra peritoneal and the anterior aspect
intraperitoneal.

29
30
31

Relations

Posterior
Iliacus, Iliolumbar ligament, Quadratus lumborum, Transverse abdominis, Diaphragm at the
tip of the last rib; Lateral cutaneous, ilioinguinal, and iliohypogastric nerves; the iliac
branches of the iliolumbar vessels, the fourth lumbar artery, gonadal vessels, ureter and the
right kidney.

Superior
Right kidney which is embedded in the perinephric fat

Medial
Mesentery which contains the ileocolic artery that supplies the right colon and terminal ileum.
A further branch , the right colic artery, also contributes to supply the hepatic flexure and
proximal transverse colon. Medially these pass through the mesentery to join the SMA. This
occurs near to the head of the pancreas and care has to be taken when ligating the ileocolic
artery near to its origin in cancer cases for fear of impinging on the SMA.
- Anterior
Coils of small intestine, the right edge of the greater omentum, and the anterior abdominal
wall.

Nerve supply
Parasympathetic fibres of the vagus nerve (CN X)

Arterial supply
Ileocolic artery and right colic artery, both branches of the SMA. While the ileocolic
artery is almost always present, the right colic can be absent in 5-15% of individuals.

http://www.emrcs.com/question/question.php?q=0

1/2

28/05/2015

Rate question:

Next question

Comment on this question

All contents of this site are 2012 E-Medical Revision Ltd

http://www.emrcs.com/question/question.php?q=0

Terms and Conditions

Privacy policy

2/2

28/05/2015

Reference ranges

Previous

Question 32 of 42

Next

Question stats

End and review

Score: 72.7%
1-3 2 / 3

35.5%

20.2%

9.8%

A. Middle finger

11.8%

B. Little finger

22.7%

C. Ring finger

35.5% of users answered this


question correctly

Which of the following fingers is not a point of attachment for the palmar interossei?

D. Index finger

4
5

8
9
10

E. None of the above

11

Search
Next question

12
Go

13

The middle finger has no attachment of the palmar interosseous.

14
15
16
17
18
19
20
21
22
23
24
25
26
27
28

Image sourced from Wikipedia

29
30
31

Hand

32

Anatomy of the hand


Bones

Intrinsic Muscles

8 Carpal bones
5 Metacarpals
14 phalanges
7 Interossei - Supplied by ulnar nerve
3 palmar-adduct fingers
4 dorsal- abduct fingers

Intrinsic muscles

Lumbricals
Flex MCPJ and extend the IPJ.
Origin deep flexor tendon and insertion dorsal extensor hood
mechanism.
Innervation: 1st and 2nd- median nerve, 3rd and 4th- deep branch of
the ulnar nerve.

Thenar eminence

Hypothenar
eminence

Abductor pollicis brevis


Opponens pollicis
Flexor pollicis brevis

Opponens digiti minimi


Flexor digiti minimi brevis
Abductor digiti minimi

http://www.emrcs.com/question/question.php?q=0

1/2

28/05/2015

Image sourced from Wikipedia

Fascia and compartments of the palm


The fascia of the palm is continuous with the antebrachial fascia and the fascia of the
dorsum of the hand. The palmar fascia is thin over the thenar and hypothenar eminences. In
contrast the palmar fascia is relatively thick. The palmar aponeurosis covers the soft tissues
and overlies the flexor tendons. The apex of the palmar aponeurosis is continuous with the
flexor retinaculum and the palmaris longus tendon. Distally, it forms four longitudinal digital
bands that attach to the bases of the proximal phalanges, blending with the fibrous digital
sheaths.
A medial fibrous septum extends deeply from the medial border of the palmar aponeurosis to
the 5th metacarpal. Lying medial to this are the hypothenar muscles. In a similar fashion, a
lateral fibrous septum extends deeply from the lateral border of the palmar aponeurosis to
the 3rd metacarpal. The thenar compartment lies lateral to this area.
Lying between the thenar and hypothenar compartments is the central compartment. It
contains the flexor tendons and their sheaths, the lumbricals, the superficial palmar arterial
arch and the digital vessels and nerves.
The deepest muscular plane is the adductor compartment, which contains adductor pollicis.
Short muscles of the hand
These comprise the lumbricals and interossei. The four slender lumbrical muscles flex the
fingers at the metacarpophalangeal joints and extend the interphalangeal joint. The four
dorsal interossei are located between the metacarpals and the four palmar interossei lie on
the palmar surface of the metacarpals in the interosseous compartment of the hand.
Long flexor tendons and sheaths in the hand
The tendons of FDS and FDP enter the common flexor sheath deep to the flexor
retinaculum. The tendons enter the central compartment of the hand and fan out to their
respective digital synovial sheaths. Near the base of the proximal phalanx, the tendon of FDS
splits to permit the passage of FDP. The FDP tendons are attached to the margins of the
anterior aspect of the base of the distal phalanx.
The fibrous digital sheaths contain the flexor tendons and their synovial sheaths. These
extend from the heads of the metacarpals to the base of the distal phalanges.
Rate question:

Next question

Comment on this question

All contents of this site are 2012 E-Medical Revision Ltd

http://www.emrcs.com/question/question.php?q=0

Terms and Conditions

Privacy policy

2/2

28/05/2015

Reference ranges

Previous

Question 33 of 42

Next

A 33 year old man sustains an injury to his forearm and wrist. When examined in clinic he is
unable to adduct his thumb. What is the most likely underlying nerve lesion?

A. Radial nerve
B. Superficial branch of the ulnar nerve

Question stats

Score: 73.5%
1-3 2 / 3

10.6%

11.6%

24.9%

10.9%

42%

42% of users answered this


question correctly

C. Median nerve

End and review

D. Posterior interosseous nerve

4
5

8
9
10

E. Deep branch of the ulnar nerve

11

Search

12
Go

Next question

13
14

Theme from April 2013 Exam


Theme from April 2014 Exam
Damage to the deep branch of the ulnar nerve may result in an inability to adduct the thumb.
This is tested clinically by trying to withdraw a piece of paper from a patients hand grasped
between thumb and index finger.

15
16
17
18

Adductor pollicis

19
20

Origin

Insertion

Nerve
supply

Actions

Tendon sheath of
flexor carpi radialis
Bases of second,
third and fourth
metacarpals
Anterior aspect of
the trapezoid and
capitate bones
Transverse head
comes from the
longitudinal ride of
the third metacarpal

Fibres of the two heads converge


on insertion into the ulnar aspect of
the base of the proximal phalanx of
the thumb

Deep
branch
of the
ulnar
(C8, T1)

Adducts the thumb


into the plane of the
palm and draws it to
the midline

21
22
23
24
25
26
27
28
29
30
31
32

Rate question:

Next question

33

Comment on this question

All contents of this site are 2012 E-Medical Revision Ltd

http://www.emrcs.com/question/question.php?q=0

Terms and Conditions

Privacy policy

1/1

28/05/2015

Reference ranges

Previous

Question 34 of 42

Next

A 6 year old sustains a supracondylar fracture of the distal humerus. There are concerns
that the radial nerve may have been injured. What is the relationship of the radial nerve to
the humerus at this point?

Question stats

Score: 74.3%
1-3 2 / 3

32.6%

10.9%

37.1%

12.1%

7.4%

A. Anterolateral
32.6% of users answered this
question correctly

B. Anteromedial

End and review

C. Posterolateral

4
5

8
9
10

D. Posteromedial

11

Search

E. Immediately anterior

12
Go

Next question

13
14

The radial nerve lies anterolateral to the humerus in the supracondylar area.

15
16

Radial nerve

17

Continuation of posterior cord of the brachial plexus (root values C5 to T1)

18
19

Path

20

In the axilla: lies posterior to the axillary artery on subscapularis, latissimus dorsi and
teres major.
Enters the arm between the brachial artery and the long head of triceps (medial to
humerus).
Spirals around the posterior surface of the humerus in the groove for the radial nerve.
At the distal third of the lateral border of the humerus it then pierces the intermuscular
septum and descends in front of the lateral epicondyle.
At the lateral epicondyle it lies deeply between brachialis and brachioradialis where it
then divides into a superficial and deep terminal branch.
Deep branch crosses the supinator to become the posterior interosseous nerve.

21
22
23
24
25
26
27
28
29

In the image below the relationships of the radial nerve can be appreciated

30
31
32
33
34

Image sourced from Wikipedia

Regions innervated
Motor (main
nerve)

Triceps
Anconeus
Brachioradialis
Extensor carpi radialis

Motor
(posterior
interosseous
branch)

Supinator
Extensor carpi ulnaris
Extensor digitorum
Extensor indicis
Extensor digiti minimi
Extensor pollicis longus and brevis
Abductor pollicis longus

Sensory

The area of skin supplying the proximal phalanges on the dorsal aspect of
the hand is supplied by the radial nerve (this does not apply to the little
finger and part of the ring finger)

http://www.emrcs.com/question/question.php?q=0

1/2

28/05/2015

Muscular innervation and effect of denervation


Anatomical
location

Muscle affected

Effect of paralysis

Shoulder

Long head of triceps

Minor effects on shoulder stability in


abduction

Arm

Triceps

Loss of elbow extension

Forearm

Supinator
Brachioradialis
Extensor carpi radialis
longus and brevis

Weakening of supination of prone hand and


elbow flexion in mid prone position

The cutaneous sensation of the upper limb- illustrating the contribution of the radial nerve

Image sourced from Wikipedia

Rate question:

Next question

Comment on this question

All contents of this site are 2012 E-Medical Revision Ltd

http://www.emrcs.com/question/question.php?q=0

Terms and Conditions

Privacy policy

2/2

28/05/2015

Reference ranges

Previous

Question 35 of 42

Next

Question stats

End and review

Score: 75%
1-3 2 / 3

11.3%

50.6%

7.8%

A. Medial pterygoid

7.4%

B. Buccinator

22.9%

C. Levator anguli oris

50.6% of users answered this


question correctly

Which of the following muscles is penetrated by the parotid duct?

D. Temporalis

4
5

8
9
10

E. Masseter

11

Search
Next question

12
Go

13

The duct crosses the masseter muscle and buccal fat pad and then penetrates the
buccinator muscle to enter the oral cavity opposite the second upper molar tooth.

14
15

Parotid gland

16
17

Anatomy of the parotid gland


18

Location

Overlying the mandibular ramus; anterior and inferior to the ear.

Salivary duct

Crosses the masseter, pierces the buccinator and drains adjacent to the
2nd upper molar tooth (Stensen's duct).

19
20
21
22

Structures passing
through the gland

Facial nerve (Mnemonic: The Zebra Buggered My Cat; Temporal


Zygomatic, Buccal, Mandibular, Cervical)
External carotid artery
Retromandibular vein
Auriculotemporal nerve

23
24
25
26

Relations

27

Anterior: masseter, medial pterygoid, superficial temporal and


maxillary artery, facial nerve, stylomandibular ligament
Posterior: posterior belly digastric muscle, sternocleidomastoid,
stylohyoid, internal carotid artery, mastoid process, styloid
process

28
29
30
31

Arterial supply

Branches of external carotid artery

Venous drainage

Retromandibular vein

Lymphatic
drainage

Deep cervical nodes

32
33
34

Nerve innervation

35

Parasympathetic-Secretomotor
Sympathetic-Superior cervical ganglion
Sensory- Greater auricular nerve

Parasympathetic stimulation produces a water rich, serous saliva. Sympathetic stimulation


leads to the production of a low volume, enzyme-rich saliva.
Rate question:

Next question

Comment on this question

All contents of this site are 2012 E-Medical Revision Ltd

http://www.emrcs.com/question/question.php?q=0

Terms and Conditions

Privacy policy

1/1

28/05/2015

Reference ranges

Question 36 of 42

Previous

Next

Question stats

End and review

Score: 75.7%
1-3 2 / 3

7.4%

12.3%

60.5%

A. It innervates the palmar interossei

8%

B. Derived from the medial cord of the brachial plexus

11.8%

C. Supplies the muscles of the thenar eminence

60.5% of users answered this


question correctly

The following are true of the ulnar nerve except:

D. Supplies the medial half of flexor digitorum profundus

4
5

8
9
10

E. Passes superficial to the flexor retinaculum

11

Search
Next question

12
Go

13

The muscles of the thenar eminence are supplied by the median nerve and atrophy of these
is a feature of carpal tunnel syndrome.

14
15

Ulnar nerve

16
17

Origin
18

C8, T1

19
20

Supplies (no muscles in the upper arm)

21

Flexor carpi ulnaris


Flexor digitorum profundus
Flexor digiti minimi
Abductor digiti minimi
Opponens digiti minimi
Adductor pollicis
Interossei muscle
Third and fourth lumbricals
Palmaris brevis

22
23
24
25
26
27
28
29
30

Path
Posteromedial aspect of ulna to flexor compartment of forearm, then along the ulnar.
Passes beneath the flexor carpi ulnaris muscle, then superficially through the flexor
retinaculum into the palm of the hand.

31
32
33
34
35
36

Image sourced from Wikipedia

Branches
Branch

Supplies

Articular branch

Flexor carpi ulnaris


Medial half of the flexor digitorum profundus

http://www.emrcs.com/question/question.php?q=0

1/2

28/05/2015

Palmar cutaneous branch (Arises near the


middle of the forearm)

Skin on the medial part of the palm

Dorsal cutaneous branch

Dorsal surface of the medial part of the hand

Superficial branch

Cutaneous fibres to the anterior surfaces of the


medial one and one-half digits

Deep branch

Hypothenar muscles
All the interosseous muscles
Third and fourth lumbricals
Adductor pollicis
Medial head of the flexor pollicis brevis

Effects of injury
Damage at the wrist

Damage at the elbow

Wasting and paralysis of intrinsic hand muscles (claw hand)


Wasting and paralysis of hypothenar muscles
Loss of sensation medial 1 and half fingers

Radial deviation of the wrist


Clawing less in 3rd and 4th digits

Rate question:

Next question

Comment on this question

All contents of this site are 2012 E-Medical Revision Ltd

http://www.emrcs.com/question/question.php?q=0

Terms and Conditions

Privacy policy

2/2

28/05/2015

Reference ranges

Previous

Question 37 of 42

Next

Question stats

End and review

Score: 76.3%
1-3 2 / 3

60.6%

13.2%

9.4%

A. None

8.1%

B. One

8.6%

C. Two

60.6% of users answered this


question correctly

How many valves lie between the superior vena cava and the right atrium?

D. Three

4
5

8
9
10

E. Four

11

Search
Next question

12
Go

13

There are no valves which is why it is relatively easy to insert a CVP line from the internal
jugular vein into the right atrium.

14
15

Superior vena cava

16
17

Drainage
18

Head and neck


Upper limbs
Thorax
Part of abdominal walls

19
20
21
22
23

Formation

24

Subclavian and internal jugular veins unite to form the right and left brachiocephalic
veins
These unite to form the SVC
Azygos vein joins the SVC before it enters the right atrium

25
26
27
28

Relations
29

Anterior

Anterior margins of the right lung and pleura

Posteromedial

Trachea and right vagus nerve

Posterolateral

Posterior aspects of right lung and pleura


Pulmonary hilum is posterior

Right lateral

Right phrenic nerve and pleura

Left lateral

Brachiocephalic artery and ascending aorta

30
31
32
33
34
35
36
37

Developmental variations
Anomalies of the connection of the SVC are recognised. In some individuals a persistent left
sided SVC drains into the right atrium via an enlarged orifice of the coronary sinus. More
rarely the left sided vena cava may connect directly with the superior aspect of the left
atrium, usually associated with an un-roofing of the coronary sinus. The commonest lesion of
the IVC is for its abdominal course to be interrupted, with drainage achieved via the azygos
venous system. This may occur in patients with left sided atrial isomerism.
Rate question:

Next question

Comment on this question

All contents of this site are 2012 E-Medical Revision Ltd

http://www.emrcs.com/question/question.php?q=0

Terms and Conditions

Privacy policy

1/1

28/05/2015

Reference ranges

Previous

Question 38 of 42

Next

Question stats

End and review

Score: 74.4%
1-3 2 / 3

17%

16.1%

10.6%

A. Ligamentum venosum is an anterior relation of the liver

23.2%

B. The portal triad comprises the hepatic artery, hepatic vein and tributary
of the bile duct

33.1%

Which of the following options in relation to the liver is true?

C. The liver is completely covered by peritoneum

33.1% of users answered this


question correctly

D. There are no nerves within the porta hepatis

4
5

8
9
10

E. The caudate lobe is superior to the porta hepatis

11

Search

12
Next question

Go

13
14

'VC goes with VC'

15

The ligamentun Venosum and Caudate is on same side as Vena Cava [posterior].

16
17

Ligamentum venosum is posterior to the liver. The portal triad contains the portal vein rather
than the hepatic vein. There is the 'bare area of the liver' created by a void due to the
coronary ligament layers being widely separated. There are sympathetic and
parasympathetic nerves in the porta hepatis.

18
19
20
21
22

Liver

23

Structure of the liver


Right lobe

24

Supplied by right hepatic artery


Contains Couinaud segments V to VIII (-/+Sg I)

25
26
27

Left lobe

Supplied by the left hepatic artery


Contains Couinaud segments II to IV (+/- Sg1)

28
29
30

Quadrate lobe

Part of the right lobe anatomically, functionally is part of the left


Couinaud segment IV
Porta hepatis lies behind
On the right lies the gallbladder fossa
On the left lies the fossa for the umbilical vein

31
32
33
34
35

Caudate lobe

Supplied by both right and left hepatic arteries


Couinaud segment I
Lies behind the plane of the porta hepatis
Anterior and lateral to the inferior vena cava
Bile from the caudate lobe drains into both right and left hepatic ducts

36
37
38

Detailed knowledge of Couinaud segments is not required for MRCS Part A


Between the liver lobules are portal canals which contain the portal triad: Hepatic
Artery, Portal Vein, tributary of Bile Duct.

Relations of the liver


Anterior

Postero inferiorly

Diaphragm

Oesophagus

Xiphoid process

Stomach
Duodenum
Hepatic flexure of colon
Right kidney
Gallbladder
Inferior vena cava

http://www.emrcs.com/question/question.php?q=0

1/2

28/05/2015

Porta hepatis
Location

Postero inferior surface, it joins nearly at right angles with the left sagittal fossa,
and separates the caudate lobe behind from the quadrate lobe in front

Transmits

Common hepatic duct


Hepatic artery
Portal vein
Sympathetic and parasympathetic nerve fibres
Lymphatic drainage of the liver (and nodes)

Ligaments
Falciform
ligament

2 layer fold peritoneum from the umbilicus to anterior liver surface


Contains ligamentum teres (remnant umbilical vein)
On superior liver surface it splits into the coronary and left
triangular ligaments

Ligamentum teres

Joins the left branch of the portal vein in the porta hepatis

Ligamentum
venosum

Remnant of ductus venosus

Arterial supply
Hepatic artery

Venous
Hepatic veins
Portal vein

Nervous supply
Sympathetic and parasympathetic trunks of coeliac plexus

Rate question:

Next question

Comment on this question

All contents of this site are 2012 E-Medical Revision Ltd

http://www.emrcs.com/question/question.php?q=0

Terms and Conditions

Privacy policy

2/2

28/05/2015

Reference ranges

Previous

Question 39 of 42

Next

Question stats

End and review

Score: 72.5%
1-3 2 / 3

13.4%

12%

12%

A. Anterior tibial artery

44.4%

B. Extensor digitorum longus

18.2%

C. Tibialis anterior

44.4% of users answered this


question correctly

Which of the following structures does not pass anterior to the lateral malleolus?

D. Peroneus brevis

4
5

8
9
10

E. Peroneus tertius

11

Search
Next question

12
Go

13

Peroneus brevis passes posterior to the lateral malleolus.

14
15

Lateral malleolus

16

Structures posterior to the lateral malleolus and superficial to superior peroneal


retinaculum

17
18

Sural nerve
Short saphenous vein

19
20
21

Structures posterior to the lateral malleolus and deep to superior peroneal


retinaculum

22
23

Peroneus longus tendon


Peroneus brevis tendon

24
25
26

The calcaneofibular ligament is attached at the lateral malleolus

27

Rate question:

28

Next question

29
Comment on this question

30
31
32
33
34
35
36
37
38
39

All contents of this site are 2012 E-Medical Revision Ltd

http://www.emrcs.com/question/question.php?q=0

Terms and Conditions

Privacy policy

1/1

28/05/2015

Reference ranges

Question 40 of 42

Previous

Next

Question stats

End and review

Score: 73.2%
1-3 2 / 3

10.9%

15.7%

9.1%

A. It runs from the symphysis pubis to the xiphoid process

49.9%

B. Its nerve supply is from the ventral rami of the lower 6 thoracic nerves

14.4%

C. It has collateral supply from both superior and inferior epigastric


vessels

49.9% of users answered this


question correctly

The following statements regarding the rectus abdominis muscle are true except:

D. It lies in a muscular aponeurosis throughout its length

4
5

8
9
10

E. It has a number of tendinous intersections that penetrate through the


anterior layer of the muscle

11

Search

12
Go

Next question

13
14
15

Rectus abdominis
16

Arises from the pubis.


Inserts into 5th, 6th, 7th costal cartilages.
The muscle lies in the rectal sheath, which also contains the superior and inferior
epigastric artery and vein.
Action: flexion of thoracic and lumbar spine.
Nerve supply: anterior primary rami of T7-12.

17
18
19
20
21
22

The aponeurosis is deficient below the arcuate line.

23
24
25

Abdominal wall

26

The 2 main muscles of the abdominal wall are the rectus abdominis (anterior) and the
quadratus lumborum (posterior).
The remaining abdominal wall consists of 3 muscular layers. Each muscle passes from the
lateral aspect of the quadratus lumborum posteriorly to the lateral margin of the rectus
sheath anteriorly. Each layer is muscular posterolaterally and aponeurotic anteriorly.

27
28
29
30
31
32
33
34
35
36

Image sourced from Wikipedia

Muscles of abdominal wall


External
oblique

Internal
oblique

Transversus
abdominis

Lies most superficially


Originates from 5th to 12th ribs
Inserts into the anterior half of the outer aspect of the iliac crest, linea
alba and pubic tubercle
More medially and superiorly to the arcuate line, the aponeurotic layer
overlaps the rectus abdominis muscle
The lower border forms the inguinal ligament
The triangular expansion of the medial end of the inguinal ligament is the
lacunar ligament.

37
38
39
40

Arises from the thoracolumbar fascia, the anterior 2/3 of the iliac crest
and the lateral 2/3 of the inguinal ligament
The muscle sweeps upwards to insert into the cartilages of the lower 3
ribs
The lower fibres form an aponeurosis that runs from the tenth costal
cartilage to the body of the pubis
At its lowermost aspect it joins the fibres of the aponeurosis of
transversus abdominis to form the conjoint tendon.

Innermost muscle
Arises from the inner aspect of the costal cartilages of the lower 6 ribs ,
from the anterior 2/3 of the iliac crest and lateral 1/3 of the inguinal
ligament
Its fibres run horizontally around the abdominal wall ending in an
aponeurosis. The upper part runs posterior to the rectus abdominis. Lower
down the fibres run anteriorly only.

http://www.emrcs.com/question/question.php?q=0

1/2

28/05/2015

The rectus abdominis lies medially; running from the pubic crest and
symphysis to insert into the xiphoid process and 5th, 6th and 7th costal
cartilages. The muscles lies in a aponeurosis as described above.
Nerve supply: anterior primary rami of T7-12

Surgical notes
During abdominal surgery it is usually necessary to divide either the muscles or their
aponeuroses. During a midline laparotomy it is desirable to divide the aponeurosis. This will
leave the rectus sheath intact above the arcuate line and the muscles intact below it.
Straying off the midline will often lead to damage to the rectus muscles, particularly below the
arcuate line where they may often be in close proximity to each other.
Rate question:

Next question

Comment on this question

All contents of this site are 2012 E-Medical Revision Ltd

http://www.emrcs.com/question/question.php?q=0

Terms and Conditions

Privacy policy

2/2

28/05/2015

Reference ranges

Previous

Question 41 of 42

Next

Question stats

End and review

Score: 71.4%
1-3 2 / 3

44.7%

19.6%

11.8%

A. The external jugular vein lies posteromedially.

16.7%

B. It is supplied by the accessory nerve.

7.2%

C. It has two heads of origin

44.7% of users answered this


question correctly

Which of the following statements relating to sternocleidomastoid is untrue?

D. It inserts into the lateral aspect of the mastoid process.

4
5

8
9
10

E. It marks the anterior border of the posterior triangle.

11

Search
Next question

12
Go

13

The external jugular vein lies lateral (i.e. superficial) to the sternocleidomastoid.

14
15

Sternocleidomastoid

16

Anatomy

17

Origin

Rounded tendon attached to upper manubrium sterni and muscular head


attached to medial third of the clavicle

Insertion

Mastoid process of the temporal bone and lateral area of the superior nuchal line
of the occipital bone

20

Innervation

Spinal part of accessory nerve and anterior rami of C2 and C3 (proprioception)

22

18
19

Action

21

23

Both: extend the head at atlanto-occipital joint and flex the cervical
vertebral column. Accessory muscles of inspiration.
Single: lateral flexion of neck, rotates head so face looks upward to the
opposite side

24
25
26
27

Sternocleidomastoid divides the anterior and posterior triangles of the neck.

28
29

Rate question:

Next question

30
31

Comment on this question

32
33
34
35
36
37
38
39
40
41

All contents of this site are 2012 E-Medical Revision Ltd

http://www.emrcs.com/question/question.php?q=0

Terms and Conditions

Privacy policy

1/1

28/05/2015

Reference ranges

Question 42 of 42

Previous

During liver mobilisation for a cadaveric liver transplant the hepatic ligaments will require
mobilisation. Which of the following statements relating to these structures is untrue?

A. Lesser omentum arises from the porta hepatis and passes the lesser
curvature of the stomach
B. The falciform ligament divides into the left triangular ligament and
coronary ligament

Question stats

Score: 72.1%
1-3 2 / 3

19.1%

19.5%

8.5%

14.7%

38.2%

38.2% of users answered this


question correctly

E. The right triangular ligament is an early branch of the left triangular


ligament

4
5

8
9
10

C. The liver has an area devoid of peritoneum


D. The coronary ligament is attached to the liver

End and review

11

Search

12
Go

13
14

The right triangular ligament is a continuation of the coronary ligament.

15
16

Liver

17
18

Structure of the liver


19

Right lobe

Supplied by right hepatic artery


Contains Couinaud segments V to VIII (-/+Sg I)

20
21
22

Left lobe

Supplied by the left hepatic artery


Contains Couinaud segments II to IV (+/- Sg1)

23
24
25

Quadrate lobe

Part of the right lobe anatomically, functionally is part of the left


Couinaud segment IV
Porta hepatis lies behind
On the right lies the gallbladder fossa
On the left lies the fossa for the umbilical vein

26
27
28
29
30

Caudate lobe

Supplied by both right and left hepatic arteries


Couinaud segment I
Lies behind the plane of the porta hepatis
Anterior and lateral to the inferior vena cava
Bile from the caudate lobe drains into both right and left hepatic ducts

31
32
33
34
35

Detailed knowledge of Couinaud segments is not required for MRCS Part A


36

Between the liver lobules are portal canals which contain the portal triad: Hepatic
Artery, Portal Vein, tributary of Bile Duct.

37
38
39

Relations of the liver


Anterior

Postero inferiorly

Diaphragm

Oesophagus

Xiphoid process

Stomach

40
41
42

Duodenum
Hepatic flexure of colon
Right kidney
Gallbladder
Inferior vena cava
Porta hepatis
Location

Transmits

Postero inferior surface, it joins nearly at right angles with the left sagittal fossa,
and separates the caudate lobe behind from the quadrate lobe in front
Common hepatic duct
Hepatic artery

http://www.emrcs.com/question/question.php?q=0

1/2

28/05/2015

Portal vein
Sympathetic and parasympathetic nerve fibres
Lymphatic drainage of the liver (and nodes)

Ligaments
Falciform
ligament

2 layer fold peritoneum from the umbilicus to anterior liver surface


Contains ligamentum teres (remnant umbilical vein)
On superior liver surface it splits into the coronary and left
triangular ligaments

Ligamentum teres

Joins the left branch of the portal vein in the porta hepatis

Ligamentum
venosum

Remnant of ductus venosus

Arterial supply
Hepatic artery

Venous
Hepatic veins
Portal vein

Nervous supply
Sympathetic and parasympathetic trunks of coeliac plexus

Rate question:
Comment on this question

All contents of this site are 2012 E-Medical Revision Ltd

http://www.emrcs.com/question/question.php?q=0

Terms and Conditions

Privacy policy

2/2

You might also like